You are on page 1of 519

INDEX CA Final Economic Law Case Study Index (Telegram channel: CA Rohit Chipper) INDEX

By CA Rohit Chipper, AIR 17 CA Final, AIR 18 CA Inter, Ex-PwC

For more CA Final notes & strategy join Telegram & Youtube Channel
Telegram Channel >>> https://t.me/carohitchipper
YouTube Channel >>> https://cutt.ly/QRYoXkn

Case Study Compilation Index & References


Find case study Index & compilation on telegram channel >>> https://t.me/carohitchipper

This compilation covers all case studies issued by ICAI

(1) Suggested Answers (Past year paper till July 2021)


(2) Sample case studies available on ICAI website
(3) Mock test paper till December 2021
(4) Case studies issued in ICAI student journal

Sr.No. Particulars Page no.


1 SA.M21 (Suggested answer May 21) 1 - 31
2 SA.J21 (Suggested answer Jan 21) 32 - 69
3 SA.N20 (Suggested answer Nov 20) 70 - 101
4 SA.N19 (Suggested answer Nov 19) 102 - 138
5 SA.M19 (Suggested answer May 19) 139 - 175
6 SA.N18 (Suggested answer Nov 18) 176 - 205
7 SA.M18 (Suggested answer May 18) 206 - 242
8 7SCS (7 Sample case studies available on ICAI website) 243 - 309
9 MTP.D21 (MTP December 21) 310 - 337
10 MTP.M21 (MTP March 21) 338 - 361
11 MTP.N20 (MTP November 20) Covered in case study digest
12 MTP.M20 (MTP May 20) Covered in case study digest
13 MTP.O19 (MTP October 19) 362 - 385
14 MTP.M19 (MTP March 19) 386 - 417
15 MTP.A18 (MTP August 18) 418 - 436
16 MTP.M18 (MTP March 18) 437 - 466
17 Journal May 21 467 - 503
18 Journal December 21 504 - 518
19 Case study digest Available on ICAI website

Page 0
2 FINAL (NEW) EXAMINATION: JULY 2021

PAPER-6D – ECONOMIC LAWS


There are five case study questions in the question paper. Candidates are required to
answer all the questions of any four case study questions.

CASE STUDY 1

AR Private Limited (ARPL) is one of the leading real estate companies in Vishakhapatnam and
has built over 20 multi storey apartments in Vizag and adjoining areas. They have a strong and
professional management team and have built their reputation of delivering the projects on time.
ARPL was run by two brothers, Mr. Sesha and Mr. Easwar, who were the directors of the
Company. The Government of India had established a new passport office in Vizag near the
airport and therefore, the area became very prominent and the demand for housing increased.
Therefore, ARPL wanted to start construction of 40 apartment(s) exclusive called "Seaview
Altius" in a 3 acre plot very close to the passport office by 1 st June 2017 and the project was
expected to be completed in 24 months.
Mr. Srinivas Manohar, a senior officer in the Indian Navy was posted to work out of the naval
base in Vizag. Since his posting is expected to be for a long term, Mr. Srinivas consulted with
his spouse, Ms. Kruthi, who was a Chartered Accountant, and entered into an agreement with
ARPL to purchase a 3BHK apartment for an amount of `109.94 lakhs or say `110 Lakhs (for
a 1527 square- feet of carpet area @ `7200 per square feet), The carpet area includes the
internal partition walls but did included the open terrace area for exclusive use of the owner and
open car parking facility to be charged separately. ARPL requested Mr. Srinivas to make an
advance payment of `15 lakhs for proceeding with the booking and registration. Ms. Kruthi felt
that this was exorbitant, however ARPL did not agree for reducing the advance and therefore
they made the payment.
A few months after booking the apartment, Mr. Srinivas got a notice from ARPL that due to
unforeseen circumstances they were not in a position to complete the project and therefore,
needed his consent for transferring of ARPL's rights and obligations to another reputed real
estate developer, GCH Private Limited (“GCHPL”). In case he does not agree, then he can get
his money refunded. Mr. Srinivas noted that around 70% of the allottees had accepted but 30%
of the allottees have contested that the Promoters have not fulfilled its obligations for transfer
of project to the third party and these 30% of the allottees have threatened to take legal action
against ARPL. Mr. Srinivas further noted that the authority had given its written approval for the
transfer of the project under Real Estate (Regulation and Development) Act, 2016 (for short "the
RERA"). Mr. Srinivas was also informed that GCHPL will rectify any structural damage for a
period of 4 years from the date of handing over. GCHPL also leased one of the 3BHK apartments
to Mr. Srinivas (and to many other allottees) for a period of 99 years by paying 90% of the sale
price along with stamp duty and registration charges and a rent of `1000 per month thereafter.

© The Institute of Chartered Accountants of India


Page 1
PAPER – 6D: ECONOMIC LAWS 3

Over the last 2 years, ARPL was constructing a large commercial complex in the suburbs of
Vizag and invested significant amounts on the project. For this purpose, ARPL had taken
secured loan of `200 crores from Addis Bank and `50 crores from Ababa Bank. In addition,
Mr. Sesha, a director in ARPL had given an unsecured loan of `10 crores to ARPL. Due to
certain structural issues in the construction, ARPL could not get the approval from the regulatory
department for the building and therefore, incurred a huge loss on the project and could not
repay the loans taken as well as pay its vendors and workmen.
The ARPL construction workers trade union, a registered trade union under the Trade Unions
Act, in which all the workmen of ARPL were members, filed an application under Insolvency and
Bankruptcy Code, 2016 (in short "IBC 2016") on behalf of all the workmen for non-payment of
salary to the workmen for the last 6 rnonths amounting to `15 crores. This application was
however rejected by the adjudicating authority since the trade union is not an "operational
creditor" or a "person" as defined under IBC, 2016. Further, no services were rendered by the
trade union to ARPL to claim any dues which can be termed as debt under IBC, 2016. In the
meantime, Addis Bank moved an application under Section 7 of IBC, 2016 which was admitted
and an Interim Resolution Professional (in short "IRP"), who subsequently became the
Resolution Professional (in short "RP") was appointed. The Committee of Creditors (comprising
of Addis Bank and Ababa Bank) was formed and a resolution plan was submitted by the
Resolution Professional. Both the directors of ARPL contended that they were not invited for the
meeting of the Committee of Creditors and the notice for the meeting and the draft resolution
plans were not shared with them by the Resolution Professional which is not in accordance with
IBC, 2016, reference was drawn to the judgement of Hon'ble Supreme Court of India in Civil
Appeal 8430 of 2018 as was held in the matter of Mr. Vijay Kumar Jain. Further, Mr. Sesha
contended that he should also be part of the Committee of Creditors since he is also a financial
creditor for ARPL.
Answer the following questions:
1.1 What is the maximum amount of booking advance that ARPL can collect from
Mr. Srinivas under the Real Estate (Regulation and Development) Act 2016?
(A) `11 lakhs;
(B) `15 lakhs;
(C) Based on the negotiation between the allottees and builder;
(D) `12.50 lakhs. (2 Marks)
1.2 What should ARPL do to ensure they are able to collect amounts from allottees for open
car parking facilitates under the Real Estate (Regulation and Development) Act, 2016?
(A) Ensure that the same is separately mentioned in the agreement;

© The Institute of Chartered Accountants of India


Page 2
4 FINAL (NEW) EXAMINATION: JULY 2021

(B) Prior approval is obtained from RERA as part of the registration of the property;
(C) They cannot charge unless the option is given to the buyer to choose specifically the
car parking slot;
(D) No amounts can be charged for open car parking slots. (2 Marks)
1.3 Ms. Kruthi seeks your advice on the appropriateness of the calculation of Carpet Area by
ARPL under the Real Estate (Regulation and Development) Act, 2016.
(A) Appropriate, since the method of calculation of carpet area is based on mutually
agreed terms between the parties;
(B) Not appropriate, both open terrace and internal partition walls to be excluded;
(C) Not appropriate, open terrace to be included but internal partition walls to be
excluded;
(D) Not appropriate, open terrace to be excluded but internal partition walls to be
included. (2 Marks)
1.4 GCHPL contends that the apartment leased to Mr. Srinivas is not covered the under Real
Estate (Regulation and Development) Act, 2016 provisions. Advice Mr. Srinivas
(A) RERA is not applicable since it is not a transaction of sale;
(B) RERA is not applicable, since 100% of the sale value is not paid at the time of transfer
of the property and it is a 99 year lease term;
(C) RERA is applicable, since a substantial amount is paid along with stamp duty and
registration charges. Further, the monthly rent is much lower than market;
(D) RERA is applicable since GCHPL is a registered promoter under RERA. (2 Marks)
1.5 Addis Bank wants your advice with regard to its rights for selecting the interim resolution
professional under the Insolvency and Bankruptcy Code, 2016.
(A) Any financial creditor can appoint the IRP in the committee of creditors meeting by
majority;
(B) Addis Bank has the right to propose the name of the IRP in its application for corporate
insolvency resolution process and the same person is appointed by the adjudicating
authority if there are no disciplinary proceedings;
(C) The Adjudicating Authority shall make a reference to the Board for the
recommendation of an insolvency professional who may act as an interim resolution
professional;

© The Institute of Chartered Accountants of India


Page 3
PAPER – 6D: ECONOMIC LAWS 5

(D) Any financial creditor or operational creditor can appoint the IRP in the committee of
creditors meeting by majority. (2 Marks)
1.6. Examine whether the contention of 30% of the allottees is correct on the basis that
Promoter have not fulfilled its obligations and have not met the required conditions for
transfer of project to the third party under the provisions of the Real Estate (Regulation
and Development) Act, 2016. (4 Marks)
1.7 In the opinion of Resolution Professional, participation of both the directors of ARPL as
member of the suspended Board of Directors is not mandatory in the meeting of Committee
of Creditors under the Insolvency and Bankruptcy Code, 2016. Whether the contention of
the Resolution Professional is correct vis-a-vis the judgement of the Hon'ble Supreme
Court as was held in the matter of Mr. Vijay Kumar Jain (Civil appeal No 8430 of 2018).
Explain briefly. (7 Marks)
1.8 "Mr. Sesha, a Director of the suspended Board of Directors, financial creditor and related
party of the Corporate Debtor have right to vote and be a member of the Committee of
Creditors." Examine this statement under the provisions of the Insolvency and Bankruptcy
Code, 2016. (4 Marks)
Answer to Case study 1
1.1 (A)
1.2 (D)
1.3 (D)
1.4 (A) & (C), Both options are correct
1.5 (B)
Answer 1.6
According to Section 15 of the Real Estate (Regulation and Development) Act, 2016 (RERA),
the obligations of a promoter in case of transfer of a real estate project to a third party are as
follows:
(1) The promoter shall not transfer or assign his majority rights and liabilities in respect of a
real estate project to a third party without obtaining prior written consent from two -third
allottees, except the promoter, and without the prior written approval of the Authority.
However, such transfer or assignment shall not affect the allotment or sale of the
apartments, plots or buildings as the case may be, in the real estate project made by the
erstwhile promoter.
(2) On the transfer or assignment being permitted by the allottees and the Authority under
sub-section (1), the intending promoter shall be required to independently comply with all

© The Institute of Chartered Accountants of India


Page 4
6 FINAL (NEW) EXAMINATION: JULY 2021

the pending obligations under the provisions of this Act or the rules and regulations made
thereunder, and the pending obligations as per the agreement for sale entered into by the
erstwhile promoter with the allottees.
In the instant case, the contention of 30% of the allottees is not correct as ARPL has taken
prior written consent from two-third allottees i.e. 70% allottees (more that 66.67%) and prior
written approval of the Authority. Thus, ARPL has fulfilled its obligations and have met the
required conditions for transfer of project to the third party under the provisions of the RERA,
2016.
Answer 1.7
As per Section 24(3) of the Insolvency and Bankruptcy Code, 2016, the members of the
committee of creditors may meet in person or by such electronic means. All the meeting of CoC
shall be conducted by the RP. Notice of meeting shall be served to the following:
(a) members of Committee of creditors, including the authorised representatives;
(b) members of the suspended Board of Directors or the partners of the corporate persons, as
the case may be;
(c) operational creditors or their representatives if the amount of their aggregate dues is not
less than ten per cent of the debt.
The directors, partners and one representative of operational creditors, as referred above, may
attend the meetings of committee of creditors, but shall not have any right to vote in such
meetings. And in their absence, shall not invalidate proceedings of such meeting.
Contention of the Resolution Professional vis-à-vis the judgement of the hon’ble Supreme Court
in Mr. Vijay Kumar Jain(Civil appeal No. 8430 of 2018), is not correct. In this judgement in fact
SC held that erstwhile BoD i.e. the suspended members being interested in resolution plan to
be discussed by the members of the committee of creditors, must be given a copy of that plans
as part of documents that have to be furnished along with the notice of such CoC meetings.
However, in light of the stated provision, Resolution professional has to give notice to all the
participants as given enumerated in section 24. As members of the suspended Board of
directors, they can attend the meeting as participants to deliberate on the issues, discuss and
give their opinion but they cannot vote.
Answer 1.8
As per section 21(2) of the Insolvency and Bankruptcy Code, 2016, for the Financial creditor or
the authorised representative of the financial creditor, if it is a related party of the corporate
debtor, shall not have any right of representation, participation or voting in a meeting of the
committee of creditors.
Further, as per section 5(24) of the Code, related party, in relation to a corporate debtor,
includes a director of the corporate debtor. Accordingly in the given instance, Mr. Sesha, a

© The Institute of Chartered Accountants of India


Page 5
PAPER – 6D: ECONOMIC LAWS 7

Director of the suspended Board of Directors, is a financial creditor as had provided unsecured
loan of ` 10 crores to the ARPL and is also a related party to ARPL.
Therefore, Mr. Sesha, shall not have any right of representation, participation or voting in a
meeting of the committee of creditors.
CASE STUDY 2
WF Private Limited (WFPL) is a well-known textile brand in Bengaluru and is known for its
authentic Mysore sarees and dress materials. Mr. Suresh Kumar is the founder and CEO of
WFPL and has been in the business for more than 30 years. WFPL made a turnover of `80
crores in Financial Year 2016-17 and was poised to grow its business to more than `100 crores
in Financial Year 2017-18. Mr. Suresh has one daughter, Ms. Revathi, who is happily married
and settled in the USA and has become a US Citizen, and one son, Mr. Mahesh Kumar, who
helps his father in the textile business. For the purpose of expanding his operations, Mr. Suresh
purchased a ground floor shop in Indira Nagar for an amount of `3 .2 crores. The payment was
made by cheque and the title deeds of the property were registered in his name after making
the payment of appropriate stamp duty. Mr. Mahesh Kumar purchased an independent villa in
Jaya Nagar for an amount of `7.2 crores (out of his own funds) and gave it on lease to a leading
Information Technology company for a monthly rent of `10 lakhs.
On account of the continued success of the textile business, WFPL commenced exporting
Mysore Sarees to all parts of the world and Suresh Kumar incorporated a company in the USA,
WF LLC (WFLLC) and made Ms. Revathi as the Shareholder and Director. Ms. Revathi wanted
to invest her earnings earned in USA to buy some ancestral property near Udupi, India and
accordingly, purchased a large farm house which has a palatial bungalow and 8 acre farm land
for an amount of `5 crores for which `4.5 crores was sent by her from USA to India through a
Nationalised Bank in India. Balance `50 lakhs was paid through USD denominated traveller's
cheques which she had with her and was encashed with the same Bank. She does not plan to
continue the farming activity after a ·period of 5 years. WFLLC also incorporated a sub sidiary
in India called as WL Private Limited (WLPL) for making leather jackets and accessories and
invested USD 700,000 as share capital through normal banking channels. WLPL made a
payment of USD 107,000 to WFLLC as reimbursement of pre-incorporation expenses, incurred
by WFLLC through its EEFC account. WLPL also obtained an external commercial borrowing
from WFLLC for an amount of USD 1,000,000 for working capital purposes at an interest rate
of 6 month LIBOR + 500 bps and repayable as a bullet repayment after 3 years. Notices were
issued to Ms. Revathi for the alleged violation of the Foreign Exchange Management Act, 1999
from Reserve Bank of India.
Mr. Suresh Kumar had a windfall in his business during the 2017 and therefore, wanted to buy
another apartment in Whitefield and therefore invested an amount of `2 crores for acquiring
the apartment in the name of his wife Ms. Seethalakshmi by paying lesser stamp duty as a

© The Institute of Chartered Accountants of India


Page 6
8 FINAL (NEW) EXAMINATION: JULY 2021

relaxation was given to women. Ms. Seethalakshmi was a home maker and was involved in lot
of social activities through NGO, apart from taking care of the family. Earlier Ms. Seethalakshmi
have visited her daughter Ms. Revathi in USA and on her return she had around USD 10,000 in
currency and travellers cheques which she retained with herself not aware of the provisions of
the Foreign Exchange Management Act, 1999 (FEMA, 1999).
Mr. Mukund, son in law of Mr. Suresh Kumar was also from Karnataka and was fully settled in
the USA as a US citizen. He was desirous of investing his funds through his NRE Bank account
in India for acquiring a good independent home in Mangaluru. So, Mr. Mukund, along with
Mr. Mahesh acquired an independent villa in Mangaluru in the joint names of Mr. Mahesh and
Ms. Revathi (siblings) for a total consideration of `1.10 crores. Mr. Mukund intended to buy the
property as a gift to Ms. Revathi and had informed her about the same. Mr. Mukund and
Mr. Mahesh each paid `30 lakhs each from their side and procured a cheque for another `20
lakhs from family friend and the balance amount of `30 lakhs was paid by Mr. Suresh Kumar
in cash. The property got registered in the name of Mr. Mahesh and Ms. Revathi for an amount
of `90 Iakhs and accordingly stamp duty was paid. Mr. Mahesh also purchased a small 2 BHK
apartment in Koramangala for an amount of ` 60 lakhs in the name of his driver, Mr.
Kumaraswamy, which was then rented out to a company under lease. Mr. Mahesh considered
that he can get the property transferred back to his name at a later point of time.
Answer the following questions:
2.1 Revathi seeks your views regarding the appropriateness of the reimbursement of pre-
incorporation expenses by WLPL to WFLLC under the provisions of the Foreign Exchange
Management Act, 1999?
(A) Appropriate, since the transaction is covered by Schedule II / Schedule III under the
provisions the Foreign Exchange Management Act, 1999;
(B) Not Appropriate, since the pre-incorporation expenses are specifically not allowed to
be reimbursed under FEMA, 1999;
(C) Appropriate, if the same is approved-by the Central Government of India;
(D) Not Appropriate, since the amount of reimbursement is above the threshold
prescribed and therefore required the approval of the Reserve Bank of India.
(2 Marks)
2.2 WFPL received a large export order on 15 May 2018 from a retailer in the UK for supplying
material for which WFPL received an advance payment of GBP 100,000 on 15 June 2018.
What is the last date by which WFPL has to supply goods under the agreement, under the
provisions of the Foreign Exchange Management Act, 1999?
(A) 15 th May 2019;

© The Institute of Chartered Accountants of India


Page 7
PAPER – 6D: ECONOMIC LAWS 9

(B) 15 th May 2019 or as per the terms of the agreement, whichever is later;
(C) 15 th June 2019;
(D) 15 th June 2019 or as per the terms of the agreement, whichever is later. (2 Marks)
2.3 Ms. Seethalakshmi went to visit Ms. Revathi in the USA for a holiday and after coming back
noted that she had around USD 10,000 in currency and travellers cheques. She is keen to
know whether she has to surrender the same to the Authorised Dealer under the provisions
of the Foreign Exchange Management Act, 1999.
(A) Yes, within a period of 180 days;
(B) Yes, she has to return the travellers cheques within 180 days, but can retain the
currency for future visits;
(C) No, she can retain both travellers cheques (subject to expiry date consideration) and
currency for future visits;
(D) No, the amount of USD 10,000 is within the limit prescribed under FEMA for
possessing foreign currency. (2 Marks)
2.4 Can Mr. Mahesh get the property purchased in the name of his driver, Kumaraswamy re-
transferred in his name?
(A) Yes, he can after paying the required stamp duty;
(B) No, he cannot unless Kumaraswamy agrees;
(C) No, not allowed under the Prohibition of Benami Property Transactions Act, 1988;
(D) Yes, since he was the person who funded the property. (2 Marks)
2.5 With regard to the property purchased in Mangaluru in the name of Mr. Mahesh and Ms.
Revathi, how much amount of the property will be considered as a benami property under
the Prohibition of Benami Property Transactions Act, 1988?
(A) `1.10 crores;
(B) `20 lakhs;
(C) `50 lakhs;
(D) None of the options. (2 Marks)
2.6 Examine whether the purchase of property in Udupi by Ms. Revati violates the provisions
of the Foreign Exchange Management Act, 1999 as alleged in the notice received by Ms.
Revati from Reserve Bank of India. (3 Marks)

© The Institute of Chartered Accountants of India


Page 8
10 FINAL (NEW) EXAMINATION: JULY 2021

2.7 The contention of Ms. Revati that the equity investment and the external commercial
borrowing investment by WFLLC are not complying with the provisions of Foreign
Exchange Management Act, 1999. Examine the contention. (3 Marks)
2.8 "There is no violation of the provisions of the Prohibition of Benami Property Transactions
Act, 1988 in respect of properties purchased/transactions specified in case study". Justify
your answer for each property purchased/transactions as per the provisions of Prohibition
of Benami Property Transactions Act, 1988. (9 Marks)
Answer to Case study 2
2.1 (D)
2.2 (C) and (D), both options are correct
2.3 (A)
2.4 (C)
2.5 (B)
Answer 2.6
As per FEM (Acquisition and Transfer of immovable property in India) Regulations, 2018, an
NRI or an OCI may Acquire immovable property in India other than agricultural land/farm
house/plantation property.
Provided that in case of acquisition of immovable property, payment of purchase price, if any
shall be made out of (i) funds received in India through normal banking channels by way of
inward remittance from any place outside India or (ii) funds held in any non-resident account
maintained in accordance with the provisions of the Act and the regulations made by the
Reserve Bank of India.
Provided further that no payment of purchase price for acquisition of immovable property shall
be made earlier by traveller’s cheque or by currency notes of any foreign country or any mode
other than those specifically permitted by this clause.
In the instant case, Ms. Revathi purchased a large farm house in India which has a palatial
bunglow and 8 acre farm land. Further, a portion of the payment ( ` 50 lakhs) was made through
traveller’s cheques.
As per FEMA Regulations, an OCI cannot acquire immovable property in India in the form of
agricultural land/farm house/plantation property and cannot make payment through traveller’s
cheques. Thus, Ms. Revathi has violated the provisions of FEMA for purchase of property in
near udupi.

© The Institute of Chartered Accountants of India


Page 9
PAPER – 6D: ECONOMIC LAWS 11

Answer 2.7
W.r.t. Equity Investment by WFLLC in WLPL:
As per Schedule II relating to the Capital Account Transactions, investment in equity capital of
a body corporate in India by a person resident outside India is a permissible transaction
Accordingly, the contention of Ms. Revati that the equity investment by WFLLC in its subsidiary
in India, WLPL of USD 700,000 is not correct It is a permitted Transaction.
W.r.t. ECB investment by WFLLC: The Minimum Average Maturity Period (MAMP) will be 5
years for ECB raised from foreign equity holder for working capital purposes.
All-in-cost ceiling per annum is Benchmark rate i.e. 6-months LIBOR rate plus 450 bps spread.
All eligible borrowers can raise ECB up to USD 750 million or equivalent per financial year under
the automatic route.
Under the approval route, the prospective borrowers are required to send their requests to the
Reserve Bank through their AD Banks for examination.
In the instant case, WLPL obtained an external commercial borrowing from WFLLC for an
amount of USD 1,000,000 for working capital purposes at an interest rate of 6 month LIBOR +
500 bps and repayable as a bullet repayment after 3 years.
Hence, amount of borrowing and interest rate is exceeding. Therefore, WLPL has to take
approval from RBI.
Hence, the contention of Ms. Revati in this regard is correct.
Answer 2.8
1. Ground floor shop in Indira Nagar: Mr. Suresh purchased a ground floor shop in Indira
Nagar for an amount of ` 3.2 crores. The payment was made by cheque and the title deeds
of the property were registered in his name after making appropriate stamp duty.
It is not a benami property.
2. Independent villa in Jaya Nagar: Mr. Mahesh Kumar purchased an independent villa in
Jaya Nagar for an amount of ` 7.2 crore (out of his own funds) and gave it on lease to a
leading Information Technology company for a monthly rent of ` 10 Lakhs.
It is not a benami property.
3. Apartment in Whitefield: Mr. Suresh Kumar purchased an apartment for ` 2 crore in
Whitefield in the name of his wife Ms. Seethalakshmi by paying lesser stamp duty as a
relaxation was given to women.
It is not a benami property.
4. Independent villa in Mangaluru: This property will be a benami property as ` 20 lakhs
has been procured from family friend and balance ` 30 Lakhs was paid by Mr. Suresh

© The Institute of Chartered Accountants of India


Page 10
12 FINAL (NEW) EXAMINATION: JULY 2021

Kumar in cash. Also, the property got registered in the name of Mr. Mahesh and Ms.
Revathi for an amount of ` 90 Lakhs and accordingly stamp duty was paid. The value of
the apartment was ` 1.10 crore but got registered for ` 90 lakhs.
Hence, it is a benami property.
5. BHK apartment in Koramangal: Mr. Mahesh also purchased a small 2 BHK apartment in
Koramangala for an amount of ` 60 Lakhs in the name of his driver, Mr. Kumaraswamy
then rented it out to a company under lease.
It is a benami property.
6. Farm House near Udipi: MS. Revathi purchased farm house which has palatial bungalow
and 8 acres farm land for an amount of ` 5 crores for which ` 4.5 crores was sent by her
from USA to India through nationalised bank in India. Balance ` 50 lakhs was paid through
USD denominated traveller’s cheques.
It is not a Benami property.
CASE STUDY 3
Mr. Prateek, Mr. Sachin and Mr. Prabhu are brothers, running their family business as directors
of their company, PSP Pvt. Ltd. A Corporate Insolvency Resolution process, under the
Insolvency and Bankruptcy Code, 2016 was initiated against PSP Pvt. Ltd. by an assignee of
an operational creditor for non-payment of dues and there was no intimation of any dispute
within the date of the demand notice and due to which the adjudicating authority admitted his
application.
After following all the due procedures prescribed in the Insolvency and Bankruptcy Code, 2016,
in the end, adjudicating authority passed an order to liquidate the corporate debtor, on intimation
from the resolution professional to do so, as decided by the committee of creditors by requisite
voting, before the approval of any resolution plan.
The financial highlights related to PSP Pvt. Ltd. for the purpose of liquidation are as given below:

Particulars Amount
( `in lakhs
Equity Share Capital 300
Preference Share Capital 200
Financial Creditors:
Secured 250
Unsecured 150
Operational Creditors:
Secured 60

© The Institute of Chartered Accountants of India


Page 11
PAPER – 6D: ECONOMIC LAWS 13

Unsecured 70
Government Dues 50
Workmen’s Dues 80
Employees’ Dues 80
Total Liabilities 1240
Property, Plant and Equipment’s
Land & Building 350
Plant & Machinery 150
Current Assets:
Stocks 100
Trade Receivables 300
Other current Assets 50
Cash & Cash equivalents 100
Recoverable from Statutory Authorities 190
Total Assets 1240

Other Information:
(1) Workmen's dues represent amount payable for the period of 30 months preceding the
liquidation commencement date.
(2) Employee liability includes `72 lakhs, outstanding to employees for a period of 12 months,
preceding the liquidation commencement date.
(3) Land & Building would realize 11.0% of its book value; Plant & Machinery would realize
60% of its book value, net of any realization cost Stock and trade receivables would realize
72% of its book value.
(4) The secured financial creditors worth `45 lakhs decided to enforce their security interest
in the other current assets and they could realize 80% of its value.
(5) There has been a pending court case against the company for use of child labour which
could result into penalty of approximately ` 30 lakhs. This has been reflected as a
contingent liability only. It has been finally decide to pay `25 lakhs and settle the case.
(6) Based on the amount realized & distributed, the cost of liquidation and insolvency period
cost is computed to be `20 lakhs and `12 lakhs respectively.

© The Institute of Chartered Accountants of India


Page 12
14 FINAL (NEW) EXAMINATION: JULY 2021

Meanwhile, when Mr. Prateek was engaged in providing professional assistance to the liquidator
as per Section 34 of the Insolvency and Bankruptcy Code, 2016, he and his wife Ms. Geetu
received a notice from the Initiating officer to start proceedings under the Prohibition of Benami
Property Transactions Act, 1988, with respect to the 50,000 unquoted shares of CFL Pvt. Ltd.,
held by Mr. Prateek in the name of his wife
The extract of the last audited financial statements of CFL Pvt. Ltd. as provided as under:

Particulars Amount
( `in lakhs
Land & Building (Market value `45 lakhs) 35
Plant & Machinery (Gross) (Market value `10 lakhs) 20
Stock and Trade Receivables 18
Income Tax paid in advance 2
Total Assets 75
Shareholder’s Funds (5 lakhs equity shares @ 1
`3 each) 35
Accumulated Depreciation 5
Trade Payables 12
Income Tax Provision 7
Provision for ascertained liabilities 6
Provision for unascertained liabilities 5
Total Liabilities 70

Other information:
Contingent liabilities - `3 lakhs (including `1 lakh relating to arrears on cumulative preference
shares).
As a result of the proceedings made by the Initiating officer as per Section 24 of the Prohibition
of Benami Property Transactions Act, 1988, after the valuation of the shares was done as per
Rule 3 of the relevant rules, the officer came to know that the source of the purchase of shares
by Mr. Prateek was the sale proceeds of one of the properties of PSP Pvt. Ltd which he had
fraudulently/wrongfully removed before 9 months of the insolvency commencement date and
accordingly the Initiating officer after taking approval of adjudicating authority informed the
Enforcement Director under the Prevention of Money Laundering Act, 2002 as now the property
appeared to be proceeds of crime. Also, Mr. Prateek, was prosecuted as per the penal
provisions of the Insolvency and Bankruptcy Code, 2016.

1
To be read `7

© The Institute of Chartered Accountants of India


Page 13
PAPER – 6D: ECONOMIC LAWS 15

Answer the following questions:


3.1 What should be the minimum value of the property which is fraudulently removed, in order
for the penal provisions under the Insolvency and Bankruptcy Code, 2016, to attract and
within how many months preceding the insolvency commencement date, such an act
should have occurred?
(A) `1 lakh or more and 12 months;
(B) `10,000 or more & 12 months;
(C) `10,000 & 12 months;
(D) `10 lakhs or more & 9 months. (2 Marks)
3.2 Under which Act's, Mr. Prateek can be prosecuted for his fraudulent act?
(A) Benami Transactions Act, 1988 and Insolvency and Bankruptcy Code, 2016;
(B) Prevention of Money Laundering Act, 2002 and Insolvency and Bankruptcy Code,
2016;
(C) Companies Act, 2013, Benami Transactions Act, 1988, Prevention of Money
Laundering Act, 2002 and Insolvency and Bankruptcy Code, 2016;
(D) Companies Act, 2013, Prevention of Money Laundering Act, 2002 and Insolvency and
Bankruptcy Code, 2016. (2 Marks)
3.3 As per the given case study, how much amount shall be distributed to government dues,
to secured creditors whose debts remain unpaid following the enforcement of security
interest and for the court case-penalty amount, if the funds available with the liquidator
after distribution to unsecured financial creditors is `64 lakhs under the Insolvency and
Bankruptcy Code, 2016?
(A) `40 lakhs to government dues, `4 lakhs to secured creditors with unpaid debt and
`20 lakhs for the court case-penalty amount;
(B) `50 lakhs to government dues, `5 lakhs to secured creditors with unpaid debt and
`9 lakhs for the-court case-penalty amount;
(C) `39.33 lakhs to government dues, `5 lakhs to secured creditors with unpaid debt
and `19.67 lakhs for the court case-penalty amount;
(D) `50 lakhs to government dues, `2.33 lakhs to secured creditors with unpaid debt
and `11.67 lakhs for the court case-penalty amount. (2 Marks)

© The Institute of Chartered Accountants of India


Page 14
16 FINAL (NEW) EXAMINATION: JULY 2021

3.4 If Mr. Prateek had purchased the shares in the name of his wife from the sale proceeds of
the immovable, property held by Mr. Prateek, as a joint owner with his mother; then whether
it can termed as a benami transaction under the provisions of the Benami Property
Transactions Act, 1988?
(A) Yes;
(B) No;
(C) Partially Yes, partially No;
(D) Can't say. (2 Marks)
3.5 What could be the punishment to PSP Pvt. Ltd. and its officers for the use of c hild labour
as per the provisions of the Prevention of Money Laundering Act, 2002 read with offences
under the Juvenile Justice (Care and Protection of Children) Act, 2020?
(A) Imprisonment for 3 to 7 years and fine without any limit;
(B) Imprisonment for 3 to 10 years and fine without any limit;
(C) Imprisonment upto 2 years and fine upto `50,000;
(D) Not an offence under the Prevention of Money Laundering Act, 2002, so not
punishable under this act. (2 Marks)
3.6 Examine whether the decision made by the Adjudicating Authority of admitting the
application filed by the assignee of an operational creditor against PSP Pvt. Ltd. is valid
as per the provisions of the Insolvency and Bankruptcy Code, 2016? (3 Marks)
3.7 How in your opinion, the property held by Prateek in the name of his wife can be considered
as proceeds of crime and whether any action can be taken by the Enforcement Director
against such property under the provisions of the Prevention of Money Laundering Act,
2002?
(5 Marks)
3.8 Assuming that the cost of acquisition and the market value based on discounted cash flow
method is `1.5 lakhs and `4 Iakhs respectively, calculate the fair market value of the
shares held by Prateek's wife of CFL Pvt. Ltd. in accordance with Rule 3 of the Prohibition
of Benami Transactions Rules, 2016. (4 Marks)
3.9 Whether in your opinion any other circumstances are there, other than the situation
mentioned in the case study, that may also had led the Adjudicating Authority to pass an
order of liquidation under the provisions of the Insolvency and Bankruptcy Code, 2016?
(3 Marks)

© The Institute of Chartered Accountants of India


Page 15
PAPER – 6D: ECONOMIC LAWS 17

Answer to Case Study 3


3.1 (B)
3.2 (D)
3.3 (A)
3.4 (D)
3.5 (A)
Answer 3.6
As per the provisions of the Insolvency and Bankruptcy Code, 2016:
Default means non-payment of debt when whole or any part or of instalment of the amount of
debt has become due and payable and is not paid by the debtor or the corporate debtor, as the
case may be. [Section 3(12)]
Operational creditor means a person to whom an operational debt is owed and includes any
person to whom such debt has been legally assigned or transferred; [Section 5(20)]
Serving of demand Notice: On the occurrence of default, an operational creditor shall first
send a demand notice and a copy of invoice to the corporate debtor.
On receipt of demand notice by corporate debtor: The corporate debtor shall, within a period
of ten days of the receipt of the demand notice or copy of the invoice bring to the notice of the
operational creditor about-
(a) existence of a dispute about debt, if any, or record of the pendency of the suit or arbitration
proceedings filed before the receipt of such notice or invoice in relation to such dispute;
(b) the payment of unpaid operational debt— It is possible that corporate debtor might have
already paid the unpaid operational debt, there in such situation, corporate debtor will
inform within 10 days-
(i) by sending an attested copy of the record of electronic transfer of the unpaid amount
from the bank account of the corporate debtor; or
(ii) by sending an attested copy of record that the operational creditor has encashed a
cheque issued by the corporate debtor. [Section 8]
If no reply is received or payment or notice of the dispute under section 8(2) from the
corporate debtor within ten days from the date of delivery of the notice or invoice
demanding payment, operational creditor can file application before Adjudicating Authority
(NCLT) for initiating a corporate insolvency resolution process as per Section 9 of the
Code.

© The Institute of Chartered Accountants of India


Page 16
18 FINAL (NEW) EXAMINATION: JULY 2021

Thus, based on the aforementioned provisions the decision made by the adjudicating
authority of admitting the application filed by the assignee of an operational creditor is valid
as operational creditor also includes a person to whom such debt has been assigned.
Answer 3.7
Section 2(1)(u) of the Prevention of Money Laundering Act, 2002defines "proceeds of crime"
as any property derived or obtained, directly or indirectly, by any person as a result of criminal
activity relating to a scheduled offence or the value of any such property or where such
property is taken/held outside the country, then the property equivalent in value held within the
country or abroad.
Paragraph 29 of Part A of Scheduled Offence prescribes offence under section 447 of the
Companies Act, 2013.
In the given case, the offence of fraudulently/ wrongfully removing the property of PSP Pvt. Ltd.
and using the sale proceeds for personal benefit is an offence punishable under section 447 of
the Act which is also a scheduled offence mentioned under the provisions of the Prevention of
Money Laundering Act, 2002 and any property derived from criminal activity relating to a
scheduled offence falls under proceeds of crime as defined above.
The action that can be taken by the Enforcement director against such property is provided on
the basis of provisions of section 5 of the Act as follows:
Where the Director or any other officer (not below the rank of Deputy Director authorised by the
Director) for the purposes of this section, has reason to believe (the reason for such belief to be
recorded in writing), on the basis of material in his possession, that—
(a) any person is in possession of any proceeds of crime; and
(b) such proceeds of crime are likely to be concealed, transferred or dealt with in any manner
which may result in frustrating any proceedings relating to confiscation of such proceeds
of crime under this Chapter,
he may, by order in writing, provisionally attach such property for a period not exceeding one
hundred and eighty days from the date of the order, in such manner as may be prescribed.
Condition for attachment: Provided that no such order of attachment shall be made unless, in
relation to the scheduled offence:
• a report has been forwarded to a Magistrate under section 173 of the Code of Criminal
Procedure, 1973, or
• a complaint has been filed by a person authorised to investigate the offence mentioned in
that Schedule, before a Magistrate or court for taking cognizance of the scheduled offence,
as the case may be, or

© The Institute of Chartered Accountants of India


Page 17
PAPER – 6D: ECONOMIC LAWS 19

• a similar report or complaint has been made or filed under the corresponding law of any
other country.
Provided further that, notwithstanding anything contained in first proviso, any property of any
person may be attached under this section if the Director or any other officer not below the rank
of Deputy Director authorised by him for the purposes of this section has reason to believe (the
reasons for such belief to be recorded in writing), on the basis of material in his possession, that
if such property involved in money-laundering is not attached immediately under this Chapter,
the non-attachment of the property is likely to frustrate any proceeding under this Act.
Provided also that for the purposes of computing the period of one hundred and eighty days,
the period during which the proceedings under this section is stayed by the High Court, shall be
excluded and a further period not exceeding thirty days from the date of order of vacation of
such stay order shall be counted.
Thus, the director can pass an order of provisional attachment of the property for a maximum
period of 180 days subject to the conditions as aforesaid.
Answer 3.8
According to section 2(16) of the Prohibition of Benami Property Transaction Act, 1988, fair
market value, in relation to a property, means—
(1) the price that the property would ordinarily fetch on sale in the open market on the date of
the transaction; and
(2) where the price referred to in sub-clause (i) is not ascertainable, such price as may be
determined in accordance with such manner as may be prescribed in Rule 3 of the
Prohibition of Benami Property Transaction Rules, 2016.
As per the said Rule, the price of unquoted equity shares shall be the higher of-
(i) its cost of acquisition;
(ii) the fair market value of such equity shares determined, on the date of transaction, by a
merchant banker or an accountant as per the Discounted Cash Flow method; and
(iii) the value, on the date of transaction, of such equity shares as determined by the formula
given in the Rules.
The value of (iii) above is determined as below:
Particulars Amount Value to be Remarks
(` in Lakhs) considered for
calculation
(` in lakhs)
Land & Building (Market 35 45 Market value to be
value ` 45 lakhs) considered

© The Institute of Chartered Accountants of India


Page 18
20 FINAL (NEW) EXAMINATION: JULY 2021

Plant & Machinery (Gross) 20 15 Book value net of


(Market Value ` 10 lakhs) (20-5) accumulated depreciation
Stock & trade Receivables 18 18 Book value
Income tax paid in advance 2 0 Not to be considered
Total Value of Assets 78
Shareholder’s Funds (5 35 0 Share capital and
lakh equity shares @ 2` 3 Reserves not to be
each) considered
Accumulated Depreciation 5 0 Considered in Value of
Plant & Machinery above
Trade Payables 12 -12 To be considered
Income Tax Provision 7 0 Not to be considered
Provision for ascertained 6 -6 To be considered
liabilities
Provision for unascertained 5 0 Not to be considered
liabilities
Contingent Liabilities 3 -1 Arrears of divided on
cumulative preference
shares to be considered
Total Value of Liabilities -19
Fair Market Value (Asset- 59
Liabilities) *Paid up Equity
Capital/ Paid up value of
equity shares
Value of equity shares 5.9
acquired i.e. 10% of total
(50,000/5,00,000)

In the said question, the cost of acquisition is assumed at ` 1.5 lakhs, the value, on the date of
transaction, of such equity shares as determined by the formula given in the rules is ` 5.9 lakhs
and the market value based on discounted cash flow method is given as ` 4 lakhs. Thus, the
fair market value of the acquisition in CFL Pvt. Ltd. will be ` 5.9 lakhs being highest of above.
Answer 3.9
It is given in the case study that before passing of the resolution plan, the committee of creditors
decided to liquidate the corporate debtor, so accordingly the other circum stances mentioned

2 To be read as ` 7

© The Institute of Chartered Accountants of India


Page 19
PAPER – 6D: ECONOMIC LAWS 21

hereunder are related to situations where resolution plan has not been received or it has been
rejected.
Section 33 of the Code, interalia, provides that where the Adjudicating Authority, —
(a) Not received a Resolution plan: Before the expiry of the insolvency resolution process
period or the maximum period permitted for completion of the corporate insolvency
resolution process or the fast track corporate insolvency resolution process, as the case
may be, does not receive a resolution plan; or
(b) rejects the resolution plan for the non-compliance of the requirements specified therein,
it shall pass an order requiring the corporate debtor to be liquidated.
CASE STUDY 4
The world of cinemas has always drawn quite a lot of dreamers and talented people into its fold
and there are many artists who have become super stars through their hard work
notwithstanding their humble beginnings. Mr. Sanjay Shankar is one such star who came to
Mumbai from a small town in Madhya Pradesh dreaming of becoming a super star. Through his
sheer hard work and commitment, he got opportunities to act in few small films and due to his
great work ethic and acting skills, he became very famous and acted in almost 30 films over a
period of 4 years and won many accolades. In 2020, he launched his own movie and digital
media production house. Futuristic Cinemas Private Limited (FCPL), with an objective to provide
opportunities to deserving professionals from humble backgrounds like himself. One such
professional he met through his production house was Ms. Chaitya Prakash, a beautiful and
talented dancer and both of them became close, were planning to get married soon.
FCPL started producing various movies and was reasonably successful and profitable and Mr.
Sanjay was able to manage both his acting career as well as the production house along with
Ms. Chaitya. In 2021, Mr. Sanjay, in his free time watched a lot of English and European digital
series through online platforms and he was interested in bringing some of the best series into
India by dubbing in Hindi and regional languages. Accordingly, FCPL entered into a contract
with the foreign production houses and obtained the rights to broadcast these digital series in
India after dubbing them in local languages. When the news of this broke out, the All India
Production Houses Association (APHA) as well as All India Movie and TV Actors Association
(AMTAA) opposed this move by FCPL for broadcasting this in India on the basis that this will
hamper the viewership of the serials currently being made in India and thereby would adversely
affect the job of local producers and artists. FCPL is of the view that the actions of APHA a nd
AMTAA are anticompetitive in nature and therefore, not in accordance with the Competition Act,
2002. Reference was drawn from Honourable Supreme Court Judgement in Competition
Commission of India Vs Coordination Committee of Artists and Technicians of W.B. Film and
Television and Others. Kishor Movies Limited (KML), one of the largest production houses in

© The Institute of Chartered Accountants of India


Page 20
22 FINAL (NEW) EXAMINATION: JULY 2021

India and also a member of APHA, also alleged that KML had the copy right for one of the digital
series and FCPL has violated the Copyright Act as well.
Mr. Sanjay and Ms. Chaitya travelled to Switzerland for a movie and dance show and settled in
Switzerland for about 15 months in Lucerne. During their stay in Switzerland, they also visited
Belgium where Ms. Chaitya purchased a beautiful Belgian diamond and gold set weighing, 30
grams for a value of EUR 2000. Mr. Sanjay purchased a swiss gold chain for himself weighing
10 grams for a value of EUR 500. When returning to India, Mr. Sanjay brought EUR 6,000 in
cash and Ms. Chaitya brought EUR 9,000 (EUR 4,500 in cash and EUR 4,500 in traveller's
cheque). When they returned to India, a heavy media contingent were waiting outside the airport
and they came through the green channel waving away to the media. Ms. Chaitya, after returning
home, handover the foreign currency and traveller's cheques to her mom, Ms. Sundari, who
kept the same in her bank locker.
On the other hand, KML had produced a big budget action movie with the biggest star of the
country acting as the hero based on India's freedom struggle and was planning to release it just
before the 75th year of Indian Independence. This was expected to be a block buster movie and
in order to utilize this opportunity, KML put forth a non-negotiable condition to all single screen
theatres (which had a market share of around 35% of the total theatres) that for purchasing the
exhibition rights for this movie, they had to necessarily acquire the exhibition rights for their next
movie which was going to release couple of months later. Some of the single screen theatres
agreed to the condition and few other declined and therefore did not get the right to exhibi t the
big budget movie. A few of the single screen theatres felt that KML was abusing its dominant
position in the industry (being one of the largest movie production houses in the country) by
forcing the theatres to buy the rights for two movies unnecessarily and thereby preventing some
of the theatres to choose to acquire the rights for the big budget movie.
Note: For the purpose of above case study, 1 USD = 0.8 EUR, and 1 USD = INR 70
Answer the following questions:
4.1 Ms. Chaitya wanted to understand if she was correct in coming through the green channel
on her return to India?
(A) Yes, the value of materials (foreign currency and jewellery) brought by her is within
the limits prescribed;
(B) No, the value of foreign currency and jewellery brought into India by her was more
than the limits;
(C) No, since the value of jewellery brought into India is more than the limit and there is
no limit for bringing in foreign currency (only a declaration is required);

© The Institute of Chartered Accountants of India


Page 21
PAPER – 6D: ECONOMIC LAWS 23

(D) Yes, the combined value of material brought into India by Mr. Sanjay and Ms. Chaitya
was within the limits. (2 Marks)
4.2 What is the maximum value of jewellery that can be brought into India by Mr. Sanjay while
coming through the green channel?
(A) Forty grams, with a maximum value of INR 100,000;
(B) Twenty grams, with a maximum value of INR 50,000;
(C) Twenty grams, with a monetary limit based on the day's market rate of jewellery in
India;
(D) None of the options. (2 Marks)
4.3. Assuming that KML had earlier obtained the copyright for one of the digital series now
acquired by FCPL, what is the status of the violation under the PMLA 2002?
(A) Prima facie, FCPL is in violation of PMLA 2002 if they have infringed on the rights of
KML;
(B) No, a violation under Copyright Act is not covered under PMLA 2002;
(C) The violation under PMLA would arise only if the copyright was acquired by FCPL
using proceeds of crime relating to another offence under PMLA 2002;
(D) None of the options. (2 Marks)
4.4 Which of the following factors are not to be considered when evaluating whether the
conditions put forth by KML on the single screen theatres has appreciable adverse effect
on competition?
(A) Directly, or indirectly determines the sale price of the tickets;
(B) Tie-in arrangement;
(C) Exclusive supply agreement;
(D) Refusal to deal or restrict any person from dealing. (2 Marks)
4.5. When evaluating the facts, the Directorate General (DG) of the Competition Commission
of India has sought your views on some of the aspects that he needs to consider for
evaluating the relevant geographic market when determining the position taken by APHA
and AMTAA ?
(A) Local specification requirements, language, consumer preferences, price of services;
(B) Consumer preferences, language, existence of specialised producers, transportation
costs;

© The Institute of Chartered Accountants of India


Page 22
24 FINAL (NEW) EXAMINATION: JULY 2021

(C) Physical characteristics or end use of services, price of services, national


procurement policies, local specification requirements;
(D) Regulatory trade barriers, local specification requirements, language, consumer
preferences. (2 Marks)
4.6 In the light of the facts mentioned in the case study regarding the items and foreign
currency brought into India by Mr. Sanjay and Ms. Chaitya, evaluate the offences as per
the provisions of the prevention of Money Laundering Act, 2002 read with offences under
the Customs Act, 1962. (5 Marks)
4.7 Whether the contention of APHA and AMTAA are in violation of the Competition Act, 2002,
vis-a-vis Hon'ble Supreme Court Judgement in Competition Commission of India Vs.
Coordination Committee of Artists and Technicians of W.B. Film & Television and others.
Explain briefly. (7 Marks)
4.8 Evaluate with reasons whether KML was abusing its dominant position in the industry by
forcing the theatres to buy the rights for two movies unnecessarily and thereby preventing
some of the theatres to choose to acquire the rights for a big budget movie. (3 Marks)
Answer to Case Study 4
4.1 (C)
4.2 (B)
4.3 (A)
4.4 (A)
4.5 (D)
Answer 4.6
Mr. Sanjay and Ms. Chaitya are guilty of offence under the Prevention of Money Laundering Act,
2002.
Mr. Sanjay and Ms. Chaitya purchased the following items from Belgium, when 1USD= 0.8 EUR
and 1US$ = ` 70:
Ms. Chaitya:
i. A diamond and gold necklace of 30 grams for EUR 2000([i.e. ` 1,75,000)
ii. EUR 9000 (EUR 4500 in cash and EUR 4500 in traveller’s cheque): [i.e. USD 5625 in cash
and USD 5625 in traveller’s cheque]
Mr. Sanjay:
i. Mr. Sanjay purchased a gold chain of 10 gms for EUR 500 (i.e. ` 43,750)

© The Institute of Chartered Accountants of India


Page 23
PAPER – 6D: ECONOMIC LAWS 25

ii. EUR 6000 in cash [i.e USD 7500]


As per Rule 5 of Baggage Rules, 2016, a passenger residing abroad for more than one year, on
return to India shall be allowed clearance free of duty in his bona-fide baggage of jewellery up
to a weight, of twenty grams with a value cap of fifty thousand rupees if brought by a gentleman
passenger, or forty grams with a value cap of one lakh rupees if brought by a lady passenger.
Since baggage item is also subject to duty beyond certain limit and in the case of Mr. Sanjay
this limit is twenty grams with a value cap of fifty thousand rupees. Since he brought the gold
chain within the prescribed amount (i.e. 10gms gold worth ` 43,750), hence Mr. Sanjay has not
committed any offence under section 135 of the Customs Act, 1962.
Ms. Chaitya brought through the green channel, jewellery which is beyond the permitted limit
(forty grams with a value cap of one lakh rupees) of baggage. But Ms. Chaitya walked through
the Green Channel with dutiable goods. She should have walked through the Red Channel and
declared her dutiable goods. Hence, Ms. Chaitya has committed an offence under section 135
of the Customs Act, 1962.
Further as per paragraph 12 of part A of Schedule to the Prevention of Money Laundering Act
2002, offences under the Section 135 of Customs Act, 1962 regarding evasion of custom duty;
and offences under the Section 132 of Customs Act, 1962 regarding false declaration, false
documents, etc. are considered as scheduled offence under the Prevention of Money
Laundering Act, 2002.
Therefore, Ms. Chaitya is guilty of an offence under section 3 of the Prevention of Money
Laundering Act, 2002.
Further, Ms. Sundari is holding foreign currency and traveller’s cheque being proceeds of crime,
is guilty of violating the provisions of the Prevention of Money Laundering Act, 2002.
Answer 4.7
In the light of the stated facts, APHA & AMTAA opposed the move taken by the FCPL for broad
casting best European and English digital series in India. The contention of APHA & AMTAA
that this will hamper the viewer ship of the serials currently being made in India and would
adversely affect the job of local producers and artists.
In the light of the said scenario, both APHA & AMTAA comes under the purview of associations
of enterprises as defined under section 2(h) of the Competition Act, 2002.
Enterprises – The said, APHA & AMTAA, associations are trade unions in nature, and so, by
the virtue of the fact that the constituent members of the associations have been indulging in
the activities relating to the production, distribution and exhibitions of the films. And therefore
the said associations falls within the ambit of “association of enterprises” as used under the Act.
Agreement – the said enterprises as the members, take decisions on behalf of every such
enterprise being engaged in similar kind of business, and also consult certain other
organizations in order to safeguard the interest of the concerned enterprises, which further

© The Institute of Chartered Accountants of India


Page 24
26 FINAL (NEW) EXAMINATION: JULY 2021

shows the collective intent of the entire associations, and fall within the ambit of “Action in
Concert” as given in Section 2(b).
As association of both the enterprises APHA & AMTAA, engaged in similar business of
productions, distributions and exhibitions of films and take decisions on behalf of every such
enterprise engaged in similar business and their decisions reflect the collective intent. Since the
associations and FCPL, both the parties working at horizontal level, hence the action of the
association attract the element of “Anti-Competitive Agreement” as specifically given under
Section 3(3), since the associations by causing restriction on the dubbing in hindi and regional
languages series, was limiting or controlling the production, supply of the serials, in India.
Moreover, the said restrictions also prevented consumers from enjoying their “right to choose”
and hence in totality it caused the “Appreciable adverse effect” on the competition of relevant
business in the relevant market, which is prohibited by Section 3 of the Act .
Answer 4.8
The facts of the case are that KML was planning to release its big budget action movie on 75 th

year of Indian Independence. KML put forth a condition before the Single Screen Theatres that
if they want to purchase the rights of the said action movie, they have to also purchase the
exhibition rights of their next movie which was to be released in couple of months later. KML
kept that as a non-negotiable condition.
These Single Screen Theatres had a market share of around 35% of the total thea tres.
The majority of the Single Screen Theatres agreed to the condition because KML is the largest
(number of films per year) producer, but some did not find it lucrative and hence declined.
Unfortunately, the ones who declined did not get the rights to exhibit both, the movies.
Since KML put forward tie-in agreement (prohibited under section 3(4) the Competition Act,
2002 and explained through explanation to said sub-section) as a non-negotiable condition in
front of Single Screen Theatres, hence guilty under section 3(1) the Competition Act, 2002 of
entering an anti-competitive agreement.
KML being the largest producer, hold the dominance over the exhibitors (as well as on other
producer and distributors) but that neither prohibited and nor considered as offence. This feature
of being the largest production house and a member of APHA, empowers the KML to put forward
the non-negotiable condition and also influences/forces the majority of the Single Screen
Theatres to agree on the condition (tie-in i.e. to purchase the rights of the film, Single Screen
Theatres have to also purchase the rights of the next film) hence KML also guilty under section
4(1) of the Competition Act, 2002 of abusing the dominance.
CASE STUDY 5
NC Private Limited (NCPL) was established in the year 2015 by Mr. Neelkant Sharma, a young
but successful real estate mogul. The Company's real estate business was managed by
Neelkant and his mother, Ms. SB Lakshmi. NCPL specialised in construction of medium sized
luxury projects with around 80- 100 apartments in each project. With the challenges in the real

© The Institute of Chartered Accountants of India


Page 25
PAPER – 6D: ECONOMIC LAWS 27

estate industry, NCPL was running in a fairly profitable manner and was able to complete its
projects timely and thereby earned a good name in the Bangalore market.
In 2019, Mr. Neelkant wanted to increase the presence of the NCPL brand and decided to
commence 4 super premium projects in Bangalore and Mysore.

Project Name Size Number of apartments


Neelkant Nandanam, Bangalore 500 sq.m 12
Neelkant Sankalp, Bangalore 50,000 sq.m 500
Neelakant Shristi, Mysore 5,000 sq.m 80

Neelkant Bhagyam, Bangalore 5,000 sq.m 100

He discussed this with his father, Mr. Yagna Sharma, who is also a Director and CFO in NCPL
and was evaluating the source of funds for these projects.
NCPL decided that the booking for all the projects will start after 15th December 2019 after
obtaining the required permissions under the Real Estate (Regulation and Development) Act,
2016 (RERA). In the Board meeting held on 5th December 2019, it was decided that in view of
the shortfall of funds at this stage, the budget for two projects was reduced. NCPL decided to
reduce the number of apartments in Neelkant Nandanam to 8 and in the case of Neelkant
Sankalp, the construction will take place in two phases. In the first phase, 25,000 sq.m will be
developed to construct 250 apartments and the balance will be done in phase II.
NCPL started to obtain bookings for the projects from 24 th December 2019 post obtaining the
required approvals with the cost of the apartments ranging from `300 lakhs to `500 lakhs in
the projects. NCPL also gave an extra 2% discount to those who book the apartment within 3
months from the commencement of the construction.
Ms. SB Lakshmi, apart from managing 'the real estate business was also providing independent
consultancy on real estate matters in India and abroad. In 2020, she went to visit her younger
brother, Mr. Anand, who was pursuing his masters in Germany. During her visit, she was invited
by the Hamburg University to give a lecture to the students on nuances in real estate
management and she earned a honorarium of USD 1,500. After her return, she remitted USD
150,000 to Mr. Anand for his education expenses which includes college fees, accommodation
and food expenses. She also remitted an amount of USD 5,000 from her RFC account as her
gift for his birthday so he could travel to the UK to watch a football match of his favourite team,
Arsenal.
Mr. Manohar Reddy, a registered real estate agent, wanted to get associated with NCPL for
selling the flats in Bangalore and Mysore and Manohar gave an advertisement without NCPL's

© The Institute of Chartered Accountants of India


Page 26
28 FINAL (NEW) EXAMINATION: JULY 2021

knowledge, in the newspapers for the sale of the apartments with an offer that whosoever book
any apartment via Manohar, they will get extra one percent discount in the booking amount.
NCPL got a very good response for the 3 projects in Bangalore. However, the project in Mysore
got a lukewarm response with only 50% booking. In the Board Meeting held in March 2020, it
was decided that the Company will sell the Mysore project to another third-party real estate
developer, SI Projects Limited (SIPL) which was approved by 26 allottees who had purchased
the apartments till date. Further, NCPL opined that the approval of 2 apartment owners were
not required, since they had acquired the apartment through a transfer from the original allottees
and therefore, are not to be considered for this purpose. After taking over the project and with
75% of the apartments sold, SIPL made certain changes in the layout of the project to move the
position of the swimming pool from the ground floor to the roof top. This would provide more
space for SIPL to include additional amenities for the use of the allottees. These changes were
approved by 38 original allottees and the 2 apartment owners who had purchased through a
transfer. The NCPL observed that SIPL have not adhered to the provisions of the Real Estate
(Regulation and Development) Act, 2016, have altered plan and specification.
In 2020, Mr. Neelkant Sharma wanted to acquire some land in Whitefield for building a large
duplex home for himself and he zeroed in on a piece of land close to his current residence,
which belonged to Mr. Deepak Kumar, his cousin brother and an NRI. Mr. Deepak Kumar had
emigrated to the USA in 2015 and married Ms. Ashlin, a citizen of USA in 2016. A consideration
of INR 700 lakhs was agreed, out of which INR 600 lakhs was paid by Mr. Neelkant Sharma
from his RFC account and the remaining amount was paid as a gift through a crossed cheque
to Mr. Deepak Kumar, who deposited this in his NRO account. In 2021, Ms. Ashlin wanted to
acquire one of the apartments constructed by SIPL, jointly with Mr. Deepak and agreed to remit
an amount of `150 lakhs from her earnings outside India through normal banking channels.
However since SIPL insisted on an immediate initial booking advance, she gave traveller's
cheque for an amount of USD 10,000 during her visit to India, however the CFO of SIPL is
skeptical regarding the compliance of the Foreign Exchange Management (Acquisition and
Transfer of Immovable property in India) Regulations, 2018.
Answer the following questions:
5.1 Mr. Yagna Sharma, seeks your views on whether NCPL can obtain External Commercial
Borrowings from an interested overseas investor for funding the new projects, under the
automatic route?
(A) Yes, subject to compliance with RBI ECB Regulations with regard to all in cost ceiling,
minimum average retention period etc;
(B) No, proceeds from ECB cannot be used for real estate activities;
(C) Yes, prior approval of the Authorised Dealer is required;

© The Institute of Chartered Accountants of India


Page 27
PAPER – 6D: ECONOMIC LAWS 29

(D) No, unless the loans are obtained by NCPL from an NBFC who in turn has obtained
the ECB. (2 Marks)
5.2 NCPL decided to construct the Neelkant Sankalp in two phases due to shortage of funds.
What shall be the impact of the decision on the project?
(A) No, if the second phase is started immediately after completion of phase 1, no
separate registration is required for the second phase;
(B) No, both the phases are part of one project and hence, no separate registration is
required;
(C) Yes, each phase will be considered as a standalone project and separate registration
is required;
(D) None of the options. (2 Marks)
5.3 Which of the projects of NCPL do not require registration with RERA?
(A) Neelkant Nandanam;
(B) Neelkant Bhagyam;
(C) Neelkant Shristi;
(D) All projects require registration. (2 Marks)
5.4 What is your view regarding the position taken by NCPL on the exclusion of 2 apartment
owners for the purpose of considering their need for approval of transfer of infrastructure
project to SIPL?
(A) Yes, the position is correct, since they are not the original acquirers of the apartment
from NCPL and NCPL is not obligated to the current owners;
(B) No, the position is incorrect, the subsequent owners of the apartment are also to be
considered as allottees under RERA;
(C) Yes, their position is correct, unless the original allottees obtaining NCPL's consent
prior to transferring their ownership to the current apartment owners;
(D) No, NCPL has obtained the approval from sufficient number of original allottees and
therefore, they do not need to consider the other allottees. (2 Marks)
5.5 Ms. SB Lakshmi seeks your advice on the next steps to be undertaken by her with regard
to the foreign currency earned by her during her visit to Germany,
(A) She is required to surrender the same to the authorised person within 180 days of
her return to India;

© The Institute of Chartered Accountants of India


Page 28
30 FINAL (NEW) EXAMINATION: JULY 2021

(B) She is allowed to retain the said sum in foreign currency without any time limit;
(C) She is allowed to retain the said sum, provided she obtains the approval of the
authorised dealer;
(D) She is allowed to retain the said sum, provided she converts the currency into
traveller's cheque. (2 Marks)
5.6 Evaluate with reasons, whether the remittances made by Ms. SB Lakshmi are in
compliance with the provisions of the Foreign Exchange Management Act, 1999. Would
your response change if the education Fee is USD 300,000 (instead of USD 150,000 as
mentioned in the Case Study). (3 Marks)
5.7 Examine and analyse the implications of the transfer of Land by Mr. Deepak to Mr. Neelkant
Sharma and remittance thereof under the Foreign Exchange Management (Acquisition and
Transfer of immovable Property in India) Regulations, 2018. (3 Marks)
5.8 SIPL's CFO is of the opinion that stipulated booking of flat by Ms. Ashlin violates the
conditions stipulated as under the Foreign Exchange Management (Acquisition and
Transfer of immovable property in India) Regulations, 2018, kindly examine. (3 Marks)
5.9 Whether the contention of NCPL is justified that SIPL is within their legal rights to alter the
plan and specification. Examine the same in light of provision of the Real Estate
(Regulation and Development) Act, 2016. (6 Marks)
Answer to Case study 5
5.1 (C)
5.2 (C)
5.3 (A)
5.4 (B)
5.5 (B)
Answer 5.6
According to Schedule III of the FEM (Current Account Transaction) Rules, 2000, individuals
can avail of foreign exchange facility for the prescribed purposes within the limit of USD 250,000
only. Any additional remittance in excess of the said limit shall require prior approval of the
Reserve Bank of India.
In the given case study, Ms. SB Lakshmi remitted USD, 150,000 to his brother, Mr. Anand for
his education expenses including fees, accommodation and food expenses. She also remitted
an amount of USD 5,000 from her RFC account as gift on account of his birthday to him.

© The Institute of Chartered Accountants of India


Page 29
PAPER – 6D: ECONOMIC LAWS 31

Accordingly, here since the remittance is for the education and his maintenance along with the
gift of amount are the prescribed purposes under Schedule III within the limit of USD 250,000
(i.e., USD 155,000= 150,000+5,000). Hence, no prior approval of RBI is necessitated.
Therefore the said remittances made by the Ms.SB Lakshmi is in compliance with the provisions
of the Foreign Exchange Management Act, 1999.
Where if, education fee is USD 300,000, then she can remit based on confirmation from
University and shall not require prior approval of the Reserve Bank of India.
Answer 5.7
Transfer of land by Mr. Deepak Kumar: As per the Foreign Exchange Management
(Acquisition and Transfer of Immovable Property in India) Regulations, 2018, an NRI or an OCI
may transfer any immovable property in India to a person resident in India. So accordingly, Mr.
Neelkant Sharma can acquire immovable property belonged to Mr. Deepak Kumar (his cousin
brother) and an NRI, residing outside India.
'Remittance outside India' means the buying or drawing of foreign exchange from an
authorised dealer in India and remitting it outside India through banking channe ls or crediting it
to an account denominated in foreign currency or to an account in Indian currency maintained
with an authorised dealer from which it can be converted in foreign currency.
As per the fact, Mr. Neelkant Sharma, remitted INR 600 lakhs from his RFC account and
remaining amount paid as gift through crossed cheque to Mr. Deepak Kumar who deposited this
in his NRO account. This mode of remittance of amount in lieu of sale of immovable property,
made to Mr. Deepak Kumar is in compliance with the said provisions.
Answer 5.8
As per the Foreign Exchange Management (Acquisition and Transfer of Immovable Property in
India) Regulations, 2018, an NRI or an OCI may acquire immovable property in India ,Provided
that the consideration, if any, for transfer, shall be made out of (i) funds received in India through
banking channels by way of inward remittance from any place outside India or (ii) funds held in
any non resident account maintained in accordance with the provisions of the Act, rules or
regulations framed thereunder.
Provided further that no payment for any transfer of immovable property shall be made either
by traveller’s cheque or by foreign currency notes or by any other mode other than those
specifically permitted under this clause.
As per the facts, remittance of an amount of ` 150 lakh to be made by Ms. Ashlin, from her
earnings through normal banking channels was permissible. However, on being insisted by SIPL
on making immediate initial booking advance, payment of an amount of USD 10,000 by
Ms. Ashlin through traveller’s cheque during her visit in India, is not permissible. Therefore

© The Institute of Chartered Accountants of India


Page 30
32 FINAL (NEW) EXAMINATION: JULY 2021

stipulated booking of flat by Ms. Ashlin violates the requirements of Foreign Exchange
Management (Acquisition and Transfer of Immovable Property in India) Regulations, 2018.
Answer 5.9
Adherence to sanctioned plans and project specifications by the promoter under RERA
(Section 14)
Section 14 of the RERA requires a promoter to adhere to the sanctioned plans and the project
specifications. According to it the proposed project shall be developed and completed by the
promoter in accordance with the sanctioned plans, layout plans and specifications as approved
by the competent authorities.
If any additions and alterations are required to be carried out it shall be done with the previous
consent of the concerned person. However, minor additions or alterations as may be required
by the allottee can be made by the promoter. In case, certain minor changes or alterations as
are necessary due to architectural and structural reasons, are to be made by the promoter, they
shall be duly recommended and verified by an authorised Architect or Engineer after proper
declaration and intimation to the allottee.
Further, in case of any other alterations or additions in the sanctioned plans, layout plans and
specifications of the buildings or the common areas within the project, the promoter is permitted
to do so but he must obtain the previous written consent of minimum two-thirds of the concerned
allottees.
If an allottee brings to the notice of promoter any structural defect, etc. or some of his other
unfulfilled obligation as per the agreement for sale within five years from the date of possession,
the promoter shall be duty-bound to rectify such defects without further charge, within thirty
days. If not so rectified within the specified time, the aggrieved allottee shall be entitled to
receive appropriate compensation.
In the given case study, changes were approved by 2/3 (i.e.40/60*100) of the original allottees,
therefore contention of NCPL is not justified and SIPL is within their legal rights to alter the plan
and specification in compliance with the stated requirements of the RERA.

© The Institute of Chartered Accountants of India


Page 31
2 FINAL (NEW) EXAMINATION: JANUARY 2021

PAPER 6D: ECONOMIC LAWS


NOTE: The question paper comprises of five case study questions. The candidates are
required to answer any four case study questions out of five.
CASE STUDY - 1
Part-A:
TJSB Sahakari Bank Ltd. (hereinafter called 'Petitioner') has sought the Corporate Insolvency
Resolution Process of Unimetal Castings Ltd. (hereinafter called the 'Corporate Debtor') on
the ground, that the Corporate Debtor committed default in repayment of loan facilities granted
to the Corporate Debtor to the extent of `6,38,78,417/- including interest of `2,07,95,568/-,
under Section 7 of the Insolvency and Bankruptcy Code, 2016 (hereafter called the 'Code')
read with Rule 4 of the Insolvency and Bankruptcy (Application to Adjudicating Authority)
Rules, 2016.
The Petition reveals that the following credit facilities were sanctioned on 25-02-2013 to the
Corporate Debtor by SVC Bank consortium wherein the Petitioner Bank is the consortium
member:
Sr. No. Facility By TJSB (Petitioner Bank) By SVC Bank
1. CC Limit 60,00,000 3,90,00,000
2. OBD Limit - 1,00,00,000
3. Term Loan 1 45,50,000 1,63,80,000
4. Term Loan 2 90,00,000 75,55,000
5. Term Loan 3 1,11,50,000 36,90,000
6. Term Loan 4 50,00,000 1,77,81,000
7. Term Loan 5 1,50,00,000 -
TOTAL 5,10,00,000 9,44,06,000
The Petitioner on 04-08-20I5 issued recall notice to the Corporate Debtor under the provisions
of Multistate Co-Operative Societies Act, 2002 and further issued SARFAESI notice on 29-03-
2016.
The Corporate Debtor submitted that:
(a) It is a medium enterprise as defined under the Micro, Small and Medium Enterprises
Development Act, 2006 (MSMED Act').
(b) The declaration of the account of the Corporate Debtor as Non-Performing Asset ('NPA')
w.e.f. 30-06-2015 is illegal, void and non-est as the same is in contravention of
Regulations and Circulars issued by the Government, Reserve Bank of India, etc.

© The Institute of Chartered Accountants of India


Page 32
PAPER – 6D: ECONOMIC LAWS 3

(c) The claim of ` 6,38,78,417 as claimed in the Petition is not due and payable by the
Corporate Debtor.
(d) The Corporate Debtor being a medium enterprise is statutorily recognised as extremely
important for the national economy and certain rights are provided under section 9 & 10
of MSMED Act.
(e) The Corporate Debtor is entitled to request the consortium members including the
Petitioner herein for restructuring the credit facilities as provided under RBl guidelines
such as "Prudential guideline on restructuring of advances by banks" and "Guidelines for
rehabilitation of sick, micro and small enterprises". The Central government has also
notified the "Framework for revival and rehabilitation- of micro, small and medium
enterprises". Despite the request of the Corporate Debtor in the year 2014 and 201 5 the
Petitioner or any other Financial Institution has not made any attempts to restructure the
facilities granted to the Corporate Debtor.
(f) Consequent to the meeting of the District Level Sick Unit Rehabilitation Committee held
on 15-03-2016 under the chairmanship of the District Collector of Kolhapur and the
meeting convened by the Joint Director of Industries, Pune, the petitioner by necessary
implications agreed to undertake the exercise of getting the requisite Eco- Techno
viability report of the Corporate Debtor in order to assist the eligibility/entitlement for the
purpose of availing the rehabilitation program but the petitioner failed to do that.
(g) The issue of SARFAESI notice dated 29-03-2016 by the petitioner under section 13(2) of
the SARFAESI Act, 2002 shows their high handedness in exploiting its dominant position
vis-a-vis the Corporate Debtor.
The Corporate Debtor further contended that, the claim of the Petitioner is barred under Article
137 of the Limitation Act, 1963 since whilst the date of alleged default was on 30-06-2015 i.e.
the date on which the account was declared as Non-Performing Asset (NPA). The cause of
action (i.e. the actual default) would have arisen much prior to the date of NPA. Hence, the
period of limitation would run starting even prior to 30-06-2015 and since this Petition was filed
on 23-08-2018, this Petition is barred by limitation.
For the above contention of the Corporate Debtor, the Petitioner submitted that the loan was
shown in the balance sheet of the Corporate Debtor which is an acknowledgement of liability
and hence the debt is not barred by limitation. The Corporate Debtor has not disputed the fact
that the loan was shown as a liability in the balance sheet of the Corporate Debtor.
The Adjudicating Authority having satisfied with the fact that the Corporate Debtor defaulted in
making payment towards the liability to the petitioner, ruled that the petition deserves to be
admitted under IBC, 2016.

© The Institute of Chartered Accountants of India


Page 33
4 FINAL (NEW) EXAMINATION: JANUARY 2021

Another operational creditor, Wonder Bearings Limited, who underwent a corporate insolvency
resolution process which got completed on 15-03-2016, filed a petition under IBC 2016 on
10-05-2017 with regard to its dues from Unimetal Castings amounting to `1,50,80,000.
The Petitioner "TJSB Sahakari Bank Limited" seeks your view on the various provisions of the
Insolvency and Bankruptcy Code, 2016 with regard to above matter.
Part-B:
In another independent development various appeals were filed by the appellants/suppliers
against the orders passed by the Hon'ble Competition Appellate Tribunal (hereinafter referred
to as COMPAT) before the Hon'ble Supreme Court of India. The COMPAT, by the said
judgment, has upheld the findings of the Competition Commission of India (for short, CCI) that
the appellants/suppliers of Liquefied Petroleum Gas (LPG) Cylinders to the Indian Oil
Corporation Ltd. (for short, IOCL) had indulged in cartelization, thereby influencing and rigging
the prices, thus, violating the provisions of Section 3(3)(d) of the Competition Act, 2002 (for
short, the Act).
These suppliers have filed the instant appeals on the ground that there was no cartelization
and they have not contravened the provisions of the Act. For the sake of convenience these
suppliers will be referred to as the appellants hereinafter. We may point out at the outset that
all these appellants are manufacturing gas cylinders of a particular specification having
capacity of 14.2 kg which are needed for use by the three oil companies in India, namely,
IOCL, Bharat Petroleum Corporation Ltd. (BPCL) and Hindustan Petroleum Corporation Ltd.
(HPCL) [all are public sector companies]. It is also a matter of record that apart from the
aforesaid three companies there are no other buyers for these cylinders manufactured by the
appellants. Insofar as IOCL is concerned, it is a leading market player in LPG as its market
share is 48%. Thus, in case a particular manufacturer is not able to supply its cylinders to the
aforesaid three companies, there is no other market for these cylinders and it may force that
company to exit from its operations. The technical bid of the subject tender was opened on 3 -
3-2010 and the price bids of 50 qualified bidders were opened on 23-3-2010. According to the
Director General, there was a similar pattern in the bids by all the 50 bidders who submitted
price bids for various States. The bids of a large number of parties were exactly identical or
near to identical for different States. The Director General had observed that there were strong
indications of some sort of agreement and understanding amongst the bidders to manipulate
the process of bidding.
As per the Director Generals report, the process of bidding followed by the IOCL in the tender
was as under: -
• The bidders would submit their quotations with the bid documents.
• The existing bidders, who were existing suppliers, were required to submit the price bids
and technical bids.
• The bidders were to quote for supplies in different States of India in keeping with their
installed capacity.

© The Institute of Chartered Accountants of India


Page 34
PAPER – 6D: ECONOMIC LAWS 5

• After price bids were opened the bidders were arranged according to the rates in the
categories of L-1, L-2 and L-3.
• The rates for the supplies in different States were approved after negotiations with L-I
bidder. In case the L-1 bidder could not supply a required number of cylinders in a
particular State, the orders of supplies went to L-2 and also L-3 bidder or likewise
depending upon the requirement in that State as per fixed formula provided in the bid
documents.
The Director General after analyzing the bids came to the conclusion that there was not only a
similarity of pattern in the price bids submitted by the 50 bidders for making supply to the
IOCL but the bids of large number of parties were exactly identical or near to identical in
different States. It was also found that bidders, who belonged to same group, might have
submitted identical rates.
The similarity of the rates was found even in case of bidders whose factories and offices were
not located at one and the same place in the States and where they were required to supply
was far off from their factories located in different place.
The D.G. had found further that though the factors like market conditions and small number of
companies were different, there was large scale collusion amongst the bidding parties. He
also arrived at a finding to the effect that the LPG Cylinder Manufacturers had formed an
Association in the name of Indian LPG Cylinders Manufacturers Association and the members
were interacting through this Association and were using the same as a platform. The date for
submitting the bids in the case of the concerned tender was 3-3-2010 and just two days prior
to it, two meetings were held on 1st and 2nd March, 2010 in Hotel Sahara Star in Mumbai. As
many as 19 parties took part and discussed the tender and, in all probability, prices were fixed
there in collusion with each other. The D.G reported that the bidders had agreed for allocation
of territories, e.g., the bidders who quoted the bids for Western India had not generally quoted
for Eastern India and that largely the bidders who quoted the lowest in the group in Northern
India, had not quoted generally in Southern India. The D.G. also concluded that this behaviour
created entry barrier and that there was no accrual of benefits of consumers nor were there
any plus factors like improved production or distribution of the goods or the provision of
services.
Ultimately, the D.G. came to the conclusion that there was a cartel like behaviour on the pa rt
of the bidders and that the factors necessary for the formation of cartel existed in the instant
case. It was also found that, there was certainly a ground to hold concerted action on the part
of the bidders. The D.G. had also noted that the rates quoted for the year 2009-10 and in
years previous to that were also identical in some cases. Thus, he came to the conclusion that
the bids for the year 2010-11 had been manipulated by 50 participating bidders. It was
thereafter that the CCI decided to supply the D.G,s investigation report to the concerned
parties and invite their objections.
The Director General seeks your advice in light of Petition filed before the Hon'ble Supreme
Court of India against the order passed by the Hon'ble Competition Appellate Tribunal.

© The Institute of Chartered Accountants of India


Page 35
6 FINAL (NEW) EXAMINATION: JANUARY 2021

Answer the following questions:


1.1 TJSB Sahakari Bank would like your views on, which of the following will not be
considered as insolvency resolution costs under the Code:
(A) The amount of any interim finance and the costs incurred in raising such finance;
(B) The fees payable to any person acting as a resolution professional;
(C) Any payment of fees for the services of an insolvency professional to any person
other than the insolvency professional;
(D) Any costs incurred at the expense of the Government to facilitate the insolvency
resolution process. (2 Marks)
1.2 When can the Committee of Creditors of Unimetal Castings Ltd. take the decision to
liquidate the Company?
(A) by simple majority any time during the resolution process but not before the
confirmation of the resolution plan and preparation of the information memorandum;
(B) by 2/3 rd majority any time during the resolution process but before the confirmation
of the resolution plan and preparation of the information memorandum;
(C) by 2/3 rd majority any time during the resolution process but not before the
confirmation of the resolution plan and preparation of the information memorandum;
(D) by 3/4 th majority any time during the resolution process but before the confirmation
of the resolution plan and preparation of the information memorandum. (2 Marks)
1.3 Is Wonder Bearings Ltd. eligible to initiate insolvency resolution process against Unimetal
Castings Ltd.?
(A) Not eligible, since requirement is to have completed the resolution process 24
months preceding the date of application;
(B) Eligible, there is no bar for a company who underwent insolvency resolution process
to initiate proceedings as long as the other requirements (existence of debt etc.)
under IBC 2016 is met;
(C) Eligible, since requirement is to have completed the resolution process 12 months
preceding the date of making of the application;
(D) Not eligible, prior consent of the Adjudicating Authority is required for filing an
application for insolvency process by Wonder Bearings Ltd., since it has itself
undergone an insolvency process. (2 Marks)
1.4 Which of the following is not the objective of the Competition Act, 2002?
(A) Promote practices having adverse effect on Competition;
(B) Sustain competition in market;
(C) Protect the interest of consumers;

© The Institute of Chartered Accountants of India


Page 36
PAPER – 6D: ECONOMIC LAWS 7

(D) Ensure freedom of trade for Indian and foreign players in markets in India. (2 Marks)
1.5 An Association of manufacturers of die cast products will not be considered as a cartel if
the objective of the association is to :
(A) limit the distribution of die cast material only to petroleum industry in view of the
huge demand and higher realization;
(B) regulate the production of die cast products to ensure optimal sale prices;
(C) represent the industry issues on a collective basis to the Government;
(D) monitor and regulate the number of dealers in each state/city. (2 Marks)
1.6 Analyze and answer the following questions in the context of the case study:
(i) Evaluate the position taken by the Adjudicating Authority that the petition deserves
to be admitted having satisfied with the fact that Unimetal Castings Ltd. has
defaulted in making payment towards the liability to the petitioner. (4 Marks)
(ii) In light of the provisions of the Competition Act 2002, whether there was any
collusive agreement between the participating bidders which directly or indirectly
resulted in bid rigging of the tender floated by IOCL? (3 Marks)
(iii) Unimetal Castings Ltd. seeks your views regarding the impact of the clarifications
issued by Ministry of Corporate Affairs (MCA) regarding approval of resolution plans
under section 30 and 31 of Insolvency & Bankruptcy Code, 2016 vide general
circular (GC) dated 25th October, 2017. (4 Marks)
(iv) TJSB Sahakari Bank Ltd. seeks your advice on the time-limit for completion of the
Corporate Insolvency Resolution Process as per the lBC, 2016. (4 Marks)
ANSWER TO CASE STUDY 1
1.1 (C)
1.2 (B)
1.3 (C)
1.4 (A)
1.5 (C)
1.6 (i)
Given situation is based on the case law, B.K. Educational Services Pvt. Ltd. vs. Parag Gupta
and Associates, of Supreme Court, Civil Appeal No.23988 of 2017, dated 11.10.2018, in which
principle was laid down that the Limitation Act, 1963 is applicable to applications filed under
Sections 7 and 9 of the Insolvency and Bankruptcy Code, 2016(Code) from the inception of
the Code .i.e. from 28th May, 2016.

© The Institute of Chartered Accountants of India


Page 37
8 FINAL (NEW) EXAMINATION: JANUARY 2021

In the given case study, the Corporate Debtor, Unimetal Castings Ltd. contended as under:
(i) Claim of the Petitioner, TJSB Sahakari Bank Ltd. is barred under Article 137 of the
Limitation Act, 1963.
(ii) Whilst the date of alleged default was 30.6.2015 (i.e. the date on which the account was
declared as Non Performing Assets (NPA)) and the cause of action (i.e. actual default)
arises much prior to the date of NPA.
(iii) The period of limitation would have started even prior to 30.06.2015 and as the petition
was filed on 23.08.2018, so it is barred by limitation.
As per the facts given in the case study, acknowledgement of liability in the Balance Sheet of
the Corporate Debtor reflects that default has already occurred i.e on 30.6.2015 and the
application for initiation of Corporate Insolvency Resolution process was filed on 23.08.2018.
Where the liability is shown in the balance sheet, it is a clear acknowledgement of debt by the
Corporate Debtor as was held in Bajan Singh Sharma Vs Wimpy International Limited,
185(2011) DLT 428 and in many other judgements.
In the light of the aforesaid ruling, the limitation period of 3 years will begin from the date of
coming of Code into enforcement i.e from 28th May 2016.
Therefore, the position taken by the Adjudicating Authority is correct and the petition deserves to
be admitted since the application filed under Section 7 of the Code is within the limitation period
and the Corporate Debtor has defaulted in making payment towards the liability of the petitioner.
1.6(ii)
As per Section 3(3) of the Competition Act, 2002 the identical bid price is not possible unless
there is some sort of prior and collective understanding. Further the contact and meeting
between the members of IOCL and Association, before submission of bids is also valid
evidence of the existence of an understanding among the parties.
In the case study as per given facts, it was found that there was large scale collusion amongst
the bidding parties. LPG Cylinder manufacturers formed an association, Indian LPG Cylinder
manufacturers Association and the members of IOCL were interacting through this
association. Two days before the date of bids i.e. on 1 st& 2 nd March, 2010 two meetings were
held and 19 parties took part and discussed the tender and prices were fixed there in collu sion
with each other. This resulted in bid rigging of the tender floated by IOCL.
In view of the above, it can be concluded that there was collusive agreements between the
participating bidders which directly or indirectly resulted in bid rigging of the te nder floated by
IOCL.
1.6 (iii)
Impact of Clarification Issued by MCA:
Vide General Circular IBC/01/2017 dated 25th October, 2017, Ministry of Corporate
Affairs issued a clarification regarding approval of resolution plans under Section 30 and 31 of

© The Institute of Chartered Accountants of India


Page 38
PAPER – 6D: ECONOMIC LAWS 9

Insolvency and Bankruptcy Code, 2016. The said clarification is sought in view of the
requirement under Section 30(2)(e) of the Code for the resolution professional to confirm that
each resolution plan received by him does not contravene any of the provisions of the law for
the time being in force.
The matter has been examined in the Ministry in the light of provisions of Sections 30 and 31
of the Code which provide a detailed procedure from the time of receipt of resolution plan by
the resolution professional to its approval by the Adjudicating Authority and there is no
requirement for obtaining approval of shareholders/members of the corporate debtor during
this process.
This clarification clears that the requirement of Section 30(2) (e) of the Code is to ensure that
the resolution plan(s) considered and approved by the Committee of Creditors and the
Adjudicating Authority is in compliant with the provisions of the applicable laws and therefore
is legally implementable.
Section 31(1) of the Code further provides that a resolution plan approved by the Adjudicating
Authority shall be binding on the Corporate Debtor and its employees, members, creditors,
guarantors and other stakeholders involved in the resolution plan.
1.6 (iv)
As per Section 12 of the Insolvency and Bankruptcy Code, 2016, the Corporate Insolvency
Resolution Process (CIRP) shall be completed within a period of one hundred and eighty days
from the date of admission of the application to initiate such process.
The Resolution Professional shall file an application to the Adjudicating Authority to extend the
period of the Corporate Insolvency Resolution Process beyond one hundred and eighty days,
if instructed to do so by a resolution passed at a meeting of the Committee of Creditors by a
vote of sixty- six per cent of the voting shares.
On receipt of an application, if the Adjudicating Authority is satisfied that the subject matter of
the case is such that CIRP cannot be completed within one hundred and eighty days, it may
by order extend the duration of such process beyond one hundred and eighty days by such
further period as it thinks fit, but not exceeding ninety days.
Provided that any extension of the period of CIRP shall not be granted more than once.
Accordingly, in the said case, time limit for competition of CIRP, will be:
01 Petition for initiation of corporate insolvency resolution was filed on 23.08.2018
02 Insolvency resolution process will be commenced within 14 days i.e, latest by
01.09.2018
03 Insolvency resolution process will be completed by 180th day from insolvency
commencement date (date of admission of the application) i.e., latest by
28.02.2019.
04 Further may extend till 29.05.2019.

© The Institute of Chartered Accountants of India


Page 39
10 FINAL (NEW) EXAMINATION: JANUARY 2021

Amendment of Section 12(3) of IBC(Amendment) Act, 2019


Section 12(3) of the IBC was amended by way of the Insolvency and Bankruptcy (Amendment)
Act, 2019 and two provisos were added:
Proviso 1 states that a CIRP must mandatorily be completed within 330 days from the
insolvency commencement date, including any extension of the period of the CIRP granted
and the time taken in legal proceedings in relation to the resolution process.
Proviso 2 states that, when the CIRP of a Corporate Debtor (CD) has been pending for over
330 days, it must be completed within 90 days from the date of the amendment.
Thus, the overall timeline for completing a CIRP now stands at 330 days from the date of
insolvency commencement date.
CASE STUDY - 2
Visio India Private Limited (Visio) is an upcoming watch manufacturing company and is based
in Vishakapatnam. The Company was started during the year 2008 by Mr. Srinivas Kumar and
his wife, Ms. Kruthi who is a Chartered Accountant.
In order to meet their expanding operations in Delhi, Visio had in the month of January 2014
pre-booked a commercial office unit of approximately 1200 sq. ft. with JV Realty Limited, a
leading developer in that area in their "SAPPHIRE COURT" Greater Noida project launched
then by paying an amount of `25,00,000 as booking amount (50% of the total consideration)
but no Builder- Buyer agreement was entered into between the parties except that an
allotment letter was issued by the developer mentioning the unit details. This project was
being developed over an area of approximately 15,000 square meters and having over 100
office units in its plan outlay.
Visio had paid almost 90% of the entire cost of the property based upon percentage of
completion (progress) of the stage of construction as of April 2017 but the developer had
failed to provide, neither possession nor had completed the project and was also not
responding to their complaints on one pretext or the other.
The legal counsel of Visio, Mr. Aswin Nakshatra, in the month of May, 2017 informed Ms.
Kruthi about Real Estate (Regulation and Development) Act, 2016 (for short "the RERA"). He
further informed that RERA was enacted by the Parliament as Act 16 of 2016 in the year 2016
and by May 1, 2017, all 92 provisions of the Real Estate (Regulation and Development) Act,
2016 (RERA or the Act) were brought into force. The Act has introduced new obligations on
real estate developers and in cases of default, prescribes penal liabilities and Visio can
contemplate bringing a legal suit against the developers under RERA.
JV Realty on the other hand is of the view that RERA is not applicable to this project as the
same was launched and construction commenced much before the RERA came into force.
One of the group companies of JV Realty, Good Looking Homes Private Limited (GLHPL) was
into construction of high rise apartment complexes and commenced a large project "Kailash
Giri Views" in Vizag.

© The Institute of Chartered Accountants of India


Page 40
PAPER – 6D: ECONOMIC LAWS 11

GLHPL took all the approvals under RERA and came up with the marketing strategy including
a brochure of 94 pages consisting of various pictures showing the following features included
in the project:
(a) Balcony at each floor
(b) Drawing room to be constructed with designed tiles at floor.
(c) Italic Marble at bedroom.
(d) Granite at kitchen.
(e) Swimming pool at the top floor.
(f) All rooms to be Centrally Air-conditioned.
(g) All floors and lifts will have CCTV camera.
(h) Open parking slot for one car.
(i) Ground covering Net for Cricket and Football.
(j) Handover of the apartments within 36 months from date of agreement.
It was also mentioned in the marketing brochure that the building will have 9 floors with
elevators and stair case and the total number of flats to be constructed would be 218 as
approved by RERA.
For the purpose of various projects, JV Realty had obtained several loans from banks and
financial institutions and there were certain allegations that some of the loan funds were
siphoned off by the promoters of JV Realty for other purposes. In five different cases, banks
and financial institutions had granted credit facilities against hypothecation/charge over certain
assets. In each of these cases, JV Realty was charged under certain provisions of the PMLA
(for offences under paragraph 2 of Part A of the Schedule) and orders were passed for
attachment of properties charged to banks and financial institutions affecting their vested
rights under other statutes such as Recovery of Debts and Bankruptcy Act (RDBA),
Securitization and Reconstruction of Financial Assets and Enforcement of Securities Interest
Act (SARFAESI) and Insolvency and Bankruptcy Code (lBC).
The Adjudicating Authority is of the view that:
• The provisions of PMLA prevail over RDBA, SARFAESI and IBC
• It is not only a "tainted property" that is to say a property acquired or obtained, directly or
indirectly, from proceeds of criminal, activity constituting a scheduled offence which can
be attached, but also any other asset or property of equivalent value of the offender of
money-laundering which has a link or nexus with the offence (or offender) of money -
laundering.

© The Institute of Chartered Accountants of India


Page 41
12 FINAL (NEW) EXAMINATION: JANUARY 2021

• If the "tainted property" is not traceable, or cannot be reached, or to the extent found is
deficient, any other asset of the person accused or charged under PMLA can be attached
provided it is near or equivalent in value, the order of confiscation being restricted to take
over by the government of illicit gains of crime.
• An order of attachment under PMLA is not illegal only because a secured creditor has a
prior secured interest in the property, within the meaning of the expressions used in
RDBA and SARFAESI. Similarly, mere issuance of an order of attachment under PMLA
does not render illegal a prior charge of a secured creditor, the claim of the latter for
release from PM LA attachment being dependent on its bonafides.
• In case of secured creditor pursuing enforcement of "security interest" in the property
sought to be attached under PMLA, such secured creditor having initiated action for
enforcement prior to the order of attachment under PMLA, the directions of such
attachment under PMLA shall be valid and operative subject to satisfaction of the charge
of such third party and restricted to such part of the value of the property as is in excess
of the claim of the said third party.
• If the order confirming the attachment has attained finality or if the order of confiscation
has been passed or if the trial of a case under Section 4 of the PMLA has commenced,
the claim of a party asserting to have acted bonafide or having legitimate interest in the
nature mentioned above will be inquired into and adjudicated upon only by the special
court.
During the course of Kailash Giri View project, it was observed that whilst the construction was
for 9 floors, the total flats constructed were 225 due to efficient realignment of the blocks and
square feet area of the individual apartments. It was also observed that due to unavoidable
reasons, the swimming pool could only be made at the ground floor only and would be allowed
to those occupants only who will specifically pay for the swimming pool facility. On completion
of 34 months, GLHPL sent an email to all allottees that due to unforeseen circumstances the
project is getting delayed by 6 months as the structure is almost complete and the work
related to interior, plastering, plumbing etc., will be completed very soon.
Answer the following questions:
2.1 Which of the following is not a condition to be fulfilled for attachment of the property of JV
Realty Ltd. for alleged offences under the PMLA?
(A) Approval of the Special Court for the attachment;
(B) Submission of report to a Magistrate under Section 173 of the Code of Criminal
Procedures;
(C) Filing of complaint for taking cognizance of the scheduled offence;
(D) None of the options. (2 Marks)

© The Institute of Chartered Accountants of India


Page 42
PAPER – 6D: ECONOMIC LAWS 13

2.2 What is the punishment which Mr. Bala Ganesh, the Managing Director of JV Realty Ltd.
is liable for under the PMLA ?
(A) No punishment, since the offence was not performed personally by him;
(B) Minimum of 3 years and maximum of 7 years, with fine;
(C) Minimum of 3 years and maximum of 10 years, without fine;
(D) Minimum of 3 years and maximum of 10 years, with fine. (2 Marks)
2.3 On receipt of a complaint under the PMLA, if the Adjudicating Authority has reasons to
believe that JV Realty Ltd. has committed an offence under section 3 or is in possession
of proceeds of crime, it may serve notice within not less than ---------------days calling
upon them to indicate the source of their income, earnings or assets etc.
(A) 15 days
(B) 60 days
(C) 30 days
(D) 7 days (2 Marks)
2.4 JV Realty Ltd. has decided to charge an amount of `5,00,000 on Visio for an open car
parking. Ms. Kruthi is of the view that JV Realty Ltd. cannot charge this amount since this
is not mentioned in the original agreement.
(A) Yes, this cannot be charged since this is not mentioned in the original agreement
between Visio and JV Realty Ltd.;
(B) Yes, this cannot be charged since JV Realty Ltd. cannot charge for open car
parking under RERA;
(C) No, this can be charged since the requirement for non-charging for open car park
under RERA is only for residential complexes and is not applicable for commercial
office space;
(D) This is purely based on mutual agreement between both parties. (2 Marks)
2.5 As per RERA, what is the maximum amount of advance or application fee which can be
collected by GLHPL from its customers?
(A) 15% of cost of apartment on entering into a Sale Deed;
(B) 10% of cost of apartment on entering into a written agreement to sell;
(C) 10% of cost of apartment on entering into a Sale Deed which is duly registered;
(D) 10% of cost of apartment on entering into a written agreement to sell which is duly
registered. (2 Marks)
2.6 Analyze and answer the following questions in the context of the case study:
(i) In the light of the given case study, evaluate if Visio can initiate legal proceedings

© The Institute of Chartered Accountants of India


Page 43
14 FINAL (NEW) EXAMINATION: JANUARY 2021

against JV Realty Ltd. for their resultant rights towards delay in completion or
whether the contention of the developer that RERA is not applicable to the Project
is correct. (4 Marks)
(ii) Discuss the provisions of powers of Director to impose fine under the PMLA,
2002. (5 Marks)
(iii) Based on the provisions of the PMLA, analyse with reasons, the contentions of
the Adjudicating Authority with regard to the following:
(a) Whether the provisions of RDBA, SARFAESI and IBC prevail over PMLA ?
(b) Whether interest created in a property prior to event of money laundering
leading up to the attachment of property, takes priority over the attachment?
(c) Whether a mere nexus between the attached property where it did not qualify
as "proceeds of crime" under the PMLA and the party accused of money
laundering was sufficient for the attachment to take place? (6 Marks)
ANSWERS TO CASE STUDY 2
2.1 (A)
2.2 (B)
2.3 (C)
2.4 (B)
2.5 (D)
2.6(i)
Whether Visio can initiate proceedings against JV Reality ?
According to Section 18 of the Real Estate (Regulation and Development) Act, 2016 (the Act),
(1) If the promoter fails to complete or is unable to give possession of an apartment, plot
or building,—
(a) in accordance with the terms of the agreement for sale or, as the case may be, duly
completed by the date specified therein; or
(b) due to discontinuance of his business as a developer on account of suspension or
revocation of the registration under this Act or for any other reason,
he shall be liable on demand to the allottees, in case the allottee wishes to withdraw from
the project, without prejudice to any other remedy available, to return the amount
received by him in respect of that apartment, plot, building, as the case may be, with
interest at such rate as may be prescribed in this behalf including compensation in the
manner as provided under this Act.

© The Institute of Chartered Accountants of India


Page 44
PAPER – 6D: ECONOMIC LAWS 15

However, where an allottee does not intend to withdraw from the project, he shall be
paid, by the promoter, interest for every month of delay, till the handing over of the
possession, at such rate as may be prescribed.
(2) If the promoter fails to discharge any other obligations imposed on him under this Act or
the rules or regulations made thereunder or in accordance with the terms and conditions
of the agreement for sale, he shall be liable to pay such compensation to the allottees, in
the manner as provided under this Act.
In the case study, Visio had paid almost 90% of the entire cost of the property. The developer
had failed to provide neither possession nor had completed the project.
Hence, Visio can initiate legal proceeding against JV Realty Ltd.
Whether the contention of the developer that RERA is not applicable to the project is
correct ?
As per Section 3 of the Act, RERA applies to projects that are ongoing on the date of
commencement of the Act and completion certificate has not been issued within a period of
three months from the date of commencement of the Act. In the given case study, completion
certificate of the project was not granted till April 2017 (even after RERA was formulated).
Hence, the contention of JV Realty Ltd. that RERA is not applicable to them, is incorrect.
2.6 (ii)
Powers of director to impose fine under the PMLA, 2002
Section 13 of the Prevention of Money Laundering Act, 2002, deals with the powers of the
Director to impose fine, which is as follows:
1. Inquiry from Director: The Director may, either of his own motion or on an
application made by any authority, officer or person, may make such inquiry or cause
such inquiry to be made, with regard to the obligations of the reporting entity.
2. Audit of records on direction of director: If at any stage of inquiry or any other
proceedings before him, the Director having regard to the nature and complexity of
the case, is of the opinion that it is necessary to do so, he may direct the concerned
reporting entity to get its records, audited by an accountant (i.e. Chartered
Accountant) from amongst a panel of accountants, maintained by the Central
Government for this purpose.
3. Bearing of expenses: The expenses of, and incidental to, any audit specified above
shall be borne by the Central Government.
4. Failure in compliance with the obligations: If the Director, in the course of any
inquiry, finds that a reporting entity or its designated director on the Board or any of
its employees has failed to comply with the obligations, then, he may-

© The Institute of Chartered Accountants of India


Page 45
16 FINAL (NEW) EXAMINATION: JANUARY 2021

(a) issue a warning in writing; or


(b) direct such reporting entity or its designated director on the Board or any of its
employees, to comply with specific instructions; or
(c) direct such reporting entity or its designated director on the Board or any of its
employees, to send reports at such interval as may be prescribed on the measures
it is taking; or
(d) by an order, impose a monetary penalty on such reporting entity or its designated
director on the Board or any of its employees, which shall not be less than ten
thousand rupees but may extend to one lakh rupees for each failure.
5. Forwarding of copy of order: The Director shall forward a copy of the order passed
above to every banking company, financial institution or intermediary or person who
is a party to the proceedings.
2.6 (iii)
(a) Whether the provisions of RDBA, SARFAESI and IBC prevail over PMLA?
Section 71 of Prevention of Money Laundering Act, 2002, which deals with the overriding
effect of the act, provides that the provisions of this Act shall have effect notwithstanding
anything inconsistent therewith contained in any other law for the time being in force.
Thus, it can be construed that the provisions of Prevention of Money Laundering Act, 2002,
shall prevail over RBDA, SARFAESI and IBC, till the time nothing inconsistent therewith is
contained in any other law for the time being in force.
(b) Whether interest created in a property prior to attachment of property, takes
priority over attachment?
As per Section 5(4) of the Prevention of Money Laundering Act, 2002, nothing in this section
shall prevent the person interested in the enjoyment of the immovable property attached under
Section 5(1) from such enjoyment.
“Person interested”, in relation to any immovable property, includes all persons claiming or
entitled to claim any interest in the property.
Accordingly, an order of attachment under money laundering Act is not said to be illegal
merely because a person interested (i.e., third party) had a prior interest in such property and
further issuance of an order of attachment under PML Act cannot, by itself, render illegal the
prior statutory right of a person interested in attached property.
Therefore, interest created in a property prior to attachment of property, takes priority over
attachment.

© The Institute of Chartered Accountants of India


Page 46
PAPER – 6D: ECONOMIC LAWS 17

(c) Whether mere nexus between the attached property whether it qualify as a
proceeds of crime and the party accused of money laundering, is sufficient for the
attachment of property?
According to Section 5 of the Prevention of Money Laundering Act, 2002, where the Director
or any other officer for the purposes of this section, has reason to believe, on the basis of
material in his possession, that—
(a) any person is in possession of any proceeds of crime; and
(b) such proceeds of crime are likely to be concealed, transferred or dealt with in any
manner which may result in frustrating any proceedings relating to confiscation of
such proceeds of crime under this Chapter,
he may, by order in writing, provisionally attach such property for a period not exceeding one
hundred and eighty days from the date of the order, in such manner as may be prescribed.
Hence, it is necessary that the attached property should qualify as ‘proceeds of crime’.
However, mere nexus between the attached property whether it qualify as a proceeds of
crime/not, the party accused of money laundering, is sufficient for the attachment of such
property to take place.
CASE STUDY - 3
The eastern part of India is very well known for the production of tea and it is exported world
over. However, the large amount of pendency of the payments by tea mills to tea produc ers
has been a cause of worry and it was decided that a common platform is an essential
requirement to provide solution to this problem. Accordingly, the Eastern Produce Co -
operative Society was formed to ensure the timely collection of sale proceeds from mills. The
Society developed a charter, in form of memorandum for its members, to regulate and control
supply, price, term of sales collection of sale proceeds and recovery if required. This
memorandum is binding on all the members of the Society.
The Society extends the support to growers, by giving them offer to sell their entire farm
produce to Society at mutually agreed price; which the Society will further sale to mills. But the
farmers who avail this facility have to necessarily sell the entire farm produce to the Society,
and the farmer cannot sell any portion of his farm produce directly in the open market.
Further, in order to trade with the mills, deal with regulatory authorities, and financial
institution, the Society decided to promote a Company named Eastern Limited. The extracts
from latest audited financial statements of Eastern Limited are as follows;
Sr. No. Particular Amount (in `crore)
1. Proceed (Net of taxes) from sale 3,500
2. Operating assets 700
3. Paid-up share capital 490
4. Net profit 100

© The Institute of Chartered Accountants of India


Page 47
18 FINAL (NEW) EXAMINATION: JANUARY 2021

With passage of time, Eastern Limited became the big hit, for the role it plays as an
intermediary and in incredible transformation in process of sale of tea by farmers.
Mr. Gaurav who is CEO of Eastern limited, heard about forward integration as method of
expansion and growth strategy. Mr. Gaurav prepared a proposal, which was duly approved by
Board of Directors and then by the members of Eastern Limited company to takeover Eastern
Tea Limited, by acquiring controlling stake from open market. Eastern Tea Limited is in the
business of running tea mills, with a global presence. Mr. Gaurav's wife, Ms. Sheetal, was
residing in Singapore and Mr. Gaurav wanted to send an amount of USD 20,000 per month to
her for her maintenance. However, the CFO of Eastern Limited mentioned to him that this is
not in accordance with FEMA.
Around 60% of sales by Eastern Tea Limited constitute exports of tea majorly to Iran. One
year back, Eastern Tea Limited opened one branch office in Iran, as Iran starts buying tea
from India, in order to settle trade balance; because Iran is blocked from the global financial
system; including using U.S. dollars to transact its oil sales. On such branch office, during last
financial year, an amount of `150 crore were incurred as expenditure for the Branch through
the EEFC account maintained by Eastern Tea Limited.
For last financial year, the turnover of Eastern Tea Limited was recorded at ` 1,200 crore,
which was `110 crore more than year earlier to last financial year; whereas operating assets
as on reporting date were ` 280 crore. The paid- up share capital was ` 130 crore. After
acquisition both the entities were not merged, and both kept their respective separate identity.
For the purpose of enhancing its global sales, Eastern Tea Limited decided to pay commission
for exports of tea under the Rupee State Credit Route at 6% of invoice value. Further, Eastern
Tea also decided to send a gift hamper to its 20 top distributors totaling to a value of USD
1,00,000 (INR 70 lakh).
Eastern Tea Limited has strong domestic Network or tie-up with retail shops and stores
through which they sale their tea under brand name 'leaf’ which constitute around 40% of sale.
Such retail shops and stores are provided with instruction not to charge the price more then
what is suggested by Eastern Tea Limited although lower prices can be charged and specific
jurisdiction is given to each retailer for resale.
According to Mr. Saurabh, who is head of marketing at Eastern Limited, also now look after
marketing at Eastern Tea Limited, in order to acquire substantial market share (in term of new
customers), Eastern Tea Limited has to sell tea at the prices lower than cost. Ignoring the
resistance from the governing body of Eastern Tea Limited, the new pricing policy
implemented. Resultantly price decreased from `150 per kilogram to `130 per kilogram. But
in order to restrict loss, on account of selling tea at price lower than cost, Eastern Tea Limited
asked all the shopkeepers and stores, not to sell more than 5 kg of leaf tea to a customer.
The Eastern Produce Co-operative Society promoted another company named South Limited,
whose object clause includes; provide weather research and forecast reports, other necessary

© The Institute of Chartered Accountants of India


Page 48
PAPER – 6D: ECONOMIC LAWS 19

technical knowledge or guidance to members of parent's society apart from conducting market
research for Eastern Limited.
In one market research conducted by South Limited, it was found that North Limited, which
holds major market share (around 30%) in retail packed tea under brand name 'Taste' (Price
of which is ` 150 per kilogram). For latest financial year, the turnover of North Limited is
recorded at 3,000 crore whereas operating assets are of ` 570 crore and paid-up share
capital is `365 crore. Since acquisition of Eastern Tea limited by Eastern Limited, remains
largely successful, hence showing trust in un-organic growth, a bear-hug letter was sent to
senior management of North Limited.
Since North Limited is already undisputed market leader, they refuse the bear hug offer.
Eastern Limited with help of South Limited performs a hostile acquisition and both the
companies acquire around 25.5% stake in voting rights each; by tender notice over the stock
exchange. Post acquisitions of North Limited, Eastern Limited got the dominance over the
market. Hence Eastern Limited decided to re-price their product which is renamed also 'Taste
leaf’ with a new price of `155 per Kilogram and to support the price rise, Eastern Limited also
started restricting supply in the end market.
Eastern Limited also entered in memorandum of understanding with West offshore Limited,
which is $ 21 million (assets base) company for transfer of technology.
Answer the following questions:
3.1 The CFO of Eastern Tea Limited seeks your views on whether the gifts sent to
distributors requires to be included in the export declaration
(A) Yes, since the aggregate value of the gifts to all distributors is more than the
prescribed limit;
(B) No, since no amounts are received from the distributors for the products;
(C) No, 'since the individual value of the gifts to each distributor is less than the
prescribed limit;
(D) Yes, any item exported should be included in the declaration, unless it will be
returned back to India. (2 Marks)
3.2 How much can Gaurav remit to his wife living in Singapore for her maintenance every
year?
(A) A maximum of USD 2,50,000 with the approval of the Reserve Bank of India;
(B) A maximum of USD 2,50,000 under the liberalized remittance scheme;
(C) Any amount subject to the approval of the Central Government;
(D) Any amount subject as long as he can prove that the amount has been earned by
him legitimately in India. (2 Marks)

© The Institute of Chartered Accountants of India


Page 49
20 FINAL (NEW) EXAMINATION: JANUARY 2021

3.3. Evaluate if the commission paid by Eastern Tea Limited is in accordance with FEMA
(A) Yes, it is a current account transaction and can be freely remitted;
(B) Yes, it is below the limit of 10% of invoice of exports of tea under Rupee State
Credit Route;
(C) No, it cannot be remitted until and unless the export proceeds are received;
(D) No, payment of commission for such exports is prohibited. (2 Marks)
3.4 Takeover (acquisition) of Eastern Tea limited by Eastern limited, will be considered as
combination if
(A) Assets of enterprise created after merger is equal to `2,000 crore;
(B) Turnover of enterprise created after merger is equal to `6,000 crore;
(C) Turnover of enterprise created after merger is more than `6,000 crore;
(D) Assets of enterprise created after merger is more than `6,000 crore. (2 Marks)
3.5 The decision of Eastern Tea Limited not to sell more than 5 kg of tea per customer
purchase can be categorized as
(A) Exclusive supply agreement;
(B) Exclusive distribution agreement;
(C) Refusal to deal;
(D) None of the options (2 Marks)
3.6 Analyze and answer the following questions in the context of the case study:
(i) In your view, explain with reasons if Eastern Produce Co-operative Society can be
considered as 'Cartel'? (3 Marks)
(ii) Does Eastern Limited hold dominance over the market, and if yes identify the
circumstances where it abuses its dominant position? (4 Marks)
(iii) Explain briefly the applicability of the Competition Act to the combinations
described in the case study and the regulatory aspects thereof. (5 Marks)
(iv) Evaluate if Eastern Limited is in compliance with the provisions of FEMA with
regard to the expenditure incurred for maintaining a branch abroad. (3 Marks)
ANSWERS TO CASE STUDY 3
3.1 (A)
3.2 (B)
3.3 (B)
3.4 (C)

© The Institute of Chartered Accountants of India


Page 50
PAPER – 6D: ECONOMIC LAWS 21

3.5 (D)
3.6(I)
Whether Eastern Produce Co-operative Society can be considered as ‘Cartel’ ?
As per Section 2 (c) of the Competition Act 2002, the term “cartel” includes an association of
producers, sellers, distributors, traders or service providers who, by agreement amongst
themselves, limit, control or attempt to control the production, distribution, sale or price of, or,
trade in goods or provision of services.
From the above, it may be noted that the term ‘cartel’ has been given inclusive meaning.
Although, Eastern Produce Cooperative Society was formed to ensure the timely collection of
sale proceeds from mills, it also developed a charter, in the form of a memorandum for its
members, to regulate and control the supply, price, term of sale, collection of sale proceeds
and also recovery, if required. This charter, in the form of a memorandum, was binding on all
the members of the Society.
Hence, Eastern Produce Cooperative Society is a ‘Cartel’ within the meaning of Section 2 (c)
of the Competition Act, 2002.
3.6(ii)
Whether Eastern Limited holds dominance over the market?
Yes, Eastern Limited holds dominance over the market because as per Explanation (a) to
Section 4 of the Competition Act, 2002, “dominant position” means a position of strength,
enjoyed by an enterprise, in the relevant market, in India, which enables it to (i) operate
independently of competitive forces prevailing in the relevant market; or (ii) affect its
competitors or consumers or the relevant market in its favour.
Circumstances where dominant position is abused
(a) Predatory Pricing after the acquisition of Eastern Tea Limited – Eastern Limited
acquired a substantial network of the retailers after the takeover of Eastern Tea Limited and
due to such takeover, it tried to penetrate the market using predatory pricing [refer Section
4(2)(a)(ii) of the Competition Act, 2002]. Eastern Tea Limited reduced the price of the leaf tea
from ` 150 to ` 130 per kilogram which was lower than the cost incurred, whereas other
players in the market like North Limited were selling leaf tea at ` 150 per kilogram.
As per Explanation (b) to Section 4 of the Competition Act, 2002, the term “predatory price”
means the sale of goods or provision of services, at a price which is below the cost, as may be
determined by regulations, of production of the goods or provision of services, with a view to
reduce competition or eliminate the competitors.
(b) Increasing the price after the acquisition of North Limited – After the hostile
acquisition of North Limited by Eastern Limited with the help of another group company South
Limited, Eastern Limited raised the price of its tea leaf ‘Taste leaf’ from ` 130 to ` 155 per
kilogram, even though North Limited was originally selling its tea leaf ‘Taste’ at ` 150 per

© The Institute of Chartered Accountants of India


Page 51
22 FINAL (NEW) EXAMINATION: JANUARY 2021

kilogram. According to Section 4 (2) (b) (i) of the Competition Act, 2002, there shall be an
abuse of dominant position under Section 4 (1), if an enterprise or a group limits or restricts
the production of goods or market therefor through unfair or discriminatory price.
(c) Cap on quantity
In order to restrict loss, on account of selling tea at price lower than cost, Eastern Tea Limited
asked all the shopkeepers and stores, not to sell more than 5 kg of leaf tea to a customer.
That would also be considered as abuse of dominance.
3.6(iii)
Provisions of the Competition Act, 2002 to “Combination”
In the context of Eastern Limited, the regulatory aspects of ‘combination’ as mentioned in
Section 5 of the Competition Act, 2002 are given as under:
Sr. Nature of Facts of the Criteria for Whether
No Combination case considering ‘Combination’ or not
‘Combination’
1 Acquisition by Eastern Limited Joint Asset over No. It is not a
single acquirer acquired ` 2,000 crores or combination.
but different Eastern Tea Turnover over Hint: Joint turnover is
goods [Section 5 Limited ` 6,000 crores ` 4,700crores
(a) (i) (A)] (3,500+1,200) which
is less than ` 6,000
crores. The joint
assets base of ` 980
crores (700+280)
which is less than
` 2,000 crores.
2 Acquisition by a Eastern Limited Group assets over No. It is not a
group with acquired North ` 8,000 crores or combination.
similar goods Limited with the turnover over Hint: Joint asset base
[Section 5 (b) (ii) help of another ` 24,000 crores of the ‘group’ is only
(A)] group company ` 1,550 crores
South Limited (980+570) and
aggregate turnover is
also ` 7,700 crores.
(4700+3000)
3 MOU for transfer Eastern Limited No criterion Not Applicable.
of technology enters into an prescribed for
MOU with West considering the
Offshore transfer of
Limited for technology as

© The Institute of Chartered Accountants of India


Page 52
PAPER – 6D: ECONOMIC LAWS 23

transfer of ‘combination’.
technology.
Note – Limits are quoted in section 5 of the Competition Act 2002 and further modified
through notification number S.O. 675(E) dated 4th March 2016
Regulation of Combinations
According to Section 6 (1) of the Act, no person or enterprise shall enter into a combination
which causes or is likely to cause an appreciable adverse effect on competition within the
relevant market in India and such a combination shall be void.
Further Section 6 (2) of the act says, any person or enterprise, who or which proposes to enter
into a combination, shall give notice to the Commission in the specified form along with a
requisite fee, disclosing the details of the proposed combination, within thirty days of:
(a) Approval of the proposal relating to merger or amalgamation by the Board of
Directors of the enterprises concerned with such merger or amalgamation;
(b) Execution of any agreement or other document for acquisition or acquiring of
control.
Further Section 6 (2A) of the Act provides, no combination shall come into effect until two
hundred and ten days have passed from the day on which the notice has been given to the
Commission or the Commission has passed orders under Section 31, whichever is earlier.
3.6(iv)
Whether eastern limited is in compliance of FEMA, 1999 for expenditure incurred on
maintenance of its branch office abroad ?
Eastern Tea Limited opened one branch office in Iran, and on such branch office, during last
financial year, an amount of ` 150 crore were incurred as expenditure for the branch through
the EEFC account maintained by Eastern Tea Limited.
As per Foreign Exchange Management Act, 1999, no branch can be opened in Pakistan,
Bangladesh, Sri Lanka, Afghanistan, China, Iran, Nepal, Bhutan, Macau or Hong Kong without
prior permission of the Reserve Bank.
As the case study does not reflect anywhere about the prior permission of the RBI, the
expenditure by Eastern Tea Limited is not in compliance under FEMA, 1999.
CASE STUDY - 4
Mr. Zebra is a real estate mogul who has developed and constructed several apartment
complexes in Mumbai through his real estate company, Delight Homes Private Limited (DHPL)
which was into the business of construction of residential premises. In May 2019, Zebra
proposed to start a new residential project named "Delight Morning Dews". The project plan

© The Institute of Chartered Accountants of India


Page 53
24 FINAL (NEW) EXAMINATION: JANUARY 2021

constituted 50 apartments with a mix of both 3 BHK and 2 BHK apartments. DHPL ensured
that the sanction plan etc. was approved appropriately under RERA.
DHPL devised the advertisement and marketing content for the project so it can be splashed
across the national and local newspapers and television channels. Mr. Zebra was of the view
that the project need not be highlighted in the website of the regulatory authority since the
same does not get a lot of views from the prospective customers and it is more efficient to
reach the customer directly through social media/television platforms. Further, to ensure more
customers are attracted, DHPL started the commercial marketing before applying for the
registration of the project under RERA.
DHPL also incorporated a subsidiary, Delight Interiors and Consultancy Private Limited
(DICPL) in India for engaging in the business of providing consultancy services on interior
designing etc. Mr. Zebra made his only daughter Ms. Rekha as the Managing Director of
DICPL. Rekha completed her masters in interior design in the London School of Design and
had her own design studio in London, which got her critical acclaim in the art and design
society. DICPL became a huge hit based on the proof of concepts it delivered in the London
School of Design and got many orders from customers located in the UK.
In the agreement to sale entered into with the aIIottees, DHPL did not specify the stage -wise
time schedule of the completion of the project, including the provisions for civic infrastructure
like water sanitation & electricity. Also, DHPL did not include any terms with regard to
cancellation of allotment etc. in the agreement. These clauses were not insisted by the
allottees since they were more than eager to buy their apartments and DHPL did not see any
reason to amend these agreements at a later date.
During the process of construction, DHPL intended to transfer the project to another real
estate construction company, Value Homes Private Limited (VHPL) through an assignment
agreement. No approval from the allottees or regulator was considered necessary since the
agreement made it clear that VHPL will take over all obligations of DHPL and there will not be
any difference for the regulator or the allottees in terms of the quality of constructions or timing
of delivery.
One of the customers of DICPL was interested in investing in the share capital of DHPL if the
same is allowed by the provisions of FEMA. However, Mr. Zebra indicated to him that FEMA
prohibits a person resident outside India to make investment in a company involved in real
estate business. However, Ms. Rekha believes that the customer can invest as long as the
money is paid directly to the bank account of DICPL through the normal banking channel and
the FIRC clearly denotes that this is for the purpose of equity investment into DICPL.
DICPL entered into various contracts to provide consultancy services to real estate
companies. Due to a downturn in the demand for real estate in the U.K. due to Brexit, some of
its customers faced a lot of difficulty in making payments to their suppliers and DICPL had
invoices outstanding amounting to GBP 2 million for more than 2 years (which include GSP
5,00,000 outstanding for more than 3 years).

© The Institute of Chartered Accountants of India


Page 54
PAPER – 6D: ECONOMIC LAWS 25

For the purpose of construction, VHPL decided to import certain raw materials such as PVC
boards and light fittings from China for a value of USD 10,00,000. VHPL paid an amount of
USD 80,000 on receipt of the products and decided to hold back the balance USD 20,000 for a
period of 1 year to ensure satisfactory performance of the products. The Authorised Dealer is
of the view that this is not in accordance with the requirements of FEMA.
Out of the 50 apartments, VHPL decided to retain the title to 10 of the apartments and entered
into a "lease" agreement with the allottees for a period of 99 years. The consideration for the
lease was 95% of the consideration for an outright sale (along with stamp duty to be paid by
the allottees) and the lease allottees were required to pay ` 1,000 per month as rent with
maintenance charges at actuals (in line with what was to be paid by the other apartment
owners). VHPL was of the view that these agreements are outside the purview of RERA since
there is no sale involved.
Answer the following questions:
4.1 A branch set and controlled by DICPL located in the U.K. will be considered as:
(A) Resident in India;
(B) Resident outside India;
(C) Resident in India, if funds are remitted back by the Branch to India on annual
basis;
(D) Resident outside India if significant portion of its funds are directly received from
its customers located outside India. (2 Marks)
4.2 One of DICPL's customers visits India and during the visit, pays an amount of USD 2,000
which was owed by the customer through cash. Is this a permissible transaction under
FEMA ?
(A) Yes, this is money rightfully owed to DICPL and money was received in foreign
exchange;
(B) No, unless DICPL had a money changer's license to accept foreign currency;
(C) No, unless DICPL obtain prior approval of the Reserve Bank of India for such
transaction;
(D) Yes, as long as DICPL deposits the money in its bank account. (2 Marks)
4.3 An advertisement in the Guardian Newspaper (London edition) by DICPL for an amount
of GBP 20,000 requires the permission of:
(A) the Central Government;
(B) the Board of Directors of DICPL;
(C) the Ministry of Finance, Department of Economic Affairs;
(D) the authorized dealer. (2 Marks)

© The Institute of Chartered Accountants of India


Page 55
26 FINAL (NEW) EXAMINATION: JANUARY 2021

4.4 Evaluate Mr. Zebra's contention that the customer of DICPL cannot invest in DHPL.
(A) The customer can invest in DHPL since DHPL, although a real estate company, is
involved in the construction of residential premises only;
(B) The customer cannot invest - Mr. Zebra is right in pointing out that FEMA prohibits
investment by a person resident outside India in real estate activities;
(C) The customer can invest with the prior approval of the Reserve Bank of India;
(D) The customer can invest since the FIRC indicates that the money is for investment
in equity capital and has come through the normal banking channel. (2 Marks)
4.5 The allottees who obtained the apartments under Lease seek your guidance on whether
those apartments would fall within the purview of RERA ?
(A) No, since there is no sale transaction and transfer of title to the allottee and
therefore it will not fall under RERA;
(B) Yes, this would be covered under RERA since substantial portion of the
consideration is paid and the lessee is also responsible for paying stamp duty,
maintenance etc.;
(C) No, since the entire sale consideration is not paid upfront and the lease is for 99
years only;
(D) Yes, since VHPL and DHPL are registered promoters under RERA. (2 Marks)
4.6 Analyze and answer the following questions in the context of the case study:
(i) Evaluate the marketing strategies adopted by DHPL with regard to the provisions
of RERA. (3 Marks)
(ii) Evaluate with reasons if DICPL is in compliance with the FEMA provisions with
regard to collection of export proceeds. Explain the steps to be taken by DICPL to
ensure compliance with FEMA. (5 Marks)
(iii) What is your advice to VHPL with regard to the position taken by the authorized
dealer? (3 Marks)
(iv) Advise the allottees of Delight Morning Dew regarding the assignment agreement
entered into between DHPL and VHPL in the context of RERA provisions. What
are the obligations of DHPL and VHPL under RERA ? (4 Marks)
ANSWERS TO CASE STUDY 4
4.1 (A)
4.2 (B)
4.3 (C)
4.4 (A)

© The Institute of Chartered Accountants of India


Page 56
PAPER – 6D: ECONOMIC LAWS 27

4.5 (C)
4.6(i)
According to Section 3 of the Real Estate (Regulation & Development) Act, 2016 (the Act), no
promoter shall advertise, market, book, sell or offer for sale, or invite persons to purchase in
any matter any plot, apartment or building, in any real estate project or part of it in any
planning area without registering the real estate project with the Real Estate Regulatory
Authority established under this Act.
Section 3 of the Act further provides that no registration of the real estate project shall be
required where the area of land proposed to be developed does not exceed 500 square
meters or the number of apartments proposed to be developed does not exceed eight
inclusive of all phases;
In the given case study, since the number of apartments are more than 8, hence registration
was required under RERA. As DHPL has not taken registration under RERA, hence it cannot
have opted for commercial marketing of the project.
4.6 (ii)
According to Foreign Exchange Management (Export of Goods and Services) Regulations,
2015:
1. The amount representing the full export value of goods / software/ services exported shall
be realized and repatriated to India within nine months or within such period as may be
specified by the Reserve Bank of India, in consultation with the Government, from time to
time, from the date of export, provided. Further the Reserve Bank of India, or subject to the
directions issued by that Bank in this behalf, the authorized dealer may, for a sufficient and
reasonable cause, extend the said period. In the present case study, an amount of GBP 2
million is outstanding for more than 2 years (which include GBP 5,00,000 outstanding for more
than 3 years) is not in compliance with the FEMA provisions.
Apart from the above compliance, the following steps must be taken by DICPL to ensure
compliance with FEMA:
1. DICPL is responsible for ensuring that the full export value of the goods exported are
realized through an authorized dealer in the manner specified in the Fore ign
Exchange Management (Manner of Receipt and Payment) Regulations, 2016.
2. In respect of export of services to which none of the Forms specified in these
Regulations apply, the exporter may export such services without furnishing any
declaration, but shall be liable to realize the amount of foreign exchange which
becomes due or accrues on account of such export, and to repatriate the same to
India in accordance with the provisions of the Act, and these Regulations, as also
other rules and regulations made under the Act.

© The Institute of Chartered Accountants of India


Page 57
28 FINAL (NEW) EXAMINATION: JANUARY 2021

3. A person resident in India to whom any amount of foreign exchange is due or has
accrued shall, save as otherwise provided under the provisions of the Act, or the
rules and regulations made thereunder, or with the general or special pe rmission of
the Reserve Bank of India, take all reasonable steps to realize and repatriate to India
such foreign exchange, and shall in no case do or refrain from doing anything, or
take or refrain from taking any action, which has the effect of securing—
(a) that the receipt by him of the whole or part of that foreign exchange is delayed;
or
(b) that the foreign exchange ceases in whole or in part to be receivable by him.
4. DICPL is also required to apply to RBI for getting the required directions for th e
purpose of securing the payment for the services performed, since there is a delay in
receipt of payment in accordance with the provisions.
4.6(iii)
advise to VHPL regarding the position taken by the authorized dealer
In terms of the FEM Regulations, remittances against imports should be completed not later
than six months from the date of shipment, except in cases where amounts are withheld
towards guarantee of performance, etc.
The Authorized Dealer (AD) can consider granting extension of time for settlement of import
dues up to a period of six months at a time (maximum up to the period of three years)
irrespective of the invoice value for delays on account of disputes about quantity or quality or
non-fulfilment of terms of contract; financial difficulties and cases where importer has filed suit
against the seller.
While granting extension of time, AD must ensure that:
a. The import transactions covered by the invoices are not under investigation by
Directorate of Enforcement / Central Bureau of Investigation or other investigating
agencies;
b. While considering extension beyond one year from the date of remittance, the t otal
outstanding of the importer does not exceed USD one million or 10 per cent of the
average import remittances during the preceding two financial years, whichever is
lower; and
c. Where extension of time has been granted by the AD, the date up to which extension
has been granted may be indicated in the ‘Remarks’ column.

© The Institute of Chartered Accountants of India


Page 58
PAPER – 6D: ECONOMIC LAWS 29

In the given case study, VHPL decides to pay USD 80,000 on receipt of products and hold
back USD 20,000 for a period of 1 year to ensure satisfactory performance (total bill amount of
raw materials was USD 1,00,000)*.
As per the above provisions, VHPL can do so. Hence, the contention of AD that VHPL cannot
withhold the amount for satisfactory performance is not correct.
*Note: In the given case study, the value of import material from China is given as USD
10,00,000. It is further given that VHPL paid an amount of USD 80,000 on receipt of the
products and decided to hold back the balance USD 20,000 for a period of 1 year. The total of
these two amounts (80,000 and 20,000) comes to 1,00,000. Hence, the total invoice amount of
USD 10,00,000 seems to be a typographical error.
4.6 (iv)
Advise to allottees:
As per the facts given in the case study, the position taken by DHPL and VHPL with regard to
transfer of project is incorrect. In terms of Section 15 of the RERA, DHPL shall not transfer or
assign his rights and liabilities in respect of the project to VHPL without obtaining the prior
written consent of two-thirds allottees of the (except the promoter). Further, prior written
approval of the RERA Authority is also mandatory. Hence, the transfer from DHPL to VHPL of
the project is not valid in accordance with the RERA.
Therefore, the allottees are advised that in order to enable the transfer effective, steps must
be taken by DHPL to get prior consent of two-thirds allottees and also the written approval of
the RERA Authority.
Obligations of the promoters: Section 15 of the RERA, 2016 provides for the obligations of
promoter in case of transfer of a real estate project to a third party under:
1. The promoter shall not transfer or assign his majority rights and liabilities in respect
of a real estate project to a third party without obtaining prior written consent from
two-third allottees, except the promoter and without the prior written ap proval of the
Authority,
However, such transfer or assignment shall not affect the allotment or sale of the
apartments, plots, or building as the case may be in the real estate project made by the
erstwhile promoter. `
2. On the transfer or assignment being permitted by the allottees and the Authority, the
intending promoter shall be required to independently comply with all the pending
obligations under the provisions of this Act or the rules and regulations made
thereunder, and the pending obligations as per the agreement for sale entered into
by the erstwhile promoter with the allottees.

© The Institute of Chartered Accountants of India


Page 59
30 FINAL (NEW) EXAMINATION: JANUARY 2021

Any transfer or assignment permitted under provisions of this section shall not result in
extension of time to the intending promoter to complete the real estate project and he shall be
required to comply with all the pending obligations of the erstwhile promoter, and in case of
default, such intending promoter shall be liable to the consequences of breach or delay, as the
case may be, as provided under this Act or the rules and regulations made thereunder.
CASE STUDY - 5
Mr. Rohit Writer is a well-known industrialist based in Pune, India and is the founder director
of Good Phones Private Limited (Good Phones), a fixed line and mobile phone manufacturer.
Good Phone is one of the largest telephone companies in India and its products are much
sought after in India and abroad. Mr. Rohit visits various countries as part of his business
travels and during these visits he spends significant time in Philanthropic activities. and social
gatherings and because of this, he is quite well known in business circles globally. Mr. Rohit
has a penchant for investing his money in buying various real estate property all over India
and passed this trait on to his son, Mr. Rahul Writer as well. Mr. Rahul completed his MBA
from Stanford University and is assisting Mr. Rohit in his business. Mr. Rohit also has a
daughter, Ms. Sonali Writer, who studies Art in Italy and has opened her own Art Studio in
Milan. Mr. Rohit is very proud of Sonali and supports her financially for her stay in Italy as wel l
as expenses towards maintaining the studio.
The marketing department of Good Phones introduced various new models in the last couple
of months with new technology such as 2 selfie cameras, faster processor and sleeker look.
Good Phones expect these phones to be major attraction in the global markets due to
attractive price range and therefore want to promote these phones extensively on a global
basis. For the purpose of advertisements, Good Phones engaged the services of Mr. David
Smith, a prominent baseball player and Ms. Emma Drew, a Miss Universe winner and agreed
to pay a 'guaranteed' fee of USD 5,00,000 each plus 10% bonus based on the sales of the
new models in year 1.
Mr. Rohit sent 5 sample mobile phones and 5 fixed line phones to his dealers abr oad
(numbering 1000 dealers), clearly marked as not for sale and other promotional material such
as brochures, 3D moulds for display in dealer shops etc. The value of the items was
approximately INR 4 Crore. He also sent 1 mobile phone to each of his dealers as token of gift
and appreciation (total value of INR 0.50 Crore). Mr. Srinivas Rajan, the CFO of Good Phones
indicated him that since these products have been sent free of cost and not for sale, these
need not be included in the export declaration to be filed by Good Phones.
On 15th February, 2018, Good Phones made a large sale to one of the dealers Delayed
Ringtone Enterprises, for USD 5 million and has received USD 2 million by 15th May, 2018
and did not receive the balance USD 3 million until 15th August, 2018, i.e. 6 months from the
date of sale. After several reminders and threatening calls to stop further shipment, another
USD 1 million was received on 10 th October, 2018 and the balance remained outstanding as at
31 st December, 2018.

© The Institute of Chartered Accountants of India


Page 60
PAPER – 6D: ECONOMIC LAWS 31

Based on the success of Good Phones, Mr. Rohit incorporated a new company, Stay
Connected Private Limited, (Stay Connected) as Internet service provider and purchased a
large consignment of networking equipment for providing internet operations through
dedicated broadband lines along with a landline facility. This would then provide Mr. Rohit
quite a few synergies with the existing Good Phone business and enable him to become an
end to end Telecom Czar. Mr. Rohit held 60% stake in Stay Connected and the balance 40%
was held by a foreign collaborator. Along with all the networking equipment, Stay Connected
hired transponders from a company in Australia and paid AUD 10 million through its
authorized dealer. Stay Connected also entered into an agreement with foreign collaborator
(holding 40% stake) to pay royalty and technical fees for the support provided by them.
During his visit to Milan to meet Ms. SonaIi, Mr. Rohit obtained EUR 10,000 from his Italian
dealer for his use during his stay in Italy and instructed the dealer to reduce the sum from the
payments to be made by the dealer for the supplies from Good Phones. Out of such funds, Mr.
Rohit used EUR 5,000 towards purchasing sweepstakes tickets in Milan, Italy, unfortunately,
he did not win any money in the sweepstakes event.
Mr. Rahul, after gaining experience in India, wanted the business in the U.A.E. (by
establishing a subsidiary of Good Phones in the U.A.E.) and therefore decided to move to the
U.A:E. along with his wife. For this purpose, he wanted to dispose off some of the properties
owned by him in India. Accordingly, Mr. Rahul sold an apartment in Mumbai owned by him to
Mr. Stuart Cooper, being an Overseas Citizen of India and a fellow student of his at Stanford
University. Mr. Stuart Cooper was planning to come to India in the next couple of months to
take up a job and therefore, wanted to secure a place for his stay. The remittance from
Mr. Stuart was received in India through banking channels.
Mr. Rahul also sold a villa and his agricultural land in Pondicherry to Mr. Rajesh
Subramanium, his professor at Stanford, who was a person of Indian origin. The payment for
the villa and agricultural land was paid by Mr. Rajesh (50%) from his FCNR account and the
balance in USD traveller cheques, which will be of use to Mr. Rahul when he visits U.A.E.
After obtaining his U.S. visa, Mr. Rahul purchased a ranch (farm house) in Texas for USD 2
million, using USD 1.50 million from RFC account and USD 500,000 sent from his INR
account through normal banking channels.
Mr. William Rutherford, one of Mr. Rohit's business acquaintances and a citizen of UAE, is
very much interested in Indian culture and practices and therefore stays in India for 8 months
(from April, 2018 to November, 2018) to attend an art of living course and to learn/practice
yoga. William believes that he has been resident in India for more than the prescribed 182
days and therefore, is a resident in India under FEMA.
Mr. Rohit, in his penchant for purchasing various properties, zeroed in on an exclusive
apartment complex in Bangalore having state-of-the art facilities. He purchased two 4-
bedroom apartments costing INR 2 crore each, one in the name of Ms. Sonali and one in the
name of Mr. Srinivas Rajan, since Mr. Rohit wanted Mr. Srinivas Rajan to feel happy and
trusted. Both the apartments were given on rent to a large multinational bank and he received

© The Institute of Chartered Accountants of India


Page 61
32 FINAL (NEW) EXAMINATION: JANUARY 2021

a rent of INR 0.20 Crore per year for each of the apartment in the bank accounts of Ms. Sonali
and Mr. Srinivas Rajan respectively, after 4 years, Mr. Srinivas Rajan transferred the property
back in the name of Mr. Rohit at zero consideration. Mr. Rohit also purchased a 3-bedroom
apartment in the same complex in his name, jointly with his brother, Mr. Sunil Writer. The
property (along with the stamp duty) was paid for by Mr. Rohit and was being used by Mr.
Sunil for his stay though the property was pending registration due to Mr. Rohit's travel
abroad.
Once the property was transferred back by Mr. Srinivas Rajan, Mr. Rohit wanted to sell the
same to Mr. Arjun De Silva, a citizen of Sri Lanka. However, he was advised by Mr. Srinivas
Rajan that Mr. Arjun De Silva cannot acquire property in India and therefore Mr. Rohit
proposed to lease it to Mr. Arjun De Silva for a period of 20 years for an upfront consideration
of INR 1 Crore and an annual rent of INR 8 Lac payable in advance.
During the review of the bank reconciliation statements of Good Phones, Mr. Srinivas Rajan
noted that an amount of INR 2 Crore had been received in one of the bank accounts without
any details relating to the same. Mr. Srinivas Rajan informed this to Mr. Rohit and Mr. Srinivas
Rajan suggested to Mr. Rohit to immediately transfer that money out of the bank of Good
Phones to Mr. Rohit's personal bank account, so that the Company's bank account are cleared
and there are no reconciling items, which Mr. Rohit agreed to. Out of INR 2 Crore, Mr. Rohit
used INR 1.75 Crore for acquiring further 20% stake in Stay Connected from the foreign
collaborator and balance INR 0.25 Crore for purchasing a stunning diamond set for his wife,
Ms. Anjali Writer, as a gift for her 50 th birthday.
The extract of the last audited financial statements of Stay Connected was provided by
Mr. Srinivas Rajan to Mr. Rohit to evaluate (FMV) his acquisition as per the provisions of the
Prohibition of Benami Property Transactions Act, 1988:
Particulars Amount in INR
(Crore)
Immovable property (market value INR 8.00 Crore) 5.00
Other fixed assets (net of depreciation of INR 1.00 Crore) 4.00
Inventory 2.00
Receivables and Loans and Advances 1.50
Deferred Advertisement Costs 0.50
Advance tax paid 1.00
Total Assets 14.00
Shareholders' Funds (including 1,000,000 equity shares of INR 10 each, 4.00
fully paid- up)
Provision for taxation 0.50
Loans from Banks 3.00

© The Institute of Chartered Accountants of India


Page 62
PAPER – 6D: ECONOMIC LAWS 33

Trade payables (including provision for unascertained liabilities-INR 1 6.50


Crore)
Total Liabilities 14.00
Other information:
(i) 'Contingent Liabilities - INR 2.00 crore (including INR 0.50 Crore relating to arrears' on
cumulative preference shares).
(ii) The Board of Directors has proposed a dividend payout of INR 1 crore to the equity
shareholders, which is pending approval of the shareholders.
The Bank, on noting the large transactions on Mr. Rohit's personal bank account, tipped the
Income Tax Authorities regarding the same and the Initiating Officer summoned information
from Mr. Rohit and Mr. Srinivas Rajan regarding the transactions to start proceedings under
the Prohibition of Benami Property Transactions Act, 1988 (PBPT Act, 1988) and inves tigate
the matter under the Foreign Exchange Management Act, 1999 (FEMA,1999).
Mr. Rohit and Mr. Srinivas Rajan reached out to you in order to understand the various
violations and implications during the course of various proceedings under the said Act's .
Answer the following questions:
5.1 Out of the below, what are the transactions that require prior approval of the Government
of India?
(A) Payment of "guaranteed" fee by Good Phones to Mr. David Smith and Ms. Emma
Drew;
(B) Payment of Royalty and Technical Fees by Stay Connected to the foreign
collaborator;
(C) Payment of hiring charges for the transponders by Stay Connected;
(D) Payment of INR 1.75 Crore by Mr. Rohit to acquire shares of Stay Connected from
the foreign collaborator. (2 Marks)
5.2 Is the use of EUR 5,000 towards purchasing sweepstakes by Mr. Rohit as per the
provisions of FEMA, 1999 ?
(A) No, drawl of foreign exchange for purchasing lottery tickets, sweepstakes etc. is
prohibited under FEMA, 1999;
(B) No, Mr. Rohit should have obtained the prior approval of the RBI before
purchasing the sweepstakes ticket;
(C) FEMA, 1999 will not be applicable, since the money was directly obtained by Mr.
Rohit from his Italian dealer outside the country;
(D) None of the options. (2 Marks)

© The Institute of Chartered Accountants of India


Page 63
34 FINAL (NEW) EXAMINATION: JANUARY 2021

5.3 Is the purchase of Ranch in Texas by Mr. Rahul in accordance with the FEMA, 1999 ?
(A) No, Rahul as a citizen of India cannot purchase a Ranch outside India;
(B) Yes, there is no specific limit under FEMA, 1999 with regard to purchase of
immoveable property outside India;
(C) No, Rahul can purchase assets outside India only if the purchase is jointly with a
relative, who is resident outside India, and there is no outflow of funds;
(D) No, since Rahul has used funds from his INR account for making the payment to
the extent of USD 500,000. (2 Marks)
5.4 In case Mr. Rohit is proven guilty of violating the provisions of PBPT Act, 1988, what is
the maximum punishment that he is liable for under the PBPT Act, 1988 ?
(A) Rigorous imprisonment for a term of one to seven years, with a fine which may
extend to 25% of the fair market value of the property;
(B) Rigorous imprisonment for a term of three to seven years, without fine;
(C) Rigorous imprisonment for a term upto seven years, with fine which may extend to
50% of the fair market value of the property;
(D) Fine which may extend to 25% of the fair market value of the property. (2 Marks)
5.5 Assuming that the transactions relating to the receipt of INR 2 Crore in the bank account
of Good Phones and the subsequent transactions are considered as benami
transactions, can the Initiating Officer take action against Mr. Srinivas Rajan ?
(A) Yes he is the CFO of Good Phones and therefore responsible for ensuring
compliance with the Law;
(B) No, he has not received, held, or acquired the proceeds in his account or
benefitted from the same;
(C) Yes, since he abets Mr. Rohit in transferring the money from the bank account of
Good Phones to Mr. Rohit's personal account;
(D) No, he is responsible only for Good Phones and he has ensured that the funds are not
retained in the books of Good Phones/used by Good Phones for its business.
(2 Marks)
5.6 Answer the following questions in the context of the provisions relating to Foreign
Exchange Management Act, 1999 :
(i) Mr. Srinivas Rajan reaches out to you to confirm his views regarding
inclusion/exclusion of the items sent free of cost to the dealers in the export
declaration. (1 Marks)
(ii) Examine the validity/appropriateness of the sale of immoveable property by Mr.
Rahul to Mr. Stuart Cooper and Mr. Rajesh Subramanium. (2 Marks)

© The Institute of Chartered Accountants of India


Page 64
PAPER – 6D: ECONOMIC LAWS 35

5.7 Examine/advise regarding the below questions relating to the Prohibition of Benami
Property Transactions Act, 1988 :
(i) Examine the appropriateness/impact of the PBPT Act, 1988 on 3 apartments
purchased by Mr. Rohit in Bangalore. How does the transfer back of the
apartment by Mr. Srinivas Rajan to Mr. Rohit affect your conclusion? (2 Marks)
(ii) The Initiating Officer, who is probing the transactions relating to the INR 2 Crore
received and spent by Mr. Rohit, seeks your advice to identify the benami
properties/transaction, the benamidars, the beneficial owner. (2 Marks)
(iii) Explain the provisions of Fair Market Value (FMV) in relation to a property as per
section 2(16) of PBPT Act, 1988. (2 Marks)
(iv) What is the process to be followed by the Initiating Officer for attachment of the
property under PBPT Act, 1988? (2 Marks)
(v) Discuss the provisions with regards to issue of notice, attachment of property
involved in benami transactions and manner of service of notice under PBPT Act,
1988. (4 Marks)
ANSWERS TO CASE STUDY 5
5.1 (C)
5.2 (A)
5.3 (D)
5.4 (A)
5.5 (C)
5.6 (i)
In the light of Regulation 4 of the Foreign Exchange Management (Export of Goods and
Services) Regulations, 2015, trade samples of goods, may be exported without furnishing the
declaration on the items, sent free of cost. In the given case, sending 5 sample mobile phones
and fixed line phones to 1000 dealers is exempted and does not require Good Phones to give
declaration for export.
With regard to sending mobile phones to the dealers as gift for a total value of INR 0.50 crore
(i.e., 50 lakh), as per the above Regulation, the exemption for sending gifts by an export is
available only if the value of the goods is not more than ` 5 lakh in value. In the case study,
since the value of the goods is more than the exempted limit, they need to be included in the
export declaration.
5.6 (ii)
Validity of the Sale of immovable property by Mr. Rahul can be given in the light of Regulati on
3 of the Foreign Exchange Management (Acquisition and Transfer of Immovable Property in
India) Regulations, 2018.

© The Institute of Chartered Accountants of India


Page 65
36 FINAL (NEW) EXAMINATION: JANUARY 2021

Sale of immovable property to Mr. Stuart Cooper: Mr. Staurt Cooper, being an Overseas
Citizen of India is entitled to acquire an apartment in Mumbai owned by Mr. Rahul and funds
were received in India through Banking Channels.
Sale of immovable property to Mr. Rajesh Subramanium: Whereas in case of Mr. Rajesh
Subramanium, being NRI, he may acquire immovable property in India other than agricultural
land/farm house/plantation property. Therefore, Mr. Rajesh Subramanium can acquire Villa
but not an agricultural land from Mr. Rahul. Further, payment was made partly from FCNR
account and in USD Travellers cheques, which was against the mode of payment prescribed
in the said regulation.
Therefore, sale of immovable property by Mr. Rahul to Mr. Stuart Cooper is valid, whereas to
Mr, Rajesh Subramanium, the said transaction is invalid.
5.7(i)
In the given case study, Mr. Rohit purchased 3 flats in Bangalore in the name of Ms. Sonali,
Mr. Srinivas Rajan, and jointly with his brother Sunil.
Apartment purchased in the name of Ms. Sonali- The property has been purchased by
Rohit in the name of his daughter Ms. Sonali, is covered under the exemption give n in Section
2(9) of the Prohibition on Benami Property Transaction Act, 1988 (i.e., property purchased in
the name of his child). Thus, it is not a benami transaction.
Apartment purchased in the name of Mr. Srinivas Rajan- This is a case of benami
transaction as the property is in the name of Srinivas Rajan but the consideration is paid by
Rohit.
Apartment purchased jointly in the name of Rohit and his brother Sunil- A property jointly
held in the name of brother and they appear as joint owners. Hence, this is not a benami
transaction.
Prohibition on retransfer of property by benamidar: As per Section 6 of the Prohibition on
Benami Property Transaction Act, 1988, (PBPT, Act) in cases where benamidar re-transfers
any benami property held by him to the beneficial owner or any other person acting on his
behalf, then such a transaction of a property shall be deemed to be null and void.
In the said above case transaction of transfer back of the apartment by Mr. Srinivas Rajan to
Mr. Rohit is void.
5.7(ii)
Advise to the Initiating Officer:
Following are the benami transactions and benamidars:
Transaction Benami Property/ Benamidar/ beneficial
owner
Receipt of INR 2 crore in the bank account Good Phones is a Benamidar w.r.t said
of Good Phones benami transaction of INR 2 crore.

© The Institute of Chartered Accountants of India


Page 66
PAPER – 6D: ECONOMIC LAWS 37

Transfer of INR 2 crore from the bank Mr. Rohit is the Beneficial owner
account of Good Phones to Mr. Rohit’s
personal bank account
Acquisition of shares of Stay Connected Shares of Stay Connected becomes
using the benami money benami property as per Section 2(8) of
PBPT Act.
Mr. Rohit is a beneficial owner.
Purchase of Jewellery as gift for Ms. Anjali The jewellery becomes benami property.
Writer Mr. Rohit is a Benficial owner as he
purchased jewellery by paying
consideration from unknown sources.
Ms. Anjali is a Benamidar, as jewellery has
been purchased in her name.
5.7 (iii)
According to Section 2(16) of the Prohibition of Benami Property Transaction Act, 1988, fair
market value, in relation to a property, means—
(1) the price that the property would ordinarily fetch on sale in the open market on the
date of the transaction; and
(2) where the price referred above is not ascertainable, such price as may be
determined in accordance with such manner as may be prescribed in Rule 3 of the
Prohibition of Benami Property Transaction Rules, 2016.
As per the said Rule, the price of unquoted equity shares shall be the higher of-
(a) its cost of acquisition;
(b) the fair market value of such equity shares determined, on the date of transaction, by
a merchant banker or an accountant as per the Discounted Free Cash Flow method;
and
(c) the value, on the date of transaction, of such equity shares as determined by the
formula given in the Rules.
5.7 (iv)
As per Section 24 of the Benami Transactions (Prohibition) Act, 1988, where the Initiating
Officer on the basis of material in his possession has reason to believe that any person is a
benamidar in respect of a property, he may, after recording reasons in writing, issue a notice
to the person to show cause within such time as may be specified in the notice why the
property should not be treated as benami property.

© The Institute of Chartered Accountants of India


Page 67
38 FINAL (NEW) EXAMINATION: JANUARY 2021

Where the Initiating Officer is of the opinion that the person in possession of the property held
benami, may alienate the property during the period specified in the notice, he may, with the
previous approval of the Approving Authority, by order in writing, attach provisionally the
property in the manner as prescribed in Rule 4 of the Benami Transactions Prohibition Rules,
2016, for a period not exceeding ninety days from the last day of the month in which the notice
is issued.
The Initiating Officer, after making inquiries, pass an order continuing the provisional
attachment of the property with the prior approval of the Approving Authority, till the passing of
the order by the Adjudicating Authority; or revoke the provisional attachment of the property
with the prior approval of the Approving Authority;
Where the Initiating Officer passes an order continuing the provisional attachment of the
property or passes an order provisionally attaching the property, he shall, within fifteen days
from the date of the attachment, draw up a statement of the case and refer it to the
Adjudicating Authority.
5.7 (v)
Notice and attachment of property involved in benami transaction [Section 24 of PBPT,
Act, 1988]
Issue of show cause notice: Section 24 (1) states that where the Initiating Officer, on the
basis of material in his possession, has reason to believe that any person is a benamid ar in
respect of a property, he may, after recording reasons in writing, issue a show cause notice to
the person.
A copy of the notice shall also be issued to the beneficial owner if his identity is known. s
possession of the property held benami, may alienate the property during the period specified
in the notice, may, with the previous approval of the Approving Authority, by order in writing,
attach provisionally the property for a period not exceeding ninety days from the last day of
the month in which the notice is issued.
After Inquiry: Initiating Officer, after making such inquires and calling for such reports or
evidence and taking into account all relevant materials, shall, within a period of ninety days
from the last day of the month in which the notice is issued —
(a) where the provisional attachment has been made
(i) pass an order continuing the provisional attachment of the property with the prior
approval of the Approving Authority, till the passing of the order by the Adjudicating
Authority or
(ii) revoke the provisional attachment of the property with the prior approval of the
Approving Authority;

© The Institute of Chartered Accountants of India


Page 68
PAPER – 6D: ECONOMIC LAWS 39

(b) where provisional attachment has not been made


(i) pass an order provisionally attaching the property with the prior approval of the
Approving Authority, till the passing of the order by the Adjudicating Authority under;
or
(ii) decide not to attach the property as specified in the notice, with the prior approval of
the Approving Authority.
Section 24 (5) states that where the Initiating Officer passes an order continuing the
provisional attachment of the property under sub-clause (i) of clause (a) of sub-section (4) or
passes an order provisionally attaching the property under sub-clause (i) of clause (b) of that
sub-section as stated above, he shall, within fifteen days from the date of the attachment,
draw up a statement of the case and refer it to the Adjudicating Authority.
Manner of Service of Notice [Section 25]
A notice under Section 24 may be served on the person named therein either by post or
as if it were a summons issued by a Court under the Code of Civil Procedure, 1908.

© The Institute of Chartered Accountants of India


Page 69
2 FINAL (NEW) EXAMINATION: NOVEMBER 2020

PAPER 6D: ECONOMIC LAWS


NOTE: The question paper comprises five case study questions. The candidates are required
to answer any four case study questions out of five.
Case Study 1
PART A
That one Mis Sun Energy (Pte.) Limited hereinafter addressed as the "petitioner" had invested
in an Indian Company 'Z', a company promoted by RR, by way of shares and debentures. The
petitioner held 51 per cent of the share capital of 'Z' respectively.
The petitioner filed writ petition with Hon'ble High Court seeking for issuance of writ of
prohibition, restraining the official respondents from in any manner proceeding with the show
cause notice dated 19-5-2017, issued by the Initiating Officer (Rank of Deputy Commissioner
Income Tax-Regular Company Circle) under section 24(1) under the Prohibition of Benami
Property Transactions Act,1988 (or in short PBPT Act), calling upon the petitioner to show
cause as to why 51 per cent shares and debentures were held by the petitioner in an Indian
Company 'Z' not be treated as a "benami property" and wanted to impose penalty under the
Prohibition of Benami Property Transactions Act, 1988. The petitioner were of the view that the
Adjudicating Authority is biased and may take adverse view on the case of the petitioner and
the petitioner even challenged the composition of the Adjudicating Authority on their
membership and qualification. The petitioner also sought for issuance of a writ of Certiorari, to
quash the impugned show cause notice dated 19-5-2017, issued under section 24(3) of the
Prohibition of Benami Property Transactions Act, 1988, intimating the petitioner that pursuant
to the provisional attachment of shares and debentures, enforced, the petitioner was
restricted/prohibited from dealing in any manner and from exercising any rights in relation to the
shares and debentures.
The petitioner stated that none of the transactions were benami transactions and the petitioner
was not a benamidar and the shares and debentures were not benami property. The
transactions done by the petitioner were completed well before the amendment to the Prohibition
of Benami Property Transactions Act, 1988. (The amendment received the assent of the
President of India on 11-8-2016 and the Act came into force with effect from 1-11-2016)
It was alleged by the petitioner that after receiving substantial investment from the petitioner,
RR was alleged to have siphoned money out of 'Z', refused to make necessary disclosures and
comply with the mandatory filings required under the Companies Act, 2013 and when the
petitioner sought for transparency of the transactions, RR and various companies controlled by
him initiated litigation against the petitioner with a view to prevent the petitioner from examining
the affairs of 'Z'. In the meanwhile, RR filed company petition before the National Company Law
Tribunal (NCLT) to restrain the petitioner from exercising its rights in relation to the shares and
debentures and also approached the High Court in this regard, where the Court initially granted
an ex parte interim injunction, which was vacated after the petitioner entered appearance and
contested the matter, by order dated 1-6-2017 and RR's plea was dismissed.

© The Institute of Chartered Accountants of India


Page 70
PAPER – 6D: ECONOMIC LAWS 3

The petitioner explained about the shareholding pattern in 'Z' and the pattern of investment
made in the company and how the debentures and shares were allotted to the petitioner. It was
submitted that on the date of issuance of the impugned show cause notice, the Initiating Officer
had no jurisdiction to issue the same, as he was not the gazette initiating officer under the Act
and thus lacked statutory jurisdiction even to issue the impugned orders. The transactions done
by the petitioner with the Indian company were completed in all aspects long before the
Amendment Act came into force i.e., on 01-11-2016 based upon the provisions of section 18
read with section 24 of the Act.
It was further submitted by the petitioner that the case of the Initiating Officer was solely based
upon the date on which, the Gazette Notification was uploaded by the Directorate of Printing at
the Government of India press to justify the jurisdiction of the Initiating Officer to initiate
proceedings. It was submitted that the notification would come into operation as soon as it is
published in the Gazette of India, i.e., the date of publication of Gazette and this being the
correct legal position, the contention of the Initiating Officer referring to the date on which the
notification was uploaded in the official website, was not sustainable based upon the provisions
of section 2(21) of the Act.
PART-B
Further to the above case scenario M/s Sun Energy (Pte.) Limited had in the month of January
2014 pre-booked a commercial office unit of approximately 1200 sq. ft. with M/s J V Realty
Limited, a leading developer in that area in their "S COURT" Greater Noida project developed
in phases launched then by paying an amount of ` 25,00,000/-as booking amount out of
`1,00,00,000 the total cost of flat but no Builder-Buyer agreement was entered into between
the parties except that an allotment letter was issued by the developer mentioning the unit
details. This project being developed over an area of approximately 15000 sq mts and having
over I 00 office units in its plan outlay and the company had paid till April 2017 almost 90% of
the entire cost of the property based upon percentage of completion (progress) of the stage of
construction but the developer had failed to provide neither possession nor had completed the
project and was also not responding to their complaints on one pretext or the other. The legal
counsel of M/s Sun Energy (Pte.) Limited in the month of May, 2017 informed the Board of
Directors of the company about Real Estate (Regulation and Development) Act, 2016 (for short
"the RERA"). They further informed that RERA was enacted by the Parliament as Act 16 of 2016
in the year 2016. Some of the provisions of the RERA came into force on a date prescribed by
the Central Government under the notification published in the official gazette. Different dates
were appointed for different provisions of the RERA. By Notification No. S.0.1544 (E), dated 26 -
4-2016, the Central Government appointed 1 st day of May 2016 as a date on which some of
provisions of the RERA came into force, namely, Sections 2, 20 to 39, 41 to 58, 17 to 78 and
81 to 92. By Notification No. S. 0.1216, dated 19-4-2017 some more provisions of the RERA
came into force, namely, Sections 3 to 19, 40, 59 to 70 and 79, 80 w.e.f 1 st May, 2017. Meaning
thereby that on May 1, 2017, all 92 provisions of the Real Estate (Regulation and Development)
Act, 2016 (RERA or the Act) were brought into force. The Act has introduced new obligations
on real estate developers and in cases of default, prescribes penal liabilities and the company

© The Institute of Chartered Accountants of India


Page 71
4 FINAL (NEW) EXAMINATION: NOVEMBER 2020

can contemplate bringing a legal suit against the developers under RERA. The developer on
the other hand is of the view that RERA is not applicable to this project as the same was
launched and construction commenced much before the RERA came into force.
Answer the following questions:
1.1 Which of the following is correct statement as per Prohibition of Benami Property
Transactions Act, 1988?
(A) Prohibition to hold benami property.
(B) Prohibition of benami transactions.
(C) Prohibition of right to recover property held benami.
(D) Prohibition on re-transfer of property by benamidar.
1.2. As per the provision of Prohibition of Benami Property Transactions Act, 1988 the appellate
tribunal or the adjudicating authority may in order to rectify any mistake apparent on face
of the record, amend any order made under section 26 and section 46 respectively within
a period
(A) of two years from the end of the quarter in which the order was passed.
(B) of three years from the end of the quarter in which the order was passed.
(C) of one year from the end of the month in which the order was passed.
(D) of one year from the date of passing of order.
1.3 Where a builder is planning to develop a particular project in different phases spread over
couple of years, then he is required to obtain registration under Real Estate (Regulation
and Development) Act, 2016.
(A) Only once for the entire project indicating all the phases.
(B) For each phase separately.
(C) As and when project commences registration will be required.
(D) As and when a particular phase is being developed registration of that phase will be
required.
1.4 A promoter shall not accept a sum of more than_________ percentage of the cost of the
apartment, plot or building, as an advance payment or an application fee from a person
without first entering________ a under the provisions of Real Estate (Regulation and
Development) Act, 2016.
(A) 15%, Sale Deed.

© The Institute of Chartered Accountants of India


Page 72
PAPER – 6D: ECONOMIC LAWS 5

(B) 10%, written agreement for sale.


(C) 15%, Sale Deed which is duly registered.
(D) 10%, written agreement to sale which is duly registered.
1.5 Where a Real Estate Agent contravenes the provisions of section 9 or section 10 of the
Real Estate (Regulation and Development) Act, 2016 he shall be liable to penalty as
determined by the Authority of____________________
(A) `10,000.
(B) `10,000 for every day during which the default continues.
(C) 1̀0,000 for every day during which the default continues upto 5% of the cost of the
plot, apartment or building of the project for which sale has been facilitated.
(D) `10,000 for every day during which the default continues upto 2% of the cost of the
plot, apartment or building of the project for which sale has been facilitated.
(5 x 2 = 10 Marks)
1.6 In the light of given case study state the quantum of penalty imposed whosoever enters
into any Benami Transaction on and after the date of commencement of the Benami
Transactions (Prohibition) Amendment Act, 2016. (3 Marks)
1.7 State the qualifications for appointment of Chairperson and Members of the Adjudicating
Authority under the Prohibition of Benami Property Transactions Act, 1988.
(2 Marks)
1.8 In the light of the given case study decide stating the provisions of the Real Estate
(Regulation and Development) Act, 2016, whether M/s Sun Energy (Pte.) Limited can
initiate legal proceedings against the developer M/s J V Realty Limited under the said Act
or the contention of the developer that the said Act is not applicable to the project is correct.
(5 Marks)
1.9 From the provisions of the Real Estate (Regulation and Development) Act, 2016, you are
of the view that the Act is applicable to the developer then decide as per the provisions of
the said Act, can the company seek refund of the entire amount paid to the developer till
date along with interest? Whether apart from principal and interest, can the company also
seek certain compensation from the developer? (5 Marks)
ANSWER TO CASE STUDY 1
1.1 Options B, C, & D
Note: In the light of the Preamble of the Prohibition of Benami Property Transaction
Act, 1988, Options B, C, & D, are correct Options

© The Institute of Chartered Accountants of India


Page 73
6 FINAL (NEW) EXAMINATION: NOVEMBER 2020

1.2 Option C
1.3 Option B
1.4 Option D
1.5 Option C
Answer 1.6
Quantum of Penalty for Benami Transactions [Section 53]
As per Section 53 of the Prohibition of Benami Property Transactions Act, 1988: [substituted for
Benami Transactions (Prohibition) Act, 1988 by the Benami Transactions (Prohibition)
Amendment Act, 2016 w.e.f. 01.11.2016] :
Where any person enters into a benami transaction in order to defeat the provisions of any law
or to avoid payment of statutory dues or to avoid payment to creditors, the beneficial owner,
benamidar and any other person who abets or induces any person to enter into the benami
transaction, shall be guilty of the offence of benami transaction.
Whoever is found guilty of the offence of benami transaction referred to above shall be
punishable with rigorous imprisonment for a term which shall not be less than one year, but
which may extend to seven years and shall also be liable to fine which may extend to twenty-
five per cent of the fair market value of the property.
Answer 1.7
As per Section 9 of the Prohibition of Benami Property Transactions Act, 1988 as amended by
the Benami Transactions (Prohibition) Amendment Act, 2016 w.e.f. 01.11.2016:
(1) A person shall not be qualified for appointment as the chairperson or a Member of the
Adjudicating Authority unless he:
(a) Has been member of the Indian Revenue Service and has held the post of
Commissioner of Income tax or equivalent post in that service
(b) Has been a member of the Indian legal service and has held the post of joint Secretary
or equivalent post in that service.
(2) The Chairperson and other members of the Adjudicating Authority shall be appointed by
the Central Government in such manner as may be prescribed.
(3) The Central Government shall appoint the senior most member to be the chairperson of
the Adjudicating Authority.
Answer 1.8
As per Section 3(1) of The Real Estate (Regulation & Development) Act, 2016, (the Act) the
promoter shall make an application to the Authority for registration of the project that is ongoing

© The Institute of Chartered Accountants of India


Page 74
PAPER – 6D: ECONOMIC LAWS 7

on the date of commencement of this Act and for which completion certificate has not been
issued within a period of three months from the date of commencement of this Act.
Further Section 3(2) of the Act says that no registration of the real estate project shall be
required :
(a) where the area of land proposed to be developed does not exceed 500 square meters or
the number of apartments proposed to be developed does not exceed eight inclusive of all
phases;
(b) where the promoter has received completion certificate for a real estate project prior to
commencement of this act;
As per the facts, “S COURT” greater Noida project was launched before the enforcement of the
Act. As M/s Sun Energy (Pte.) Limited had pre-booked in January 2014 with M/s J V Realty
Limited. So it was an ongoing project on the date of commencement of this Act and for which
completion certificate has also not been issued within a period of three months from the date of
commencement of this Act.
Further project was developed over an area of approximately 15000 sq. mts. and having over
100 office units in its plan outlay, which exceed 500 square meters and the number of
apartments exceeding eight inclusive of all phases.
Hence in the given case, M/s Sun Energy (Pte.) Ltd can initiate legal proceedings against
developer M/s J V Realty Limited under the Act stating the violation of the above mentioned
provisions under the Act and the contention of the developer that the said Act is not applicable
to the project, is incorrect.
Answer 1.9
Return of Amount and Compensation (Section 18)
Section 18 of the of The Real Estate (Regulation & Development) Act, 2016, (RERA) provides
for the return of amount and compensation.
(1) If the promoter fails to complete or is unable to give possession of an apartment, plot or
building,—
(a) in accordance with the terms of the agreement for sale or, as the case may be, duly
completed by the date specified therein; or
(b) due to discontinuance of his business as a developer on account of suspension or
revocation of the registration under this Act or for any other reason:
he shall be liable on demand to the allottees, in case the allottee wishes to withdraw from
the project, without prejudice to any other remedy available, to return the amount received
by him in respect of that apartment, plot, building, as the case may be, with interest at such

© The Institute of Chartered Accountants of India


Page 75
8 FINAL (NEW) EXAMINATION: NOVEMBER 2020

rate as may be prescribed in this behalf including compensation in the manner as provided
under this Act.
However, where an allottee does not intend to withdraw from the project, he shall be paid,
by the promoter, interest for every month of delay, till the handing over of the possession,
at such rate as may be prescribed.
(2) The promoter shall compensate the allottees in case of any loss caused to him due to
defective title of the land, on which the project is being developed or has been developed,
in the manner as provided under this Act, and the claim for compensation under this sub -
section shall not be barred by limitation provided under any law for the time being in force.
(3) If the promoter fails to discharge any other obligations imposed on him under this Act or
the Rules or Regulations made thereunder or in accordance with the terms and conditions
of the agreement for sale, he shall be liable to pay such compensation to the allottees, in
the manner as provided under this Act.
Therefore in the given case study as per the provision of Section 18 reproduced herein above,
the Company can seek refund if they wish to withdraw and also claim interest apart from
compensation.
CASE STUDY 2
Mr. Kamal is engaged in the real estate business of development of townships through his
company- M/s P Homes Ltd. During the course of business, he has accumulated enormous
amount of wealth in the form of cash which was generated through illegal businesses. Police
cases under several sections of various Indian laws have also been registered against
Mr. Kamal.
Mr. Kamal has a son Mr. Vimal who was residing in India during F.Y. 2016-17. He left for UAE
on 25th August 2017 to undergo training for a period of 4 years. Mr. Shyam, brother of Mr.
Kamal, has a daughter, Ms. Priyadarshini pursuing higher studies in UAE. Mr. Shyam intends
to:
(a) open a bank account in foreign currency in UAE.
(b) remit money from India to his daughter in her account for studies.
Separately, Ms. Priyadarshini has requested Mr. Shyam to sponsor a chess tournament in UAE
which will involve remittance amounting to USD 85,000 (after conversion). Mr. Shyam generally
remits money through TZB Bank Ltd. after complying necessary formalities.
On the other hand, since Mr. Vimal's interest lies in India, he intends to invest money in India in
the following manner:
(a) Incorporating a Company in India followed by infusion of capital in the said company.
(b) Buying an agricultural farm in his individual capacity.

© The Institute of Chartered Accountants of India


Page 76
PAPER – 6D: ECONOMIC LAWS 9

Above investments require funding which will be sought from Mr. Kamal.
From the business of real estate, total wealth generated by Mr. Kamal amounts to approx.
`775 Crore. The said amount was utilized by him in the following manner:
(a) Around `100 crore were used for meeting certain cash expenses and paying certain bribes.
(b) `325 crore were transferred through hawala transaction to Mr. Vimal.
Transferring money through hawala route was chosen by Mr. Kamal since the money available
with him in his bank account was not sufficient to remit legally under various provisions of
Foreign Exchange Management Act, 1999. Therefore, he decided to strike a deal with Mr. Bhola,
a hawala agent operating in India. Terms of the deal are as under:
• Mr. Kamal will pay `325 crore + commission in cash to Mr. Bhola.
• Mr. Bhola, through his counterparts in UAE, will pay equivalent USD (after conversion) to
Mr. Vimal against invoice for professional services dated 1 st October 2018.
Further Mr. Kamal and Mr. Shyam are promoters and directors of M/s KS Cinemas Ltd., a
company engaged in the business of producing motion films in India.
For a very large upcoming film project, M/s KS Cinemas Ltd. has taken loan from TZB Bank Ltd.
amounting to `350 crore after mortgaging all the assets of the company including rights related
to the film. However, due to controversies surrounding the film, the Censor Board withheld the
certification of the film. Even the Honorable High Court turned down plea of the producers that
the film is not against the interest of the country or public at large. The Reserve Bank of India
during the course of annual audit sent a notice to TZB Bank Ltd on suspicion of non-compliance
of the provisions of the Foreign Exchange Management Act, 1999 by TZB Bank Ltd. In the said
notice, the Reserve Bank of India sought certain information on the transactions carried out by
Mr. Shyam. However, lawyer of TZB Bank Ltd. suggested not to provide any response to such
notice, since such notice is generally issued to every bank as a part of audit procedure and is
of routine in nature.
One of the disgruntled crew members filed a complaint against Mr. Kamal in police station under
Indian Penal Code (IPC) for investigation. The complaint was accompanied with the details of
how Mr. Kamal acquired massive amount of wealth and huge properties in his name and also
in joint names. The accused person accumulated movable and immovable properties and assets
not only in India but in abroad also. Those properties were acquired otherwise and were not
included in their disclosed assets. Their criminal acts indicated misappropriation of public
money. Accordingly, the complaint was registered under the Indian Penal Code, 1860 and the
Prevention of Corruption Act, 1988.
Later on, the investigation was taken over by the CBI., while the CBI. was proceeding with the
investigation, the Enforcement Directorate on the basis of allegation made, lodged Enforcement
Case Information Report (ECIR) against Mr. Kamal. Similarly, as per the said ECIR when
complaint was filed under Section 45 of the Prevention of Money Laundering Act, 2002,

© The Institute of Chartered Accountants of India


Page 77
10 FINAL (NEW) EXAMINATION: NOVEMBER 2020

cognizance of the offence was taken against Mr. Kamal under section 3 of the Prevention of
Money Laundering Act, 2002, punishable under section 4 of the said Act. The Enforcement
Directorate issued a notice dated 27 th January 2018 to Mr. Kamal, which was received by him
on 31 st January,2018 directing him to pay penalty.
Subsequently, an order was issued by the authorities to provisionally attach properties
belonging to Mr. Kamal. Mr. Kamal now intends not to challenge the action taken against him
under the Prevention of Money Laundering Act, 2002 before the Adjudicating Authorities. On
01 st May,2018 a meeting was held with you in the said meeting Mr. Kamal informed that he
wanted to engage you to advise for understanding, powers and remedy for his matters under
the various provisions of theForeign Exchange Management Act, 1999 and the Prevention of
Money Laundering Act, 2002.
Answer the following questions:
2.1 Which of the following remittance will require prior approval of Government of India for
drawl of foreign exchange under the Foreign Exchange Management Act, 1999?
(A) Payment related to 'call back services' of telephones.
(B) Opening of foreign currency account abroad with a bank.
(C) Remittance of prize money / sponsorship of sports activity abroad by a person other
than International / National / State Level bodies, if the amount involved is USD
90,000.
(D) Remittance of freight of vessel charted by a Public Sector Undertaking.
2.2 As per the provisions of the Prevention of Money Laundering Act, 2002, person on whose
behalf a transaction is being conducted is known as:
(A) Client.
(B) Financial Institution
(C) Beneficial Owner.
(D) Authorized Dealer.
2.3 Under the Prevention of Money Laundering Act, 2002, Adjudicating Authority consists of
following:
(A) 3 persons including chairman.
(B) 4 persons including chairman.
(C) 2 persons one of whom can be appointed as a chairman
(D) 5 persons including a member from Ministry of Law and Justice.

© The Institute of Chartered Accountants of India


Page 78
PAPER – 6D: ECONOMIC LAWS 11

2.4 Among other things, what is the qualification of a person to be appointed as a Public
Prosecutor before the Special Court under the provisions of the Prevention of Money
Laundering Act, 2002?
(A) Minimum 10 years of experience as an advocate.
(B) Minimum 5 years of experience as an advocate.
(C) Minimum 7 years of experience as an advocate.
(D) Minimum 15 years of experience as an advocate.
2.5. Under the Prevention of Money Laundering Act, 2002, property can be provisionally
attached for___________.
(A) Not exceeding 60 days.
(B) Not exceeding 90 days.
(C) Not exceeding 180 days.
(D) Not exceeding 300 days. (5 x 2 = 10 Marks)
2.6 Answer the following in light of the provisions of the Foreign Exchange Management
Act,1999:
Advise Mr. Kamal whether:
I. he can invest in M/s P Homes Ltd. engaged in the business of building low budget
homes.
II. he can buy agricultural farm in his individual capacity.
III. he can make payment through foreign currency notes. (3 Marks)
2.7 For investing activities in India by Mr. Kamal, he approached you on 1 st May 2018 with a
notice dated 27 th January, 2018 received by him from the office of Enforcement Directorate
on 31 st January 2018 directing him to pay penalty. Kindly advise Mr. Kamal on timelines to
pay the penalty and powers of the officers to recover the same. Mr. Kamal has informed
that he doesn't intend to file an appeal. (3 Marks)
2.8 On suspicion of non-compliance of the provisions of the Foreign Exchange Management
Act, 1999 by TZB Bank Ltd., the Reserve Bank of India had sent a notice to the bank
seeking certain information on the transactions carried out by Mr. Shyam. However, lawyer
of TZB Bank Ltd. had suggested not to provide any response to such notice since such
notice is generally issued to every bank as a part of audit procedure and is of routine in
nature. Explain the powers of the Reserve Bank of India in case of non-compliance to
notice. (3 Marks)

© The Institute of Chartered Accountants of India


Page 79
12 FINAL (NEW) EXAMINATION: NOVEMBER 2020

2.9 Explain the following in light of the provisions of the Prevention of Money Laundering Act,
2002:
I. Money Laundering does not mean just siphoning of funds. In light of this statement,
explain the significance and aim of the Prevention of Money Laundering Act, 2002
and its three distinct stages. (2 Marks)
II. Mr. Kamal seeks your advice on the remedy available with him under the Act against
the said attachment order. (2 Marks)
III. Properties confiscated under the provisions of the Prevention of Money Laundering
Act, 2002 shall be available for disposal by Ministry of Finance as and when
necessary. Examine correctness of the statement. (2 Marks)
ANSWER TO CASE STUDY 2
2.1 Option D
2.2. Option C
2.3 Option A
2.4 Option C
2.5 Option C
Answer 2.6
(I) As per Foreign Exchange Management (Permissible Capital Account Transactions)
Regulations, 2000, no person resident outside India shall make an investment in India, in
any form, in any company or partnership firm or proprietary concern or any entity, whether
incorporated or not, which is engaged or proposes to engage in real estate business. Since
Mr. Kamal is a person resident in India, he can invest in M/s P Homes Ltd. engaged in the
business of building low budget homes.
(II) Yes, Mr. Kamal can buy agricultural farm in his individual capacity, since prohibitions as
regard the purchase of agricultural farm is exercised in favour of person resident outside
India. In other words there is no specific prohibition on person resident in India on buying
of agricultural farm in his individual capacity.
(III) A person resident in India can open, hold and maintain with an authorized dealer in India,
a Resident Foreign Currency (Domestic) Account, out of foreign exchange acquired in the
form of currency notes, Bank notes and travellers cheques from any of the sources like,
payment for services rendered abroad. Yes Mr. Kamal can make payment through foreign
currency notes through an authorized dealer.

© The Institute of Chartered Accountants of India


Page 80
PAPER – 6D: ECONOMIC LAWS 13

Answer 2.7
Recovery of Fine or Penalty [Section 69 of the Prevention of Money Laundering Act, 2002]
Where any fine or penalty imposed on any person under Section 13 or Section 63 of Prevention
of Money Laundering Act, 2002 is not paid within 6 months from the day of imposition of fine or
penalty, the Director or any other officer authorized by him in this behalf may proceed to recover
the amount from the said person in the same manner as prescribed in Schedule II of the Income-
tax Act, 1961 for the recovery of arrears and he or any officer authorized by him in this behalf
shall have all the powers of the Tax Recovery Officer mentioned in the said Schedule for the
said purpose. Accordingly, Mr. Kamal must pay penalty latest by 31 st July, 2018.
Answer 2.8
Reserve Bank’s powers to issue directions to authorized person [Section 11]
(1) The Reserve Bank may, for the purpose of securing compliance with the provisions of this
Act and of any rules, regulations, notifications or directions made thereunder, give to the
authorized persons any direction in regard to making of payment or the doing or desist
from doing any act relating to foreign exchange or foreign security.
(2) The Reserve Bank may, for the purpose of ensuring the compliance with the provisions of
this Act or of any rule, regulation, notification direction or order made thereunder, direct
any authorized person to furnish such information, in such manner, as it deems fit.
(3) Where any authorized person contravenes any direction given by the Reserve Bank under
this Act or fails to file any return as directed by the Reserve Bank, the Reserve Bank may,
after giving reasonable opportunity of being heard, impose on the authorized person a
penalty which may extend to ten thousand rupees and in the case of continuing
contravention with an additional penalty which may extend to two thousand rupees for
every day during which such contravention continues.
As per above provisions, Reserve Bank of India may impose penalty on TZB Bank Ltd. for non -
compliance to notice.
Answer 2.9
(I) Money laundering does not mean siphoning of fund. It actually refers to a whole process
or an entire system by which money is actually generated from serious crimes but they are
given such shape (by disguising its origin into a series of transactions) that it looks like it
has originated from legitimate sources.
The Prevention of Money Laundering Act, 2002 As stated in the Preamble to the Act, it
is an Act to prevent money-laundering and to provide for confiscation of property derived
from, or involved in, money-laundering and to punish those who commit the offence of
money laundering.

© The Institute of Chartered Accountants of India


Page 81
14 FINAL (NEW) EXAMINATION: NOVEMBER 2020

Money laundering is a single process however; its cycle can be broken down into three
distinct stages:
1. Placement: It is the first and the initial stage when the crime money is injected into
the formal financial System.
2. Layering: Then under the second stage, money injected into the system is layered
and moved or spread over various transactions in different accounts and different
countries. Thus, it will become difficult to detect the origin of the money.
3. Integration: Under the third and final stage, money enters the financial system in
such a way that original association with the crime is sought to be obliterated so that
the money can then be used by the offender or person receiving as clean money.
II. Section 25 of the Prevention of Money Laundering Act, 2002 (the Act) empowers the
Central Government to establish an Appellate Tribunal to hear appeals against the orders
of the Adjudicating Authority and the Authorities under the Act.
Section 26 of the Act deals with the rights and time frame to make an appeal to the
appellate Tribunal. Any person aggrieved by an order made by the Adjudicating Authority
may prefer an appeal to the Appellate Tribunal within a period of 45 days from the date on
which copy of the order is received by him. The appeal shall be in such form and be
accompanied by such fee as may be prescribed. The Appellate tribunal may extend the
period if it is satisfied that there was sufficient cause for not filing it within the period of 45
days.
The Appellate Tribunal may, after giving the parties to the appeal an opportunity of being
heard pass such orders thereon as it thinks fit, confirming, modifying or setting aside the
order appealed against.
According to Section 42 of the Act, any person aggrieved by any decision or order of the
Appellate Tribunal may file an appeal to the High Court within 60 days from the date of
communication of the order of the Appellate Tribunal.
In the light of the above provisions of the Act, Mr. Kamal is advised to prefer an appeal to
the Appellate Tribunal in the first instance.
(III) Management of Properties confiscated (Section 10)
Under Section 10 of the Prevention of Money laundering Act, 2002:
The Central Government may, by order published in the Official Gazette, appoint as many
of its officers (not below the rank of a Joint Secretary to the Government of India) as it
thinks fit, to perform the functions of an Administrator.

© The Institute of Chartered Accountants of India


Page 82
PAPER – 6D: ECONOMIC LAWS 15

(1) The Administrator appointed under sub-section (1) shall receive and manage the
property in relation to which an order has been made under sub-section (5) or sub-
section (6) or sub-section (7) of Section 8 or Section 58B or sub-section (2A) of
Section 60 in such manner and subject to such conditions as may be prescribed.
(2) The Administrator shall also take such measures as the Central Government may
direct to dispose of the property which is vested in the Central Government under
Section 9.
In view of the above, the state that the properties under the Act shall be available for
disposal by the Ministry of Finance as and when necessary is correct.
CASE STUDY 3
SSTPL is one of India's leading television manufacturers and has its manufacturing plant in
Chennai, with more than 200 dealers across the country. SSTPL specializes in manufacturing
LED Smart televisions both for direct retail sales as well as contract manufacture for other
television manufacturers. SSTPL has a very robust Board of Directors who are highly involved
in the operations of the entity.
During one of the Board Meetings held in the month of July 2019, the Board of Directors
reviewed the amounts receivable from the dealers of SSTPL and noted the following:
Age Amount in Lakh ` Number of Dealers
0 to 180 days 1505 135
180 to 720 days 280 34
> 720 days 905 1
Total 2,690 170
The CFO went on to explain that the amount which is outstanding for more than 2 years is
receivable from DMPL and the Company has been following up with the dealer on a regular
basis. The independent director on the Board asked the CFO to explore the possibility of taking
action against DMPL under the Insolvency and Bankruptcy Code, 2016 (in short 'IBC 2016').
The CFO informed that the financial creditors of DMPL has already commenced the process
and the Interim Resolution Professional (IRP) reached out to the CFO last week to understand
the claims of SSTPL against DMPL.
The IRP identified the following assets and liabilities of DMPL:
• Bank loans taken by DMPL from Bank A amounting to `1500 lakh and Bank B amounting
to `1050 lakh.
• Loan taken from the son Mr. 'X' of the promoter of DMPL amounting to `75 lakh attended
Board Meetings to provide guidance/directions on policy making process.
• Payable to SSTPL `905 lakh.

© The Institute of Chartered Accountants of India


Page 83
16 FINAL (NEW) EXAMINATION: NOVEMBER 2020

• Outstanding wages to workmen amounting to `75 lakh.


• Statutory employer contributions to the tune of `30 lakh.
• Realisable value of the fixed assets of DMPL, `2800 lakh.
• Receivables from various customers, `225 lakh, out of which 50% is not realisable.
• Bank balance off `22.5 lakh.
The IRP also received information that MCL, a Company registered in Germany, pursuant to an
agreement entered with DMPL and supplied spares to DMPL for an amount of EVR 500,000
(INR 400 lakh) (though this claim is not disputed by DMPL, the same was not recorded in the
books of accounts of DMPL inadvertently). Since this amount was not paid by DMPL even after
several reminders, MCL filed an application under the IBC 2016. However, this application was
rejected by the Adjudicating Authority since as per the agreement between MCL and DMPL, any
disputes between the parties are to be decided by the courts in Germany. DMPL, in its
agreement, with its distributors, specified that the distributors be necessarily required to
purchase spares for 2 models of cars on a bundled basis (the sale price fixed based on fair
market value/mutual discussion). On 14 th April 2020, ACL, another supplier of DMPL, to whom
DMPL owed INR 75 lakh, also wanted to initiate Corporate Insolvency Resolution Process
against DMPL for non-payment of undisputed dues.
During the aforesaid Board Meeting of SSTPL, the CFO also placed a revised draft agreement
to be entered into with all the dealers after introduction of GST and as part of the same, the
following clauses were proposed to be included:
• Dealers are required to obtain specific approval of SSTPL prior to making change in the
marketing model or technical developments to the prejudice of customers.
• Specify the geographical area where the dealers can market the cars.
• Limit the operation of service centres by specifying dealers who can operate service
centres.
• Bar transactions or transfer of cars and spares between dealers.
• Mandate the floor price at which services may be provided by the dealers.
• Higher pricing of substitutable products and services.
• Mandate the dealers to acquire certain number of cars of the base version, when ordering
high end variants.
The agreement envisaged that no sale would be made to dealers who do not comply with the
above conditions. The Directors of the Company felt that some of these clauses are not in
compliance with the provisions of the Competition Act 2002.

© The Institute of Chartered Accountants of India


Page 84
PAPER – 6D: ECONOMIC LAWS 17

Answer the following questions


3.1 What is the percentage share of Bank A in the Committee of Creditors of DMPL under IBC,
2016 proceedings?
(A) 57.14%.
(B) 58.82%.
(C) 41.27%.
(D) 42.13%.
3.2 Out of the below, identify who is a related party of DMPL under the IBC, 2016?
(A) Mr. A, who holds 15% shares in DMPL.
(B) Indigenous Private Limited, who has one common independent director (with no
shareholding) with DMPL.
(C) Mr. X, who although not an employee or director of DMPL, is close to the promoter
and attends Board Meetings to provide guidance/directions on policy making process;
(D) Ms. Y, who controls the composition of Board of Directors of SSTPL.
3.3 Does the contract entered into by DMPL with its distributors cause an appreciable adverse
effect on competition under the Competition Act, 2002?
(A) Yes, since this is in the nature of a tie-in arrangement.
(B) No, this is a contract between a 'willing buyer' and 'willing seller' and they are free to
determine the contract terms;
(C) Yes, since transaction is in the nature of predatory pricing by DMPL to reduce
competition from other spares manufacturers.
(D) No, the contract actually promotes and sustains competition in the market.
3.4 The plan of SSTPL to consider a higher cost of substitutable goods and services for the
dealers is covered under which of the below factors under the Competition Act, 2002?
(A) Appreciable adverse effect on competition.
(B) Abuse of dominant position.
(C) Price rigging.
(D) Collusive pricing.

© The Institute of Chartered Accountants of India


Page 85
18 FINAL (NEW) EXAMINATION: NOVEMBER 2020

3.5 Can ACL file Corporate Insolvency Resolution Process against DMPL under IBC, 2016?
(A) Yes, ACL is an operational creditor and all the conditions under IBC, 2016 have been
fulfilled.
(B) No, ACL is not a financial creditor.
(C) No, since the amount of default is less than the minimum amount of default ( `100
lakh) for being covered under Section 4 of IBC, 2016;
(D) Yes, since the amount of default is not, disputed by DMPL and there is no ongoing
dispute. (5 x 2 = 10 Marks)
3.6 Answer the following questions:
(i) Advice the IRP with regard to the appropriateness of the order of the Adjudicating
Authority regarding, the application made by MCL under the provisions of the
Insolvency and Bankruptcy Code, 2016. (4 Marks)
(ii) Calculate the amount receivable by SSTPL from DMPL based on the facts given in
the case study (assume-no liquidation costs) as per Section 53 of the Insolvency and
Bankruptcy Code, 2016. (5 Marks)
(iii) Evaluate the terms of the agreement proposed to be entered into by SSTPL with the
dealers based as per the provisions of the Competition Act, 2002. (6 Marks)
ANSWER TO CASE STUDY 3
3.1 Option (C)
3.2 Option (C)
3.3 Option (A)
3.4 Option (B)
3.5 Option (C)
Answer 3.6
Enabling provisions for cross border transactions: India is no more an isolated business
place. India is now part of global business hub. Indian businesses have investments outside
India while many businesses outside India have presence in India. India is now a global village.
Enabling provisions in the Code are Sections 234 and 235 for this purpose.
Agreements with Foreign Countries: The Central Government may enter into an agreement
with the Government of any country outside India for enforcing the provisions of this Code.
Letter of request to a country outside India in respect of assets: If, in the course of
insolvency resolution process, or liquidation or bankruptcy proceedings, as the case may be,

© The Institute of Chartered Accountants of India


Page 86
PAPER – 6D: ECONOMIC LAWS 19

under this Code, the resolution professional, liquidator or bankruptcy trustee, is of the opinion
that assets of the corporate debtor or debtor, are situated in a country outside India with which
reciprocal arrangements have been made under section 234 of Insolvency and Bankruptcy
Code, 2016, he may make an application to the Adjudicating Authority that evidence or action
relating to such assets is required in connection with such process or proceeding.
The Adjudicating Authority on receipt of an application and, on being satisfied that evidence or
action relating to assets, is required in connection with insolvency resolution process or
liquidation or bankruptcy proceeding, may issue a letter of request to a Court or an authority of
such Country competent to deal with such request. [Section 235]
Accordingly, in the given case, order of the Adjudicating Authority of rejection of filing an
application under IBC, 2016 by MCL (a Company registered in Germany) is not in order because
as per Section 235, the Adjudicating Authority on receipt of an application on being satisfied
that evidence or action relating to assets, is required in connection with insolvency resolution
process or liquidation or bankruptcy proceeding, may issue a letter of request to a Court or an
authority of such Country competent to deal with such request.
Answer 3.6 (ii)
Section 53 of the Insolvency & Bankruptcy Code, 2016 lays the provisions related to distribution
of assets or the proceeds from the sale of the liquidation assets.
Distribution of proceeds from the sale of the liquidation assets: The proceeds from the sale
of the liquidation assets shall be distributed in the following order of priority —
(a) the insolvency resolution process costs and the liquidation costs paid in full;
(b) the following debts which shall rank equally between and among the following :—
(i) workmen's dues for the period of twenty-four months preceding the liquidation
commencement date; and
(ii) debts owed to a secured creditor in the event such secured creditor has relinquished
security in the manner set out in section 52;
(c) wages and any unpaid dues owed to employees other than workmen for the period of
twelve months preceding the liquidation commencement date;
(d) financial debts owed to unsecured creditors;
(e) the following dues shall rank equally between and among the following:—
(i) any amount due to the Central Government and the State Government including the
amount to be received on account of the Consolidated Fund of India and the
Consolidated Fund of a State, if any, in respect of the whole or any part of the period
of two years preceding the liquidation commencement date;

© The Institute of Chartered Accountants of India


Page 87
20 FINAL (NEW) EXAMINATION: NOVEMBER 2020

(ii) debts owed to a secured creditor for any amount unpaid following the enforcement of
security interest;
(f) any remaining debts and dues;
(g) preference shareholders, if any; and
(h) equity shareholders or partners, as the case may be.
Realisable value of the fixed assets + realisable value of receivables (50% of Rs. 225 lakhs) +
Bank Balance Amount=
(Rs 2800 + 112.5 + 22.5) lakhs = Rs. 2935 lakhs
Less outstanding wages to workmen = Rs.75 lakhs*
Less unpaid dues on account of statutory employer’s contribution treating them as workmen’s
dues = 30 lakhs
Less amount debts owed to a secured creditor** = (1500 + 1050) = Rs. 2550 lakhs
Less Loan taken from Mr. X = 75 lakhs
Balance amount available = 2935 – (75+30+2550+75) lakhs = 205 lakhs (which to be shared
between SSTPL and ACL***)
Therefore, amount receivable by SSTPL (205 / 980****x905) = Approx. Rs. 189.31 lakhs.
* It is assumed that outstanding wages of Rs. 75 Lakhs due to the workmen relate to the period
of 24 months preceding to the date of commencement of liquidation. [The question does not
mention the date of commencement of liquidation. Moreover, the term IRP needs to be replaced
by the term Liquidator since it is a case of Liquidation of a Corporate Person].
** It is assumed that the both the banks have relinquished their security interest and their
securities have been realized by the Liquidator for inclusion in the Liquidation estate. [In fact,
consolidated amount of Rs. 2800 lakhs being the realizable value of fixed assets validates this
assumption.]
***In respect of MCL, a Company registered in Germany, the Adjudicating Authority (AA) has
rejected its application filed under IBC, 2016. Further, no direction has been issued by the AA
regarding the outstanding amount of Rs. 400 lakhs. MCL has also not approached the Appellate
Authority for revival of rejected application. No stay order has been issued favouring MCL. In
addition, MCL has not filed any suit in Germany against DMPL for recovery of dues till the date
of commencement of liquidation. In such a case, from the facts of the questions which are not
elaborated in nature and from the limited information available, MCL cannot be treated as an
operational creditor at par with SSTPL or ACL.
**** After considering SSTPL (Rs. 905 lakhs) and ACL (Rs. 75 lakhs) as operational creditors.

© The Institute of Chartered Accountants of India


Page 88
PAPER – 6D: ECONOMIC LAWS 21

Answer 3.6 (iii)


Any agreement entered into between enterprises or associations of enterprises or persons or
associations of persons or between any person and enterprise or practice carried on or decision
taken by any association of enterprises or association of persons, including cartels’ engaged in
identical or similar trade of goods or provision of services shall be presumed to have an adverse
effect on competition which:
(a) directly or indirectly determines purchase or sale prices
(b) limits or contracts production, supply, markets, technical development, investment or
provision of services
(c) shares the market or source of production or provision of services by way of allocation of
geographical area of market or type of goods or services or number of customers in the
market or any other similar way.
(d) directly or indirectly results in bid rigging or collusive bidding
Any agreement entered into between enterprises or persons at different levels of the production
chain in different markets in respect of production, supply, distribution, storage, sale or price of
trade in goods or provision of services shall be a void agreement if it causes or is likely to cause
an appreciable adverse effect on competition in India including:
(a) Tie-in agreement
(b) Exclusive supply agreement
(c) Exclusive distribution agreement
(d) Refusal to deal
(e) Resale price maintenance
Accordingly the clauses proposed in the revise draft agreement by SSTPL, is limiting and
restricting to the production of goods or provision of services or market therefore specifying
geographical areas where dealers can market the cars , restricting technical or scientific
development relating to goods or services to the prejudice of consumers; resulting in denial of
market access by limiting the operations of service centers, bar on the transactions of car and
spares between dealers themselves, imposes unfair price in purchase or sale by mandating
floor price, higher pricing of substitutable products and services and imposing the dealers to
acquire certain numbers of cars while ordering high end variants
These all terms of agreement entered by SSTPL with dealers shows the abuse of dominant
position as per section 4 of the Competition Act, 2002.

© The Institute of Chartered Accountants of India


Page 89
22 FINAL (NEW) EXAMINATION: NOVEMBER 2020

CASE STUDY 4
The decade of 1960 was known as the golden period for goldsmiths in India and there was
tremendous interest in the minds of the people to buy and wear gold jewelry. Hard work and
expertise in making this jewelry made many goldsmiths millionaires in a very short period. Two
such goldsmiths were Mr. Selva Chetty and Mr. Thiagu Chetty, brothers who lived in Sivaganga
district, Tamil Nadu. Using the boom period, the Selva ventured to start several new business,
one of which was a small real estate company called Gangaikondan Holiday Properties Limited
(GHPL).
In the year 1970, Mr. Thiagu migrated to the United Kingdom and started his jewelry business
there. He used to visit India every year and give substantial sums to Mr. Selva to invest in India
on behalf of Mr. Thiagu and for his benefit to use once he comes back to India Mr. Selva
mentioned to him that it may be worthwhile to invest the money in buying large tracts of land
near Sivaganga and the same is expected to appreciate significantly in the next 10 years.
Mr. Thiagu was very much interested in this and therefore, in the year 1989, Mr. Selva
purchased 10 acres of land from the Government in his name, in the capacity as fiduciary
relationship/trustee of Mr. Thiagu and hold the property on behalf of and for the benefit of
Mr. Thiagu. Mr. Selva used the land for cultivation of crops and was using the crops for his
consumption and for sale. The proceeds from the sale was deposited by Mr. Selva in his bank
account.
In the meantime, Mr. Selva got married and was blessed with a son Mr. Venkat. In the year
1971, when Mr. Venkat was 6 years old, Mr. Selva acquired a new residential house comprising
of 4 individual units in the name of Mr. Venkat since he felt that buying the new home in his
son's name will be auspicious for Mr. Selva and the new home. For this purpose, Mr. Selva took
a 5 year loan from Bank of Sivaganga and was repaying the loans promptly on the due dates
and got back the title deeds from the Bank once the loan was repaid. The new home was
occupied by Mr. Selva and his family and Mr. Selva rented out 2 portions on rent to tenants. Mr.
Selva paid the property taxes for the property and maintained the property on his own account.
In 1980, Mr. Selva was blessed with another child who was named Ms. Bhagyalakshmi. In 1984,
Mr. Selva prepared his will as per which he considered that the residential house will belong to
Mr. Venkat and Ms. Bhagyalakshmi in equal measure, which was not disclosed to anyone.
GHPL commenced construction of a large apartment complex in an upcoming industrial belt of
Sivaganga. There was tremendous expectation that several large companies were going to set
up factories in the location and therefore, the demand for housing expanded significantly. A lot
of housing companies commenced projects in the location.
In one of the discussions between the real estate companies, GHPL was approached by other
leading real estate developers who were constructing high rise apartments in the vicinity to have
a tacit (unwritten) understanding for jacking up the prices of the apartments and also in
unbundling of the open car parking given to the allottees from the total price and charging
separately for the same. This would help the companies in providing the best-in-class facilities
to the apartment buyers at the same time ensure good profitability for the companies. GHPL did

© The Institute of Chartered Accountants of India


Page 90
PAPER – 6D: ECONOMIC LAWS 23

not immediately agree to the same but wanted to evaluate the implications of such an
agreement. One of the real estate developers wanted to extend the understanding to the
infrastructure projects by these companies in UAE also (since many of them are constructing
homes in UAE as well).
In the year 1986, Mr. Venkat got married and declared that he is the absolute owner of the
residential house since the house is in his name and was purchased by his father in his name
purely for his benefit when he was a minor and to help him settle down in his life. He then asked
for vacation of the property by Mr. Selva and his family as well as the tenants. Mr. Selva was
enraged by this act of Mr. Venkat and filed a suit for declaring the property as a benami property
where Mr. Venkat was a benamidar and he was the rightful owner of the same. They discussed
the matter with various consultants for determination of a benami transaction as decided by
Hon'ble Supreme Court of India.
In May 2017, GHPL is evaluating the acquisition of another large real estate company in
Sivaganga and is contemplating the implications of the Competition Act, 2002 in this regard.
Answer the following questions:
4.1 The CFO of GHPL seeks your views to understand which of the following would not be a
violation of the provisions of the Competition Act, 2002?
(A) Predatory Pricing.
(B) Limiting production of goods.
(C) Agreement for Protection of rights under the Designs Act, 2000.
(D) Denial of market access.
4.2 What is the term of the members of the Competition Commission under the Competition
Act, 2002 which is reviewing the agreement / tacit understanding between the real estate
companies in the case study?
(A) 5 years, eligible for re-appointment for one more term.
(B) 5 years, eligible for re-appointment.
(C) 5 years, not eligible for re-appointment.
(D) Upto the discretion of the Central government.
4.3 Assuming that the acquisition of another real estate company by GHPL happened in the
year 2019, what is the maximum amount of assets and revenue that can be acquired by
GHPL for being accepted from the provisions of Section 5 of the Competition Act, 2002
(A) Post-acquisition (incl. GHPL) asset value off ` 350 crore and `1000 crore
respectively.

© The Institute of Chartered Accountants of India


Page 91
24 FINAL (NEW) EXAMINATION: NOVEMBER 2020

(B) Asset value off `350 crore and turnover off `1000 crore of the target entity being
acquired.
(C) Post-acquisition (incl. GHPL) value off `1000 crore or turnover of `3000 crore of the
target entity.
(D) Asset value off ` 350 crore or turnover of ` 1000 crore of the target entity being
acquired.
4.4 Assuming that the proposed combination is covered under Section 5 of the Competition
Act, 2002, and GHPL gave notice to the Commission on 15 th May, 2018, what is the latest
date by when the combination will come into effect (no orders have been passed by the
Commission)?
(A) 13 th August 2018.
(C) 15 th May 2019.
(B) 11 th December 2018.
(D) 11 th November 2018.
4.5 Under the Prohibition of Benami Property Transactions Act, 1988, who is responsible for
issuing notice for furnishing evidence to Selva and Venkat?
(A) Approving Authority
(C) Adjudicating Authority.
(B) Initiating Officer
(D) Administrator. (5 x 2 = 10 Marks)
4.6 Answer the following questions:
(I) Discuss the judicial pronouncements on tests for determination of a benami
transaction as decided by Hon'ble Supreme Court of India under Prohibition of
Benami Property Transactions Act, 1988. (6 Marks)
(II) Analyse the case with regard to Mr. Selva's contention regarding the house
purchased by him in the name of Mr. Venkat and Mr. Selva's rights under the
Prohibition of Benami Property Transactions Act, 1988 to recover the property.
(4 Marks)
(III) GHPL reaches out to you for your advice regarding the proposal from the other real
estate developers under the Competition Act, 2002. (5 Marks)

© The Institute of Chartered Accountants of India


Page 92
PAPER – 6D: ECONOMIC LAWS 25

ANSWER TO CASE STUDY 4


4.1 Option (C)
4.2 Option (B)
4.3 Option (C)
4.4 Option (B)
4.5 Option (A)
Answer 4.6
(I) Judicial pronouncements on tests for determination of a benami transaction:
In the matter of Bhim Singh & Anr vs Kan Singh (And Vice Versa) 1980 AIR 727, 1980 SCR
(2) 628, the Hon’ble Supreme Court of India, observed –
The principle governing the determination of the question whether a transfer is a benami
transaction or not may be summed up thus:
(a) The burden of showing that a transfer is a benami transaction lies on the person who
asserts that it is such a transaction;
(b) if it is proved that the purchase money came from a person other than the person in
whose favour the property is transferred, the purchase is prima facie assumed to be
for the benefit of the person who supplied the purchase money, unless there is
evidence to the contrary;
(c) the true character of the transaction is governed by the intention of the person who
has contributed the purchase money and
(d) the question as to what his intention was has to be decided on
(i) the basis of the surrounding circumstances,
(ii) the relationship of the parties,
(iii) the motives governing their action in bringing about the transaction and
(iv) their subsequent conduct etc.
All the four factors stated above may have to be considered cumulatively [O P Sharma
vs. Rajendra Prasad Shewda & Ors. (CA 8609-8610 of 2009) (SC)].
In the matter of Valliammai(D) by LRS.V.Subramaniam and Others (2004) 7 SCC
2330 the Honorable Supreme Court observed that the essence of a benami
transaction is the intention of the party or parties concern and often, such intention is
shrouded in a thick veil which cannot be easily pierced through. But such difficulties

© The Institute of Chartered Accountants of India


Page 93
26 FINAL (NEW) EXAMINATION: NOVEMBER 2020

do not relieve the person asserting the transaction to be benami of any part of the
serious onus that rests on him nor justify the acceptance of mere conjectures or
surmises as a substitute for proof.
Answer 4.6 (II)
"Benami transaction" as per Section 2(9) of the Prohibition of Benami Transaction Act, 1988
means, a transaction or an arrangement where the property is held for the immediate or future
benefit, direct or indirect, of the person who has provided the consideration, except when the
property is held by any person being an individual in the name of his spouse or in the name of
any child of such individual and the consideration for such property has been provided or paid
out of the known sources of the individual.
In the instant case, Mr. Selva purchased the house in the name of his son Mr. Venkat through
a 5 year bank loan and used 2 units for his family and rented out 2 portions on rent.
In the light of the above provisions, the said transaction is not a benami transaction and Mr.
Venkat is not a benamidar and is a real owner.
Right of Mr. Selva under Section 4 of the PBPTA, 1988
No suit, claim or action to enforce any right in respect of any property held benami against the
person in whose name the property is held or against any other person shall lie by or on behalf
of a person claiming to be the real owner of such property.
Moreover, the transaction in question was registered in the year 1978. The suit was filed in the
year 1986, which was before coming into force of the PBTP Act in 1988. Since, the PBTP Act
cannot have any retrospective applicability.
Accordingly, Mr. Selva’s right is prohibited to recover the property.
Answer 4.6 (III)
As per Section 3 of the Competition Act, 2002, any agreement entered into between
enterprises or associations of enterprises or persons or associations of persons or between any
person and enterprise or practice carried on, or decision taken by, any association of enterprises
or association of persons, including cartels, engaged in identical or similar trade of goods or
provision of services, shall be presumed to have an appreciable adverse effect on competition,
which—
(a) directly or indirectly determines purchase or sale prices;
(b) limits or controls production, supply, markets, technical development, investment or
provision of services;
(c) shares the market or source of production or provision of services by way of allocation of
geographical area of market, or type of goods or services, or number of customers in the
market or any other similar way;

© The Institute of Chartered Accountants of India


Page 94
PAPER – 6D: ECONOMIC LAWS 27

(d) directly or indirectly results in bid rigging or collusive bidding.


However, any agreement entered into by way of joint ventures, if such agreement increases
efficiency in production, supply, distribution, storage, acquisition or control of goods or provision
of services, shall not be considered to be an anti-competitive agreement.
Therefore the proposal of other leading real estate to have understanding with GHPL, in the
light of facts, will increase efficiency in providing best class facilities to the apartment buyers
and at the same time ensure good profitability for the companies. This proposal shall not be an
anti-competitive agreement.
CASE STUDY 5
An Investigation was carried out at the office of WWL Mumbai by the Assistant Director under
the Prevention of Money Laundering Act, 2002, in the process they came across violation of
the Foreign Exchange Management Act,1999. The Assistant Director discussed the case with
you and apprised the matter as under:-
WWL is based in Mumbai and is India's premier watch manufacturing company and specializes
in designing and manufacturing high-end watches. Its products are sold across premier stores
in India and abroad. WWL was established by Mr. Virender Kohli, a first-time entrepreneur. The
marketing department of WWL introduced new models in the past 4 months and expects these
watches to be a major attraction in the global markets especially UK, France and US markets.
For the purpose of advertisements, WWL engaged the services of Mr. George Mckenzie, a
prominent NBA player and Ms. Rudy Hobbs, a Miss Universe winner and agreed to pay a
"guaranteed" fee of USD 1,000,000 each plus 5% bonus based on the sales of the new models
in 1st year. The marketing strategy was highly successful and Virender earned a significant
amount through the sale of 10% stake in WWL to a private equity investor.
This was invested in his various businesses to acquire agricultural farm land (to grow and export
opium), acquiring and selling (export) of antiquities etc. A Marks majority of his dealings on the
farm and antiquities businesses were done through cash transactions or through a specific bank
account maintained with ABC Bank Limited. Amounts were received in cash from his
international customers through a hawala agent known to Mr. Virender. He also purchased villas
in India and in Spain using the money earned through his farm and antiquities businesses. Mr.
Virender also established Sure Returns Private Limited, a small non-banking finance company
for securing the lives of his employees and their families. Virender invested an amount of `5
crore in Sure Returns out of the funds received from his antiquities business.
WWL sent 10 watches to his 500 dealers abroad, clearly marked as riot for sale and other
promotional material, for display in dealer shops etc. The value of the items were approximately
INR 6 crore. He also sent 1 watch for each of his dealers as a token of gift and appreciation
(total value of INR 40 lakh). The CFO of WWL is of the view that since these products have
been sent free of cost and not for sale, these need not be included in the export declaration to
be filed by WWL.

© The Institute of Chartered Accountants of India


Page 95
28 FINAL (NEW) EXAMINATION: NOVEMBER 2020

Mr. Virender attended one of the manufacturing conferences held in Mumbai, in which he met
one Mr. Alex Smith, who runs a watch designing studio in Italy and showed quite a few exhibits
to Mr. Virender. Mr. Virender was impressed by the designs and the prices quoted by Alex. Alex
was also amenable to receive funds in cash in India through an intermediary and then provide
the material to Virender from Italy. Based on the same, Mr. Virender arranged for making cash
payment to the extent of INR 3 crore to an intermediary in Delhi and the material was received
from Alex in a month. During his visit to India, Alex noted that his Euro passport got expired and
he did not realise the same. Since he did wanted to leave India immediately, he got in touch
with a travel agent, who helped him get a forged passport, for which Mr. Alex paid INR 3 lakh in
cash.
In order to clear the imported material critical for its manufacturing process, WWL used cash
amounting to INR 30 lakhs to pay amounts to various intermediaries to facilitate timely and
smooth import process and the amounts were paid by the intermediaries to Mr. Raghav Kapoor.
Using this money, Mr. Raghav purchased a 1 acre farm house in Munnar in the name of his
spouse, Ms. Anu Kapoor, who was not aware of the source of the funds and was residing in the
farm house along with her parents. The Enforcement Directorate, as part of the proceedings
against Mr. Raghav Kapoor sought to attach and confiscate the farm house owned / purchased
in the name of Ms. Anu. This was challenged by Mr. Raghav on the basis that this property was
owned and possessed by Anu who is not charged under a scheduled offence under the
Prevention of Money Laundering Act, 2002. With Mr. Alex's help, Mr. Virender transferred an
amount of INR 260 lakh to an intermediary in Delhi and invested the amount to incorporate a
shell company in the Isle of Mann. The funds were then transferred back by the Shell Company
to the bank account of WWL. For this purpose, WWL raised export invoices in its books on the
Shell Company for providing professional services relating to watch designing. Based on these
invoices, WWL claimed export incentives under the relevant laws in India and received INR 15
lakh as export incentive.
On 30 th March 2018, WWL made a large sale to one of the dealers in Switzerland for EURO 8
million and had received EURO 3 million by 15 th May 2018 and did not receive the balance
EURO 5 million until 30th October 2018, i.e. 7 months from the date of sale. After several
reminders and threating calls to stop further shipment, another EURO 1 million was received on
10 th October 2018 and the balance remained outstanding as at 31 st December 2018. The CFO
of WWL reaches out to Mr. Z and seek Mr. Z support to evaluate the level of compliances as
stipulated under the Foreign Exchange Management Act, 1999.
Based on investigation carried out, the Assistant Director sought to arrest Virender and also
wanted to attach the property for contravention of provision of Prevention of Money laundering
Act, 2002 (in short 'PMLA, 2002')
After the discussions the Assistant Director sought your views on powers for attachment of
property involved in money-laundering and on punishment for the offence of money laundering
under the provisions of Prevention of Money Laundering Act, 2002.

© The Institute of Chartered Accountants of India


Page 96
PAPER – 6D: ECONOMIC LAWS 29

Answer the following questions:


5.1 Out of the below, which are the items that require inclusion in the export declaration by
WWL under the, Foreign Exchange Management Act, 1999?
(A) Goods imported free of cost for re-export.
(B) Publicity materiality supplied free of cost; ·
(C) Gift of goods for a value of INR 10 lakh.
(D) Unaccompanied personal effects of travellers.
5.2 Out of the below, what is not part of the responsibility of ABC Bank Limited under the
Prevention of Money Laundering Act, 2002?
(A) Report suspicious transactions undertaken by Mr. Virender and the Group;
(B) Furnish all-information requested by the Director;
(C) Verify the identity of the clients and beneficial owners;
(D) Maintain records of transaction for a period of 5 years;
5.3 A friend of Mr. Virender is an Indian citizen resident outside India, is seeking to transfer his
agricultural property held by him in India. Who can he transfer the property to?
(A) Any person resident in India.
(B) Any person resident outside India if he is a citizen of India or a person of Indian origin.
(C) Any person resident in India and any person resident outside India if he is a citizen
of India or a person of Indian origin.
(D) Neither any person resident in India nor any person resident outside India if he is a
citizen of India or a person of Indian origin.
5.4 Mr. Virender bought gold watches worth INR 25 lakh from Italy through the green channel
which he asked his Italian dealer to pay and deduct from their monthly payments to WWL.
Is this an offence under the Prevention of Money Laundering Act, 2002?
(A) Yes, because he came through the green channel and evaded duty of customs.
(B) No, whilst it is an offence, it is not actionable under the Prevention of Money
Laundering Act, 2002.
(C) No, since he did not pay any cash for the purchase.
(D) Yes, since import of gold items from European countries requires specific consent as
per the agreement entered with foreign countries as per Section 56 of Prevention of
Money Laundering Act, 2002.

© The Institute of Chartered Accountants of India


Page 97
30 FINAL (NEW) EXAMINATION: NOVEMBER 2020

5.5 Does the Assistant Director have powers to arrest a person under the Prevention of Money
Laundering Act, 2002?
(A) Director or Deputy Director or Assistant Director have the powers to arrest an offender
without prior approval of Central Government
(B) Any arrest under the Prevention of Money Laundering Act, 2002 requires the prior
approval of the Central Government
(C) Only a Director or Deputy Director have the powers to arrest without prior approval of
the Central Government
(D) Any arrest under the Prevention of Money Laundering Act, 2002 requires the prior
approval of the special court. (5 x 2 = 10 Marks)
5.6 Answer the following questions:
(I) The Enforcement Directorate wanted to take your view on powers for attachment of
property involved in money-laundering and your views on punishment for the offence
of money laundering under the provisions of the Prevention of Money Laundering Act,
2002. Express your views on the same. (7 Marks)
(II) The Enforcement Directorate, as part of the proceedings against Mr. Raghav Kapoor
sought to attach and confiscate the farm house owned /purchased by Anu, This was
challenged by Mr. Raghav on the basis that this property was owned and possessed
by Anu who is not charged under a scheduled offence under the Prevention of Money
Laundering Act, 2002. Advice Mr. Raghav on the validity or otherwise of his
contention. (4 Marks)
(III) The CFO of WWL reaches out to Mr. Z and seek Mr. Z support to evaluate if there is
a non-compliance under the Foreign Exchange Management Act, 1999 regarding the
sale made to the dealer in Switzerland and the receipt of the proceeds and if so, the
quantum, the consequences and the future course of action that needs to be taken
by WWL relating to the same. (4 Marks)
ANSWER TO CASE STUDY 5
5.1 Option (C)
5.2 Option (A)
5.3 Option (A)
5.4 Option (A)
5.5 Option (B)

© The Institute of Chartered Accountants of India


Page 98
PAPER – 6D: ECONOMIC LAWS 31

Answer 5.6 (I)


Attachment of property involved in money-laundering [Section 5 of the Prevention of
Money Laundering Act, 2002]
1. Where the Director or any other officer (not below the rank of Deputy Director authorized
by the Director) for the purposes of this section, has reason to believe on the basis of
material in his possession, that—
(a) any person is in possession of any proceeds of crime; and
(b) such proceeds of crime are likely to be concealed, transferred or dealt with in any
manner which may result in frustrating any proceedings relating to confiscation of
such proceeds of crime under this Chapter,
he may, by order in writing, provisionally attach such property for a period not exceeding
one hundred and eighty days from the date of the order, in such manner as may be
prescribed.
Conditions for Attachment: Provided that no such order of attachment shall be made
unless, in relation to the scheduled offence:
- a report has been forwarded to a Magistrate under Section 173 of the Code of
Criminal Procedure, 1973, or
- a complaint has been filed by a person authorized to investigate the offence
mentioned in that Schedule, before a Magistrate or Court for taking cognizance of the
scheduled offence, as the case may be, or
- a similar report or complaint has been made or filed under the corresponding law of
any other Country.
2. The Director, or any other officer not below the rank of Deputy Director, shall, immediately
after attachment forward a copy of the order, along with the material in his possession, to
the Adjudicating Authority, and such Adjudicating Authority shall keep such order and
material for such period as may be prescribed.
3. Every order of attachment made shall cease to have effect after the expiry of the period
specified in that sub-section or on the date of an order made under sub-section (3) of
Section 8, whichever is earlier.
4. The Director or any other officer who provisionally attaches any property under sub-section
(1) shall, within a period of thirty days from such attachment, file a complaint st ating the
facts of such attachment before the Adjudicating Authority.

© The Institute of Chartered Accountants of India


Page 99
32 FINAL (NEW) EXAMINATION: NOVEMBER 2020

Section 4 provides for the Punishment for Money-Laundering - Whoever commits the offence of
money-laundering shall be punishable with rigorous imprisonment for a term which shall not be
less than three years but which may extend to seven years and shall also be liable to fine.
But where the proceeds of crime involved in money-laundering relate to any offence under the
Narcotic Drugs and Psychotropic Substances Act, 1985, the maximum punishment may extend
to ten years instead of seven years.
Answer 5.6 (II)
Section 2(1)(u) of the Prevention of Money Laundering Act, 2002 , "proceeds of crime" can
be understood as any property derived or obtained, directly or indirectly, by any person as a
result of criminal activity relating to a scheduled offence or the value of any such property or
where such property is taken/held outside the country, then the property equivalent in value held
within the country or abroad.
As per the stated facts, farm house was purchased by Mr. Raghav on the name of his spouse
Ms. Anu who was not aware of sources of the funds. ED sought to attach the farm house and
confiscate as a part of proceeding against Mr. Raghav. Here the contention of Mr. Raghav is
not valid because the said property was derived from the proceeds of crime.
Answer 5.6(III)
Period within which export value of goods/software/ services to be realized:-
(1) The amount representing the full export value of goods / software/ services exported shall
be realized and repatriated to India within nine months or within such period as may be
specified by the Reserve Bank, in consultation with the Government, from time to time.
(a) that where the goods are exported to a warehouse established outside India with the
permission of the Reserve Bank, the amount representing the full export value of
goods exported shall be paid to the authorised dealer as soon as it is realis ed and in
any case within fifteen months or within such period as may be specified by the
Reserve Bank, in consultation with the Government, from time to time;
(b) further that the Reserve Bank, or subject to the directions issued by that Bank in this
behalf, the authorised dealer may, for a sufficient and reasonable cause shown,
extend the said period, as the case may be.
Delay in Receipt of Payment:
Where in relation to goods or software export of which is required to be declared on the
specified form and export of services, in respect of which no declaration forms has been
made applicable, the specified period has expired and the payment therefor has not been
made as aforesaid, the Reserve Bank may give to any person who has sold the goods or
software or who is entitled to sell the goods or software or procure the sale thereof, such
directions as appear to it to be expedient, for the purpose of securing,

© The Institute of Chartered Accountants of India


Page 100
PAPER – 6D: ECONOMIC LAWS 33

(a) the payment therefor if the goods or software has been sold and
(b) the sale of goods and payment thereof, if goods or software has not been sold or re-
import thereof into India as the circumstances permit, within such period as the
Reserve Bank may specify in this behalf;
Provided that omission of the Reserve Bank to give directions shall not have the effect of
absolving the person committing the contravention from the consequences thereof.
Quantum: In the given case, out of total sale of EUR 8 million, an amount of EUR 4 million
was received within the stipulated time period of 9 months and the balance EUR 4 million
is outstanding for a period of more than 9 months. Accordingly, WWL is required to apply
for an extension of time with the Authorized Dealer giving sufficient and reasonable
reasons for the delay in receipt.
As per Section 8, where any amount of foreign exchange is due or has accrued to any
person resident in India, such person shall take all reasonable steps to realize and
repatriate to India such foreign exchange within such period and in such manner as may
be specified by the Reserve Bank. WWL will act in compliance with the above provisions.

© The Institute of Chartered Accountants of India


Page 101
DISCLAIMER

This Suggested Answers do not constitute the basis for evaluation of the student’s answers in

the examination. The answers are prepared by the Faculty of the Board of Studies with a view

to assist the students in their education. While due care is taken in preparation of the answers,

if any errors or omissions are noticed, the same may be brought to the attention of the Director

of Studies. The Council of the Institute is not in anyway responsible for the correctness or

otherwise of the answers published herein.

Further, in the Elective Papers which are Case Study based, the solutions have been worked

out on the basis of certain assumptions/views derived from the facts given in the question or

language used in the question. It may be possible to work out the solution to the case studies

in a different manner based on the assumption made or view taken.

© The Institute of Chartered Accountants of India


Page 102
2 FINAL (NEW) EXAMINATION: NOVEMBER, 2019

PAPER 6D: ECONOMIC LAWS

The solutions to case studies have been worked out on the basis of certain assumptions/views
derived from the facts given in the question or language used in the question. It may be
possible to work out different solutions based on the assumption made or view taken. Further,
there should be no negative marking for wrong answers in MCQ based questions.

NOTE: There are five case study questions in the question paper. Candidates are
required to answer all the questions of any four case study questions.
Citation of case laws, sections, subsections, rules may not form part of the answer.

CASE STUDY 1
1. Delta Corporation, a government corporation purchases Aluminium Phosphide Tablets
(APT) on bulk basis through a formal tender process for the past several years. The main
market of APT in India was that of the institutional sales and a majority of buyers were
Government agencies. The number of private buyers was insignificant.
APT is manufactured only by 4 companies in the country, namely M/s. Easy,
M/s. Samurai, M/s. Multicrop and, M/s. Agro Chemicals. Sometime during the year 2018,
Mr. Rohit the Chairman and Managing Director of Delta Corporation, as part of his review
of the operations, analysed the purchase of APT over the last several years, and noted a
trend that the four manufacturers of APT had formed a cartel by entering into an
anticompetitive agreement amongst themselves and on that basis they had been
submitting their bids for last eight years by quoting identical rates in the tenders invited
by the Delta Corporation for the purchase of APT. Based on the above, Mr. Rohit wrote a
complaint to the Competition Commission of India (CCI) on February 4, 2018 and the CCI
assigned the complaint to the Director General (DG) for investigation.
Based on the investigation carried out, the DG noted the following:
• Right from the year 2009, upto the year 2016, all the four parties used to quote
identical rates, excepting for the year 2014. In 2009, `245 was the rate quoted by
these four parties and in the year 2012 it was ` 310 (though the tender was
scrapped in this year). In November, 2012, though the tenders were invited, all the
parties had abstained from quoting. In 2014, M/s. Samurai had quoted the price
which was much below the price of other competitors. In 2015, all the parties
abstained from quoting, while in 2016 only the three appellants, barring Agro
Chemicals, participated and quoted uniform rate of 388, which was ultimately
brought down to `386 after negotiations.

© The Institute of Chartered Accountants of India


Page 103
PAPER – 6D: ECONOMIC LAWS 3

• It was also found that the tender documents were usually submitted in-person and
the rates were normally filled with hand;
• In respect of the tender floated in March, 2016, the three appellants had quoted
identical rates of `388.
• The DG also analysed the bidding pattern for tenders issued by other corporations
during the period from 2014 to 2018 and concluded that the pricing pattern was
similar between the parties in such tenders as well, as indicated below :
Corporations Year Price Quoted
Easy Samurai Multicrop Agro Chemical
A 2014 225 225 - -
B 2015 260 260 - -
C 2015 450 - 450 -
C 2016 414 414 - -
Delta 2016 388 388 388 -
B 2016 399 - - 399
D 2016 - - 399 399
B 2017 419 - - 410
C 2017 421 421 421 -
B 2018 - 415 - 415

Based on the investigation carried out above, the DG concluded that:


• The pricing pattern definitely showed the practice of quoting identical pricing by all
the parties.
• The explanation given by the parties (rise in price was mostly attributed to increase
in price by China) for the common pricing was unconvincing since it was noticed
that even during the period when the Phosphorours prices had fallen, no reflection
thereof was seen in the high prices quoted by the parties.
• Examination of the cost structure of each company reflected that there was nothing
common between the parties as far as the said cost structure was concerned and,
therefore, quoting of identical prices by all the parties was unnatural.
• Joint boycotting by the parties, at times, showed their concerted action, which
happened again in March, 2018 when the Delta Corporation had issued e-tender,
which was closed on July 25, 2018.

© The Institute of Chartered Accountants of India


Page 104
4 FINAL (NEW) EXAMINATION: NOVEMBER, 2019

On the basis of the aforesaid findings, the DG framed an opinion that the appellants had
contravened the provisions of Sections 3(3)(a), 3(3)(b) and 3(3)(d) read with Section 3(1) of
the Competition Act, 2002.
The CCI called for the responses of the parties for the above observations of the DG and the
responses of the parties are as under:
• In so far as tender of 2018 is concerned, it was contended that inquiry in respect of
boycotting the said tender by the appellants was without jurisdiction in as much as the
Delta Corporation in its complaint dated February 04, 2018 did not mention about the
said tender.
• On the merits, increase in the price over a period of time, particularly between years
2016 and 2018, was sought to be justified on the ground that the "price of yellow
phosphorous, which was to be procured from China, had increased". It was further
submitted that merely because there was identical prices quoted by the parties, it would
not mean that there was any bid rigging or formation of cartel by the parties. Submission
in this behalf was that· the market forces brought the situation where the prices became
so competitive and it had led to the aforesaid trend.
• It was further submitted that, notwithstanding the same price quoted by the parties, each
time the tender was evaluated by a Committee of Officers of the Delta Corporation and
no such suspicion was raised by the Committee. On the contrary, this aspect was
specifically gone into and the Committee was satisfied that quoting of identical price was
not due to any cartelisation.
The CCI rejected each of the responses provided by the parties and concluded 'that the
parties had entered into an agreement or understanding, and indulged in anti-competitive
activities while submitting their bids in response to the tenders issued by the Delta
Corporation.
Prosper Extractors Limited (PEL) is one of the key operational creditors of Multicrop and was
the sole supplier of Phosphorous to Multicrop for the manufacture of the APT. The
arrangement between PEL and Multicrop was formally documented through a blanket
Purchase Order on an annual basis with weekly supply schedule and a 30 days credit period.
Due to the financial issues including losses of Multicrop, there was a significant backlog in the
payment by Multicrop and in line with the terms of the purchase order, the matter was referred
to an Arbitral Tribunal with claims and counter claims by both parties. The Arbitral Tribunal
delivered its award in favour PEL for the entire balance payable (including receivables
assigned to the bank without recourse basis) by Multicrop and rejected the cross claims of
Multicrop. Multicrop proceeded to file a petition under the Arbitration and Conciliation Act,
1996 challenging the award of the Arbitral Tribunal. Based on the opinion of CFO that the
object of IBC, 2016 is also to hold promoters personally financially liable for default of the
firms they control, an application was then filed by PEL under Section 9 of the IBC, 2016 as
the sole operational creditor of Multicrop. Th e NCLT, based on the application; admitted the

© The Institute of Chartered Accountants of India


Page 105
PAPER – 6D: ECONOMIC LAWS 5

same since there is a clear evidence of a demand and the appropriate notice has been
submitted by PEL as per the IBC, 2016.
Answer the following questions :
1.1 Which of the following is not part of the objectives for introduction of the IBC, 2016?
(A) Avoiding destruction of value.
(B) Hold Promoters personally financially liable for default of the firms that they control
as opined by CFO in the case study.
(C) Improve handling of conflicts between creditors and debtor through process of
negotiation.
(D) Clear allocation of losses during downturn. (2 Marks)
1.2 Which of the following is not covered under the definition of a financial debt under IBC,
2016?
(A) Interest on Unsecured debentures issued by a corporate debtor.
(B) Market value of a derivative taken to hedge foreign currency fluctuations of an ECB
loan.
(C) Amount raised from an allottee of an apartment under a real estate project.
(D) Receivables assigned to a Bank on without recourse basis. (2 Marks)
1.3 The IRP appointed for Multicrop is seeking your views on the constitution of the
Committee of Creditors of Multicrop. Multicrop does not have any financial debt other
than a loan obtained from Mr. Ajay Jhawar, son of the Mr. Vijay Jhawar, the Managing
Director of Multicrop. Considering the above, identify the appropriate constitution of the
Committee of Creditors out of the following :
(A) Mr. Ajay Jhawar, 18 largest operational creditors, and 1 representative of all
workmen.
(B) 18 largest operational creditors, 1 representative of workmen and 1 representative
of employees.
(C) Only Mr. Ajay Jhawar since he is the only financial creditor.
(D) 18 largest operational creditors, 1 representative of workmen and 1 representative
of employees and the resolution professional. (2 Marks)
1.4 Which of the following are not factors which need to be considered for determining the
relevant product market under the Competition Act, 2002 ?
(A) Existence of specialised producers

© The Institute of Chartered Accountants of India


Page 106
6 FINAL (NEW) EXAMINATION: NOVEMBER, 2019

(B) Market structure and size of market


(C) Consumer preferences
(D) Actual end use of the products (2 Marks)
1.5 When evaluating whether the arrangement between the parties involved shall be
presumed to be anti-competitive and likely to have an appreciable adverse effect on
competition, which of the following are not factors to be considered by the Director
General ?
(A) Limit and control the use of technology used by all parties in manufacturing APT.
(B) Allocate the supply of APT in India between the parties and limit new entrants.
(C) Collectively determine the purchase price of the key raw material (phosphorous)
from the vendors.
(D) Joint venture between the parties to share distribution channels and logistics
services to reduce cost. (2 Marks)
1.6 Answer the following questions in the context of the provisions relating to Competition
Act, 2002.
(i) Analyse whether the CCI can consider the tender called for in March, 2009 and
negotiations finalised in July, 2009 for examination under Section 3, which became
operational only on 20th May, 2009. (3 Marks)
(ii) Whether CCI was barred from investigating the matter pertaining to the tender
floated by Delta Corporation in March, 2018 on the basis that this was not a subject
matter contained in the complaint submitted by Delta Corporation on 4 February,
2018. (3 Marks)
(iii) Analyse based on the facts of the case, regarding the conclusion of CCI that the
appellants had entered into an agreement to indulge in collusive bidding by forming
a cartel, resulting into contravention of Section 3 of the Act. (5 Marks)
1.7 Examine/advise regarding the below questions relating to the Insolvency and Bankruptcy
Code, 2016:
What is your view with regard to the stand taken by NCLT in admitting the application of
PEL for initiating insolvency proceedings against Multicorp? (4 Marks)
ANSWER TO CASE STUDY 1
1.1 (B)
1.2 (D)

© The Institute of Chartered Accountants of India


Page 107
PAPER – 6D: ECONOMIC LAWS 7

1.3 (B)
1.4 (B)
1.5 (D)
Descriptive Answers
1.6 (i) According to Section 36 of the Competition Act, 2002, the Commission shall have,
for the purposes of discharging its functions under this Act, the same powers as are
vested in a Civil Court under the Code of Civil Procedure, 1908, while trying a suit,
in respect of the following matters, namely:
(a) summoning and enforcing the attendance of any person and examining him on
oath;
(b) requiring the discovery and production of documents;
(c) receiving evidence on affidavit;
(d) issuing commissions for the examination of witnesses or documents;
(e) requisitioning, subject to the provisions of Sections 123 and 124 of the Indian
Evidence Act, 1872, any public record or document or copy of such of record
or document from any office.
The Commission may also direct any person:
(a) to produce before the Director General or the Secretary or an Officer
authorized by it, such books, or other documents in the custody or under the
control of such person so directed as may be specified or described in the
direction, being documents relating to any trade, the examination of which may
be required for the purposes of this Act;
(b) to furnish to the Director General or the Secretary or any other Officer
authorized by it, as respects the trade or such other information as may be in
his possession in relation to the trade carried on by such person, as may be
required for the purposes of this Act.
Hence, CCI can also consider the tender called for in March, 2009.
Alternate Solution
The bid was called in March 2009 and negotiations finalized in July, 2009 by which
date, Section 3 of the Competition Act, 2002 had already been activated. Therefore,
the principle of retro-activity shall become applicable as the process of finalization
of the tender was still on. Therefore, the inquiry into the tender of March, 2009 by
the CCI is covered by Section 3 of the Act in as much as the tender process, though
initiated prior to the date when Section 3 became operational, continued much

© The Institute of Chartered Accountants of India


Page 108
8 FINAL (NEW) EXAMINATION: NOVEMBER, 2019

beyond May 20, 2009, the date on which the provisions of Section 3 of the Act were
enforced.
In the light of the above, it can be concluded that CCI can consider the tender called
for.
(ii) According to Section 19 of the Competition Act, 2002, the Commission is
empowered to inquire into any alleged contravention of the provisions contained in
Section 3(1) or Section 4(1) either on its own motion or on:—
(a) receipt of any information in such manner and accompanied by such fee as
may be determined by regulations, from any person, consumer or their
association or trade association; or
(b) a reference made to it by the Central Government or a State Government or a
Statutory Authority.
As per the situation given and provisions of the Act, CCI is empowered to inquire
into any alleged contravention of the provisions contained in Section 3(1) or Section
4(1) on its own motion also. Hence, CCI can also investigate the matter
pertaining to the tender floated by Delta Corporation in March, 2018 (though it
was not the subject matter contained in the complaint submitted by Delta
Corporation on 4 th February, 2018).
(iii) "Bid Rigging" means any agreement, between enterprises or persons engaged in
identical or similar production or trading of goods or provision of services, which has
the effect of eliminating or reducing competition for bids or adversely affecting or
manipulating the process for bidding.
As per the facts of the case study, there seems to be collusive bid rigging by
forming cartel due to the following reasons:
1. All the parties (namely M/s Easy, M/s Samurai, M/s Multicrop and M/s Agro
Chemicals) quoted identical rates from 2009 to 2014.
2. In tender floated in March 2016, the three applicants quoted identical prices.
3. If we see the bidding patterns for other corporations also (i.e. A, B, C and D)
we see that participating applicants quoted identical prices always.
Further, the response given by the parties (namely M/s Easy, M/s Samurai, M/s
Multicrop and M/s Agro Chemicals) did not support that there was no cartelization,
on the following grounds:
1. CCI is empowered to inquire into any alleged contravention of the provisions
contained in Section 3(1) or Section 4(1) on its own motion also. Hence, CCI
can also investigate the matter pertaining to the tender floated by Delta

© The Institute of Chartered Accountants of India


Page 109
PAPER – 6D: ECONOMIC LAWS 9

Corporation in March, 2018 (though it was not the subject matter contained in
the complaint submitted by Delta Corporation on 4 th February, 2018).
2. The said parties pleaded that the price rise of APT was due to increase of
price of yellow phosphorous, which was to be procured from China, had
increased. However, all the parties quoted identical prices which has resulted
in adversely affecting/ manipulating the process of bidding.
1.7 Initiation of Insolvency resolution by PEL (operational creditor) against Multicrop.
According to Section 8 of the IBC, 2016, following requirements are to be met for
initiation of corporate insolvency resolution process by operational creditor, i.e. by PEL
against the corporate debtor, Multicrop:
(1) On the occurrence of default, an operational creditor shall first send a demand
notice and a copy of invoice to the corporate debtor.
(2) The corporate debtor shall, within a period of ten days of the receipt of the
demand notice or copy of the invoice bring to the notice of the operational
creditor about existence of a dispute about debt, if any, record of the pendency of
the suit or arbitration proceedings filed before the receipt of such notice or invoice in
relation to such dispute; Where corporate debtor might have already paid the
unpaid operational debt, there in such situation, corporate debtor will inform within
10 days send an attested copy of the record of electronic transfer of the unpaid
amount from the bank account of the corporate debtor; or sends an attested copy
of record that the operational creditor has encashed a cheque issued by the
corporate debtor. [Section 8]
According to Section 9 of the IBC, an application for initiation of corporate
insolvency resolution process by operational creditor may be filed, if no reply is
received or payment or notice of the dispute under Section 8(2) from the corporate
debtor within ten days from the date of delivery of the notice or invoice demanding
payment, operational creditor can file application before Adjudicating Authority
(NCLT) for initiating a corporate insolvency resolution process.
As per the facts stated in the case study, PEL had not served demand notice and a
copy of invoice to the Multicrop. In fact it directly went to the Arbitral tribunal, for
settlement of the claim as per the term of agreement. Award was passed in the
favour of PEL. However, the award of the Arbitral Tribunal was challenged by the
Multicrop. Whereas PEL also filed an application before the NCLT for initiation of
CIRP against Multicrop.
According to the above provision, due to prima facie non-compliance of serving of
demand notice and a copy of invoice to the Multicrop by the operational creditor

© The Institute of Chartered Accountants of India


Page 110
10 FINAL (NEW) EXAMINATION: NOVEMBER, 2019

(PEL) and of further no notice of dispute about debt regarding the pendency of the
suit in appeal before Appellate Arbitration by the Corporate Debtor (Multicrop).
Therefore NCLT stand as regard the admission of application of PEL on
initiation of CIRP against Multicrop, is not appropriate.

CASE STUDY - 2
Teddy Bear Technology Private Limited (TBTPL), is one of India's fastest growing start-up
companies. TBTPL was incorporated in the year 2015 by two promoters Mr. Sudhir Shankar
and Mr. Ajay Vinod, who were college mates at IIT Bombay and completed their masters in the
United States of America (USA). Both Mr. Sudhir Shankar and Mr. Ajay Vinod worked in the
USA for more than 10 years.
Post that they came back to India in 2015 (and continue to stay in India) to serve the country
and established TBTPL to develop technology and software relating to aviation technology
and machine learning. TBTPL has around 300 employees in India and has several clientele in
US and the company is also looking at rapid expansion over the next 3 years. The Company is
registered with the Software Technology Parks but is not a status holder exporter.
The details of export sales and realization of export proceeds by TBTPL during the last 3
financial years is as under :
Particulars 2015-16 2016-17 2017-18 Average
Export Turnover (USO) 500,000 2,500,000 4,500,000 2,500,000
Realisation of Export Proceeds (USD) 300,000 2,000,000 3,000,000 1,600,000
One of the export invoices amounting to USD 200,000 raised by TBTPL in the financial year
2016-17 was outstanding for more than one year as of 31 st March, 2018 and the Company's

auditors insisted on the Company taking action for recovery. However, even after the best
efforts, no amounts could be recovered and therefore, during the financial year 2018-19, the
Company wrote off the entire amount of USD 200,000 without obtaining the approval from the
Authorised Dealer (AD). Out of the export proceeds received by TBTPL, the Company lent an
amount of USD 500,000 in foreign currency to one of its key Indian vendors to enable them to
create / maintain core working capital. The Management convinced the Board of Directors to
approve the loan since the vendor was providing, critical services for business continuity of
TBTPL. Further, this loan has been guaranteed by the holding company of the vendor, which
is located in Mauritius.
In order to expand its operations, TBTPL was intending to lease a commercial property in
India in Mumbai for a period of 5 years at an upfront lease premium of `5 crores, TBTPL was
in great urgency to complete the transaction soonest in view of the great demand for the
property and therefore, M/s. DoCorrect Consultants, the agency assisting TBTPL used a
counterfeit government stamp paper for the purpose of registering the lease deed and this was
informed by the agency to Mr. Ajay Vinod at the time of transaction to minimise the cost of

© The Institute of Chartered Accountants of India


Page 111
PAPER – 6D: ECONOMIC LAWS 11

stamp duty. The funds for acquiring the stamp papers was paid by the agency and was in-turn
billed by the agency on TBTPL as part of its invoice for agency fee / commission. The invoice
was settled by TBTPL to the agency in cash without deduction of tax, even though the CFO of
TBTPL was of the view that the same is not in accordance with the applicable statutory
requirements.
For the purpose of enhancing its capabilities, TBTPL engaged the services of two reputed
organizations to train the employees of TBTPL. For this purpose, TBTPL paid an amount of
USD 500,000 to one company and USD 1,500,000 to the second company. For the purpose of
investing money into the business, TBTPL sold a commercial plot owned by it in India to a
friend of Mr. Ajay Vinod who was a Non-resident Indian in the USA, through an agent based in
Chicago, USA for an amount of USD 500,000. In accordance with the terms of the agreement
with the agent, TBTPL paid an amount of USD 30,000 as commission to the agent. TBTPL
also published an advertisement costing USD 100,000 in the New York Times weekend
edition calling for employees to join its proposed office in New York.
Mr. Siddarth Shankar, brother of Sudhir Shankar who works as a CFO in a listed entity in
India, provided certain price sensitive information to Mr. Sudhir Shankar about his employer
based on which Mr. Sudhir Shankar purchased equity shares of the entity and made a profit of
`2 crores. With these proceeds, he sent `1 crore to his wife Ms. Anne Shankar (as part of
the liberalised remittance scheme) to purchase a small apartment in the USA. He also
purchased a very old statue of an Indian king in an amount of `0.20 crores and sent it to his
wife for display in his home in USA. He invested the balance amount of ` 0.80 crores in
TBTPL as an equity investment.
During one of the discussions with the customers in USA, Mr. Ajay Vinod indicated to the
customer that TBTPL has capabilities to develop new robotic technology on aviation and
accordingly, entered into a contract for an amount of USD 2,000,000. TBTPL developed the
robotic platform in 2 months and delivered to the customer, although the patent and copyright
was owned by another competitor of TBTPL. TBTPL is of the view that the company rightfully
owns the patent for the same, although it has not applied / registered for the same.
The Enforcement Directorate (ED) got wind of the transactions carried out by TBTPL and the
Directors, through one of the employees of the Company and have issued a notice to the
Company and the Directors.
Answer the following questions:
2.1 Which of the following are not actions that could be taken by the ED on TBTPL or its
employees, for not complying with its orders under PMLA, 2002?
(A) Issue a warning in writing.
(B) Direct the entity or its employees to directly send reports.
(C) Direct the relevant courts to take civil or criminal proceedings against TBTPL or its
employees.

© The Institute of Chartered Accountants of India


Page 112
12 FINAL (NEW) EXAMINATION: NOVEMBER, 2019

(D) Impose a monetary penalty on TBTPL or its employees. (2 Marks)


2.2 In order to obtain more information from Mr. Sudhir Shankar, the ED wanted to detain Mr.
Sudhir Shankar for a period of 3 days to make enquiries and get the relevant information
from him. Evaluate if this is appropriate under PMLA, 2002.
(A) Yes, the Director is well within his powers to detain Sudhir until all informations are
collected.
(B) No, maximum period of detention under PMLA is 24 hours before which Sudhir
should be presented before the superior ranking office or the magistrate.
(C) Yes, however, the Director is required to take the prior approval of his superior
ranking officer.
(D) No, the Director is not within his rights to detain Sudhir. (2 Marks)
2.3 The Appellate Tribunal has concluded that the Director who searched Mr. Sudhir Shankar
and his property indulged in a vexatious search without recording proper reasons in
writing and has sought your views on the next course of action :
(A) Suspension / Dismissal from service, as may be decided by the central government.
(B) Fine which may extend to `2 lakhs.
(C) Imprisonment for a term which may extend to four years and fine which may extend
to `2 lakhs.
(D) Imprisonment for a term which may extend to two years or fine which may extend to
`50,000 or both. (2 Marks)
2.4 What is the maximum amount of export receivables which can be written off by TBTPL
during the financial year 2018-19?
(A) With approval of AD - USD 450,000; Without approval of AD – USD 225,000
(B) With approval of AD - USD 250,000; Without approval of AD – USD 125,000
(C) With approval of AD - USD 300,000; Without approval of AD – USD 150,000
(D) With approval of AD - USD 160,000; Without approval of AD – USD 80,000
(2 Marks)
2.5 Under FEMA, 1999, what is the amount that can be paid by TBTPL for publishing an
advertisement in New York Times ?
(A) USD 10,000
(B) USD 100,000

© The Institute of Chartered Accountants of India


Page 113
PAPER – 6D: ECONOMIC LAWS 13

(C) USD 250,000, subject to the approval of the Reserve Bank of India.
(D) None, all such transactions require approval of the government of India. (2 Marks)
2.6 Advise the Board of Directors of TBTPL on the compliance with FEMA, 1999 with regard
to the below transactions :
a. Payments made by TBTPL for consultancy services
b. Payment of commission
c. Loan provided in foreign currency to vendor in India and the validity of the
guarantee provided by the vendor's holding company. (7 Marks)
2. 7 Examine / advise regarding the below questions relating to the Prevention of Money
Laundering Act, 2002 :
(i) The Enforcement Directorate has sought your advice on identifying all the offences
committed by the parties under the PMLA, 2002 described in the case study.
Identify :
(a) the offences along with explanations,
(b) the parties involved, and
(c) the proceeds of crime. (5 Marks)
(ii) The Enforcement Directorate is proposing to perform a search of M/s. DoCorrect
Consultants premises in connection with the investigation of TBTPL's transactions.
This has been challenged by M/s DoCorrect consultants. Evaluate the
appropriateness of the position taken by M/s. DoCorrect Consultants. (3 Marks)
ANSWER TO CASE STUDY 2
2.1 (C)
2.2 (B)
2.3 (D)
2.4 (C)
2.5 (D)
Descriptive Answers
2.6 As per Schedule III, the following remittances by persons other than individuals shall
require prior approval of the Reserve Bank of India:

© The Institute of Chartered Accountants of India


Page 114
14 FINAL (NEW) EXAMINATION: NOVEMBER, 2019

(i) Commission, per transaction, to agents abroad for sale of residential flats or
commercial plots in India exceeding USD 25,000 or five percent of the inward
remittance whichever is more.
(ii) Remittances exceeding USD 1,00,00,000 per project for any consultancy services in
respect of infrastructure projects and USD 10,00,000 per project, for other
consultancy services procured from outside India.
a. TBTPL made a payment of USD 500,000 to one Company and USD 1,500,000
to another Company for training the employees of TBTPL. Thus, in total, made
a payment of USD 2,000,000.
As per the provision of law and facts of case study, TBTPL require prior
approval of the Reserve Bank of India to make a payment of USD 200,000 as
it exceeds the limit of USD 1,000,000 given under law.
Alternate Solution
a. As per Schedule III, the following remittances by persons other than
individuals shall require prior approval of the Reserve Bank of India:
(i) Commission, per transaction, to agents abroad for sale of residential flats
or commercial plots in India exceeding USD 25,000 or five percent of the
inward remittance whichever is more.
(ii) Remittances exceeding USD 10,000,000 per project for any consultancy
services in respect of infrastructure projects and USD 1,000,000 per
project, for other consultancy services procured from outside India.
TBTPL made a payment of USD 500,000 to one Company and USD 1,500,000
to another Company for training the employees of TBTPL.
Therefore, the prior approval of the RBI is required for the payment of USD
1,500,000 to the second Company. No specific approval of the RBI is required
for the payment of USD 500,000 to the first Company.
b. TBTPL made a payment of USD 30,000 as commission to agent abroad for
selling a commercial plot owned by it in India to a Non- resident Indian in USA.
As per facts of case and provision of law, TBTPL can make a remittance of
USD 25,000 or five percent of the inward remittance from sale of commercial
plot, without RBI approval.
Thus, TBTPL have to take prior approval of RBI to make a payment of USD
30,000 as commission to agent abroad (as it exceeds the limit of USD 25,000
or 5% of USD 500,000, whichever is higher).
c. As per FEMA provisions, a resident cannot lend to another resident in foreign
currency. However, Loan and guarantee can be extended to an overseas

© The Institute of Chartered Accountants of India


Page 115
PAPER – 6D: ECONOMIC LAWS 15

entity only if there is already an existing equity / CCPS ( Compulsorily


Convertible Preference Shares) participation by way of direct investment.
In the given case study, TBTPL lent an amount of USD 500,000 in foreign
currency to one of its vendor. This loan was guaranteed by the holding
Company of the vendor, which is located in Mauritius.
As per the facts of the case study and the provision enumerated above,
TBTPL cannot give loan to its vendor.
2.7 (i) In the given case study, Enforcement Directorate identified following offences
committed by the parties under the PMLA, 2002-
(a) Offences with Explanation: (1) Use of counterfeit government stamp paper
for the purpose of registering the lease deed to minimise the cost of stamp
duty- offence under Part A of the Schedule , (2) invoice for agency fees
/commission for acquiring the stamp papers, settled in cash without deduction
of tax by TBTPL- Offence under Part C of the Schedule (3) Use of patent and
copyright owned by another competitor of TBTPL -- offence under Part A of the
Schedule (4) Providing of price sensitive information to Mr. Sudhir Shankar of
an employee on the basis of which he purchased equity shares of the entity-
offence under Part A of the Schedule (5) sending to ` 1 Crore out of proceeds
from purchase of an equity shares to Ms Anne Shankar- offence under Part A
of the Schedule.
(b) Parties Involved: Offence pertaining to use of counterfeit government stamp
paper- TBTPL, Mr. Ajay Vinod, Agency M/s DoCorrect Consultants, CFO of
TBTPL.
Invoice for agency fees /commission for acquiring the stamp papers- Agency
M/s DoCorrect Consultants, TBTPL, CFO of TBTPL.
Use of patent and copyright owned by another competitor by TBTPL: TBTPL,
Mr. Ajay Vinod.
Purchase of equity shares of an entity on the price sensitive information: Mr.
Siddarth Shanker & Mr. Sudhir Shankar
Out of proceeds obtained above, sent certain amount to Ms. Anne- Mr.
Siddarth Shanker, Mr. Sudhir Shankar, Ms. Anne Shanker.
(c) Proceeds of Crime: ` 5 Crore (Lease premium), USD 2,000,000 (for
development of Robotic Platform under the patent & copyright owned by
another), & ` 2 Crore (obtained by the purchase of equity shares).
(ii) According to Section 17 of the Prevention of Money Laundering Act, 2002, where
the Director or any other officer not below the rank of Deputy Director authorised by
him for the purposes of this Section, on the basis of information in his possession,

© The Institute of Chartered Accountants of India


Page 116
16 FINAL (NEW) EXAMINATION: NOVEMBER, 2019

has reason to believe (the reason for such belief to be recorded in writing) that any
person—
(i) has committed any act which constitutes money-laundering, or
(ii) is in possession of any proceeds of crime involved in money-laundering, or
(iii) is in possession of any records relating to money-laundering, or
(iv) is in possession of any property related to crime,
then, subject to the rules made in this behalf, he may authorise any officer
subordinate to him to—
enter and search any building, place, vessel, vehicle or aircraft where he has
reason to suspect that such records or proceeds of crime are kept;
Thus, the Enforcement Directorate can perform a search of M/s DoCorrect
Consultants’ premises in connection with the investigation of TBTPL’s connection.
Hence, the position taken by M/s DoCorrect Consultants is not appropriate
based on the above legal provisions.

CASE STUDY: 3
The Indian pharmaceutical manufacturing industry comprises of 3 large companies, LPPL,
SMCL and HLL. The above 3 companies, in total supply more than 90% of the across the
counter medicine market in India and their products were available across India through the
sale of medicines to registered agencies / stockists, who in turn supplied to the local chemists
and drugstores. In addition to the business of manufacturing across the counter medicines, all
the 3 entities were also engaged in the manufacture of 'Active Product Ingredients' (API),
which were supplied to global pharmaceutical companies for production of medicines. The
entire API manufacturing in India is performed only by the 3 companies.
During one of the discussions between LPPL and its overseas customer based in Canada, the
overseas customer requested LPPL to supply API for manufacturing diabetes medicines and
also stated that as per the latest research carried out by them, coca leaves have a lot of
medicinal properties and have tremendous potential to supress diabetes and other ailments.
LPPL stated that they could supply coca leaves from India and pursuant to a purchase order
from the customer, LPPL sold coca leaves for an amount of Rs. 5 crores and the CFO of LPPL
ensured that the proceeds was received from the customer into LPPL’s EEFC account in
compliance with FEMA, 1999. For the purpose of increasing their operations in Canada, LPPL
wanted to set up its branch office in Canada and accordingly, used the consideration received
for acquiring Land and Building in Toronto, Canada for an amount of `4 crores. The CFO of
LPPL was informed by the internal auditor that the above acquisition of immovable property in
Canada was in accordance with the provisions of FEMA, 1999.
During the year 2017, the Pharmaceutical Agents Association of Uttar Pradesh filed a
complaint against the 3 companies with the Director General that the companies were

© The Institute of Chartered Accountants of India


Page 117
PAPER – 6D: ECONOMIC LAWS 17

engaging in anti-competitive market activities by forcing stockists to obtain a Non-Objection


Certificate from the local chemists and druggists association and the companies were denying
the supply of medicines to the stockists solely because they were not able to obtain the NOC.
LPPL, SMCL and HLL responded to the DG that sub-clause (a) of Clause 28 of the Drugs
(Price Control) Order, 2013 creates an obligation on a pharmaceutical company/distributor to
sell drugs/medicines unless there is a ‘good and sufficient reason' to refuse sale. Based on
their evaluation of the facts and circumstances, the non-availability of NOC from the local
chemists association tantamount to 'good and sufficient reason’.
Based on the investigation carried out by the DG and analysis of all the documents and
information provided by the Pharmaceutical manufacturing companies, the stockists etc. and
notwithstanding the above views of the pharmaceutical manufacturers, the DG concluded that
the 3 companies, LPPL. SMCL and HLL contravened the provisions of Section 3(3)(b) read
with Section 3(1) of the Competition Act, 2002. For indulging in anti-competitive practices in
violation of the provisions of Section 3 of the Act, the CCI imposed penalties upon all the three
appellants at 9% of average 3 years’ total turnover of these appellants under the Act.
LPPL, SMCL and H LL accepted the order of the DG in principle and accepted to remove, the
condition of obtaining NOC for supply to the stockists. However, they contested the manner in
which the DG had computed the penalty by considering the total turnover of the entities (as
per the Statement of Profit and Loss) without considering that the turnover includes incomes
from the API business, which is not forming part of the investigation of the DG. They filed an
appeal before the Appellate Tribunal that the penalty could be calculated only based on the
turnover relating to the "Across the Counter" operations of the pharmaceutical companies.
In the meantime during the year 2018, LPPL entered into an agreement with HLL to acquire
the API business of HLL for a consideration of ` 200 Crores. The latest available financial
information relating to the entities are as under:
`in Crores
Particulars LPPL HLL
Total entity API business Total Entity API business
Assets 900 800 500 300
Turnover 2800 2400 1000 800
Note: The entities do not have any business / operations outside India.
SMCL is of the view that the above arrangement will cause an appreciable adverse effect on
competition in the API manufacturing market in India and requires the approval of the
Competition Commission.
The Authorised dealer, when reviewing the export invoices raised by LPPL noted the sale of
coca leaves and informed the income tax authorities regarding the same. The authorities, after
review of the documents and other information, concluded that the transactions was in

© The Institute of Chartered Accountants of India


Page 118
18 FINAL (NEW) EXAMINATION: NOVEMBER, 2019

violation of the Prevention of Money Laundering Act, 2002 and have sent a notice to LPPL,
who is not a willful defaulter.
Answer the following questions:
3.1 Which of the following terms and conditions as per the agreement between LPPL and
HLL is not likely to cause an appreciable adverse effect on competition under the
Competition Act, 2002?
(A) All purchase of raw materials by HLL should be made from SMCL or from LPPL
only.
(B) The API manufactured by HLL should be sold to the customers as mandated by
LPPL.
(C) Any purchase. of API by HLL should be along with purchase of the packing material
and preservatives.
(D) A maximum price ceiling on the resale price that may be charged by HLL for
ultimate sale of the goods purchased by it from LPPL. (2 Marks)
3.2 Considering the nature of the operations of LPPL and HLL, what is the requirement of
giving notice regarding the· proposed combination as per Form Il as specified in the
Schedule II to the Competition Commission of India (Procedure in regard to the
transaction of business relating to combinations) Regulations, 2011 ?
(A) Mandatory, if the combined market share after such combination is more than 15 %
of the market.
(B) Optional, 'if the combined market share after such combination is more than 25% of
the market.
(C) Optional, if the combined market share after such combination is more than 15% of
the market.
(D) Mandatory, if the combined market share after such combination is more than 25%
of the market. (2 Marks)
3.3 Which of the following are not included within arrangements entered into by central
government with another country, in relation to reciprocal arrangements under PMLA,
2002?
(A) Enforcement of the provisions of PMLA, 2002.
(B) Prevention of offence in India under the corresponding PMLA law in force in the
other country.
(C) Exchange the history of LPPL if it is wilful offenders under the PMLA on annual
basis.
(D) Exchange information to prevent any offence under PMLA, 2002. (2 Marks)

© The Institute of Chartered Accountants of India


Page 119
PAPER – 6D: ECONOMIC LAWS 19

3.4 The composition of an Adjudicating Authority (AO) under the PMLA, 2002 referred in the
case study is :
(A) One Chairperson, appointed by central government and two other members.
(B) Three members, one of whom will be a Chairperson, as per seniority.
(C) Four members, each of whom will be a Chairperson on rotation.
(D) Five members, appointed by central government and four other members. (2 Marks)
3.5 On the basis that the transactions entered into by LPPL is considered to be in
contravention of the PMLA, 2002, what is the punishment that the CFO of LPPL would be
liable under the PMLA, 2002 ?
(A) Minimum 3 years and maximum 10 years with fine.
(B) No punishment since he is not a director of LPPL and therefore cannot be held
liable under PMLA, 2002.
(C) Minimum 3 years and maximum 7 years with fine.
(D) No punishment since he was not aware that the transaction was indeed a non-
compliance under PMLA, 2002. (2 Marks)
3.6 Answer the following questions in the context of the provision relating to Competition Act,
2002 with reasons and explanations:
(i) SMCL has reached out to you to seek your advice on their views regarding the
impact of the provisions of the Competition Act on the proposed combination
between LPPL and HLL. (4 Marks)
(ii) Whether penalty under Section 27(b) of the Act has to be on total/entire turnover of
the offending company or it can be only on "relevant turnover", i.e., relating to the
product in question. (3 Marks)
3. 7 Answer the following questions in the context of the provisions relating to PMLA, 2002
with reasons and explanations:
(i) LPPL has challenged the notice and without admitting to any of the offences, is of
the view that only immovable property held within India is to be considered for
identifying proceeds of crime under PMLA. Evaluate. (4 Marks)
(ii) In the above case study, what is the mechanism to be followed by the Enforcement
Directorate for attachment of property situated in Canada? (4 Marks)
ANSWER TO CASE STUDY 3
3.1 (D)
3.2 (C)
3.3 (C)

© The Institute of Chartered Accountants of India


Page 120
20 FINAL (NEW) EXAMINATION: NOVEMBER, 2019

3.4 (A)
3.5 (A)
Descriptive answers
3.6 (i) The given proposed combination between LPPL & HLL in terms of Section 5 of the
Competition Act, 2002, is a combination of the enterprises by acquisition where the
parties to the acquisition, being the acquirer and the enterprise, whose control,
shares, voting rights or assets have been acquired or are being acquired jointly
have in India, the assets of the value of more than rupees one thousand crores or
turnover more than rupees three thousand crores.
Pursuant to Notification No. S.O. 675 (E) dated March 4, 2016 the value of
assets and the value of turnover has been enhanced by the Central Government by
100% for the purposes of Section 5 of the Act.
So, the revised value of assets and turnover is presently more than ` 2000 crore
and ` 6000 Crore.
Since, here the proposed combination between LPPL and HLL was to acquire the
API business of HLL only, therefore, it will not be valid as they have not met with
the requirement of assets of the value of more than ` 2000 crore [i.e., total value of
asset of LPPL (900+800) + value of asset of API business of HLL (300)] and
turnover of ` 6000 crore [i.e., total turnover of LPPL (2800+2400) + turnover of API
business of HLL (800)]
(ii) As per Section 27 of the Competition Act, 2002, where after inquiry the Commission
finds that any agreement referred to in Section 3 or action of an enterprise in a
dominant position, is in contravention of Section 3 or Section 4, as the case may be,
it may impose such penalty, as it may deem fit, which shall be not more than ten per
cent of the average of the turnover for the last three preceding financial years, upon
each of such person or enterprises which are parties to such agreements or abuse.
In case any agreement referred to in Section 3 has been entered into by a cartel,
the Commission may impose upon each producer, seller, distributor, trader or
service provider included in that cartel, a penalty of up to three times of its profit for
each year of the continuance of such agreement or ten per cent of its turnover for
each year of the continuance of such agreement, whichever is higher.
Accordingly, the penalty under Section 27(b) of the Act has to be on total/
entire turnover of the offending Company.
3.7 (i) In the light of Section 2(1)(u) of the Prevention of Money Laundering Act, 2002,
"proceeds of crime" means as any property derived or obtained, directly or

© The Institute of Chartered Accountants of India


Page 121
PAPER – 6D: ECONOMIC LAWS 21

indirectly, by any person as a result of criminal activity relating to a scheduled


offence or the value of any such property or where such property is taken/held
outside the country, then the property equivalent in value held within the country or
abroad.
In the said case, LPPL challenged the notice and not admitting to any of the
offences on the ground that only immovable property held within India is to consider
for identifying proceeds of Crime under PMLA. According to the above stated
provision, LPPL challenge to the notice and not admitting to any of the offences
pertaining to the immovable property held outside India, is not valid and therefore
the notice served on LPPL cannot be challenged.
(ii) Following are the ways for attachment of property situated in Canada in the given
case study in the light of Section 60 of the PMLA, 2002 -
1. Issue of letter of request: Where the Director has made an order for
attachment of any property under Section 5 or for freezing under sub-Section
(1A) of Section 17 or where an Adjudicating Authority has made an order
relating to a property under Section 8 or where a Special Court has made an
order of confiscation relating to a property under sub-Section (5) or sub
Section (6) of Section 8, and such property is suspected to be in a
contracting State,
- the Special Court, on an application by the Director or the Administrator
appointed under sub- Section 10(1), as the case may be, may issue a letter of
request to a court or an authority in the contracting State for execution of such
order.
2. Forwarding of letter of request for execution on its receipt by CG : Where
a letter of request is received by the Central Government from a Court or an
Authority in a Contracting State requesting attachment, seizure, freezing or
confiscation of the property in India, derived or obtained, directly or indirectly,
by any person from the commission of an offence under a corresponding law
committed in that Contracting State, the Central Government may forward
such letter of request to the Director, as it thinks fit, for execution in
accordance with the provisions of this Act.
3. Issue of Order of confiscation: Where on closure of the criminal case or
conclusion of trial in a criminal court outside India under the corresponding law
of any other country, such court finds that the offence of money-laundering
under the corresponding law of that country has been committed, the Special
Court shall, on receipt of an application from the Director for execution of
confiscation under sub-Section (2), order, after giving notice to the affected
persons, that such property involved in money-laundering or which has been
used for commission of the offence of money-laundering stand confiscated to

© The Institute of Chartered Accountants of India


Page 122
22 FINAL (NEW) EXAMINATION: NOVEMBER, 2019

the Central Government.


4. The provisions of this Act relating to attachment, adjudication, confiscation and
vesting of property in the Central Government shall apply to the property in
respect of which letter of request is received from a court or contracting State
for attachment or confiscation of property.
CASE STUDY - 4
Highcity Partners LLP (Highcity), is a recently established limited liability partnership between
Seaview Constructions Private Limited, a real 'estate development company owned by Mr.
Vyas Chakraborty (Seaview constructions) and Mr. Ved Chakraborty. Highcity was established
for the purpose of acquiring an existing apartment complex "Riverview Bliss" (comprising of 12
luxury apartments) in Kolkata and redevelopment of the same. Seaview Constructions is a
very successful real estate company and has completed more than 20 apartment complexes
and is known for quality constructions, adherence to timelines and profitable growth.
6 of the 12 apartments in Riverview Bliss is currently owned by SPZ Private Limited (SPZ) and
the balance 6 are owned by the senior employees of SPZ. Due to the strategic location of the
property and the quality of construction, Highcity and the current owners have agreed for a
price of `3 crores for each of the 12 apartments and therefore the total consideration to be
paid by Highcity is `36 crores.
SPZ is an associate Company of True & Fair Finance Company Limited (TFFC), a listed
company in the business of providing loans for large corporate projects. Both SPZ Private
Limited and TFFC have common promoters and senior employees and operate out of the
same registered office.
In the past, S eaview Constructions has obtained loans from TFFC for many of their projects
and has established a strong professional relationship with them on account of the mutual
benefit realised by both the entities from the transactions between them. Therefore,
considering the size of the transaction to be entered into by Highcity, Mr. Vyas Chakraborty
had discussions with TFFC and based on the business case submitted by Highcity, TFFC
approved a secured loan of `30 crores to Highcity to enable purchase of the apartments in
Riverview Bliss from its owners. The loan was fully utilised by Highcity to acquire the
apartments and a charge was created against the property for the secured loan obtained from
TFFC. Highcity obtained further loans amounting to `10 crores from SPZ for the purpose of
the redevelopment of the property.
During the scrutiny assessment of Highcity, the Income tax authorities noted the details of the
transactions and concluded that the entire transaction is a benami transaction where Highcity
is the Benamidar and SPZ / TFFC are the beneficial owners. The Initiating Officer sent a show
cause notice under Section 24(1) of the Prohibition of Benami Property Transactions Act, 1988
(PBPT Act, 1988) and on the same day, an order was passed by the Dy. Commissioner of
Income Tax for provisional attachment of the Riverview Bliss property based on the following
averments :

© The Institute of Chartered Accountants of India


Page 123
PAPER – 6D: ECONOMIC LAWS 23

• Highcity did not have any business or operations prior to the acquisition of the benami
property.
• Mere approvals in the name of benamidar do not prove in any way that the benefits from
the property are actually enjoyed by it and not by the beneficial owner.
• Highcity received huge amounts of money from SPZ which it used for the development of
property, thereby establishing that SPZ is directly involved in the development of project
in order to derive future benefits arising out of the same.
• The entire transaction is only for the benefit of TFFC and SPZ, who are owned by
common promoters since the person providing the consideration i.e. TFFC and person
reaping the benefits of such transaction i.e. SPZ are same as they are linked to each by
means of common directors and promoters.
• The benefits to the beneficial owner arising out of property held in the name of the
benamidar need not be direct and immediate and· that indirect and future benefits are
also covered under the definition of a benami transaction under section 2(9)(A) of the
PBPT Act, 1988.
The Initiating Officer further stated in his show cause notice seeking response and proof from
Highcity and SPZ that the above transactions are not benami transactions. Highcity is of the
strong view that the above averments are incorrect and that the entire transaction is a genuine
business transaction and the loan from TFFC was obtained in the ordinary course of business
(similar to the other loans taken by Seaview Constructions).
Seaview Constructions was operating as a profit making company until 2016 and whilst it was
having debt, the entity was able to service the debt promptly from its business cash flows.
However, due to the downturn of the real estate industry and commencement of additional
businesses, Seaview Construction's profits and operations started to deteriorate and it had to
obtain significant borrowings during 2017 from a consortium of banks for working capital
purposes. However, due to the difficulties in the business operations and the economic
slowdown, Seaview Constructions could not repay its borrowings and the entire net worth got
eroded due to significant operating losses. This led to Seaview Constructions filing a petition
under the Insolvency and Bankruptcy Code, 2016. The petition was accepted by the National
Company Law Tribunal (NCLT) and an Interim Resolution Professional (IRP) was appointed,
who was later approved as the Resolution Professional (RP). The Committee of Creditors,
comprising of the financial creditors was formed with the following vote share:
Particulars Voting Share (%)
A Bank 22.33%
B Bank 14.39%
C Bank 15.15%.

© The Institute of Chartered Accountants of India


Page 124
24 FINAL (NEW) EXAMINATION: NOVEMBER, 2019

D Bank 26.36%
E Bank 10.94%
F Bank 10.83%
The resolution plan submitted by the RP was placed before the Committee of Creditors at its
meeting held on 4 th December, 2018 wherein, the resolution plan was approved by A Bank, B
Bank and C Bank. D Bank rejected the resolution plan and provided its reasons in writing to
the RP. E Bank and F Bank did not approve or reject the proposal and abstained from voting
at the meeting. Seaview Constructions (the Corporate Debtor) is of the view that the resolution
plan has been approved by the Committee of Creditors since the resolution plan has been
approved by more than the prescribed percentage of creditors who actually voted in the
meeting (i.e. after excluding the percentage relating to the creditors who abstained). The RP
did not agree to this view since more than 25% of the creditors present in the meeting had out
rightly rejected the resolution plan and therefore, proceeded for liquidation under the IBC
since no resolution plan was approved within the prescribed time limit under the Code.
M/s. Sunflower Estates Private Limited (Sunflower Estates), a Company under the common
control of the promoter of Seaview Constructions had also subscribed to the secured
debentures of Seaview Constructions to the extent of ` 50 Crores (representing 15% of the
total financial debts of Seaview Constructions). The IRP rejected the request received from
Sunflower Estates for inclusion into the Committee of Creditors.
Answer the following questions:
4.1 The owner (one of the employees of SPZ) of one of the apartments in Riverview Bliss is
not aware of his ownership of the apartment. He is seeking your advice on the impact on
the same under PBPT Act, 1988.
(A) No impact, since the property has already been sold off to Highcity.
(B) The property is not a benami property since the employee had continuous
possession of the property through the period he was the owner.
(C) The property is not a benami property since the sale agreement was registered
appropriately and stamp duty was also paid.
(D) The property is a benami property since the owner of the property is not aware of
such ownership. (2 Marks)
4.2 Mr. Vyas Chakraborty is of the view that the Initiating Authority does not have the right to
send the notice for attachment of the property and those powers are vested with the
adjudicating authority, as per PBPT, 1988 and seeks your advice :
(A) Yes. Initiating Authority has only powers to summon and conduct inquiries.

© The Institute of Chartered Accountants of India


Page 125
PAPER – 6D: ECONOMIC LAWS 25

(B) No. The adjudicating authority's function is to confiscate and vest the property. The.
Initiating Officer has powers to send the notice for attachment of property.
(C) No. The approving authority has to send the notice for attachment of property and
the adjudicating authority is required to confiscate and vest the property.
(D) Yes. The initiating authority can provisionally attach properties only with the prior
approval of the adjudicating authority. (2 Marks)
4.3 Assuming that the Riverview Bliss property is considered as a benami property, the
Initiating Officer seeks your views on whether the rental income earned by Highcity from
the lease of the apartment (pending commencement of redevelopment) is also a benami
transaction.
(A) No, the rental income is an independent transaction between a landlord and a
tenant for legitimate use of the property.
(B) No, as long as Highcity remits Income tax on the rental income earned.
(C) Yes, benami transaction includes any income or proceeds received or earned out of
a benami property,
(D) Yes, if the proceeds from the rental income are used by Highcity for making interest
payment or loan repayment to TFFC or SPZ. (2 Marks)
4.4 How should the voting share of each of the Banks who have lent to Seaview
Constructions be determined under IBC, 2016 ?
(A) Based on the financial debt owed by Seaview Constructions to each bank as a
proportion to the total debt (financial + operational) owed by Seaview Constructions.
(B) Based on the financial debt owed by Seaview Constructions to each bank as a
proportion to the total financial debt owed by Seaview Constructions to third parties
(i.e. other than related parties).
(C) Based on· the financial debt owed by Seaview Constructions to each bank as a
proportion to the total financial debt owed by Seaview Constructions.
(D) Based on the financial debt owed by Seaview Constructions to each bank as a
proportion to the total financial debt and statutory dues owed by Seaview
Constructions. (2 Marks)
4.5 Which of the following operational creditors of Seaview Constructions are eligible to
initiate corporate insolvency process against Seaview Constructions ?
(A) G Limited, completed a corporate insolvency resolution process 15 months prior to
the date of making the application.

© The Institute of Chartered Accountants of India


Page 126
26 FINAL (NEW) EXAMINATION: NOVEMBER, 2019

(B) H Limited, who is currently undergoing a insolvency resolution process.


(C) I Limited, who could not meet its resolution plan under a insolvency resolution
process.
(D) J Limited, who supplied goods to ACL one month prior to the date of making the
application and the invoice demanding payment is in transit. (2 Marks)
4.6 Answer the following questions in the context of the provision relating to PBPT Act, 1988
with reasons and explanations :
(i) Analyse based on the facts of the case provided above, whether the Initiating
Officer's actions were appropriate in concluding that the transaction was a benami
transaction. (3 Marks)
(ii) What are the factors that will need to be considered for the purpose of determining
whether a. transaction is a benami transaction ? (4 Marks)
4.7 Answer the following questions in the context of the provision relating to IBC, 2016 with
reasons and explanations :
(i) Examine the appropriateness of the approval or otherwise of the resolution plan of
Seaview Constructions and whether the view taken by the RP is appropriate.
(5 Marks)
(ii) Advise Sunflower Estates with regard to the rejection of the request for inclusion
into the Committee of Creditors of Seaview Constructions. (3 Marks)
ANSWER TO CASE STUDY 4
4.1 (D)
4.2 (B)
4.3 (C)
4.4 (B)
4.5 (A) [In order to drive the answer for initiation of CIRP against Seaview
constructions, word “Operational Creditor” is to be assumed as corporate
person].
Descriptive Answers
4.6 (i) Course of action taken by Initiating Officer under Section 24 of the PBPT Act,
2016: Where the Initiating Officer, on the basis of material in his possession, has
reason to believe that any person is a benamidar in respect of a property, he may,
after recording reasons in writing, issue a notice to the person to show cause within

© The Institute of Chartered Accountants of India


Page 127
PAPER – 6D: ECONOMIC LAWS 27

such time as may be specified in the notice why the property should not be treated
as benami property.
Where the notice specifies any property as being held by a benamidar, a copy of the
notice shall also be issued to the beneficial owner if his identity is known. Where the
Initiating Officer is of the opinion that the person in possession of the property held
benami may alienate the property during the period specified in the notice, he may,
with the previous approval of the Approving Authority, by order in writing, attach
provisionally the property in the manner as prescribed in Rule 4 of the Benami
Transactions Prohibition Rules, 2016, for a period not exceeding ninety days
from the date of issue of notice.
The Initiating Officer, after making such inquires and calling for such reports or
evidence as he deems fit and taking into account all relevant materials, shall, within
a period of ninety days from the date of issue of notice —
(a) where the provisional attachment has been made —
(i) pass an order continuing the provisional attachment of the property with
the prior approval of the Approving Authority, till the passing of the order
by the Adjudicating Authority; or
(ii) revoke the provisional attachment of the property with the prior approval
of the Approving Authority;
(b) where provisional attachment has not been made—
(i) pass an order provisionally attaching the property with the prior approval
of the Approving Authority, till the passing of the order by the Adjudicating
Authority; or
(ii) decide not to attach the property as specified in the notice, with the prior
approval of the Approving Authority.
Where the Initiating Officer passes an order continuing the provisional attachment of
the property or passes an order provisionally attaching the property, he shall, within
fifteen days from the date of the attachment, draw up a statement of the case and
refer it to the Adjudicating Authority under Section 26 of the PBPT Act, 2016.
Yes, the actions taken by the initiating officer, were appropriate in the compliance of
the above stated provisions.
Alternate Solution
For a transaction to be covered under Section 2 (9)(A) of PBPT Act, 1988 the
following two conditions are to be met.

© The Institute of Chartered Accountants of India


Page 128
28 FINAL (NEW) EXAMINATION: NOVEMBER, 2019

(i) The consideration for the property has been provided or paid by another
person and
(ii) The property is held for immediate or future benefit direct or indirect of the
person who provided the consideration
In order to ascertain whether a particular sale is Benami and the apparent
purchaser is not the real owner, the burden lies on the person who sets up the case
and such burden has to strictly discharged based on legal evidence of definite
nature. Therefore, the Initiating Officer (IO) cannot show cause High City and seek
proof as to why the transaction cannot be treated as a Benami transaction.
Therefore, the onus is on the IO to prove, if at all the transaction is a Benami
transaction. Moreover, if it is shown by the parties to the alleged Benami transaction
that such transaction is done through a registered sale deed and valid loan
agreements, the burden of proof would be shifted upon the IO to prove the
transaction as a Benami transaction.
Merely because the source of consideration paid by the alleged benamidar is
funded by way of loan received from a party related with the alleged beneficial
owner, it cannot be ipso facto held that the consideration has been provided by the
alleged beneficial owner and more so when evidences has been brought on record
to show that the aforesaid loan was a genuine transaction and was done at arms
length in the normal course of business. Therefore the IO’s action is not
appropriate in concluding that the transaction was a Benami transaction.
(ii) Factors that will need to be considered for the purpose of determining of a
benami transaction:
As per Section 2(9) of the PBPT Act, 2016 in order to be a "Benami Transaction"
such transaction /arrangement w.r.t a property is to be considered as a benami—
(i) where a property is transferred to, or is held by, a person, and the
consideration for such property has been provided, or paid by, another person;
and
(ii) the property is held for the immediate or future benefit, direct or indirect, of the
person who has provided the consideration,
(iii) a transaction or an arrangement in respect of a property carried out or made in
a fictitious name; or
(iv) a transaction or an arrangement in respect of a property where the owner of
the property is not aware of, or, denies knowledge of, such ownership;
(v) a transaction or an arrangement in respect of a property where the person
providing the consideration is not traceable or is fictitious;

© The Institute of Chartered Accountants of India


Page 129
PAPER – 6D: ECONOMIC LAWS 29

4.7 (i) Procedure of seeking approval of the Resolution plan in the light of Section 30
of the Insolvency and Bankruptcy Code:
(i) Seeking approval of CoC: The resolution professional shall present such
resolution plans to the committee of creditors for its approval by a vote of not
less than sixty-six per cent of voting share of the financial creditors.
(ii) Submission of the Resolution Plan: The resolution professional shall submit
the resolution plan as approved by the committee of creditors to the
Adjudicating Authority. [Section 30]
(iii) Approval of Resolution Plan: If the Adjudicating Authority is satisfied that
the resolution plan as approved by the committee of creditors meets the
requirements as per Section 30(2), it shall by order approve the resolution
plan.
(iv) Rejection of the Resolution Plan: Where the Adjudicating Authority is
satisfied that the resolution plan does not confirm to the above requirements, it
may, by an order, reject the resolution plan.
The resolution applicant shall obtain the necessary approval pursuant to the
resolution plan approved, within a period of one year from the date of approval of
the resolution plan by the Adjudicating Authority or within such period as provided
for in such law, whichever is later.
In the given instance, the resolution plan of Seaview Constructions will not be
passed as it was not approved by a vote of sixty-six per cent of voting share of the
financial creditors. Out of Total six financial creditors, four financial creditors voted
on the resolution plan and two abstained from voting. Further out of 4 Financial
creditors, 3 Financial creditors with the voting share (22.33% + 14.39%+15.15% =
51.87%) approved the Resolution plan. However,1 Financial creditor with voting
share 26.36% voted against the resolution plan. Resolution professional stand
was correct as regarding the filing of liquidation as the resolution plan was not
approved by the CoC with the requisite majority of 66% of voting share.
(ii) When Financial Creditor /authorized representative is not entitled to
participate in the CoC:
As per Section 21 of the IBC, for the Financial Creditor or the authorised
representative of the financial creditor referred to in Section 24(6), 24(6A), or 24(5),
related to the conduct of meetings of creditors, if it is a related party of the
corporate debtor, shall not have any right of representation, participation or voting
in a meeting of the committee of creditors.
Here the rejection of the request to Sunflower Estates, for inclusion into the
committee of creditors of Seaview constructions, is valid.

© The Institute of Chartered Accountants of India


Page 130
30 FINAL (NEW) EXAMINATION: NOVEMBER, 2019

CASE STUDY - 5
Decor Design Constructions Private Limited (Decor Constructions) is a reputed construction
company based in Pune, India and specialises in construction mid-sized apartments
(approximately 20 apartments in each project). Decor Constructions was founded by 2
brothers, Mr. Ravi Rao and Mr. Giri Rao, and are the Directors of Decor Constructions. Mr.
Ravi Rao studied civil engineering in the UK and worked extensively in the UK in various
infrastructure and construction companies before moving back to India to establish Decor
Constructions. During the year 2014, Decor Constructions commenced a new project called as
Decor Dream Home, which comprises of 30 apartments, each having a super built-up area of
1,800 square feet and carpet area of 1,500 square feet. All the 30 apartments were sold by
Decor Constructions within a period of 3 months and they entered into a sale agreement with
the allottees in the month of November, 2014. The following were the key features of the sale
agreement:
• The apartments were sold to the allottees at a square feet rate of `5,000 per square feet
and the total consideration for each of the apartments were calculated based on the
super built-up area.
• The application fee to be paid prior to entering into the sale agreement was fixed as 8%
of the total consideration.
• The entire amount of consideration should be paid by the allottee within 6 months from
the sale agreement, irrespective of the date / stage of completion of the construction.
This is to facilitate the speedy completion of construction. Decor Constructions has
already factored in a discount in the per square feet rate to compensate the allottees for
the upfront payment.
• Free open car parking to the allottees who pay the entire consideration at the time of sale
agreement. For other allottees, the open car parking will be allotted on payment of
`200,000.
• The apartment will be handed over to the allottees within 30 months from the date of the
agreement i.e. by 31 st May, 2017.
All the 30 allottees made the payment to Decor Constructions in accordance with the
agreement (10 of the allottees paid the full amount on the date of the sale agreement thereby
getting a free open car park) and an amount of ` 2,700 lakhs was received by Decor
Constructions. During the month of August 2016, Decor Constructions sent an e-mail to all the
30 allottees that the Promoter has filed the required forms for approval from the Municipal
Corporation for water, sewerage and electricity connections and this is taking substantial time
to complete, which is not in the control of the Promoter and therefore, the date of handing over
will get slightly delayed to 31 st December, 2017. None of the allottees responded to the
communication. In the meanwhile, with the introduction of Maharashtra Real, Estate
(Regulation and Development) Act with effect from 1st May,2017, Decor Constructions

© The Institute of Chartered Accountants of India


Page 131
PAPER – 6D: ECONOMIC LAWS 31

registered the project under the RERA and as part of the registration stated the expected date
of completion as 30th June, 2018.
Although Mr. Ravi Rao has been in India for more than three years, his ultimate aim is to settle
down in Switzerland, which is the home country of his spouse, Ms. Anne Rao. Therefore, Ravi
wanted to buy a colonial villa in Switzerland for an amount of EUR 2 million. Mr. Giri Rao is of
the view that the FEMA rules does not allow Mr. Ravi Rao to invest in immovable property
outside India when he is resident in India.
Ms. Anne Rao (spouse of Mr. Ravi Rao) who is a citizen of USA, wants to purchase an
immovable property (apartment) in India jointly along with Mr. Ravi Rao. For this purpose, Ms.
Anne Rao is proposing to take a housing loan in her personal name from Bank of Bengaluru, a
bank operating in India. However, considering the fact that she is a citizen of USA, the Bank
has included a pre-condition that the loan be guaranteed by Decor Constructions. Based on
such request, Decor Constructions has provided the required guarantee in favour of Bank of
Bengaluru. Ms. Anne Rao is also interested in investing USD 200,000 in a Special Purpose
Vehicle (in the form of an unincorporated joint venture) which is engaging in the business of
providing managed farm to its investees and provide the land after a period of 20 years. Ms.
Anne Rao before attempting further transactions approached the consultant to advise on the
transactions which are not capital account transactions.
In the month of June 2017, Decor Constructions sent another e-mail to the 30 allottees that
the construction of the super structure of Decor Dream Home is almost complete and what is
left is only to complete the interior plastering, flooring, plumbing etc. and this will get
completed by 31 st March, 2018 and the slight extension of the timeline is only on account of
labour shortage at Pune due to the extensive construction spree happening in the city. Decor
Dream Home also suggested to the allottees that they were ready to handover the apartment
in the month of December, 2017 (before receiving the occupancy certificate) to the allottees
for them to get the interior/furnishing work done so that the allottees can occupy the
apartments in March/April, 2018 as soon as occupancy certificate is received.
All the 30 allottees were not happy on account of the further delay in completion and filed a
complaint against Decor Constructions under the Maharashtra RERA provisions. Out of the 30
allottees, 25 allottees sought cancellation of the sale agreement and refund of the amounts
paid by the allottees along with interest at 21% p.a. The balance 5 allottees wanted to be
compensated by Decor Constructions for the delay in completion-but do not want to cancel the
sale agreement.
Decor Constructions has subm itted before the RERA authorities the following:
• Notwithstanding the registration of the project under RERA as per the requirements of
Section 3 of the RERA, the sections relating to compensation for delay etc. do not apply
to the project since the date of commencement of project / date of sale agreement is
prior to the date when RERA came into effect.

© The Institute of Chartered Accountants of India


Page 132
32 FINAL (NEW) EXAMINATION: NOVEMBER, 2019

• Even otherwise, the date of completion stated in the RERA registration is 30th June 2018
and therefore, the date of handover finally indicated allottees is 31 st March 2018, which is
well within the timelines and therefore, there is no non-compliance with the RERA
requirements.
• The Company had already informed the reasons for the delay of the project upto
31 st December, 2017 in August, 2016 itself and there was no response / issue raised by
the allottees at that time. Further, Decor Constructions has also agreed to provide the
apartments for interior work during December, 2017 and therefore, it is effectively agreed
to handover the apartment as per the revised timelines communicated in August, 2016.
• Even presuming the applicability of the RERA provisions, there is no unanimity in the
decisions of the allottees on the way forward (since 25 have opted for cancellation and 5
have opted for compensation) and therefore, this cannot be anyway given effect to under
RERA.
Accordingly, Decor Constructions has submitted that they are not liable for any compensation
to be paid under RERA and have re-iterated that they will handover the apartments to the
allottees by the revised timelines indicated in the e-mail sent in June, 2017.
Answer the following questions:
5.1 What is your view regarding the terms of the agreement relating to the open car
parking arrangement with the allottees ?
(A) Decor Constructions is free to stipulate any terms and conditions in this regard,
since this is a transaction between a willing buyer and a willing seller.
(B) Decor Constructions is required to provide open car parking for all allottees on
equitable terms and there cannot be a discrimination based on payment schedule.
(C) Open parking areas cannot be sold for consideration since they are to be
considered as common area of the Project.
(D) Open parking is part of internal development works and is part of overall project
costs which can be charged by the Promoter equally to all allottees. (2 Marks)
5.2 One of the allottees of Decor Dream Home have-reached out to you for your advice on
whether the 'collection of the entire consideration by Decor Constructions without regard
to the stage of constructions is appropriate.
(A) Appropriate. The terms/timing of payment are governed by the sale agreement
between the promoter and allottee.
(B) Not appropriate. The timing of payment should be in line with the stage wise
completion / construction schedule.

© The Institute of Chartered Accountants of India


Page 133
PAPER – 6D: ECONOMIC LAWS 33

(C) Appropriate, since the necessary discount has already been factored into the
consideration by Decor Constructions.
(D) Appropriate, provided Decor Constructions has obtained the approval of the terms
at the time of registration of the Project under RERA. (2 Marks)
5.3 Advice of the consultant to Ms. Anne Rao for the transaction which do not fall under the
definition of a capital account transaction under FEMA, 2002 will be:
(A) Transactions which alter the assets and liabilities of non-residents in India.
(B) Transactions which alter the assets and liabilities (including contingent liabilities) of
residents outside India.
(C) Transactions relating to transfer of a security by a branch in India of a company
resident outside India.
(D) Transactions which alter the assets and liabilities (including contingent liabilities) of
non-residents in India. (2 Marks)
5.4 Mr. Vishy Rao, brother of Mr. Ravi Rao, is a resident of Singapore and he owns an
immovable property in Chennai which he inherited from his father, who was a resident of
India. Can Mr. Vishy Rao continue to hold the property?
(A) No, he cannot hold transfer or invest in India, since he is resident outside India.
(B) Yes, he can continue to hold in India, since he is a person of Indian Origin and the
property is located in India.
(C) Yes, he can continue to hold the property, since this was inherited from a person
who was resident in India.
(D) Yes, he can continue to hold the property, since his brother (Mr. Ravi Rao) uses the
property whenever he travels to Chennai. (2 Marks)
5.5 Decor Constructions is in the process of entering into certain business transactions with
international agencies and in this context Mr. Girl Rao seeks your views on the maximum
amount that can be paid by Decor Constructions under the Liberalised Remittance
Scheme and how much he can pay in his own individual capacity under the Scheme, per
year ?
(A) Decor Constructions - USD 250,000; Individually - USD 250,000.
(B) Decor Constructions - USD Nil; Individually - USD 250,000.
(C) Decor Constructions - No limit for specified objects; Individually - USD 200,000.

© The Institute of Chartered Accountants of India


Page 134
34 FINAL (NEW) EXAMINATION: NOVEMBER, 2019

(D) Decor Constructions - USD 500,000 (USD 250,000 for each director); Individually -
USD Nil, since the same is considered under Decor Constructions' limit. (2 Marks)
5.6 Answer the following questions in the context of the provisions relating to Real Estate
Regulation Act, 2016 (RERA):
(i) Analyse whether the provisions of RERA (which came into effect from 1 st May, 2017)
are applicable to the Decor Dream Home project and if Decor Constructions is liable
for obligations under RERA. (3 Marks)
(ii) Analyse based on the facts of the case, regarding each of the averments of Decor
Constructions with regard to its obligations under RERA for the alleged delay in
handover of the apartments to the allottees and whether it is liable for payment of
compensation under RERA. (6 Marks)
5.7 Examine / advise regarding the below questions relating to the Foreign Exchange
Management Act, 1999 :
(i) How would you advise Mr. Ravi Rao with regard to his aim of acquiring a colonial
villa in Switzerland when he is a resident in India. (2 Marks)
(ii) Evaluate the implications of the transactions proposed to be entered into by
Ms. Anne Rao, including the consequential / related transactions. (4 Marks)
ANSWER TO CASE STUDY 5
5.1 (C)
5.2 (A)
5.3 (D)
5.4 (C)
5.5 (B)
Descriptive Answers
5.6 (i) The project was commenced in November, 2014 and was in progress on the
effective date of coming into force of RERA, 2016 i.e. on 1 st May, 2017. As per
Section 3(1) of RERA, 2016, the promoter shall make an application to the Authority
for registration of the project that is ongoing on the date of commencement of this
Act and for which completion certificate has not been issued within a period of three
months from the date of commencement of this Act.
Accordingly, the provisions of RERA are said to be applicable to the Décor
Dream Home Project as no completion certificate has been issued within a period of
three months from the date of commencement of this Act i.e., uptill July end 2017.

© The Institute of Chartered Accountants of India


Page 135
PAPER – 6D: ECONOMIC LAWS 35

(ii) Return of amount and compensation (Section 18)


This Section provides for the return of amount and compensation.
If the promoter fails to complete or is unable to give possession of an apartment,
plot or building, in accordance with the terms of the agreement for sale or, as the
case may be, duly completed by the date specified therein; he shall be liable on
demand to the allottees, in case the allottee wishes to withdraw from the project,
without prejudice to any other remedy available, to return the amount received by
him in respect of that apartment, plot, building, as the case may be, with interest at
such rate as may be prescribed in this behalf including compensation in the manner
as provided under this Act.
However, where an allottee does not intend to withdraw from the project, he shall be
paid, by the promoter, interest for every month of delay, till the handing over of the
possession, at such rate as may be prescribed.
If the Promoter fails to discharge any other obligations imposed on him under this
Act or the rules or regulations made thereunder or in accordance with the terms and
conditions of the agreement for sale, he shall be liable to pay such compensation to
the allottees, in the manner as provided under this Act.
According to the relevant provisions, Décor Constructions will not be liable under
RERA for handover of the apartments to the allottees as it was within the expected
date of completion i.e., 30 th June, 2018. Therefore, Decor Constructions shall not
be liable for payment of compensation.
Alternate Solution
Analysis of each of Averments of Décor Constructions with regard to its
obligations under RERA for the alleged delay in handing over the apartments to the
allottees:
AVERTMENT (1): Even though, the date of completion stated in RERA registration is
30 th June, 2018 and therefore, the date of handover finally indicated to the allottees is
31 st March, 2018 which is well within the timelines and therefore, there is no non-
compliance with the RERA requirements
As per Section 18, if the promoter fails to complete or is unable to give possession of an
apartment, plot or building, in accordance with the terms of the agreement for sale or, as
the case may be, duly completed by the date specified therein; he shall be liable on
demand to the allottees, in case the allottee wishes to withdraw from the project, without
prejudice to any other remedy available, to return the amount received by him in respect
of that apartment, plot, building, as the case may be, with interest at such rate as may be
prescribed in this behalf including compensation in the manner as provided under this
Act.

© The Institute of Chartered Accountants of India


Page 136
36 FINAL (NEW) EXAMINATION: NOVEMBER, 2019

On a plain reading of this provision, it becomes clear that date of completion referred to
in this provision is the date specified in the agreement. The word “therein” refers to the
“agreement” and not the date of completion revised by the Promoters unilaterally while
registering the project. Hence, the submission of Décor Constructions that as till the date
of completion mentioned in the registration certificate is not crossed, there is no delay
in not valid.
AVERTMENT – 2: The Company had already informed the reasons for the delay of the
project upto 31 st December, 2017 in August, 2016 itself and there was no response /
issues raised by the Allottees at that time. Further, Décor Constructions has also agreed
to provide the apartments for interior work during December,2017 and therefore, it is
effectively agreed to handover the apartment as per the revised timelines communicated
in August, 2016.
From the facts of the case, it appears that Décor Constructions is of the view that since
the complainants did not object to the extended time, hence, the complainants by their
conduct agreed to extend the period of delivery of the possession of the flats. This is not
acceptable because a party cannot take unilateral decision and impose it upon the other
party. The parties have decided to withdraw from the project since the flats were not
delivered on time and no where have they agreed to the new dates as unilaterally
declared by the Company. The handover of the apartments prior to obtaining the
occupancy certificate is mere paper possession and possession without such certificate
is illegal and cannot be permitted in law. Therefore, this offer has been rejected by the
complainants and have exercised their right to claim back their money.
AVERTMENT – 3: Even presuming the applicability of the RERA provisions, there is no
unanimity in the decisions of the allottees on the way forward (since 25 have opted for
cancellation and 5 have opted for compensation) and therefore, this cannot be anyway
given effect to under RERA.
Section 18 offers two options to the allottees – one is for return of the amounts, or
compensation, if the allottees decide not to withdraw from the project. It is not
necessary for unanimity in the decision of the allottees and the promoter is liable
to refund / compensate the allottees based on the option that they choose.
5.7 (i) As per FEMA, 1999 under Section 6(4), a person resident in India may hold, own,
transfer or invest in foreign currency, foreign security or any immovable property
situated outside India if such currency, security or property was acquired, held or
owned by such person when he was resident outside India or inherited from a
person who was resident outside India.
The RBI vide A.P. (DIR Series) Circular No. 90 dated 9 th January, 2014 has
issued a clarification on Section 6(4) of the Act. According to which a person
resident in India may freely utilize all their eligible assets abroad as well as income
on such assets or sale proceeds thereof received after their return to India for

© The Institute of Chartered Accountants of India


Page 137
PAPER – 6D: ECONOMIC LAWS 37

making any payments or to make any fresh investments abroad without approval of
Reserve Bank, provided the cost of such investments and/or any subsequent
payments received therefor are met exclusively out of funds forming part of eligible
assets held by them and the transactions is not in contravention to extant FEMA
provisions.
Accordingly, Mr. Ravi Rao aim of acquiring a colonial villa in Switzerland when
he is resident in India is possible and in compliance with the above provision.
(ii) In the given case, Ms. Anne Rao proposed for two types of investments in India:
(i) Purchase of immovable property in India Jointly with Mr. Ravi Rao
(ii) Investing USD 2,00,000 in special purpose vehicle
W.r.t. part (i) of the transaction proposed by Ms Anne Rao, according to Section
6(3), the Reserve Bank may, by regulations, prohibit, restrict or regulate the giving
of a guarantee or surety in respect of any debt, obligation or other liability incurred
by a person resident outside India.
Therefore, proposed transaction as to purchase of immovable property to be
entered by Ms. Anne Rao, is valid on the guarantee of Décor Construction.
W.r.t. part (ii) of the transaction proposed, investments (or financial commitment) in
JV/WOS abroad by Indian parties through the medium of a Special Purpose Vehicle
(SPV) are also permitted under the Automatic Route if the Indian party is not
appearing in the Reserve Bank's caution list or is under investigation by the
Directorate of Enforcement or included in the list of defaulters to the banking system
circulated by the Reserve Bank/any other Credit Information Company as approved
by the Reserve Bank.
As in the given case, investment in a Special Purpose Vehicle in the form of an
unincorporated joint venture, is invalid in line with the above provision.
Alternate Solution to Part (ii)
As per the Foreign Exchange Management (Permissible Capital Account
Transactions) Regulations, 2000, the person resident outside India is prohibited
from making investments in India in any form, in any Company, or partnership firm
or proprietary concern or any entity whether incorporated or not which is engaged or
proposes to engage in agricultural or plantation activities.
Accordingly, Ms. Anne Rao cannot invest in the aforesaid business since
managed farm business is included under prescribed business of agricultural
and plantation activities.

© The Institute of Chartered Accountants of India


Page 138
DISCLAIMER

This Suggested Answers do not constitute the basis for evaluation of the

student’s answers in the examination. The answers are prepared by the

Faculty of the Board of Studies with a view to assist the students in their

education. While due care is taken in preparation of the answers, if any

errors or omissions are noticed, the same may be brought to the attention

of the Director of Studies. The Council of the Institute is not in anyway

responsible for the correctness or otherwise of the answers published

herein.

Further, in the Elective Papers which are Case Study based, the solutions

have been worked out on the basis of certain assumptions/views derived

from the facts given in the question or language used in the question. It may

be possible to work out the solution to the case studies in a different manner

based on the assumption made or view taken. Further, there should be no

negative marking for wrong answers in MCQ based questions.

© The Institute of Chartered Accountants of India


Page 139
2 FINAL (NEW) EXAMINATION: MAY 2019

PAPER-6D – ECONOMIC LAWS


There are three case study questions in the question paper. Candidates are required to
answer all the questions of any two case study questions.

CASE STUDY 1
Mr. Rohit writer is a well known industrialist based in Pune, India and is the founder director of
M/s. Good Phones Private Limited (Good Phones), a fixed line and mobile phone manufacturer.
Good Phones is one of the largest telephone companies in India and its products are much
sought after in India and abroad. Mr. Rohit visits various countries as part of his business travels
and during these visits he spend significant amount of time in Philanthropic activities and social
gatherings and because of this, he is quite well known in the business circles globally. Mr. Rohit
has a penchant for investing his money in buying various real estate property all over India and
has passed this trait on to his son, Mr. Rahul Writer as well. Mr. Rahul completed his MBA from
Stanford University and is assisting Mr. Rohit in his business. Mr. Rohit also has a daughter,
Mr. Sonali Writer, who studies Art in Italy and has opened her own Art Studio in Milan. Mr. Rohit
is very proud of Sonali and supports her financially for her stay in Italy as well as expenses
towards maintaining the studio.
The marketing department of Good Phones introduced various new models in the last couple of
months with new technology such as 2 selfie cameras, faster processor and sleeker look. Good
Phones expect these phones to be a major attraction in the global markets due to the attractive
price range and therefore wanted to promote these phones extensively on a global basis. For
the purpose of advertisements, Good Phones engaged the services of Mr. David Smith, a
prominent baseball player and Ms. Emma Drew, a Miss Universe winner and agreed to pay a
“guaranteed” fee of USD 500,000 each plus 10% bonus based on the sales of the new models
in year 1.
Mr. Rohit sent 5 sample mobile phones and 5 fixed line phones to his dealers abroad (numbering
1000 dealers), clearly marked as not for sale and other promotional material such a brouchers,
3D moulds for display in dealer shops etc. The value of the items were approximately INR 4
crore. He also sent 1 mobile phone to each of his dealers as a token of gift and appreciation
(total value of INR 0.50 Crores). Mr. Srinivas Rajan, the CFO of Good Phones indicated him
that since these products have been sent free of cost and not for sale, these need not be
included in the export declaration to be filed by Good Phones.
On 15 February 2018, Good Phones made a large sale to one of the dealers M/s. Delayed
Ringtone Enterprise, Germany, for USD 5 million and had received USD 2 million by 15 May
2018 and did not receive the balance USD 3 million until 15 August 2018, i.e. 6 months from the
date of sale. After several reminders and threatening calls to stop further shipment, another
USD 1 million was received on 10 October 2018 and the balance remained outstanding as at
31 December 2018.

© The Institute of Chartered Accountants of India


Page 140
PAPER – 6D: ECONOMIC LAWS 3

Based on the success of Good Phones, Mr. Rohit incorporated a new company, M/s. Stay
Connected Private Limited, (Stay Connected) an Internet service provider and purchased a large
consignment of networking equipment for providing internet operations through dedicated
broadband lines along with a landline facility. This would then provide Mr. Rohit quite a few
synergies with the existing Good Phones business and enable him to become an end to end
Telecom Czar. Mr. Rohit held 60% stake in Stay Connected and the balance 40% was held by
a foreign collaborator. Along with all the networking equipment, Stay Connected hired
transponders from a company in Australia and paid AUD 10 million through its authorized healer.
Stay Connected also entered into an agreement with foreign collaborator (holding 40% stake)
to pay royalty and technical fees for the support provided by them.
During his visit to Milan to meet Ms. Sonali, Mr. Rohit obtained EUR 10,000 from his Italian
dealer for his use during his stay in Italy and instructed the dealer to reduce the sum from the
payments to be made by the dealer for the supplies from Good Phones. Out of such funds,
Mr. Rohit used EUR 5,000 towards purchasing sweepstakes tickets in Mila, Italy, unfortunately,
he did not win any money in the sweepstakes event.
Mr. Rahul, after gaining experience in India, wanted to expand the business in the USA (by
establishing a subsidiary of Good Phones in the USA) and therefore decided to move to the
USA along with his wife. For this purpose, he wanted to dispose off some of the properties
owned by him in India. Accordingly, Mr. Rahul sold an apartment in Mumbai owned by him to
Mr. Stuart Cooper, a citizen of USA, and a fellow student of his at Stanford University. Mr. Stuart
was planning to come to India for the first time in the next couple of months to take up a job and
therefore, wanted to secure a place for his stay. Mr. Rohit also sold a villa and his agricultural
land in Pondicherry to Mr. Rajesh Subramanian, his professor at Stanford, who was a person of
India origin. The payment for the villa and agricultural land was paid by Mr. Rajesh partly (50%)
from his FCNR account and the balance in USD traveller cheques, which will be of use to Mr.
Rahul when he visits USA.
After obtaining his US visa, Mr. Rahul purchased a ranch (farm house) in Texas for USD 2
million, using USD 1.50 million from his RFC account and USD 500,000 sent from his INR
account through normal banking channels.
Mr. William Rutherford, one of Mr. Rohit’s business acquaintances and a citizen of the USA, is
very much interested in Indian culture and practices and therefore stays in India for 8 months
(from April 2018 to November 2018) to attend an art of living course and to learn/practice yoga.
William believes that he has been resident in India for more than the prescribed 182 days and
therefore, is a resident in India under FEMA.
Mr. Rohit, in his penchant for purchasing various properties, zeroed in on an exclusive
apartment complex in Bangalore having state-of-the art facilities. He purchased two 4 bedroom
apartments costing INR 2 crore each, one in the name of Ms. Sonali and one in the name of Mr.
Srinivas Rajan, since Mr. Rohit wanted Mr. Srinivas Rajan to feel happy and trusted. Both the
apartments were given on rent to a large multinational bank and he received a rent of INR 0.20
Crores per year for each of the apartments in the bank accounts of Ms. Sonali and Mr. Srinivas

© The Institute of Chartered Accountants of India


Page 141
4 FINAL (NEW) EXAMINATION: MAY 2019

Rajan, respectively. After 4 years, Mr. Srinivas Rajan transferred the property back in the name
of Mr. Rohit at zero consideration. Mr. Rohit also purchased a 3 bedroom apartment in the same
complex in his name, jointly with his brother, Mr. Sunil Writer. The property (along with the stamp
duty) was paid for by Mr. Rohit and was being used by Mr. Sunil for his stay though the property
was pending registration due to Rohit’s travel abroad.
Once the property was transferred back by Mr. Srinivas Rajan, Mr. Rohit wanted to sell the same
to Mr. Arjun De Silva, a citizen of Shi Lanka. However, he was advised by Mr. Srinivas Rajan
that Mr. Arjun De Silva cannot acquire property in India and therefore, Mr. Rohit proposed to
lease it to Mr. Arjun De Silva for a period of 20 years for an upfront consideration of INR 1 crore
and an annual rent of INR 8 lakhs payable in advance.
During the review of the bank reconciliation statements of Good Phones, Mr. Srinivasan Rajan
noted that an amount of INR 2 Crore had been received in one of the bank accounts without
any details relating to the same. Mr. Srinivas Rajan informed this to Mr. Rohit and Mr. Srinivas
Rajan suggested to Mr. Rohit to immediately transfer that money out of the bank of Good Phones
to Mr. Rohit’s personal bank account, so that the Company’s bank accounts are cleared and
there are no reconciling items, which Mr. Rohit agreed to. Out of the INR 2 Crore, Mr. Rohit
used INR 1.75 Crores for acquiring further 20% stake in Stay Connected from the foreign
collaborator and the balance INR 0.25 Crores for purchasing a stunning diamond set for his
wife, Ms. Anjali Writer, as a gift for her 50 th birthday.
The extract of the last audited financial statements of Stay Connected was provided by
Mr. Srinivas Rajan to Mr. Rohit to evaluate his acquisition.
Particulars Amount in
INR (Crores)
Immovable property (market value INR 8.00 Crores) 5.00
Other fixed assets (net of depreciation of INR 1.00 Crores) 4.00
Inventory 2.00
Receivables and Loans and Advances 1.50
Deferred Advertisement Costs 0.50
Advance tax paid 1.00
Total Assets 14.00
Shareholders’ Funds (including 1,000,000 equity shares of INR 10 each, 4.00
fully paid up)
Provisions for taxation 0.50
Loans from Banks 3.00
Trade Payables (including provision for unascertained liabilities - INR 6.50
1 crore)
Total Liabilities 14.00

© The Institute of Chartered Accountants of India


Page 142
PAPER – 6D: ECONOMIC LAWS 5

Other information:
(i) Contingent liabilities-INR 2.00 Crores (including INR 0.50 cores relating to arrears on
cumulative preference shares).
(ii) The Board of Directors has proposed a dividend payout of INR 1 crore to the equity
shareholders, which is pending approval of the shareholders.
The Bank, on noting the large transactions on Mr. Rohit's personal bank account, tipped
the Income tax authorities regarding the same and the Initiating Officer summoned
information from Mr. Rohit and Mr. Srinivas Rajan regarding the transactions to start
proceedings under the Prohibition of Benami Property Transactions Act, 1988 (PBPT Act,
1988).
Answer the following questions:
1.1 An Indian citizen resident outside India is permitted to transfer his agricultural property in
India to:
(A) any person resident in India
(B) a person resident in India if he is a citizen of India or a person of Indian origin
(C) a person resident in or outside India if he is a citizen of India or a person of Indian
origin
(D) any person who is resident in the country where the Indian citizen currently resides
(2 Marks)
1.2 Out of the below, what are the transactions that requires prior approval of the Government
of India ?
(A) Payment of "guaranteed" fee by Good Phones to Mr. David Smith and Ms. Emma
Drew.
(B) Payment of Royalty and Technical fees by Stay Connected to the foreign collaborator.
(C) Payment of hiring charges for the transponders by Stay Connected.
(D) Payment of INR 1.75 Crores by Mr. Rohit to acquire shares of Stay Connected from
the foreign collaborator (2 Marks)
1.3 Is the use of EUR 5,000 towards purchasing sweepstakes by Mr. Rohit as per the
provisions of FEMA, 2002 ?
(A) No, drawal of foreign exchange for purchasing lottery tickets, sweepstakes, etc. is
prohibited under the FEMA 2002

© The Institute of Chartered Accountants of India


Page 143
6 FINAL (NEW) EXAMINATION: MAY 2019

(B) No, Mr. Rohit should have obtained the prior approval of the RBI before purchasing
the sweepstakes ticket
(C) FEMA 2002 will not be applicable, since the money was directly obtained by
Mr. Rohit from his Italian dealer outside the country
(D) None of the above (2 Marks)
1.4 As per the provisions of FEMA, 2002, Mr. William Rutherford is :
(A) a person resident in India in the financial year 2018-19 as per FEMA, "since he has
resided for more than 182 days during the year
(B) a person resident in India in the financial year 2019-20 as per FEMA, since he has
resided for more than 182 days during the previous financial year
(C) not a person resident in India since he is a foreign citizen
(D) not a person resident in India, since he is on a short term visit to India and is not on
a long term visa (2 Marks)
1.5 Is the purchase of Ranch in Texas by Mr. Rahul in accordance with FEMA, 2002?
(A) No, Rahul, as a citizen of India cannot purchase a Ranch outside India.
(B) Yes, there is no specific limit under FEMA 2002 with regard to purchase of
immoveable property outside India.
(C) No, Rahul can purchase assets outside India only if the purchase is jointly with a
relative, who is a resident outside India, and there is no outflow of funds.
(D) No, since Rahul has used funds from his INR account for making the payment to the
extent of USD 500,000. (2 Marks)
1.6 In case Mr. Rohit is proven guilty of violating the provisions of PBPT Act, 1988, what is the
maximum punishment that he is liable for under the PBPT Act, 1988 ?
(A) Rigorous imprisonment for a term of one to seven years, with fine which may extend
to 25 of the fair market value of the property.
(B) Rigorous imprisonment for a term of three to seven years, without fine.
(C) Rigorous imprisonment for a term upto seven years, with fine which may extend to
50% of the fair market value of the property.
(D) Fine which may extend to 25% of the fair market value of the property. (2 Marks)
1.7 Once the benami property acquired by Mr. Rohit and his family have been identified, which
authority has the power to confiscate and vest the property?

© The Institute of Chartered Accountants of India


Page 144
PAPER – 6D: ECONOMIC LAWS 7

(A) Initiating Officer


(B) Administrator
(C) Approving Authority
(D) Adjudicating Authority (2 Marks)
1.8 Which of the following is a benami property under the PBPT Act, 1988 ?
(A) Property held by a member of a Hindu Undivided Family (not the Karta) which is held
for the benefit of the members of the family
(B) Property purchased by a person in the name of his spouse paid out of money received
from his father in law as a gift.
(C) Property purchased by a person in the name of his brother out of his own funds.
(D) Property purchased in the name of an individual, for which consideration was paid by
another person and such another person enjoys the possession of the property.
(2 Marks)
1.9 Assuming that the transactions relating to the receipt of INR 2 crores in the bank account
of Good Phones and the subsequent transactions are considered as benami transactions,
can the Initiating Officer take action against Mr. Srinivas Rajan ?
(A) Yes, he is the CFO of Good Phones and therefore, responsible for ensuring
compliance with the law.
(B) No, he has not received, held, or acquired the proceeds in his account or benefitted
from the same.
(C) Yes, since he abets Mr. Rohit in transferring the money from the bank account of
Good Phones to Mr. Rohit's personal account.
(D) No, he is responsible only for Good Phones and he has ensured that the funds are
not retained in the books of Good Phones / used by Good Phones for its business.
(2 Marks)
1.10 The Initiating Officer believes that Ms. Anjali is not a benamidar under the PBPT Act,
1988. What is your view?
(A) No, she is not a benamidar, since she has not purchased the diamond set, but
received as a Gift.
(B) Yes, she is a benamidar as she is in possession of a property acquired out of benami
funds.

© The Institute of Chartered Accountants of India


Page 145
8 FINAL (NEW) EXAMINATION: MAY 2019

(C) No, she is not a benamidar, since the transaction is not a benami transaction.
(D) Yes, she is a benamidar, but will not be liable for any prosecution under the PBPT
Act, since she is not a party to any of the transactions but only a beneficiary.
(2 Marks)
1.11 Answer the following questions in the context of the provisions relating to the Foreign
Exchange Management Act, 2002 :
(i) Srinivas Rajan reaches out to you to confirm his views regarding inclusion / exclusion
of the items sent free of cost to the dealers in the export declaration.
(4 Marks)
(ii) Examine the validity / appropriateness of the sale of immoveable property by Mr.
Rahul to Mr. Stuart Cooper and Mr. Rajesh Subramanian. (4 Marks)
(iii) Srinivas Rajan reaches out to you and seek your support to evaluate if there is a non-
compliance with the FEMA regulations regarding the sale made to M/s Delayed
Ringtone and the receipt of the proceeds and if so, the quantum, the consequences
and the future course of action that needs to be taken by Good Phones relating to the
same. (4 Marks)
(iv) Arjun disagrees with the advice received from Srinivas Rajan and asks your views on
why he cannot purchase the home from Mr. Rohit and if not, whether the terms and
conditions of the lease are acceptable. (2 Marks)
1.12 Examine/advice regarding the below questions relating to the Prohibition of Benami
Property Transaction Act, 1988:
(i) Examine the appropriateness/impact of the PBPT Act 1988 on the 3 apartments
purchased by Mr. Rohit in Bangalore. How does the transfer back of the apartment
by Mr. Srinivas Rajan to Mr. Rohit affect your conclusion? (4 Marks)
(ii) The Initiating Officer, who is probing the transactions relating to the INR 2 crores
received and spent by Mr. Rohit, seeks your advice on identify the benami
properties/transaction, the benamidars, the beneficial owner. (4 Marks)
(iii) Assuming that the cost of acquisition and the market value based on discounted cash
flow method is INR 2 crores, calculated the fair market value of M/s. Stay Connected
in accordance with Rule 3 of the Prohibition of Benami Transactions Rules, 2016.
(4 Marks)
(iv) What is the process to be followed by the Initiating Officer for attachment of the
property under a benami transaction? (3 Marks)

© The Institute of Chartered Accountants of India


Page 146
PAPER – 6D: ECONOMIC LAWS 9

Answer Case study 1


1.1 (A)
1.2 (C).
1.3 (A)
1.4 (D)
1.5 (D)
1.6 (A)
1.7 (D)
1.8 (D)
1.9 (C)
1.10 (B)
1.11 (i) As per the Regulation 3 of the Foreign Exchange Management (Export of Goods and
Services) Regulations, 2015, in case of exports taking place through Customs manual
ports, every exporter of goods or software to any place outside India, shall furnish to
the specified authority, a declaration as regards the export value of the Goods. In
respect of export of services to which none of the Forms specified in these
Regulations apply, the exporter may export such services without furnishing any
declaration, but shall be liable to realise the amount of foreign exchange whic h
becomes due or accrues on account of such export, and repatriate the same to India.
However, Regulation 4 of the said Regulation states of exemptions w.r.t. export of
goods / software may be made without furnishing the declaration on the following
items which are sent free of cost:
• trade samples of goods and publicity material supplied free of payment;
• by way of gift of goods accompanied by a declaration by the exporter that they
are not more than five lakh rupees in value.
Therefore, sending 5 sample mobile phones and fixed line phones to 1000 dealers is
exempted and does not require Good Phones to include in the export declaration.
With regard to sending mobile phones to the dealers as gift for a total value of
INR 50 lakhs, as per the above Regulation, the exemption for sending gifts by an
export is available only if the value of the goods are not more than ` 5 lakhs in value.
In the case study, since the value of the goods is more than the exemption limit, they
need to be included in the export declaration.
(ii) As per regulation 3 of the Foreign Exchange Management (Acquisition and Transfer
of Immovable Property in India) Regulations, 2018, an NRI or OCI, may acquire

© The Institute of Chartered Accountants of India


Page 147
10 FINAL (NEW) EXAMINATION: MAY 2019

immovable property in India other than agricultural land/farm house/plantation


property. Provided that the consideration, if any, for transfer, shall be made out of (i)
funds received in India through banking channels by way of inward remittance from
any place outside India or (ii) funds held in any non resident account maintained in
accordance with the provisions of the Act, rules or regulations framed thereunder.
Mr. Staurt Cooper, being an Overseas Citizen of India is entitled to acquire an
apartment in Mumbai owned by Mr. Rahul.
Whereas in case of Mr, Rajesh Subramanian, being NRI, he may acquire immovable
property in India other than agricultural land/farm house/plantation property.
Provided further that no payment for any transfer of immovable property shall be
made either by traveler's cheque or by foreign currency notes or by any other mode
other than those specifically permitted under this regulation. Since with related
transactions, payment was made partly from FCNR account and in USD Traveler
cheques, which was against the mode of payment prescribed in the said regulation.
Therefore, sale of immovable property by Mr. Rahul to Mr. Stuart Cooper is valid,
whereas to Mr. Rajesh Subramanian, the said transaction is invalid.
(iii) Regulation 9 of the Foreign Exchange Management (Export of Goods and Services)
Regulations, 2015 specifies the Period within which export value of
goods/software/services to be realised.
According to it the amount representing the full export value of
goods/software/services exported shall be realised and repatriated to India within
nine months from the date of export.
Further that the Reserve Bank, or subject to the directions issued by that Bank in this
behalf, the authorised dealer may, for a sufficient and reasonable cause shown,
extend the period of nine months.
Due to delay in of the proceeds of sale, M/s Delayed Ringtone contravened provision
of this Act & Regulations.
According to Regulation 14 of the Foreign Exchange Management (Export of Goods
and Services) Regulations, 2015, where in relation to goods or software export of
which is required to be declared on the specified form and export of services, in
respect of which no declaration forms has been made applicable, the specified period
has expired and the payment therefor has not been made as aforesaid, the Reserve
Bank may give to any person who has sold the goods or software or who is entitled
to sell the goods or software or procure the sale thereof, such directions as appear
to it to be expedient, for the purpose of securing,
(a) the payment therefor if the goods or software has been sold and
(b) the sale of goods and payment thereof, if goods or software has not been sold
or reimport thereof into India as the circumstances permit, within such period as

© The Institute of Chartered Accountants of India


Page 148
PAPER – 6D: ECONOMIC LAWS 11

the Reserve Bank may specify in this behalf;


Provided that omission of the Reserve Bank to give directions shall not have the effect
of absolving the person committing the contravention from the consequences thereof.
Therefore, in such situation, M/s Delayed Ringtone shall, upon adjudication under
Section 13 of the FEMA, be liable to a penalty up to thrice the sum involved in such
contravention where such amount is quantifiable, or up to two lakh rupees where the
amount is not quantifiable, and where such contravention is a continuing one, further
penalty which may extend to five thousand rupees for every day after the first day
during which the contravention continues.
Hence, Mr. Delayed will be liable to USD 6 million [2 million (sum involved in such
contravention) x 3)] and further penalty up to five thousand rupees for every day after
the first day during which the contravention continues.
(iv) As per Regulation 9 of the Foreign Exchange Management (Acquisition and Transfer
of Immovable Property in India) Regulations, 2018, prohibition on acquisition or
transfer of immovable property in India is cast on the citizens of certain countries.
Accordingly, no person being a citizen of Sri Lanka, without prior permission of the
Reserve Bank shall acquire or transfer immovable property in India, other than lease,
not exceeding five years.
Since in the given case, Arjun, being a citizen of Sri Lanka, is prohibited to acquire
property in India, except on lease for 5 years. If Mr. Rohit proposes to lease Mr. Arjun
for a period of 20 years, can do so only on the prior permission of the RBI.
1.12 (i) In the given case study, Mr. Rohit purchased 3 flats in Bangalore in the name of Ms.
Sonali, Mr. Srinivas Rajan, and jointly with his brother Sunil.
Apartment purchased in the name of Ms. Sonali- The property has been purchased
by Rohit in the name of his daughter Ms. Sonali. Therefore, although the
consideration for the purchase has been made by Mr. Rohit, since this is covered by
the exemtion provided (since the property has been purchased in the name of his
child), it is not covered as a benami transaction.
Apartment purchased in the name of Mr. Srinivas Rajan- A benami transaction is
defined as a transaction where a property is transferred to or held by one person and
consideration is paid by some other person. In this case, the property is in the name
of Srinivas Rajan although the consideration is paid by Rohit. Therefore, this is a
benami transaction.
Apartment purchased jointly in the name of Rohit and his brother Sunil- A property
jointly held in the name of brother and they appear as joint owners. Hence, this is not
a benami transaction.

© The Institute of Chartered Accountants of India


Page 149
12 FINAL (NEW) EXAMINATION: MAY 2019

First two flats were rented to a large multi-national bank. After 4 years, Mr, Srinivas
Rajan transferred the property back in the name of Mr. Rohit without any
consideration.
As per section 6 of the Prohibition of Benami Property Transactions Act, 1988, no
person, being a benamidar shall re-transfer the benami property held by him to the
beneficial owner or any other person acting on his behalf. In cases where any property
is re-transferred, then such a transaction of a property shall be deemed to be null and
void.
In the said above case transaction of transfer back of the apartment by Mr. Srinivas
Rajan to Mr. Rohit is void.
(ii) The Initiating Officer, who is probing the transactions relating to the INR 2 crores
received and spent by Mr. Rohit, seeks your advice on identifying the benami
properties/ transactions, the benamidars, the beneficial owner.
The following are the benami transactions and benamidars:
Transaction Benamidar
Receipt of INR 2 crores in the bank Good Phones
account of Good Phones
Transfer of INR 2 crores from the Mr. Rohit (person abetting the
bank account of Good Phones to Mr. transaction, Mr. Srinivas Rajan)
Rohit’s personal bank account
Acquisition of shares of Stay Shares of Stay Connected becomes
Connected using the benami money benemi property
Purchase of Jewellery as gift for Ms. The jewellery becomes benami property.
Anjali Writer Mrs. Anjali also becomes Benamidar.
The original beneficial owner is not identified- the person who should have been the
original recipient of the funds, may be the original beneficial owner, although these
funds are not being held by Rohit/ Anjali for their benefit.
(iii) According to section 2(16) of the Prohibition of Benami Property Transaction Act,
1988, fair market value", in relation to a property, means—
(1) the price that the property would ordinarily fetch on sale in the open market on
the date of the transaction; and
(2) where the price referred to in sub-clause (i) is not ascertainable, such price as
may be determined in accordance with such manner as may be prescribed in
Rule 3 of the Prohibition of Benami Property Transaction Rules, 2016.
As per the said Rule, the price of unquoted equity shares shall be the higher of-
(i) its cost of acquisition;
(ii) the fair market value of such equity shares determined, on the date of

© The Institute of Chartered Accountants of India


Page 150
PAPER – 6D: ECONOMIC LAWS 13

transaction, by a merchant banker or an accountant as per the Discounted Cash


Flow method; and
(iii) the value, on the date of transaction, of such equity shares as determined by
the formula given in the Rules.
The value of (iii) above is determined as below:
Particulars Amount Value to be Remarks
(` In considered for
Crores) calculation
(` In Crores)
Immovable Property 5.00 8.00 Market value to be
considered
Other fixed assets 4.00 4.00 Book value net of
depreciation
Inventories 2.00 2.00 Book value
Receivables and Loans and 1.50 1.50 Book value
Advances
Deferred Advertisement Costs 0.50 0.00 Not to be considered
Advance tax paid 1.00 0.00 Not to be considered
Total Value of Assets 15.50
Shareholders’ funds -4.00 0.00 Share capital and
Reserves not to be
considered
Provision for taxation -0.50 0.00 Not to be considered
Loans from Banks -3.00 -3.00
Trade Payables -6.50 -5.50 Provision for
unascertained
liabilities not to be
considered
Contingent Liabilities -2.00 -0.50 Arrears of divided on
cumulative preference
shares to be
considered
Proposed dividend on equity -1.00 -0.00 Not to be considered
shares
Total Value of Liabilities -9.00
Fair Market Value (Asset- 6.50
Liabilities) *Paid up Equity

© The Institute of Chartered Accountants of India


Page 151
14 FINAL (NEW) EXAMINATION: MAY 2019

Capital/ Paid up value of equity


shares
Value of equity shares 1.30
acquired i.e. 20% of total
In the said question, the cost of acquisition is assumed at ` 2.00 crores, the value,
on the date of transaction, of such equity shares as determined by the formula given
in the Rules is ` 1.30 crores and the market value based on discounted cash flow
method is given as ` 2 crores. Thus, the fair market value of the acquisition in M/s.
Stay Connected will be ` 2 crores being highest of above and ` 10 crores for the
company as a whole (i.e. 2.00/0.20).
(iv) As per section 24 of the Prohibition of Benami Property Transactions Act, 1988, where
the Initiating Officer, on the basis of material in his possession, has reason to believe
that any person is a benamidar in respect of a property, he may, issue a show cause
notice to the person.
Where the notice specifies any property as being held by a benamidar, a copy of the
notice shall also be issued to the beneficial owner if his identity is known.
Where the Initiating Officer is of the opinion that the person in possession of the
property held benami may alienate the property during the period specified in the
notice, he may, with the previous approval of the Approving Authority, by order in
writing, attach provisionally the property in the manner as prescribed in Rule 4 of the
Benami Transactions Prohibition Rules, 2016, for a period not exceeding ninety days
from the date of issue of notice.
The Initiating Officer, after making such inquires and calling for such reports or
evidence as he deems fit and taking into account all relevant materials, shall, within
a period of ninety days from the date of issue of notice -
(i) pass an order continuing the provisional attachment of the property with the prior
approval of the Approving Authority, till the passing of the order by the
Adjudicating Authority; or
(ii) revoke the provisional attachment of the property with the prior approval of the
Approving Authority;
where provisional attachment has not been made, pass an order provisionally
attaching the property with the prior approval of the Approving Authority, till the
passing of the order by the Adjudicating Authority; or decide not to attach the property
as specified in the notice, with the prior approval of the Approving Authority.
Where the Initiating Officer passes an order continuing the provisional attachment of
the property or passes an order provisionally attaching the property, he shall, within
fifteen days from the date of the attachment, draw up a statement of the case and
refer it to the Adjudicating Authority.

© The Institute of Chartered Accountants of India


Page 152
PAPER – 6D: ECONOMIC LAWS 15

CASE STUDY 2
Winner Builders Private Limited ("Winner") is a premium real estate builder who specializes in
constructing mid-sized apartment complexes (20 - 40 apartments) in South India. Winner was
started in the year 2004 by Mr. Vijay Nair, Managing Director and has its head office in Kochi,
Kerala with branches in Trivandrum, Bengaluru, Chennai and Tirupati. Mr. Vijay Nair has been
in the real estate business for more than 30 years and comes from a family of civil engineers
who are highly respected by their customers. Mr. Arun Nair, son of Mr. Vijay Nair, is a Chartered
Accountant and is the Chief Financial Officer of Winner. Mr. Vijay and Mr. Arun together own 60
of the share capital of Winner and the balance is held by a large private equity investor.
Although the company is a private limited company, the affairs of the company are handled in
the most professional manner akin to a listed company and Mr. Arun ensures that the financial
statements are properly prepared and presented to the Board of Directors (Mr. Vijay, Mr. Arun
and a representative of the PE investor) on a quarterly basis. The financial performance of
Winner has been reasonable and being a conservative person, Mr. Vijay was never in the mind-
set of taking aggressive positions with regard to business. Over the last few months, the PE
investor has been pushing the company in making changes in the operational mechanism, sale
prices etc. to increase the profits of the company and ensure decent return on their investment.
Due to this, Mr. Vijay and Mr. Arun are under tremendous pressure to complete the ongoing
projects fast and start new projects immediately and increase the revenues / profits of the
Company.
In June 2018, Winner announced a new 80 apartment project in Kochi named as "Winner
Shikaram", an ultra-modem luxury apartment complex with a variety of amenities including
swimming pool, skating rink, basketball court, fully equipped club house with all amenities, etc.
As per RERA regulations the Company applied for registration of the project on 15 th June 2018.
On 20 th June 2018, the Company announced the launch of the project and commenced a big
advertisement campaign in the TV media and also through release of promotion material
through social media. It also collaborated with a regional TV Channel and announced a free
home in "Winner Shikaram" for the first prize winner of a popular reality show. The property was
registered by RERA on 10 th July 2018 after scrutiny of the information provided by the promoter.
Based on the past performance of the Winner group and the general image of Mr. Vijay Nair,
there was tremendous demand for the apartments in the project and all the apartments were
booked within 1 month from the date of launch (20 th June, 2018). The following were some of
the conditions mentioned in the agreement to sale entered into by Winner with its allottees:
1. Expected date of completion of construction -31st March, 2020.
2. Expected date of handover-31 st May, 2020, subject to a grace period of 4 months.
3. Booking Advance amount to be paid prior to entering into agreement to sale – 20% of
total cost of apartment
4. Open car parking cost-INR 200,000

© The Institute of Chartered Accountants of India


Page 153
16 FINAL (NEW) EXAMINATION: MAY 2019

5. Any delay in payment of dues by the allottees will liable for interest on such delayed
payments.
6. Return of booking amount shall not be entertained for any reason whatsoever.
7. Winner Group shall be liable for any deficiency in quality of construction for a period
of 3 years from the date of handing over the apartments.
Winner Group collected a total amount of INR 80 crores from the allottees and deposited an
amount of INR 60 crores in an escrow account for exclusive use for construction of the complex.
Separately, an amount of INR 5 lakhs each was collected from the 80 allottees in cash,
aggregating to INR 400 lakhs towards interior work, modular kitchen, supplying fans and lights,
etc. This money was accounted as receipt in a separate company, M/s. Wonderful Interiors,
which was owned by Ms. Anusha Nair, daughter of Mr. Vijay Nair and Mr. Arun Nair.
Although the construction was proceeding apace, the Company encountered severe rock
formations under the ground in one section of the land area which was previously not known
and due to the same, the Company concluded that the swimming pool could not be constructed
as designed and the size of the same had to be reduced. Winner got in' touch with the allottees
and proposed that the reduction of the size of swimming pool will be compensated suitably by
Winner by providing a Jacuzzi and Spa inside the club house. This was accepted by majority
(45 of the 80) of the allottees and, accordingly, Winner proceeded with the construction based
on the amended plan.
Few of the allottees reached out to Mr. Vijay Nair and stated that the carpet area for their
apartments was lesser than the size stipulated in the sale agreement and therefore, wanted to
be compensated. Mr. Vijay Nair mentioned to them that the reduction in the area was on account
of the exterior walls appurtenant to their apartments and this is the case with all the apartments
and not specific to their homes alone.
Mr. Arun Nair attended one of the real estate conclaves held in Bangalore, in which he met one
Mr. Henry Stewart, who runs an interior designing warehouse in Dubai UAE and showed quite
a few exhibits to Arun. Arun was impressed by the designs and the prices quoted by Mr. Henry.
Mr. Henry was also amenable to receive funds in cash in India through an intermediary and then
provide the material to Arun from UAE. Based on the same, Arun arranged for making cash
payment to the extent of INR 200 lakhs (Out of the INR 400 lakhs received by M/s Wonderful
Interiors) to an intermediary in Delhi, and the material was received from Henry in a month.
During his visit to India, Henry noted that his UAE passport got expired and he did not realise
the same. Since he did wanted to leave India immediately, he got in touch with a travel agent,
Mr. Anil Kumar, who helped him get a forged passport, for which Mr. Henry paid INR 2 lakhs in
cash.
Out of the balance INR 200 lakhs cash available with Wonderful Interiors, Arun used cash
amounting to INR 25 lakhs to pay amounts to various intermediaries to facilitate timely and
smooth registration process of thie apartments of Winner Shikharam, which was paid by the

© The Institute of Chartered Accountants of India


Page 154
PAPER – 6D: ECONOMIC LAWS 17

intermediaries to the officials of the Sub- Registrar. With Henry's help, Arun transferred the
balance amount of INR 175 lakhs to an intermediary in Delhi and invested the amount to
incorporate a shell company in the Cayman Islands. The funds were then transferred back by
the Shell Company to the bank account of Winner. For this purpose, Mr. Arun raised export
invoices in the books of Winner on the Shell Company for providing professional services
relating to real estate business. Based on these invoices, Winner claimed export incentives
under the relevant laws in India and received INR 20 lakhs as export incentive.
On 30th March 2019, a meeting was organised by the Company and all the allottees during which
Mr. Vijay Nair provided a status update on the project and stated that bulk of the construction
activities will be completed by the timeline mentioned in the sale agreement (31 st May, 2020)
and the apartments will be handed over by 31 st July, 2020 (i.e. within the grace period). The
common areas will be completed in parallel and handed over by 30 th September, 2020. The
slight delay in completion was on account of non-availability of quality labour and he wanted
only the best labour to work on the project to ensure that the home owners have a happy life
after hand over. He also mentioned that the labour rates increased by 15% after the sale
agreements were entered and the Company did not ask for increase in prices from the allottees
only for good will reasons. The allottees were unhappy with the delay but, accepted the same,
since there was no other choice.
As one of the shareholders of Wonderful Interiors, Ms. Anusha Nair decided to visit Dubai to
see the interior designs and then place an order for the upcoming projects. During her visit, she
purchased 500 grams worth of gold (costing INR 15 lakhs) and since, she did not have enough
money, she asked Mr. Arun Nair to make the payment through the intermediary in Delhi. Based
on the discussion with the intermediary, Mr. Arun Nair provided an antique painting which he
got from one of his social friends to the intermediary as consideration for the gold purchased by
Ms. Anusha Nair in Dubai. Based on the same, Ms. Anusha brought the gold with her through
the green channel.
One of the employees of Wonderful Interiors, noting the substantial amount of cash transactions,
informs the Bank regarding the same, which in turn informs the enforcement directorate. The
ED has issued a show cause notice to all the parties regarding the above transactions.
Answer the following questions:
2.1 RERA authorities sent a notice to Winner that their advertisement campaign was not
in accordance with the RERA 2016. Evaluate.
(A) Valid, Since Winner decided to use Social media platform for promotion, without
obtaining specific approval from RERA.
(B) Valid, Since Winner collaborated with a TV channel to give a free home in Winner
Shikaram when the construction itself was not complete.
(C) Valid, Since Winner launched the project and commenced marketing even before the
project received registration from RERA.

© The Institute of Chartered Accountants of India


Page 155
18 FINAL (NEW) EXAMINATION: MAY 2019

(D) Not Valid, Since Winner applied for the registration prior to the launch of campaign
and the registration was ultimately received within the stipulated period. (2 Marks)
2.2 As per RERA, Winner is required to enable the formation of the association of allottees
of Winner Shikaram within-months.
(A) 3 months of the majority of the allottees having booked their apartment.
(B) 3 months of the receipt of occupancy certificate.
(C) 3 months of the majority of the allottees registering their apartments with the sub-
registrar.
(D) 3 months of all the allottees making the full payment for the apartments. (2 Marks)
2.3 After registering the apartments in the name of the allottees, Winner informed the
allottees that they need to pay the water and electricity charges to the concerned
departments for their apartments. Evaluate.
(A) The registration of the apartments denote that the allottees are now the legal owners
of the apartments and hence, need to bear the water and electricity charges.
(B) The promoter is liable for making payment for the water and electricity charges until
the physical possession is transferred to the allottees.
(C) This is dependent on the terms of the agreement of sale between Winner and the
allottees.
(D) This amount need to be paid equally by Winner and the allottees, since the
registration is completed and only transfer of physical possession is pending.
(2 Marks)
2.4 The time limit within which the allottees of winner Shikaram are required to take
physical possession of the apartment after issuance of occupancy certificate is:
(A) Three months
(B) One month
(C) Five months
(D) Two months (2 Marks)
2.5 As per provisions of RERA, collection of cash by Wonderful Interiors for interior work,
modular kitchen, supplying fans and lights, etc. :
(A) May be appropriate, since RERA does not specify the mode of collection.

© The Institute of Chartered Accountants of India


Page 156
PAPER – 6D: ECONOMIC LAWS 19

(B) May not be appropriate, since collection should be done as per the stipulations of
RERA.
(C) May be appropriate, since provisions of RERA are not applicable.
(D) May not be appropriate, since Wonderful Interiors are not registered with RERA.
(2 Marks)
2.6 What are the three distinct stages of Money Laundering?
(A) Information, Interrogation, Indictment
(B) Placement, Layering, Integration
(C) Planning, Comingling, Profiting
(D) Monitoring, Adjudicating, Punishing (2 Marks)
2.7 Which of the following are not circumstances which need to be considered by the
Director of Enforcement for performing search of the offices of Winner and other parties
mentioned in the case study?
(A) Possession of any property related to crime
(B) Possession of any records relating to money laundering
(C) Possession of records relating to RERA compliance by Winner
(D) Possession of any proceeds of crime involved in money laundering (2 Marks)
2.8 Ms. Anusha Nair brought gold jewellery worth INR 15 lakhs from D ubai through the
green channel. Is this an offence under the PMLA 2002 ?
(A) Yes, because she came through the green channel and evaded duty of customs.
(B) No, whilst it is an offence, it is not actionable under the PMLA 2002.
(C) No, she did not pay any cash for the purchase.
(D) Yes, since purchase of gold from gulf countries requires specific consent as per the
agreement entered with foreign countries as per Section 56 of PMLA 2002. (2 Marks)
2.9 As per RERA 2016, what is the minimum amount that Winner was required to deposit
in the escrow account ?
(A) INR 50 crores
(B) INR 56 crores
(C) INR 54 crores

© The Institute of Chartered Accountants of India


Page 157
20 FINAL (NEW) EXAMINATION: MAY 2019

(D) INR 58.8 crores (2 Marks)


2.10 Of the below, which of the practices are not common schemes of money laundering?
(A) Bribery and Corruption
(B) False declarations under Customs act
(C) Usage of false trade Marks/copyrights
(D) Possession of foreign currency over and above permitted limit (2 Marks)
2.11 Answer the following questions in the context of the prov isions relating to the Real
Estate (Regulation & Development) Act, 2016 (RERA 2016).
(i) Examine the appropriateness of the conditions mentioned in the agreement to sale,
in the context of the provisions of RERA 2016. (4 Marks)
(ii) What are the provisions in RERA 2016 relating to the changes in design of the
construction from the sanctioned plans ? Analyse if the changes made by Winner are
appropriate in this context. (4 Marks)
(iii) What would be your advice if the customers of Winner reach out to you for your views
with regard to the validity of the explanations provided by Mr. Vijay Nair on the
reduction of carpet area? (3 Marks)
(iv) Evaluate the statements made by Mr. Vijay Nair in the meeting with the allottees on
30th March 2019 regarding the delay and the increase in labour costs in the context
of provisions of RERA 2016. (4 Marks)
2.12 Examine / advice regarding the below questions relating to the Prevention of Money
Laundering Act, 2002 (PMLA 2002).
(i) As a leading consultant on PMLA matters, the enforcement directorate has sought
your advice on identifying :
(a) the offences (b) the parties involved and (c) the punishment for the offence of
money laundering. (8 Marks)
(ii) The Bank, in which Winner holds its bank account, has reached out to you to
understand their obligations for maintaining and reporting of transactions under the
PMLA 2002. Advise. (4 Marks)
(iii) When the Enforcement Directorate proposed to take action against Mr. Vijay Nair
under the PMLA 2002, Mr. Vijay Nair contended that he was not a party to any of the
alleged offences and he was managing the real estate business of Winner only.

© The Institute of Chartered Accountants of India


Page 158
PAPER – 6D: ECONOMIC LAWS 21

Examine whether his statement is valid. What would be the position of t he nominee
director of the PE investor? (3 Marks)

Answer Case study 2


2.1 (C)
2.2 (A)
2.3 (B)
2.4 (D)
2.5 (C)
2.6 (B)
2.7 (C)
2.8 (B)
2.9 (B)
2.10 (C)
2.11 (i) 1. Expected date of completion of construction- 31st March, 2020 -This
condition is valid.
2. Expected date of handover- 31 st May 2020, subject to a grace period of 4
months. -This condition is valid.
3. According to Section 13, a promoter shall not accept a sum more than ten
per cent of the cost of the apartment, plot, or building as the case may be,
as an advance payment or an application fee, from a person without first
entering into a written agreement for sale with such person and register the
said agreement for sale, under any law for the time being in force.
Hence, the condition in the agreement for sale for booking advance amount
to be paid prior to entering into agreement to sale @20% of total cost of
apartment is not valid.
4. Section 2(n) of RERA, 2016 defines ‘common areas’ to include ‘open
parking areas’, thus open parking areas cannot be sold to the allottees.
Hence, the condition in the agreement for sale for open car parking cost
` 2,00,000 is not valid.
5. As per section 19(7) of RERA, 2016, the allottee shall be liable to pay
interest, at such rate as may be prescribed, for any delay in payment
towards any amount or charges to be paid.
Hence, the condition about any delay in payment of dues by the allottees
will be liable for interest on such delayed payments, is valid.

© The Institute of Chartered Accountants of India


Page 159
22 FINAL (NEW) EXAMINATION: MAY 2019

6. The allottee shall be entitled to claim the refund of amount paid along with
interest at such rate as may be prescribed and compensation in the manner
as provided under this Act, from the promoter, if the promoter fails to
comply or is unable to give possession of the apartment, plot or building,
as the case may be, in accordance with the terms of agreement for sale or
due to discontinuance of his business as a developer on account of
suspension or revocation of his registration under the provisions of this Act
or the rules or regulations made thereunder.
Hence, the condition for return of booking amount shall not be entertained
for any reason whatsoever is not valid.
7. The builder has to provide five-year warranty for any structural defects in
the building. They are liable to pay equal rate of interest in case of default
or delays as home buyers.
Hence, the condition that Winner Group shall be liable for any deficiency
in quality of construction for a period of 3 years from the date handing over
the apartments is not valid.
(ii) Adherence to sanctioned plans and project specifications by the promoter
(Section 14)
(1) The proposed project shall be developed and completed by the promoter
in accordance with the sanctioned plans, layout plans and specifications
as approved by the competent authorities.
(2) Notwithstanding anything contained in any law, contract or agreement,
after the sanctioned plans, layout plans and specifications and the nature
of the fixtures, fittings, amenities and common areas, of the apartment, plot
or building, as the case may be, as approved by the competent authority,
are disclosed or furnished to the person who agree to take one or more of
the said apartment, plot or building, as the case may be, the promoter shall
not make—
(i) any additions and alterations in the sanctioned plans, layout plans
and specifications and the nature of fixtures, fittings and amenities
described therein in respect of the apartment, plot or building, as the
case may be, which are agreed to be taken, without the previous
consent of that person.
Provided that the promoter may make such minor additions or
alterations as may be required by the allottee, or such minor changes
or alterations as may be necessary due to architectural and structural
reasons duly recommended and verified by an authorised Architect or
Engineer after proper declaration and intimation to the allottee.
Explanation.—For the purpose of this clause, "minor additions or

© The Institute of Chartered Accountants of India


Page 160
PAPER – 6D: ECONOMIC LAWS 23

alterations" excludes structural change including an addition to the


area or change in height, or the removal of part of a building, or any
change to the structure, such as the construction or removal or cutting
into of any wall or a part of a wall, partition, column, beam, joist, floor
including a mezzanine floor or other support, or a change to or closing
of any required means of access ingress or egress or a change to the
fixtures or equipment, etc.
(ii) any other alterations or additions in the sanctioned plans, layout plans
and specifications of the buildings or the common areas within the
project without the previous written consent of at least two-thirds of
the allottees, other than the promoter, who have agreed to take
apartments in such building.
Explanation.—For the purpose of this clause, the allottees,
irrespective of the number of apartments or plots, as the case may
be, booked by him or booked in the name of his family, or in the case
of other persons such as companies or firms or any association of
individuals, etc., by whatever name called, booked in its name or
booked in the name of its associated entities or related enterprises,
shall be considered as one allottee only.
(3) In case any structural defect or any other defect in workmanship, quality or
provision of services or any other obligations of the promoter as per the
agreement for sale relating to such development is brought to the notice of
the promoter within a period of five years by the allottee from the date of
handing over possession, it shall be the duty of the promoter to rectify such
defects without further charge, within thirty days, and in the event of
promoter's failure to rectify such defects within such time, the aggrieved
allottees shall be entitled to receive appropriate compensation in the
manner as provided under this Act.
In the instant case, the proposal of Winner for reduction of the size of
swimming pool and the same to be compensated by providing a Jacuzzi
and spa inside the club house was accepted by majority (45 of the 80) of
the allottees and accordingly, Winner proceeded with the construction
based on the amended plan.
According to the above provisions, the promoter shall not make any other
alterations or additions in the sanctioned plans, layout plans and
specifications of the buildings or the common areas within the project
without the previous written consent of at least two-thirds of the allottees,
other than the promoter, who have agreed to take apartments in such
building.
Hence, approval by majority (45 of 80) is not valid.

© The Institute of Chartered Accountants of India


Page 161
24 FINAL (NEW) EXAMINATION: MAY 2019

(iii) As per section 2(k) of the Real Estate (Regulation & Development)Act, 2016
"carpet area" means the net usable floor area of an apartment, excluding the
area covered by the external walls, areas under services shafts, exclusive
balcony or verandah area and exclusive open terrace area, but includes the area
covered by the internal partition walls of the apartment.
Accordingly, Sale of property will be on carpet area, not super built area.
Therefore, the homebuyer will have to pay only for the carpet area, that is the
area within walls, and the builder cannot charge for the super built-up area.
Therefore, the explanations provided by Mr. Vijay Nair on the reduction of the
carpet area was invalid. So, home buyers/ customers are liable to pay only for
the carpet area, that is the area within walls.
(iv) As given in the question that on 30th March, 2019, meeting was organized by
the company with all the allottees. During the meeting, Mr. Vijay Nair provided
a status update on the project and of the construction activities to be completed
and the other information mentioned in the sale agreement. As per the Section
11 of Real Estate (Regulation & Development) Act, 2016, it is the duty of the
promoter, to alter a project plan, structural design and specifications of the plot,
apartment or a building, the promoter has to get the consent of minimum two-
third allottees (buyers) after the necessary disclosures.
Since in the given case no approval of 2/3rd of the allottees was taken w.r.t. to
delay and the increase in labour costs i.e., as to the updation of the status of the
said project. This act of Mr. Vijay Nair is not in compliance with the Law.
2.12 (i) (a) Offences: The term “scheduled offence" has been defined in clause (y) of sub-
section (1) of section 2. It means –
(a) the offences specified under Part A of the Schedule; or
(b) the offences specified under Part B of the Schedule if the total value
involved in such offences is one crore rupees or more; or
(c) The offences specified under Part C of the Schedule.
The Schedule to the Act gives a list of all the above offences. The Schedule is
divided into three parts- Part A, Part B and Part C, which are given in Annexure
to the Chapter.
(b) The parties involved: Clause (p) of sub section (1) of section 2 provides that
"money-laundering" has the meaning assigned to it in section 3. Moving to
section 3, it is observed that whosoever directly or indirectly attempts to indulge
or knowingly assists or knowingly is a party or is actually involved in any process
or activity connected with the proceeds of crime including its concealment,
possession, acquisition or use and projecting or claiming it as untainted property
shall be guilty of offence of money laundering.

© The Institute of Chartered Accountants of India


Page 162
PAPER – 6D: ECONOMIC LAWS 25

(c) The punishment for the offence of money laundering: Section 4 provides for the
Punishment for Money-Laundering - Whoever commits the offence of money-
laundering shall be punishable with rigorous imprisonment for a term which shall
not be less than three years but which may extend to seven years and shall also
be liable to fine.
But where the proceeds of crime involved in money-laundering relate to any
offence specified under paragraph 2 of Part A of the Schedule (i.e. Offences
under the Narcotic Drugs and Psychotropic Substances Act, 1985), the
maximum punishment may extend to ten years instead of seven years.
(ii) Obligation of Banking Companies, Financial Institutions and Intermediaries
Reporting entity to maintain records
Section 12 provides for the obligation of Banking Companies, Financial Institutions
and Intermediaries.
1. Maintenance of records: According to sub-section (1), every reporting entity
shall –
(a) maintain a record of all transactions, including information relating to
transactions covered under clause (b), in such manner as to enable it to
reconstruct individual transactions;
(b) furnish to the Director within such time as may be prescribed, information
relating to such transactions, whether attempted or executed, the nature
and value of which may be prescribed;
(c) verify the identity of its clients in such manner and subject to such
conditions, as may be prescribed;
(d) identify the beneficial owner, if any, of such of its clients, as may be
prescribed;
(e) maintain record of documents evidencing identity of its clients and
beneficial owners as well as account files and business correspondence
relating to its clients.
2. Confidentiality: Every information maintained, furnished or verified, save as
otherwise provided under any law for the time being in force shall be kept
confidential.
3. Maintenance of records (for clause a): The records referred to in clause (a)
of sub-section (1) shall be maintained for a period of five years from the date of
transaction between a client and the reporting entity.
4. Maintenance of records (for clause e): The records referred to in clause (e)
of sub-section (1) shall be maintained for a period of five years after the business
relationship between a client and the reporting entity has ended or the account

© The Institute of Chartered Accountants of India


Page 163
26 FINAL (NEW) EXAMINATION: MAY 2019

has been closed, whichever is later.


5. Exemption by the Central Government: The Central Government may, by
notification, exempt any reporting entity or class of reporting entities from any
obligation under this chapter.
Access to information [Section 12A]
1. The Director may call for from any reporting entity any of the records referred to
in sub-section (1) of section 12 and any additional information as he considers
necessary for the purposes of this Act.
2. Every reporting entity shall furnish to the Director such information as may be
required by him under sub-section (1) within such time and in such manner as
he may specify.
3. Save as otherwise provided under any law for the time being in force, every
information sought by the Director under sub-section (1), shall be kept
confidential.
(iii) Offences by companies [Section 70]
1. Where a person committing a contravention of any of the provisions of this Ac t
or of any rule, direction or order made thereunder is a company, every person
who, at the time the contravention was committed, was in charge of and was
responsible to the company, for the conduct of the business of the company as
well as the company, shall be deemed to be guilty of the contravention and shall
be liable to be proceeded against and punished accordingly.
Nothing contained in this sub-section shall render any such person liable to
punishment if he proves that the contravention took place without his knowledge
or that he exercised all due diligence to prevent such contravention.
2. Notwithstanding anything contained in sub-section (1), where a contravention of
any of the provisions of this Act or of any rule, direction or order made
thereunder has been committed by a company and it is proved that the
contravention has taken place with the consent or connivance of, or is
attributable to any neglect on the part of any director, manager, secretary or
other officer of any company, such director, manager, secretary or other officer
shall also be deemed to be guilty of the contravention and shall be liable to be
proceeded against and punished accordingly.
In the instant case, Mr. Vijay Nair contended that he was not a party to any of
the alleged offences and he was managing the real estate business of Winner
only.
His statement is not valid on the grounds of section 70 of the PMLA 2002.
The position of the nominee director of the PE investor would be same as of

© The Institute of Chartered Accountants of India


Page 164
PAPER – 6D: ECONOMIC LAWS 27

Mr. Vijay Nair and he shall be deemed to be guilty of the contravention and shall
be liable to be proceeded against and punished accordingly.
CASE STUDY 3
(Part-A)
The appellant "M/s Transmission Corporation of Andhra Pradesh Limited" is a successor of
Andhra Pradesh State Electricity Board (for short, 'APSEB') and is in the activities relating to
transmission of electricity. It had awarded certain contracts to the respondent "M/s Equipment
Conductors & Cables Limited" herein for supply of goods and services. Some disputes arose
and the respondent initiated arbitration proceedings. As many as 82 claims were filed by the
respondent before Haryana Micro and Small Enterprises Facilitation Council (hereinafter
referred to as 'Arbitral Council'). These proceedings culminated into Award dated June 21st,
2010. The Arbitral Council came to the conclusion that the claims made on the basis of Invoice
Nos. 1 - 57 were barred by law of limitation and, therefore, no amount could be awarded against
the said claims. In respect of Invoice Nos. 58-82, the award was passed in favour of the
respondent. Against the aforesaid award rejecting claims in respect of Invoice Nos. 1 - 57 as
time barred, the respondent herein filed an application under Section 34 of the Arbitration and
Conciliation Act before the Additional District Judge, Chandigarh. The Additional District Judge
passed the order dated August 28, 2014 in the said application thereby remanding the case
back to the Arbitral Council for fresh decision. Against this order, the appellant filed the appeal
before the High Court of Punjab and Haryana at Chandigarh. This appeal was allowed by the
High Court by its order dated January 29, 2016 thereby setting aside the direction of the
Additional District Judge remanding the matter to Arbitral Council for fresh consideration.
The respondent herein filed execution petition for execution of judgment dated January 29, 2016
passed by the High Court of Punjab and Haryana as well as the award dated June 21, 2010
passed by the Arbitral Council. In so far as award of Arbitral Council is concerned, as noted
above the respondent's claim pertaining to Invoice Nos. 58 - 82 was allowed and the execution
thereof was sought. The respondent, however, filed another execution petition seeking
execution of amount in respect of Invoice Nos. 1 - 57 also. This application was entertained and
both the petitions were directed to be dealt with simultaneously vide orders dated August 17,
2016. The High Court vide its order dated November 08,2016 allowed the said Revision Petition
holding that there was no award in respect of claim towards Invoice Nos. 1 - 57 and, therefore,
it was not permissible for the respondent to seek the execution. When the things rested at that,
the respondent approached the NCLT by means of a Company Petition under Section 9 of IBC,
2016 read with Rule 6 of Insolvency and Bankruptcy (AAA) Rules, 2016. In this petition, the
respondent stated that it had served demand notice dated October 14, 2017 upon the appellant
under the provisions of the IBC, thereby claiming the amount of `45,69,31,233/- which was not
paid by the appellant. As mentioned above, this petition was dismissed by the NCLT filed under
section 9 of IBC vide its order dated April 09, 2018 being non maintainable on account of
existence of a dispute between the parties and this assertion of the NCLT is based on the fact
that these very claims of the respondent were subject matter of arbitration and the award was
passed rejecting these claims as time barred. Against this order, the respondent has filed appeal

© The Institute of Chartered Accountants of India


Page 165
28 FINAL (NEW) EXAMINATION: MAY 2019

before the NCLAT in which impugned orders dated September 04,2018 have been passed. The
Honourable NCLAT passed an order stating "Prima facie case has been made out by the
Appellant in view of the part decree awarded by the competent court under Section 34 of the
Arbitration and 'Conciliation Act, 1996. However, taking into consideration the fact that if appeal
is allowed and Corporate Insolvency Resolution Process is initiated against the Respondent -
"Transmission Corporation of Andhra Pradesh Ltd. ", the government undertaking may face
trouble. Therefore, by way of last chance we grant one opportunity to respondents to settle the
claim with the appellant, failing which this Appellate Tribunal may pass appropriate order on
merit."
This very order of the National Company Law Appellate Tribunal, (for short, 'NCLAT) dated
September 04, 2018 is the subject matter of challenge before the Honourable Supreme Court
by the appellant M/s Transmission Corporation of Andhra Pradesh Limited and prays that the
same be reversed as there exists a Dispute and the application under IBC cannot be accepted.
(Part-B)
One Shri Rajendra Singh (Informant') filed an information under Section 19(1)(a) of the
Competition Act, 2002 (the 'Act') against Ghaziabad Development Authority (‘OP' /'GDA')
alleging contravention of the provisions of Section 4 of the Act. As per the information, the
Informant is an allottee of a flat under the Adarsh Vihar residential housing scheme for the
Economically Weaker Sections (EWS) ('Scheme') being developed by the OP in Ghaziabad, U.
P. in 2008. It is informed that OP is constituted under Section 4 of the Urban Planning and
Development Act, 1973 of Uttar Pradesh and is, inter alia, engaged in the activity of
development and sale of real estate in Ghaziabad, U. P.
It is further stated that the OP had conducted a lottery draw for allotment of EWS flats under the
aforesaid scheme. On being successful in the said lottery draw, the Informant was allotted a flat
bearing no. A-1/222 and accordingly, an allotment letter dated 04.05.2009 was issued in favour
of the Informant mentioning the final price of the flat as ` 2,00,000 and other conditions relating
to payment plan, date of giving possession, penal interest in case of delay in the payment of the
balance amount etc. As per the condition of the scheme, the Informant paid
`20,000 to the OP as registration amount constituting 10% of the total price of the said house.
It is averred by the Informant that on 27.11.2015, the OP issued a letter to all the allottees of
the aforesaid scheme asking them to pay `7,00,000 as sale price of the flats allotted to them
failing which their allotment would stand cancelled. It is alleged that the OP has arbitrarily
increased the sale price of the said flat to ` 7,00,000 from `2,00,000 which was mentioned in
the allotment letter dated 04.05.2009. As per the Informant, the OP has indulged in unfair and
arbitrary practices and has misused its dominant position in the market. It is further averred that
the OP has indulged in the said practice even after knowing that the allottees of the scheme
belong to the weaker sections of the society and they are not in a position to challenge the OP
for its unfair and arbitrary conduct. Further, it is stated that the allottees of the said scheme are
dependent upon the OP for the residential flats under the said scheme. The Informant has

© The Institute of Chartered Accountants of India


Page 166
PAPER – 6D: ECONOMIC LAWS 29

averred that the alleged conduct of OP is in contravention of the provisions of Section 4(2)(a)
of the Act.
Based on the above submissions, the Informant has prayed before the Commission to initiate
an inquiry against the OP under the provisions of the Act, set aside the impugned letter dated
27.11.2015 of the OP demanding `7,00,000 for the aforementioned flat, and direct the OP to
deliver possession of the flat to the informant under the said scheme at the price of `2,00,000
per flat. Besides hearing the parties on 27.12.2016, the Commission has perused the
information available on record and the documents submitted by the OP. From the facts of the
case, it appears that the grievance of the Informant relates to the letter dated 27.11.2015 of the
OP demanding a higher price of `7,00,000 for a EWS flat allotted to the Informant under the
aforesaid scheme as compared with the price of `2,00,000 as declared in the scheme's initial
brochure and intimated to the Informant vide allotment letter dated 04.05.2009. It is the case of
the Informant that the OP has abused its dominant position by arbitrarily increasing the price of
the said flat in contravention of the provisions of Section 4 of the Act.
The Commission observes that GDA is established under Section 4 of the Urban Planning and
Development Act, 1973 of the State of Uttar Pradesh. It has a common seal with power to
acquire, hold or dispose of both movable and immovable properties. The Urban Planning and
Development Act, 1973 of Uttar Pradesh empower GDA to pursue activities for promoting and
securing development of Ghaziabad in a planned manner. GDA has the power to acquire, hold,
manage and dispose of land and other properties in Ghaziabad and to carry out building,
engineering, mining and other operations, etc. Further, GDA is, inter alia, engaged in the
activities of development and sale of buildings, flats, complexes etc. for residential, commercial,
institutional and other purposes and with regard to the relevant geographic market. The
Commission is of the view that the geographic area of Ghaziabad district of the State of Uttar
Pradesh exhibits homogeneous and distinct market conditions for the development and sale of
low cost residential flats under affordable housing schemes for EWS and can be distinguished
from the conditions of competition prevailing in other adjacent areas of Ghaziabad such as Delhi,
Noida, etc. It may be noted that a consumer intending to buy a low cost residential flat under
affordable housing scheme for EWS in Ghaziabad may not prefer to purchase the same in other
adjacent areas of Ghaziabad because of factors such as difference in regulatory authorities (and
hence, different rules, regulation and taxes), distance to locations frequently commuted,
regional or personal preferences, transport connectivity etc.
Simultaneously, the Competition Commission of India (CCI) received a complaint from the Tamil
Nadu State Government alleging that two companies, M/s Sun Limited, a company engaged in
the business of manufacturing solar panels, and M/s Shine Limited, a company engaged in the
sale, installation and maintenance of solar energy generation plants, have entered into an
informal agreement to limit or control the production, supply and marketing of the products to
ensure maximum price realisation. M/s Sun Limited sells its manufactured panels on an
exclusive basis to M/s Shine Limited, which is India's largest solar power generation company
supplying solar plants to more than 60% of the current market.

© The Institute of Chartered Accountants of India


Page 167
30 FINAL (NEW) EXAMINATION: MAY 2019

Therefore, it is the case of the Tamil Nadu State Government that the agreement between M/s
Sun Limited and M/s Shine Limited is anti-competitive and has an adverse effect on competition
since the entities have abused their dominance in the market.
Answer the following questions:
3.1 Which of the following are not duties of the Competition Commission of India?
(A) To promote and sustain competition in markets in India.
(B) To protect the interests of consumers.
(C) To ensure freedom of trade carried on by Indian suppliers in global market.
(D) To eliminate practices having adverse effect on competition. (2 Marks)
3.2 Notwithstanding anything contained in sub-regulation (2), the Commission may, after
recording reasons, invalidate a notice filed under regulation 5 or regulation 8 of The
Competition Commission of India (Procedure in regard to the transaction of business
relating to combinations) Regulations, 2011 as amended when it comes to the knowledge
of the Commission that such notice is not valid as per sub-regulation (1) and, in that case,
the Secretary shall convey the decision of the Commission to the parties to the combination
within _____________.
(A) Seven days of such decision of the Commission.
(B) Fourteen days of such decision of the Commission.
(C) Seven working days of such decision of the Commission.
(D) Fourteen working days of such decision of the Commission. (2 Marks)
3.3 Operate independently of competitive forces prevailing in the relevant market is-------------
--- component.
(A) Abuse of Dominance
(B) Anti-Competition agreements
(C) Combinations Regulation
(D) Competition Advocacy (2 Marks)
3.4 Which of the following are not functions of Insolvency Professional Agencies (IPAs)?
(A) Monitoring, Inspecting and Investigating members.
(B) Recommending Insolvency Professionals to Committee of Creditors.
(C) Drafting detailed standards and code of conduct for insolvency professionals.

© The Institute of Chartered Accountants of India


Page 168
PAPER – 6D: ECONOMIC LAWS 31

(D) Addressing grievances, hearing complaints and taking suitable action. (2 Marks)
3.5 Following are the liabilities of M/s A Limited, which is under insolvency process under IBC
2016.
(i) Loan from Bank - INR 100 crores.
(ii) Secured Debentures issued to M/s B Limited - INR 20 crores.
(iii) Trade Payable (10 creditors, including B Ltd., whose outstanding is `2 crores) - INR
14 crores.
(iv) Amounts payable to workmen - INR 4 crores. (2 Marks)
Calculate the voting share of M/s B Limited in the Committee of Creditors.
(A) 15.9420%
(B) 16.6667%
(C) 16.4179%
(D) 16.1290% (2 Marks)
3.6 The liquidation process relating to corporate debtors under IBC 2016 will not be initiated
under which of the following circumstances ?
(A) The Committee of Creditors do not approve a resolution plan within 180 days.
(B) The NCLT rejects the resolution plan submitted to it on technical grounds.
(C) The Committee of Creditors resolve to liquidate the debtor with a majority (> 50%).
(D) The debtor contravenes the agreed resolution plan and an affected person makes an
application to the NCLT to liquidate the debtor. (2 Marks)
3.7 The liquidator of M/s Wrongway has sought your help in prioritizing the claims against M/s
Wrongway, as per IBC 2016:
(1) Costs payable to liquidator and resolution professional.
(2) Property tax payable to Government of Goa.
(3) Salary payable to the Finance team for past 6 months.
(4) Amounts payable to M/s Dhara Bank towards secured loans, where security
was relinquished.
(5) Amounts payable to Holding company of M/s Wrong way for Royalty fees.
(A) (1), (2), (4), (3), (5)

© The Institute of Chartered Accountants of India


Page 169
32 FINAL (NEW) EXAMINATION: MAY 2019

(B) (2), (1), (4), (5), (3)


(C) (1), (4), (2), (3), (5)
(D) None of the above (2 Marks)
3.8 Whether an operational creditor can assign or legally transfer any operational debt to a
financial creditor ?
(A) Yes. However, the transferee shall be considered as an operational creditor to such
extent of transfer.
(B) Yes but the transferee shall be considered as a financial creditor in relation to such
transfer.
(C) No. An operational creditor cannot assign or legally transfer any operational debt to
a financial creditor.
(D) No. An operational creditor can assign or legally transfer an operational debt only to
an operational creditor. (2 Marks)
3.9 Which of the following agenda items should be taken up in the first meeting of committee
of creditors (COC) ___________.
(A) Appointment of interim resolution professional as insolvency professional or
replacement of the interim resolution professional by another resolution professional
(B) Preparation of draft resolution plan.
(C) Discussion of the status of the corporate debtor as on the present date and the road
map ahead.
(D) Collection of information on corporate debtor from independent sources (2 Marks)
3.10 What is quorum in case of meeting of committee of creditors (CoC)?
(A) Members of the Committee representing at least 33% of the voting rights present
either in person or video conference or other audio visual means.
(B) Members of the Committee representing at least 50% of the voting rights are present
either in person or proxy.
(C) Members of the Committee representing at least 50% of the voting rights are present
either in person or video conference or other audio visual means or proxy.
(D) Members of the Committee representing at least 66% of the committee present in
person or proxy. (2 Marks)

© The Institute of Chartered Accountants of India


Page 170
PAPER – 6D: ECONOMIC LAWS 33

3.11 What is to be construed as a "Dispute" under the Insolvency & Bankruptcy Code, 2016 ?
State its significance for the maintainability of an application filed under section 9 of the
Code.
In the given case study whether the appellant M/s Transmission Corporation of Andhra
Pradesh Limited will succeed in its appeal ? Decide. (11 Marks)
3.12 (i) Examine the provisions of the Competition Act, 2002:
(a) Decide whether the agreement between Sun Limited and Shine Limited is
covered under the scope of the Act with reasons. Also, clarify the nature of the
agreement based on facts provided. (4 Marks)
(b) What factors shall the CCI consider while evaluating the views of the
Government of Tamil Nadu? (6 Marks)
(c) What orders can the CCI pass on completion of the inquiry? (4 Marks)
(ii) In the given case study, decide with reasons whether Rajendra Singh (Informant) will
succeed against the Opposite Party (OP) for alleged violation of Section 4(2)(a) of
the Competition Act, 2002 ? (6 Marks)
Answer Case Study 3
3.1 (C)
3.2 (C) Note: Prior to Amendment in the combination Regulation, it was Option A.
3.3 (A)
3.4 (B)
3.5 (B)
3.6 (C)
3.7 (D)
3.8 (A)
3.9 (A)
3.10 (A)
3.11 Meaning of dispute: As per section 5(6) of the Insolvency and Bankruptcy Code, 2016 the
word “Dispute” includes a suit or arbitration proceedings relating to—
(a) the existence of the amount of debt;
(b) the quality of goods or service; or
(c) the breach of a representation or warranty;
Significance of “dispute” for filing of an application by operational creditor under
section 9 of the IBC: If there is any dispute about debt, the corporate debtor is required to

© The Institute of Chartered Accountants of India


Page 171
34 FINAL (NEW) EXAMINATION: MAY 2019

reply within ten days of receipt of copy of invoice, existence of a dispute, or record of the
pendency of the suit or arbitration proceedings filed before the receipt of such notice or invoice
in relation to such dispute [section 8(2)(a) of Insolvency & Bankruptcy Code, 2016].
After the expiry of the period of ten days from the date of delivery of the notice or invoice
demanding payment, if the operational creditor does not receive payment from the
corporate debtor or notice of the dispute, the operational creditor may file an application
before the Adjudicating Authority for initiating a corporate insolvency resolution process.
The operational creditor shall, along with the application furnish the relevant documents,
containing an affidavit to the effect that there is no notice given by the c orporate debtor
relating to a dispute of the unpaid operational debt. The Adjudicating Authority shall, within
fourteen days of the receipt of the application, by an order admit the application and
communicate such decision to the operational creditor and the corporate debtor if no notice
of dispute has been received by the operational creditor or there is no record of dispute in
the information utility.
In the given case study, appellant M/s Transmission Corporation, filed an appeal against
the order of NCLAT before the Supreme court on the ground of existence of a dispute, so
the application under IBC cannot be accepted and so order passed in the favour of
respondent (M/s Equipment Conductors & Cables Ltd.) to be revered.
As per the facts given in the case study, respondent filed petition seeking execution of
amount in respect of Invoice Nos. 1-57. Vide order dated Nov, 8, 2016 high court held that
this revision petition holding that there was no award in respect of claim towards Invoice
Nos. 1- 57 and therefore, it was not permissible for the respondent to seek the execution.
Against this order, respondent approached NCLT on the ground that it has served demand
notice dated October 2014, 2017. This application was dismissed by the NCLT Vide Order
April 9, 2018. Against this order of NCLT, respondent filed appeal before NCLAT. NCLAT
challenged the orders and passed an order dated 4 th September 2018.
Cause of action arised when, high court rejected on the execution of the petition which was
holding that there was no award in respect of claim towards Invoice Nos. 1- 57, was
passed, which means that still the dispute is pending. Against this order, respondent,
served demand notice dated October 14, 2017.
As per the Code, if there is dispute about claim of debt between parties prior to issue of
demand notice by operational creditor, application cannot be admitted. On the basis of this
ground, Appellant challenged the subject matter of the order passed by NCLAT dated 4 th
September 2018 before Supreme Court.
In the judicial pronouncement, it was held that, application by operational creditor to initiate
insolvency process was accepted when it was found that there was no existing dispute
prior to date of demand notice and dispute raised after receipt of demand notice was not
genuine [Badjate Stock v. Snowblue Trexim (2018) 145 SCL 441 = 89 taxmann.com 64
(NCLT)].

© The Institute of Chartered Accountants of India


Page 172
PAPER – 6D: ECONOMIC LAWS 35

Also, If appeal has been filed under section 34 of Arbitration Act, the proceedings are
pending as appeal is continuation of the adjudication proceedings. Hence, application for
insolvency resolution is not maintainable. [CG Power & Industrial Solutions Ltd. v. ACC
Ltd. (2018) 91 taxmann.com 363 (NCLT)].
Therefore, in the light of the given facts and circumstances, Appellant M/s Transmission
Corporation of Andhra Pradesh Limited will succeed in its appeal.
3.12 (i) (a) Anti-Competitive Agreements [Section 3]:
It shall not be lawful for any enterprise or association of enterprises or person
or association of persons to 'enter' into an agreement in respect of production,
supply, storage, distribution, acquisition or control of goods or provision of
services, which causes or is likely to cause an appreciable adverse effect on
competition within India. All such agreements entered into in contravention of
the aforesaid prohibition shall be void.
Any agreement entered into between enterprises or associations of enterprises
or persons or associations of persons or between any person and enterprise or
practice carried on, or decision taken by, any association of enterprises or
association of persons, including cartels, engaged in identical or similar trade of
goods or provision of services, shall be presumed to have an appreciable
adverse effect on competition, which—
(a) directly or indirectly determines purchase or sale prices;
(b) limits or controls production, supply, markets, technical development,
investment or provision of services;
(c) shares the market or source of production or provision of services by way
of allocation of geographical area of market, or type of goods or services,
or number of customers in the market or any other similar way;
(d) directly or indirectly results in bid rigging or collusive bidding.
In the instant case, M/s Sun Limited and M/s Shine Limited have entered into an
informal agreement to limit or control the production, supply and marketing of
the products to ensure maximum price realization. M/s Sun Ltd. sells its
manufactured panels on an exclusive basis to M/s Shine Ltd. which is India’s
largest solar power generation company supplying solar plants to more than
60% of the current market.
The above agreement is covered under the scope of the Competition Act, 2002
as it is an Anti-competitive agreement under section 3.
(b) Dominant position of enterprise: The Commission shall, while inquiring
whether an enterprise enjoys a dominant position or not, have due regard to all
or any of the following factors, namely:—
(a) market share of the enterprise;

© The Institute of Chartered Accountants of India


Page 173
36 FINAL (NEW) EXAMINATION: MAY 2019

(b) size and resources of the enterprise;


(c) size and importance of the competitors;
(d) economic power of the enterprise including commercial advantages over
competitors;
(e) vertical integration of the enterprises or sale or service network of such
enterprises;
(f) dependence of consumers on the enterprise;
(g) monopoly or dominant position whether acquired as a result of any statute
or by virtue of being a Government company or a public sector undertaking
or otherwise;
(h) entry barriers including barriers such as regulatory barriers, financial risk, high
capital cost of entry, marketing entry barriers, technical entry barriers,
economies of scale, high cost of substitutable goods or service for consumers;
(i) countervailing buying power;
(j) market structure and size of market;
(k) social obligations and social costs;
(l) relative advantage, by way of the contribution to the economic
development, by the enterprise enjoying a dominant position having or
likely to have an appreciable adverse effect on competition;
(m) any other factor which the Commission may consider relevant for the inquiry.
(c) Orders by Commission after inquiry into agreements or abuse of dominant
position [Section 27]
Where after inquiry the Commission finds that any agreement referred to in
section 3 or action of an enterprise in a dominant position, is in contravention of
section 3 or section 4, as the case may be, it may pass all or any of the following
orders, namely:—
(a) direct any enterprise or association of enterprises or person or association
of persons, as the case may be, involved in such agreement, or abuse of
dominant position, to discontinue and not to re-enter such agreement or
discontinue such abuse of dominant position, as the case may be;
(b) impose such penalty, as it may deem fit which shall be not more than ten
per cent. of the average of the turnover for the last three preceding financial
years, upon each of such person or enterprises which are parties to such
agreements or abuse.
In case any agreement referred to in section 3 has been entered into by a
cartel, the Commission may impose upon each producer, seller, distributor,
trader or service provider included in that cartel, a penalty of up to three

© The Institute of Chartered Accountants of India


Page 174
PAPER – 6D: ECONOMIC LAWS 37

times of its profit for each year of the continuance of such agreement or
ten per cent. of its turnover for each year of the continuance of such
agreement, whichever is higher.
(c) Omitted
(d) direct that the agreements shall stand modified to the extent and in the
manner as may be specified in the order by the Commission;
(e) direct the enterprises concerned to abide by such other orders as the
Commission may pass and comply with the directions, including payment
of costs, if any:
(f) Omitted
(g) pass such other order or issue such directions as it may deem fit.
While passing orders under this section, if the Commission comes to a finding,
that an enterprise in contravention to section 3 or section 4 of the Act is a
member of a group as defined in clause (b) of the Explanation to section 5 of
the Act, and other members of such a group are also responsible for, or have
contributed to, such a contravention, then it may pass orders, under this section,
against such members of the group.
(ii) Section 4(2)(a) of the Competition Act, 2002 says there shall be an abuse of
dominant position under sub-section (1), if an enterprise or a group,—
(a) directly or indirectly, imposes unfair or discriminatory—
(i) condition in purchase or sale of goods or services; or
(ii) price in purchase or sale (including predatory price) of goods or service; or
Explanation.—For the purposes of this clause, the unfair or discriminatory
condition in purchase or sale of goods or services referred to in sub-clause (i)
and unfair or discriminatory price in purchase or sale of goods (including
predatory price) or service referred to in sub-clause (ii) shall not include such
discriminatory conditions or prices which may be adopted to meet the
competition.
As Opposite Party (OP) has increased the price of the flat allotted to Rajendra
Singh from ` 2,00,000 to ` 7,00,000. OP has been indulged in unfair and
arbitrary practices and has misused its dominant position in the market.
Hence, OP has violated section 4(2)(a) of the Competition Act, 2002.

© The Institute of Chartered Accountants of India


Page 175
DISCLAIMER

This Suggested Answers do not constitute the basis for evaluation of the student’s answers in

the examination. The answers are prepared by the Faculty of the Board of Studies with a view

to assist the students in their education. While due care is taken in preparation of the answers,

if any errors or omissions are noticed, the same may be brought to the attention of the Director

of Studies. The Council of the Institute is not in anyway responsible for the correctness or

otherwise of the answers published herein.

Further, in the Elective Papers which are Case Study based, the solutions have been worked

out on the basis of certain assumptions/views derived from the facts given in the question or

language used in the question. It may be possible to work out the solution to the case studies

in a different manner based on the assumption made or view taken.

© The Institute of Chartered Accountants of India


Page 176
2 FINAL (NEW) EXAMINATION: NOVEMBER, 2018

Case Study 1
Mr. Sharp was appointed as whole time member of the Competition Commission of India in
2015 and is presently a whole time member. Before joining Competition Commission of India,
during 2000-2014, he was acting as a trustee of several charitable trusts. For his contribution
towards the society, he was awarded several times by the State Government. Prior to that
during 1995-1999, he was acting as a managing director of 'Poor' Ltd., a Public Limited
Company engaged in the commercial real estate and was overall in-charge of finance and
sales function. During 2005, based on complaints filed by foreign investors, investigation into
the affairs of the company was initiated by the SFIO and CBI. The Report of CBI and SFIO
issued in October 2017 has revealed that affairs of the company were not managed in the
interest of the company during 1998-2005 and has resulted into financial loss of `400 Cr to
the shareholders and the Government. Considering the outcome of the report, Central
Government has issued an order of expulsion of Mr. Sharp from the post of whole time
member of the Competition Commission of India with immediate effect. He is in double mind to
challenge the order keeping in view the grounds for disqualification and the fact that
enactment of the Competition Act, 2002 is to provide an establishment of a Commission with
certain objectives or join back 'Poor' Ltd. 'Poor' Ltd. has been a party to a proceeding before
the Commission on the following issues:
(a) Whether a person who is purchasing goods for resale can also be considered as a
consumer?
(b) Whether all agreements which causes or is likely to cause an appreciable adverse effect
on competition in India, entered into in contravention shall be void?
(c) Whether the Commission also has powers to enquire into the acts taking place outside
India?
Poor', Ltd., now is in default in repayment of mainly on account of the General slowdown in
construction activities resulting in low capacity utilization and inadequate cash generation for
timely repayment of dues to all concerned. Repeated follow-up by the Financial Institutions
with the corporate debtor, 'Poor' Ltd., for submitting its specific plan of action for repayment of
dues did not evoke any meaningful response. Therefore, after a joint lenders' meeting, all the
financial creditors unanimously decided to apply under the provisions of the Insolvency and
Bankruptcy Code, 2016 to the National Company Law Tribunal (NCLT) for starting the process
of insolvency resolution in respect of corporate debtor, 'Poor' Ltd'. Financial Creditors filed an
application before NCLT which was admitted by NCLT on 20th May, 2018 and orders issued
for commencement of a moratorium period of 180 days, appointment of Mr. Ram, an Interim
Resolution Professional and for his making a public announcement inviting claims from all
concerned. With the advent of the public announcement the following creditors were identified:
(1) Financial debts owed to unsecured creditors (D1)- `10 crores.

© The Institute of Chartered Accountants of India


Page 177
PAPER – 6D: ECONOMIC LAWS 3

(2) Workmen's dues for the period of 24 months preceding the liquidation commencement
date (D2)- `30 crores.
(3) Debts owed to a secured creditor who has relinquished his security (D3)- `60 crores.
(4) Debts owed to the Central Government (D4)-34 Crores.
(5) Debts owed to a secured creditor for an amount unpaid following the enforcement of
security interest (D5)- `52 Crores.
Mr. Ram who has been appointed as Interim Resolution Professional wants to know the
functional responsibilities of Insolvency Professional Agency (IPA).
Mr. Ram, in the last financial year, has given some legal opinions on financial matters to 'Poor'
Ltd. and has charged fees.
Smart was the statutory auditor of the corporate debtor. Mr. Sharp is the whole time member
of the Competition Commission of India and has been identified as a relative of Mr. Dull,
present Managing Director of 'Poor' Ltd. Mr. Dull is not dear on the provisions of Insolvency
and Bankruptcy Code, 2016 (IBC) and requested Company Secretary to advise him on the
vital objectives which are intended to be achieved with the Code and also whether the
initiation of insolvency resolution process can be done by creditors only or by debtor also.
Mr. Dull also wants to know the specified procedure and term of appointment of an IRP. In
case, 'Poor' Ltd. approach NCLT before the financial creditors and decide to appoint Mr. Ram
as Interim Resolution Professional, advise Mr. Ram on the consent to be provided by him as
required by regulations.
Answer the following questions:
(1.1) Mr. Ram who has been appointed as the resolution professional can take the following
actions without the approval of the Committee of Creditors:
(A) Undertake transactions with Mr. Sharp.
(B) Make changes in the appointment of Smart, the statutory auditor.
(C) File applications for avoidance of preferential or undervalued transactions.
(D) Record any change in the ownership interest of 'Poor' Ltd. (2 Marks)
(1.2) The Adjudicating Authority has by an order declared moratorium period on the 'Poor' Ltd.
Vide the moratorium order, the following shall not be prohibited:
(A) the action to foreclose security interest created by the corporate debtor in respect of
its property.
(B) the institution of arbitration proceedings.

© The Institute of Chartered Accountants of India


Page 178
4 FINAL (NEW) EXAMINATION: NOVEMBER, 2018

(C) the recovery of any property by an owner or lessor where such property is occupied
by or in the possession of the corporate debtor.
(D) The supply of raw material essential for construction of commercial real estate from
its suppliers. (2 Marks)
(1.3) The NCLT rejected the resolution plan for want of compliance with the Insolvency and
Bankruptcy Code, accordingly the proceeds from the sale of liquidation shall be
distributed in the following order of priority :
(A) D2- D1-D3-D4 & D5 (ranked equally).
(B) D2 & D5 (ranked equally)-D3-D1-D4.
(C) D2 & D5 (ranked equally)-D1-D3-D4.
(D) D3 & D2 (ranked equally)-D1-D5 & D4 (ranked equally). (2 Marks)
(1.4) The NCLT rejected the resolution plan for want of compliance with the Insolvency and
Bankruptcy Code and proceeded to initiate liquidation proceedings. During the course of
liquidation, it was found that 'Poor' Ltd. had gifted some valuable assets of the Company
to another friendly company Soft Ltd. on 20th April, 2016 and D1 (unsecured financial
creditors) reported the transaction to the National Company Law Tribunal by way of an
application. The National Company Law Tribunal may pass an order:
(A) Rejecting the application of D1.
(B) Requiring the Insolvency and Bankruptcy Board to initiate disciplinary proceedings
against the liquidator.
(C) Require any person to pay sums in respect of benefits received by such person to
the liquidator.
(D) Require any person to submit relevant documents of transaction to Insolvency and
Bankruptcy board. (2 Marks)
(1.5) The management of the affairs of 'Poor' Ltd., the corporate debtor undergoing corporate
insolvency resolution process vests in the _______.
(A) Mr. Ram, Interim Resolution Professional
(B) Board of Directors
(C) Committee of Creditors
(D) Insolvency and Bankruptcy Board of India (2 Marks)
(1.6) Mr. Sharp cannot be removed from the Competition Commission of India by the Central
Government, if he:

© The Institute of Chartered Accountants of India


Page 179
PAPER – 6D: ECONOMIC LAWS 5

(A) has engaged at any time, in any paid employment.


(B) has become physically or mentally incapable of acting as a member.
(C) has been convicted of an offence which, in the opinion of the Central Government,
involves moral turpitude.
(D) is, or at any time has been, adjudged as an insolvent. (2 Marks)
(1.7) Mr. Sharp shall not for a period ________of years from the date on which cease to hold
office in the Competition Commission of India, accept any employment in, or be
connected with the management of administration of, any enterprise which has been a
party to a proceeding before the Competition Commission of India.
(A) 1
(B) 2
(C) 1
(D) None of the above (2 Marks)
(1.8) ‘Agreement’ under the Competition Act, 2002 includes any arrangement or understanding
or action in concert:
(A) if it is in writing only.
(B) if it is enforceable by legal proceedings only.
(C) if it is in writing and enforceable by legal proceedings only.
(D) If it is whether or not, in formal or writing or whether or not enforceable by legal
proceedings. (2 Marks)
(1.9) Any agreement under the Competition Act, 2002 shall be presumed to have an
appreciable adverse effect on competition, which:
(A) directly or indirectly determines purchase or sale prices.
(B) limits or controls production, supply, markets, technical development, investment or
provision of services.
(C) directly or indirectly results in bid rigging or collusive bidding.
(D) All of the above (2 Marks)
(1.10) If Central Government issues expulsion order to Mr. Sharp, the order:
(A) is valid from the date of his joining the Commission.
(B) is not valid but cannot be challenged.

© The Institute of Chartered Accountants of India


Page 180
6 FINAL (NEW) EXAMINATION: NOVEMBER, 2018

(C) is not valid and can be challenged.


(D) is valid on the basis of outcome of the report from the date of receipt of the order. (2 Marks)
(1.11) Answer the following based on the facts given in the question with reference to the
provisions of the Insolvency and Bankruptcy Code, 2016 (Code):
(A) Advise Mr. Dull on vital objectives which are intended to be achieved with the Code.
(2 Marks)
(B) Advise Mr. Ram on the functional responsibilities of Insolvency Professional
Agencies (IPA). (2 Marks)
(C) Advise Mr. Ram on the independence with the Corporate Debtor. (2 Marks)
(D) Advise 'Poor' Ltd. whether the initiation of insolvency resolution process can be
done by creditor only or by corporate debtor also. (2 Marks)
(E) Interim Resolution Professional is to be appointed by following the specified
procedure and for a specific term. Examine and advise Mr. Ram on the consent to
be provided by him in Form 2 as required by the relevant rules. (7 Marks)
(1.12) Answer the following under the provisions of the Competition Act, 2002:
(A) Enactment of the Competition Act, 2002 is to provide for an establishment of a
Commission with objectives. Advise Mr. Sharp for taking decision whether to
challenge the order of the Supreme Court? (3 Marks)
(B) A person who is purchasing goods for resale can also be considered as a
'Consumer'. Examine and advise 'Poor' Ltd. (3 Marks)
(C) All agreements which causes or is likely to cause an appreciable adverse effect on
competition in India, entered into in contravention shall be void. Examine and advise
'Poor' Ltd. (5 Marks)
(D) Advise 'Poor' Ltd. whether the Commission also has power to enquire into the acts
taking place outside India. (4 Marks)
Answer to Case Study 1
(1.1) Option (C): File applications for avoidance of preferential or undervalued transactions
(1.2) Option (D): The supply of raw material essential for construction of commercial real
estate from its suppliers.
(1.3) Option (D): D3 & D2 (ranked equally)-D1-D5 & D4 (ranked equally)
(1.4) Option (A): Rejecting the application of D1.
(1.5) Option (A): Mr. Ram, Interim Resolution Professional

© The Institute of Chartered Accountants of India


Page 181
PAPER – 6D: ECONOMIC LAWS 7

(1.6) Option (A) : has engaged at any time, in any paid employment.
(1.7) Option (B) : 2
(1.8) Option (D) : if it is whether or not, in formal or writing or whether or not enforceable by
legal proceedings
(1.9) Option (D) : All of the above
(1.10) Option (C): Is not valid and can be challenged.
(1.11) (A) Vital objectives which are intended to be achieved with the IBC: The Insolvency
and Bankruptcy Code, 2016 is intended to strike the right balance of interests of all
stakeholders of the business enterprise so that the corporates and other business
entities enjoy availability of credit and at the same time the creditor do not have to
bear the losses on account of default. The purpose of enactment of the Insolvency
and Bankruptcy Code, 2016 is as follows:
(a) To consolidate and amend the laws relating to re-organization and insolvency
resolution of corporate persons, partnership firms and individuals.
(b) To fix time periods for execution of the law in a time bound manner.
(c) To maximize the value of assets of interested persons.
(d) To promote entrepreneurship
(e) To increase availability of credit.
(f) To balance the interests of all the stakeholders including alteration in the order
of priority of payment of Government dues.
(g) To establish an Insolvency and Bankruptcy Board of India as a regulatory body
for insolvency and bankruptcy law.
(B) Functional responsibilities of Insolvency Professional Agencies (IPA):
It will perform three key functions:
(i) Regulatory functions
• drafting detailed standards and codes of conduct through bye-laws, that
are made public and are binding on all members
(ii) Executive functions
• monitoring, inspecting and investigating members on a regular basis
• gathering information on their performance, with the over-arching
objective of preventing frivolous behaviour, and
• malfeasance in the conduct of IP duties

© The Institute of Chartered Accountants of India


Page 182
8 FINAL (NEW) EXAMINATION: NOVEMBER, 2018

(iii) Quasi-judicial functions


• addressing grievances of aggrieved parties, hearing complaints against
members and taking suitable actions
(C) Eligibility of an insolvency Professional to be appointed as a Resolution
Professional: As per Regulation 3 of the Insolvency and Bankruptcy (Insolvency
Resolution process for Corporate Persons) Regulation, 2016, an insolvency
professional shall be eligible for appointment as a resolution professional for a
corporate insolvency resolution process if he and all partners and directors of the
insolvency professional entity of which he is partner or director are independent of
the corporate debtor:-
(a) He is eligible to be appointed as an independent director on the board of the
corporate debtor under section 149 of the Companies Act, 2013, where the
corporate debtor is a company.
(b) He is not a related party of the corporate debtor.
(c) He is not an employee or proprietor or a partner of a firm of auditors or
company secretaries in practice or cost auditors of the corporate debtor in the
last three financial years.
(d) He is not an employee or proprietor or a partner of a legal or consulting firm
that has or had any transaction with the corporate debtor amounting to ten per
cent or more of the gross turnover of such firm in the last three financial years.
(D) As per Section 6 of the IBC, 2016, where any corporate debtor commits a default, a
financial creditor, an operational creditor or the corporate debtor itself may initiate
corporate insolvency resolution process in respect of such corporate debtor in the
manner as provided under this Chapter (Chapter II of part II). Therefore, Insolvency
resolution process can be initiated by creditor as well as by the corporate debtor.
(E) Appointment of IRP: As per the Code, Adjudicating authority shall appoint an
Interim Resolution Professional within 14 days from the commencement date of the
Insolvency process. Section16 of the Code lays down the procedure for
appointment of an Interim Resolution Professional.
Where the application for corporate insolvency resolution process is made by a
financial creditor or the corporate debtor, the resolution professional as proposed in
the application shall be appointed as the interim resolution professional, if no
disciplinary proceedings are pending against him.
Where the application for corporate insolvency resolution process is made by an
operational creditor and
(a) No proposal for an interim resolution professional is made. The
Adjudicating Authority shall make a reference to the Board for the
recommendation of an insolvency professional who may act as an interim

© The Institute of Chartered Accountants of India


Page 183
PAPER – 6D: ECONOMIC LAWS 9

resolution professional.
(b) A proposal for an interim resolution professional is made the proposed
resolution professional shall be appointed as the interim resolution
professional, if no disciplinary proceedings are pending against him.
The Board shall recommend the name of an insolvency professional to the
Adjudicating Authority against whom no disciplinary proceedings are pending, within
ten days of the receipt of a reference from the Adjudicating Authority.
Period of appointment of IRP: The term of Interim Resolution Professional shall
not exceed 30 days from the date of appointment. [Section 16]
As per Form 2, written consent by proposed IRP is given to the Adjudicating
authority under the relevant rule of the Insolvency and Bankruptcy (Application to
Adjudicating Authority) Rules, 2016.
(1.12)(A) The preamble of the Competition Act, 2002 provides that it is an Act to establish a
Commission to prevent anti-competitive practices, promote and sustain competition,
protect the interests of the consumers and ensure freedom of trade in markets in
India.
However Section 53T of the Competition Act, 2002, provides that the Central
Government or any State Government or the Commission or any statutory authority
or any local authority or any enterprise or any person aggrieved by any decision or
order of the Appellate Tribunal may file an appeal to the Supreme Court.
Since the Supreme Court(SC) as per the Indian constitution is the apex body, so the
decision of SC is binding on the Central Government / any State Government / the
Commission / any statutory authority / any local authority /any enterprise / any
person. So Mr. Sharp ‘s decision to challenge the order of the Supreme court is not
possible.
(B) The term ‘consumer’ is defined in section 2(f) of Competition Act, 2002. Accordingly,
‘consumer’ means any person who buys any goods for a consideration, which has
been paid or promised or partly paid and partly promised, whether such purchase of
goods is for resale or for any commercial purpose or for personal use.
Hence, it is not necessary that a person must purchase the goods for personal use
in order to be considered as a ‘consumer’ under Competition Act, 2002. Even a
person purchasing goods for resale or for any commercial purpose will also be
considered as a ‘consumer’ within the meaning of Section 2(f) of the Competition
Act, 2002.
(C) All agreements which causes or is likely to cause an appreciable adverse effect on
competition in India, entered into in contravention shall be void

© The Institute of Chartered Accountants of India


Page 184
10 FINAL (NEW) EXAMINATION: NOVEMBER, 2018

It shall not be lawful for any enterprise or association of enterprises or person or


association of persons to 'enter' into an agreement in respect of production, supply,
storage, distribution, acquisition or control of goods or provision of services, which
causes or is likely to cause an appreciable adverse effect on competition within
India. All such agreements entered into in contravention of the aforesaid prohibition
shall be void.
Any agreement entered into between enterprises or associations of enterprises or
persons or associations of persons or between any person and enterprise or
practice carried on, or decision taken by, any association of enterprises or
association of persons, including cartels, engaged in identical or similar trade of
goods or provision of services, shall be presumed to have an appreciable adverse
effect on competition, which—
(a) directly or indirectly determines purchase or sale prices;
(b) limits or controls production, supply, markets, technical development,
investment or provision of services;
(c) shares the market or source of production or provision of services by way of
allocation of geographical area of market, or type of goods or services, or
number of customers in the market or any other similar way;
(d) directly or indirectly results in bid rigging or collusive bidding.
However, any agreement entered into by way of joint ventures, if such agreement
increases efficiency in production, supply, distribution, storage, acquisition or control
of goods or provision of services, shall not be considered to be an anti-competitive.
(D) Acts taking place outside India but having an effect on competition in In dia
(Section 32)
The Commission shall, notwithstanding that,—
(a) an agreement referred to in section 3 has been entered into outside India; or
(b) any party to such agreement is outside India; or
(c) any enterprise abusing the dominant position is outside India; or
(d) a combination has taken place outside India; or
(e) any party to combination is outside India; or
(f) any other matter or practice or action arising out of such agreement or
dominant position or combination is outside India;
have power to inquire in accordance with the provisions contained in sections 19,
20, 26, 29 and 30 the Act into such agreement or abuse of dominant position or

© The Institute of Chartered Accountants of India


Page 185
PAPER – 6D: ECONOMIC LAWS 11

combination if such agreement or dominant position or combination has, or is likely


to have, an appreciable adverse effect on competition in the relevant market in India
and pass such orders as it may deem fit in accordance with the provisions of this
Act.
Case Study 2
Mr. Cute had given an application to the state authorities for purchase of land for farming and
agricultural use. This application was made by him through his company M/s Hip Hop Farms
Ltd. (HHFL). HHFL was initially incorporated in 2003 with two shareholders Mr. A and Mr. B.
Through an executed share transfer deed, shares of this company were transferred to
Mr. Cute and his wife Mrs. Pretty. Consequent to transfer of shares, first directors were also
replaced with new directors i.e. Mr. D, Mrs. E and Mr. Sharp. Mr. D and Mrs. E are parents of
Mrs. Pretty.
To enable HHFL to purchase the said piece of land, Mr. Cute, had given unsecured loan
amounting to ` 11 Crore to HHFL. Since the subject piece of land was an agricultural land,
during the time of representation, Mr. Sharp declared himself an agriculturist. Accordingly, the
additional collector allowed the purchase of the land on condition that it would be used for
farming within two years.
During the year 2010, Ms. F (sister of Mrs. Pretty) was appointed as director of HHFL in place
of Mr. Sharp.
HHFL was preparing financial statements on a regular basis and was compliant in filing
various documents with the Registrar of Companies. Financial Statements for the year ended
on 31 st March, 2018 and previous years did not show any income from farm activities or
agricultural activities. Instead, the said piece of land was developed by HHFL and constructed
a palatial bungalow with swimming pool and a dedicated space to facilitate landing and
parking fixed wing aircrafts.
Mr. Cute had separately obtained a loan for his personal use from a Non- Banking Finance
Company amounting to ` 65.Crore. The said loan was secured by the mortgage created on
the property owned by HHFL. Mr. Cute defaulted on payment of last few installments and tried
persuading bank to restructure the covenants of the loan agreement. Bank officials did not
agree to his request and decided to take action against him and the said mortgaged property.
This particular case came under the scanner of the authorities when the Collector of the
region claimed that this particular property along with other 110 properties have allegedly
flouted other applicable regulations prevailing in the State. Due to this matter, the case was
forwarded to the Income Tax department. Acting proactively on the matter, the Income Tax
department had issued an attachment notice under the Prohibition of Benami Property
Transaction Act, 1988 to HHFL for provisionally attaching the property and filed a report
before an adjudication authority to confirm the attachment.

© The Institute of Chartered Accountants of India


Page 186
12 FINAL (NEW) EXAMINATION: NOVEMBER, 2018

Mr. Cute had engaged a lawyer to prepare a reply in response to the notice received. His
lawyer had advised that maximum penalty for contravention, if any would be 10% of the cost
of the property. Further, he has stated that in the worst case situation, attachment in no case
under the provisions of the Prevention of Benami Property Transactions Act, 1988 exceed 3
months.
Mr. Cute after knowing the provisions, had instructed the lawyer to furnish a fabricated reply in
response to the notice and include a point as to why notice has been issued to him. The said
notice should have been issued to HHFL only.
After the legal proceedings were completed, the order was passed by the adjudicating
authorities. After, perusing the order, Mr. Cute identified certain errors and misplaced facts,
and asked his lawyer to discuss the same with the authorities. However, his lawyer forgot the
matter due to other cases in hand. When he was reminded again after almost 11 months, he
responded that the matter is time barred.
Besides the said piece of land on which bungalow and swimming pool were constructed, there
were other seven pieces of non-agricultural land just adjacent to the land. Survey numbers of
the same were 112/1, 112/2, 112/3, 112/4, 112/5, 112/6 and 112/7.
Mr. D and his family were quite affluent and generally they were seen in lavish social
gatherings apart from managing their real estate development business. During a family
function in 2012, they made a fixed deposit amounting to `10 crore in the name of Mrs. Pretty
which was a gift for her.
Mrs. Pretty on maturity of the said deposit, transferred the amount in the name of Mr. Cute for
his personal use. During 2015, the said amount was used by Mr. Cute to buy a piece of land
bearing survey number 112/1 in the name of Mrs. Pretty. Owner as per the land records was
Mrs. Pretty and payment for the said land bearing survey number 112/1 was made by Mr.
Cute.
Mr. D was the owner of land bearing survey numbers 112/2, 112/3, 112/4, 112/5, 112/6 and
112/7. During the third quarter of financial year 2017-18, he developed and launched a new
residential-cum-commercial project on the said pieces of land after seeking registration under
the Real Estate (Regulation and Development) Act, 2016 (RERA). For the said project Ms. F
was acting as an authorized agent for marketing. When the commercial launch was organized,
it was announced by Ms. F that the project is available at an attractive rate of`8,800 per
square feet and the units are very spacious since they admeasure 1500 square feet built-up
with total 100 units.
Also, marketing brochure contains following features included in the project:
(1) Italian marble in the kitchen
(2) 5 Star rated Air Conditioners
(3) 3 Star rated Geysers

© The Institute of Chartered Accountants of India


Page 187
PAPER – 6D: ECONOMIC LAWS 13

(4) French Windows of reputed brand


(5) Elevators of top brands
(6) Open parking slot at a nominal price of `11,000
(7) Massive multi-level kids play area
(8) Ducts attached to each flat
(9) Comprehensive insurance for the project
Marketing brochure mentioned that builder provides warranty of 5 years of the products with
additional free 1 year warranty.
It was informed in the marketing material that the project would be completed in a time frame
of 5 years. One of the allottee complained about Ms. F for project's registration to which she
replied that project is already registered and since she is daughter of the promoter, she is not
required to take the separate registration, only outsiders are required to take registration
under RERA. Ms. F receives facilitation fees from the company owning the project.
Mr. Bhakt was one of the allottee who bought flat number 205 in Tower 1 of the project after
several rounds of meeting with Ms. F. It was told to him that a Ganesh Temple would be
constructed as a part of the project in the eastern side of Podium 2.
During the course of the project, an intimation along with a certificate from engineer was sent
to all the allotees that due to a technical objection received from fire department, temple will
have to be shifted from Podium 2 to Podium 3.
When this fact came in the knowledge of Mr. Bhakt, he consulted his lawyer who advised to
file a complaint against the builder with the authorities. Also, he mentioned in the complaint
that he bought flat through Ms. F who was not registered under RERA and reported several
defects in the features contained in the marketing brochure.
Just before the completion of the project, the promoter got an offer to sell the entire project to
an American builder at an attractive price. The acquirer informed the promoter that since it is
the deal between us and I have never defaulted on the delivery in projects in last 50 years
there is practically no use of seeking approval of allottees. There were several rounds of
discussions between the promoter and the acquirer; however, the deal did not go through due
to difference in valuation.
The project was completed on time and the invitation was sent to all the allottees to take
physical possession of their respective units. After staying for about 8 months in the flat
number 406 in Tower 4, Mr. Sultan informed builder that he is facing serious issues with the
quality of MCB provided and there is a potential risk of short circuit which could lead to
massive losses to the building as a whole. On investigation by an independent electrician
appointed by Mr. Sultan, it was found that lining of electricity wire was done along with water
pipe lines and due to internal damage, problem is arising. However, the promoter was harping

© The Institute of Chartered Accountants of India


Page 188
14 FINAL (NEW) EXAMINATION: NOVEMBER, 2018

on the fact that the issue is in the MCB and not in the wirings. The investigation done by
electrician was confirmed by other electricians who surveyed a few other flats.
Further, Mr. Sultan complained that the grass given by the builder in the flower bed area was
of sub-standard quality and needs replacement.
Answer the following Questions:
(2.1) Who is Benamidar in the above case as per Prevention of Benami Property Transactions
Act, 1988?
(A) HHFL.
(B) Mr. D.
(C) Mr. E.
(D) All of the above (2 Marks)
(2.2) Whether is it a requirement under Prevention of Benami Property Transactions Act, 1988
that Benamidar shall be aware that property is registered in his / her name to categorize
a transaction as Benami ?
(A) Yes, it is necessary.
(B) No, it is not necessary
(C) Can't say
(D) None of the above (2 Marks)
(2.3) Under Prevention of Benami Property Transactions Act, 1988, property which has been
declared as Benami can be confiscated by which authority?
(A) The President of India.
(B) State Government.
(C) Central Government.
(D) None of the above (2 Marks)
(2.4) In a scenario where authorities conclude that the subject property is hit by the provisions
of the Prevention of Benami Property Transactions Act, 1988, what could be the quantum
of penalty?
(A) 25% of the cost of the property.
(B) 10% of the fair market value of the property.
(C) 10% of the cost of the property.

© The Institute of Chartered Accountants of India


Page 189
PAPER – 6D: ECONOMIC LAWS 15

(D) 25% of the fair market value of the property. (2 Marks)


(2.5) Under Prevention of Benami Property Transactions Act, 1988, notice for initiating action
shall be submitted by following means?
(A) By Post.
(B) By way of summons.
(C) By e-mail.
(D) Either (A) or (B) (2 Marks)
(2.6) As per the provisions of RERA, which of following are treated as part of common area?
(A) Kids play area.
(B) Duct attached to the units.
(C) Balcony attached to the living room.
(D) All of the above (2 Marks)
(2.7) Under RERA, 20% of the flat cost cannot be accepted unless:
(A) Property is registered.
(B) Marketing brochure mentioned terms of payment.
(C) 20% project is completed.
(D) All of the above (2 Marks)
(2.8) Under RERA, provision related to 5 years warranty is applicable to following:
(A) Chipped beam in the kitchen.
(B) Loose tiles in the washrooms.
(C) Leakage in the internal pipe lines.
(D) All of the above (2 Marks)
(2.9) Under RERA, when all documents in connection with insurance shall be handed over by
the promoter to the allotees?
(A) On receipt of final payment / installment.
(B) On receipt of occupancy certificate.
(C) On receipt of NOC from fire department.
(D) On formation of society. (2 Marks)

© The Institute of Chartered Accountants of India


Page 190
16 FINAL (NEW) EXAMINATION: NOVEMBER, 2018

(2.10) On completion of the project and after receipt of occupancy certificate, when can an
allottee take physical possession of the flat?
(A) Within two months.
(B) Within three months.
(C) Within six months.
(D) None of the above (2 Marks)
(2.11)Explain the following in light of the provisions of the Prevention of Benami Property
Transactions Act, 1988:
(A) Owner of the land as per land records shall make payment for the land standing in
his/her name. Examine the correctness of the statement. (3 Marks)
(B) Whether action proposed by the officials of the bank is defensible? Advise officials
of the bank. (3 Marks)
(C) Examine legal ramifications of the instructions made by Mr. Cute to his lawyer and
advice by his lawyer in the matter. (6 Marks)
(D) Mr. Cute has approached you after hearing response from his lawyer after 1 month.
Please advise him. (3 Marks)
(2.12)Explain the following in light of the Provisions of the Real Estate (Regulation and
Development) Act, 2016 (RERA):
(A) Mr. Bhakt has approached you to confirm advice given by his lawyer. Kindly assist
Mr. Bhakt on the points mentioned by the lawyer. (9 Marks)
(B) Promoter of the project has appointed you to advise on the issue raised by
Mr. Sultan. (4 Marks)
(C) Examine legal validity of the proposal given by the American builder. (3 Marks)

Answer to Case Study 2


(2.1) Option (A): HHFL
(2.2) Option (B): No, it is not necessary.
(2.3) Option (C): Central Government
(2.4) Option (D): 25% of the fair market value of the property
(2.5) Option (D): Either (A) or (B)
(2.6) Option (D): All of the above

© The Institute of Chartered Accountants of India


Page 191
PAPER – 6D: ECONOMIC LAWS 17

(2.7) Option (A): Property is registered


(2.8) Option (D): All of the above
(2.9) Option (D): On formation of society
(2.10) Option (A): Within two months
(2.11)(A) As per section 2(9) of the Prohibition of Benami Property Transactions Act, 1988, all
such type of transaction or an arrangement made in respect to a property, where -
• suc h a property is transferred to or held by one person and consideration is
paid by some other person,
• suc h a property c arried out or m ade in a fictitious name,
• owner of a property is not aware of, or, denies knowledge of, suc h
ownership;
• where the person providing the c onsideration is not trac eable or is
fic titious.
Such a transaction is said to be a benami transaction.
Accordingly, in the light of the above provisions, the owner of the land as per land
records shall make payment for the land standing in his/her name in order to be
valid transaction and not to be considered as benami transaction in the terms of
section 2(9) of the said Act.
(B) As per the facts given in the case study, Mr. Cute defaulted in the payment of few
installments on the loan secured on the property owned by the HHFL. He tried
persuading bank to restructure the covenants of loan agreement. Bank Officials did
not agree to his request and decided to take action against him and the said
mortgaged property.
As per Section 18 of the Prohibition of Benami Property Transactions Act, 1988, the
following Authorities shall be there for the purposes of this Act, namely:—
(a) the Initiating Officer;
(b) the Approving Authority;
(c) the Administrator; and
(d) the Adjudicating Authority.
The authorities shall exercise all or any of the powers and perform all or any of the
functions conferred on, or, assigned, as the case may be, to it under this Act or in
accordance with such rules as may be prescribed.

© The Institute of Chartered Accountants of India


Page 192
18 FINAL (NEW) EXAMINATION: NOVEMBER, 2018

The authorities under this Act shall have the same powers as are vested in a civil
court (Under section 19).
Accordingly, denial to agree to the request of Mr. Cute to restructure the covenants
of loan agreement by bank officials, is right.However, the decision to take action
against him and the said mortgaged property is not available with the Bank officials
under the Prohibition of Benami Property Transactions Act, 1988. Any property,
which is subject matter of benami transaction, shall be liable to be confiscated by
the Central Government under section 5 of the Prohibition of Benami Property
Transactions Act, 1988.
(C) Following will be the legal ramifications of the instructions m ade by Mr. Cute to his
Lawyer and advice by his lawyer with respect to furnishing of a fabricated reply in
response to the notice and to include a point as to why notice has been issued to
him. The said notice shall be issued to HHFL only-
As per section 24 of the Prohibition of Benami Property Transactions Act, 1988,
where the Initiating Officer, on the basis of material in his possession, has reason to
believe that any person is a benamidar in respect of a property, he may, after
recording reasons in writing, issue a notice to the person to show cause within such
time as may be specified in the notice why the property should not be treated as
benami property.
Where the notice specifies any property as being held by a benamidar, a copy of the
notice shall also be issued to the beneficial owner if his identity is known. Where the
Initiating Officer is of the opinion that the person in possession of the property held
benami may alienate the property during the period specified in the notice, he may,
with the previous approval of the Approving Authority, by order in writing, attach
provisionally the property in the manner as prescribed in Rule 4 of the Benami
Transactions Prohibition Rules, 2016, for a period not exceeding ninety days from
the date of issue of notice.
The Initiating Officer, after making such inquires and calling for such reports or
evidence as he deems fit and taking into account all relevant materials, shall, within
a period of ninety days from the date of issue of notice —
(a) where the provisional attachment has been made —
(i) pass an order continuing the provisional attachment of the property with
the prior approval of the Approving Authority, till the passing of the order
by the Adjudicating Authority; or
(ii) revoke the provisional attachment of the property with the prior approval
of the Approving Authority;

© The Institute of Chartered Accountants of India


Page 193
PAPER – 6D: ECONOMIC LAWS 19

(b) where provisional attachment has not been made—


(i) pass an order provisionally attaching the property with the prior approval
of the Approving Authority, till the passing of the order by the Adjudicating
Authority; or
(ii) decide not to attach the property as specified in the notice, with the prior
approval of the Approving Authority.
Where the Initiating Officer passes an order continuing the provisional
attachment of the property or passes an order provisionally attaching the
property, he shall, within fifteen days from the date of the attachment, draw up
a statement of the case and refer it to the Adjudicating Authority.
Parties to be issued notice: On receipt of a reference under Section 24, the
Adjudicating Authority shall issue notice, to furnish such documents,
particulars or evidence as is considered necessary on a date to be specified
therein, on the following persons, namely:—
(a) the person specified as a benamidar therein;
(b) any person referred to as the beneficial owner therein or identified as
such;
(c) any interested party, including a banking company;
(d) any person who has made a claim in respect of the property.
Therefore, as per the above given provisions, the Adjudicating Authority shall
issue notice, to furnish such documents, particulars or evidence as is
considered necessary on a date to be specified therein in the notice to the
person specified as a benamidar therein; and to as the beneficial owner
therein or identified as such.
(D) After persuing the order passed by Adjudicating authorities, Mr. Cute identified
certain errors and misplaced facts. He asked his lawyer to discuss the same with
the authorities. However, his lawyer forgot the same. Reminding after 11 months, he
responded that the said matter is time-barred.
According to section 47 of the Prohibition of Benami Property Transactions Act,
1988, the Appellate Tribunal or the Adjudicating Authority may, in order to rec tify any
mistake apparent on the face of the record, amend any order made by it under
section 26 and section 46 respectively, within a period of one year from the end of
the month in which the order was passed.
No amendment shall be made, if the amendment is likely to affect any person
prejudicially, unless he has been given notice of intention to do so and has been
given an opportunity of being heard.

© The Institute of Chartered Accountants of India


Page 194
20 FINAL (NEW) EXAMINATION: NOVEMBER, 2018

Accordingly, above stated course of action may be available to the Mr. Cute in
compliance with the said provision.
2.12 (A) Mr. Bhakt, an allottee was told that Ganesh Temple would be constructed as a part
of the project in podium 2. Due to technical objection, an intimation along with the
certificate from engineer was sent to allottees stating that as of consequences
temple will have to be shifted from podium 2 to podium 3. Also filed a complained
against Ms. F and towards builders for several defects in the features contained in
the marketing brochure.
According to section 18 of the Real Estate (Regulation & Development) Act, 2016, if
the promoter fails to complete or is unable to give possession of an apartment, plot
or building, in accordance with the terms of the agreement for sale or, as the case
may be, duly completed by the date specified therein;
He shall be liable on demand to the allottees, in case the allottee wishes to
withdraw from the project, without prejudice to any other remedy available, to return
the amount received by him in respect of that apartment, plot, building, as the case
may be, with interest at such rate as may be prescribed in this behalf including
compensation in the manner as provided under this Act.
However, where an allottee does not intend to withdraw from the project, he shall be
paid, by the promoter, interest for every month of delay, till the handing over of the
possession, at such rate as may be prescribed.
If the promoter fails to discharge any other obligations imposed on him under this
Act or the rules or regulations made thereunder or in accordance with the terms and
conditions of the agreement for sale, he shall be liable to pay such compensation to
the allottees, in the manner as provided under this Act.
Further Section 9 of the Act specifies that no real estate agent shall facilitate the
sale or purchase of or act on behalf of any person to facilitate the sale or purchase
of any plot, apartment or building, as the case may be, in a real estate project or
part of it, being the part of the real estate project registered under section 3, being
sold by the promoter in any planning area, without obtaining registration under this
section. In case of contravention, Ms. F will be liable under section 62 of the said
Act.
Accordingly, Mr. Bhakt will have above remedies under the RERA against the Ms. F
and against builder with the authorities.
(B) According to section 14 of the RERA, the proposed project shall be developed and
completed by the promoter in accordance with the sanctioned plans, layout plans
and specifications as approved by the competent authorities.

© The Institute of Chartered Accountants of India


Page 195
PAPER – 6D: ECONOMIC LAWS 21

As per section 14(3) of the Act, in case any structural defect or any other defect in
workmanship, quality or provision of services or any other obligations of the
promoter as per the agreement for sale relating to such development is brought to
the notice of the promoter within a period of five years by the allottee from the date
of handing over possession, it shall be the duty of the promoter to rectify such
defects without further charge, within thirty days, and in the event of promoter's
failure to rectify such defects within such time, the aggrieved allottees shall be
entitled to receive appropriate compensation in the manner as provided under this
Act.
In the given case, Mr. Sultan after 8 months of his staying, informed the builder of
the quality of MCB with a potential risk of short circuit.Considering it a structural
defect Mr. Sultan intimated within time frame. So promoter of the project will be
liable here.
Whereas complain of grass given by builder in flower bed area for replacement is in
the nature of "minor additions or alterations”, so promoter will be discharged of his
liabilities.
(C) As per section 15(1) of the Real Estate (Regulation & Development) Act, 2016, the
promoter shall not transfer or assign his majority rights and liabilities in respect of a
real estate project to a third party without obtaining prior written consent from two-
third allottees, except the promoter, and without the prior written approval of the
Authority.
However, such transfer or assignment shall not affect the allotment or sale of the
apartments, plots or buildings as the case may be, in the real estate projec t made
by the erstwhile promoter.
In the case study, the American Builder’s, proposal was not valid as no prior written
consent from two-third allottees and the authority were obtained.
Case Study 3
Question 3
Mr. Inder and Mr. Sunder are promoter directors of India Exports Limited having registered
office in Jammu, is engaged in the export of software products to various countries in the
world. One of the customer in U.S. to whom the company exported certain products failed to
pay the amount due for these exports resulting into non-repatriation of amount to India. The
Adjudicating Authority on coming to know about this, levied a penalty on the company under
the provisions of the Foreign Exchange Management Act, 1999. The Company has sought
advice on the followings:
(a) Relevant provisions for realization of export amount and its timeline.

© The Institute of Chartered Accountants of India


Page 196
22 FINAL (NEW) EXAMINATION: NOVEMBER, 2018

(b) Timeline to surrender the realized foreign exchange under the Act.
(c) Cases where realization and repatriation enjoy exemption.
Later, the company settled the amount for 50% with the customer and the amount was
transferred through Hawala to India. The money so received was partly used by the company
to part finance it's office building in Mumbai. During search in the premises of Hawala
businessman, some documentary evidence was captured by the search officer and based on
which, the Adjudicating Authority appointed under the Prevention of Money Laundering Act,
2002 issued an order attaching the office of the company alleged to be involved in scheduled
offence of money laundering Mr. Prabhat, one of the employee was sent to Japan to develop a
software program on deputation for 2 years. He earned a sum of US$ 3000 as a honorarium.
Ms. Lilly, the daughter of Mr. Inder is an air hostess with the British Airways and flies for 12
days in a month and thereafter takes a break for 18 days. During the break, she
accommodated of 'base', which is normally the city, outside India where the airways are
headquartered. However, for security considerations, she was based on Mumbai, during the
current financial year and was accommodated at Mumbai for more than 182 days.
Mr. Victor, son of Mr. Sunder, having Indian origin and resident of USA desires to acquire two
immovable properties in India comprising a residential flat in Noida and a farm house on the
outskirts of Delhi. Further, Mr. Sunder has won lottery and want to remit the amount to his son
Mr. Victor in USA for buying-immovable property in USA under joint ownership of 50% with Mr.
Sunder. Mr. Sunder also wants to remit money to meet his obligation of 50% in the above
immovable property.
The balance of the money received through Hawala was used by the company to part finance
the residential flat in Noida purchased by Mr. Victor.
The Adjudicating Authority appointed under the Prevention of Money Laundering Act, 2002
issued an order attaching the flat alleged to be involved in scheduled offence of money
laundering. The company decides to challenge the action of the Adjudicating Authority.
In the meantime, Mr. Sunder requested the Chief Financial Officer to examine the following
issues under the Prevention of Money Laundering Act, 2002:
(a) Process of money laundering
(b) Multiple method of money laundering
(c) The connection between 'proceeds of crime' and 'criminal activity'
(d) The request from a contracting state for investigation.
(e) The powers of the authority under the Act to survey
Answer the following questions:
(3.1) Which of these is not a permissible capital account transactions?

© The Institute of Chartered Accountants of India


Page 197
PAPER – 6D: ECONOMIC LAWS 23

(A) Investment by person resident in India in Foreign Securities.


(B) Foreign currency loans raised in India and abroad by a person resident in India.
(C) Export, Import and holding of currency/currency notes.
(D) Trading in transferable development rights. (2 Marks)
(3.2) Mr. Prabhat can retain the honorarium earned by him on deputation to the extent of US $:
(A) 3000
(B) 2000
(C) 1000
(D) Nil (2 Marks)
(3.3) The residential status of Ms. Lilly for the current financial year under FEMA would be:
(A) Non-Resident irrespective of her citizenship.
(B) Resident irrespective of her citizenship.
(C) Non-Resident since she is British citizen.
(D) Resident though she is British citizen. (2 Marks)
(3.4) The time limit within which the appeal can be lodged against the decision of the
Adjudicating Authority by India Export Limited:
(A) Within 30 days from receipt of order.
(B) Within 60 days from receipt of order.
(C) Within 45 days from receipt of order.
(D) Within 90 days from receipt of order. (2 Marks)
(3.5) Mr. Victor can acquire the following properties by following the steps as mentioned in the
provisions of the Foreign Exchange Management Act, 1999:
(A) a farm house in outskirt of Delhi, only.
(B) both farm house in the outskirt of Delhi and a flat in Noida.
(C) a flat in Noida, only.
(D) None of above (2 Marks)
(3.6) Section 2 of the Prevention of Money Laundering Act, 2002 defines the term 'scheduled
offence', which accordingly means:

© The Institute of Chartered Accountants of India


Page 198
24 FINAL (NEW) EXAMINATION: NOVEMBER, 2018

(A) the offences specified under Part A of the Schedule.


(B) the offences specified under Part B of the Schedule if the total value involved in
such offences is thirty lakh rupees or more.
(C) the offences specified under Part C of the Schedule.
(D) All of the above (2 Marks)
(3.7) Whoever commits offence of Money Laundering shall be punishable with:
(A) imprisonment only.
(b) fine only
(C) imprisonment or fine.
(D) Imprisonment and fine. (2 Marks)
(3.8) Money Laundering is a single process however, its cycle can be broken down into
following three distinct stages:
(A) Integration, Layering and Placement.
(B) Layering, Placement and Integration.
(C) Placement, Layering and Integration.
(D) Placement, Integration and Layering. (2 Marks)
(3.9) Where an order of confiscation has been made under the provisions of section 58B of the
Prevention of Money Laundering Act, 2002, in respect of any property of a person, all
rights and title in such property shall vest absolutely in the _______free from all
encumbrances.
(A) Central Government
(B) Supreme Court
(C) President of India
(D) None of the above (2 Marks)
(3.10)The offences under the Prevention of Money Laundering Act, 2002 shall be:
(A) cognizable and bailable.
(B) non-cognizable and non-bailable.
(C) cognizable and non-bailable.
(D) non-cognizable and bailable. (2 Marks)

© The Institute of Chartered Accountants of India


Page 199
PAPER – 6D: ECONOMIC LAWS 25

(3.11)Answer the following with reference to the provisions of the Foreign Exchange
Management Act, 1999 (FEMA):
(A) The FEMA extends to the whole of India. Examine and advise India Export Ltd. as
they have registered office in Jammu. (2 Marks)
(B) The drawal of foreign exchange is prohibited for certain current account
transactions. Examine and advise Mr. Sunder whether he can remit the amount of
lottery won by him to Mr. Victor in USA. (2 Marks)
(C) The Act restricts acquisition or transfer of immovable property outside India by a
person resident in India. Examine and advise Mr. Sunder whether he can remit
amount to buy immovable property in USA. (2 Marks)
(D) The amount representing full export value shall be realized within time limit
permitted under the Act. Explain and advise India Export Ltd. the relevant provisions
for realization of export value and its timeline. (4 Marks)
(E) The realized foreign exchange is to be surrendered within the period specified
under the Act. Examine and advise India Export Ltd. (2 Marks)
(F) The realization and repatriation in certain cases enjoy exemption. Examine and
advise India Export Ltd. (3 Marks)
(3.12) Examine and advise Mr. Sunder on the following with reference to the provisions of the
Prevention of Money Laundering Act, 2002 (PMLA):
(A) Money Laundering is a process. (3 Marks)
(B) There are multiple methods of money laundering. (3 Marks)
(C) The 'proceeds of crime' and 'criminal activity' have connection. (2 Marks)
(D) The request from a contracting state can be accepted for investigation. (3 Marks)
(E) The Authority under the Act can make survey only based on the material in his
possession. (4 Marks)

Answer to Case Study 3


(3.1) Option (D) :Trading in transferable development rights
(3.2) Option (B) : 2000
(3.3) Option (A) : Non-Resident irrespective of her citizenship
(3.4) Option (C) :within 45 days from receipt of order

© The Institute of Chartered Accountants of India


Page 200
26 FINAL (NEW) EXAMINATION: NOVEMBER, 2018

(3.5) Option (C):a flat in Noida, only.


(3.6) Option (D) :All of the above
(3.7) Option (D) :imprisonment and fine
(3.8) Option (C) :Placement, Layering and Integration
(3.9) Option (A) :Central Government
(3.10) Option (C) :Cognizable and non-bailable
(3.11)(A) Extent and Application [Sections 1 of FEMA, 1999]
FEMA, 1999 extends to the whole of India. In addition, it shall also apply to all
branches, offices and agencies outside India owned or controlled by a person
resident in India and also to any contravention thereunder committed outside India
by any person to whom this Act applies.
Accordingly, FEMA does not apply to citizens of India who are outside India unless
they are resident of India. The scope of the Act has been further extended to
include branches, offices and agencies outside India. The scope is thus wide
enough because the emphasis is on the words “Owned or Controlled”. Even
contravention of the FEMA committed outside India by a person to whom this Act
applies will also be covered by FEMA.
(B) According to Section 5 of the FEMA, 1999 and rules/regulations made thereunder,
the drawal of foreign exchange for certain current account transactions is
prohibited, a few need permission of appropriate Govt. of India authority and some
other transactions would require RBI permission if they exceed a certain ceiling.
According to Schedule I, Remittance out of lottery winnings is prohibited.
Hence, Mr. Sunder cannot remit the amount of lottery won by him to Mr. Victor in
USA.
(C) According to Regulations on Acquisition and Transfer of Immovable Property
outside India, a person resident in India may acquire immovable property outside
India, jointly with a relative who is a person resident outside India, provided there is
no outflow of funds from India.
In the instant case, Mr. Sunder wants to remit money to meet his obligation of 50%
in the immovable property in USA under joint ownership with his son Mr. Victor.
Hence, as per the regulations, Mr. Sunder cannot remit amount to buy immovable
property in USA.
(D) Period within which export value of goods/software/ services to be realized
[Foreign Exchange Management (Export of Goods and Services) Regulations, 2015]

© The Institute of Chartered Accountants of India


Page 201
PAPER – 6D: ECONOMIC LAWS 27

(1) The amount representing the full export value of goods / software/ services
exported shall be realised and repatriated to India within nine months from the
date of export, provided
(a) that where the goods are exported to a warehouse established outside
India with the permission of the Reserve Bank, the amount representing
the full export value of goods exported shall be paid to the authorised
dealer as soon as it is realised and in any case within fifteen months from
the date of shipment of goods;
(b) further that the Reserve Bank, or subject to the directions issued by that
Bank in this behalf, the authorised dealer may, for a sufficient and
reasonable cause shown, extend the period of nine months or fifteen
months, as the case may be.
(2) (a) Where the export of goods / software / services has been made by Units
in Special Economic Zones (SEZ) / Status Holder exporter / Export
Oriented Units (EOUs) and units in Electronics Hardware Technology
Parks (EHTPs), Software Technology Parks (STPs) and Bio-Technology
Parks (BTPs) as defined in the Foreign Trade Policy in force, then, the
amount representing the full export value of goods or software shall be
realised and repatriated to India within nine months from the date of
export.
Provided further that the Reserve Bank, or subject to the directions
issued by the Bank in this behalf, the authorised dealer may, for a
sufficient and reasonable cause shown, extend the period of nine months.
(b) The Reserve Bank may for reasonable and sufficient cause direct that the
said exporter/s shall cease to be governed by sub-regulation (2);
Provided that no such direction shall be given unless the unit has been
given a reasonable opportunity to make a representation in the matter.
(c) On such direction, the said exporter/s shall be governed by the provisions
of sub-regulation (1), until directed otherwise by the Reserve Bank.'
Explanation—For the purpose of this regulation, the “date of export” in
relation to the export of software in other than physical form, shall be
deemed to be the date of invoice covering such export.
(E) Period for surrender of received/ realised/ unspent/ unused foreign exchange by
Resident individuals [Regulation 5 of Foreign Exchange Management (Realisation,
repatriation and surrender of foreign exchange) Regulations, 2000]:A Person being
an individual resident in India shall surrender the received/realised/unspent/ unused
foreign exchange whether in the form of currency notes, coins and travellers
cheques, etc. to an authorised person within a period of 180 days from the date of

© The Institute of Chartered Accountants of India


Page 202
28 FINAL (NEW) EXAMINATION: NOVEMBER, 2018

such receipt/realisation/purchase/acquisition or date of his return to India, as the


case may be.
(F) Exemption from realisation and repatriation in certain cases [Section 9 of
FEMA, 1999]
The provisions of sections 4 and 8 shall not apply to the following, namely:
(a) possession of foreign currency or foreign coins by any person up to such limit
as the Reserve Bank may specify;
(b) foreign currency account held or operated by such person or class of persons
and the limit up to which the Reserve Bank may specify;
(c) foreign exchange acquired or received before the 8th day of July, 1947 or any
income arising or accruing there on which is held outside India by any person
in pursuance of a general or special permission granted by the Reserve Bank;
(d) foreign exchange held by a person resident in India up to such limit as the
Reserve Bank may specify, if such foreign exchange was acquired by way of
gift or inheritance from a person referred to in clause (c), including any income
arising there from;
(e) foreign exchange acquired from employment, business, trade, vocation,
service, honorarium, gifts, inheritance or any other legitimate means up to
such limit as the Reserve Bank may specify; and
(f) such other receipts in foreign exchange as the Reserve Bank may specify.
(3.12) (A) Money laundering is a process: It is the process by which illegal funds and assets
are converted into legitimate funds and assets. In other words, it is basically the
process of converting illegal or black money of a person in a legal or white money.
It is the process used by criminals to wash their “tainted” money to make it “clean.”
Money laundering is a single process however; its cycle can be broken down into
three distinct stages
1. Placement: It is the first and the initial stage when the crime money is injected
into the formal financial System.
2. Layering: Then under the second stage, money injected into the system is
layered and moved or spread over various transactions in different accounts
and different countries. Thus, it will become difficult to detect the origin of the
money.
3. Integration: Under the third and final stage, money enters the financial system
in such a way that original association with the crime is sought to be
obliterated so that the money can then be used by the offender or person
receiving as clean money.

© The Institute of Chartered Accountants of India


Page 203
PAPER – 6D: ECONOMIC LAWS 29

(B) Multiple methods of money laundering: There are multiple methods through
which money can be laundered and huge profit is being made, some of them are:
• Cash Smuggling: Moving cash from one location to another or depositing the
cash in Swiss Bank Account;
• Structuring: Cash is broken down into formal receipts to buy money orders
etc., smaller amounts are hard to detect;
• Laundering via Real Estate: Buying a land for money and then selling it making
the profits legal.
• Stock Markets scams
• By creating bogus companies.
• Drug Trafficking;
• Bribery and Corruption;
• Kidnapping and Extortion.
(C) Section 2(1)(u) defines "proceeds of crime"as any property derived or obtained,
directly or indirectly, by any person as a result of criminal activity relating to a
scheduled offence or the value of any such property or where such property is
taken/held outside the country, then the property equivalent in value held within the
country.
(D) Letter of Request to a Contracting State in Certain Cases [Section 57 of the
PMLA, 2002]
1. If, in the course of an investigation into an offence or other proceedings under
this Act, an application is made to a Special Court by the Investigating Officer
or any officer superior in rank to the Investigating Officer that any evidence is
required in connection with investigation into an offence or proceedings under
this Act and he is of the opinion that such evidence may be available in any
place in a contracting State, and the Special Court, on being satisfied that
such evidence is required in connection with the investigation into an offence
or proceedings under this Act, may issue a letter of request to a court or an
authority in the contracting State competent to deal with such request to-
(i) examine facts and circumstances of the case,
(ii) take such steps as the Special Court may specify in such letter of
request, and
(iii) forward all the evidence so taken or collected to the Special Court issuing
such letter of request.
2. The letter of request shall be transmitted in such manner as the Central

© The Institute of Chartered Accountants of India


Page 204
30 FINAL (NEW) EXAMINATION: NOVEMBER, 2018

Government may specify in this behalf.


3. Every statement recorded or document or thing received shall be deemed to
be the evidence collected during the course of investigation.
(E) Power of authority to make survey [Section 16(1) of the PMLA]: Where an
authority, on the basis of material in his possession, has reason to believe (the
reasons for such belief to be recorded in writing) that an offence under section 3
has been committed, he may enter any place—
(i) within the limits of the area assigned to him; or
(ii) in respect of which he is authorised for the purposes of this section by such
other authority, who is assigned the area within which such place is situated,
at which any act constituting the commission of such offence is carried on, and may
require any proprietor, employee or any other person who may at that time and
place be attending in any manner to, or helping in, such act so as to,-
(i) afford him the necessary facility to inspect such records as he may require and
which may be available at such place;
(ii) afford him the necessary facility to check or verify the proceeds of crime or any
transaction related to proceeds of crime which may be found therein; and
(iii) furnish such information as he may require as to any matter which may be
useful for, or relevant to, any proceedings under this Act.
Explanation. - For the purposes of this sub-section, a place, where an act which
constitutes the commission of the offence is carried on, shall also include any other
place, whether any activity is carried on therein or not, in which the person carrying
on such activity states that any of his records or any part of his property relating to
such act are or is kept.
Therefore, Authority under the Act can make survey as per the above stated section.

© The Institute of Chartered Accountants of India


Page 205
DISCLAIMER
This Suggested Answers hosted on the website do not constitute the basis for

evaluation of the student’s answers in the examination. The answers are

prepared by the Faculty of the Board of Studies with a view to assist the

students in their education. While due care is taken in preparation of the

answers, if any errors or omissions are noticed, the same may be brought to

the attention of the Director of Studies. The Council of the Institute is not in

anyway responsible for the correctness or otherwise of the answers published

herein.

Further, in the Elective Papers which are Case Study based, the solutions

have been worked out on the basis of certain assumptions/views derived from

the facts given in the question or language used in the question. It may be

possible to work out the solution to the case studies in a different manner

based on the assumption made or view taken.

© The Institute of Chartered Accountants of India


Page 206
2 FINAL EXAMINATION: MAY 2018

PAPER 6D: ECONOMIC LAWS

NOTE: There are three case study questions in the question paper. Candidates are
required to answer all the questions of any two case study questions.
Citation of case laws, sections, subsections, rules may not form part of the answer.
Case Study No. 1
Question
(A) A complaint was made by a complainant (Informant) to the Competition Commission of
India (CCI) against the practices adopted by certain Insurance Companies in
implementation of the Insurance scheme, Country Peoples Plan (CPP) by an imaginary
State Government 'Z' in India.
The CCI after going through the complaint, on merit, ordered a detailed investigation by the
Director General of Investigation under the Competition Act, 2002 (as amended in 2007,
briefly referred to hereinafter as the "Act"). The facts of the case are mentioned as under:
(i) CPP is the health insurance scheme introduced by the Central Government for
below poverty line (BPL) families. The task of implementation of this scheme was
entrusted to the respective State Governments of the country with the Central
Government bearing 75% of the expenses incurred in relation to the annual
premiums.
(ii) A tender was floated by a State Government 'Z' through its agency ULTRA (on
1.11.2009) for selecting and insurance service provider for the implementation of
the CPP for the year beginning 2010-11 for a period of three years. The State
Government 'Z' issued a tender for the implementation of CPP scheme for the
selection of the insurance provider. In this regard, bids were invited from: (a)
insurance companies licensed and registered with the Insurance Regulatory and
Development Authority; and (b) agencies enabled by any central legislation to
undertake health insurance related activities. The last date for submission of the-
tender was 31.1.2010.
(iii) Four Public Sector Insurance Companies A, B, C & D Insurance Company, each
submitted their offer in response to the above tender before its last date of
submission. All these companies formed an Insurance Facilitation Group (IFG) with
the objective of a common cause of furtherance and Development of insurance
business in India and all these companies were members of the IFG. Before
submitting their bids against the above tender, officials of these companies attended
a meeting of IPG as per their practice, held on 27.12.2009 at XYZ place in the State
'Z' with the sole agenda to discuss the Tender Notice on CPP dated 1.11.2009 of the
State Government 'Z'. They agreed on a business sharing model of sharing the
business in the ratio of 55% by the winning company and 15% each by the

© The Institute of Chartered Accountants of India


Page 207
PAPER – 6D: ECONOMIC LAWS 3

remaining companies of the total business generated. They also agreed on the
premiums to be quoted by each of them in response to the tender. The minutes of
the meeting signed by officials of aforementioned companies stated to share the
business among the four Insurance Companies with insurance Company with 55%
and other Companies with 15% each. D Insurance Company will be L1 and other
three insurance companies will be L-2 to L-4 in the quotation being submitted on
28 th December, 2009 as per the decision taken in the above meeting.
(iv) Seven insurance companies including the A, B, C, & D Insurance Company
submitted the tender documents. The Technical Evaluation Committee (TEC) formed
by the State Government ‘Z’ evaluated the bids on the basis of a scoring system.
The TEC decided that the companies which scored 50 marks and above (a
benchmark set by the TEC through ratings) would be declared successful in the
technical rounds. As such, only C and D insurance Company were declared
successful and their financial bids were opened in the presence of the
representatives of the respective insurance companies. TEC recommended
acceptance of D Insurance Company’s bid for implementation of CPP scheme being
the lowest in the State ‘Z’ for a period of three years subject to yearly basis
renewals. D Insurance Company was awarded the tender on the basis of
comparative bids mentioned as under:
Details of Price Bids relating to the Tender dated 1.11.2009 for 2010-11.
S. Participating Whether Marks Awarded Premium Amt. as
No. Insurance Technically in Technical stated in Bid ( `)
company Qualified Evaluation
Without With ST
S.T @ 10.3%
1 D Yes 76 521 575
2 C Yes 63 597 658
3 E No 49 509 561
4 F No 45 599 652
5 B No 49 590 651
6 A No 47 580 640
7 G No 48 775 854
(v) Accordingly, D Insurance Company won the tender for 2010-11 and later on shared
its business with A, B & C Insurance Company in their agreed mutual model sharing
ratio. The tender was issued for a period of three years. However, towards the end
of the first year of the contract, D Insurance Company sought for an upward revision
of premium to `1,000/- per family. When this request of D Insurance Company was
turned down by the State Government 'Z'; D Insurance Company invoked the exit
clause of the contract. As a result of this action, the State Government retendered.

© The Institute of Chartered Accountants of India


Page 208
4 FINAL EXAMINATION: MAY 2018

(vi) Post Retendering Scenario: It was found that the price rise effected by the
Insurance companies - A, B, C & D Insurance Company could not have been based
on any rational business justification as the retender for the year
2011- 12 and 2012-13 was won by E Insurance Company at a much lower premium
of ` 840/- per family. The awarded contract was even extended with the same
premium for the year 2012-13, 2013-14 and 2014-15 i.e. for a period of three years
and this contract was renewed for the year 2014-15 at the same price. E Insurance
Company confirmed that the company was not incurring any losses for providing
health insurance services under CPP scheme. The details of rates of these
Insurance companies in relation to the tenders of 2010-11 to 2012-13 are mentioned
as under:
Details of Insurance companies rates bids in relation to tenders of 2010-11
to 2013-14
Price Bids ( `)
S. Name of 2010-11 2011-12 2012-13 2013-14
No. Insurance
company
Without With Without With Without With Without With
S.T ST S.T ST S.T ST S.T ST
1 2 3 4 5 6 7 8 9 10
1 A 580 640 850 938 1700 1875 900 994
2 B 590 651 850 938 1250 1392 1100 1214
3 C 597 658 910 1004 1400 1546 920 1016
4 D 521 575 1000 1104 1000 1104 1000 1104
5 E **509 561 840 927 840 927 840 927
** Not technically qualified
(vii) It was observed that the State Government entrusted its agency named ULTRA to
implement CPP scheme in letter and spirit in the State and this agency had actually
facilitated continuance of D Insurance Company as the insurer under these
schemes by employing an arbitrary practices. A, B, C & D Insurance Companies
have claimed that until 2002, all of them were owned by General Insurance
Company.
It was also submitted that pursuant to the enactment of the General Insurance
Business (Nationalization) Amendment Act, 2002, Government of India holds 100%
shares of each of them and controls the management and affairs of the companies
through Department of Financial Services (Insurance Division), Ministry of Finance.
In this regard, a reference may be had to the policy reforms introduced by the
Government of India in 1991 which led to the de-regulation of the Indian economy.

© The Institute of Chartered Accountants of India


Page 209
PAPER – 6D: ECONOMIC LAWS 5

With the commencement of private participation, a need was felt to modify the
existing market structure of certain select sectors, including, the insurance sector so
as to promote orderly growth of these sectors.
In this regard, the Government of India established a committee in the year 1993
under the chairmanship of Shri R. N. Malhotra (former Governor of the Reserve
Bank of India) to propose reforms for the insurance sector. Pursuant to the
recommendations of the Malhotra Committee, two major regulatory changes were
introduced, including, ending the monopoly of General Insurance Company in the
general insurance business and ending the control exercised by General Insurance
Company over its wholly owned subsidiaries.
These regulatory changes were ushered in to allow the public sector insurance
companies to act independently and to compete with the private players to offer
better services to consumers.
(viii) Further, A, B, C & D Insurance Companies submitted that all decisions relating to
submission of bids, determination of bid amounts, business sharing arrangements,
etc. were taken internally at company level without any ex ante approval/ directions
from Ministry of Finance. Even the decisions taken by the companies were not
notified ex post to the Ministry. These companies participated in the above said
tenders, independent of Ministry of Finance.
(ix) Details of Business Sharing Arrangement among A, B, C & D Insurance Companies
relating to the Tender dated 1.11.2009 are tabulated as under:
Details of Business Sharing Arrangement relating to the Tender dated
1.11.2009
Total Business Generated for D Insurance Company: `92,94,65,400/-
S. Name of Insurance Business Sharing Business sharing (in
No. Company (in term of %) term of revenue ( `)
1 A 15 13,94,19,810.00
2 B 15 13,94,19,810.00
3 C 15 13,94,19,810.00
4 D 55 51,12,05,970.00

(x) Turnover of the A,.B,C & D Insurance Companies in the last three financial years
based on the financial statements were as under :
S. Name of the Insurance Annual Turnover ( `In crore)
No. Company
2010-11 2011-12 2012-13
1 A 6000 7660 9575

© The Institute of Chartered Accountants of India


Page 210
6 FINAL EXAMINATION: MAY 2018

2 B 5400 6745 7853


3 C 7600 7500 8765
4 D 6745 7352 7872
You are required to analyse, with reference to the Competition Act Provisions.
Q.1 Whether the public sector insurance companies i.e., A, B, C & D Insurance
Company constitute a single economic entity? Explain. (5 Marks)
Q.2 Examine whether the A, B, C & D Insurance Companies by their conduct have
entered into an agreement and have contravened any of the provisions of the
Competition Act. Explain. (10 Marks)
Q.3 The State Government 'Z' has now desired to include a specific clause in the bid
document to prevent abuse of the Competition Act. What key clauses would you
recommend? Please draft your reply within a total of 100-200 words. (5 Marks)
Q.4 Assume a situation where the agreement and the meeting of IFG took place outside
India. Explain whether the provisions of the Act still be applicable. (4 Marks)
Q.5 Chairman of the Competition Commission of India, based upon the facts of the
above case, has requested you as an officer of the Commission to draft a brief show
cause notice that should be issued to the insurance companies alleged to be in
default. Your notice should cover the following aspects namely Authority issuing the
notice, Defendant details, Alleged contraventions, Facts as available and Time line
for the response by the defendant. Also include the relevant provisions which
empower such notices to be issued. (6 Marks)
(B) You are the Chairman of Competition Commission of India (CCI) under the Competition
Act, 2002 (hereafter, the Act) as amended in 2007 and subsequently you are chairing the
Bench to deal with information filed under section 19(1) (a) of the Act relating to the radio
taxi market, alleging abuse of dominance and predatory pricing. You do not own a car.
For official journeys, you are provided with an office vehicle. For private use, you
generally avail of the facility available in the market of radio taxis, fitted with GPS
instruments. Therefore, you are fully aware of the radio taxis available in the market and
exposed to the methodology of requisitioning a taxi for personal use and of paying for the
service.
Informants A and B are engaged in the business of providing radio taxi services in a
certain city XXX in South India under the brand names “Press and Hail a Taxi” and “Taxi
before you blink”, A large Radio Taxi provider C is also in the market competing with
Radio Taxi providers A and B and some others too. Informants A and B filed before the
CCI separate information under Section 19 (1) (a) of the Act alleging that Radio Taxi
provider C had abused its dominant position by engaging in predatory pricing in the
relevant market by offering heavy discounts to passengers and incentives to cab drivers,
in contravention of Section 4 (2) (a)(ii) of the Act. Radio Taxi provider C was in the habit

© The Institute of Chartered Accountants of India


Page 211
PAPER – 6D: ECONOMIC LAWS 7

of having oral agreements with customers thus practising an opaque behaviour


prejudicing the interests of A and B.
Informants alleged that C controlled over 50% of a highly concentrated market,
demonstrating C's dominance. The Informants also alleged that there were considerable
entry barriers present which had made it difficult for a new player to effectively compete.
Consistent payment of high incentives and discounts along with exclusivity clauses in
agreements with drivers allowed C to thwart effective competition, lock-in drivers and
create a wide base of customers.
Additionally, the Informants alleged that the presence of an extensive network of C
across the city XXX had acted as a sufficient detriment to any countervailing buying
power available with consumers. They alleged that the presence of a large network of C
had restricted the power of consumers to negotiate and had substantially restricted
competition in the market for other Radio Taxis in the city XXX.
Based on the high market share of C, the Commission arrived at the prima facie view that
C held a dominant position in the relevant market of "Radio Taxi services" in city XXX and
directed the Director General ("DG") to conduct a detailed investigation into the matter.
Findings of the DG
The DG recognized the different business models prevailing in the radio taxi service
industry i.e. asset-owned model, aggregator model and hybrid model. He noted that while
C functioned under the aggregator model, its services were functionally substitutable with
those provided by other taxis operating under the different business models.
Accordingly, the DG concluded that the relevant product market would be the "market for
radio taxi services" and the relevant geographic market would be the city of XXX.
The DG compared the number of trips/rides undertaken by different players in the
relevant market between 2012 and 2016 to observe that while C did grow at a meager
rate of 63% between January and September of 2015, Informant A's trip size registered a
phenomenal growth of 1200% in the same period. He noted that A was an aggressive
player in the market and that the rise of A as a healthy competitor defeated the argument
of the presence of entry barriers. The DG concluded that C was not in a dominant
position, given these facts.
Informants had alleged that C had access to funds and had availed of the same in big
measure, thwarting the other operators to avail of funds. This, according to them, was an
entry barrier. DG found that no evidence had been supplied by the Informants to
substantiate this entry barrier allegation. DG dismissed the allegation as not proved.
Answer the following 10 Multiple Choice Questions by selecting the most appropriate
answer from the options given for each question. Write a few lines justifying your stance.
(10 x 2= 20 Marks)

© The Institute of Chartered Accountants of India


Page 212
8 FINAL EXAMINATION: MAY 2018

(i) The oral agreements between Radio Taxi provider C and some customers, falling
within Section 2(b) of the Act ___________.
(a) are not legally enforceable
(b) are legally enforceable
(c) are not anti-competitive
(d) are not actions in concert
(ii) Dominance under the Act should be determined on the basis of ______________.
(a) market share
(b) price leadership.
(c) profitability
(d) ability to operate independently of competitive forces in the relevant market
(iii) Relevant market is made up of ______________.
(a) relevant geographic market
(b) relevant product market
(c) relevant geographic market and relevant product market
(d) market structure and size alone
(iv) Abuse of dominance by a dominant enterprise arises ___________.
(a) if the enterprise imposes unfair or discriminatory condition in purchase or sale
of goods or service
(b) if the enterprise imposes discriminatory condition or price to meet competition
(c) if the enterprise makes a sizeable profit in its activities
(d) if the enterprise is a price leader
(v) Predatory pricing arises when an enterprises __________.
(a) prices its product very high
(b) prices its product just below the prevalent market price
(c) prices its product to clear inventory
(d) prices its product below its cost of production with a view to reducing
competition or eliminating competitors
(vi) Two Enterprises ____________.
(a) can be in a dominant position at the same time
(b) cannot be in a dominant position at the same time

© The Institute of Chartered Accountants of India


Page 213
PAPER – 6D: ECONOMIC LAWS 9

(c) can be dominant only if they merge


(d) can be dominant only if one acquires the other
(vii) Abuse of dominance does not arise if___________.
(a) the enterprise limits or restricts production of goods or provision of services.
(b) the enterprise limits or restricts technical and scientific development relating to
goods or services to the prejudice of consumers.
(c) the enterprise does not indulge in practices resulting in denial of market
access.
(d) the enterprise uses its dominance in one relevant market to enter into other
relevant market.
(viii) CCI cannot make enquiry into alleged contravention of the provisions in Section 3
and 4 ______.
(a) on unfounded rumours
(b) on its own motion
(c) on receipt of information from consumers or trade associations
(d) on receipt of a reference from the Central Government or State Government
(ix) The parties requesting for confidentiality of information or documents submitted
during the investigation shall have to satisfy the conditions laid down in regulation
_____ of the Competition Commission of India (General) Regulations, 2009.
(a) 42
(b) 39
(c) 35
(d) None of the above
(x) Relevant product market will have to reckon
(a) regulatory trade barriers
(b) physical characteristics or end-use of goods
(c) national procurement policies
(d) transport costs
Answers to Part (A) of Case study 1

Answer 1
Yes, the Public insurance companies, A, B, C & D Insurance company constitute a single
economic entity, which means that companies associated with each other through the virtue of

© The Institute of Chartered Accountants of India


Page 214
10 FINAL EXAMINATION: MAY 2018

common control operate. These Companies formed an Insurance Facilitation Group (IFG) with
the objective of a common cause of furtherance and development of insurance business in
India and all these companies were members of IFG.
This common control operate can be considered as cartel defined in section 2(c) of the
competition Act, 2002.
"Cartel" includes an association of producers, sellers, distributors, traders or service providers
who, by agreement amongst themselves, limit, control or attempt to control the production,
distribution, sale or price of, or, trade in goods or provision of services.
Alternative answer
It is observed that although the public sector insurance companies namely A, B, C and D
Insurance company are presently under the supervision of the Central Government, each of
them placed a separate bid in response to the tenders issued by the State Government for
implementation of the CPP scheme.
Further, the Insurance companies themselves have submitted that all decisions relating to
submission of bids, determination of bid amounts, business sharing arrangements, etc. were
taken internally at company level without any ex ante approval/ directions from the Ministry of
Finance. Thus, it is apparent that these companies participated in the impugned tenders
independent of Ministry of Finance.
In view of the above, it is concluded that bid offers submitted by the A, B, C and D Insurance
companies in response to the Tender issued by the State Government ‘Z’ in relation to the
CPP were based on their own volition and the Ministry of Finance had no role to play. The
Ministry of Finance did not exercise any de facto or de jure control over business decisions of
these companies in submitting bids for impugned tenders. As such, these insurance
companies do not constitute a single economic unit.
Answer 2
Yes, A, B, C, & D insurance companies have entered into an agreement for sharing the
business on a basis of business sharing model in the ratio of 55% by the winning company
and 15% each by the remaining companies of the total business generated.
Insurance companies through an agreement between them quoted the bids rate, which has
the effect of eliminating or reducing competition for bids or adversely affecting or manipulating
the process for bidding.
These are anti-competitive agreements defined under section 3 of the Competition Act, 2002.
According to the section, it shall not be lawful for any enterprise or association of enterprises
or person or association of persons to 'enter' into an agreement in respect of production,
supply, storage, distribution, acquisition or control of goods or provision of services, which
causes or is likely to cause an appreciable adverse effect on competition within India. All such
agreements entered into in contravention of the aforesaid prohibition shall be void.

© The Institute of Chartered Accountants of India


Page 215
PAPER – 6D: ECONOMIC LAWS 11

Any agreement entered into between enterprises or associations of enterprises or persons or


associations of persons or between any person and enterprise or practice carried on, or
decision taken by, any association of enterprises or association of persons, including cartels,
engaged in identical or similar trade of goods or provision of services, shall be presumed to
have an appreciable adverse effect on competition, in the following manner, where it—
(a) directly or indirectly determines purchase or sale prices;
(b) limits or controls production, supply, markets, technical development, investment or
provision of services;
(c) shares the market or source of production or provision of services by way of allocation of
geographical area of market, or type of goods or services, or number of customers in the
market or any other similar way;
(d) directly or indirectly results in bid rigging or collusive bidding.
Accordingly, in the given case, the agreement between them A, B,C,& D insurance companies
results in the anti-competitive agreements, and thus contravened the provisions of the
Competition Act.
Alternative Answer
“In the given case, these insurance companies had held a meeting under the auspices of IFG
on 27.12.2009 at XYZ place in the State 'Z' with the sole agenda to discuss the ‘Tender Notice
on CPP dated 1.11.2009 of the State Government 'Z', The meeting was held to discuss about
sharing of business and submission of quotation for the above business", The minutes of the
meeting of IFG signed by officials of aforementioned companies indicated that a decision was
taken ‘to share the business among the four PSUs with D Insurance Ltd. with 55% and other
Companies with 15% each …D Insurance Company will be LI and other three insurance
companies will be L-2 to L-4 in the quotation being submitted on 28th December, 2009'.
It is a fact that the decision taken by these companies in the above mentioned IFG meeting
was implemented by them. It is clear that the price quoted by these companies in their price
bids was in accordance with the decision taken in the IFG meeting held on 27.12.2009. In line
with the decision taken in the IFG meeting, D Insurance Company was the L-l bidder.
In terms of the provisions contained in section 3(1) of the Act, no enterprise or association of
enterprises or person or association of persons can enter into any agreement in respect of
production, supply, distribution, storage, acquisition or control or goods or provision of
services, which causes or is likely to cause an appreciable adverse effect on competition
within India. Section 3(2) of the Act declares that any agreement entered into in contravention
of the provisions contained in sub-section (1) shall be void.
By virtue of the presumption contained in subsection (3), any agreement entered into between
enterprises or associations of enterprises or persons or associations of persons or between
any person and enterprise or practice carried on, or decision taken by, any association of
enterprises or association of persons, including cartels, engaged in identical or similar trade of

© The Institute of Chartered Accountants of India


Page 216
12 FINAL EXAMINATION: MAY 2018

goods or provision of services, which-(a) directly or indirectly determines purchase or sale


prices; (b) limits or controls production, supply, markets, technical development, investment or
provision of services; (c) shares the market or source of production or provision of se rvices by
way of allocation of geographical area of market, or type of goods or services, or number of
customers in the market or any other similar way; (d) directly or indirectly results in bid rigging
or collusive bidding, shall be presumed to have an appreciable adverse effect on competition.
It may also be pointed out that explanation appended to section 3(3) of the Act defines 'bid
rigging' as any agreement, between enterprises or persons engaged in identical or similar
production or trading of goods or provision of services; which has the effect of eliminating or
reducing competition for bids or adversely affecting or manipulating the process for bidding.
In view of the above, it is concluded that conduct of A, B, C & D Insurance Companies have
resulted in manipulation of the bidding process initiated by the State Government in
contravention of the provisions of section 3(1) read with section 3(3)( d) of the Act. In case of
agreements as listed in section 3(3) of the Act, once it is established that such an agreement
exists, it will be presumed that the agreement has an appreciable adverse effect on
competition; the onus to rebut the presumption would lie upon the opposite parties".
Further, the insurance companies A, B, C & D have entered into an agreement (in writing as
per the minutes of IFG meeting) to manipulate the tendering process initiated by Z State
Government/ULTRA for implementation of the scheme for the years 2010-11, 2011-12, 2012-
13 in accordance with the provisions of section 2(b) of the Act. It is clearly and unequivocally
established. Section 2(b) of the Act defines the term 'Agreement'. Accordingly, the term
Agreement includes arrangement or understanding or action in concert (i) whether or not, such
arrangement, understanding or action is formal or in writing (ii) whether or not such
arrangement, understanding or action is intended to be enforceable by legal proceedings .”
Answer 3
To prevent abuse of Competition Act. It is advised that the following clauses be included by
the State Government “Z” to prohibits abuse of dominant position by any enterprise or group.
An enterprise or a group, does not-
(a) directly or indirectly, imposes unfair or discriminatory condition in purchase or sale of
goods or services; or price in purchase or sale (including predatory price) of goods or
service, or
(b) limits or restricts the production of goods or provision of services or market therefor; or
technical or scientific development relating to goods or services to the prejudice of
consumers; or
(c) indulges in practice or practices resulting in denial of market access in any manner; or
(d) makes conclusion of contracts subject to acceptance by other parties of supplementary
obligations which, by their nature or according to commercial usage, have no connection
with the subject of such contracts; or

© The Institute of Chartered Accountants of India


Page 217
PAPER – 6D: ECONOMIC LAWS 13

(e) uses its dominant position in one relevant market to enter into, or protect, other relevant
market.
Answer 4
As per section 32 of the Competition Act, 2002, where-
(a) an agreement referred to in section 3 has been entered into outside India; or
(b) any party to such agreement is outside India; or
(c) any enterprise abusing the dominant position is outside India; or
(d) a combination has taken place outside India; or
(e) any party to combination is outside India; or
(f) any other matter or practice or action arising out of such agreement or dominant position
or combination is outside India;
The Commission shall, have power to inquire under the various provisions of the Act into such
agreement or abuse of dominant position or combination if such agreement or dominant
position or combination has, or is likely to have, an appreciable adverse effect on competition
in the relevant market in India and pass such orders as it may deem fit in accordance with the
provisions of this Act.
Though the agreement and the meeting of IFG took place outside India, but have an
appreciable adverse effect on competition in the relevant market in India, so the provisions of
the Competition Act are applicable.
Answer 5
Drafting of show cause notice
To,
A, B, C, & D Insurance Companies
-----------------------
New Delhi-110014
12 th May, 2018
Subject: Show cause notice for entering into anti-competitive agreement or combination
of an enterprise for abusing of dominant position
The Chairman, CCI, has noticed that an agreement / combination of the A, B, C, & D
Insurance Companies in response to the tender issued by the State government (Z), for
selection of the insurance service provider for implementation of the CPP, insurance, is likely
to cause, or has caused an appreciable adverse effect on competition and abuse of dominant
provision under section 3 and 4, within the relevant market in India.

© The Institute of Chartered Accountants of India


Page 218
14 FINAL EXAMINATION: MAY 2018

All the service providers as aforesaid, are required to respond within thirty days of the receipt
of the notice, as to why investigation in respect of such an agreement/combination should not
be conducted under section 29 of the Competition Act, 2002.
Chairman
CCI

Answers to Part B of Case study 1


(i) Answer (a): Are not legally enforceable
Reasoning: The oral agreements between radio taxi provider C and customers are
presumed to have an appreciable adverse effect on competition. These agreements
are void, so they are not legally enforceable.
(ii) Answer (d): Ability to operate independently of competitive forces in the relevant
market
Reasoning: Dominant position means a position of strength, enjoyed by an
enterprise, in the relevant market, in India, which enables it to —
(a) operate independently of competitive forces prevailing in the relevant market; or
(b) affect its competitors or consumers or the relevant market in its favour.
[Explanation to section 4]
(iii) Answer (c): Relevant geographic market and relevant product market
Reasoning: "Relevant Market" means the market, which may be determined by the
Commission with reference to the relevant product market or the relevant geographic market
or with reference to both the markets; [Section 2(r)]
(iv) Answer (a): If the enterprise imposes unfair or discriminatory condition in purchase or
sale of goods or service
Reasoning: According to section 4 of the Competition Act, 2002, there shall be abuse of
dominant position if an enterprise or a group, directly or indirectly, imposes unfair or
discriminatory condition in purchase or sale of goods or services; or price in purchase or
sale (including predatory price) of goods or service.
(v) Answer (d): prices its product below its cost of production with a view to reducing
competition or eliminating competitors
Reasoning: "predatory price" means the sale of goods or provision of services, at a
price which is below the cost, as may be determined by regulations, of production of
the goods or provision of services, with a view to reduce competition or eliminate the
competitors.

© The Institute of Chartered Accountants of India


Page 219
PAPER – 6D: ECONOMIC LAWS 15

(vi) Answer (b): cannot be in a dominant position at the same time


Reasoning: Dominant position can be enjoyed by an enterprise, in the relevant
market, in India, which enables it to operate independently of competitive forces
prevailing in the relevant market. Therefore two enterprises cannot be in a dominant
position at the same time.
(vii) Answer (c): the enterprise does not indulge in practices resulting in denial of market
access
Reasoning: According to Section 4(2)(c) of the Competition Act, 2002, there shall be abuse
of dominant position if an enterprise or a group indulges in practice or practices resulting in
denial of market access in any manner. Therefore non indulgences in practices resulting in
denial of market access by the enterprise is not a abuse of dominance.
(viii) Answer (a): on unfounded rumours
Reasoning: Section 19 of the Competition Act, 2002, lays down the procedure for any inquiry
which can be initiated suo motu by the Commission, on receipt of a reference from the
Central Government or a State Government and on the on receipt of an information from
consumers or trade associations.
(ix) Answer (c): 35
According to section 30(3), the parties requesting for confidentiality shall file an affidavit
as specified in regulation 42 of the Competition Commission of India (General)
Regulations, 2009 stating that the conditions prescribed in regulation 35 of the
Competition Commission of India (General) Regulations, 2009 are satisfied.
(x) Answer (b): Physical characteristics or end use of goods
Reasoning: "Relevant Product Market" means a market comprising all those products or
services which are regarded as interchangeable or substitutable by the consumer, by
reason of characteristics of the products or services, their prices and intended use;
[Section 2(t)]

Case study No. 2


(A) A Corporate Insolvency Resolution process, under the Insolvency and Bankruptcy Code
2016 was initiated by M/s A Limited as a Corporate Debtor. The company was in default
to its creditors and the assets were insufficient to meet the liabilities of the company.
Attempts to resolve the insolvency of the corporate debtors failed and in the last, it was
decided to go for liquidation of the company. The balance sheet and additional
information of A Ltd. are given below:

© The Institute of Chartered Accountants of India


Page 220
16 FINAL EXAMINATION: MAY 2018

Key Financial Information:


Data Amount Data Amount
( `In crore) ( `In
crore)
Equity Share Capital 11,000 Land & Building 16,500
Preference Share Capital 3,800 Fixtures & Fittings 1,000
Term Loan 1,500 Stocks 640
Working Capital Loan 1,200 Debtors 550
Unsecured Financial
1,000 Other current Assets 625
Creditors
Government dues 400 Cash 175
Workman dues 240 Accumulated Losses 2,350
Employee Liability 300
Operational Creditors 2,400
21,840 21,840

Additional Information:
Creditors
(1) Term loan is secured against fixed charge on land & building and fixtures & fittings.
Bank A with an `800 crore term loan outstanding has first charge on the assets and
Bank B with `700 crore outstanding has second charge on the assets.
(2) Working capital loan is provided by Bank C and secured against a floating charge
on debtors stock of the company.
(3) Unsecured financial creditors include a Director X who owns 3% of the share capital
of M/s A Limited with an outstanding loan due to him of `50 crores.
Other Liabilities:
(1) Workman dues represents amount payable for the period of 24 months preceding
the liquidation commencement date.
(2) Employee liability includes `25 crore is outstanding for employees for a period of
12 months.
(3) Last three years of tax assessment pending total demand raised by the department
is `1200 crore. This has not been included in the balance sheet, but reflected as a
contingent liability only. However the liquidator has managed to get an assessment
completion certificate and agreed to a final liability of `300 crore.

© The Institute of Chartered Accountants of India


Page 221
PAPER – 6D: ECONOMIC LAWS 17

Fixed Assets & Other Assets :


(1) Land & Building realized 70% of book value and there would be a cost of ` 175
crore in realizing the assets.
(2) Fixtures & fittings would realize 30% of book value, net of any realization cost.
Stock, debtors & other current assets would realize 65% of book value.
Other information:
(1) There was a pending insurance claim filled by the company for a quality breach by a
supplier, which was not recorded in the books. The liquidator has managed to
recover `150 crore from the insurance company.
(2) Lease for the office premises had a lock in period of 10 years, out of which three
years have expired. The landlord has submitted a claim of ` 120 crore for the
remaining seven years of the lease period.
(3) Based on the amount realized & distributed, the cost of liquidation is computed to
be `140 crores.
(4) The pending insolvency period cost was ` 80 crore, mainly including interim
funding, remuneration of the IP and other such costs as permitted under the Code.
(5) The secured creditors have decided to relinquish their security interest to the
liquidation estate and receive proceeds from the sale of the liquidation assets by the
liquidator as per provisions laid under the Insolvency and Bankruptcy Code, 2016.
You are required to find out following with reference to the relevant provisions laid under
the Insolvency and Bankruptcy Code, 2016:
Q.1 What would have been the constitution of the Committee of Creditors and what
would have been the voting share of each of the members of the committee?
(2+3 = 5 Marks)
Q.2 Total value realized by liquidator. (4 Marks)
Q.3 Order of Priority with Notes indicating the relevant section of the Code.
(8 Marks)
Q.4 You have been appointed as the Interim Resolution professional of A Ltd. Draft a
public notice as required under the Act and Regulations. (5 Marks)
Q.5 The application before NCLT was filed on 5 th January, 2018. The case was admitted
on 20 th January, 2018. The IRP who was appointed on 20 th January, 2018, received
the order on the same day and issued public notice on 23rd January, 2018 seeks
your guidance on the various time lines to be compiled with. Prepare a checklist for
his ready reference. (5 Marks)
Q.6 In the said case, assume that A Ltd. has transferred an amount of `500 crore to its
subsidiary abroad. The subsidiary has acquired assets for its business purposes.

© The Institute of Chartered Accountants of India


Page 222
18 FINAL EXAMINATION: MAY 2018

How will you, as the liquidator treat the assets of the subsidiary and the shares held
in the subsidiary? (3 Marks)
(B) You are a Chartered Accountant specialising in FEMA related matters. You are back in
office after a short trip and your assistant has compiled all clients' queries on which your
opinion is requested. Choose the most appropriate reply and write a few lines justifying
your stance.
(i) Mr. Patel's mother requires to travel to USA for a complicated brain surgery. The
estimate given by the hospital in USA is USD 3,00,000 over and above Mr. Patel
would need USD 50,000 towards lodging boarding and other incidental expenses.
Mr. Patel had already spent USD 2,00,000 during the concerned Financial Year. Mr.
Patel can remit from India _________.
(a) USD 2,50,000
(b) USD 3,00,000
(c) USD 3,50,000
(d) USD 1,00,000
(ii) Mr. Smith is deputed to India by his company to develop a strategic software for a
period of five years from 1st January, 2015. He is paid salary to his Indian bank
account. On 1 st May, 2017 he wants to remit his entire salaries ended till 30 th April,

2017 to his home country USA. Mr. Smith can __________.


(a) remit the salary after payment of applicable taxes and contribution to
applicable social security schemes
(b) cannot remit any amount as salary is credited to his bank account in India
(c) remit gross salary before taxes and can make payment of taxes at the year
end
(d) remit salary only upon completion of assignment after payment of taxes and
filing of Income tax return
(iii) Mr. John, an Australian citizen of non-Indian origin is engaged in construction of
farm houses in Australia. He intends to take 50% stake in an Indian company which
is engaged in construction of residential premises in Jammu. Mr. John __________.
(a) cannot make any investment in the Real Estate Sector
(b) can invest through his company in Australia
(c) can make direct investment for construction of residential premises
(d) Both (a) and (b) above
(iv) Mr. Mehra intends to return to India for good after 30 years of stay in USA.
Mr. Mehra needs to ____________.

© The Institute of Chartered Accountants of India


Page 223
PAPER – 6D: ECONOMIC LAWS 19

(a) close all his bank accounts in USA and remit funds to India
(b) liquidate all his investments before returning to India
(c) bring minimum of USD 2,50,000 to India for his survival
(d) can retain his money, bank accounts, investments etc. abroad without any
restrictions
(v) Mr. Kale migrated to UK 20 year ago. He later on acquired UK citizenship. He
inherited 50 acres of agricultural land in Maharashtra which has an inbuilt Farm
House. Mr. Kale intends to gift or sell this property to his only son who has UK
citizenship, but settled in India. Mr. Kale ___________.
(a) can gift this property to his son but cannot sale it
(b) can neither gift nor sale this property to his son
(c) can sale this property to his son but cannot gift it
(d) can do both, gift as well as sale this property to his son
(vi) Mr. lyer an Indian resident acquired a residential flat in Malaysia in contravention of
FEMA regulations. Fearing actions, he intends to gift the same to his nephew Mr.
Kartik, who is a resident of India at present but will soon be migrating to Malaysia
for higher studies. Mr. Kartik _________.
(a) can acquire the flat from his uncle by way of gift
(b) cannot acquire the flat from his uncle by way of gift
(c) can acquire the flat by way of inheritance but not as a gift
(d) can acquire the flat by way of sale, gift or inheritance
(vii) M/s Charming Garments has a warehouse in Amsterdam to which goods worth
` 10 crore are exported. The firm needs to realise the proceeds of exports
_________
(a) as soon as exports are made
(b) within nine months from the date of export
(c) as soon as goods are sold or within fifteen months from the date of shipment
of goods whichever is earlier.
(d) within twelve months from the date of shipment of goods
(viii) Mr. Gotad travelled to Germany for attending a conference. He acquired USD 5,000
from his travel agent in India, out of which he saved currency notes worth USD
2,500. Upon his return to India, Mr. Gotad _________.
(a) needs to surrender USD 2,500 to his Authorised Dealer (AD) within six months
of date of return

© The Institute of Chartered Accountants of India


Page 224
20 FINAL EXAMINATION: MAY 2018

(b) needs to surrender USD 2,500 to his AD within ninety days of date of return
(c) can retain USD 2,000 and surrender USD 500 within 90 days of his return to
India
(d) can retain USD 2,500 for his next trip
(ix) For any contravention of FEMA Regulations under section 13 of the Act, where the
sum involved is quantifiable, the quantum of penalty would be __________.
(a) three times of sum involved
(b) rupees two lacs only
(c) upto Rupees five thousand per day of the offence in continue
(d) Both (a) and (c) above
(x) The time limit for compounding of offences under section 13 of FEMA by the
Directorate of Enforcement is
(a) Nine months from the date of application
(b) Six months from the date of committing such contravention
(c) 180 days from the date of receipt of application by the Directorate of
Enforcement
(d) 180 days from the date of application to the Directorate of Enforcement
Answers to Part (A) of Case study 2
Answer 1
In the given case, the committee of creditors will be constituted as per section 21 of the
Insolvency and Bankruptcy Code, 2016.
The members of the committee will comprise all financial creditors excluding related party who
will not have right of representation, participation or voting in the meeting of the committee of
creditors.
Accordingly, the committee of creditors and their voting share will be as under:
S.No. Members Loan Amount Voting Share %
(Rs. Crores)
1 Bank A 800 21.92
2 Bank B 700 19.18
3 Bank C 1200 32.88
4 Unsecured unrelated financial creditors 950 26.02
3650 100

© The Institute of Chartered Accountants of India


Page 225
PAPER – 6D: ECONOMIC LAWS 21

The director X who is an unsecured financial creditor with ` 50 crores, since related party of
the corporate debtor, shall not have any right of representation, participation or voting in the
committee of creditors.
Answer 2
Total Assets that can be realized by the Liquidator of M/S A Limited will be as follows:
Land & Building realized 70% of book value = ` 11,550 Crore
Less: Cost of realization = ` 175 Crore
Net value = ` 11375 Crore
Fixtures & Fittings realize 30 % of book value = ` 300 Crore
Stock, debtor & other current assets would realize 65% of book value = ` 1179.75 Crore
Insurance claim recovered by the liquidator from insurance company = ` 150 Crore
Total value realized by liquidator = ` 13,004.75 Crore.
[Note: Answer may also be given on the assumption of inclusion of amount of cash
available in the amount of total value released by liquidator. In such case total value
released will be 13,179.75 Crore].
Answer 3
Section 53 of the Code lays the provisions related to distribution of assets or the proceeds
from the sale of the liquidation assets.
Distribution of proceeds from the sale of the liquidation assets: The proceeds from the
sale of the liquidation assets shall be distributed in the following order of priority —
(a) the insolvency resolution process costs and the liquidation costs paid in full;
(b) the following debts which shall rank equally between and among the following :—
(i) workmen's dues for the period of twenty-four months preceding the liquidation
commencement date; and
(ii) debts owed to a secured creditor in the event such secured creditor has
relinquished security in the manner set out in section 52;
(c) wages and any unpaid dues owed to employees other than workmen for the period of
twelve months preceding the liquidation commencement date;
(d) financial debts owed to unsecured creditors;
(e) the following dues shall rank equally between and among the following:—
(i) any amount due to the Central Government and the State Government including the
amount to be received on account of the Consolidated Fund of India and the
Consolidated Fund of a State, if any, in respect of the whole or any part of the
period of two years preceding the liquidation commencement date;

© The Institute of Chartered Accountants of India


Page 226
22 FINAL EXAMINATION: MAY 2018

(ii) debts owed to a secured creditor for any amount unpaid following the enforcement
of security interest;
(f) any remaining debts and dues;
(g) preference shareholders, if any; and
(h) equity shareholders or partners, as the case may be.
Fees to liquidator: The fees payable to the liquidator shall be deducted proportionately from
the proceeds payable to each class of recipients, and the proceeds to the relevant recipient
shall be distributed after such deduction.
Particulars Amount (` in Crores)
Value Realized by Liquidator 13,004.75
Add: Cash 175.00
Total Amount of Funds Available 13,179.75
Less: Section 53(1)(a)
insolvency resolution process costs and the liquidation costs.
(i) Cost of Liquidation 140.00
(ii) Insolvency Professional related costs* 80.00 220.00
Balance Available 12,959.75
Less: Section (53)(1)(b)
(i) Workmen's dues for the period of 24 months
preceding the liquidation commencement date 240.00
(ii) Debt owed to a secured creditors
(a) Term loans 1500.00
(b) Working capital loan 1200.00 2940.00
Balance Available 10,019.75
Less: Section(53)(1)(c)
Wages and any unpaid dues owed to employees other than 25.00
workmen for the period of twelve months preceding the
liquidation commencement date
Balance available 9994.75
Less: Section(53)(1)(d)
Financial debts owed to unsecured financial creditors 1000.00
Balance Available 8994.75
Less: Section(53)(1)(e)
Amount due to the Central Government and the State
Government

© The Institute of Chartered Accountants of India


Page 227
PAPER – 6D: ECONOMIC LAWS 23

(i) Government dues 400.00


(ii) Income Tax Liability 300.00 700.00
Balance Available 8294.75
Less: Section(53)(1)(f)
(i) Employee liability (300-25) 275.00 2675.00
(ii) Operational Creditors 2400.00 5619.75
Balance Available
Less: Section(53)(1)(g)
Amount to be given to Preference Shareholders 3800.00
Balance Available 1819.75
Less: Section(53)(1)(h)
Amount to be given to Equity Shareholders 1819.75
Balance Available NIL
[Note 1: Rent claim for unexpired lease period has been considered at nil value as
based on the relevant provisions, payment of periodic nature can only be claimed till
the time order for liquidation is passed
*Note 2: It is assumed that ‘pending insolvency cost of ` 80 crores has not been paid in
full before and now being paid in full].
Answer 4
Draft Public notice to the Creditors of A Ltd., the corporate debtor is as under:
Form A
PUBLIC ANNOUNCEMENT
(Under Regulation 6 of the Insolvency and Bankruptcy Board of India (Insolvency
Resolution Process for Corporate Persons) Regulation, 2016.)
FOR THE ATTENTION OF THE CREDITORS OF A LIMITED
RELEVANT PARTICULARS
1. NAME OF CORPORATE DEBTOR A LIMITED
2. DATE OF INCORPORATION OF CORPORATE DEBTOR
3. AUTHORITY UNDER WHICH CORPORATE DEBTOR IS INCORPORATED /
REGISTERED
4. CORPORATE IDENTITY NUMBER / LIMITED LIABILITY IDENTIFICATION
NUMBER OF CORPORATE DEBTOR
5. ADDRESS OF THE REGISTERED OFFICE AND PRINCIPAL OFFICE (IF ANY ) OF
CORPORATE DEBTOR
6. INSOLVENCY COMMENCEMENT DATE IN RESPECT OF CORPORATE DEBTOR

© The Institute of Chartered Accountants of India


Page 228
24 FINAL EXAMINATION: MAY 2018

7. ESTIMATED DATE OF CLOSURE OF INSOLVENCY RESOLUTION PROCESS


8. NAME AND REGISTRATION NUMBER OF THE INSOLVENCY PROFESSIONAL
ACTING AS INTERIM RESOLUTION PROFESSIONAL
9. ADDRESS AND E -MAIL OF THE INTERIM RESOLUTION PROFESSIONAL , AS
REGISTERED WITH THE BOARD
10 ADDRESS AND E -MAIL TO BE USED FOR CORRESPONDENCE WITH THE
. INTERIM RESOLUTION PROFESSIONAL , IF DIFFERENT FROM THOSE GIVEN
AT SL . NO .9.
11 LAST DATE FOR SUBMISSION OF CLAIMS
.
Notice is hereby given that the National Company Law Tribunal has ordered the
commencement of a corporate insolvency resolution process against the M/S A Ltd. on ---------
-------- [insolvency commencement date].
The creditors of M/S A Ltd., are hereby called upon to submit a proof of their claims on or
before----------------- [within fourteen days from the appointment of the interim resolution
professional] to the interim resolution professional at the address mentioned against item 8.
The financial creditors shall submit their proof of claims by electronic means only. The
operational creditors including workmen and employees may submit the proof of claims by in
person, by post or by electronic means.
Submission of false or misleading proofs of claim shall attract penalties.
Name and Signature of Interim Resolution Professional:
Date and Place:
Answer 5
Checklist for ready reference of various time lines to be complied by IRP within the
Insolvency and Bankruptcy Code are:
Sl.No Process of Insolvency process Timelines
1. Filing of application before NCLT 5th January 2018
2. Admission of application 20 th January, 2018
3. Appointment of Interim Resolution 20 th January 2018 (within 14 days from the
Professional(IRP)- Actual date commencement date)
4. Public announcement -Actual date Uptil 23rd January, 2018(within 3 days from
the date of appointment of the Interim
Resolution Professional)
5. Collation of claims Within 14 days of the date of appointment of
Interim Resolution Professional

© The Institute of Chartered Accountants of India


Page 229
PAPER – 6D: ECONOMIC LAWS 25

6. Verification of claims Within 7 date from last date of submission


of claims
7. Constitution of Committee of Immediate after verification of claims
Creditors
8. Holding first meeting of Committee Within 7 days of constitution of Committee
of Creditors of Creditors
9. Filing of report to Adjudicating Before 30 th day of appointment of IRP
Authority
10. Moratorium 180 days from the date of admission of
application i.e. 18 th July, 2018.
Answer 6
According to section 36 of the code, for the purposes of liquidation, the liquidator shall form an
estate of the assets, which will be called the liquidation estate in relation to the corporate
debtor. The liquidation estate shall comprise all liquidation estate assets which shall include
any depository recording securities of the corporate debtor or by any other means as may be
specified by the Board, including shares held in any subsidiary of the corporate debtor.
However, as per the Insolvency and Bankruptcy Code, 2016, assets of any Indian or foreign
subsidiary of the corporate debtor shall not be included in the liquidation estate assets and
shall not be used for recovery in the liquidation.
So, according to the above provision, the assets of the foreign subsidiary of A Ltd., is
excluded for recovery in the liquidation.
Answers to Part (B) of Case study 2

(i) Answer (c): USD 3,50,000


Reasoning: As per Schedule III of the FEM (Current Account Transactions) Rules, 2000,
Individuals can avail of foreign exchange facility within the limit of USD 2,50,000 only. Any
additional remittance in excess of the said limit shall require prior approval of the Reserve
Bank of India. However, for the purposes of expenses in connection with medical treatment
abroad, the individual may avail of exchange facility for an amount in excess of the limit
prescribed if it is so required by a medical institute offering treatment. Mr. Patel can remit from
India 3,00,000+ 50,000= USD 3,50,000.
(ii) Answer (a): remit the salary after payment of applicable taxes and contribution to
applicable social security schemes
Reasoning: As per Schedule III of the FEM (Current Account Transactions) Rules, 2000, a
person who is resident but not permanently resident in India, who is on deputation to the
office or branch of a foreign company or subsidiary or joint venture in India of such foreign
company, may make remittance up to his net salary, after deduction of taxes, contribution to

© The Institute of Chartered Accountants of India


Page 230
26 FINAL EXAMINATION: MAY 2018

provident fund and other deductions. Accordingly, Mr. Smith can remit the salary after
payment of taxes and contributions related to social security schemes.
(iii) Answer (c): can make direct investment for construction of residential premises
Reasoning: As per the FEM (Permissible Capital Account Transactions) Regulations,
2000, the person resident outside India is prohibited from making investments in India
in any form, in any company, or partnership firm or proprietary concern or any entity whether
incorporated or not which is engaged or proposes to engage in real estate business, or
construction of farm houses. In “real estate business” the term shall not include shall not
include development of townships, construction of residential /commercial premises, roads or
bridges and Real Estate Investment Trusts (REITs) registered and regulated under the SEBI
(REITs) Regulations 2014.
(iv) Answer (d): can retain his money, bank accounts, investments etc. abroad without any
restrictions
Reasoning: As per the Foreign Exchange Management (Foreign Currency Accounts by a
person resident in India) Regulations, 2015, a citizen of a foreign state resident in India may
open, hold and maintain a foreign currency account with a bank outside India.
[Note: This regulation does not form part of the study material. Correct answer given in
common parlance, may be taken into consideration]
(v) Answer (a): can gift this property to his son but cannot sale it
Reasoning: As per the FEM (Acquisition and transfer of immovable property in India)
Regulation, a person of Indian origin resident outside India may transfer any immovable
property in India other than agricultural land/farm house/plantation property, by way of sale to
a person resident in India. Since in the question it an agricultural land, so it will fall in
exception for transfer of property by the way of sale to a person resident in India.
(vi) Answer (b): cannot acquire the flat from his uncle by way of gift
Reasoning: A person resident in India may acquire immovable property outside India, a
person resident in India may acquire immovable property outside India, by way of inheritance
or gift from a person resident in India who has acquired such property in accordance with the
foreign exchange provisions in force at the time of such acquisition. Since in the given case
there was contravention of FEMA regulations, so Mr. Kartik cannot acquire the flat.
(vii) Answer (c): as soon as goods are sold or within fifteen months from the date of
shipment of goods whichever is earlier.
Reasoning: As per FEM (Export of goods and services) Regulation, the amount
representing the full export value of goods / software/ services exported shall be realised and
repatriated to India within nine months from the date of export, provided that where the
goods are exported to a warehouse established outside India with the permission of
the Reserve Bank, the amount representing the full export value of goods exported
shall be paid to the authorised dealer as soon as it is realised and in any case within

© The Institute of Chartered Accountants of India


Page 231
PAPER – 6D: ECONOMIC LAWS 27

fifteen months from the date of shipment of goods.


(viii) Answer (c): can retain USD 2,000 and surrender USD 500 within 90 days of his return to
India
Reasoning: According to Foreign Exchange Management (Possession and Retention
of Foreign Currency) Regulations, 2015, a person resident in India can retain foreign
currency notes, bank notes and foreign currency traveller’s cheques not exceeding
USD 2,000 or its equivalent in aggregate, provided that such foreign exchange in the
form of currency notes, bank notes and travellers cheques represents unspent
amount of foreign exchange acquired by him from an authorised person for travel
abroad.
(ix) Answer (d): Both (a) and (c) above i.e., three times of sum involved and up to rupees
five thousand per day of the offence in continue
Reasoning: According to section 13 of the Foreign Exchange Management Act, 1999,
if any person contravenes any provisions of this Act, or contravenes any rule,
regulation, notification, direction or order issued in exercise of the powers under this
Act, or contravenes any condition subject to which an authorisation is issued by the
Reserve Bank, he shall, upon adjudication, be liable to a penalty up to thrice the sum
involved in such contravention where such amount is quantifiable, or up to two lakh
rupees where the amount is not quantifiable, and where such contravention is a
continuing one, further penalty which may extend to five thousand rupees for every
day after the first day which the contravention continues.
(x) Answer (c): 180 days from the date of receipt of application by the Directorate of
Enforcement
Reasoning: According to section 15 of the Foreign Exchange Management Act,
1999, any contravention under section 13 may, on an application made by the person
committing such contravention, be compounded within one hundred and eighty days
from the date of receipt of application by the Director of Enforcement or such other
officers of the Directorate of Enforcement and Officers of the Reserve Bank as may
be authorised in this behalf by the Central Government in such manner as may be
prescribed.
Case Study No. 3
Everbullish Inc. USA has a subsidiary in Singapore, namely Everbullish Steel Asia Pvt. Ltd.
(ESA) looking after the entire south east Asia, including India.
ESA has following entities operating under it.
(i) A branch in China for manufacturing of steel
(ii) A liaison office in India for marketing of steel exported by ESA directly to Indian
customers.

© The Institute of Chartered Accountants of India


Page 232
28 FINAL EXAMINATION: MAY 2018

(iii) A project office in Afghanistan


(iv) A commission agent in Bangladesh
(v) A warehouse in Srilanka
ESA upgraded its Liaison Office (LO) in India to a full fledged subsidiary as 1 st April, 2016 and
transferred all its balances to the newly formed subsidiary, name Everbullish Indian Steel Pvt.
Ltd. (EISPL)
Note In each of the above situations, you are required to give relevant 'FEMA' and 'Prohibition
of Benami Property Transaction Act, 1988 and references options or steps to regularize the
contraventions, if any.
(A) ESA was advised that since it has a permission to operate as a LO till 31.3.2018, there is
no need to obtain separate approval from RBI for converting or upgrading the same into
a subsidiary. Hence No permission was taken by ESA or EISPL. Incorporation expenses
were spent by the Indian LO out of funds remitted by ESA. EISPL started local trading in
India. The LO was not closed by the ESA and no intimation was filed with RBI till
31-10-2018. (5 Marks)
Question
Are there any FEMA violations in the above transactions, and if so, then what is the way
out? (5 Marks)
(B) Sensing something wrong, EISPL decided to undergo voluntary FEMA compliance audit.
EISPL has appointed you as a FEMA auditor. In the process of audit, you discover
several transactions where FEMA regulations were not adhered to, or compliances
pending. You are required to give your expert opinion on following matters as to what are
the contraventions under FEMA and how they can be regularized?
Question 1
Receipt of Share application money from ESA amounting to `One crore on 1 st April
2017. No compliances are made in this respect as the company was advised that
activities of the EISPL falls under the automatic route of RBI. (5 Marks)
Question 2
ESA had bought a large commercial property on 1 st January, 2016 which was then leased
to EISPL w.e.f, 1 st April 2016 and part of the premises was leased to an unrelated Indian
company w.e.f. 1 st April, 2017. (5 Marks)
Question 3
ESA had sent an adhoc amount of `two crore to EISPL for its day to day requirements.
The funds have been received by the EISPL on 1 st January, 2018. Again no FEMA
compliances are made in this respect. (4 Marks)


date 31-10-2018 will be taken as 31.03.2018

© The Institute of Chartered Accountants of India


Page 233
PAPER – 6D: ECONOMIC LAWS 29

Question 4
EISPL has exported steel worth ` 10 crore to solid steel Gmbh an unrelated German
Company on 1st January 2017. Solid steel has run into financial trouble and therefore
refused to pay. Despite best efforts, EISPL is unable to recover the sum. The directors of
EISPL used to follow up for recovery over phone only and therefore no documentary
evidence is available.
(i) Assuming that the total exports of EISPL for the year ended 31 st March 2017 is
likely to cross `50 crore, can it write off this sum? (3 Marks)
(ii) Assuming that EISPL has imported steel ingots from solid steel amounting to
` 11 crore, in Dec. 2016, which is still outstanding. Can it net off and make the
payment for the balance of `1 crore only? (3 Marks)
(iii) Will your answer change if the import and export transactions would have happened
in December, 2017 and January, 2018 respectively? (2 Marks)
Question 5
EISPL remitted ` one crore to the project office of the ESA in Afganistan in February,
2018. Is it permissible? Will your answer be different if instead of money, steel worth of
`one crore is exported to the Afganistan P.O.? (3 Marks)
Question 6
EISPL exported goods to Srilanka. For that purpose it hired the warehouse of ESA and
paid warehousing charges. Is it permissible? What is the time limit for realising goods
exported by EISPL to its Srilankan Warehouse? (5 Marks)
Question 7
EISPL wants to remit commission to the agent of ESA for exports made by Bangalore.
However the Agent has requested to pay `one crore extra, as advance to be adjusted
against future commission. Looking at the present business scenario, it may take 5 years
to adjust the advance commission paid to the Bangladesh Agent. Is it okay from FEMA
perspective? (5 Marks)
Question 8
One of the directors, of the EISPL is a person of India origin with US citizenship. He
wants to acquire a commercial premises in India and then lease it to the company. Is this
permissible under FEMA? Will your answer be different if that director is a US citizen of
non-Indian origin? (5 Marks)
Question 9
In the process of audit it is observed that one of the directors Mr. Valia of EISPL who,
recently joined company has acquired a large bunglow in Bangalore in the name of his
son who has settled in USA. He purchased the same by paying `10 crore. However, his
son is still studying and has not disclosed this property in his US tax returns. Upon

© The Institute of Chartered Accountants of India


Page 234
30 FINAL EXAMINATION: MAY 2018

enquiry Mr. Valia’s son denies of holding any such property. What are the consequences
in this case under the provisions of the “Prohibition of Benami Property Transaction Act,
1988”. (5 Marks)
Answer (A)
According to the Foreign Exchange Management (Establishment of a Branch Office or a
Liaison office or a project office or any other place of business) Regulations, 2016:
1. Liaison Office (LO) means a place of business to act as a channel of communication
between the principal place of business or Head office or by whatever name called and
entities in India but which does not undertake any commercial/ trading / industrial activity,
directly or indirectly, and maintains itself out of inward remittances received from abroad
through normal banking channel.
2. The validity period of an LO is generally for three years, except in the case of Non -
Banking Finance Companies (NBFCs) and those entities engaged in construction and
development sectors, for whom the validity period is two years only. No further extension
would be considered for liaison offices of entities which are Non-Banking Finance
Companies and those engaged in construction and development sectors (excluding
infrastructure development companies). Upon expiry of the validity period, the offices
shall have to either close down or be converted into a Joint Venture / Wholly Owned
Subsidiary in conformity with the extant Foreign Direct Investment policy.
The question states that ESA has the permission to operate as a LO till 31.3.2018.
Hence, we can deduce that ESA must have got the permission to operate as a LO on
1.4.2016. The facts of the case study also states that ESA upgraded its LO in India to full
fledged subsidiary on 1.4.2016.
From the definition of LO, it can be inferred that trading is not included in the permissible
operation of a LO. As per the question ESA has got the permission to operate as a LO
and not as a subsidiary, hence, the decision to operate in the nature of subsidiary without
informing the concerned authority is incorrect.
In every financial year, liaison office have to submit the annual activity certificate
confirming the activities undertaken along with the Audited financial statements, including
the receipt and payment of account on or before 30 th September of the Year.
Failure to comply the above, will attract penalty as provided in the Foreign Exchange
Management Act, 1999.
[Note: The question has provided that LO is liaisoning for steel business, hence it has
been taken to be in the categories of those engaged in construction and development
sectors.]
Alternative answer
ESA was wrongly advised that it can form a subsidiary without any compliances under FEMA.
RBI grants permission for the Liaison office (LO) office for a Special duration and for specified

© The Institute of Chartered Accountants of India


Page 235
PAPER – 6D: ECONOMIC LAWS 31

activities only. A LO is supposed to adhere to all names under FEMA and comply with
conditions mentioned in the permission from RBI.
So, ESA needs to set right things as follows:
(i) Intimate RBI about closure of LO and transfer of all its assets and liabilities in the new
formed subsidiary EISPL.
(ii) File all pending returns of LO with the Income tax authority and audited accounts with
ROC as well as activity certificate under FEMA with its authorized dealer for the onward
submission to RBI.
(iii) Spending funds on incorporation of a company by a LO is in violation of conditions
attached to the activities of the LO and utilization of funds.
(iv) It is given that EISPL started local trading in India. EISPL can do local trading only in
respect of Cash & Carry wholesale Trading under automatic route of RBI. For any other
category it requires prior approval of RBI.
(v) For various offences/contravention mentioned above ESA needs to approach RBI for
compounding of offences. EISPL shall ensure that its activities remain within the purview
of FEMA reputations. For allotment of share to ESA against the balances transferred from
the LO as well as incorporation expenses. If EISPL intends to remit fund to ESA instead
of allotments of shares, they still it needs to obtain RBI approval.
Answer B.
Answer 1
According to Schedule II to the Foreign Exchange Management (Permissible Capital Account
Transactions) Regulations, 2000, investment in India by a person resident outside India,
through issue of securities by a body corporate or an entity in India and investment therein by
a person resident outside India, is a permissible transaction.
Further, according to the Master Directions on Foreign Investment in India-
An Indian company issuing shares /convertible debentures under FDI Scheme to a person
resident outside India shall receive the amount of consideration required to be paid for
such shares /convertible debentures by:
(i) inward remittance through normal banking channels.
(ii) debit to NRE / FCNR account of a person concerned maintained with an AD
category I bank.
(iii) conversion of royalty / lump sum / technical know how fee due for payment /import of
capital goods by units in SEZ or conversion of ECB, shall be treated as consideration
for issue of shares.

© The Institute of Chartered Accountants of India


Page 236
32 FINAL EXAMINATION: MAY 2018

(iv) conversion of import payables / pre incorporation expenses / share swap can be
treated as consideration for issue of shares with the approval of FIPB. (Now Line
Ministry as FIPB is abolished on 17 th April, 2017.)
(v) debit to non-interest bearing Escrow account in Indian Rupees in India which is
opened with the approval from AD Category – I bank and is maintained with the AD
Category I bank on behalf of residents and non-residents towards payment of share
purchase consideration.
If the shares or convertible debentures are not issued within 180 days from the date
of receipt of the inward remittance or date of debit to NRE / FCNR(B) / Escrow
account, the amount of consideration shall be refunded.
It can be regularized on an application filed to RBI where amount outstanding
towards issue of security is beyond the period of 180 days from the date of receipt.
Alternative answer
Compliances on Shares allotment
Two stages compliance is required in respect of receipt of funds and allotment of shares
under FEMA:
(i) Form ARF needs to be submitted to the authorized dealer AD bank of the company within
30 days of receipt of remittance towards equity shares KYC and Foreign Inward
remittance certificate (FIRC) need to be submitted alongwith form ARF.
(ii) Form ECGPR needs to be filed with AD bank within 30 days of allotment of shares. This
form should be certified by a company Secretary certifying all compliances under the
Companies Act, 2013 and a valuation certificate from a Chartered Accountant certifying
the valuation of shares as per the pricing guidelines under FEMA.
(iii) The FEMA regulation provides that the allotment of shares needs to be completed within
six months of the receipt of funds. Under the companies Act, the shares needs to be
allotted in 3 months. Since share are not allotted within the time frame nor intimation
filed, therefore EISPL need to obtain RBI permission for allotment of shares and apply for
compounding of office.
(iv) It may be noted that automatic route of RBI is available only in respect of compliances
made within the prescribed time frame.
Answer 2
According to the Foreign Exchange Management (Acquisition & Transfer of Immovable
Property in India) Regulations, 2000, provides that a person resident outside India cannot
lease/ rent any part of the property acquired by him.

© The Institute of Chartered Accountants of India


Page 237
PAPER – 6D: ECONOMIC LAWS 33

Hence, ESA cannot lease the said commercial property to EISPL & to an unrelated Indian
Company.
Alternative answer
Foreign companies are allowed buy immovable property in India for the purpose of carrying on
its own business. Form IP is to be filed with RBI for intimating the purchase of property.
However, remittance of sale proceed needs prior approval of RBI. If ESA has ceased its
activities as LO, it cannot continue to hold and lease property to others. Recently RBI has
permitted to lease additional place to related enterprises.
Under the circumstances, ESA needs to regularize the leasing of premises to EISPL. As
LO cannot earn any income in India, a question would arise for the leasing income.
Step to be taken by ESA
(i) Approach RBI with facts of the case
(ii) Obtain specific approval for lease of premises or sale its subsidiary EISPL.
(iii) Apply for compounding of offence as per advice from RBI.
Answer 3
Schedule I to the Foreign Exchange Management (Permissible Capital Account
Transactions) Regulations, 2000, allows loans and overdrafts (borrowing) by a person
resident in India from a person outside India subject to the compliance of guidelines
issued by RBI in this regard. Hence, ESA is advised to comply with the Newspaper
guidelines.
Alternative answer
EISPL has received on adhoc amount of ` 2 crore for its day to day requirements for ESA
on 1 s t January, 2018.
EISPL can take external commercial borrowing (ECB) from its parent company subject to
conditions prescribe in the ECB regulations.
However, any loan under ECB regulations can be drawn only after obtaining loan
Registrations Number (i.e. LRN). In the instance case EISPL has already received the
funds from ESA on 1st January, 2018. It would be better to treat these funds towards
subscription of compulsory convertible debentures (CCDs). Authorized Dealer Bank may
be approached for necessary changes in the FIRC. ESA & EISPL can pass necessary
resolution in this behalf. CCDs are treated at par with equity shares. Hence, EISPL needs
to comply with necessary formalities under the Foreign Direct Investment (FDI)
regulations.
RBI may levy nominal compounding fees for delay in intimation of receipt of funds.
Answer 4
(i) Section 7 of FEMA deals with provisions of Export of Goods and Services.

© The Institute of Chartered Accountants of India


Page 238
34 FINAL EXAMINATION: MAY 2018

It is the duty of the exporter to see that foreign exchange is realized within the
prescribed time limit. The normal time limit for realization of exports is nine months
from the date of export. If for any reason export proceeds are not realized in time,
the AD/RBI bank may be informed and requested to external the time limit.
As per Master Direction – Export of Goods and Services;
An exporter who has not been able to realize the outstanding export dues despite
best efforts, may either self-write off or approach the AD Category – I banks, who
had handled the relevant shipping documents, with appropriate supporting
documentary evidence. The limits prescribed for write-offs of unrealized export bills
are as under:
Self “write off” by an exporter (Other than status holder exporter) 5%*
Self “write off” by status holder exporter 10%*
“Write off” by AD Bank 10%*
* of the total export proceeds realized during the previous calendar year
The above limits will be related to total export proceeds realized during the previous
calendar year and will be cumulatively available in a year.
Thus, EISPL can write off the amount to the extent as prescribed in the above provisions.
Alternative answer
Section 7 of FEMA deals with provisions of Export of Goods and Services.
It is the duty of the exporter to see that foreign exchange is realized within the
prescribed time limit. The normal time limit for realization of exports is nine months
from the date of export. If for any reason export proceeds are not realized in time,
the AD/RBI bank may be informed and requested to extend the time limit.
Exporter needs to main robust documentations of steps taken to realize the
outstanding dues. In the instant case the directions followed up for payment only
over phone and therefore would land up in trouble as they will not be able to prove
that all reasonable efforts were put in to realize the export proceeds. Under the
circumstances, the company may face stringent actions from the Enforcement
Directorate.
Self-write off of exports is permitted upto 10% of the average annual realization of
exports in past 3 years subject to fulfilment of certain other conditions. As EISPL
does not fall into this category specific approval from RBI is advisable.
(ii) As per Master Direction – Export of Goods and Services;
EISPL can set off the amount and make payment for 1 crore only by following the
conditions:

© The Institute of Chartered Accountants of India


Page 239
PAPER – 6D: ECONOMIC LAWS 35

AD category –I banks may deal with the cases of set-off of export receivables against
import payables, subject to following terms and conditions:
(1) The import is as per the Foreign Trade Policy in force.
(2) Invoices/Bills of Lading/Airway Bills and Exchange Control copies of Bills of
Entry for home consumption have been submitted by the importer to the
Authorized Dealer bank.
(3) Payment for the import is still outstanding in the books of the importer.
(4) Both the transactions of sale and purchase may be reported separately in R-
Returns and FETERS (Foreign Exchange Transactions- Electronic Reporting
System).
(5) The relative EDF (Export Declaration Form) will be released by the AD bank
only after the entire export proceeds are adjusted / received.
(6) The set-off of export receivables against import payments should be in respect
of the same overseas buyer and supplier and that consent for set -off has been
obtained from him.
(7) The export / import transactions with ACU countries should be kept outside the
arrangement.
(8) All the relevant documents are submitted to the concerned AD bank who should
comply with all the regulatory requirements relating to the transactions.
Alternative answer
Netting off export of goods receivable and import payable from same party is
permitted under the automatic route, provided the outstanding amounts are within the
time frame prescribed in FEMA. In this case both are overdue and hence specific
approval from RBI would be required.
(iii) The position as stated above in part (ii) will not change even if the import and export
transactions would have happened in December 2017 and January 2018 respectively.
Answer 5
According to Foreign Exchange Management (Export and import of currency) Regulations,
2015, any person resident in India may take outside India (other than to Nepal and Bhutan)
currency notes of Government of India and Reserve Bank of India up to an amount not
exceeding `25,000 (Rupees twenty five thousand only).
Hence, EISPL cannot remit amount of ` 1 crore to the project office of ESA in Afghanistan.
However, EISPL can export steel worth ` 1 crore to project office of ESA in Afghanistan (by
following the guidelines as issued by RBI).

© The Institute of Chartered Accountants of India


Page 240
36 FINAL EXAMINATION: MAY 2018

Answer 6
Remittance of funds for the warehouse rent falls within the current account transactions and
therefore EISPL can freely remit warehouse charges to Sri Lanka.
According to Foreign Exchange Management (Export of Goods and Services) Regulations,
2015, where goods are exported to a warehouse established outside India with the permission
of the Reserve Bank of India, the amount representing the full export value of goods exported
shall be paid to the authorised dealer as soon as it is realised and in any case within fifteen
months from the date of shipment of goods;
The Reserve Bank of India, or subject to the directions issued by that Bank in this behalf, the
authorised dealer may, for a sufficient and reasonable cause shown, extend the period of
fifteen months.
Hence, EISPL can send goods to the warehouse in Sri Lanka. Also, the amount representing
the full export value of goods exported shall be paid to the authorised dealer as soon as it is
realised and in any case within fifteen months from the date of shipment of goods. However,
this period can be extended as mentioned above.
Answer 7
Payment of export commission to an overseas agent is a current account transaction and
hence freely permitted. However, payment of advance commission, lasting for five years
would be regarded as capital account transaction and therefore would require prior approval of
RBI.
EISPL is well advised to approach RBI for remitting advance commission which is in the
nature of loan.
Alternative answer
The Foreign Exchange Management Act, 1999 does not provide for a prohibition for
payment of commission to an agent provided it does not exceed 12.50% of the invoice
value. Hence, EISPL can remit commission to agent of ESA for exports made by
Bangladesh within the above limit.
In view of above, the request to pay ` One crore extra, as advance to be adjusted against
future commission cannot be accepted and is not okay from FEMA perspective as the relative
shipment has not been made.
Answer 8
According to Acquisition and transfer of immovable property in India, Regulations,
A person of Indian origin and resident outside India may acquire immovable property in India
other than an agricultural property, plantation, or a farm house:
Provided that in case of acquisition of immovable property, payment of purchase price, if any,
shall be made out of (i) funds received in India through normal banking channels by way of

© The Institute of Chartered Accountants of India


Page 241
PAPER – 6D: ECONOMIC LAWS 37

inward remittance from any place outside India or (ii) funds held in any non-resident account
maintained in accordance with the provisions of the Act and the regulations made by the
Reserve Bank of India:
Provided further that no payment of purchase price for acquisition of immovable property shall
be made either by traveller’s cheque or by currency notes of any foreign country or any mode
other than those specifically permitted by this clause.
Thus, in the given situation, the said director who is a person of Indian origin with US
citizenship can acquire the commercial premises in India.
According to section 6(3) of the Foreign Exchange Management Act, 1999, a person
resident outside India can acquire or transfer the immovable property in India, other than
a lease not exceeding five years. Thus, the director can lease the said commercial
premises but not for a period exceeding 5 years.
If the director would have been a US citizen of non Indian origin then he will not be
allowed to acquire the property in India.
Answer 9
In the given instant, a director Mr. Valia of EISPL has acquired bungalow in Bangalore in the
name of his son who has settled in USA. Upon enquiry Mr. Valia's son denies of holding any
such property and has also not disclosed in his US tax returns.
The given situation falls within the purview of section 2(9) of the Prohibition of benami
Property Transaction Act, 1988. According to the section benami transaction "means a
transaction or an arrangement (a) where a property is transferred to, or is held by, a person,
and the consideration for such property has been provided, or paid by, another person; and
(b) the property is held for the immediate or future benefit, direct or indirect, of the person who
has provided the consideration.
As per the exception to the above clause, Mr. Valia can hold the property in the name of his
son provided the consideration is paid out of the known sources of the Mr. Valia. This source
is also not disclosed so it is assumed that it is an unauthorized source.
Further, on enquiry, denial of Mr. Valia's Son of holding of any such property, is known in
respect of such property, as a benami transaction.
As of consequential holding of benami transactions, section 3 states that no person shall enter
into any benami transaction. Whoever enters into any benami transaction shall be punishable
with imprisonment for a term which may extend to three years or with fine or with both.
So, Mr. Valia shall be liable under the Prohibition of Benami Property Transaction Act, 1988.

© The Institute of Chartered Accountants of India


Page 242
ELECTIVE PAPER 6D: ECONOMIC LAWS

CASE STUDY 1

Mr. Bhanu Pratap Taneja is a leading real estate developer based in Delhi. In the
last decade, his company Garvit Bhoomi Developers Pvt. Limited having registered
office in Bhikaji Cama Place, New Delhi, had successfully developed four housing
projects – two in Gurgaon and one each in Jaipur and Lucknow. They had a robust
management team. Having been the name behind developing more than five
thousand luxurious apartments with modern amenities, they had the reputation of
delivering the projects well within the promised time.
In the beginning of the year 2015, they launched another project in Indirapuram, UP
by the name Omega Capetown Residency in which 1000 residential units consisting
of 2BHK and 3BHK apartments were to be developed. They were to be completed in
all respects by January 2018 and delivered to the consumers by that date. This
project was being carried on smoothly when the Real Estate (Regulation and
Development) Act, 2016 came to be enacted w.e.f. 1 st May, 2016. Section 3, which

was enacted later w.e.f. 1 st May 2017. Since Omega Capetown Residency consisted
of 1,000 residential units, it was required to be registered and so the company
submitted the requisite documents with concerned authorities.
As the application for registration was found to be complete in all respects, the
Omega Capetown Residency Project was granted registration by RERA (UP) within
the statutory period and was provided with a registration number including a log-in
ID for assessing the website of the Authority and to create webpage.
In the meantime, Mr. Taneja was approached by some of the influential developers
that an understanding had been reached among them to control the price of
apartments to be built by them. However, because of legal tangles such
understanding could not be brought into writing and it was also not intended to be
enforced by legal proceedings. Mr. Taneja did not agree to the proposal because
even though the understanding was not in writing and it was not intended to be
enforceable by legal proceedings, it was still illegal as per the Competition Act, 2002.
This revelation made by Mr. Taneja discouraged the intending developers and they
desisted from being a party to this proposal.
Mr. Taneja’s son Garvit, who was a commerce graduate and holder of law degree,
had a college friend Rohit whose father Mr. Dev Kumar dealt in sale, purchase and
renting of properties under the title ‘Dev Property Dealers’ from the Yusuf Sarai
market. Since Rohit had joined his father’s business, it was thought prudent to
convert the existing proprietary business into a registered partnership firm titled as
‘Dev & Sons Property Dealers’. Because of the enactment of Real Estate Act, Rohit
consulted his friend Garvit regarding its implications in case of real estate agents.
Accordingly, the firm was got registered as real estate agent with the help of Garvit’s
legal advisor.
Further, Garvit made a proposal to Rohit and his father that they could associate
themselves with his Omega Capetown Residency, a registered RERA project in
Indirapuram for facilitating sale of apartments which they readily accepted. Garvit
also cautioned them that as per the Act, since their firm was now a registered real
estate agent they were not supposed to facilitate sale/purchase of any plot,
© The Institute of Chartered Accountants of India

Page 243
apartment or building in a real estate project being sold by the promoter in any
planning area, if such project was not registered with RERA of the concerned State.
In addition, Garvit’s legal advisor told them that as required by Section 10, a
registered real estate agent would maintain and preserve proper books of accounts
and other necessary documents. Further, such agent would not involve himself in
any unfair trade practice like making a false statement regarding services to be
provided by him. He would also not permit the publication of any advertisement
whether in any newspaper or otherwise of services that were not intended to be
offered. Besides, the agent would also have to help the intending buyers in getting
the required information and documents to which they were entitled, at the time of
booking of any property.
Rohit had a friend Tarun whose father Dr. Sreenivas Sharma was a surgeon in a
government hospital and was residing in a rented government flat in the hospital
campus itself. He had an intense desire to have a luxurious flat of his own. Tarun
had joined IBM after doing MBA from IIFT, New Delhi. So, with the combined salary
of both, they decided to buy a flat. Tarun contacted Rohit to help him in searching a
suitable apartment for his family. In turn, Rohit informed him that one particular 3BHK
flat at an ideal location was available in Omega Capetown Residency in Indirapuram
as the original allottee had withdrawn from the scheme; otherwise the booking under
this project was already full. Dr. Sharma got interested in the information and went
to the Omega Capetown Residency along with his family to see the concerned
apartment. He liked its strategic location and gathered more information regarding
sanctioned plan, layout plan along with the other specifications, etc. He then asked
for stage-wise time schedule of completion of the project and also enquired regarding
provision of water, sanitation and other amenities. Since, Rohit personally knew
Garvit and his father Mr. Taneja - the promoters of the project - Dr. Sharma and his
family had a lively and fruitful meeting with them. Subsequently, he and his son jointly
entered into an agreement for sale with the promoters of the project and made
payment of 75% of the cost of the apartment, while remaining 25% of the cost was
to be paid at the time when the apartments were ready for occupation.
A few months after booking the apartment, Dr. Sharma got a notice from the
promoters of Omega Capetown Residency that due to unforeseen circumstances
they were not in a position to complete the project and needed the allottees’ consent
for transferring of their majority rights and liabilities to another reputed developer
M/s. Sai Developers Pvt. Limited of New Delhi. In case any of the allottees was not
agreeable to this proposal he could get his money refunded. Since Dr. Sharma was
very much attached to the location of the flat, he accepted the proposal after
enquiring with Rohit and his father. He also learnt that 95% of the allottees had
already given their written permission. Further, the Authority had given its written
approval to the proposal for transfer and completion of Project by M/s. Sai
Developers Pvt. Limited. Dr. Sharma was also assured by Mr. Bhanu Pratap Taneja,
the erstwhile promoter with whom he had earlier interacted satisfactorily, that all the
pending obligations would be fulfilled by the new developer and in no case the date
of completion of the project would be extended; otherwise it would attract penalty. It
was also disclosed by Mr. Taneja that the new promoter would rectify any structural
defect if it occurred within a period of five years from the date of handing over the
possession of the apartments. Dr. Sharma, thus felt relieved.
© The Institute of Chartered Accountants of India

Page 244
M/s. Sai Developers completed the project on time and received Completion
Certificate from the Competent Authority. As per the agreement for sale, Dr. Sharma
made payment of the remaining 25% of the cost. Thereafter, he received Occupancy
Certificate and took physical possession of the apartment well before two months
since the allottees were supposed to take physical possession within statutory period
of two months from the issue of Occupancy Certificate. He was also given other
necessary documents and plans, including that of common areas. He also became
a member of the RWA formed by the allottees. In the meantime, the promoter
executed a registered conveyance deed in favour of each of the allottees along with
the undivided proportionate title in the common areas to the RWA.
I. Multiple Choice Questions
1. Registration of a real estate project shall not be required –
(a) where the area of land proposed to be developed does not exceed
five hundred square meters or the number of apartments proposed to
be developed does not exceed eight.
(b) where the area of land proposed to be developed does not exceed
five thousand square meters or the number of apartments proposed
to be developed does not exceed eighty.
(c) where the area of land proposed to be developed does not exceed
two hundred fifty square meters or the number of apartments
proposed to be developed does not exceed four.
(d) where the area of land proposed to be developed does not exceed
three hundred square meters or the number of apartments proposed
to be developed does not exceed three.
2. Who is required to submit a copy of duly obtained approvals and
commencement certificate for getting the project registered with RERA:
(a) Allottee
(b) Promoter
(c) Real Estate Agent
(d) None of the above
3. A registered real estate agent shall -
(a) Facilitate the sale/purchase of any plot, apartment or building, being
sold by the promoter in any planning area, which is registered with
the Authority;
(b) maintain and preserve prescribed books of account, records and
documents;
(c) not involve himself in any unfair trade practices
(d) All of the above.

© The Institute of Chartered Accountants of India

Page 245
4. The promoter is required to rectify any structural defect if it occurs within a
period of ----- years from the date of handing over the possession of the
apartments to allottees –
(a) Two years
(b) Three years
(c) Four years
(d) Five years
5. Registration of on-going project for which completion certificate is yet to be
received is mandatory -
(a) Yes, if the area of land (developed or to be developed) exceeds five
hundred square meters or the number of apartments (developed or
to be developed) exceeds eight.
(b) No, irrespective of the area of land or the number of apartments
(c) Can’t say
(d) None of the above
6. A real estate developer can leave the project mid-way by selling that project
to another developer if he has taken a written approval of ---------- of allottees
along with approval of the Authority.
(a) 2/3rd
(b) 1/3rd
(c) 3/4th
(d) 1/4 th
7. The time limit within which an allottee is required to take physical possession
of the apartment after issue of occupancy certificate is -
(a) one month
(b) two months
(c) three months
(d) four months
8. A certificate certifying that the real estate project has been developed
according to the sanctioned plan, layout plan and specifications as approved
by the competent authority under the local laws is called -
(a) Commencement Certificate
(b) Completion Certificate
(c) Occupancy Certificate
(d) None of the above

© The Institute of Chartered Accountants of India

Page 246
9. The flat purchased by Dr. Sharma jointly with his wife Mrs. Neelima Sharma
though funded by him would be held as ‘benami transaction’ under the
Prohibition of Benami Property Transactions Act, 1988 -
(a) Yes
(b) No
(c) Can’t say
(d) None of the above
10 As per the Competition Act, 2002 ‘Agreement’ includes any arrangement or
understanding or action in concert:
(a) Whether or not, such arrangement, understanding or action is formal
or in writing; or
(b) Whether or not such arrangement, understanding or action is
intended to be enforceable by legal proceedings.
(c) Whether or not, such arrangement, understanding or action is formal
or in writing; or whether or not such arrangement, understanding or
action is intended to be enforceable by legal proceedings.
(d) None of the above.
II. Descriptive Questions
1. (i) Examine the following given aspects with reference to the allottee in
the situation given in the case study :
(a) Rights exercised by Dr. Sharma as an allottee.
(b) Duties fulfilled by Dr. Sharma as an allottee.
(c) Right which was not exercised by him and duty which was not
required to be fulfilled by Dr. Sharma.
(ii) The promoters of Omega Capetown Residency transferred majority
of rights and liabilities to Sai DeveIopers Pvt Ltd. for the completion
of the project. Advise as to the validity of such transfer of a real estate
project to a Sai Developer’s Pvt Ltd in the case study?
2. In the given case study Omega Capetown Residency has got itself registered
under the Real Estate Regulatory Authority, as it consisted of 1,000
residential units. However, if Omega Capetown Residency consisted of only
250 residential units, then was it necessary to get itself registered under the
Real Estate (Regulation and Development) Act, 2016: if yes, name the
various important documents and declarations which are required to be
submitted by a ‘real estate developer’ while registering a project with the
Real Estate Regulatory Authority (RERA) having only 250 residential units
and not 1,000 residential units.
3. Mr. Bhanu Pratap Taneja was approached by some of the influential
developers to join their association so as to reach an understanding whereby
they could inflate the price of the apartments built by them. Even though the
deal was in favour of Mr. Bhanu Pratap Taneja, he rejected the proposal from
other developers. In the light of the provisions of the Competition Act, 2002,
discuss whether the decision of Mr. Bhanu Pratap Taneja is lawful?
© The Institute of Chartered Accountants of India

Page 247
ELECTIVE PAPER 6D – ECONOMIC LAWS
SUGGESTED SOLUTION – CASE STUDY 1

I. ANSWERS TO OBJECTIVE TYPE QUESTIONS

1. (a) [Hints: Refer Section 3 of the Real Estate (Regulation and Development) Act, 2016]

2. (b) [Hint: Refer Section 4 (2) (c) of the Real Estate (Regulation and Development) Act,
2016]

3. (d) [Hint: Refer Section 10 of the Real Estate (Regulation and Development) Act, 2016]

4. (d) [Hint: Section 14 (3) of the Real Estate (Regulation and Development) Act, 2016]

5. (a); [Hint: Refer Section 3 of the Real Estate (Regulation and Development) Act, 2016.
It may not be mandatory in a particular State if the State has granted exemption to
such on-going project]

6. (a); [Hint: Section 15 of the Real Estate (Regulation and Development) Act, 2016. It is to
be noted that if a consumer or his family holds more than one unit in the project then
he will be considered as one consumer only]

7. (b); [Hint: Refer Section 19 (10) of the Real Estate (Regulation and Development) Act,
2016]

8. (b); [Hint: Refer Section 2 (q) of the Real Estate (Regulation and Development) Act,
2016]

9. (b); [Hint: Refer Section 2 (9) (iii) of the Prohibition of Benami Property Transactions Act,
1988]

10. (c). [Hint: According to Section 2 (b) of the Competition Act, 2002, ‘Agreement’ includes
any arrangement or understanding or action in concert:

(i) Whether or not, such arrangement, understanding or action is formal or in


writing; or

(ii) Whether or not such arrangement, understanding or action is intended to be


enforceable by legal proceedings.]

© The Institute of Chartered Accountants of India


Page
1 248
II. ANSWERS TO DESCRIPTIVE QUESTIONS

1. (i) (a) With reference to Section 19 of the Real Estate (Regulation and
Development) Act, 2016, Dr. Sharma, as an allottee, exercised the following
rights:

(I) Obtained the information relating to sanctioned plans, layout plans


along with the specifications as approved by the competent
authority.

(II) Demanded to know stage-wise time schedule of completion of the


project, including the provisions for water, sanitation, electricity and
other amenities.

(III) Claimed physical possession of the said apartment.

(IV) Obtained the necessary documents and plans, including that of


common areas, after getting the physical possession of the
apartment from the promoter.

(b) With reference to Section 19 of the Real Estate (Regulation and


Development) Act, 2016, Dr. Sharma, as an allottee, fulfilled the following
duties:

(i) Made necessary payments within the time as specified in the


agreement for sale.

(ii) Became a member of the RWA formed by the allottees.

(iii) Took physical possession of the apartment within a period of two


months from the issue of Occupancy Certificate.

(iv) Participated towards registration of the conveyance deed of the


apartment.

(c) (i) With reference to Section 19 of the Real Estate (Regulation and
Development) Act, 2016, Dr. Sharma, as an allottee, did not exercise
the following right:

The right to claim the refund of amount paid along with prescribed
rate of interest. It was so because the promoter was able to give

© The Institute of Chartered Accountants of India


Page
2 249
possession of the apartment in accordance with the terms of
agreement for sale.

(ii) With reference to Section 19 of the Real Estate (Regulation and


Development) Act, 2016, Dr. Sharma, as an allottee, was not
required to fulfill the following duty:

The duty to pay interest at prescribed rate for delay in making any
payment. It was so because he had made the payments in
accordance with the terms of agreement for sale.

(ii) As per section 15 of the Real Estate (Regulation and Development) Act, 2016, a
promoter is permitted to transfer his majority rights and liabilities in respect of a real
estate project to a third party.

The provisions given below are to be adhered to by the promoter for transfer:

(a) Obtain prior written consent from two-third of allottees. Such consent will not
include the consent given by the promoter.

(b) Also obtain prior written approval of the Authority.

Note: It is to be ensured that such transfer shall not affect the allotment or sale of
the apartments, plots or buildings, as the case may be, in the real estate project
developed by the promoter.

(i) After obtaining the required consent of both allottees and the Authority, the
new promoter shall be required to independently comply with all the pending
obligations under the provisions of the Act or the rules and regulations made
thereunder.

(ii) The new promoter is also required to comply with the pending obligations as
per the agreement for sale entered into by the erstwhile promoter with the
allottees.

(iii) Further, the new promoter must note that any transfer so permitted shall not
result in extension of time to him to complete the real estate project.

Note: In case of default, he shall be liable to the consequences for delay, as per the
provisions of the Act or the rules and regulations made thereunder.

© The Institute of Chartered Accountants of India


Page
3 250
Since in the given case study , 95% of the allottees had already given their written
permission. Further, the Authority had given its written approval to the proposal for
transfer and completion of Project by M/s. Sai Developers Pvt. Limited in compliance
with the requirements given in the said provisions. Such transfer of a real estate
project to a Sai Developer’s Pvt Ltd. is valid.

2. According to proviso to section 3 of the Real Estate (Regulation and Development) Act, 2016,
projects that are on going on the date of commencement of the Act, and for which the
completion certificate has not been issued, the promoter of the project are required to make
and application to the concerned Authority for the registration of the said project within a
period of 3 months from the date of commencement of the Act.

Further, the section provides that no registration of real estate project shall be required where
the area of land proposed to be developed does not exceed 500 square meters or the number
of the apartments proposed to be developed does not exceed 8 inclusive of all phases.

Hence, the Act requires registration of on-going projects where completion certificate was
yet to be obtained as well as new projects, if the area to be developed exceeded 500 sq.
mtrs. or apartments to be built under the project exceeded eight. Thus, registration of Omega
Capetown Residency was must with the Real Estate Regulatory Authority of UP (RERA, UP),
as consisted of 1,000 residential units.

Further, even if Omega Capetown Residency consisted of only 250 residential units (i.e.
more than 8 units), it will be compulsory to get itself registered under the Act. The process
of registering a project with the Real Estate Regulatory Authority (RERA) which consists of
1,000 units or 250 units is same which is given under section 4 of the Act.

With reference to Section 4, various important documents and declaration required to be


submitted while registering a project with RERA are as under:

• Details of the project such as name, address, type, names and photographs of the
promoters, etc.

• Details of the project which were already launched by the real estate developer in
the preceding 5 years and their present status.

• Approvals and commencement certificates obtained from the competent authority for
each phase of the project separately.

© The Institute of Chartered Accountants of India


Page
4 251
• Sanctioned layout plan, the development plan for the project and details of basic
facilities being made available like drinking water, electricity etc.

• Proforma of allotment letter, agreement for sale and conveyance deed to be signed
with the consumers.

• Location of the project with clear demarcation of the land for the project.

• Number, type and carpet areas of units to be sold.

• The details of open areas if any like terraces, balconies etc.

• Details of associated engineers, contractors, architects and intermediaries in the


project.

• a declaration, duly supported by an affidavit, stating the following important matters:

o that the promoter has a legal title to the land and it is free from all encumbrances
along with legally valid documents;

o the time period required for completion of the project;

o that seventy per cent. of the amount realised from the allottees, from time to
time, shall be deposited in a separate escrow account and shall be used only
for the purpose of completion of project;

o that the promoter shall get his accounts audited within six months after the end
of every financial year by a chartered accountant in practice; and shall take all
the pending approvals on time from the competent authorities; etc.

3. According to section 2(b) of the Competition Act, 2002, ‘Agreement’ includes any
arrangement or understanding or action in concert:

(i) Whether or not, such arrangement, understanding or action is formal or in writing, or

(ii) Whether or not such arrangement, understanding or action is intended to be


enforceable by legal proceedings.

Further, section 2(c) of the Competition Act, 2002, "Cartel" includes an association of
producers, sellers, distributors, traders or service providers who, by agreement amongst
themselves, limit, control or attempt to control the production, distribution, sale or price of,
or, trade in goods or provision of services.

© The Institute of Chartered Accountants of India


Page
5 252
An association for the welfare of the trade or formed for any other purpose not mentioned in
the aforesaid definition will not be a cartel Thus, it is only when an association, by agreement
amongst themselves, limits control or attempts to control the production, distribution, sale or
price of, or, trade in goods or provision of services, that it will be a cartel.

Hence, an agreement which prohibits an enterprise or person or their association for entering
into an agreement in respect of production, supply, distribution, storage, acquisition or
control of goods or services, which causes or is likely to cause an appreciable adverse affect
on competition. Such agreements entered in contravention of the above are void. These
agreements are presumed to have an appreciable adverse affect on competition.

Here, in the given situation, the agreement between Mr. Bhanu Pratap Taneja and other
builders would have fallen into the ambit of section 2(b) and 2(c) of the Competition Act,
2002 as the aim of the association was to increase the price of the apartments. Thus, such
an association would be void.

© The Institute of Chartered Accountants of India


Page
6 253
ELECTIVE PAPER 6D: ECONOMIC LAWS

CASE STUDY - 2

Mr. Manohar Mehta, renowned builder of Mumbai, owns a reputed building construction company known
as “Sri Ram Building Construction and Real Estate”. Due to his vast business empire, he is known as the
“King of the Property World”.

On the personal front, Mr. Mehta in his family has an elderly mother, wife, and three children. His father,
Mr. Sri Ram Mehta, had recently expired after prolonged sickness. Mr. Manohar Mehta’s mother, Mrs.
Rama Devi, is a religious lady always dedicating her time in worship and holy works. Wife, Urvashi, is a
home maker and a socialite. She is mostly involved in all the social activities and runs her own NGO
named “AAWAZ”. Mr. Mehta regularly give financial contribution to his wife’s NGO. These minor donations
gave Mr. Mehta a Noble man tag in the social circle and better business prospects.

Sonia, eldest daughter of Mr. Mehta, is married and well settled with her husband in Australia. She has
recently started her own import export business in Australia with the help of her father. Mr. Mehta would
transfer the amount to his daughter and she regularize the amount in the books of accounts of her
business.

Recently, a project was started by Mr. Mehta in Marol area of Mumbai. The project was named as “Shubh
Appartment”. Under this project a 5 storey building comprising of 2 flats (2 BHK) at each floor were
constructed. The actual construction cost of each flat was Rs. 50 lakh. The flat was sold at Rs. 60 Lakh.
The advance booking charges of Rs. 6 lakh for each flat was collected from the buyers by cheque. Proper
receipt was issued to all the buyers for the advance payment. Out of the 10 flats, 4 flats were sold at an
increased price of Rs. 62 Lakh. Rs. 2 lakh each was taken in cash from the 4 buyers. Therefore, he earned
in total Rs. 8 Lakh for these 4 flats. This amount of Rs. 8 lakh was send to Sonia via an independent agent.
Sonia utilized this amount in her business and taken into record via some entry in her books of accounts.

Mr. Manohar Mehta has two sons, Rohit and Sorav. Sorav is the youngest son. He is pursuing his
graduation from one of the best universities of Chicago. For his education, Mr. Mehta remitted foreign
exchange of USD 2, 00,000 through authorized person. During course of his studies, Sorav was caught
with the seasonal influenza, so there he required an emergency medical treatment. Mr. Mehta transmitted
additional amount of USD 70,000 for treatment through authorized person who was well known to him for
hassle free transfer.

Rohit, the elder son, after successful completion of his M.B.A. Finance degree, is now actively supporting
his father in his real estate business. To give a start to his career, Mr. Mehta handed over the project

1
© The Institute of Chartered Accountants of India
Page 254
“Royal Aashiana” to be constructed in Kharghar. The said project was proposed to be developed in 1000
sq. mts. Rohit was working on the project under guidance of Mr. Mehta. He marketed about the said
project and invited persons to purchase the flats in the Royal Ashiana. It was an ongoing project, Rohit
without registration of the project made an agreement to sell some of the flats.

As per Mr. Mehta’s regular morning routine, he one day read his favorite column “Property for Sale” in the
newspaper. He came across one advertisement regarding the sale of the residential plot in Panvel district
of Maharashtra. He discussed about the advertisement with his manager, Mr. Shyam Pareekh. He asked
his manager to visit the actual site of the mentioned property.

Mr. Pareekh called the land owner, Mr. R. Thakker, and took the appointment for the visit. He went to
Panvel to meet the owner and see the property. It was a 10,000 square feet plot near the city area. Mr.
Thakker quoted a price of Rs. 1crore for selling his property. After two rounds of meeting the final
negotiation with the land owner was done and deal was locked for Rs. 90 Lakh. On mutual consensus
between them, down payment of Rs. 20 lakh was made to Mr. Thakker in cash. Further, a payment of Rs.
70 Lakh was done by cheque and the property was registered in the name of his (Mr. Mehta) mother.
Being a sacred woman, she was not interested in all such types of transactions or arrangements made on
her name by Mr. Mehta.

After few months, Mr. Manohar Mehta from his sources came to know that an agricultural land is on sale by
a farmer, Mr. Bhima Singh. The farmer’s 5 acres of agricultural land was located in Thane district of
Mumbai. Mr. Manohar Mehta thought it would be a great deal to buy the agricultural land around the lush
green vicinity of the Thane district. He further thought that he can resale this property after converting it to
farm houses to the potential buyers.

After the detail discussion with his management regarding the purchase of land, Mr. Manohar Mehta went
to Thane to see the agricultural land. The land was just 500 meters away from the highway. After visiting
the land Mr. Mehta became keen to buy the property. They had a talk with the farmer, Mr. Bhima Singh.
The farmer being illiterate hardly knew about the legal sale/purchase of the land. Mr. Manohar Mehta and
Mr. Pareekh negotiated and finalized the deal in Rs. 80 Lakh.

Since Mr. Manohar Mehta required funds for purchasing the Thane property, he decided to sell his Panvel
plot which was in the name of his mother. He retransferred the Panvel property (Which was in the name of
his mother) to himself and then sold the Panvel plot for Rs. 1.10 Crore. He took partial amount by cheque
and rest by cash. This way he safeguarded himself from showing the capital gain on financial record. Mr.
Mehta received Rs. 80 Lakh in cheque and rest Rs. 30 Lakh in cash.

2
© The Institute of Chartered Accountants of India
Page 255
Whereas, Mr. Mehta induced Bhima Singh, and paid him Rs. 50 Lakh through cheque and Rs. 30 lakh
through cash.

Mr. Manohar Mehta was still having Rs. 30 Lakh out of sale of 1.10 Crore panvel property, at his disposal.
Mr. Mehta decided to deposit Rs. 2,000 each to his wife, two sons and mother, saving accounts every
month. He would continue deposition of Rs. 2000 each per month for next couple of years.

During one of the corporate parties while having a discussion, Mr. Mehta’s friend advised him to invest the
remaining amount in the shell company outside India. Mr. Mehta liked the suggestion and decided to send
Rs. 10 Lakh to invest in the shell company in Singapore via Hawala. He learned about Mr. Varun Das who
runs a business of hawala under the veil of running a financial company. Mr. Mehta contacted Varun Das
who agreed to transfer the fund via Hawala on 1% commission basis. In this way Mr. Mehta managed to
circulate the amount in the shell company outside India.

Mr. Manohar Mehta also donated Rs. 50,000 in cash to his wife’s NGO ÄAWAZ”.

After few months, Mr. Mehta decided to buy a new car, worth Rs. 50 Lakh. He did the down payment of Rs.
5 Lakh via cheque. For the remaining Rs. 45 lakh he took 3 years auto loan, so that he can deposit the
monthly installment in the bank. Hence in this way the remaining Rs. 10 Lakh, which he gained from the
sale of the Panvel property, was utilized..

Due to frequent transactions of hefty amount and his conduct of other financial activities in a year, Mr.
Mehta bank accounts and his family members account of transactions were in the scrutiny of the Income
Tax Department.

On further investigation it was discovered that Mr. Mehta, Mr. Thakkar, Rohit and Sonia being guilty for
different offences punishable under the different Acts.

I. Multiple Choice Questions

1. Sonia plans to make investment in India. She was permitted to do so as per the FEM
regulation in -

a. Trading in transferable development rights

b. Real Estate business and construction of farm houses

c. Agricultural or plantation activities

d. Chit funds subscribed through banking channel and on non-patriation basis

3
© The Institute of Chartered Accountants of India
Page 256
2. Sonia ordered exports of goods from India for her business. The amount (export value) of
good shall be released and repatriated to India within period -

a. 3 Months from date of export

b. 6 Months from date of export

c. 9 Months from date of export

d. 9 Months from date of invoice covering such export

3. Amount released for the real estate project from allottees in separate account can be
withdrawn by promoter after it is certified by-

a. Cost accountant and an Architect

b. Engineer, and a Chartered Accountant

c. an Architect and an Engineer

d. Engineer, an Architect, and a Chartered Accountant in practice

4. "Who according to the Provision of Prevention of Money Laundering Act is/ are held to be
liable in dealing of Panvel property-

a. Mr. Manohar Mehta


b. Mr. Thakkar
c. Both (a) and (b)
d. Mr. Shyam Pareekh
5. Who among the following is liable for an offence of money laundering as per the Part C of
the Schedule given in the Prevention of money laundering Act-

a. Mr. Mehta
b. Mr. Thakkar
c. Mr. Atul
d. Ms. Sonia
6. How much amount of penalty Mr. Mehta has to pay on illegal remittance of money
transferred to Sorav

a. USD 6,00,000
b. USD 1,80,000
c. USD 60,000

4
© The Institute of Chartered Accountants of India
Page 257
d. USD 50,000
7. Suppose any project started by Mr. Mehta was not completed within preferred time due to
force majeure. Remedy available to Mr. Mehta in this situation

a. Registration may be interim cancelled by Authority


b. Registration need to be freshly applied
c. Registration granted may be extended for period not exceeding 1 year on
application
d. Registration may be extended for reasonable period on application.
8. Mr. Mehta sells a flat of Royal Aashiana to Mr. X for the amount Rs. 75 lakh. Mr. X made
the advance payment. The correct value of the advance payment is

a. Rs. 7.5 Lakhs


b. Rs. 8 Lakhs
c. Rs. 8.5 Lakhs
d. Rs. 9 Lakhs
9. In case of dispute between Mrs. Rama Devi and Mr. Mehta, can Mr. Mehta legally claim
her right over the Panvel Plot?

a. Yes, because he is the beneficial owner in the transaction


b. No, because the transaction is Null and void
c. Yes, because he paid consideration for the transaction
d. None of the above
10. Mr. Mehta files an application for initiation of voluntary liquidation proceeding of his Real
Estate construction company. Mr. X, a home buyer of a flat in one of the project of Mr.
Mehta claimed for the re-fund of paid amount or demanded for handover of possession of
flat. Which amongst following is not incorrect statement

a. X cannot claim the amount due to pending of Insolvency process


b. X can file a suit for the default committed by Mr. Mehta under the Insolvency &
Bankruptcy code
c. X’s right & interest is protected after execution of an agreement to sale till the
conveyance of the flat
d. None of the above

5
© The Institute of Chartered Accountants of India
Page 258
II. Descriptive Question

1. What would be the consequences in the following given situations:

(a) Where if Mr. Mehta remitted Foreign Exchange USD 2,00,000 and USD 70,000 as
educational and medical expenses to Sorav .

(b) Sorav used USD 20,000 out of the remitted medical expenses (i.e., USD 70,000)
and used remaining amount to purchase immovable property jointly with Mr.
Mehta in Chicago.

2. Suppose Royal ashiana, is a 15 year old building of Mr . X. It was purchased by Mr .


Mehta in January 2016. He planned to re-develop the said building into residential
apartments. He launched the project in end of January 2016. During the course, the
Government enacted the Real estate(Regulation and Development) Act, 2016.Mr. Mehta
seek advise of his legal consultant on the issues related to the registration of the said
Project-

(1) Is the Registration for the re-development of society is mandatory?

(2) If he plans to develop a new society under new name with new allotments . Is the
registration mandatory of the project?

(3) State the position where project have been completed and obtained the certificate
of completion before the commencement of RERA?

(4) Where if the project is to be devolved into phases ?

(5) Where if the authority has not taken any decision on application for the registration
within the prescribed period.

3. (i) What remedy is available to Mr. Mehta, in case he want to compound for the
commission of illegal remittance to Sorav under FEMA Act 1999?

(ii) According to the case study the property bought by Mr. Mehta in the name of his
mother, Mrs. Rama Devi, is a Benami Transaction and will be confiscated.
Suppose the property rightfully belonged to Mrs. Rama Devi then what measures
she can take to save the property from confiscation. Explain?

6
© The Institute of Chartered Accountants of India
Page 259
ELECTIVE PAPER 6D – ECONOMIC LAWS
SUGGESTED SOLUTION – CASE STUDY 2

I. ANSWERS TO OBJECTIVE TYPE QUESTIONS

1. (d) [Hint: As per regulation 4 (b) explanation (ii) of the FEMA (permissible capital
account transaction) Regulation 2000.]

2. (c) [As per regulation of FEMA (Export of goods and services) regulation 2016]

3. (d) [Hint: Section 4 (2), proviso to (D) of clause (l) of the RERA, 2016]

4. (c) [Hint: Refer section 3]

5. (d) [Hint: Refer Part C of the Schedule to the Prevention of Money Laundering Act, 2002]

6. (c) [Hint: Refer Section 13 of Foreign Exchange Management Act 1999 along with
Schedule III of the FEM (Permissible Current Account Transactions) Regulations,
2000]

7. (c) [Hint: Refer Section 6 of RERA, 2016]

8. (a) [Hint: As per Section 13 of RERA, 2016]

9. (b) [Hint: Section 6 Prohibition of Benaim Property Transactions Act, 1988 ]

10. (c) [Hint: Section 11 (4) read with section 89 of the RERA, 2016]

II. ANSWERS TO DESCRIPTIVE QUESTIONS

1 (a) As per section 13 (1) of the FEMA, 1999, If any person contravenes any provision of
this Act, or contravenes any rule, regulation, notification, direction or order issued in
exercise of the powers under this Act, or contravenes any condition subject to which
an authorisation is issued by the Reserve Bank, he shall, upon adjudication, be liable
to a penalty up to thrice the sum involved in such contravention where such amount
is quantifiable, or up to two lakh rupees where the amount is not quantifiable.

Any Adjudicating Authority adjudging any contravention to above provisions, may, if


he thinks fit in addition to any penalty which he may impose for such contravention
direct that any currency, security or any other money or property in respect of which
the contravention has taken place shall be confiscated to the Central Government

© The Institute of Chartered Accountants of India


Page
1 260
and further direct that the foreign exchange holdings, if any of the persons
committing the contraventions or any part thereof, shall be brought back into India
or shall be retained outside India in accordance with the directions made in this
behalf.

According to the above provisions, Mr. Mehta will be penalized thrice of the extra
amount (USD, 20,000) remitted above the prescribed limit (USD 2, 50,000). Hence
liable to pay a penalty of USD 60,000 to the Government.

(b) The second issue is related to sections 13(1A), 13(1C) & 37A of the FEMA Act, 1999
read with Regulation 5 of the FEM(Acquisition & transfer of immovable property
outside India)Regulation , 2015.

As per section 13(1A), if any person is found to have acquired any foreign exchange,
foreign security or immovable property, situated outside India, of the aggregate value
exceeding the threshold prescribed under the proviso to sub-section (1) of section
37A, he shall be liable to a penalty up to three times the sum involved in such
contravention and confiscation of the value equivalent, situated in India, of the
foreign exchange, foreign security or immovable property.

13(1C) of FEMA says that if any person is found to have acquired any foreign
exchange, foreign security or immovable property, situated outside India, of the
aggregate value exceeding the threshold prescribed under the proviso to sub-section
(1) of section 37A, he shall be, in addition to the penalty imposed under sub-section
(1A), punishable with imprisonment for a term which may extend to five years and
with fine.

According to Section 37A of the FEMA, upon receipt of any information or otherwise,
if the Authorised Officer prescribed by the Central Government has reason to believe
that any foreign exchange, foreign security, or any immovable property, situated
outside India, is suspected to have been held in contravention of section 4, he may
after recording the reasons in writing, by an order, seize value equivalent, situated
within India, of such foreign exchange, foreign security or immovable property:

Provided that no such seizure shall be made in case where the aggregate value of
such foreign exchange, foreign security or any immovable property, situated outside
India, is less than the value as may be prescribed.

© The Institute of Chartered Accountants of India


Page
2 261
As per regulation 5 of the FEM (Acquisition & transfer of immovable property outside
India) Regulation, 2015, a person resident in India may acquire immovable property
outside India jointly with a relative who is a person outside India. Provided there is
no outflow of funds from India.

Since in the given case, Mr. Mehta remitted Foreign exchange to Sorav in excess to
the limit prescribed under the FEMA. Sorav partially used USD 20,000 for medical
treatment and rest USD 50,000 to purchase property outside India jointly with Mr.
Mehta. So Both Mr. Mehta and his son Sorav will be liable under sections 13(1),
13(1A), 13(1C) of the FEMA, 1999.

2. (1) According to section 3(2) of the Real Estate (Regulating ) Authority Act, 2016, no
registration of the real estate project shall be required for the purpose of renovation
or repair or re-development which does not involve marketing, advertising selling or
new allotment of any apartment, plot or building, as the case may be, under the real
estate project. So Registration for the re-development of society (Royal Ashiana)
was not required.

(2) According to the above provision no registration is required when any project is
renovated or repair or re-development and it does not involve marketing, advertising
selling or new allotment of any apartment, plot or building, as the case may be, under
the real estate project. However, in the given situation in the question, Mr. Mehta
plans to develop it as a new society under new name with new allotments. So
registration of the said project was necessaited as the Act.

(3) As per the proviso to section 3(1) of the RERA, projects that are ongoing on the date
of commencement of this Act and for which the completion certificate has not been
issued, the promoter shall make an application to the Authority for registration of the
said project within a period of three months from the date of commencement of this
Act. In the given case, where the project have been completed and obtained the
certificate of completion before the commencement of RERA, such project shall not
require registration.

(4) As per the explanation to section 3 of the RERA, where the real estate project is to
be developed in phases, every such phase shall be considered a stand alone real
estate project, and the promoter shall obtain registration under this Act for each

© The Institute of Chartered Accountants of India


Page
3 262
phase separately. Therefore if the said project is to be developed in phases , it needs
separate registration for each phase.

(5) As per Section 5 of the RERA, the Authority has to decide on the application within
30 days of its receipt. It further provides that in case the Authority fails to take a
decision within the said period of 30 days the project shall be deemed to be
registered.

3. (i) According to provision of Section 15 (1)

(1) Any contravention under section 13 may, on an application made by the


person committing such contravention, be compounded within one hundred
and eighty days from the date of receipt of application by the Director of
Enforcement or such other officers of the Directorate of Enforcement and
officers of the Reserve Bank as may be authorised in this behalf by the
Central Government in such manner as may be prescribed."

(2) Where a contravention has been compounded under sub-section (l), no


proceeding or further proceeding, as the case may be, shall be initiated or
continued, as the case may be, against the person committing such
contravention under that section, in respect of the contravention so
compounded.

As per the above mention provision Mr. Mehta will submit the application to the
concerned authority for compounding of the offences committed in contravention to
the FEMA Act.

(ii) The Section 5 of PMLA authorizes the Director or any other officer not below the
rank of Deputy Director to attach the property. Section 8 of PMLA lays down an
elaborate procedure for adjudication of complaint under Section 5 of PMLA. It calls
for a show cause notice to be issued to the offender/ person from whom property
has been seized, so as to give the person an opportunity to make a case against
attachment. Such a person in order to avoid confiscation, can demonstrate the
legitimate source of his income/earning or assets, out of which or by means of which
he has acquired the property attached. The evidence on which he realized and other
relevant information and particulars, and to basically convince the authority about
the property which should not be declared to be the property involved in money
laundering.

© The Institute of Chartered Accountants of India


Page
4 263
If the authority reached the conclusion that the offense has not taken place it shall
order release of such property to the person entitled to receive it.

Hence as per the above provision, Mrs. Rama Devi aggrieved by the provisional
attachment may file her objection before the adjudicating authority.

© The Institute of Chartered Accountants of India


Page
5 264
FINAL COURSE
ELECTIVE PAPER 6D: ECONOMIC LAWS

CASE STUDY - 3

Mr. Krishnakant Mathur was a lecturer in agricultural college of Pushkar. After 25 years of his service he retired
from his job last month. To utilise his time, Mr. Mathur keeps the accounts of his society and has also started a
coaching classes for the needy and poor students. Mr. Mathur has three children, two daughters Jaya & Lata and
son Neeraj. His wife Asha Devi is a house wife. Neeraj is eldest of all the three children. Jaya and Lata are still
studying in Class 12 th and 10 th respectively. At the time of retirement of Mr. Mathur, Neeraj has completed his
engineering degree.
During his work tenure, Mr. Mathur purchased one property (area 2000 Sq feet), at Mansarover colony, Jaipur in
the name of his son Neeraj Mathur. The property already has a constructed house over it. Mr. Mathur also owns
an ancestral property in his village Titari. Since he was posted at Jaipur, so he resided at Mansarover colony house
with his family.
Neeraj got a good job in Maharashtra State Electricity Board (MSEB) as a Junior Engineer. He was posted at Pune.
He was very desperate to earn more money and become rich. For clearing the contractors billing and giving
necessary approvals he started giving favours to the contractors and vendors. Various projects were in the hands
of MSEB, they authorised Neeraj to finalise the tenders related to supply of required articles, goods and services
for attainment of the government projects. Many contractors pleased him by cash or kind for acceptance of their
tenders for the projects. Neeraj used his position in the said department by manipulating the bidding process.
Soon, Neeraj got promotion. He decided to celebrate his grand success and planned for a trip to Hongkong with
his friend. There in Hongkong, he came to know about XBL Company, which was a joint ventures abroad of Indian
Companies. He thought to make an equity investment in XBL Company. He talked with the Indian Service Providers
for the investment of funds to Securities Exchange of Hongkong for buying equity in XBL Company. Neeraj applied
for drawl of foreign exchange, but due to legal compliances, authorised dealer denied for the said transaction.
Neeraj, through known authorise dealer, on payment of commission exported the foreign exchange for the equity
investment in XBL Company. So in this way, Neeraj made an overseas investments.
In the meantime, Neeraj, while residing in Pune, met a girl in his office named Shalini. They both fell in love with
each other. After couple of months they both decided to marry without informing any of the family members as they
both are of different caste. As a witness Rahul, Neeraj’s friend, was the only one who knew about his marriage.
Rahul is located in Dubai and works as a Senior Manager with Al-Aadil Works LLC.
Rahul insisted him to come to Dubai. So Neeraj and Shalini planned their visit to Dubai. Shalini discussed about
her trip with her friend. Her friend told that Dubai is a beautiful place and is a hub for Gold and electronics. Since
Shalini had a keen interest in buying gold and Neeraj had a restriction in carrying foreign currency, he came up
with an idea. He gave Rahul’s family Rs. 1 Lakh in cash, in India and took the equivalent foreign currency from
Rahul in Dubai. They both had a good time in Dubai. They visited the Gold Souq in Deira. Shalini purchased gold
jewellery worth Rs. 1.5 Lakh. Rahul also took them around to see some latest Electronic items. Neeraj found that
the price of the Smart Television was less and the shopkeeper proposed further discount without a bill. So Neeraj
also bought a new Television worth Rs. 60000. After their return, they did not declare it to the Indian customs and
passed through the Green channel. Custom officials on the matter of doubt withheld them at the airport and
interrogated. After compliance with the required formalities under the legal prospects, the matter was sorted.
In 2016, during the period of demonetisation, Neeraj was holding a cash of Rs. 10 Lakh. He deposited Rs. 3 lakh
is his and his wife’s account within the permissible limit imposed by the government. To settle the rest of the amount
he took the help of his friend Jai Bakshi. Jai is a renowned hotelier based in Mumbai. As per the discussion Jai
told that he could manage Rs. 2 Lakh by depositing it in his account. Further Jai told that he will charge 10 percent

1
© The Institute of Chartered Accountants of India
Page 265
of Rs. 2 Lakh to accommodate the amount in his record books. Also, he will return this amount only after six months
due to scrutiny of income tax department. Neeraj found that the market rate of changing the demonetised 500 and
1000 rupees notes were 50% of the given amount. So he thought it is better to agree to the terms and conditions
of Jai and gave him Rs. 2 Lakh. With the remaining Rs. 5 Lakh, he booked a flat in Shubh Laxmi apartment near
Badlapur Mumbai in the name of his wife Shalini. Neeraj on record showed above that paid amount of Rs. 5 lakh
as a loan taken from Shalini’s uncle.
Whereas, Shubh Laxmi Apartment, constructed by J.K. Builders was registered with the Maharashtra Real Estate
Regulatory Authority. The flat was sold by Himmat Chand to Neeraj. In the meantime J.K. Builders filed an
application for bankruptcy and shed of their responsibilities as to the completion and handover of the possession
of the flats to the buyers. Neeraj approached to the office of J.K. Builders and they denied from their responsibilities
by saying that the said flat was sold by Himmat Chand, so he owns the responsibility. Whereas Himmat chand
took the plea that ultimate responsibility lies with the J.K. Builders, being a promotor. Neeraj went to the consumer
forum for the relief.
In the meantime, due to the tip off received from unknown sources and on grounds of noticing the suspicious
activities of Neeraj, Income Tax Department issued a scrutiny notice to him. According to the Notice, Neeraj was
asked to clarify the mode of payment for the loan amount from his uncle as there was no entry of cash credited to
Shalini’s account. Neeraj and Shalini, however, managed and came safely out of a situation on benefit of doubt.
After few months, Neeraj went to Jaipur with his wife Shalini to meet his family. Mr. Mathur was extremely angry
and shattered to see his son married without his consent. Mr. Mathur and his son had a heated argument and he
turned both of them out of his house. Neeraj told that this house is in his name and legally belongs to him. Hence
his father has no right to throw them away.
Mr. Mathur files a case for claiming ownership over the property as he does not want to give the possession of his
house to Neeraj. As per plaint allegation, Mr. Mathur has purchased the property in 18/2/2015 in the name of his
son Neeraj Mathur. As per plaint allegation, he has purchased the property out of his own income. He has claimed
relief that he be declared as owner of the property and Neeraj should be permanently restrained from interfering
in possession of the property. Plaint clearly reveals that Mr. Mathur has purchased the property out of his own
income in the name of his son Neeraj. It was his own property and he had claimed declaration of ownership right
over the property. He has also prayed for permanent injunction against his son. Entire plaint allegation does not
whisper that it was joint Hindu family property or purchased by Mr. Mathur for joint Hindu family.
On the other hand, Neeraj claimed that he has purchased the property from his own income, his father was not
having any right over the property.
During pendency of the suit, Mr. Mathur died. According to the procedure of the Tribunal in case of death of one
of several plaintiffs or of sole plaintiff, the right to sue survives. The Tribunal, on an application made in that behalf
shall cause the legal representative of the deceased plaintiff to be made a party and shall proceed with the suit.
So, his wife and two daughters were impleaded as legal representatives of the deceased. His legal representatives
have amended the plaint and have claimed declaration that the property is Joint Hindu family property. By detail
amendment, made in the plaint, his legal representatives pleaded that Mr. Mathur was “Karta” of Joint Hindu
Family.
The Tribunal held that in the present case, as per the claim of Mr. Mathur he purchased the property in the name
of defendant i.e. the defendant was Benamidar. So, Mr. Mathur was not entitled to claim any right over such
property in the light of Section 4 of the Prohibition of Benami Property Transactions Act, 1988.
Further, the Tribunal held that Late Mr. Mathur or legal representatives of the plaintiff have neither pleaded nor
proved that the defendant was holding the property as a trustee or in a fiduciary capacity for the benefit of other
persons for whom he was a trustee or was standing in a fiduciary capacity, inter alia, as per original pleading and
amended pleading, the property was owned by Mr. Mathur as his self-acquired property or property was owned by
Joint Hindu Family. Nothing has been pleaded by original plaintiff or his legal representatives that Neeraj was a

2
© The Institute of Chartered Accountants of India
Page 266
trustee or standing in a fiduciary capacity for the present legal representatives, original plaintiff and for others. Inter
alia, it has been specifically pleaded that the owner was the plaintiff and he has purchased the property in the
name of the present defendant.
Objective Type Questions:
1. In the given case study, Neeraj and Shalini bought gold jewellery worth Rs. 1.5 Lakh from Dubai. They have
custom clearance through green channel. State whether the given act will constitute an offence under the
Prevention of Money Laundering Act, 2002?
a. Yes, because the gold bought is beyond the permissible limit
b. Yes, because of an evasion of duty chargeable thereon goods on an & above the permissible limit.
c. No, because they are carrying the original bill of the purchased gold
d. No, because gold bought is within the permissible limit
2. Will Shalini’s uncle be liable for punishment under the Money Laundering Act, if he lends the loan amount
from his known sources of income?
a. Yes, because he is knowingly associating in the crime
b. No, because he is not a party to the crime
c. Yes, because he is actually involved in the process
d. Not sure
3. During the period of demonetisation Neeraj deposited Rs. 2 Lakh in Jai’s account. who is the beneficial person
in the light of the Prevention of Money-Laundering Act, 2002?
a. Neeraj
b. Jai
c. Both a and b
d. Neither a nor b
4. Suppose Mr. Mathur acquired a property from undisclosed & unaccounted sources of funds. Later, he created
a trust of his entire property for his family benefit and appointed Neeraj as his trustee. Will Neeraj be held
liable for such transaction made by Mr. Mathur-
a. Yes
b. No
c. Partially Liable
d. Not Sure
5. Neeraj in the course of his duty took commission in clearing the bill. This act can be termed as?
a. Money laundering through Prevention of Corruption Act, 1988
b. Money laundering through unlawful activities (Prevention) Act, 1967
c. Money laundering through customs
d. Money laundering through Indian Penal Code, 1860
6. Can Neeraj resale his house located in Jaipur to his father?
a. No, it is prohibited under Benami Transaction

3
© The Institute of Chartered Accountants of India
Page 267
b. Yes, Because Neeraj has the right to sale the property
c. Yes, because Neeraj is an owner
d. Both b and c
7. State which statement as to the drawl of the foreign exchange by Neeraj for transaction related to equity
investment in XBL Company is correct –
a. Neeraj can do such transaction through authorised dealer by providing commission on export for equity
investment in XBL Company.
b. Neeraj can do such transaction by directly buying equity in XBL Company
c. Neeraj cannot transact for equity investment in XBL Company
d. Drawl of foreign exchange by Neeraj for payment of commission on exports towards equity investment
in XBL Company, is prohibited.
8. Neeraj can claim relief for the completion and handover of the possession of the flat purchased in Shubh
Lakshmi Apartment against-
a. J.K. Builders
b. Himmat Chand
c. Both (a) & (b)
d. Maharashtra Real Estate Authority
9. When any transaction cannot said to be benami transaction-
a. If a person deposits old currency in account of another person in an understanding that account
holder will return the money in new currency
b. Individual held property in joint name with his grand children with the consideration paid by an
individual.
c. Company raising share capital through fictitious shareholders
d. Person takes a loan and not able to prove the genuinity of the lender
10. Suppose there are 3 bidders X, Y, & Z in a tender process initiated for MSEB. X & Y bidders were removed
by neeraj unethically showing them incompetent to make bid in the tender process for supply of goods to
MSEB for its project. This relates to-
a. Regulation of combinations
b. Abuse of dominant position
c. Controlling affairs or management
d. All of the above
Descriptive Questions:
1. a. Neeraj without consulting his Chief Engineer Authority, affixes his sign and seal to a document
certifying Mr. X (Fictitious Client) as a registered dealer of electronic goods. He there by obtains
his share for certifying the said document.
Mr. X misappropriates this forged document for his business to obtain various projects of
government for supply of electric goods.
Examine the given situation as to the nature of offence committed under the Prevention of Money
Laundering Act, 2002.

4
© The Institute of Chartered Accountants of India
Page 268
b. Neeraj While returning to India from Dubai trip gave wrong declaration about his gold and electronic
purchase at the airport. State the nature of liability of Neeraj for the commission of the above act?
2. a. Neeraj was given an offer by a company vendor to disclose him the lowest bid quoted by other
vendors. Neeraj accessed the computer of his Executive Director and passed on the lowest
quotation to the vendor and thus helped him in quoting the lowest among all the bids. Examine and
analyse the situation and conclude how Neeraj will be held liable under PMLA?
b. If suppose Neeraj with one of his friend registers a company and quotes bid to get a tender of
MSEB to get an extra income. Neeraj being at the back foot helps his friend unofficially to get
maximum tenders allotted to their company. They need to raise a share capital of the company. So
Neeraj decided to invest from his unknown sources of his income in the name of fictitious
shareholders.
Determine in the given scenario, the liability of Neeraj being covered under which Act?
3. (i) The Competition Commission of India (CCI) has received a complaint from a State Government
alleging that Shubh Limited and Mangal Limited have entered into an informal agreement, not
enforceable at law, to limit or control production, supply and market, to determine the sale price of
their products. Such an action of these companies has an appreciable effect on competition.
Examining the provisions of the Competition Act, 2002:
(A) Decide whether the above agreement has appreciable effect on competition.
(B) What factors shall the Competition Commission of India consider while taking the above
decision?
(C) What orders can the Competition Commission of India pass on completion of the inquiry?
(ii) Explain the restrictions, if any, under Foreign Exchange Management Act, 1999 in respect of the
following issue and transfer of shares:
Issue of Equity Shares of ` 1 crore at face value accounting for 45 percent of post-issue capital to
non-resident Indians in U.S.A. on non-repatriation basis. The shares are issued by M/s ABC
Knitwear Limited to finance the modernization of its plant.

5
© The Institute of Chartered Accountants of India
Page 269
ELECTIVE PAPER 6D- ECONOMIC LAWS

SOLUTION – CASE STUDY 3

I. ANSWERS TO OBJECTIVE TYPE QUESTIONS

1. (b)

[Hint: As per Schedule I of PMLA, 2002 it is a predicate offence related to custom Act]

2. (b)

[Hint: Section 3 of the Prevention of Money-Laundering Act, 2002 states of commission of an


offence of money-laundering.—Whosoever directly or indirectly attempts to indulge or knowingly
assists or knowingly is a party or is actually involved in any process or activity connected with
the proceeds of crime and projecting it as untainted property shall be guilty of offence of money-
laundering. Since Shalini’s uncle has given the loan from the known sources of his income so
he is not a part of this crime.]

3. (b)

[ Hint: 2(fa) of PMLA, 2002 ]

4. (b)

[Hint: As per exception of Section 2 (9) (A) Neeraj is not liable for Benami Transaction as he
stand in a fiduciary capacity for the benefit of other person.]

5. (a)

[Hint: Refer the schedule of PMLA, 2002]

6. (a)

[Hint: Refer Section 6 of the Prohibition of Benami Property Transactions Act, 1988]

7. (d)

[Hint: Refer Schedule I of FEM (Current Account Transaction ) Rules, 2000]

8. (c)

[Hint: Refer section 2(v) of the RERA]

9. (b)

[Hint: Refer section 2(9) of the Prohibition of Benami Property Transactions Act, 1988]

© The Institute of Chartered Accountants of India


Page 270
10. (b)

[Hint: Refer section 4 of the Competition Act, 2002]

II. ANSWERS TO DESCRIPTIVE QUESTIONS

1. a. According to provision of the PMLA, 2002 the money earned by Neeraj is not from the
legitimate sources. Since Neeraj forged the sign and seal of his Chief Engineer so the
money earned by him is proceed of crime. According to Schedule Part A of Para 1 of
PMLA, Neeraj has committed an offence under section 472 and 473 of Indian Penal
Code. These Section deals with the offence of making or possessing counterfeit seal,
etc., with intent to commit forgery. Whoever makes or counterfeits any seal, plate or
other instrument for making an impression, intending that the same shall be used for
the purpose of committing any forgery which would be punishable under section 467 of
this Code and under any other section under this Code, or, with such intent, has in his
possession any such seal, plate or other instrument, knowing the same to be counterfeit,
shall be punished with imprisonment for life, or with imprisonment of either description
for a term which may extend to seven years, and shall also be liable to fine.

b. Neeraj is liable under the provision of PMLA Act. His act is covered under the provision of
Part B of the Schedule. Part B of the Schedule refers to offence under the Customs Act,
1962. Section 132 of the Customs Act states that whosoever makes sign, or use or cause
to be made, sign or use any declaration, statement or document in relation to customs
knowingly or having reasons to believe that such declaration statement etc. is false shall be
punishable for a term which may extend to two years or fine or both.

2. a. Neeraj has contravened the Prevention of Money Laundering Act under Part A Para 22
of the Information Technology Act 2000. According to the provision of Section 72 of
Information of Technology Act 2000, if any person who, in pursuance of any of the
powers conferred under this Act, rules or regulations made thereunder, has secured
access to any electronic record, book, register, correspondence, information, document
or other material without the consent of the person concerned discloses such electronic
record, book, register, correspondence, information, document or other material to any
other person shall be punished with imprisonment for a term which may extend to two
years, or with fine which may extend to one lakh rupees, or with both.

Neeraj in the given case, without the consent of his Executive Director accessed the
electronic records and passed on the official information to the vendor without permission.
This information can produce large profits and legitimize the ill-gotten gains through money

© The Institute of Chartered Accountants of India


Page 271
laundering. Hence it is punishable under the Section 72 of the Information of Technology
Act, 2000.

b. The given issues falls within the ambit of the Prohibition of Benami Property Transactions
Act, 1988. According to Section 2 (26) "Property" means assets of any kind, whether
movable or immovable, tangible or intangible, corporeal or incorporeal and includes any
right or interest or legal documents or instruments evidencing title to or interest in the
property and where the property is capable of conversion into some other form, then the
property in the converted form and also includes the proceeds from the property.

According to Section 2 (10) "benamidar" means a person or a fictitious person, as the case
may be, in whose name the benami property is transferred or held and includes a person
who lends his name.

According to Section 2 (9) (B), a transaction or an arrangement in respect of a property


carried out or made in a fictitious name; or (D) a transaction or an arrangement in respect
of a property where the person providing the consideration is not traceable or is fictitious.
Hence, according to all the above provisions, Neeraj has done Benami transactions. He has
done the investment in the share as a Benamidar as he holds the share in the fictitious
name of shareholders. Hence this is a Benami transaction and is liable for punishment
under the Benami Transaction Act.

3. (i) (A) The term ‘agreement’ as defined in section 2 (b) of the Competition Act, 2002,
includes any arrangement or understanding or action in concert.

(i) whether or not such arrangement, understanding or action is formal or in


writing, or

(ii) whether or not such arrangement, understanding or action is intended to


be enforceable by legal proceedings.

Thus an agreement between Shubh Ltd. and Mangal Ltd. satisfies the above
ingredients of an agreement as per section 2 (e) of the Act, so agreement has
appreciable effect on competition.

(B) Factors to be considered:

(1) creation of barriers to new entrants in the market.

(2) driving existing competitors out of the market.

(3) foreclosure of competition by hindering entry into the market.

(4) accrual of benefits to consumers.

© The Institute of Chartered Accountants of India


Page 272
(5) improvements in production or distribution of goods or provision of
services.

(C) Orders of CCI: If after enquiry by the Director General, the Commission finds the
agreement entered into by Shubh Ltd. and Mangal Ltd. are in contravention of
section 3, it may pass all or any of the following orders:

(1) direct Shubh Ltd. and Mangal Ltd. to discontinue and not to re-enter such
agreement.

(2) impose such penalty as it may deem fit which shall not be more than 10% of
the average of the turnover for the last 3 preceding financial years, upon each
of such person or enterprises which are parties to such agreement or abuse;

(3) direct that agreement shall stand modified to the extent and in the manner
as may be specified in the order by the commission

(4) direct Shubh Ltd. and Mangal Ltd to abide by such other orders as the
commission may pass and comply with the directions including payment
of cost, if any.

(5) pass such other orders or issue such directions as it may deem fit.

(ii) Issue of equity shares to NRI’s and transfer of shares by NRIs are capital account
transactions.

RBI may in consultation with the Central Government specify any class or classes of
transactions which are permissible [Section 6(2)(a)].

According to Regulation 3(1) of the Foreign Exchange Management (Permissible capital


Account Transactions) Regulations, 2000 issued by RBI, Investment in India by a person
resident outside India is a permissible capital account transactions (Schedule II).

Further RBI is empowered under Section 6(3)(b) to prohibit, restrict or regulate, by


regulations, transfer or issue of any security by a person resident outside India.

According to Regulation 5(3)(ii) of the said regulations a NRI may purchase shares of an
Indian Company which is not engaged in Print Media Sector on non-repatriation basis
without any limit (para 2 of Schedule 4). The shares may be issued by the company either
by public issue or private placement. The only condition is that the amount of consideration
for purchase of shares shall be paid by way of inward remittance through normal banking
channels from abroad or out of funds held in NRE/FCNR/NRO/NRSR/N&NR account
maintained with an authorized dealer or as the case may be with an authorised bank in India
(Para 3 of Schedule 4).

© The Institute of Chartered Accountants of India


Page 273
FINAL COURSE
ELECTIVE PAPER 6D: ECONOMIC LAWS
CASE STUDY - 4
Albert John, right from his childhood had high dreams and aspirations. His hobbies included travelling, scuba
diving, gliding, trekking, and other adventurous sports. To go around the globe was his biggest wish.
Therefore, after his schooling, to satisfy his prime interest in travelling, he opted for a two years’ Diploma and
thereafter, a 18 months’ Post Graduate Diploma in Tourism and Travel Industry Management, from the
University of Mumbai.
After post-graduation he had a lot of options like to become a travel agent or join any immigration and customs
services, travel agencies, airline catering or laundry services, etc. Initially, however, he got a chance to join
a reputed travel agency where he gathered on-hand experience and continued with the job for about two
years but his innermost desire was to do his own business and therefore, he opened up a proprietary travel
agency under the name of ‘John Travel Agents’ in Mumbai. Within a period of about five years he could earn
substantially from his business. He used to arrange foreign currency also through his contacts for needy
tourists who did not want to use banking channels for this purpose.
In the meantime, he got married to Neelima George and was comfortably settled in his 2BHK flat in Mumbai.
Keeping in view the future expansion of his travel agency business, he decided to form a private limited
company by the name ‘John and George Travel Agency Pvt. Ltd. having its registered office in Mumbai. After,
it was got registered with ROC, Mumbai with the authorised capital of Rs. 10 lacs, Albert wrapped up its
business conducted so far under ‘John Travel Agents’.
His business was flourishing. By now he had seen most of the touring destinations of Europe and East Asia
by taking his clients around. One of such clients was Chimanbhai Patel, a leading and famous exporter of
Mumbai. He was a rich and dynamic businessman dealing in gold, diamonds and precious as well as semi -
precious stones. He had three companies i.e. Shining Gold Jewellery Pvt. Ltd., which dealt in gold jewellery;
Red Star Pvt. Ltd. which dealt in diamonds and jewellery studded with diamonds; and Blue Sapphire Pvt. Ltd.
which dealt in precious and semi-precious stones. He was the owner of two palatial bungalows situated at
Bandra and Juhu.
Once he took Chimanbhai, his wife and both of their married daughters as well as sons-in-law to Europe for
a family vacation trip. He arranged for them Royal Caribbean’s cruise liner ‘Harmony of the Seas’ which was
like a five star hotel at sea, for a ‘seven night’ cruise starting from Barcelona. While on board, Chimanbhai
proposed him a business deal which required him to deliver gold biscuits worth Rs. one crore to one of his
close friends on his next visit to Hong Kong and for accomplishing this job he would get some hefty
commission. After some hesitation he agreed to do the assigned work and the deal was done. Albert managed
to somehow pass on the tainted wealth as directed by Chimanbhai. In return he got rich kickbacks in the form
of commission; and also admired this new way of earning quick money. The bond between Albert and Patel
grew intense and he accomplished many such assignments including converting of Indian currency
representing black money into foreign currency and delivering it outside India to a safe haven.
This way slowly and gradually Albert entered into money laundering activities. Time passed on. He was a rich
person now. He purchased a new 3BHK flat in the same locality and rented out his old flat. He also acquired
properties in Uttrakhand and Rajasthan and at the same time invested additional funds in purchasing gold
jewellery and diamonds including buying a rust coloured Mahindra XUV500. Though the current line of activity
helped him in fulfilling his high dreams and aspirations but in actuality he was converting proceeds of crime
to make them appear as legitimate money. He was a changed person now keeping the moral ethics at bay.

© The Institute of Chartered Accountants of India


Page 274
Once, while travelling in a Vayu Airways flight from Hong Kong to Mumbai he was impressed by the hospitality
provided by Neetu Bhatia, a member of the cabin crew. An idea clicked him. Albert knew very well that flight
attendants had access to secure venues at airports. At times, they did not require baggage screening. The
cabin crews underwent minimum security check and therefore, needle of suspicion and surveillance was also
minimal in their case. Thinking so, Albert befriended her and through his mesmerizing talks he could gather
that she was a resident of Vile Parle, Mumbai and a regular employee on this route. He developed a story
which reflected his persona as that of a business tycoon. Neetu was highly impressed by the sweet talks and
manners exhibited by Albert and they exchanged mobile numbers.
A few days later, Chimanbhai sent requisite Indian currency to Albert for conversion into around one lac US
Dollars and its deliverance to his business associate in Hong Kong. This time Albert thought of Neetu, called
her and they met at a high profile restaurant ‘Green Tea Day’ in Worli. Albert disclosed her about delivering
of USD one lac in exchange for handsome commission to one of his known and trusted business associates
who would get the money lifted from the Hong Kong International Airport itself. Initially, Neetu was a little bit
hesitant but the desire to earn some quick cash without putting in much effort prevailed over her good senses
and she relented. Both of them, however, knew that it was a criminal conspiracy.
Albert had drawn a very simple modus operandi for her. He wrapped the stacks of dollars in aluminium foils
and carbon sheets to dodge x-ray machine at the Mumbai airport. After placing them in her suitcase he put
her make-up kit and clothes over them. At the security check, as he had anticipated, the foils were passed
off as chocolates. After landing at Hong Kong Airport, Albert’s local conduit picked up the cash from there.
As promised, Neetu got the commission for deliverance. Not being caught in her first operation, Neetu’s
confidence level rose to a considerable extent. A few other consignments, delivered through her, were a no
glitch operation but in the seventh one Albert’s luck ran out and this operation was spoiled by Enforcement
of Directorate (ED) officials who caught her before the plane could take off from the Chhatrapati Shivaji
International Airport, Mumbai.
After being caught, Neetu got frightened and spilled the beans. She was taken under custody by Directorate
of Enforcement Officers and her thorough investigation revealed the involvement of Albert. Her offence of
carrying foreign currency on the behest of Albert was considered to be a Scheduled Offence falling in Part A
of the Schedule to the PML Act (i.e. criminal conspiracy involving Section 120B of the IPC). After following
due procedures including filing a complaint before the jurisdictional Magistrate for taking cognizance of the
scheduled offence, her residence was also searched and gold jewellery worth Rs. 21 lacs was recovered and
Dy. Director duly authorised by the Director took steps to provisionally attach the recovered jewellery in the
presence of two independent witnesses.
Simultaneously, following due procedures, a search team headed by Dy. Director raided the house of Albert.
By the time the officers of ED entered his house, Albert was almost ready to go on a trip to Dubai as a part
of routine job but with a special mission. The officers could smell a rat and took him to his rust coloured
Mahindra XUV500. Immediate search of his car gave way to the recovery of 24 kgs. of gold which was going
to be smuggled out of India through various conduits. It transpired from Albert that the gold belonged to
Chimanbhai Patel, a famous exporter of Mumbai.
From the search of Albert’s residence, various incriminating documents were also recovered. In one of the
almirahs, there was a hidden bottom drawer but the hawk eyes of ED officers were able to detect it. Albert
was asked to open it but he did not oblige giving lame excuse that the keys were misplaced. This compelled
the officers to break open it. When opened forcibly, this secret drawer contained five silver pouches where
narcotic drugs were securely kept. On further enquiry it was found that he had two lockers in two different
banks. A search of the lockers gave way to the recovery of fixed deposits receipts worth Rs. 1.25 crores, hard
cash Rs. 50 lacs and property papers showing properties in Uttrakhand and Rajasthan. In both the properties
his name was not registered as the owner. The title documents of residential property at Uttrakhand contained

© The Institute of Chartered Accountants of India


Page 275
the name of Raj Karan, his driver and the property in Rajasthan was a farm house wh ich was in the name of
Sanju who was the husband of the full time maid-servant Rani working at his house. Both the properties
seemed to be benami properties. Recovery of a green diary from one of the lockers confirmed the name of
Chimanbhai Patel and the various transactions Albert had with him. Time was up for both of them.
Since dealing in drugs was a Scheduled offence under the PML Act, the ED Officers, filed a complaint before
the jurisdictional Magistrate for taking cognizance of the scheduled offence. Thereafter, following the property
attachment procedures, the Dy. Director duly authorised by the Director provisionally attached and seized all
the movable and immovable properties as well as records. Identification marks were placed and an inventory
was made in respect of seized property and records. This was done in the presence of two independent
witnesses.
At the time when Albert revealed the involvement of Chimanbhai, immediately, a search team under the
supervision of duly authorised Dy. Director was sent to the palatial bungalow of Chimanbhai Patel situated at
Bandra. It was found that the bungalow was spread over 5000 sq.ft. approximately. A search of the basement
of his bungalow revealed presence of narcotic drugs and psychotropic substances. In between the wooden
partitions used in the basement, they also found counterfeit Indian currency valuing Rs. 40 crores. It was a
Scheduled offence falling in Part A of the Schedule to the PML Act where amount involved had no
consideration. It was alleged by the Dy. Director that Chimanbhai Patel possessed proceeds of crime but
tried to project the same as untainted property and therefore he was guilty of offence of money-laundering
under Section 3 of the PML Act.
The other - Jal Tarang Mahal residence - a 7 BHK villa of Chimanbhai Patel at Juhu was also raided
concurrently by the ED officials. It was really a humongous, palatial sea-facing bungalow covering
approximately 25,000 sq.ft. area and fully done up with imported and handpicked interiors. The building had
basement, ground floor and a first floor. In a two-day long search, the officials seized, inter-alia, thirty diamond
rings worth Rs. 30 crores; fifty watches worth Rs. five crores; the choicest of rare paintings by M. F. Hussain,
Hebbar, Tyeb Mehta and Amrita Shergil valuing approximately Rs. 21.5 crores which were displayed in a
special air-tight hall so that moisture in the air could not damage them; high end and antique jewellery valued
at Rs. 46 crores; high end cars which included Rolls Royce Ghost, Mercedes Benz, Porsche Panamera, Ford
Mustang, Toyota Fortuner and Innova. The total attachment and seizure of diamonds, gold, precious and
semi-precious stones and other movable and immovable assets stood at Rs. 6562 crores. His various
companies were also searched and a number of incriminating documents, files, computers, etc. were seized.
The intensive search revealed that Chimanbhai used to bring his own black money from about twenty shell
companies based at Hong Kong and Dubai into the accounts of his three main companies in India as foreign
direct investment. He subsequently diverted these funds into the accounts of various shell companies
describing transfers as unsecured loans from where the funds were siphoned off through various means
including cash withdrawals.
An investigation was also conducted under FEMA, 1999 for alleged violations of Sections 3 and 4 of FEMA
for dealing in and acquiring and holding foreign exchange in his account with United Royal Bank of
Switzerland whose value in Indian currency was approximately Rs. 3,600 crores.
Being a Scheduled offence falling in Part A, it was required of authorised ED Officers, to file a complaint
before the jurisdictional Magistrate for taking cognizance of the offence which was done immediately.
Thereafter, following the provisions of Section 17, the ED Officers seized all the movable and immovable
properties as well as records in the presence of two independent witnesses. As in the case of Albert,
identification marks were placed and an inventory of the seized property was also made.
All of them were arrested by the authorised ED Officers, since ED officers, on the basis of material in their
possession, had reason to believe that they were guilty of an offence punishable under PML Act. I mmediately

© The Institute of Chartered Accountants of India


Page 276
after their arrest, the officers forwarded a copy of the order along with the material in their possession to the
Adjudicating Authority in a sealed envelope, in the prescribed manner. Further, the guilty persons were, within
24 hours, taken to the jurisdictional Magistrate.
As we have noted earlier, keeping in view Section 5 (1), in all the above cases, the ED Officers, through
written orders provisionally attached the properties because it was suspected that they were derived from the
proceeds of crime. The ED Officers knew that the maximum period of attachment would be limited to 180
days from the date of the order.
Thereafter, the ED Officers forwarded the copies of the orders provisionally attaching the properties of Neetu,
Albert and Chimanbhai Patel along with the various documents in his possession to the Adjudicating Authority
in a sealed envelope.
The ED Officers also filed complaints stating the facts of such provisional attachments before the Adjudicating
Authority within thirty days of such attachments.
The Adjudicating Authority served on Neetu, Albert and Chimanbhai Patel notices to explain in not less than
30 days their source of income, earning or assets out of which they had acquired the attached property.
The attachment of the properties was confirmed by the Adjudicating Authority bearing in mind that such properties
were involved in money laundering being obtained through the proceeds of crime. However, such confirmation was
made only after considering the replies of the aggrieved persons as well as after hearing them.
In terms of confirmation order passed by the Adjudicating Authority, the ED Officers, forthwith took the
possession of the attached properties.
The trial of the above money laundering offences is being done by the jurisdictional Special Court. The Central
Government in consultation with the Chief Justice of the High Court is empowered to designate one or more
Courts of Sessions as Special Court or Special Courts for trial of offence of money laundering .
Under Section 4, if Neetu, Albert and Chimanbhai Patel are found to have committed the offence of money-
laundering, then they shall be punishable with rigorous imprisonment which shall be minimum three years
and maximum seven years and shall also be liable to fine. In case it is proved that th e proceeds of crime
involved in money-laundering relate to any offence specified under paragraph 2 of Part A of the Schedule,
the maximum punishment may extend to ten years instead of seven years.
On conclusion of a trial, if the Jurisdictional Special Court finds that the offence of money-laundering has
been committed, it shall order that the properties involved in the money laundering shall stand confiscated to
the Central Government.
If on conclusion of a trial, the Special Court finds that the offence of money laundering has not taken place,
it shall order release of such property to the person entitled to receive it.
Required (MCQ of 2 marks each)
Select the most appropriate answer from the options given for each question:
1. Whether the 2BHK flat owned by Albert but rented out can be considered to have been derived from the
proceeds of crime:
(a) Yes it can be considered because Albert, the owner, is involved in money laundering activities;
(b) No, it cannot be considered because Albert did not purchase it from funds obtained through money
laundering activities;
(c) No, it cannot be considered because Albert has rented it out;
(d) None of the above.

© The Institute of Chartered Accountants of India


Page 277
2. Adjudicating Authority may serve a notice of not less than --------- on Neetu, Albert and Chimanbhai
Patel who are believed to have committed offence of money laundering to explain their source of income,
earning or assets out of which they had acquired the attached property.
(a) 14 days
(b) 30 days
(c) 60 days
(d) None of the above
3. After provisional order of attachment is confirmed by the Adjudicating Authority, the Director shall
forthwith -------------.
(a) confiscate the attached properties;
(b) take the possession of the attached properties;
(c) seize the attached properties;
(d) None of the above
4. Provisional attachment of property of Chimanbhai Patel suspected to be involved in money laundering
ensures that he is prohibited to:
(a) transfer the attached property;
(b) convert the attached property;
(c) dispose of the attached property;
(d) All of the above
5. A complaint with the Adjudicating Authority is to be filed within a period of --------- days by the Director
who provisionally attaches the property involved in money laundering.
(a) 15
(b) 20
(c) 30
(d) 60
6. If on conclusion of a trial by the Jurisdictional Special Court, the guilt of Neetu, Albert and Chimanbhai
Patel is proved, it shall make an order to --------
(a) Freeze the attached property;
(b) Confiscate the attached property;
(c) Seize the attached property;
(d) None of the above.
7. Chimanbhai Patel was found to have been in possession of counterfeit Indian currency which is a
Scheduled offence belonging to:
(a) the Unlawful Activities (Prevention) Act, 1967;
(b) the Indian Penal Code;
(c) the Prevention of Corruption Act, 1988;
(d) None of the above.
8. An offence specified in Part B of the Schedule shall be considered as Scheduled offence under PML Act
only if the total value involved in such offence is --------.
5

© The Institute of Chartered Accountants of India


Page 278
(a) Rs. 30.00 lacs or less;
(b) Rs. 50.00 lacs or less;
(c) Rs. 1.00 crore or more;
(d) None of the above.
9. Both Raj Karan and Sanju are to be considered as Benamidar because they are the:
(a) fictitious persons who have not made any payment for purchase of properties;
(b) persons in whose name the benami properties are held without making any payment by them;
(c) persons who have lent their names to be owners of the properties without making any payment by
them;
(d) All of the above.
10. Any property found to be involved in money laundering cannot be provisionally attached by the Director,
ED for more than ---------------.
(a) 30 days
(b) 60 days
(c) 90 days
(d) 180 days
Required (Descriptive Questions of 10 marks each)
1. As per the facts, Albert through laundered money purchased 3 BHK. Suppose if the said flat is purchased
by him jointly on his and his wife’s name, Neelima Goerge. Examine in the light of the Prevention of
Money Laundering Act 2002, the following situtaions:
(a) Will Neelima be also liable for holding of the such joint property.
(b) If property is claimed by a person, other than whom the notice has been issued. Discuss the legal
position of the person claiming the property.
2. (a) Suppose Mr. X, a non-resident Indian, purchases a flat of Albert in India, for Rs. 50,00,000 and
paid 30,00,000 in by account payee cheque of his own account and rest in cash. The registry was
done at a value of Rs 30,00,000 which was paid by cheque. Discuss the nature of the transaction.
(b) (i) Albert was assigned by Chiman Bhai to deliver counterfeit currency-notes to one of his close
friends to Honkong for which hefty commission was fixed by the Chiman Bhai. Discuss,
whether the said act can be considered as money laundering. Who shall be liable for the
commission of the money Laundering.?
(ii) State whether maintenance of foreign currency accounts in India and outside India by Albert
is permittable in FEMA, due to his nature of business.
3. (i) Chiman bhai is a person resident in India. He has different business units as to manufacturing &
designing of jewellary in Hongkong which is owned by him. How will you determine whether a
particular business units of Chiman bhai is a ‘person resident in India’ under the Foreign Exchange
Management Act, 1999?
(ii) Suppose if ‘Blue Sapphire Pvt. Ltd.” is a Singapore based company having several business units
all over the world. It has a unit for cutting & manufacturing precious and semi-precious stones in
the form to be used for the jewellary with its Headquarters in Mumbai. It has a Branch in Dubai
which is controlled by the Headquarters in Mumbai. What would be the residential status under the
FEMA, 1999 of units of Blue Sapphire Pvt. Ltd in Mumbai and that of Dubai branch?

© The Institute of Chartered Accountants of India


Page 279
ELECTIVE PAPER 6D- ECONOMIC LAWS

SOLUTION – CASE STUDY 4

I. ANSWERS TO OBJECTIVE TYPE QUESTIONS

1. (b)

[Hint: Based on Section 2 (1) (u) of the Prevention of Money-Laundering Act, 2002]

2. (b);

[Hint: Refer Section 8 of the Prevention of Money-Laundering Act, 2002].

3. (b);

[Hint: Refer section 8 (4) of the Prevention of Money-Laundering Act, 2002].

4. (d);

[Hint: Based on Section 2 (1) (d) of the Prevention of Money-Laundering Act, 2002]

5. (c);

[Hint: Refer Section 5 (5) of the Prevention of Money-Laundering Act, 2002].

6. (b);

[Hint: Refer Section 8 (3) and 8 (6) of the Prevention of Money-Laundering Act, 2002].

7. (b);

[Hint: Refer Schedule to the Prevention of Money-Laundering Act, 2002]

8. (c);

[Hint: Refer Section 2 (1) (y) of the Prevention of Money-Laundering Act, 2002].

9. (d);

Hint: Refer Section 2 (10) of the Prohibition of Benami Property Transactions Act, 1988.

10. (d):

[Hint: Refer Section 5 (1) of the Prevention of Money-Laundering Act, 2002].

II. ANSWERS TO DESCRIPTIVE QUESTIONS

1. According to section 8 of the Prevention of Money Laundering Act, 2002, on receipt of a


complaint or applications, if the Adjudicating Authority has reason to believe that any person

© The Institute of Chartered Accountants of India


Page 280
has committed an offence of money laundering or is in possession of proceeds of crime, he may
serve a notice of not less than thirty days.

Such person shall be called upon to indicate the sources of his income, earning or assets, out
of which or by means of which he has acquired the property so or, seized or frozen.

However, where a notice specifies any property as being held by a person on behalf of any other
person, a copy of such notice shall also be served upon such other person. Where if, such
property is held jointly by more than one person, such notice shall be served to all persons
holding such property.

The Adjudicating Authority shall, after hearing the aggrieved person and the Director or any
other officer authorised by him in this behalf, and taking into account all relevant materials
placed on record before him, by an order, record a finding whether all or any of the properties
referred to in the notice issued , are involved in money-laundering. Provided that if the property
is claimed by a person, other than a person to whom the notice had been issued, such person
shall also be given an opportunity of being heard to prove that the property is not involved in
money-laundering.

According to the above stated provisions, following are the answers:

(a) Since in the given case, Alberts holds the property jointly in his and his wife’s name i.e.
Neelima George. As per the above law, such notice shall be served to all persons holding
such property. So accordingly, Neelima will also be served the notice, and being heard.
Taking into account all relevant materials placed on record before him, by an order, record
a finding whether all or any of the properties referred to in the notice issued , are involved
in money-laundering, then in such case Neelima will also be liable for holding of the joint
property.

(b) If property is claimed by a person, other than whom the notice has been issued therein,
such person shall also be given an opportunity of being heard to prove that the property is
not involved in money-laundering.

2. (a) According to the Explanation given to section 2(9) of the Prohibition to Benami
Transaction Act, Benami transaction shall not include any transaction involving the
allowing of possession of any property to be taken or retained in part performance of a
contract, where—

(i) consideration for such property has been provided by the person to whom
possession of property has been allowed but the person who has granted
possession thereof continues to hold ownership of such property;

(ii) stamp duty on such transaction or arrangement has been paid; and

© The Institute of Chartered Accountants of India


Page 281
(iii) the contract has been registered

Since the property is in the name of Mr. X and not in others name and it is registered on
duly paid stamp duty, it is not a Benami Transaction.

(b) (i) As per the Prevention of Money Laundering Act, 2002, whosoever directly or
indirectly attempts to indulge or knowingly assists or knowingly is a party or is
actually involved in any process or activity connected with the proceeds of crime
including its concealment, possession, acquisition or use and projecting or
claiming it as untainted property shall be guilty of offence of money laundering
(Section 3).

“Proceeds of crime” means any property derived or obtained, directly or indirectly,


by any person as a result of criminal activity relating to a scheduled offence or the
value of any such property [Section 2(1)(u)].

Every Scheduled Offence is a Predicate Offence. The occurrence of the scheduled


Offence is a pre requisite for initiating investigation into the offence of money
laundering.

In the given case , Chiman Bhai assigned Albert to deliver counterfeit currency
notes to be given to his friends in Hongkong , which is an offence falling within the
purview of scheduled offence in Part A of the PMLA, 2002 under section 489B of
the IPC. This section deals with the using as genuine, forged or counterfeit
currency-notes or bank-notes. According to the section whoever sells to, or buys
or receives from, any other person, or otherwise traffics in or uses as genuine, any
forged or counterfeit currency-note or bank-note, knowing or having reason to
believe the same to be forged or counterfeit, shall be liable under the Prevention of
Money Laundering Act.

Hence, Albert, Chiman Bhai and his friends in Hongkong, all are said to be liable
under the Prevention of Money Laundering Act.

(ii) The Foreign Exchange Management (Permissible Capital Account Transactions)


Regulations, 2000 specifies list of transactions, which are permissible in respect of
persons resident in India in Schedule I.

Schedule I states the list of permissible classes of transactions made by persons


resident in India. List specifies, maintenance of foreign currency accounts in India
and outside India by a person resident in India. Accordingly, maintenance of foreign
currency accounts in India and outside India by Albert is permittable in FEMA.

© The Institute of Chartered Accountants of India


Page 282
3. (i) Person resident in India

Section 2(v) of FEMA, 1999 defines the term “person resident in India”. According to Section
2(v) (iii), all business units in India will be “resident in India” even though these units are
owned or controlled by a person resident outside India.

Similarly all business units outside India will be ‘resident in India’ provided the business
units are either owned or controlled by a person resident in India [Section 2(v) (iv)].

It is necessary to determine the residential status of the person (i.e.,Chiman bhai) who
owns or controls the business units in outside India.

(ii) Blue Sapphire Pvt. Ltd., being a Singapore based company would be person resident
outside India [(Section 2(w)]. Section 2 (u) defines ‘person’ under clause (vii) thereof, as
person would include any agency, office or branch owned or controlled by such person. The
term such person appears to refer to a person who is included in clause (i) to (vii).
Accordingly Blue Sapphire Pvt. Ltd. unit in Mumbai, being a branch of a company would be
a ‘person’.

Section 2(v) defines a person resident in India. Under clause (iii) thereof person resident in
India would include an office, branch or agency in India owned or controlled by a person
resident outside India. Blue Sapphire Pvt. Ltd unit in Mumbai is owned or controlled by a
person resident outside India, and hence it, would be a ‘person resident in India.’

However, Dubai Branch though not owned, is controlled by Blue Sapphire Pvt. Ltd. unit in
Mumbai which is a person resident in India. Hence prima facie, it may be possible to hold a
view that the Dubai Branch is a person resident in India.

© The Institute of Chartered Accountants of India


Page 283
ELECTIVE PAPER 6D: ECONOMIC LAWS

Case Study 5
Mrs. Shakuntala Bisht was a dynamic woman entrepreneur running her factory of manufacturing designer candles
and other items made of wax as a proprietary concern in Dehradun (Uttrakhand) titled as M/s. Bisht Designer
Candles since 2003. She had appointed a number of dealers pan India for selling her designer products.
She was residing in a bungalow on Subhash Road in Dehradun along with her family. Her husband Mr. O. P. Bisht
was joint-secretary in Uttrakhand Sachivalaya. Her son Varun had done his B.E. (Bachelor of Engineering) from
IIT, Kharagpur and thereafter MBA from IIM, Kolkata in the year 2013. Her daughter Latika was pursuing B.Sc.
(Hons.) in Physics from DIT University, Dehradun.
Varun, being a brilliant student, secured a job in Accenture through Campus placement. He attended a three
months’ residential training programme and joined as Assistant Manager (Operations) in Pune branch of the
company. He took a one BHK flat on rent at Hinjewadi locality, purchased some furniture and other daily household
items and got himself settled in the new atmosphere. He was happy and content as the package offered to him
was very lucrative.
Mrs. Bisht had high aspirations and was desirous of expanding her business further. Therefore, in the year 2013,
she thought of exporting her products to various countries crossing the borders of India. After discussing with her
family members, she decided to convert her proprietary concern into a private limited company. Accordingly, she
got registered her company under the title Bisht Designer Candles Pvt. Ltd. in which she and her daughter were
directors while all of the four family members were shareholders. Thereafter, she completed various formalities
required for exporting her product which, inter-alia, included obtaining a ten digit importer-exporter code (IEC)
number from Directorate General of Foreign Trade (DGFT).
In the year 2015, she sent her first export consignment of designer candles to a foreign buyer in Berlin, Germany.
The order amounted to € 20800 and the importer was required to make payment in three months after shipment.
As per the terms and conditions a Letter of Credit (L/C) was opened by the Deutsche Bank on behalf of the importer.
Before shipping goods, Mrs. Bisht had to fill requisite export declaration form since the consignment did not fall in
exempted category as mentioned in Regulation 4 of the FEM (Export of Goods and Services), Regulations, 2015.
After shipment of goods, she submitted the documentary bill of exchange drawn under L/C to Syndicate Bank,
Dehradun and got it discounted under her sanctioned bills discounting limit. On the due date Syndicate Bank
received the export payment and squared off her liability.
Subsequently, she explored candle market in the USA and came in contact with M/s. Williams’ Art Gallery in Boston
which had a five storey departmental store. In this store, one of the floors was meant only for designer candles and
other items made of wax. After due negotiations with the CEO Mr. Williams, she managed to get an advance of
50,000 USD being 50% of the total export value. It was well within her knowledge that in case an advance was
received against export to a foreign buyer, the shipment of goods was to be made within one year of receipt of
advance and the export documents were required to be routed through the same authorised dealer which received
the advance on her behalf. She shipped the goods much before one year and also got payment well within the
statutory period of nine months from the date of export.
On the of successful settlement of her first export consignment to M/s. Williams’ Art Gallery of Boston, she took
steps to complete another export order from the same party for USD 1,00,500. However, this time no advance
payment was made by the importer and on the basis of his firm order, she dispatched the consignment of designer
candles. After shipment of goods, she submitted the documentary B/E to Syndicate Bank, Dehradun for
discounting. As per the agreement, the importer was to make payment on the completion of five months from the

© The Institute of Chartered Accountants of India


Page 284
receipt of consignment at his godown. However, by the time five months were over, the importer could make
payment of only 40% of the total export value.
Being in need of funds, she started raising and collecting funds from various sources. In one of the cases she had
given an unsecured loan of Rs. 5 lacs to a private limited company in which a distant relative of her husband was
a director. However, when she demanded her loan back from the company, it was transpired that the company
was under liquidation process before the National Company Law Tribunal under Insolvency and Bankruptcy Code,
2016.
Varun was doing his job at Pune to the complete satisfaction of his superiors. In the next three years’ time after
joining Accenture, Varun could save a lot of money as he was a man of few needs. One day, a casual talk with the
local grocer Ajay Gupta gave him an idea to buy a flat in a housing society. Ajay gave him the phone number of a
known property dealer, Mr. Rajnikant. Thereafter, a meeting was fixed in the office of Mr. Rajnikant where he
noticed a Certificate of Registration hanging on the wall of his office. On enquiry, he was told that now it was
mandatory for the property agents to get themselves registered under Real Estate (Regulation and Development)
Act, 2016. After seeing the certificate Varun could conclude that he was dealing with a genuine person. After due
negotiations, a ground floor 2BHK apartment was finalised in Vayudoot Apartments at a cost of Rs. 62.35 lacs. He
himself arranged Rs. 30 lacs out of his savings; obtained a housing loan of Rs. 20 lacs from Axis Bank while the
remaining amount of Rs. 12.35 lacs was given by his father out of his personal savings. The title deeds got
registered in his name after making payment of stamp duty and other statutory dues. On an auspicious day Varun
shifted to his new flat.
About after a month of shifting to his own flat, Varun’s boss called him and informed that recognizing his hard work
and devotion towards the company he was being transferred to Boston, USA on promotion as Manager
(Operations). He was beaming with happiness and thanked his boss from the bottom of his heart. He was supposed
to join within next one month.
He went back to Dehradun, completed various formalities including obtaining of visa, packed his belongings and
bade goodbye to his family. On the advice of his father he leased out his flat on rent to a reputed private company
and then flew to Boston and joined his job. Over there, he was provided with a furnished apartment by the company
in the suburbs of Boston. As daily commuting was a bit difficult, he purchased a second-hand SUV. Slowly and
gradually he settled in his new home, new office and new country.
Here in Dehradun, Mrs. Bisht was pursuing vigorously to obtain export payment from M/s. Williams’ Art Gallery
because the statutory period of nine months was over long back and the remaining payment was yet to be received.
In the meantime, the authorities at Syndicate Bank also started pressurizing Mrs. Bisht to get the foreign exchange
realised at the earliest since the statutory period of nine months was already over. They opined that in cases of
default the Reserve Bank of India may also issue appropriate directions for the purpose of securing the payment if
the goods were sold in USA or if they were still unsold to get them re-imported into India within the specified period.
Though the Reserve Bank had not so far issued any directions but according to her bankers, omission on the part
of RBI to give directions did not absolve her from the consequences of committing the contravention. Therefore,
she was duty bound to realise the export payment as early as possible.
Besides taking various steps, she also persuaded her son Varun who was already in Boston to follow the matter
vigourously and advised him to meet Mr. Williams personally and settle the case. A meeting was fixed and during
conversation, it was transpired that though Mr. Williams had sold whole of the consignment, the purchaser was yet
to make payment because of some mismanagement. However, on the vigourous persuasion of Varun, Mr. Williams
exerted pressure on the local purchasers and within next one month, remaining payment along with interest was
realised and repatriated to India.

© The Institute of Chartered Accountants of India


Page 285
Varun had a school friend Raman Verma in India who had done MBA from Symbiosis, Pune after his graduation
from Dehradun and had joined sales team of LIC at Shimla. From time to time after joining Accenture in Boston,
Varun was persuading him to visit Boston and nearby areas along with his wife Vaishnavi Verma. At last, Raman
and his wife agreed for the foreign visit and both of them obtained visa.
Raman approached Canara Bank, Shimla for purchase of USD 12,000 for a private visit to the USA. The bank
without much formalities gave him the required amount in foreign currency since it was well within USD 2,50,000,
i.e. an amount which could be remitted by a resident individual in a financial year under Liberalised Remittance
Scheme (LRS). Moreover, the foreign currency was not required to be remitted for any prohibited current account
transaction [mentioned in Schedule I to the FEM (Current Account Transactions) Rules, 2000] like participation in
lottery schemes or lottery like schemes existing under different names like money circulation scheme or
remittances for the purpose of securing prize money/awards, etc. He was asked to submit a simple letter containing
the basic information, viz., his name, address and that of beneficiary (i.e. self), SB account number, amount to be
remitted and the purpose of remittance along with a cheque of equivalent amount in rupees. In no time, both of
them reached the USA.
Varun received Raman and his wife with open heart at the Logan International Airport, Boston and all of them
drove to his residence. The next ten days were full of fun and frolic. They visited a number of famous sites which
included John F. Kennedy Presidential Museum & Library, Boston Public Library which was opened in 1852 as the
first free publicly-supported municipal library in America, Museum of Fine Arts having world's most comprehensive
art collections, Boston Public Garden famous for its Swan Boats and having over 600 varieties of trees, Old North
Church & Historic Site where the two famous signal lanterns were hung launching the American Revolution, New
England Holocaust Memorial where its six glass towers represented the six million Jews who perished in the
holocaust, Bunker Hill Monument, etc. In between, they had an overnight stay at New York as well.
Varun helped them in purchasing some nice dresses, chocolates, perfumes, cosmetic items and also some
souvenirs for their relatives and friends in India. They enjoyed their trip to USA to the fullest and flew back to India
with nice memories.
Raman still had with him unspent amount of USD 3500. On enquiry with his bankers regarding surrender of this
amount he was informed that he could surrender to the bank any unspent foreign exchange within a period of 180
days from the date of his return to India. Even if he approached the bank after this period, the bank would not
refuse to purchase unspent foreign exchange merely because the prescribed period of 180 days had expired. He
was further informed that he was permitted to retain with him foreign currency notes up to USD 2000 and foreign
coins without any ceiling beyond 180 days and he could utilize this amount for his subsequent visit abroad.
Varun wanted to be inform regarding sale of his flat in Pune if he was to settle down in the USA permanently since
his family at Dehradun was not that much inclined to keep the flat. He once again approached Mr. Rajnikant and
enquired whether he, as NRI, could sell his flat. Mr. Rajnikant after obtaining necessary information from one of
his lawyer friends, informed him that he was permitted to sell his flat in India to a person resident in India. Further,
he could also sell the flat (since it was not an agricultural or plantation property or farm house) to a person resident
outside India who is an Indian citizen or to a person of Indian origin resident outside India. Such permission was
available under Regulation 3 of FEM (Acquisition and Transfer of Immovable Property in India), Regulations, 2000.
As regards purchase of immovable property at Boston, Varun was informed that FEMA did not restrict such
acquisition by a non-resident Indian and he had to follow local laws in this respect. However, if his family members
in India remitted to him funds under the Liberalised Remittance Scheme (LRS) for purchasing immovable property
outside India, then the said property should be in the name of all the members who made the remittances. Even
as per Section 6(4) of the FEMA, if he becomes a person resident in India in future, he would be allowed to hold,
own or transfer the immovable property situated outside India because such property was acquired by him when
he was resident outside India.
3

© The Institute of Chartered Accountants of India


Page 286
I. Objective Type Questions (2 marks each)
Select the most appropriate answer from the options given for each question:
1. Which of the following remittance would require prior approval of the Reserve Bank of India?
(a) Donation exceeding 0.5% of foreign exchange earning during the previous three financial years or USD
40,00,000, whichever is less for contribution to funds promoted by educational institutes,
(b) Commission per transaction to agents abroad for sale of commercial plots in India of USD 20,000 or 4%
of the inward remittance whichever is more,
(c) Remittance exceeding USD 10,00,000 per project for other consultancy services procured from outside
India.
(d) Remittance of 4% of investment brought into India or USD 90,000 whichever is higher, by an entity in
India by way of reimbursement of pre-incorporation expenses.
2. Mr. O. P. Bisht’s name does not appear in the registration papers relating to Pune apartment purchased by
Varun though he contributed Rs. 12.35 lacs towards the cost of the apartment.
(a) It is a benami transaction to the extent of Rs. 12.35 lacs.
(b) It is wholly a benami transaction.
(c) It is not a benami transaction
(d) None of the above
3. Export of the following goods/software would require furnishing of the declaration under FEMA, 1999?
(a) Goods imported free of cost on re-export basis
(b) Publicity material supplied free of payment
(c) By way of gift of goods accompanied by a declaration by the exporter that they are of six lakh rupees in
value
(d) Unaccompanied personal effects of travellers
4. An exporter receiving advance payment against exports from the foreign buyer is required to make the
shipment of the goods within ---------- of receiving advance payment, if export agreement does not mention
anything to the contrary regarding time period:
(a) 6 months
(b) 9 months
(c) One year
(d) One and a half years
5. An Indian citizen resident outside India is permitted to transfer his agricultural property in India to:
(a) any person resident in India
(b) any person resident outside India if he is a citizen of India or a person of Indian origin.
(c) Neither (a) nor (b)
(d) both (a) and (b)
6. Foreign exchange purchased from an authorised dealer by a resident individual, if remains unspent, needs
to be surrendered to the authorised dealer within ------------ of purchase or date of his return to India:
(a) 60 days
4

© The Institute of Chartered Accountants of India


Page 287
(b) 90 days
(c) 120 days
(d) 180 days
7. In case of goods valuing up to Rs. 5,00,000 as declared by the exporter and sent by way of gift to an importer
in a foreign country:
(a) an export declaration need to be furnished
(b) an export declaration need not be furnished
(c) furnishing of export declaration depends upon the discretion of the authorised dealer who handles
export documents
(d) furnishing of export declaration depends upon the discretion of the Custom authorities
8. The term 'Moratorium' in the Insolvency and Bankruptcy Code, means-
(a) A temporary prohibition on an activity by the competent authority.
(b) A period declared by the NCLT, during which no action can be taken against the Company or the
assets of the Company.
(c) Suspension order of the Board on the debtor's operations.
(d) Order issued by the NCLT prohibiting an action against the creditor.
9. Is it possible for a non-resident Indian to acquire immovable property outside India:
(a) No, it is not possible
(b) Yes, it is possible
(c) Yes, it is possible but subject to the permission of RBI
(d) None of the above
10. As per the Insolvency and Bankruptcy Code, 2016, an Interim Resolution professional approved by the
committee of Creditors:
(a) Can never be replaced until the conclusion of the resolution process
(b) Has a fixed term of 180 days
(c) Can be replaced with 75% voting in favour of the decision and approval of the Board
(d) Can be replaced with 75% voting in favour of the decision.

II Descriptive Questions (10 marks each)


1. Analyse the following situations under the Foreign Exchange Management Act, 1999:
(i) Forex Dealers Ltd. is an Authorised Person within the meaning of Foreign Exchange Management Act,
1999. Reserve Bank of India issued certain directions to the said Authorised Person to file certain returns,
which it failed to file. You are required to state the penal provisions to which the said Authorised Person has
exposed itself.
(ii) Mr. Shekhar resided for a period of 150 days in India during the Financial year 2016-2017 and thereafter
went abroad. He came back to India on 1 st April, 2017 as an employee of a business organization. What
would be his residential status during the financial year 2017-2018?

© The Institute of Chartered Accountants of India


Page 288
(iii) ‘Printex Computer’ is a Singapore based company having several business units all over the world. It has
a unit for manufacturing computer printers with its Headquarters in Pune. It has a Branch in Dubai which is
controlled by the Headquarters in Pune. What would be the residential status under the FEMA, 1999 of printer
units in Pune and that of Dubai branch?
2. Examine with reference to the provisions of the Foreign Exchange Management Act, 1999 whether there
are any restrictions in respect of the following:-
(i) Drawal of Foreign Exchange for payments due for depreciation of direct investment in the ordinary
course of business.
(ii) A person, who is resident of U.S.A. for several years, is planning to return to India permanently. Can
he continue to hold the investment made by him in the securities issued by the companies in U.S.A.?
(iii) A person resident outside India proposes to invest in the shares of an Indian company engaged in
construction of farm houses.
(iv) A person, who is resident of Canada, is planning to acquire an immovable property in Mumbai.
3. Analyze the following situations under the Real Estate (Regulation and Development) Act, 2016:
(i) Mr. Ram booked a 4 BHK flat under the Gateways project. The project is under supervision of Mr.
Pankaj. Mr. Pankaj without telling the allottees reduced the number of rooms from 4 to 3 himself.
Whether this is allowed under the Act and what remedies does the Allottees have.
(ii) Mr. Vivaan booked a 4 BHK flat under the Flower Valley project for a total cost of Rs. 2 Crore. The
project is under supervision of Mr. Shyam. Mr. Shyam put a condition to pay Rs. 50 Lakhs as an
application fee before entering into a written agreement for sale with Mr. Vivaan. Decide whether the
contention of Mr. Shyam is valid?

© The Institute of Chartered Accountants of India


Page 289
ELECTIVE PAPER 6D: ECONOMIC LAWS

SOLUTION

Case Study 5

ANSWER 1
I ANSWERS TO OBJECTIVE TYPE QUESTIONS

1. (c) [Hints: Refer Regulation 2 of Schedule III of Foreign Exchange Management (Current
Account Transactions) Rules, 2000]
2. (c) [Hint: Refer Section 2 (9) of the Prohibition of Benami Property Transactions Act, 1988.
It is not a benami transaction because all statutory dues have been paid and his father
knew about the transaction. Therefore, it falls under exempted category. The amount so
contributed can be a loan or gift to the son.]
3. (c) [Hint: Refer Regulation 4 of the Foreign Exchange Management (Export of Goods and
Services) Regulations, 2015]
4. (c) [Hint: Refer Regulation 15 of the FEM (Export of Goods and Services), Regulations, 2000]
5. (a) [Hint: Refer Regulation 3 of FEM (Acquisition and transfer of immovable property in India)
Regulations, 2000]
6. (d) [Hint: Refer Regulation 7 of FEM (Realisation, Repatriation and surrende r of Foreign
Exchange) Regulations, 2015]
7. (b) [Hint Refer Regulation 4 the FEM (Export of Goods and Services), Regulations, 2015
which has exempted such export transaction from furnishing of export declaration]
8. (b) [Hint: Section 14 of the Insolvency and Bankruptcy Code, 2016, describes moratorium. It
is an order passed by the adjudicating authority (NCLT) declaring a moratorium on the
debtor's operations for the period of the Insolvency Resolution Process, during which no
action can be taken against the Company or the assets of the Company. This operates
as a 'calm period' during which no judicial proceedings for recovery, enforcement of
security interest, sale or transfer of assets, or termination of essential contracts can take
place against the debtor.]
9. (b) [Hint: FEMA does not impose any restriction on acquisition of immovable property outside
India by a non-resident Indian. Further, when at a future date the person concerned
becomes a person resident in India, Section 6(4) even permits him to hold, own or transfer
immovable property situated outside India since such property was acquired by him when
he was resident outside India]
10. (c) [Hint: As per section 22 of the Insolvency and Bankruptcy Code, 2016, an Interim
Resolution professional approved by the Committee of Creditors can be replaced with
75% voting in favour of the decision and approval of the Board.
II. ANSWERS TO DESCRIPTIVE QUESTIONS
1. (i) Section 11(3) of the Foreign Exchange Management Act, 1999 states that where any
Authorised person contravenes any direction given by the Reserve Bank of India under the
said Act or fails to file any return as directed by the Reserve Bank of India, the Reserve Bank
of India may, after giving reasonable opportunity of being heard, impose on Autho rised
Person, a penalty which may extend to ten thousand rupees and in the case of continuing

© The Institute of Chartered Accountants of India


Page 290
contraventions with an additional penalty which may extend to two thousand rupees for every
day during which such contravention continues.
Since as per the facts given in the question, the Authorised person, namely, Forex Dealers
Ltd., has failed to file the returns as directed by the Reserve Bank of India. According to the
above provisions, it has exposed itself to a penalty which may extend to ten thousand rupees
and in the case of continuing contraventions in the nature of failure to file the returns, with an
additional penalty which may extend to two thousand rupees for every day during which such
contravention continues.
(ii) According to the provisions of section 2(v) of the Foreign Exchange Management Act, 1999,
a person in order to qualify for the purpose of being treated as a "Person Resident in India"
in any financial year, must reside in India for a period of more than 182 days during the
preceding financial year. In the given case, Mr. Shekhar has resided in India for a period of
only 150 days, i.e., less than 182 days, during the financial year 2016-2017. Hence, he cannot
be considered as a "Person Resident in India" during the financial year 2017-2018 irrespective
of the purpose or duration of his stay.
(iii) Printex Computer being a Singapore based company would be person resident outside India
[(Section 2(w)]. Section 2 (u) defines ‘person’ under clause (vii) thereof, as person would
include any agency, office or branch owned or controlled by such person. The term such
person appears to refer to a person who is included in clause (i) to (vii). Accordingly printex
unit in Pune, being a branch of a company would be a ‘person’.
Section 2(v) defines a person resident in India. Under clause (iii) thereof person resident in
India would include an office, branch or agency in India owned or controlled by a person
resident outside India. Printex unit in Pune is owned or controlled by a person resident outside
India, and hence it, would be a ‘person resident in India.’
However, Dubai Branch though not owned is controlled by Printex unit in Pune which is a
person resident in India. Hence prima facie, it may be possible to hold a view that the Dubai
Branch is a person resident in India.
2. Capital Account Transactions: All the transactions referred to in the question are capital account
transactions.
Section 6(2) of FEMA, 1999 provides that the Reserve Bank may in consultation with the Centra l
Government specify the permissible capital account transactions and the limit upto which foreign
exchange will be allowed for such transactions.
(i) Depreciation of direct investments: According to proviso to section 6(2), the Reserve bank
shall not impose any restriction on the drawal of foreign exchange for certain transactions.
One such transaction is drawal of foreign exchange for payment due for depreciation of direct
investment in the ordinary course of business. Hence this transaction is permissi ble without
any restrictions.
(ii) Person resident in USA returning permanently to India: When the person returns to India
permanently, he becomes a resident in India. Section 6(4) provides that a person resident in
India may hold, own, transfer or invest in foreign currency, foreign security, etc. if such
currency, security or property was acquired, held or owned by such person when he was
resident outside India or inherited from a person who was resident outside India. In view of
this, the person who returned to India permanently can continue to hold the foreign security
acquired by him when he was resident in U.S.A.
(iii) Investment in shares of Indian company by non-resident: Reserve Bank issued Foreign
Exchange Management (Permissible Capital Account Transactions) Regulations, 2000.
Regulation 4(6) of the said Regulations prohibits a person resident outside India from making
investment in India, in any form, in any Company or partnership firm or proprietary concern
or any entity, whether incorporated or not, which is engaged or proposes to engage in

© The Institute of Chartered Accountants of India


Page 291
construction of farm houses. Hence it is not possible for a person resident outside India to
invest in the shares of a company engaged in construction of farm houses as such investment
is prohibited.
(iv) Acquisition of immovable property by person resident outside India: Reserve Bank
issued Foreign Exchange Management (Permissible Capital Account Transactions)
Regulations, 2000. The regulations specify the classes of capital account transactions of
persons resident outside India in Schedule II. Under this schedule, acquisition and transfer of
immovable property in India by a person resident outside India is permissible. Hence, the
person resident of Canada can acquire the immovable property in Mumbai.
3. (i) Adherence to sanctioned plans and project specifications by the promoter (Section 14)
The proposed project shall be developed and completed by the promoter in accordance with
the sanctioned plans, layout plans and specifications as approved by the competent
authorities.
Notwithstanding anything contained in any law, contract or agreement, after the sanctioned
plans, layout plans and specifications and the nature of the fixtures, fittings, amenities and
common areas, of the apartment, plot or building, as the case may be, as approved by the
competent authority, are disclosed or furnished to the person who agree to take one or more
of the said apartment, plot or building, as the case may be, the promoter shall not make —
(1) any additions and alterations in the sanctioned plans, layout plans and specifications
and the nature of fixtures, fittings and amenities described therein in respect of the
apartment, plot or building, as the case may be, which are agreed to be taken, without
the previous consent of that person.
Provided that the promoter may make such minor additions or alterations as may be
required by the allottee, or such minor changes or alterations as may be necessary due
to architectural and structural reasons duly recommended and verified by an authorised
Architect or Engineer after proper declaration and intimation to the allottee.
Explanation.—For the purpose of this clause, "minor additions or alterations" excludes
structural change including an addition to the area or change in height, or the removal
of part of a building, or any change to the structure, such as the construction or removal
or cutting into of any wall or a part of a wall, partition, column, beam, joist, floor including
a mezzanine floor or other support, or a change to or closing of any required means of
access ingress or egress or a change to the fixtures or equipment, etc.
(2) any other alterations or additions in the sanctioned plans, layout plans and specifications
of the buildings or the common areas within the project without the previous written
consent of at least two-thirds of the allottees, other than the promoter, who have agreed
to take apartments in such building.
Explanation.—For the purpose of this clause, the allottees, irrespective of the number of
apartments or plots, as the case may be, booked by him or booked in the name of his
family, or in the case of other persons such as companies or firms or any association of
individuals, etc., by whatever name called, booked in its name or booked in the name of
its associated entities or related enterprises, shall be considered as one allottee only.
In case any structural defect or any other defect in workmanship, quality or provision of
services or any other obligations of the promoter as per the agreement for sale relating to
such development is brought to the notice of the promoter within a period of five years by the
allottee from the date of handing over possession, it shall be the duty of the promoter to rectify
such defects without further charge, within thirty days, and in the event of promoter's failure
to rectify such defects within such time, the aggrieved allottees shall be entitled to re ceive
appropriate compensation in the manner as provided under this Act.

© The Institute of Chartered Accountants of India


Page 292
Hence, in the instant case, reducing the number of rooms does not come under minor
additions or alterations. The promoter i.e. Mr. Pankaj Gupta shall not make any additions and
alterations in the sanctioned plans, layout plans and specifications within the project without
the previous written consent of at least two-thirds of the allottees, other than the promoter,
who have agreed to take apartments in such buildings.
(ii) No deposit or advance to be taken by promoter without first entering into agreement
for sale
According to section 13 of the said Act, a promoter shall not accept a sum more than ten per
cent of the cost of the apartment, plot, or building as the case may be, as an advance payment
or an application fee, from a person without first entering into a written agreement for sale
with such person and register the said agreement for sale, under any law for the time being
in force.
In the instant case, the cost of the flat is Rs. 2 crore and Mr. Shyam put a condition to pay
Rs. 50 Lakhs as an application fee before entering into a written agreement for sale with Mr.
Vivaan. This is invalid as a promoter can accept only Rs.20 Lakhs (10% of Rs. 2 Crore) as an
advance or an application fee without first entering into a written agreement for sale.

© The Institute of Chartered Accountants of India


Page 293
ELECTIVE PAPER 6D: ECONOMIC LAWS

Case Study 6
Ronit Chawla was a Fellow Chartered Accountant (FCA) practicing in the field of corporate and economic laws. He
represented his clients before Company Law Board (CLB) and thereafter in National Company Law Tribunal
(NCLT). After coming into force of Insolvency and Bankruptcy Code, 2016 w.e.f. 28 May, 2016, he learnt about the
Limited Insolvency Examination (LIE) for becoming Insolvency Professional (IP). Since he had about eleven years
of experience as practicing CA, he attempted the very first examination of LIE conducted by Insolvency and
Bankruptcy Board of India (IBBI) in December 2016 and successfully cleared it. He then enrolled himself with a
reputed Insolvency Professional Agency (IPA) and got registered with IBBI by fulfilling the requisite formalities
including payment of non-refundable application fee of Rs. ten thousand.
His father Roopesh Chawla, a resident of Green Park, New Delhi, was recently posted as Chief Manager in Bank
of India, Delhi which was a full-fledged Foreign Exchange (FX) branch though Roopesh, being unable to get a
chance to work in a FX branch, had very little knowledge of rules relating to Foreign Exchange. Therefore, he used
to consult his son Ronit in the matters of foreign exchange from time to time. His mother Rukmani Chawla was a
senior teacher in Kendriya Vidyalaya, New Delhi, taking commerce classes.
Rajnish Sinha, a close friend of Roopesh, was heading a Delhi branch of Punjab National Bank (PNB) and knew
that Roopesh’s son Ronit besides being a Chartered Accountant was also an Insolvency Professional. Rajnish, on
behalf of PNB, wanted to initiate corporate insolvency resolution process (CIRP) before NCLT in the case of its
customer Manohar Masale Pvt. Ltd. (MMPL) of Delhi which had defaulted in repaying the dues of the bank totaling
approximately Rs. 23.00 lacs. Accordingly, PNB being financial creditor, while making an insolvency resolution
application to NCLT proposed the name of Ronit as Interim Resolution Professional (IRP). MMPL was sanctioned
cash credit limit of Rs. 10.00 lacs against hypothecation of stock of raw material and finished goods and another
bill discounting limit of Rs. 5.00 lacs against actionable claims. MMPL was registered with an authorised capital of
Rs. 25.00 lacs but its paid up capital was to the tune of Rs. 10.00 lacs.
Initially started as a registered partnership concern (Manohar Masale & Co.) by two brothers, namely, Ram
Manohar and Shyam Manohar, it did profitable business and keeping an eye on future business growth, it was
converted into a private limited company with Ram, Shyam and Shyam’s elder son Shivam as directors. Shyam’s
younger son Dwapam, an alumnus of IIFT, Delhi and also a law graduate, did not have any interest in the family
business and was more inclined to continue with his current employment in a German MNC having its office in
Gurugram.
MMPL’s factory in Okhla Industrial Area was located on the one-fourth portion of the plot which was co-owned by
the brothers. However, the bank had created an equitable mortgage on the plot as well as factory building while
sanctioning the working capital limits to the company. The elder brother Ram Manohar was the anchoring person
who steered the company to newer heights due to his sheer business acumen and inherent managerial skills but
one day, all of a sudden, he had a massive heart attack resulting in his untimely death. Since he was not married,
the business of ‘masale making’ was now run by Shyam and his son Shivam.
However, the father-son duo could not manage the business properly because of the lack of foresight, faulty inter-
personal relations and poor organisational skills. Their authoritative style of leadership resulted in demotivation of
workers which led to labour unrest and all sort of other conflicts. The paternalistic approach towards them which
Ram always displayed was missing altogether. Needless to say, the output started declining and wastage of raw
material turned north. Since there was no vigourous follow-up as well, the debtors to the tune of around Rs. 12.00
lacs were long overdue. Consequently, the company started suffering losses and also defaulted on dues from the
bank.

© The Institute of Chartered Accountants of India


Page 294
When PNB, even after repeated reminders to MMPL, could not realise its dues and the liability touched the height
of around Rs. 23.00 lacs (including normal and overdue interest), Rajnish Sinha, on behalf of PNB, decided to file
corporate insolvency resolution application duly supported by ledger extracts and other specified evidences
(services of Information Utility could not be used as by the time application was filed there was no IU registered
with IBBI) with Adjudicating Authority i.e. NCLT, New Delhi for initiating CIRP against MMPL.
NCLT considered the corporate insolvency resolution application along with the proposed name of Ronit as Interim
Resolution Professional (IRP). Within next 10 days of receipt of application (which was lesser than the statutory
period of 14 days) NCLT ascertained that there existed default because the defaulted amount was much more
than the minimum required of Rs. one lac. Since the CIRP application was complete in all respects, NCLT admitted
it and within the statutory period of next seven days after admission, it conveyed its order of commencement of
CIRP to the financial creditor (i.e. PNB) and the corporate debtor (i.e. MMPL).
The order of NCLT confirmed the proposed appointment of Ronit as IRP for 30 days, for Ronit had a clean record
without any disciplinary proceedings pending against him. It was also stated in the order that a moratorium period
of 180 days had become applicable during which all suits and legal proceedings, etc. against MMPL (i.e. corporate
debtor) were to be held in abeyance so as to give time to the ailing company to resolve its status. MMPL was also
barred from transferring or disposing of any of its assets or any legal rights therein. However, the supply of specified
essential goods and services to the MMPL as mentioned in the order, were not to be interrupted during moratorium
period.
In the meantime, Ronit’s father Roopesh faced a peculiar problem related to the foreign exchange matter at his
branch. His FX officer brought to his knowledge that one of their exporter customers who had received an advance
of USD 75,000 from an importer based at California, USA against export of ready-made jeans had not shipped the
requisite items worth USD 2,00,000 by utilizing the advance so received. The exporter, not willing to ship the goods,
wanted to refund the advance to the importer along with interest for which permission of Roopesh was required.
Roopesh did not allow the refund immediately and in turn, advised the FX officer to gather more knowledge about
FX provisions whether refund along with interest was permissible. At the same time he also discussed the matter
with his son Ronit who advised him to refer FEM (Export of Goods and Services) Regulations, 2015. A scrutiny of
the relevant banking records revealed that 14 months had already expired since advance of USD 75,000 was
received. Further, he came to know that if goods were not shipped within one year of receipt of advance, such
advance could not be refunded without the permission of the RBI. Accordingly, he advised the customer to seek
permission of RBI through his branch.
After his appointment as IRP, Ronit assumed full control of the affairs of MMPL. Since powers of the board of
directors stood suspended he was empowered to exercise such powers. Accordingly, he took immediate custody
and control of all the assets of the MMPL including its business records.
Following the orders of NCLT, Ronit took steps to make a public announcement within three days from the date of
his appointment regarding the initiation of CIRP against MMPL.
Public announcement, included the following aspects:
• Name and address of the corporate debtor (i.e. MMPL) and its registration/incorporating authority.
• His details as IRP and the fact that he would be vested with the management of the corporate debtor and
be responsible for receiving claims.
• Penalties for false or misleading claims.
• The last date for the submission of the claims.
• The date on which the CIRP would end.

© The Institute of Chartered Accountants of India


Page 295
After the expiry of last date for submission of claims, a Committee of Creditors was constituted which included
PNB and five trade creditors who had cumulative dues of Rs. 3.00 lacs. Within seven days of its constitution, the
first meeting of the committee was called. In the meantime, Ronit electronically submitted an Information
Memorandum to the creditors after they had given an undertaking regarding maintaining of confidentiality. This
Information Memorandum contained details of assets and liabilities of the MMPL with their estimated values,
audited financial statements for the last two financial years and provisional financial statements for the current
financial year made just eight days earlier from the date of the application, a list of creditors and the amounts
claimed by them which were duly admitted and other prescribed information.
In the meeting of the Committee of Creditors it was resolved to let Ronit continue as full-fledged Resolution
Professional (RP) since he was eligible to be appointed as an independent director and was not a related party of
the MMPL and such decision was conveyed to the NCLT as well as MMPL. As RP, Ronit assumed all those powers
which were conferred on him as IRP. He was required to manage the operations of the MMPL during the CIRP
period.
Based on the Information Memorandum, Rajnish on behalf of PNB as resolution applicant undertook to prepare a
resolution plan as per the provisions of the Code for onward submission to Ronit. Before finalizing the resolution
plan, he along with his two officers took up the matter with Shyam and his son Shivam regarding the revival of
MMPL and repayment of long outstanding dues or face liquidation if they were not inclined to revive the company.
The fear psychosis of liquidation made them think frantically to save their company from imminent death. Having
woken up from their slumber they started exploring ways to bring in short term finance and also to rope in some
professional who would help the company in its revival.
Shyam saw a ray of hope in his younger son Dwapam and persuaded him to participate in the management of the
affairs of the company at least for the first three months to which he ultimately agreed. In the meantime Shyam,
with a view to raise short term finance, consulted his elder sister Rama Devi to lend at least Rs. 5.00 lacs for a
short period of about one year and also convinced his daughter Ria, her husband Dushyant as well as Dwapam to
invest at least Rs. 3.00 lacs each in the share capital of the company. Shivam who had invested funds in the share
market agreed to sell his securities to raise Rs. 3.00 lacs against which he was to be allotted shares in the MMPL.
As per the advice of the bankers, Shyam also started inter-acting with long overdue debtors for recovery who
eventually agreed to pay 50% of Rs. 12.00 lacs in the current month and remaining amount in the next month. Out
of the raised amount, the operational creditors were to be paid fully while dues of PNB were to be satisfied to the
extent of Rs. 12.00 lacs. Further, Rs. 2.50 lacs were to be allocated towards insolvency resolution process costs
including fee of RP and remaining amount was to be utilized as working capital. Since both the directors of MMPL
had consented to repay Rs. 12.00 lacs in one lump sum, Rajnish on behalf of PNB assured them that he would
take up the matter of waiving of overdue interest up to Rs. 2.00 lacs with his Dy. General Manager and would also
seek permission to revive MMPL’s limits which were currently frozen.
Based on the experience he gathered while working with two MNCs, Dwapam assumed the role of a leader to set
the company on rails. He took note of the prevailing situation from which the ailing MMPL was passing through.
He observed that the current as well as liquid ratios were much far away from the standard norms of 2:1 and 1:1
respectively. The turnover ratios were also unhealthy and at the same time the operating ratio was very high - not
a good sign for any business. An investment of about Rs. 5.00 lacs was tied up in raw material like whole red
chillies, coriander seeds, turmeric, black pepper, dry mango, etc.
Since currently the business of spices was run in a traditional manner, Dwapam decided to take the following short,
medium and long term measures:
Short term measures:
• to understand the needs and wants of customers in the target market;

© The Institute of Chartered Accountants of India


Page 296
• to apply the principles of scientific management;
• to set standards for raw material, wastage, working conditions, etc.;
• to conduct time and motion studies;
• to provide financial incentives and to adopt social security plans for the workers;
• to secure registration with FSSAI immediately;
• to appoint an Administrative Officer and, if need be, to appoint another one in future;
• to devise competitive pricing strategy;
• to create a corporate brand identity by assigning the product a brand name ‘Manohar Uttam Masale’ which
would help in building a brand image;
• to design an attractive package and label by using a graphic design of spices combining green, yellow
and red colours for different varieties of masale;
• to promote the masale by advertising initially in leading newspapers and depending upon income
generation in future, to advertise on FM radio, TV as well as cinema halls;
• to adopt sales promotion measures like free gift offers, contests, free sample distribution, etc.
• to select the similar channels of distribution as used by the competitors;
• to conduct SWOT analysis of MMPL and important competitors;
• to create an effective Website of the company;
• to take decisions regarding various activities under physical distribution of masale like order processing,
transportation, warehousing and inventory control;
• to adopt strict credit policy by reducing debtors’ days with a regular follow-up;
• to use an accounting software;
• to submit various Government Returns within the prescribed time limits so that avoidable hefty penalties
are not levied.
Medium and Long Term Measures:
• to stop heavy expenditure on repairs and maintenance by installing new machines and grinders;
• to establish direct contacts with the cultivators for obtaining raw material which would help in avoiding
middlemen and their high commissions;
• to develop the remaining three-fourth portion of the plot and rent out some of the developed portion to a
commercial establishment;
• to renovate the factory building.
• to manufacture more types of different spices like Rajma Masala, Pindi Chana Masala, Shahi Paneer
Masala, Dal Makhni Masala, Mushroom Matar Masala, etc;
• to diversify MMPL’s operations by manufacturing Jams and Ketchups;
• To explore offshore markets.
Rajnish prepared a resolution plan containing the above strategies and submitted it to Ronit for his consideration.
Later on, a meeting of committee of creditors was called by Ronit and the resolution plan was presented for its
approval. The plan was duly approved by full majority. Thereafter, Ronit submitted the approved resolution plan to
the NCLT for its approval.
Since the resolution plan was approved by the committee of creditors much before the statutory period of 180 days
and also met the prescribed requirements, NCLT approved it and passed an order to this effect. Now the plan was
binding on the MMPL and its employees, members, PNB and operational creditors as well as other stakeholders
involved in the resolution plan.

© The Institute of Chartered Accountants of India


Page 297
I. Objective Type Questions (2 marks each)
Select the most appropriate answer from the options given for each question:
1. “Default” under the IBC is said to be occurred on the fulfillment of condition/s-
(a) Debts becoming due and payable
(b) Non- payment of the debt
(c) Liability /obligation in respect of a claim which is due
(d) Both (a) & (b)
2. In the case study PNB initiated Corporate Insolvency Resolution Process against MMPL for the default in the
capacity of-
(a) Corporate debtor
(b) Operational debtor
(c) Financial creditor
(d) Resolution applicant
3. If the goods against which an advance payment is received from a foreign buyer are not shipped within one
year and there exists no agreement regarding timing of shipment, the advance payment:
(a) shall be refunded within reasonable time without prior approval of Reserve Bank.
(b) Shall be refunded within one year from the date of receipt of advance payment without the prior approval
of Reserve Bank
(c) Shall be refunded within one year from the date of receipt of advance payment with the prior approval of
Reserve Bank
(d) Shall be refunded after one year from the date of receipt of advance payment on the basis of reasonable
cause.
4. PNB through an assignment agreement, assigned here the debt to the X trust. X trust filed the petition for
initiation of corporate Insolvency resolution process (CIRP) against MMPL. State the correct statement with
respect to the competency of the X trust in the filing of the petition in the above situation-
(a) X Trust is not a competent applicant as per section 6 of the IBC
(b) X Trust is being authorized by the PNB to file an application
(c) X Trust in the capacity of financial creditor can file a valid petition.
(d) None of the above
5. As per the Insolvency & Bankruptcy Code, 2016, resolution plan is prepared by ----------- is submitted to ------
----------- for examination and submission to ------------- for approval.
(a) Committee of Creditors, Adjudicating Authority, Resolution Professional
(b) Resolution applicant, committee of creditors, Adjudicating Authority
(c) Resolution applicant, Resolution Professional, Committee of Creditors
(d) Committee of Creditors, Resolution Professional, Adjudicating Authority
6. The maximum duration during which the appointment of Interim Resolution Professional (IRP) is valid shall
not exceed -------- days.
(a) 10
(b) 20
(c) 30
5

© The Institute of Chartered Accountants of India


Page 298
(d) 40
7. In the case study, the expenses of public announcement shall be borne by the-
(a) MMPL
(b) Ronit
(c) Roopesh
(d) PNB
8. In the case study, committee of creditors of MMPL was constituted on 17.3.2018. Time limit, within which the
first meeting of committee of creditors should be held, is ----------------.
(a) 20.3.2018
(b) 22.3.2018
(c) 24.3.2018
(d) 31.3.2018
9. Ronit, being an Insolvency Professional can be appointed as Resolution Professional, if:
(a) he is eligible to be appointed as an independent director under section 149 of the Companies Act, 2013
(b) he is not a related party of the corporate debtor
(c) only (a)
(d) Both (a) and (b)
10. MMPL finds material irregularity in exercise of the powers of the Ronit during the corporate insolvency
resolution period. Remedy available to MMPL-
(a) File a complaint to the adjudicating authority
(b) Complain to the committee of creditor’s
(c) Complaint filed before the IBBI
(d) File an appeal against the order of adjudicating authority against the approval of resolution plan.
II. Descriptive Questions (10 marks each)
1. Suppose the resolution plan prepared by Rajneesh was delayed in approval by committee of creditors. Ronit,
further presented the said resolution plan, before NCLT after 180 days of insolvency commencement date.
Answer the following-
(i) What step shall be taken by NCLT on such presented resolution plan.
(ii) What, if MMPL contravened the resolution plan which effected its employees and stake holders.
(iii) What consequences be there where liquidator continued the business of MMPL during liquidation
process.
2. Ronit in an examination of sale of property of MMPL finds that a transaction was made by the MMPL to Rama
devi (the elder sister of Shyam) in 6 months preceding the Insolvency Commencement date, was undervalued.
Give the following answers in reference to the above situation-
(i) State the validity of the conduct of such transaction by MMPL to Ramadevi.
(ii) What will be the consequences when resolution professional determines such transactions undervalue
and fails to report that same to NCLT?
(iii) What order NCLT shall pass when MMPL entered into an undervalued transaction?

© The Institute of Chartered Accountants of India


Page 299
3. (a) Discuss the legal position and liability of Mr. Shyam in the following given situations-
(i) Where Mr. Shyam fraudulently transferred his holding of shares in favour of his sister of Rs.1 lakh
within 1 year immediately preceding the insolvency commencement date.
(ii) Mr. Shyam makes false entry in the books of account of MMPL to defraud creditors on insolvency
commencement date.
(iii) Shyam permitted Shivam to provide information for initiation of CIRP which is false in material
particular and omits material fact related to a books of accounts of a specified period in the
application.
(b) What course of action can be taken by NCLT against the directors of the MMPL for transactions
defrauding creditors?

© The Institute of Chartered Accountants of India


Page 300
ELECTIVE PAPER 6D: ECONOMIC LAWS

SOLUTION

Case Study 6

I. ANSWERS TO OBJECTIVE TYPE QUESTIONS


1. (d) [Hints: As per section 3(12), Default means non-payment of debt when whole or any part
or instalment of the amount of debt has become due and payable and is not repaid by the
debtor or the corporate debtor, as the case may be]
2. (c) [Hints: Financial creditor means any person to whom a financial debt is owed and includes
a person to whom such debt has been legally assigned or transferred to;{section 5(7)}]
3. (b) [Hint: Refer Regulation 15 of the FEM (Export of Goods and Services), Regulations, 2000]
4. (c) [Hint: Refer Section 5 (7) of the Code]
5. (c) [Hint: Refer Section 5 (25) read with section 28 of the Code]
6. (c) [Hint: Refer Section 16 of the Code]
7. (d) [Hint: Refer Section 15 of the Code]
8. (c) [Hint: Refer Section 22 (1)]
9. (d) [Hint: Refer Regulation 3 of the Insolvency and Bankruptcy (Insolvency Resolution
Process for Corporate Persons) Regulations, 2016]
10. (d) [Hint: section 61(3) of the IBC]

II ANSWERS TO OBJECTIVE TYPE QUESTIONS

1. (i) According to section 33 of the Insolvency and Bankruptcy Code, 2016, where the Adjudicating
Authority before the expiry of the insolvency resolution process period does not receive a
resolution plan as approved by the committee of creditors, it shall—
(a) pass an order requiring the corporate debtor to be liquidated as per the relevant
provisions
(b) issue a public announcement stating that the corporate debtor is in liquidation; and
(c ) require such order to be sent to the authority with which the corporate debtor is
registered.
According to section 12 of the Insolvency and Bankruptcy Code, 2016, the corporate
insolvency resolution process (CIRP) shall be completed within a period of one hundred and
eighty days from the date of admission of the application to initiate such process.
As per the facts, Ronit, presented the approved resolution plan, before NCLT after the
prescribed period for the completion of CIRP i..e, after 180 days of insolvency commencement
date.
According to the above stated provisions, NCLT, shall pass an order requiring the corporate
debtor (MMPL) to be liquidated. It shall issue a public announcement of its liquidation and
send such order to the Registrar of companies.
(ii) As per Section 33(3) of the Insolvency and Bankruptcy Code, 2016, where the resolution plan
approved by the Adjudicating Authority is contravened by the concerned corporate debtor,
any person other than the corporate debtor, whose interests are prejudicially affected by such
contravention, may make an application to the Adjudicating Authority for a liquidation order

© The Institute of Chartered Accountants of India


Page 301
as referred above. Accordingly, the employees and the stakeholders of MMPL, whose
interests are affected by contravention in compliances of the resolution plan, may make an
application to NCLT for initiation of liquidation. On receipt of an application, if the Adjudicating
Authority determines that the MMPL has contravened the provisions of the resolution plan, it
shall pass a liquidation order.
(iii) As per section 33(7) of the Insolvency and Bankruptcy Code, 2016, the order for liquidation
shall be deemed to be a notice of discharge to the officers, employees and workmen of the
corporate debtor.
However, where the business of the corporate debtor when continued during the liquidation
process by the liquidator, it shall not be deemed to be notice of discharge to the officers,
employees and workmen of the corporate debtor.
So the Conduct of business of MMPL during liquidation process by the liquidator is tenable
and shall not be deemed to be notice of discharge to the officers, employees and workmen of
the MMPL.
2. (i) Validity of the conduct of undervalued transaction : As per the provisions given in section
45 of the Insolvency and Bankruptcy Code, 2016, Ronit, on an examination of the
transactions of the MMPL, determines that certain transactions were made by MMPL with a
related party (Rama devi) within the period of two years preceding the insolvency
commencement date (in 6 months preceding the Insolvency Commencement date), which
were undervalued. Ronit, shall make an application to the NCLT to declare such transactions
as void and reverse the effect of such undervalued transaction and requiring the person who
benefits from such transaction to pay back any gains he may have made as a result of such
transaction.
(ii) Failure to report to NCLT of undervalued transactions: As per the stated facts given in
the light of the provisions laid in Section 47 of the Insolvency and Bankruptcy Code, an
undervalued transaction has taken place and Ronit (Resolution Professional) has not reported
it to the NCLT, in such case, a creditor , member or a partner of a MMPL, as the case may
be, may make an application to the NCLT to declare such transactions void and reverse their
effect in accordance with the relevant provisions of this Code.
(iii) Order of NCLT: Where the NCLT, after examination of the application made above, is
satisfied that undervalued transactions had occurred; and Ronit (RP) after having sufficient
information or opportunity to avail information of such transactions did not report such
transaction, there it shall pass an order of —
(a) restoring the position as it existed before such transactions and reversing the effects
thereof in the manner as laid down in section 45 and section 48 of the Code. The order
of the Adjudicating Authority may provide for the following:—
(1) require any property transferred as part of the transaction, to be vested in the
corporate debtor(MMPL);
(2) release or discharge (in whole or in part) any security interest granted by the
corporate debtor (MMPL);
(3) require any person to pay such sums, in respect of benefits received by such
person, to the Ronit (RP), as the Adjudicating Authority may direct; or
(4) require the payment of such consideration for the transaction as may be determined
by an independent expert.
(b) requiring the Board(IBBI) to initiate disciplinary proceedings against Ronit.
3. (a) (i) As per the provisions given in section 68 of the Code, Mr. Shyam, Director (an officer in
default) has within the twelve months immediately preceding the insolvency
commencement date, fraudulently transferred his holding of shares in favour of his sister

© The Institute of Chartered Accountants of India


Page 302
of Rs.1 lakh (which is more than value of ten thousand rupees ). So, Mr. shyam, shall
be punishable with imprisonment for a term which shall not be less than three years but
which may extend to five years, or with fine, which shall not be less than one lakh rupees,
but may extend to one crore rupees, or with both: However, he shall not be liable to any
punishment under this section if he proves that he had no intent to defraud or to conceal
the state of affairs of the corporate debtor.
(ii) According to section 71 of the Code, on and after the insolvency commencement date,
Mr. Shyam, makes a false entry in the books of account of MMPL with an intent to
defraud or deceive any person, he shall be punishable with imprisonment for a term
which shall not be less than three years, but which may extend to five years, or with fine
which shall not be less than one lakh rupees, but may extend to one crore rupees, or
with both.
(iii) As per Section 77 of the Code, as Shyam permitted Shivam to provide informations in
the application under section 10,which is false in material particular and omits material
fact related to a books of accounts of a specified period, so he shall be punishable
with imprisonment for a term which shall not be less than three years, but which
may extend to five years or with fine which shall not be less than one lakh rupees,
but which may extend to one crore rupees, or with both.
(b) As per section 69 of the Code, on or after the insolvency commencement date, where the
directors of the MMPL—
(a) has made transfer of, or charge on, or has caused or connived in the execution of a
decree or order against, the property of the corporate debtor;
(b) has concealed or removed any part of the property of the corporate debtor within two
months before the date of any unsatisfied judgment, decree or order for payment of
money obtained against the corporate debtor,
such directors of MMPL, shall be punishable with imprisonment for a term which shall not be
less than one year, but which may extend to five years, or with fine, which shall not be less
than one lakh rupees, but may extend to one crore rupees, or with both.
However, directors of MMPL, shall not be punishable under this section if the acts mentioned
in clause (a) were committed more than five years before the insolvency commencement date;
or if he proves that, at the time of commission of those acts, he had no intent to defraud the
creditors of the corporate debtor.

© The Institute of Chartered Accountants of India


Page 303
ELECTIVE PAPER 6D: ECONOMIC LAWS

Case Study 7

During March 2017, XMC Pvt. Ltd., a car manufacturing company, launched its TXI model of car with a lot of
advertisements and promotions in all types of media platforms, inter alia, highlighting the Ex-showroom price
of the said car model in Mumbai as Rs. 6.25 lacs.
Mr. Nazir, a prospective buyer of the said model, visited an authorised dealer of XMC Pvt. Ltd. i.e. M/s Ratan
Lal & Sons located at Bandra, Mumbai and after due consultation/ discussion with the representatives of M/s
Ratan Lal & Sons, booked a vehicle of the aforesaid model on 11 th May, 2017 on payment of Rs. 100,000/-.
M/s Ratan Lal & Sons in turn provided the money receipt for the aforesaid transaction with serial number
ABC/1010 as well as booking reference number 218/ 2017 to Mr. Nazir. He was assured by the
representatives of M/s Ratan Lal & Sons that the booked vehicle will be delivered within three months from
the date of booking i.e. by 10 th August, 2017. However, the representative of M/s Ratan Lal & Sons have
stated to Mr. Nazir that as per XMC Pvt. Ltd.’s policy, five months’ time is given in writing so as to keep some
buffer for delays which may arise due to unforeseen exigencies or transportation of vehicle or other logistic
problems. Mr. Nazir, inter alia, noted the conditions in the booking document that “the vehicle would be
delivered within six months from the date of booking”. Believing the assurance given by the representative of
M/s Ratan Lal & Son, Mr. Nazir accepted the terms of the booking and thought that he will get the vehicle
within three months from the date of booking as assured by the representatives of M/s Ratan Lal & Son and
in worst scenario he will get delivery of the vehicle within six months from the date of booking as per the
terms and conditions of booking of the vehicle.
However, within three months of booking of the vehicle, M/s Ratan Lal & Son failed to deliver the vehicle to
Mr. Nazir despite repeated request and after 10 th August, 2017, Mr. Nazir contacted the representatives of
M/s Ratan Lal & Sons many times for delivery of the vehicle and they kept on giving assurances that the
delivery of the vehicle will be done within six months from the date of booking as per the conditions of booking.
After five months, on 15 th February 2018, Mr. Nazir written an e-mail to XMC Pvt. Ltd. highlighting the issue
of delay in delivery of the booked vehicle, but did not get any response. Then he wrote an email to the
President of XMC Pvt. Ltd. and got the reply that his grievances will be looked into by the sales team of the
Company and the concerned dealer.
Despite the assurance of the president of XMC Pvt. Ltd., the booked vehicle was not delivered to Mr. Nazir.
Rather, through M/s Ratan Lal & Sons, he was informed that due to delay in production of the said model,
the Company is not able to deliver the same and he was asked to wait for some more time. Subsequently, he
received a letter from XMC Pvt. Ltd wherein, inter alia, it was informed that due to unprecedented number of
bookings for the said model the delivery of the car will be delayed for two months. Through the said letter, it
was also informed that the price of the booked car will be revised and it will be effective from the date of
booking by dealer to the customer.
About the market and the state of competition
As per Mr. Nazir, XMC Pvt. Ltd. is a big player in the car manufacturing market. Its financial strength and
brand name is much more compared to other players in the market. Also, it commands largest market share
in terms of sales and revenue compared to its competitors and in the last financial year XMC Pvt. Ltd. acquired
a loss making car manufacturing company i.e. Trisha Ventures Pvt. Ltd. As per Mr. Nazir, XMC Pvt. Ltd. has
taken recourse to terms and conditions of the booking documents to enforce price hike and also not honouring
the commitment made for the delivery within the given time period despite repeated correspondence. XMC
Pvt. Ltd. and its dealer at Mumbai M/s Ratan Lal & Sons started the gimmick of non-delivery due to production
delay and started informing that there will be higher price of the vehicle. Mr. Nazir alleged that he and other
similarly situated consumers are being not given with delivery of the vehicle in due time and the delay tactics
done by XMC Pvt. Ltd. is to increase the price of the vehicle and to exploit the consumers by not giving the
benefit of initial launch price which is not fair in a competitive market.

© The Institute of Chartered Accountants of India


Page 304
Concerns raised
As per Mr. Nazir, XMC Pvt. Ltd. has abused its powers to fix the price of the vehicle. It has initially priced
attractively and launched with heavy advertisements and promotions to lure the customers and take maximum
bookings by taking interest free amount of Rs. 100,000/- as booking amount. By doing this XMC Pvt. Ltd. has
been able to not only generate huge amounts of cash which is interest free but also create buzz in the market
because of publicity in the media regarding heavy bookings of the said vehicle. It is stated that XMC Pvt. Ltd.
has arbitrarily increased the price of the vehicle to encash on the market demand. Not only that, the Company
has also not passed on the benefit of recent GST reduction on the passenger cars by Government to the
consumers in the said car model. However, it has passed on the benefit of the GST reduction on its other car
models to the customers which are not in such demand. Most of the other car manufacturers have duly passed
on the GST reduction to the customers. As per Mr. Nazir, XMC Pvt. Ltd. has indulged in unfair practices in
connivance with its dealers by manipulating its delivery policy and price policy. After seeing huge response
because of attractive initial offer price, it not only delayed in giving delivery of t he booked car but also
increased the price which is nearly two times of the offer price at the time of booking. It has not honored the
commitment of delivery and price to the buyer who had booked on the very first day and first hour of the
launch. Mr. Nazir stated that it is not just an individual issue but it involves the larger interests of car buyers,
who do not have any recourse to effective mechanism against the abuse of dominant position by such auto
manufacturers for imposing anti-competitive terms on the buyers.
Based on the above submissions Mr. Nazir alleged that the aforesaid conduct of XMC Pvt. Ltd. is not in
tandem with the provisions of the Competition Act, 2002 and it has acted in a manner which can be termed
as anti-competitive.
I. Objective Type Questions (2 marks each)
Select the most appropriate answer from the options given for each question:
1. Which of the following is the appropriate authority for redressal of the grievances of Mr. Nazir?
(a) District Consumer Redressal Forum
(b) Competition Commission of India
(c) Car Manufacturers Association of India
(d) Both (a) and (b)
2. Under which provisions of the Competition Act, 2002, the grievances of Mr. Nazir can be examined?
(a) Prohibition of horizontal anti-competitive agreement under section 3(3) of the Competition Act, 2002
(b) Prohibition of abuse of dominant position under section 4 of the Competition Act, 2002
(c) Prohibition of vertical anti-competitive agreement under section 3(4) of the Competition Act, 2002
(d) Regulation of combination under section 6 of the Competition Act, 2002
3. Mr. Nazir stated that “it is not just an individual issue but it involves the larger interests of car buyers, who do
not have any recourse to effective mechanism against the abuse of dominant position by such auto
manufacturers for imposing anti-competitive terms on the buyers”. What would be his prime intention in stating
so?
(a) The car manufacturer’s conduct towards him is exploitative
(b) The car manufacturer is imposing anti-competitive terms on him.
(c) The conduct of car manufacturer is not conducive to the market as it affects larger consumers’ interest.
(d) All the above
2

© The Institute of Chartered Accountants of India


Page 305
4. Let, Mr. Nazir approached the Competition Commission India for his grievances and you are the person in the
Commission to take a decision in the matter and according to you the matter pertains to abuse of dominance.
What would be your sequence of analysis of the matter?
(a) XMC Pvt. Ltd. is dominant or not
(b) Whether the alleged conduct is abusive under section 4 of the Competition Act, 2002
(c) Whether XMC Pvt. Ltd. falls under the definition of enterprise as defined under the Competition Act,
2002
(d) Define the relevant market where XMC Pvt. Ltd. is operating
5. Let Mr. Nazir approached the Competition Commission India for his grievances and you are the person in the
Commission to take a decision in the matter and according to you the matter pertains to vertical restraint under
section 3(4) of the Competition Act, 2002. What would be your sequence of analysis of the matter?
(a) Whether XMC Pvt. Ltd. and M/s Ratan Lal & Sons have entered into an agreement
(b) Whether XMC Pvt. Ltd. and M/s Ratan Lal & Sons are placed at vertical level.
(c) Whether there is any appreciable adverse effect on competition because of anti-competitive agreement
between XMC Pvt. Ltd. and M/s Ratan Lal & Sons.
(d) Whether XMC Pvt. Ltd. and M/s Ratan Lal & Sons have agreed on some issues which are anti-
competitive in terms of section 3(4) of the Competition Act, 2002.
6. If you think delineation of relevant market is necessary to examine the fact of the case, then what should be
the relevant product market in this case?
(a) Market for passenger car
(b) Market for dealership services for passenger car
(c) Market for motor vehicle
(d) Market for non-commercial passenger car
7. Mr. Nazir submitted that XMC Pvt. Ltd. is a dominant market player in the relevant market, if you agree with
his submission, what would be your reasoning?
(a) Market share of XMC Pvt. Ltd. is largest
(b) Competitors of XMC Pvt. Ltd. have lesser financial strength
(c) XMC Pvt. Ltd. is a known brand
(d) Consumers are dependent on XMC Pvt. Ltd.
8. Given the facts that XMC Pvt. Ltd. and M/s Ratan Lal & Sons, in connivance with each other, have delayed
the delivery of the booked passenger car to Mr. Nazir and revised the price of the said car, it cannot be a case
of cartelization. What would be the possible reason?
(a) The fact does not reveal any exclusive agreement between XMC Pvt. Ltd. and M/s Ratan Lal & Sons.
(b) The fact does not reveal any agreement of XMC Pvt. Ltd. with other car manufacture in fixing the price
(c) The fact does not reveal that M/s Ratan Lal & Sons is involved in price fixation of delay in giving delivery
of the car to Mr. Nazir
(d) None of the above

© The Institute of Chartered Accountants of India


Page 306
9. In case the Competition Commission of India ordered that Mr. Nazir should approach in the appropriate forum,
what would be your reaction?
(a) The Competition Commission of India is rightly ordered so because the allegations of Mr. Nazir do not
raise any competition concerns in any market.
(b) The order of the Competition Commission of India should be challenged in National Company Law
Appellate Tribunal as it failed to address the concerns of Mr. Nazir in terms of the provisions of
Competition Act, 2002.
(c) Since it is grievance of an individual consumer, Consumer Redressal Forum is the appropriate authority
to deal this matter.
(d) None of the above
10. If you think that XMC Pvt. Ltd. has abused its dominant position, then which of the following conduct of XMC
Pvt. Ltd. is abusive in terms of Section 4 of the Competition Act, 2002?
(a) not giving delivery of the booked car within the assured time
(b) The President of XMC Pvt. Ltd. vide its mail to Mr. Nazir informed that the price of the booked vehicle
will revised and it will be applicable on the date of invoice by dealer to the customer
(c) XMC Pvt. Ltd has not passed the benefit of tax deduction to the consumers
(d) None of the above.
II. Descriptive Questions (10 marks each)
1. Do you think that the concerns raised by Mr. Nazir can be examined through the provisions of the
Competition Act, 2002? If yes, explain the steps through which the matter can be examined.
2. Do you think that not giving delivery of the booked car within the assured time without enabling
provisions in the booking form is tantamount to imposition of unfair conditions and revision of price of
the vehicle with effect from the date booking tantamount to imposition of unfair price on Mr. Nazir?
Examine the given situations in terms of the provisions of the Competition Act, 2002.
3. What is relevant market? State the provisions of the Competition Act, 2002 to delineate the relevant
market. Delineate the relevant market in the instant case.

© The Institute of Chartered Accountants of India


Page 307
ELECTIVE PAPER 6D: ECONOMIC LAWS

SOLUTION

Case Study 7

I. ANSWERS TO OBJECTIVE TYPE QUESTIONS


1. (a) [Hint: The matter relates to concerns of an individual consumer regarding non-delivery of
booked vehicle in the given time]
2. (b) [Hint: The allegations essentially relate to abuse of dominance by a car manufacturing
company, directly or through its authorized dealer]
3. (c) [Hint: The tried to portray his issue as an issue of consumer exploitation to draw the attention
of the competition authority]
4. c, d, a, b [Hint: To examine a case under section 4 of the Competition Act, 2002, it is to be seen
first whether the alleged entity is an enterprise or not before defining the relevant market,
assessment of its position of dominance in the relevant market and examination of its conduct]
5. b, a, d, c [Hint: To examine a case under section 3 (4) of the Competition Act, 2002, first it is to be
seen whether the alleged two entities are in a vertical chain and whether they have entered into
any agreement as defined under the Competition Act, 2002. Then it is to seen whether such
agreement is anti-competitive and it has appreciable adverse effect on competition]
6. (a) [Hint: Essentially, the allegations relate to passenger car market]
7. (d) [Hint: All are the factors prescribed under section 19 (6) the Competition Act, 2002 to assess
dominance of an enterprise in a relevant market]
8. (b) [Hint: Cartelization requires agreement amongst players placed at horizontal level]
9. (a) [Hint: No competition concerns raised in the matter as delay in giving delivery to a consumer
or not passing the benefit of tax reduction to consumer or increasing the price cannot said to
be anti-competitive]
10. (d) [ Hint: Refer section 4 of the Competition Act, 2002]
II. ANSWERS TO DESCRIPTIVE QUESTIONS
1. Even though the concerns raised by Mr. Nazir cannot be redressed by the competition authority
as it essentially relates to grievances of an individual consumer of a passenger car manufactured
by XMC Pvt. Ltd, however if the matter is placed before the competition authority it will be examined
in terms of section 4 of the Competition Act, 2002. It is so because the allegations of Mr. Nazir
essentially relate to abuse of dominance by XMC Pvt. Ltd, directly or through its authorized dealer
M/s Ratan Lal & Sons.
To examine the matter under section 4 of the Competition Act, 2002, it is to be seen first whet her
the alleged entity is an enterprise or not before defining the relevant market, assessment of its
position of dominance in the relevant market and examination of its conduct.
Enterprise: Yes, XMC Pvt. Ltd. is an enterprise in terms of Section 2 (h) of the Act.
Relevant Product Market: The market for passenger car [section 2 (t)]
Relevant Geographic Market: whole of India [see section 2 (s)]
Relevant Market: the market for passenger car in India [section 2 (r)]
Assessment of Dominance of XMC Pvt. Ltd.: Appear to be dominant in the market for
passenger car in India as it has highest market share and financial strength besides its brand name
and dependence of the consumer on it.

© The Institute of Chartered Accountants of India


Page 308
Assessment of the alleged conduct of XMC Pvt. Ltd.: Not appear to be abusive. Delay in giving
delivery of a product to a consumer or not passing the benefit of tax reduction to consumer or
increasing the price cannot said to be anti-competitive in terms of section 4 of the Competition Act,
2002.
2. To examine the matter under section 4 of the Competition Act, 2002, it is to be seen first whether
the alleged entity is an enterprise or not before defining the relevant market, assessment of its
position of dominance in the relevant market and examination of its conduct. (all the steps of
answer no. 1 above to be followed)
Delay in giving delivery of a product to a consumer without enabling provisions in the booing
document may be an issue of breach of contract between two parties. It may not be a case of
imposition of unfair condition in term of the provisions of section 4 of the Competition Act, 2002.
Further, increasing price is a commercial decision of an enterprise which is taken considering the
market demand conditions of the product. If market is competitive then excess price, if any, can be
wiped out in the long run, no intervention of the competition authority is required. However, if the
company raised the price after negotiation with the consumer, it can be challenged in other
appropriate forum.
3. As per section 2(r) of the Act, ‘relevant market’ means the market which may be determined by the
Commission with reference to the relevant product market or the relevant geographic market or
with reference to both the markets. Further, the term ‘relevant product market’ has been defined in
section 2(t) of the Act as a market comprising all those products or services which are regarded as
interchangeable or substitutable by the consumer, by reason of characteristics of the products or
services, their prices and intended use. And, the term ‘relevant geographic market’ has been
defined in section 2(s) of the Act to mean a market comprising the area in which the conditions of
competition for supply of goods or provision of services or demand of goods or services are
distinctly homogenous and can be distinguished from the conditions prevailing in the neighbouring
areas.
In order to determine the ‘relevant product market’, the Commission, in terms of the factors
contained in section 19(7) of the Act, is required to have due regard to all or any of the following
factors viz. physical characteristics or end- use of goods, price of goods or service, consumer
preferences, exclusion of in-house production, existence of specialized producers and
classification of industrial products. Similarly in order to determine the ‘relevant geographic market’,
the Commission, in terms of the factors contained in section 19(6) of the Act, is required to have
due regard to all or any of the following factors viz., regulatory trade barriers, local speci fication
requirements, national procurement policies, adequate distribution facilities, transport costs,
language, consumer preferences and need for secure or regular supplies or rapid after - sales
services.
As stated above, as per the provisions of the Competition Act, 2002 the relevant market comprises
of the relevant product market and relevant geographic market. In the instant matter, the relevant
product market may be considered as the ‘market for passenger car’. It may be noted that the
allegations of Mr. Nazir pertains to purchase and after sale service of a passenger car which cannot
be substitutable with other type of vehicle in terms of price, end use, characteristics, etc. The
relevant geographic market in this matter may be considered as ‘India’ because the condition of
competition in passenger car market in India is homogeneous throughout India. A consumer can
buy a passenger car from any part of India with similar competitive condition. Thus, the market for
passenger car in India may be considered as the relevant market in this case.

© The Institute of Chartered Accountants of India


Page 309
Test Series: November, 2021
MOCK TEST PAPER
FINAL (NEW) COURSE: GROUP – II
PAPER-6D: ECONOMIC LAWS

Time Allowed – 4 Hours Maximum Marks – 100

Case Study -1
Jain Bikes Ltd. (JBL) is engaged in the business of manufacturing and selling of motor cycles. Its registered
office and corporate office, both are in Chandigarh. It has captured the market area of almost North India. It
covers the whole Rajasthan, Punjab, Haryana, Himachal Pradesh, Jammu and Kashmir, Uttarakhand, Uttar
Pradesh, Delhi and Chandigarh. JBL has appointed its sole distributors in almost in every district of the
states of North India.
JBL have its own terms and conditions for granting of distributorship. It dictates the following conditions:
 Security Deposits with the company: Rs. 10 lakh.
 Space for showroom (own or rental): 10000 Sq. feet
 Space for workshop: 2400 Sq feet
 Two Trained Engineers (training shall be provided by the JBL at the cost of distributor)
 Monthly sales target of bikes:
 for Tier 1 Cities: 1000 (minimum)
 for Tier 2 Cities: 700 (minimum)
 for Tier 3 Cities: 500 (minimum)
 Each of the distributor shall provide 3 free servicing of the bikes.
 Each distributor shall not sale any of the vehicles of other companies / make.
 Each distributor shall keep in reserve at least 100 bikes in its showroom to meet the sudden rise in the
demand.
 Distributor shall sale the vehicles at the pre-determine price, fixed by the company, neither up, nor down.
Under the JBL group, there are following group companies:
 JBL General Insurance Company Ltd.
 JBL Finance Company Ltd.
 JBL Tubes and Tyres Ltd.
 JBL Marketing Ltd.
The JBL General Insurance Company Ltd. provides the comprehensive insurance on the bikes, purchased
by the customers. JBL Finance Company Ltd. is a NBFC and it provides finance to customers. JBL has
made it mandatory for the customers to avail finance from its NBFC and buy insurance policy only from its
group insurance company.
The company’s bike is a monopoly product. Bike costs around Rs. 75,000/-. It have 175 cc engine and is
rough and tough in running on any road. It gives 50 km average. However, its spare parts are costly and

Page 1 of 14

© The Institute of Chartered Accountants of India


Page 310
parts made by the JBL can only be used in it. The average life of the vehicle is estimated for 3 lakh km run /
10 years.
The company also provide the road side assistance, provided the customer gets the renewal of insurance
only from its group company.
Rohan is one of a distributor of JBL, having its showroom in Jaipur. He is selling the bikes of JBL for the
last 5 years. His yearly turnover of sale of bikes is 50,000. Recently, the JBL put some more stringent
conditions on distributors, who are having their showrooms in the State Capital City. The JBL advised all
such distributors to deposit additional security amount of Rs. 20 lakh. The yearly sales targets were
increased by 10% of the last FY’ sales figures. The company also insisted that each and every customer
should avail the loan facility from the group NBFC company. Without finance, no bike should be sold.
Further buying of the insurance policy from its group company is also mandatory.
The JBL insisted that all walk-in customers in the showroom, must be asked for their name, mobile number,
e-mail id and address and prepare a data bank and send it to the company. The JBL through its group
company, JBL Marketing Ltd. will approach the customers as listed in the data bank and will call on and
often to the prospective clients and to pursue them to buy the company’s product. Each distributor is
supposed to create such data bank of at least 5000 prospective customers in a month.
The JBL also put a condition on the distributors that if any of customer of their showroom, who has availed
loan from its group NBFC, and not paying the EMI should contact the concerned defaulter and will assist in
recovery of the loan. If any such customer’s loan account becomes non-performing, it shall be the
responsibility of the distributor to make good the losses on account of NPA to the JBL Company.
Rohan was very annoyed with the dictatorship of the company. It was not possible for him to make the
recovery from the defaulters and make good the losses to the company. Further, for every month, sending
the data to the company is also difficulty. Many of the people have complained to the distributor that the
marketing and recovery persons on and often and that too on odd times, visits to the residential address of
the defaulting borrowers and harassing them.
Aggrieved with the attitude of JBL, Rohan wanted to file a case against the company alleging the following
points:
 The JBL had already security deposit of Rs. 10 lakh at the time of taking distributorship and again asked to
deposit addition security deposit of Rs. 20 lakh, resulting a total deposit of Rs. 30 lakh on which no interest
is payable by the company.
 The JBL has insisted to assist in the recovery from the defaulters and if the loan with its Group NBFC of
JBL is not liquidated, the distributor shall have to make good the losses.
 The distributors are also under stress to sale the vehicles only through loan and also there is compulsion to
buy insurance from the JBL group company, which, many a times, the customers have objected to it. There
are many customers who do not want to avail the loan, but due to the pre-condition, they are either
reluctant to buy the bike or are forced to purchase the bike on loan
 Providing of the data bank every month to the JBL is a cumbersome task and the company harasses the
persons listed in the data bank, by calling them on and often.
 There is exclusive supply agreement on the part of the distributors. They cannot sale the spare parts and
other peripherals of other’s make.

Page 2 of 14

© The Institute of Chartered Accountants of India


Page 311
Based on the captioned facts, answer the following questions:
Multiple Choice Questions (5 questions of 2 marks each)
1.1 Where Rohan can file case against the JBL:
(a) Consumer Court
(b) National Company Law Tribunal
(c) Competition Commission of India (CCI)
(d) Insolvency and Bankruptcy Board of India (IBBI)
1.2 Which provision of the Competition Act, 2002 shall be applicable in this case:
(a) Anti-competitive agreements – Section 3
(b) Abuse of dominant position- Section 4
(c) Combination-Section 5
(d) Some parts of Section 3 and 4, both
1.3 Selling of bike along with compulsion to buy insurance from the group insurance company and that too
on finance / loan, are an example of:
(a) Refusal to deal
(b) Resale price maintenance
(c) Tie in arrangement
(d) Exclusive distribution agreement
1.4 Normally every company, while giving the distributorship, put some reasonable conditions. Which
among the following may be treated as reasonable conditions:
(a) Customers to purchase bike only through loan from its Group NBFC Company
(b) Buy compulsory insurance from its Group Insurance Company
(c) To sale the bike at predetermined price fixed by the JBL
(d) Create a data bank of 5000 customers on monthly basis and send it to JNL
1.5 The prime responsibility of making recovery of the EMI from the borrowers is of:
(a) The Lender (JBL Finance Company Ltd.)
(b) The Bike manufacturer (Jain Bikes Ltd -JBL)
(c) The Distributor (Rohan)
(d) JBL General Insurance Company Ltd.
Descriptive Questions (15 Marks)
1.6 Any agreement which requires a purchaser of goods, as a condition of such purchase, to purchase
some other goods, normally prevails in the business world. Would you treat this condition as an anti-
competitive agreement? (8 Marks)
1.7 Which agreements entered between the two parties shall be presumed to have an appreciate adverse
effect on competition? (7 Marks)

Page 3 of 14

© The Institute of Chartered Accountants of India


Page 312
Case Study 2
Ms. Drishel Patel is a young dynamic IT professional and currently resides in America. She holds the NRI
status. Ms. Drishel works for Blip LLC, which has a wholly own subsidiary Blip India Private Limited (here-
in-after referred to as Blip). Blip deals in the mobile operating system. Blips’ operating system ‘Diordna’ is
widely popular among the mobile phone manufacturers in India. Blip also offers proprietary applications and
services (Such as Blip Maps, Blip Internet Explorer, and Blip Tube, etc.). Blips Mobile Services (BMS) is a
bundled suite of Blips’ applications and services and such apps and services are not available in isolation.
In trade parlance, the mobile OS is different from OS designed for desktop as they have additional
handheld use features. 80% of mobile phone, which are in use has Diordna as an operating system.
If a mobile manufacturer wants to manufacture a ‘bare” Diordna mobile, it needs to only pass technical tests
and accept the Diordna License Agreement; but in bare Diordna mobile manufacturer are not permitted to
include any of BMS such as Blip Maps, Blip Internet Explorer, Blip Tube. If a manufacturer wants to
manufacture a mobile having Diordna with pre-installed BMS, he has to enter into two additional
agreements with Blip i.e. Mobile Application Distribution Agreement and Anti Fragmentation Agreement.
BMS couldn’t be availed directly by the end-users, in case it is not pre-installed.
Ms. Drishel got married to Mr. Joe Harris around a year back. The marriage took place with a traditional
saptapadi ceremony in the backyard of Harris family’s resident where only close relatives were present.
Marriage was registered six months later due to a widely observed lockdown to prevent the widespread of
COVID-19.
Indian traditions have a deep-rooted impact on William's family because the grandmother of Joe is from
India. Joe’s grandfather is also influenced by Indian culture, hence willing to migrate to India along with
Joe’s grandmother to spend the rest of their life. Considering this in the month of January 2021, Drishel and
Joe acquired a luxurious apartment in joint name in India; so that Joe’s grandparent can stay there
comfortably. Half of the consideration was paid by Ms. Drishel out of the Non-Resident Account maintained
by her, and the remaining half by Joe through proper banking channel, and too in the manner prescribed.
To identify the flat and fulfill the legal requirement for registration of the same Ms. Drishel took the help of
his elder cousin Mr. Arya Patel, who is permanently residing in India.
Mr. Arya along with two of his friends owns a cement manufacturing company in India called ‘Strong
Cement Private Limited’ (SCPL). The SCPL supplies cement to various builders and retail consumers
through a network of stockiest and retailers. An understanding has been reached among the manufacturers
of cement to control the price and supply of cement, but the understanding is not in writing and it is also not
intended to be enforced by legal proceedings.
Rock Solid Private Limited (RSPL) is the substantial supplier of clay, slate, blast furnace slag, silica sand which
are essential raw materials of cement, and a shortage of same observed in the market. Mr. Arya on behalf of
SCPL has executed a supply agreement with RSPL on 20th October 2020 wherein it is provided that RSPL will
not supply these raw materials to any other cement manufacturer, against this the purchase commitment has
been made from SCPL for all their (RSPL) output at price mentioned in such agreement.

Solid Cement Limited (SCL) who is another cement manufacturer is not happy with the RSPL, because
RSPL not supplied the slate and silica power to SCL against the PO (Purchase Order) placed by SCL dated
18th October 2020, hence board of directors of SCL is considering taking legal remedy against RSPL in the
capacity of the consumer. SCL borne loss on account of the stock-out situation emerged from the non-
availability of raw material. It was found that only half of the consideration paid and 30 days credit was
available for making payment of the remaining balance, regarding which payment promise is made by SCL.

Page 4 of 14

© The Institute of Chartered Accountants of India


Page 313
Mr. Alok who is co-owner of Mr. Arya in SCPL conducts the market study and concluded that the RMC
segment has favourable opportunities because currently competition is relatively less in RMC and RMC
based block segments. Moreover, RMC based block has wide acceptance as an economical replacement of
the brick-based structure. Hence SCPL must diversify into the RMC segment. Mr. Arya expresses his
concerns over the availability of funds for the same. Mr. Anil the third member of SCPL, advices both the
co-owners to float capital through the capital market. After numerous rounds of discussions, SCPL decided
to go for public issue and listing of its equity shares, largely for business expansion, initially with setting up
a new large scale RMC plant.
Mrs. Patel, the Mother of Ms. Drishel, who also resides with her daughter and son-in-law in states and
holds NRI status, acquired two immovable properties (one farmhouse for residential purposes and another
an agricultural land, because she studies botany during her master and willing to develop botanical garden
there) in their native place situated near to Rajkot district of Gujarat in India in the year 2020-2021 for total
consideration equivalent to USD 4,70,000. She made payment for the same out of her non-resident
account.
Multiple Choice Questions (5 questions of 2 marks each)
2.1 Whether the understanding reached among the manufacturers of cement be termed as an agreement
(a) No, because it is not in writing
(b) No, because it is also not intended to be enforced by legal proceedings
(c) No, because it is not in writing and also not intended to be enforced by legal proceedings
(d) Yes
2.2 The agreement is executed among SCPL and RSPL on 20 th October 2020, can be categories as
(a) Exclusive supply agreement
(b) Tie-in arrangement
(c) Refuse to deal agreement
(d) None of these
2.3 Can SCL assume the position of the consumer for the purpose of competition laws?
(a) No, because only half of the consideration paid by SCL
(b) No, because SCL is not buying slate and silica sand for personal use or direct resale
(c) No, because only an individual can be a consumer
(d) Yes
2.4 Which of the following statements is correct regarding the acquisition of immovable property in India
by Mrs. Patel?
(a) Mrs. Patel not allowed to acquire any sort of immovable property in India
(b) Mrs. Patel not allowed to acquire farmhouse and agricultural land in India
(c) Mrs. Patel may acquire the farmhouse, but not agricultural land in India
(d) Mrs. Patel may acquire both the farmhouse and agricultural land in India

Page 5 of 14

© The Institute of Chartered Accountants of India


Page 314
2.5 SCPL decided to go for public issue and listing of its equity shares, largely for business expansion,
initially with setting up a new large scale RMC plant. In the context of shares, which one of the
following statements is correct under the Competition Act, 2002?
(a) Shares can’t be considered as “goods” because nothing has to do with manufacturing,
processing, or mining.
(b) Shares shall be considered as “goods” only if fully paid-up.
(c) Shares shall be considered as “goods” after the application made for shares since application
monies are paid for the acquisition of shares.
(d) Shares shall be considered as “goods” after allotment.
Descriptive Questions (15 Marks)
2.6 Decide, whether Blip has dominance and does it abused its dominant position? Support your decision
with legal backing. (7 Marks)
2.7 Can Mr. Joe acquire immovable property in India, independently? Is the acquisition of a flat by Drishel and Joe,
jointly as per the provisions of the Foreign Exchange Management Act and relevant regulations made
thereunder? Can Joe acquire another property which is agricultural land, in joint ownership with
Drishel for investment purposes? (8 Marks - 2+3+3)

Case Study 3
Ronak Builders Pvt. Ltd. (RBL) is engaged in the business of land development, site planning, construction
and selling of residential and commercial complexes. After enactment of the Real Estate (Regulation and
Development) Act, 2016 (RERD), every project launched by the RBL was registered with the Real Estate
Regulatory Authority (RERA). RBL has earned a good reputation in the real estate market and is a trusted
name, as it complies the provisions of REDA.
RBL in January, 2017 planned to launch a new real estate project in Panvel, Navi Mumbai. It got
registration of the project with the RERA. RBL after doing all the legal formalities relating to the construction
of flats prepared some sample flats of 3BHK. The name of this complex was kept as Ronak Heights. It will
be of G+20 story building with 15 sections and will have all the modern facilities like, Jim, Swimming Pool,
Separate Sports area for Children, Senior Citizen’s Lobby, Temple and a Conference Hall. The number of
flats in the complex will be 315.
After preparing sample flats, RBL started doing advertising and marketing of the project. The cost of the
each flat was fixed for Rs. 80 lakh, which was payable as under:
At the time of booking : Rs. 10 lakh
At the time of Registering : Rs. 20 lakh
After completing of 5 th Floor : Rs. 10 lakh
After completing of 8 th Floor : Rs. 10 lakh
After completing of 11 th Floor : Rs. 10 lakh
After completing of 15 th Floor : Rs. 10 lakh
After completing of 20 th Floor : Rs. 10 lakh
Total : Rs. 80 lakh

Page 6 of 14

© The Institute of Chartered Accountants of India


Page 315
The possession of the flat will be given to the allottees in the month of December, 2020. Just after making
the advertisements, all the flats were booked. The booking amount was collected from all the 315 allottees.
The legal documents and stamp papers of each of the flat was prepared and collected Rs 30 lakh each
from the allottees.
Thereafter, the work of construction went on full swing. It completed the construction upto 20th floor and
collected all the remaining amount due i.e. Rs. 50 lakh each from the allottees. The construction of 20 th

floor was completed by the end of June 2019 and the allottees were expecting to get the allotment of the
flat by the due date i.e. December 2020. However, the furnishing and finishing, installation of electric and
sanitary work etc. all of sudden stopped and no construction related activity was going on after September
2019.
Some of the allottees who resides nearby Panvel area used to go at the location, on and often to observe
the progress of the work. When the work stopped they approached to the RBL and enquired about the
development. The RBL ensured that due to upcoming festival time the labours have gone to their native
places and after the end of Diwali, the work will resume again.
However, the work, as promised by the RBL could not be started. Many of the allottees contacted at the
RBL’s office to know the status, but got no satisfactory replies. Ultimately the dead line of giving possession
of the flats, which was pre-determined as December 2020, crossed.
Meanwhile it came to the knowledge of some of the allottees that RBL has transferred this real estate
project to a third party named Ganpati Constructions Ltd (GCL) without obtaining the consent of the
allottees.
This transfer was objected by the allottees and threated to move to RERA and to take other legal recourse,
if the RBL is not taking back the completion of the work. After a prolonged discussions held with the officials
of the RBL and GCL and the representatives of the allottees, the RBL agreed that all the liabilities
pertaining to this real estate project shall be of RBL only and GCL will only do the incomplete work as a
sub-contractor.
Some 100 allottees of the flat, demanded their money back from RBL on account of non-completion of the
project in time. The RBL requested all such allottees to remain in the project, but the allottees were
adamant and were not turning back. As a result, the promoter had to pay off the amount raised from such
allottees along with the interest including the compensation as per the provisions contained in the RERD.
However, the remaining 215 (315-100 = 215) allottees desired to stay with the project till the completion
and possession but demanded from the promoter to compensate them by paying interest since they are
suffering for not providing the flat and had to pay the rent to their present landlord in which they are
residing. RBL agreed to their demand.
The site was completed by the end of April 2021. All the remaining allottees were given the possession of
the flats.
Based on the captioned facts, answer the following questions:
Multiple Choice Questions (5 questions of 2 marks each)
3.1 In the given case, the promoter transferred his rights and liabilities to a third party. Can he do so:
(a) The promoter cannot transfer his rights and liabilities to a third party
(b) The promoter can transfer his rights and liabilities to a third party
(c) The promoter can transfer his rights and liabilities to a third party after obtaining prior written

Page 7 of 14

© The Institute of Chartered Accountants of India


Page 316
consent of at least 10% of allottees
(d) The promoter can transfer his rights and liabilities to a third party after obtaining prior written
consent from two-third allottees and without the prior written approval of the Authority.
3.2 In the given case, RBL obtained the booking amount from the prospective allottees a sum of Rs. 10
lakh. Is it justified:
(a) RBL can receive any amount as a commitment from the prospective allottees towards the
booking of flat
(b) RBL cannot received any amount at the time of booking of the flat
(c) RBL cannot accept a sum more than 10% of the cost of the flat. The cost of flat is Rs 80 lakh, so
he cannot accept more that Rs. 8 lakh
(d) RBL cannot accept a sum more than 20% of the cost of the flat. The cost of flat is Rs 80 lakh, so
he can accept upto Rs. 16 lakh, while he had actually accepted only Rs, 10 lakh which is
permissible
3.3 Were the 215 allottees correct in demanding from the promoter to compensate them to pay interest
since they are suffering for not providing the flat and had to pay the rent to their present landlord in
which they are residing. Choose the correct option as per the provisions of the Real Estate
(Regulation and Development) Act, 2016:
(a) Yes, these allottees shall be given interest for every month of delay, till the handing over of the
possession
(b) These allottees shall not be given any interest
(c) These allottees shall be given the notional rent of the flat till the handing over of the possession
(d) These allottees shall be given special discount at the time giving of possession
3.4 Can the allottees of the flat, file an application for initiation of Corporate Insolvency Resolution
Process (CIRP) under the Insolvency and Bankruptcy Code, 2016 (IBC):
(a) No, the allottees can not file CIRP under IBC
(b) The allottees can file CIRP if the NCLT permits as a special case
(c) The allottees can file CIRP under IBC provided the number of allottees are not less than 100
under the same real estate or 10% of the total number of allottees, whichever is less
(d) The RERD was specifically enacted to regulate the real estate business and to safeguard the
interest of the home buyers, so the allottees can seek remedies by filing an application before the
RERA.
3.5 As per the Insolvency and Bankruptcy Code, 2013, in the given case scenario, the amount that was
raised by the promoter from the allottees for Ronak Heights, shall be deemed to be an amount of the
nature:
(a) Advance received against booking of flat
(b) Commercial effect of borrowing
(c) Consideration for agreement to sale flat
(d) Reservation of flat against money

Page 8 of 14

© The Institute of Chartered Accountants of India


Page 317
Descriptive Questions (15 Marks)
3.6 The Real Estate (Regulation and Development) Act, 2016 (RERD) was enacted to regularise the
unregulated real estate business. Substantiate your answer in light of the given case. (10 Marks)
3.7 Where the possession of the flat is delayed as per the agreed terms, what recourse is available before
the allottees under the IBC and RERD? (5 Marks)
Case Study 4
Alta Modern Builders Ltd (AMBL) is a builder and have built many residential complexes and shopping
malls in Mumbai, Pune, Bangalore and Chennai.
AMBL was established initially as a private sector company in 1980. As the business grew up, it was
converted into Public Ltd. in 2005. It have its registered office and corporate office in Versova, Mumbai.
In the year 2017, it purchased a land near Kurla, Mumbai admeasuring of 50,000 sq feet and developed and took
approval from the competent authority for construction of flats. The AMBL has plan to construct 500 residential
flats in that area consisting of 2 BHK and 3 BHK. The cost of the flat was kept as Rs. 99 lakh.
The AMBL constructed some sample flats and started giving advertisements of the flats. The registration of
this real estate project with RERA was pending and before registration it booked and taken the deposit
amount of Rs. 10 lakh from each of the 300 allottees.
Some of the real estate agents who seen the advertisement of booking of the flats by the AMBL
approached the builder. The negotiations were made between the agents and the AMBL that, AMBL will
pay Rs. 1,00,000 to each of the real estate agent, if he brings in any prospective buyer to book the flat, and
through his efforts the flat is booked.
Ganpati Estate Agent was one of such real estate agent, who brought in 50 prospective home buyers with
the builder and all have booked the flats. Ganpati Estate Agent has not got registration with RERA as an
agent. The AMBL paid Rs. 50 lakh to this agent.
The RERA officials of the Mumbai, Maharashtra came to know that AMBL has received the amount without
getting itself registered with the RERA. The RERA officials imposed penalty on the AMBL. Further during
the course of investigation of books of the AMBL, the RERA official came to know that one Ganpati Estate
Agent, who brought in 50 customers, was also not registered with the RERA. It called on the explanation of
Ganpati Estate Agent and when no satisfactory reply was received, imposed a penalty on the agent, for not
getting registered with the RERA.
The AMBL after paying the penalty immediately registered the project with RERA and created its web-page
on the website of the RERA and put in all the relevant information as mentioned in the RERD. While
displaying such information, the AMBL submitted some false information of its web-page, so that by reading
this, more customers can be attracted. The false information was about the carpet area, free parking for
one car for one flat and providing of the modern amenities in the complex, such as school up to the age of
5th class, sports club house, conference hall etc.
Actually, the AMBL has falsely represented for free parking. It was not free parking of a car but of two
wheeler only. The carpet area as mentioned at the web-page of the RERA, was actually captured by the
surrounding walls. When the complaints were lodged by some of the allottees with RERA that there are
differences between, what has been displayed on the web-page of AMBL at the website of the RERA and
what documents have been provided to allottees. The RERA officials again investigated the matter and
imposed file on the company.

Page 9 of 14

© The Institute of Chartered Accountants of India


Page 318
The AMBL who was doing business of real estate since 1980 did not took seriously even after the
enactment of the RERD. It thought that noting has changed and the terms and conditions of the builder will
prevail as was being done previously. However, after the enactment of the RERD, the Regulator RERA is
there and the promoter have to adhere to the compliances as prescribed under the Act.
Based on the captioned facts, answer the following questions:
Multiple Choice Questions (5 questions of 2 marks each)
4.1 the given case, the promoter has booked some of the flats prior to registration of real estate project
with the Real Estate Regulatory Authority (RERA). What penalty can be imposed on the promoter by
the RERA:
(a) The penalty may extend up to 5% of the estimated cost of the real estate project as determined by the
Authority
(b) The penalty may extend up to 10% of the estimated cost of the real estate project as determined by
the Authority
(c) The penalty may extend up to 12% of the estimated cost of the real estate project as determined by
the Authority
(d) The penalty may extend up to 15% of the estimated cost of the real estate project as determined by
the Authority
4.2 The promoter while submitting the application for registration of real estate project under section 4 of
the RERD submitted false information that he have the legal title to the land, but in fact at the time of
application for registration, he was not having such clear title. What penalty can be imposed on the
promoter:
(a) The promoter shall be liable to a penalty which may extend up to 1% of the estimated cost of the real
estate project, as determined by the Authority.
(b) The promoter shall be liable to a penalty which may extend up to 3% of the estimated cost of the real
estate project, as determined by the Authority
(c) The promoter shall be liable to a penalty which may extend up to 5% of the estimated cost of the real
estate project, as determined by the Authority
(d) The promoter shall be liable to a penalty which may extend up to 7% of the estimated cost of the real
estate project, as determined by the Authority
4.3 What is the time limit within which the promoter shall execute a registered conveyance deed in favour
of the allottee, in the absence of any local law:
(a) Within one month form the date of issue of occupancy certificate
(b) Within two months form the date of issue of occupancy certificate
(c) Within three months form the date of issue of occupancy certificate
(d) Within six months form the date of issue of occupancy certificate
4.4 What is the time limit within which the allottee shall take physical possession of the apartment:
(a) Within a period of one month of the occupancy certificate
(b) Within a period of two months of the occupancy certificate

Page 10 of 14

© The Institute of Chartered Accountants of India


Page 319
(c) Within a period of three months of the occupancy certificate
(d) Within a period of four months of the occupancy certificate

4.5 In the given case, the real estate agent, who was not registered with the RERA shall bear
the penalty of ________________ for every day during which such default continues, which
may extend up to five per cent. of the cost of the apartment:
(a) Rs. 5000/-
(b) Rs. 7000/-
(c) Rs 10000/-
(d) Rs. 15000/-

Descriptive Questions (15 Marks)


4.6 Real Estate Agent is a person who merely assist any person in sale / purchase of flat in any apartment
and also provides ‘To-let’ services to the flat owners / tenants. He have no role in developing of land,
getting approval of the site plan, registering and constructing the apartments. He have no role to play
in the Real Estate (Regulation and Development) Act, 2016. Comment on the statement. (6 Marks)
4.7 The post RERD era have put stringent compliance norms on the part of the promoter to ensure the
provisions of the RERD. State the compliances which are to be adhered by the promoter in terms of
the RERD. (9 Marks)

Case Study 5
Sun Private Limited (SPL) is a fully integrated operation from taking a 3D model as input to the design and
manufacturing of tools to the manufacturing of finished products. The Company is also into Engineering
Services with headquarters in Mumbai, India managed and run mainly by the promoters Mr. S (Managing
Director), Mr. T (Director), and Mr. U (Director). All three are Indian residents.
SPL has a marketing office with warehouse facility Sun Trading Spolka Z.O.O (STS) in Poland, fully owned
and controlled by it, to cater to the demands of European customers. LTS has been established with the
permission of the Reserve Bank of India, duly complying with the required statutory formalities.
On 1st January 2020, SPL, shipped some engineering products with a CIF value of EUR 265,000 to STS;
the cost of the products is EUR 250,000, Insurance EUR 3,000, and Freight EUR 12,000. Also, some of the
products worth CIF GBP 126,000 were shipped to one of the customers in the UK on the same date. The
total value of Exports of SPL during the calendar year 2020 from various customers from different countries
was USD 12 Million.
SPL during the normal course of business also entered into a Supply (Export) Agreement with one of its
customers Royal Group (RG) in the UK for the supply of two machines, a total export value estimated to be
CIF GBP 4 Million.
As per the terms of supply:
a. Two Machines, as specified, worth about CIF GBP 2 Million each are to be exported by SPL to RG.
b. Exact value of each of the Machinery can be ascertained only after the export to the UK since some
more processes are involved during installation and commissioning.
c. An advance of GBP 1 Million is to be remitted to India by RG to SPL for the purchase or import of

Page 11 of 14

© The Institute of Chartered Accountants of India


Page 320
critical components required for the manufacture of the said machines.
d. Interest shall be payable on Advance payment by SPL to RG up to the date of the bill of lading of the
first shipment.
e. The first Machinery is to be supplied within 15 months from the date of receipt of advance payment in
India and the second one within a period not exceeding 27 months.
Accordingly, as per the terms of supply, a sum of GBP 1 Million was received by SPL from RG on 1st July
2021 as an advance towards exports through the State Bank of India. The First machinery was supplied on
time and the relevant export declaration was furnished to the specified authority in a specified manner.
Other export formalities were duly complied with.
SPL also established a marketing office in Dubai, UAE - Sun Emirates LLC (SEL) for conducting normal
business activities of the Indian entity, to cater to the requirements of customers from the Middle East. For
promoting business in the Middle East Region, SPL sponsored a T20 Cricket match in Dubai International
Cricket stadium and approached the State Bank of India for remittance of USD 250,000 towards
sponsorship Fees.
SPL is holding certain properties in the form of some residential flats in UAE ready for sale. Prestige Real
Estate LLC (PREL) is a well-reputed real estate agent in UAE and has experience in marketing, advertising,
and selling real estate property. While on travel to Dubai, Mr. S and T, on behalf of SPL entered into an
Agency Agreement PREL for the sale of properties in UAE. As per the Agreement
a. PL grants PREL the exclusive rights to sell all the residential flats in UAE.
b. Any and all offers and negotiations in regards to the said properties shall be conducted by PREL
c. PREL shall do everything possible to entertain and vet offers made. It is the Agent’s sole purpose to
sell the properties and as so shall be permitted to employ additional Brokers to assist in the selling
and advertising process.
d. Any offers considered valid should be reported to the Seller within 2 days and it shall be at the
discretion of SPL to accept or decline.
e. SPL agreed to remit PREL a flat commission of a certain percentage of the final sale price, on a case-
to-case basis.
PREL is also authorized to sell one of the commercial plots owned by SPL in India on similar terms as
stated above. For one of the plots owned by SPL in Pune, PREL finds a buyer from UAE. Because of the
efforts of PREL, such a plot could be sold at USD 400,000. PREL transferred USD 400,000 to India, as sale
proceeds. As per the Agreement, USD 22,000 is to be transferred as Commission to PREL.
Mr. U wants to remit USD 250,000 under the Liberalized Remittance Scheme (LRS) to buy lottery tickets
abroad making use of his business connections.
Answer the following questions in the light of given information.
Multiple Choice Questions (5 questions of 2 marks each)
5.1 For one of the plots owned by SPL in Pune, PREL find a buyer from UAE. Because of the efforts of
PREL, such a plot could be sold at USD 400,000. PREL transferred USD 400,000 to India, as sale
proceeds. As per the Agreement, USD 22,000 is to be transferred as Commission to PREL. In the
context of commission which of the following statements is correct:
(a) Without any pre-approval from the Reserve Bank of India upto USD 100,000 or 5% of the amount

Page 12 of 14

© The Institute of Chartered Accountants of India


Page 321
remitted, whichever is higher, can be transferred as a commission by SPL to PREL
(b) Without any pre-approval from the Reserve Bank of India any amount upto USD 25,000 or 5% of
the amount remitted, whichever is higher can be transferred as a commission by SPL to PREL
(c) Without any pre-approval from the Reserve Bank of India only USD 20,000 can be transferred as
a commission by SPL to PREL in the given case.
(d) Without any pre-approval from the Reserve Bank of India upto USD 50,000 or 5% of the amount
remitted, whichever is lesser, can be transferred as a commission by SPL to PREL.
5.2 The First machinery was supplied on time and the relevant Export Declaration was furnished to the
specified authority in a specified manner. In the context of the Export Declaration, which one of the
following statements is not correct?
(a) Export of goods can be made without furnishing the specified Declaration when goods are
imported free of cost on a re-export basis;
(b) Export of goods can be made without furnishing the specified Declaration when goods are sent
outside India for testing subject to re-import into India.
(c) Export of goods can be made without furnishing the specified Declaration when defective goods
are sent outside India for repairs at an agreed price with the supplier outside, subject to re-import
into India.
(d) Export of goods can be made without furnishing the specified Declaration in case of
unaccompanied personal effects of travelers.
5.3 STS (Sun Trading Spolka Z.O.O) in Poland in the stated case shall be treated as:
(a) Person resident outside India
(b) Person resident in India
(c) Person not ordinary resident in India
(d) No relevance to LTS of residential status with reference to Indian laws
5.4 For promoting business in the Middle East Region, SPL sponsored a T20 cricket match in Dubai
International Cricket stadium and approached the State Bank of India for remittance of USD 250,000
towards sponsorship Fees.
(a) State Bank of India can remit USD 250,000 towards cricket sponsorship without any limits and
any pre-approval.
(b) State Bank of India can remit USD 250,000 with the approval from Reserve Bank of India.
(c) State Bank of India can remit USD 250,000 with prior approval from the appropriate ministry of
the Government of India.
(d) Remittance by State Bank of India of USD 250,000 towards T20 cricket sponsorship in Dubai is a
transaction for which remittance of foreign exchange is prohibited.
5.5 Mr. U wants to remit USD 250,000 under the Liberalized Remittance Scheme (LRS) to buy lottery
tickets abroad making use of his business connections.
(a) Remittance to buy lottery tickets abroad is a prohibited item under LRS
(b) Remittance of more than USD 100,000 for buy lottery tickets abroad is prohibited under LRS

Page 13 of 14

© The Institute of Chartered Accountants of India


Page 322
(c) Remittance upto USD 250,000 per financial year is permitted to buy lottery tickets abroad under
LRS
(d) Remittance only upto USD 150,000 per financial year is permitted to buy lottery tickets abroad
under LRS
Descriptive questions (15 Marks)
5.6 On 1st January 2020, SPL shipped some engineering products with a CIF value of EUR 265,000 to
STS; the cost of the products is EUR 250,000, Insurance is EUR 3,000, and Freight is EUR 12000. In
this regard answer the following; (9 Marks)
A. What is the period within which the export value of goods shipped to STS to be realized and
repatriated to India, and does it make any difference, if only the cost price is realized but not
Insurance and Freight within the period specified?
B. Will your answer change to part A above, in the case of the transaction wherein goods worth
GBP 126,000 shipped/exported to one of the customers in the UK and not to STS?
C. Will your answer change to part A above, in case SPL has an Export Oriented Unit in Mumbai
and goods/software/services are shipped therefrom? Explain.
5.7 SPL is holding certain properties in the form of some residential flats in the UAE. What are the
possible ways by which these properties might have been legally acquired by SPL in the UAE?
(6 Marks)

Page 14 of 14

© The Institute of Chartered Accountants of India


Page 323
(c) Remittance upto USD 250,000 per financial year is permitted to buy lottery tickets abroad under
LRS
(d) Remittance only upto USD 150,000 per financial year is permitted to buy lottery tickets abroad
under LRS
Descriptive questions (15 Marks)
5.6 On 1st January 2020, SPL shipped some engineering products with a CIF value of EUR 265,000 to
STS; the cost of the products is EUR 250,000, Insurance is EUR 3,000, and Freight is EUR 12000. In
this regard answer the following; (9 Marks)
A. What is the period within which the export value of goods shipped to STS to be realized and
repatriated to India, and does it make any difference, if only the cost price is realized but not
Insurance and Freight within the period specified?
B. Will your answer change to part A above, in the case of the transaction wherein goods worth
GBP 126,000 shipped/exported to one of the customers in the UK and not to STS?
C. Will your answer change to part A above, in case SPL has an Export Oriented Unit in Mumbai
and goods/software/services are shipped therefrom? Explain.
5.7 SPL is holding certain properties in the form of some residential flats in the UAE. What are the
possible ways by which these properties might have been legally acquired by SPL in the UAE?
(6 Marks)

Page 14 of 14

© The Institute of Chartered Accountants of India


Page 324
Test Series: November, 2021
MOCK TEST PAPER
FINAL (NEW) COURSE: GROUP – II
PAPER-6D: ECONOMIC LAWS

SUGGESTED ANSWERS/HINTS

Case Study -1
1.1 (c)
1.2 (d)
1.3 (c)
1.4 (c)
1.5 (a)
1.6 The purpose behind the enactment of the Competition Act, 2002 was to prevent practices having
adverse effect on competition, to promote and sustain competition in markets, to protect the interests
of consumers and to ensure freedom of trade carried on by other participants in markets, in India.
Healthy competition brings many good things. It curtails the monopoly, improving the quality of the
goods and services, competitive prices and continuous innovations to improve the quality.
Anti-competitive agreement
Section 3(1) provides that no enterprise or association of enterprises or person or association of
persons shall enter into any agreement in respect of production, supply, distribution, storage,
acquisition or control of goods or provision of services, which causes or is likely to cause an
appreciable adverse effect on competition within India.
As per Sub-section (2), any agreement entered into in contravention of the provisions contained in
sub-section (1) shall be void.
In the given case, the JBL is almost enjoying the monopolistic conditions. Besides the selling of the
bike, it has put several conditions on the persons who are having its dealership. These conditions
are not reasonable conditions. It prevents the entry of other insurance companies to provide the
insurance coverage on the bikes, by simply putting a condition on the buyers of bike to buy
insurance policy only from its Group Insurance company.
Further the JBL has also prevented the entry of other finance companies / NBFCs/ Banks who can
provide the loan facility to the buyers of the bike. JBL have put conditions that no cash sale of the
bike will be allowed and the buyer have been forced to avail loan from its Group NBFC company.
Section 3(4) provides that any agreement amongst enterprises or persons at different stages or
levels of the production chain in different markets, in respect of production, supply, distribution,
storage, sale or price of, or trade in goods or provision of services, including—
(a) tie-in arrangement;
(b) exclusive supply agreement;
(c) exclusive distribution agreement;

Page 1 of 13

© The Institute of Chartered Accountants of India


Page 325
(d) refusal to deal;
(e) resale price maintenance,
shall be an agreement in contravention of sub-section (1) if such agreement causes or is likely to
cause an appreciable adverse effect on competition in India.
Meaning of Tie-in arrangement:
The explanation (a) to this sub-section defines the meaning of tie-in arrangements. It provides an
inclusive definition (not exclusive). “Tie-in arrangement” includes any agreement requiring a
purchaser of goods, as a condition of such purchase, to purchase some other goods. Such type of
conditions which are not reasonable, can be put in the category of tie-in arrangement.
What conditions are reasonable
The conditions which may be put under the banner of reasonable conditions have been described
under section 3(5) of the Act. It provides that nothing contained in this section shall restrict—
(i) the right of any person to restrain any infringement of, or to impose reasonable conditions, as
may be necessary for protecting any of his rights which have been or may be conferred upon
him under—
(a) the Copyright Act, 1957 (14 of 1957);
(b) the Patents Act, 1970 (39 of 1970);
(c) the Trade and Merchandise Marks Act, 1958 (43 of 1958) or the Trade Marks Act, 1999 (47
of 1999);
(d) the Geographical Indications of Goods (Registration and Protection) Act, 1999 (48 of
1999);
(e) the Designs Act, 2000 (16 of 2000);
(f) the Semi-conductor Integrated Circuits Layout-Design Act, 2000 (37 of 2000).
In the given case, since the JBL has put very unreasonable condition on the distributor, so it
is treated as tie-in arrangements.
1.7 Section 3(3) of the Competition Act, 2002 (the Act) lists out some of the agreements which are
presumed to have an appreciable adverse effect on competition. This section provides that -
Any agreement entered into between enterprises or associations of enterprises or persons or
associations of persons or between any person and enterprise or practice carried on, or decision
taken by, any association of enterprises or association of persons, including cartels, engaged in
identical or similar trade of goods or provision of services, which—
(a) directly or indirectly determines purchase or sale prices;
(b) limits or controls production, supply, markets, technical development, investment or provision of
services;
(c) shares the market or source of production or provision of services by way of allocation of
geographical area of market, or type of goods or services, or number of customers in the market or
any other similar way;
(d) directly or indirectly results in bid rigging or collusive bidding,
shall be presumed to have an appreciable adverse effect on competition.

Page 2 of 13

© The Institute of Chartered Accountants of India


Page 326
It is to be mentioned here the above instances mentioned in the Act have been presumed to have an
appreciable adverse effect on competition and there is no need to prove it.
However, the agreements which have been entered into by way of joint ventures shall not be
presumed to have an appreciable adverse effect on competition, if such agreement increases
efficiency in production, supply, distribution, storage, acquisition or control of goods or provision of
services.
Determination of Appreciable Adverse Effect on Competition
Section 19(3) of the Act provides that the Commission shall, while determining whether an
agreement has an appreciable adverse effect on competition under section 3, have due regard to all
or any of the following factors, namely:—
(a) creation of barriers to new entrants in the market;
(b) driving existing competitors out of the market;
(c) foreclosure of competition by hindering entry into the market;
(d) accrual of benefits to consumers;
(e) improvements in production or distribution of goods or provision of services;
(f) promotion of technical, scientific and economic development by means of production or
(g) distribution of goods or provision of services.
In the given case, the JBL have entered into the agreement with dealers/ distributors that no bike
shall be sold in cash. The Bikes shall be sold only if the customer avails loan from its Group Finance
Company and shall have to buy insurance cover on the bike only from the Group Insurance
Company. These two major conditions restrict the entry of other insurance companies and finance
companies to sale insurance /distribute loan facility particularly to the bike segment manufactured by
the JBL.
Case Study 2
2.1 (d)
2.2 (c)
2.3 (d)
2.4 (b)
2.5 (d)
2.6 Facts in the given case are more or less similar to the case (No. 39 of 2018, Competition
Commission of India dated 16.04.2019) of Umar Javeed and Google LLC, wherein legal issue also
about dominance and its abuse and act of Google found in violation of Section 4(2) of the
Competition Act, 2002.
In the said case, CCI observed to form a prima facie view about the alleged abusive conduct, it
would be first appropriate to define the relevant market and to determine the dominance of accused
enterprise therein if any. In the present case, it is clearly mentioned that mobile OS due to additional
handheld use features are different from OS designed for desktop hence all OS for other devices
such as desktop or laptop shall be excluded from the relevant market. Blip appears to be dominant
in the relevant market as 80% of mobile phones, which are in use have Diordna as the operating

Page 3 of 13

© The Institute of Chartered Accountants of India


Page 327
system.
The signing of the Mobile Application Distribution Agreement and Anti Fragmentation Agreement is
a pre-condition for mobile manufacturers to pre-install BMS (while using Diordna as OS. Further,
BMS is also a bundled suite of Blips’ applications and services. In this manner Blip reduced the
ability of device manufacturers to develop viable alternatives with selected applications and s ervices
out of the BMS suite, hence dis-incentivize them. Thereby restricting technical development to the
prejudice of consumers in violation of Section 4 of the Competition Act, 2002.
While reading Section 4 with Section 32 of the Act, it is important to note that the conduct of Blip to
tie or bundle applications and services is an attempt to eliminate effective competition from the
market. There exists an element of coercion as the mobile manufacturers are coerced to purchase
the BMS suite altogether which results in consumer harm through a reduction in choice of products.
2.7 As per regulation 6 of Foreign Exchange Management (Acquisition and Transfer of Immovable
Property in India) Regulations, 2018, a person resident outside India, not being a Non-Resident
Indian or an Overseas Citizen of India, who is a spouse of a Non-Resident Indian or an
Overseas Citizen of India may acquire one immovable property (other than agricultural land/
farmhouse/ plantation property), jointly with his/ her NRI/ OCI spouse, subject to following
conditions
(i) The consideration for the transfer, shall be made out of funds received in India through
banking channels by way of inward remittance from any place outside India or funds held in
any non-resident account maintained in accordance with the provisions of the Act and the
regulations made by the Reserve Bank;
(ii) No payment for any transfer of immovable property shall be made either by travellers’ cheque
or by foreign currency notes or by any other mode other than those s pecifically permitted
under this clause;
(iii) The marriage has been registered and subsisted for a continuous period of not less
than two years immediately preceding the acquisition of such property;
(iv) The non-resident spouse is not otherwise prohibited from such acquisition.
No, Mr. Joe (a person resident outside India, not being a Non-Resident Indian or an Overseas
Citizen of India) can’t acquire immovable property in India, independently
No, the acquisition of a flat by Drishel and Joe, jointly is not aligned (hence legally invalid, and
amount to violation) to the provisions of FEMA and relevant regulations made thereunder, because
marriage has been registered and subsisted for a continuous period of fewer than two years
immediately preceding the acquisition of such property.
No, Joe can’t acquire another property being agricultural land in joint ownership with Drishel for
investment purposes because;
i. The acquisition of agricultural land, farmhouse, and plantation property is specifically
prohibited; and
ii. The time since the marriage took place and subsisted is less than two years; and
iii. There is a maximum ceiling limit of owning one property

Page 4 of 13

© The Institute of Chartered Accountants of India


Page 328
Case Study -3
3.1 (d)
3.2 (c)
3.3 (a)
3.4 (c)
3.5 (b)
3.6 The real estate business before the enactment of the RERD was highly unregulated. The builders
were dictating their own terms and conditions and the home buyers were helpless, since taking legal
recourse takes much more time. The common person was not able to understand the real estate
business jugglery like common area, floor area, wall area, carpet area, parking- covered parking-
open parking etc.
The home buyers were also not provided with the copies of the sanctioned plan, lay out of the flats,
the quality of the materials to be used and most important the date of possession, which was the
practice on the part of the builders to linger on and sometimes, it passes through 5, 7 or even 10
years from the promised date of possession.
However, after the enactment of the RERD the promoter is duty bound to first get the project
registered with RERA. The promoter has to make disclosures of all the relevant information as per
the Act. Some of the provisions of the Act, are mentioned here below, which advocates in favour of
the statement that the RERD has regularised the un-regulated real estate business.
Act deals with the matter relating to the registration of real estate project and registration of real
estate agents. It consists of section 3 to 10, which discusses over the following issues:
▪ Section 3. Prior registration of real estate project with Real Estate Regulatory Authority.
▪ Section 4. Application for registration of real estate projects.
▪ Section 5. Grant of registration.
▪ Section 6. Extension of registration.
▪ Section 7. Revocation of registration.
▪ Section 8. Obligation of Authority consequent upon lapse of or on revocation of registration.
▪ Section 9. Registration of real estate agents.
▪ Section 10. Functions of real estate agents
Act also defines the matter relating to the functions and duties of promoter being covered under
section 11 to 18, which elaborate on the following points:
▪ Section 11. Functions and duties of promoter.
▪ Section 12. Obligations of promoter regarding veracity of the advertisement or prospectus.
▪ Section 13. No deposit or advance to be taken by promoter without first entering into agreement
for sale.
▪ Section 14. Adherence to sanctioned plans and project specifications by the promoter.
▪ Section 15. Obligations of promoter in case of transfer of a real estate project to a third party.

Page 5 of 13

© The Institute of Chartered Accountants of India


Page 329
▪ Section 16. Obligations of promoter regarding insurance of real estate project.
▪ Section 17. Transfer of title.
▪ Section 18. Return of amount and compensation.
Some of the benefits that the allottees will have due to Insolvency and Bankruptcy Code are:
1. In the given case, RBL has taken the booking amount more than that prescribed under the
Act. The Act provides that booking amount shall not be more than 10% of the cost of the flat.
2. Further RBL has delayed in handing over the possession of flats. Some of the allottees have
demanded the refund of the amount along with interest and compensation, which RBL had to
passed on to them.
3. Further the remaining allotees intended to stay till the possession of the flats, but in this case
also RBL has to pay the monthly interest for every month of delay, till the handing over of the
possession.
4. Rights of the allottees
The rights given to the allottees have been described under Chapter IV of the RERD. Sub-section (1)
to (5) of section 19 provides the rights attached to the allottees, which are as under:
(1) The allottee shall be entitled to obtain the information relating to sanctioned plans,
layout plans along with the specifications, approved by the competent authority and such
other information as provided in this Act or the rules and regulations made thereunder or the
agreement for sale signed with the promoter.
(2) The allottee shall be entitled to know stage-wise time schedule of completion of the
project, including the provisions for water, sanitation, electricity and other amenities and
services as agreed to between the promoter and the allottee in accordance with the terms and
conditions of the agreement for sale.
(3) The allottee shall be entitled to claim the possession of apartment, plot or building, as the
case may be, and the association of allottees shall be entitled to claim the possession of the
common areas, as per the declaration given by the promoter under sub-clause (C) of clause
(l) of sub-section (2) of section 4.
(4) The allottee shall be entitled to claim the refund of amount paid along with interest at
such rate as may be prescribed and compensation in the manner as provided under this Act,
from the promoter, if the promoter fails to comply or is unable to give possession of the
apartment, plot or building, as the case may be, in accordance with the terms of agreement
for sale or due to discontinuance of his business as a developer on account of suspension or
revocation of his registration under the provisions of this Act or the rules or regulations made
thereunder.
(5) The allottee shall be entitled to have the necessary documents and plans, including that
of common areas, after handing over the physical possession of the apartment or plot or
building as the case may be, by the promoter.
This all happened only to due to the enactment of the Act. Prior to the enactment the home allottees
were not entitled to get any amount from the builder nor any interest and compensation.

Page 6 of 13

© The Institute of Chartered Accountants of India


Page 330
Thus, it is right to say that the enactment of this Act has regularised the un-regulated real estate
business.
3.7 Under the provisions of IBC
Explanation (i) to Section 5(8)(f) of the Insolvency and Bankruptcy Code, 2016 provides that any
amount raised from an allottee under a real estate project shall be deemed to be an amount having
the commercial effect of a borrowing.
Thus, the allottees are treated as financial creditors. Further the second proviso to section 7(1)
provides that the financial creditors who are allottees under a real estate project, an application for
initiating CIRP against the corporate debtor shall be filed-
▪ jointly by not less than 100 of such allottees under the same real estate project; or
▪ not less than 10% of the total number of such allottees under the same real estate project,
whichever is less.
In the given case the total number of allottees are 315. Thus, not less than 100 number of allottees
or 10% of 315 (say 32 after rounding off), whichever is less, means at least 32 allottees together can
initiate CIRP against RBL.
Under the provisions of RERD
Section 18 of the RERD deals with the return of amount and compensation. It provides that -
If the promoter fails to complete or is unable to give possession of an apartment, plot or building, —
(a) in accordance with the terms of the agreement for sale or, as the case may be, duly
completed by the date specified therein; or
(b) due to discontinuance of his business as a developer on account of suspension or revocation
of the registration under this Act or for any other reason,
he shall be liable on demand to the allottees, in case the allottee wishes to withdraw from the
project, without prejudice to any other remedy available, to return the amount received by him in
respect of that apartment, plot, building, as the case may be, with interest at such rate as may be
prescribed in this behalf including compensation in the manner as provided under this Act.
The language of the above section bears the sentence “without prejudice to any other remedy
available”, it means that if any other action/ remedy is available in other Act, will not affect the
remedy provided to the allottees under the Insolvency and Bankruptcy Code, 2016. Thus, it is very
much clear the allottees can take the advantages of RERD as well as of the IBC.
Where the allottee does not intend to withdraw
The proviso to section 18 of the Insolvency and Bankruptcy Code, 2016, provides that where an
allottee does not intend to withdraw from the project, he shall be paid, by the promoter, interest for
every month of delay, till the handing over of the possession, at such rate as may be prescribed.
Case Study 4
4.1 (b)
4.2 (c)
4.3 (c)

Page 7 of 13

© The Institute of Chartered Accountants of India


Page 331
4.4 (b)
4.5 (c)
4.6 The statement mentioned in the question is not correct. In fact, apart from the promoter, the real
estate agent have also been put under the RERD to obtain prior registration and have cast certain
duty of him.
Section 9 of the Act deals with the registration of real estate agents. Its sub-section (1) provides
that no real estate agent shall facilitate the sale or purchase of or act on behalf of any person to
facilitate the sale or purchase of any plot, apartment or building, as the case may be, in a real es tate
project or part of it, being the part of the real estate project registered under section 3, being sold by
the promoter in any planning area, without obtaining registration under this section.
Further, sub-section (7) provides that where any real estate agent who has been granted registration
under this Act commits breach of any of the conditions thereof or any other terms and conditions
specified under this Act or any rules or regulations made thereunder, or where the Authority is
satisfied that such registration has been secured by the real estate agent through misrepresentation
or fraud, the Authority may, without prejudice to any other provisions under this Act, revoke the
registration or suspend the same for such period as it thinks fit.
Further section 10 of the Act deals with the functions of the real estate agents and provides
that –
Every real estate agent registered under section 9 shall—
(a) not facilitate the sale or purchase of any plot, apartment or building, as the case may be, in a real
estate project or part of it, being sold by the promoter in any planning area, which is not registered
with the Authority;
(b) maintain and preserve such books of account, records and documents as may be prescribed;
(c) not involve himself in any unfair trade practices, namely:—
(i) the practice of making any statement, whether orally or in writing or by visible
representation which—
(A) falsely represents that the services are of a particular standard or grade;
(B) represents that the promoter or himself has approval or affiliation which such
promoter or himself does not have;
(C) makes a false or misleading representation concerning the services;
(ii) permitting the publication of any advertisement whether in any newspaper or otherwise of
services that are not intended to be offered.
(d) facilitate the possession of all the information and documents, as the allottee, is entitled to, at the
time of booking of any plot, apartment or building, as the case may be;
(e) discharge such other functions as may be prescribed.
Thus, from reading of section 9 and 10, it is clear that the real estate agent is required to get the
registration with RERA and cast duty on such agent to adhere the provisions of RERD failing which
penal provisions are attracted as specified in section 62. This section deals with the penalty for non -
registration and contravention under section 9 and 10 and provides that if any real estate agent fails
to comply with or contravenes the provisions of section 9 or section 10, he shall be liable to a penalty

Page 8 of 13

© The Institute of Chartered Accountants of India


Page 332
of ten thousand rupees for every day during which such default continues, which may cumulatively
extend up to five per cent. of the cost of plot, apartment or building, as the case may be, of the real
estate project, for which the sale or purchase has been facilitated as determined by the Authority.
4.7 Yes, it is true to say that post era of RERD have tighten the promoters. Earlier, the pro moters were
not having the transparency in dealing with the home buyers and often the possession of flats were
delayed.
The functions and duties of the promoter have been mentioned in Chapter III of the RERD consisting
of section 11 to 18. A brief overview of the aforesaid sections are as under:
Section 11: Functions and duties of promoter
(1) The promoter after getting the login id and password has to create his web page on the
website of the RERA and enter all details of the proposed project in all the fields as provided,
for public viewing, including:
(a) details of the registration granted by the Authority;
(b) quarterly up-to-date the list of number and types of apartments or plots, as the case may
be, booked;
(c) quarterly up-to-date the list of number of garages booked;
(d) quarterly up-to-date the list of approvals taken and the approvals which are pending
subsequent to commencement certificate;
(e) quarterly up-to-date status of the project; and
(f) such other information and documents as may be specified by the regulations made by the
Authority.
(2) The advertisement or prospectus issued or published by the promoter shall mention
prominently the website address of the Authority, wherein all details of the registered project
have been entered and include the registration number obtained from the Authority and such
other matters incidental thereto.
(3) The promoter, at the time of the booking and issue of allotment letter shall be responsible to
make available to the allottee, the following information, namely:—
(a) sanctioned plans, layout plans, along with specifications, approved by the competent
authority, by display at the site or such other place as may be specified by the
regulations made by the Authority;
(b) the stage wise time schedule of completion of the project, including the provisions for
civic infrastructure like water, sanitation and electricity.
(4) The promoter shall—
(a) be responsible for all obligations, responsibilities and functions under the provisions of this
Act or the rules and regulations made thereunder or to the allottees as per the agreement
for sale, or to the association of allottees, as the case may be, till the conveyance of all the
apartments, plots or buildings, as the case may be, to the allottees, or the common areas
to the association of allottees or the competent authority, as the case may be: Provided
that the responsibility of the promoter, with respect to the structural defect or any other
defect for such period as is referred to in sub-section (3) of section 14, shall continue even

Page 9 of 13

© The Institute of Chartered Accountants of India


Page 333
after the conveyance deed of all the apartments, plots or buildings, as the case may be, to
the allottees are executed.
(b) be responsible to obtain the completion certificate or the occupancy certificate, or
both, as applicable, from the relevant competent authority as per local laws or other laws
for the time being in force and to make it available to the allottees individually or to the
association of allottees, as the case may be;
(c) be responsible to obtain the lease certificate, where the real estate project is developed
on a leasehold land, specifying the period of lease, and certifying that all dues and charges
in regard to the leasehold land has been paid, and to make the lease certificate available to
the association of allottees;
(d) be responsible for providing and maintaining the essential services, on reasonable
charges, till the taking over of the maintenance of the project by the association of the
allottees;
(e) enable the formation of an association or society or co-operative society, as the case
may be, of the allottees, or a federation of the same, under the laws applicable:
Provided that in the absence of local laws, the association of allottees, by whatever name
called, shall be formed within a period of three months of the majority of allottees having
booked their plot or apartment or building, as the case may be, in the project;
(f) execute a registered conveyance deed of the apartment, plot or building, as the case
may be, in favour of the allottee along with the undivided proportionate title in the common
areas to the association of allottees or competent authority, as the case may be, as
provided under section 17 of this Act;
(g) pay all outgoings until he transfers the physical possession of the real estate project to
the allottee or the associations of allottees, as the case may be, which he has collected
from the allottees, for the payment of outgoings (including land cost, ground rent, municipal
or other local taxes, charges for water or electricity, maintenance charges, including
mortgage loan and interest on mortgages or other encumbrances and such other liabilities
payable to competent authorities, banks and financial institutions, which are related to the
project):
Provided that where any promoter fails to pay all or any of the outgoings collected by him
from the allottees or any liability, mortgage loan and interest thereon before transferring the
real estate project to such allottees, or the association of the allottees, as the case may be,
the promoter shall continue to be liable, even after the transfer of the property, to pay such
outgoings and penal charges, if any, to the authority or person to whom they are payable
and be liable for the cost of any legal proceedings which may be taken therefor by such
authority or person;
(h) after he executes an agreement for sale for any apartment, plot or building, as the case
may be, not mortgage or create a charge on such apartment, plot or building, as the case
may be, and if any such mortgage or charge is made or created then notwithstanding
anything contained in any other law for the time being in force, it shall not affect the right
and interest of the allottee who has taken or agreed to take such apartment, plot or
building, as the case may be;
(5) The promoter may cancel the allotment only in terms of the agreement for sale:
Provided that the allottee may approach the Authority for relief, if he is aggrieved by such

Page 10 of 13

© The Institute of Chartered Accountants of India


Page 334
cancellation and such cancellation is not in accordance with the terms of the agreement for
sale, unilateral and without any sufficient cause.
(6) The promoter shall prepare and maintain all such other details as may be specified, from time
to time, by regulations made by the Authority.
In addition to the functions and duties mentioned in section 11, few more sections of Chapter II
highlight the compliance to be adhered by the promoter. These are:
▪ Section 12. Obligations of promoter regarding veracity of the advertisement or prospectus.
▪ Section 13. No deposit or advance to be taken by promoter without first entering into agreement
for sale
▪ Section 14. Adherence to sanctioned plans and project specifications by the promoter
▪ Section 15. Obligations of promoter in case of transfer of a real estate project to a third party
▪ Section 16. Obligations of promoter regarding insurance of real estate project
▪ Section 17. Transfer of title
▪ Section 18. Return of amount and compensation
In the given, the promoter has not complied with the provisions of RERD and was therefore
penalised.

Case Study 5
5.1 (b)
5.2 (c)
5.3 (b)
5.4 (c)
5. 5 (a)
5.6 (A) Regulation 9 of the Foreign Exchange Management (Export of Goods & Services)
Regulations, 2015 deals with the period within which the export value of goods is to be
realized. It is provided in Sub Regulation 9 (1) (a) that that where the goods are exported to a
warehouse established outside India with the permission of the Reserve Bank, the amount
representing the full export value of goods exported shall be paid to the authorized dealer as
soon as it is realized and in any case within fifteen months or within such period as may be
specified by the Reserve Bank, in consultation with the Government, from time to time from
the date of shipment of goods.
It is further provided that RBI, or subject to the directions issued by that bank on this behalf,
the authorised dealer may, for a sufficient and reasonable cause shown, extend the said
period.
Since in the given case, STS is a warehouse facility of SPL established with the permission of
RBI in Poland and the goods were shipped and/or exported on 1st January 2020, EUR
265,000 is expected to be realized within the next 15 months i.e. by March 31st, 2021, unless
the period is further extended as above. It is the full value of export i.e. CIF value (EUR
265,000) is to be realized within the period stipulated in Regulation 9.

Page 11 of 13

© The Institute of Chartered Accountants of India


Page 335
(B) Regulation 9 of the Foreign Management (Export of Goods & Services) Regulations, 2015
deals with the period within which export value of goods to be realized. As per sub -regulation
9 (1), the amount representing the full export value of goods / software/ services exported
shall be realized and repatriated to India within nine months or within such period as may be
specified by the Reserve Bank, in consultation with the Government, from time to time from
the date of export,
It is further provided that the Reserve Bank, or subject to the directions issued by that Bank
on this behalf, the authorized dealer may, for a sufficient and reasonable cause shown,
extend the said period.
Since in the given case, the goods were shipped and/or exported on 1st January 2020, GBP
126,000 is expected to be realized within the next 9 months i.e. by September 30th, 2020,
unless the period is further extended as above. It is the full value of export i.e. GBP 126,000
is to be realized within the period stipulated.
(C) Regulation 9 of the Foreign Management (Export of Goods & Services) Regulations, 2015
deals with the period within which export value of goods to be realized. As per Regulation 9
(2) (a) Where the export of goods / software / services has been made by Units in Special
Economic Zones (SEZ) / Status Holder exporter / Export Oriented Units (EOUs) and units in
Electronics Hardware Technology Parks (EHTPs), Software Technology Parks (STPs) and
Bio-Technology Parks (BTPs) as defined in the Foreign Trade Policy in force, then
notwithstanding anything contained in sub-regulation (1), the amount representing the full
export value of goods or software shall be realized and repatriated to India within nine months
or within such period as may be specified by the Reserve Bank, in consultation with the
Government, from time to time from the date of export.
It is further provided that the Reserve Bank, or subject to the directions issued by that Bank in
this behalf, the authorized dealer may, for a sufficient and reasonable cause shown, extend
the said period.
Since in the given case, the goods were shipped and/or exported on 1st January 2020, EUR
265,000 is expected to be realized within the next 9 months i.e. by September 30th, 2020,
unless the period is further extended as above. It is the full value of export i.e. CIF value
(EUR 265,000 is to be realized within the period stipulated.
5.7 According to section 6 (4) of the Foreign Exchange Management Act, 1999 (here-in-after referred to
as the Act) read with regulation 5 of Foreign Exchange Management (Acquisition and transfer of
immovable property outside India) Regulations, 2015,
(1) A person resident in India may acquire immovable property outside India;
(a) By way of gift or inheritance from a person referred to in sub-section (4) of Section 6 of
the Act, or referred to in clause (b) of regulation 4;
(b) By way of purchase out of foreign exchange held in Resident Foreign Currency (RFC)
account maintained in accordance with the Foreign Exchange Management (foreign
currency accounts by a person resident in India) Regulations, 2015;
(c) Jointly with a relative who is a person resident outside India, provided there is no
outflow of funds from India;
(2) A person resident in India may acquire immovable property outside India, by way of

Page 12 of 13

© The Institute of Chartered Accountants of India


Page 336
inheritance or gift from a person resident in India who has acquired such property in
accordance with the foreign exchange provisions in force at the time of such acquisition.
(3) A company incorporated in India having overseas offices, may acquire immovable property
outside India for its business and for residential purposes of its staff, in accordance with the
direction issued by the Reserve Bank of India from time to time.
These are the possible ways by which these properties might have been legally acquired by SPL in
UAE.

Page 13 of 13

© The Institute of Chartered Accountants of India


Page 337
Compiled By: Shivam Gopal Mundad

Test Series: April, 2021


MOCK TEST PAPER
FINAL (New) GROUP – II
PAPER – 6D: ECONOMIC LAWS
Question Paper comprise of five case study questions. The candidates are requested to answer any
four case study questions out of five.
Time Allowed – 4 Hours Maximum Marks – 100
Case Study 1
Mr. Aman Chawla belongs to Delhi based business family and has ancestral roots in Kharar, a Town in the
Sahibzada Ajit Singh Nagar (Mohali) district in the state of Punjab (around 15 KMs away from Chandigarh).
Chawla family owns the chain of restaurants, snacks points, and Ice-Cream parlours across the nation. Few
of these are owned properties, but a large number are leased properties. The holding company is Chawla
Snacks and Refreshment Limited (CSRL). Mr. Aman is an electrical engineer, joined an MNC in the role of
system engineer after college. But Mr. Aman is inspired by constructing the buildings, towers, landscapes,
hence decided to quit the job to pursue his passion.
Despite the Chawla family owning a major stake in the business, the business model is unlike to autocratic
monarchy. It is managed professionally and listed on the stock exchange. Family members (father, grand-
mother and elder brother of Mr. Aman) are part of the Board of Directors, whereas few other family members
are also engaged with CSRL but in form of employment (or in a professional capacity).
Mr. Aman joined his brother-in-law, Mr. Vivek, in his construction business. Mr. Aman assists
Mr. Vivek in ongoing projects, and one among them is Rishi Enclave whose centre of attraction is state of art
yoga centre which will be one of its type in the world apart from the common area which is turned into with
mesmerising landscapes. The project is located near Jolly Grant Airport on out-skirt of the holy town of
Rishikesh. Rishi Enclave (Project) consists of 120 units of 2BHKs, 3BHKs (Flats and Floors), and Independent
Houses or Villas in totality. The project is registered under the RERA. All 120 units’ subscr ibed/booked by
allottees except 2 Flats kept by Mr. Vivek (promoter). Mr. Tirlochan Negi booked 3 floors one in his own name,
another one in the name of his daughter in law and the third one in name of his company. Mr . Dabral also
booked a flat and a villa (both in his name). Rest all allottee booked one unit each. Soon allottees form a
residential association. Considering the latest NGT decisions and amendments in policy about the
environment (applicable for civil construction in hill or foothill area concerning the height of the building),
certain structural changes relating to the height and common area landscape is required in sanctioned plan
of the project. Mr. Vivek is of opinion since the alteration in sanctioned plan enforced by changes in policy
matter hence the approval of allottees is not required.
Mr. Aman recently visited Kharar after a long time to meet his friends Mr. Onkar Singh and
Mr. Dipan Ahuja of early childhood. They all admitted that the town has developed substantially especially
the townships and Skyscrapers as tri-city (Mohali, Chandigarh, and Panchkula) turns into metropolitan and
hub of service entities. The lifestyle of people also improves. Mr. Onkar is settled in Canada and holding a
Canadian passport and citizenship as his family migrate there when he was in school only. In Canada, he
own a transport business. Currently, he is on a visit to India to attend the marriage of a relative. Mr. Dipan
Ahuja is a supplier of construction materials and planning to venture into the solar panel business under make
in India drive, considering the enhancing role of solar energy for household and commercial uses.
Mr. Dipan believes Mr. Aman (considering his electrical engineering background) should join him in his solar
panel venture.
The ancestral property of Mr. Onkar’ family has been unoccupied for a long, hence turned into a mud house.
Mr. Onkar offered Mr. Aman to develop residential apartments on such property after the name of his grand -
father ‘Satnam Apartments’. A chunk of land on the backside of such property is also available for sale at a
Page 338 1
Compiled By: Shivam Gopal Mundad

reasonable price because it has no connectivity. Mr. Aman found it a good idea to develop the residential
apartments as backside land can be acquired at a cheaper rate than prevailing in the mark et. Mr. Onkar
talked to his father [property inherited, hence registered in his name in land revenue records after the death
of grandfather (who was resident in India) of Mr. Onkar] and ready to transfer (sale) the property for INRs 2.5
Crore. The Father of Mr. Onkar is a resident outside India who never registered as OCI. Mr. Aman after
communicating with Mr. Vivek agreed to deal.
Mr. Aman heard about the importance of keeping capital low to generate more wealth and attain high ROI
(Return on Investment). He decided to borrow money from a private investor from the States (US) based on
showing growth prospect in his business to his investor. The investor was a good friend of Mr. Dipan and
originally from Mohali named Mr. Tarun and settled in Philadelphia (Pennsylvania, US). Mr. Tarun agreed to
invest US$1 Million in the said real estate project.
The money got transferred from an overseas branch in Philadelphia of some Indian bank (through banking
channel) to the Kharar branch (Mohali, India). The Branch Manager in India, is the friend of an elder brother
of Mr. Aman and was excited to get one project in Mohali and thus approved the investment without any
opinion from any Finance Professional.
CSRL witnessed the bad jolts (of financial turbulence) as revenue vanished and reserves are socked to meet
maintenance costs of properties & employee cost due to lock-down and afterword restrictions. The financial
cost and lease rentals not only erode the working capital but also forces the CSRL to land into a debt trap
situation wherefrom meeting financial obligations seems near to impossible. The only way left to management
is restructuring of business hence Board decided to shut a few points and parlours (to reduce lease rental
obligation, and free-up one-two owned properties so that sale proceed can be infused as working capital)
One of the properties sold by CSRL, acquired by Ms. Vijeta in name of her mother-in-law (as she is a senior
citizen female – to bear less registration cost in form of stamp duty), consideration for which is paid out of the
known sources of the Ms. Vijeta.
Despite the best efforts made by management at CSRL, still, the bottom line is in deep red, resulting in default
in repayment of financial debts and such default continues since the 2nd quarter of Fiscal 2020-21.
Management gave assurance to financial creditors that soon it will overcome the solvency issue and they
already took corrective measures. On 19 th March, 2021, one of the financial creditors moved an application
for initiation of corporate insolvency resolution process (CIRP) whose outstanding claim was INRs 120 lakh.
On 26 th March, 2021, another financial creditor filed an application to NCLT for initiation of CIRP against
CSRL in their case amount of default was INRs 35 lakh and such default took place in the 3 rd Quarter of fiscal
2020-21.
Multiple Choice Questions (2 Marks each for correct answer)
1.1 Regarding the state of art yoga centre and common area situated in Rishi Enclave, which of the following
statement is correct;
(a) Promoter will keep the possession and title both
(b) Promoter may handover physical possession of these to the association of allottees or competent
authority as per the local laws
(c) In absence of any local law promoter shall hand over within thirty days after obtaining the
occupancy certificate.
(d) In absence of any local law promoter shall hand over within thirty days after obtaining the
completion certificate.
1.2 State the legal position of mother-in-law of Ms. Vijeta as benamidar in the case study-
(a) Yes, the mother-in-law of Ms. Vijeta is benamidar
(b) No, the mother-in-law of Ms. Vijeta is not benamidar as she is covered under the exceptions stated
Page 339 2
Compiled By: Shivam Gopal Mundad

(c) No, mother-in-law of Ms. Vijeta is not benamidar as consideration is paid out of the known source
of Ms Vijeta
(d) Both b and c above.
1.3 Which of the following statements is correct regarding the acquiring, holding, owning and transfer of
property, in a case by the father of Mr. Onkar in India-
(a) Being a person resident outside India he can acquire, hold, own and transfer any immovable
property in India, but with RBI permission only
(b) Being a person resident outside India he can acquire, hold, own and transfer any immovable
property in India, but only in joint ownership with any person resident in India
(c) Being a person resident outside India he can acquire, hold, own and transfer any immovable
property in India, if inherited by him from the person who was a resident of India
(d) Being a person resident outside India he can acquire, hold, own and transfer any immovable
property in India, if inherited by him when he himself was resident in India
1.4 Whether the application moved on 19 th March, 2021 can be admitted by NCLT to initiate CIRP against
CSRL-.
(a) Yes, because CSRL made default in repayment of financial debts
(b) Yes, because the amount of default is more than one crore
(c) No, because management gave assurance to financial creditors that soon it will overcome the
solvency issue and they already took corrective measures
(d) No, because an application for initiation of CIRP shall not be filled.
1.5 Whether the application moved on 26 th March, 2021 can be admitted by NCLT to initiate CIRP against
CSRL.
(a) Yes, because CSRL made default in repayment of financial debts
(b) Yes, because the application for initiation of CIRP may be filled by the financial creditor as a period
of suspension of section 7 is over.
(c) No, because the amount of default is less than one crore
(d) No, because default occurred during a period of suspension.
Descriptive Questions
1.6 Mr. Vivek is of opinion since the alteration in sanctioned plan enforced by changes in policy matter hence
the approval of allottees is not required. Are the changes in sanctioned plan minor in nature? Evaluate
the opinion of Mr. Vivek in the context of the provision contained in the RERA 2016? Support your
answer with appropriate reason. (6 Marks)
1.7 What would be your opinion related to the repatriation of funds in India as an Investment of US$1 million
into the real estate project in Kharar (Mohali, India)? (5 Marks)
1.8 Can the father of Mr. Onkar repatriate the sale proceed of ancestral property inherited by him to Canada
from India? Elucidate in the light of the relevant provision of the applicable law, the mentioned legal
issue. (4 Marks)

Page 340 3
Compiled By: Shivam Gopal Mundad

Case Study 2
Mr. David Pinto is a Goa-based businessman. He holds an Indian passport and usually resides in India, but
occasionally used to visits other countries of the world for business and vacations. During the preceding
financial year, he was out of India for 42 days. Mr. Pinto owns beach restaurants with water -sports facilities,
resorts, spa centres, and floating restaurants apart from a fleet of boats and ferries in form of Travel Thrill
Private Limited (TTPL). TTPL had a global turnover of INRs 2735 crore in the immediately preceding year
with an asset base of worth INRs 960 crore in India. Mr. Pinto has marketing tie-ups with travel agents and
leading tour and travel companies all across the globe. Mr. Pinto recently launched a web-based portal ‘Fun
Holidays’ offering a wide variety of travel packages including stay and food. Fun Holidays become a big hit
and buzz.
Singapore-based Thanjai Tours & Travels Pte Ltd. (TTTPL) targets TTPL to acquire and ready for a hostile
one. Mr. Pinto along with other Board members of TTPL decided to attempt the reverse acquisition of TTTPL
to defend the threat of their hostile acquisition. Finally, TTPL acquired control over TTTPL. Thanjai Tours &
Travels Pte Ltd. had a turnover of US$ 3400 million in the immediately preceding year with an asset base of
US$ 1200 million across the globe including assets worth INRs 80 crore in India.
TTPL received notice from the deputy commissioner of Income Tax regarding some suspicious and high -
value benami transactions. Hence the book of accounts called and detailed enquiry was initiated into the
matter under the Prohibition of Benami Property Transaction Act, 1988. Mr. Pinto is of opinion that deputy
commissioner of Income Tax has no jurisdiction to inquire into the matter under the Prohibition of Benami
Property Transaction Act, 1988.
Ms. Anna, the daughter of Mr. Pinto is architecture, stays in Georgia and holds NRI status. In India, there are
many assets and properties vested in name of Ms. Anna, from which she earns the rental income.
Ms. Anna willing to manage her Indian income through the use of a bank account, but is confused with NRE,
NRO, FCNR, RFC Accounts, and their utility and implications.
Mr. Joe, the son of Mr. Pinto recently joined him in business and starts working on diversification. He identified
the opportunity in the export of seafood, especially to European countries. Mr. Joe contacted a shipping
company for an operational tie-up to reserve the space of two containers on the vessel moved on every
Friday. After recording an initial good response for few days, Mr. Joe decided to form a company ‘Silver
Exports Private Limited’ (SEPL) with overseas branches to expand the business. Despite the directors and
managers warn Mr. Joe to consolidate rather than keep on expanding highlighting financial vulnerability, but
Mr. Joe further decided to buy own ship named ‘sliver-line’ for logistic (because according to him the
arrangement of shipping through the vessel, as discussed above is economical but inflexible) of seafood. The
decision of purchasing a ship puts the burden on financial leverage. In the years, Mr. Joe realized his mistake
because increased logistic and financial cost starts causing troubles.
The situation became worse when different countries announced lockdowns across the globe to prevent the
widespread of COVID-19, including India. International trade and transport almost halted to nil. After lock -
down lifted, the finances of SEPL were really in bad shape. While reaching towards the end of F.Y 2020-
2021, SEPL left with only two weeks of working capital (on the 20 th March, 2021). Somehow SEPL is serving
its debt. On the evening of 22 nd March, 2021 silver-line left the mormugao port for its destination in Europe
through the Indian Ocean and the Mediterranean Sea via the Red Sea. On the morning of
23 rd March, 2021, the Suez Canal was blocked after the grounding of Ever Given, a 20,000 TEU container
ship, hence the silver-line stuck and forced (amid ambiguity) to take another route (circumnavigate the African
continent) to its destination, which results in a huge increase in logistic cost and the SEPL made default again
in meeting its debt obligation due on 30th March, 2021 as working capital turned negative. The amount
regarding default that took place was 2.1 crore. One of the financial creditor with an outstanding of 42 lakh
moved to NCLT for initiation of the Corporate Insolvency Resolution Process.

Page 341 4
Compiled By: Shivam Gopal Mundad

Ms. Rita, the wife of Ms. Joe who is also a director in the SEPL and designated as officer -in-charge and
signing authority for legal filing, forex and export documentations. Custom officers identified Sliver-line
engaged in illegal trans-shipment of psychotropic substance worth INRs 4- 4.5 crore. Captain of the ship, Ms.
Rita, and One of the other directors (who is a senior citizen) of SEPL arrested under the Prevention of Money
Laundering Act 2002. Ms. Rita argued that she was unaware of the matter and her arrest is without a warrant.
Captain and Director also argued against their arrest stating the officer is not empowered to arrest them
without a warrant and investigate without prior authorization from the Government.
Mr. Sadanand Yadav, father of Ms. Rita started his journey as a real estate agent around two decades ago
and currently owning a construction business with the name ‘Nand Developers and Infrastructures’ (NDI).
NDI recently launched another residential project ‘Surya Enclave’, developed near to Haniman Chauraha on
Sahara Hospital Road in Viraj Khand, Gomti Nagar (Lucknow, UP). The project consists of 320 residenti al
units with different dimensions and specifications. The project got an overwhelming response and all the unit
was subscribed within a week of the start of the booking. Allocation was duly made and the allottee develops
an informal association among themselves and it was decided that a residential society will be formed as and
when projects reach the stage of completion.
Due to lockdown and afterward restrictions the construction work at Surya Enclave halted, since a large part
of workforce (labour) are daily-based casual migrant workers who walked back to their villages and towns;
hence delay in completion of the project as compared to sanctioned plan is expected. The financial cost and
blocked working capital are causing trouble at NDI to manage its projects including Surya Enclave. Mr.
Sadanand along with other directors of NDI thinks it’s better to transfer the project to some other realtor or
developer.
Mr. Dev Manohar is a business partner of the brother-in-law of Sadanand Yadav and a renowned landlord of
the region. Mr. Sadanand who has political ambition also, willing to contest in the upcoming assembly election
in the state. To arrange funds for the campaign and allied expenditures, Mr. Sadanand approached Mr. Dev
to buy a piece of land he owns near to highway on the way to Kanpur. Mr. Dev ready to buy the plot, but
considering the limit on landholding and land already owned by him in his own name, he instructed Vasika
Navis (Deed Writer) to draft the deed for registration of the property in four equal part in name of him, his
wife, his mother, and wife of his elder brother.
Multiple choice questions (2 Marks each for correct answer)
2.1. Ms. Rita, Captain of Ship, and Director (who is a senior citizen) approached the special court constituted
under the Prevention of Money Laundering Act 2002 for bail. Explain the validity of bail applications.
(a) Ms. Rita (being a woman) can get bail
(b) Ms. Rita (being a woman) and Director (being a senior citizen) can get the bail
(c) None of them get the bail as the amount involved in the offence is more than INRs one crore.
(d) None of them get bail as the offence is non-bailable.
2.2 Who among the following is benamidar?
i. Wife of Mr. Dev
ii. Mother of Mr. Dev
iii. Wife of the elder brother of Mr. Dev
(a) ii only
(b) iii only
(c) Both ii and iii
(d) None of i, ii, and iii

Page 342 5
Compiled By: Shivam Gopal Mundad

2.3 Regarding admissibility of application furnished by financial creditor to initiate the Corporate Insolvency
Resolution Process in case of SEPL, which of the following statements is correct?
(a) Since the default is caused by a bad financial position that arises due to COVID -19, hence
suspension provision applies here.
(b) Amount of default is less than the threshold limit for filing an application.
(c) Default took place after the suspension clause expired, this is sufficient cause to admit the
application
(d) None of these.
2.4 Which of the following statements are correct requirements regarding the transfer of project ‘Surya
Enclave’ in the case of NDI?
i. Project can be transferred, after obtaining the consent from at least two-thirds of allottees
ii. Project can be transferred, after obtaining the written approval from the authority
iii. Allocation or sale done by original or erstwhile promoters remain unaffected
(a) i and ii only
(b) ii and iii only
(c) i and iii only
(d) All i, ii, and iii
2.5 Who out of the following can act as initiating officer under the Prohibition of Benami Property Transaction
Act,1988?
(i) Assistant Commissioner
(ii) Joint Commissioner
(iii) Deputy Commission
(a) ii only
(b) both ii and iii only
(c) both i and iii only
(d) All i, ii and iii
Descriptive Questions
2.6 Differentiate between NRE and NRO accounts while highlighting its purpose and implications, how the
residential status of the person is relevant to them, and can Ms. Anna open and operate a joint NRO
account with his father? (2+1+2 Marks)
2.7 Does the acquisition of TTTPL by TTPL results in the formation of a combination under the Competition
Act, 2002? Mention the thresholds. Is combination prohibited under the Competition Act 2002?
(2+2+1 Marks)
2.8 Ms. Rita argued that she was unaware of the matter and her arrest is without a warrant. Captain and
Director also argued against their arrest stating the officer is not empowered to arrest them without a
warrant and investigate without prior authorization from the government. Are their arguments legally
tenable? (5 Marks)
Page 343 6
Compiled By: Shivam Gopal Mundad

Case Study 3
Mr. Yogandra Prasad, Mr. Srinivasan, and Mr. Venkatesh Rao started the business of manufacturing the saw
machines around seven-eight years ago as business partners. Mr. Prasad is the only son in his Salem-based
agriculture family. A decade back when Mr. Prasad completed his ITI diploma in mechanics, moved to
Madurai for employment in a company that was engaged in manufacturing different machines and hand tools;
because the size of a farm owned by Mr. Prasad's family not that big that it requires two -person to manage
and Prasad’s father performs the farm-related activities easily with the occasional support of Prasad’s mother.
There he met with Mr. Srinivasan who was an accountant in the company, has an additional charge of looking
after the dealership-related stuff, and Mr. Rao was plant supervisor. Soon they become good friends. Mr.
Srinivasan always stressed upon the starting of own-business; Mr. Prasad and Mr. Srinivasan were also
inspired by his thought.
With help of the elder brother of Mr. Rao, who is based in Madurai only; they (all 3) established the workshop.
Mr. Prasad and Mr. Rao started repairing the machines there in the evening or on holidays;
Mr. Srinivasan also helped them in to look after the general management and client dealings. Soon they
started getting more and more installation and repair requests from nearby, even one saw machine
manufacturer outsourced the task of repairing the defective machine or those upon whom the warranty is
invoked. They (all 3) quit the job and start spending full time in workshops and hire few workers as well. In a
couple of years, they installed a manufacturing facility of saw machines and turned workshops into the plant
and their informal partnership changed into the duly incorporated company (PRS Machine Limited).
Mr. Prasad looks after the production and operations, Mr. Rao looks after general management, HR, and
control, Mr. Srinivasan looks after finance and marketing. Business resources were applied with utmost
caution, like idle cash immediately invested in marketable securities, working capital optimized, production
process streamlined and cash operating cycle (using floats) was minimized. The business was really doing
well and able to acquire a substantial market share because it has a low cost of pr oduction. Due to the low
cost of production PRS machine Limited (here-in-after PRS) able to offer deep discounts to its customer,
especially first-time customers. The Standard portable saw machine of 16 th Inches which PRS was able to

manufacture under INRs 3500/- and sold for in a range of INRs 3800-4000, other competitors were able to
manufacture the same in and around INRs 4000/-.
Out of rivalry, a business competitor uses its political nexus to annoy the management at PRS Machine
Limited, through frivolous searches of the Prevention of Money Laundering Act 2002 and notices from the
Enforcement Directorate. PRS faced notices from the Competition Commission of India as well.
In 2019, Mr. Prasad got married to the younger sister of Mr. Rao, Ms. Rukmani. They visited Europe after
marriage. Mr. Prasad and Rukmani had few unspent Euros in the form of currency notes and coins with them,
which they forgot to exchange at the airport.
To accelerate the growth (organic and inorganic) PRS need more fund and keeping the hurdle rate low is
another target in front of management, hence it was decided to raise the fund through External Commercial
Borrowings. Despite Mr. Srinivasan managed the finance function but not well-aware of the latest directions
of the apex bank (money market regulator) on External Commercial Borrowings. The floatation process has
been completed in the week first of March, 2020. Lockdown announced in the fourth week of March, 2020,
hindered the expansion projects in midways and idle funds (raised through ECB) is need to be parked
domestically. Management is thinking to repay some of the existing loans as regular cash flow is barely
enough to meet the needs and commitments of PRS.
Mr. Prasad booked a 4BHK flat in royal residency so that the entire family including his wife and parents
migrate to Madurai. Prasad deposits 12% of the cost of the property with the promoter (Shiva Estate and
Relators) in advance after entering the agreement to sell. Brochure carrying due date and amoun t of
instalment handed over to Mr. Prasad along with receipt of deposit made by him. Mr. Prasad also becomes
a member of the association of the allottees, the legal formation of the association is in process.

Page 344 7
Compiled By: Shivam Gopal Mundad

Mr. Prasad figured out that his father is getting older now and it may be difficult for him to manage farms,
hence insisted his parent to shift with him. But his parents decided to stay back at Salem only because they
wish to spend their remaining life in the house and village, where they spent a substantial part of life (post-
marriage in the case of his mother). Father gave him (Mr. Prasad) INRs 50 lakh to register the property.
Mr. Prasad registered the property in his own and his wife’s name using such INRs 50 lakh as part of the
consideration paid.
Although by the end of 3 quarter of 2020 the construction was expected to be completed and possession was
planned to be given, but COVID-19 hit the schedule, and projects are running delay with 4-5 months.
Considering this the association of allottees talked to representatives of Shiva Estate and Relators and
demand interest on their money deposited with the promoter in form of advance and part payments. Promoter
promises to complete the project at the earliest, pointing out pandemic as the only reason for delay. Further,
he denies paying interest to allottees on money deposited with the promoter in form of advance and part
payments. The period for which the registration granted to Shiva Estate and Relators under section 5 of the
RERA 2016 also approaching to end.
Multiple Choice Questions (2 Marks each for correct answer)
3.1 Assess the correctness of following statements in view of provisions contained in the Prevention of
Money Laundering Act 2002, regarding the offence of frivolous search without recording the reason in
writing by authority or officer;
i. Cognizance can be taken by the court at its own
ii. Such authority or officer shall be punishable with both fine and imprisonment
(a) Only i is correct
(b) Only ii is correct
(c) Both i and ii are correct
(d) Both i and ii are incorrect
3.2 Mr. Prasad and Rukmani had few unspent Euros in the form of currency notes with them. These amounts
can be retained with him:
(a) For 120 days from the date of acquisition
(b) For 120 days from the date of return to India
(c) For 180 days from the date of acquisition
(d) For 180 days from the date of return to India
3.3 Regarding parking of ECB proceeds domestically pick the correct statement out of the following;
(a) In term deposit with AD Category I bank for 6 Months
(b) In term deposit with AD Category I or II bank for 6 Months
(c) In term deposit with AD Category I bank for 12 Months
(d) In term deposit with AD Category I or II banks for 12 Months
3.4 Which of the following statement is correct regarding the deposit of advance for 4BHK under the RERA
2016:
(a) Promoter shall not accept any deposit
(b) Promoter may accept the deposit to any percentage
(c) Promoter shall not accept any deposit unless entered into a written agreement for sale
(d) Promoter is allowed to accept the deposit less than ten percent in any case

Page 345 8
Compiled By: Shivam Gopal Mundad

3.5 Whether the father of Mr. Prasad is benamidar?


(a) Yes, because he is not among the person in whose name property is registered
(b) Yes, because his daughter in law is also part owner of the property
(c) No, because he provides money which is part of the total consideration
(d) No, because the property is not held for the immediate or future benefit, direct or indirect, of him
Descriptive Questions
3.6 Is PRS abusing the dominance through predatory pricing of their product? What factor, commission shall
consider while determining the existence of dominance? The cost determined under the regu lation is
INRs 3800/-. (2+2 Marks)
3.7 Mr. Srinivasan is not clear about end uses of ECB hence approached Mr. Kartik (you), a chartered
accountant to know, whether -
(i) ECB can be availed of for making equity investment domestically or buying goodwill? Can ECB be
availed of for making a contribution in an LLP? (1+1 Mark)
(ii) ECB raised and used for repayment of Rupee loans availed domestically for purposes other than
capital expenditure? Will it make any difference if Rupee loans availed domestically for purposes
of capital expenditure? Will it make any further difference if the loan is not availed domestically?
(2+1+1 Marks)
3.8 In light of provision contained in the Real Estate (Regulation and Development) Act, 2016, decide:
(i) Can an unregistered allottees association can file a complaint with the authority? (3 Marks)
(ii) Can Shiva Estate and Realtors apply for an extension of registration for the project? (2 Marks)
Case study 4
State Metro Rail Corporation is responsible for the construction and smooth operation of the metro rail
network in the states’ capital. In the first phase, two lines (tracks) are in operation, North -South (Red Line)
and East-West (Blue Line); on which train (with the length of 4 coaches) runs both ways. A tender notice was
floated by State Metro Rail Corporation, for procurement of battery sets for metro coaches running in the
state’s capital, from the list of approved vendors across the globe. None of the global firms responds to tender
considering logistic-related uncertainties due to the second wave of Covid-19. Only 8 firms from different
parts of India submitted the response. The general manager found the rate quoted by most of the firms was
the same per unit, on an all-inclusive basis. The quoted rate found 25% higher than, the rate at which
procurement of similar battery sets (exactly same specifications) was done by National Capital’s Metro Rail
Corporation very recently. It is also observed that the quantity quoted by each of the firms was, far or less
near to 20% of the total tender quantity.
The General Manager at State Metro Rail Corporation in presence of the Project Director talked to the Head
of Legal Division regarding this, to express his suspicions over the possible cartelization among the bidders.
A letter regarding this sent to Directors’ office (State Metro Rail Corporation) seeking permission to lodge a
complaint with the Competition Commission of India.
Metal and Casting Iron Limited (MCIL) is one of the largest suppliers to State Metro Rail Corporation, supplies
a wide range of metals of different shapes and specifications. To lower down the cost of operation, MCIL
decided to go for floating funds from the international market. Mr. Mukund Yadav, VP finance made a
presentation in front of the Board of directors including the CEO, whereat he expressly favours the External
Commercial Borrowings (ECBs) as a cheap and long-term source of finance, above others. In the next board
meeting at MCIL, the board approves the decision to raise fund through ECBs in two tranches, out of which
one shall be used for the repayment of rupee loans availed domestically. It was also decided that borrowing
from foreign branches and subsidies of any sort of Indian financial institutions shall not be accepted. When it
comes to deciding the duration of borrowing, there is a difference of opinion, regarding the minimum average
Page 346 9
Compiled By: Shivam Gopal Mundad

maturity period (MAMP).


MCIL to establish itself as a global brand that carries a sustainable strategic vision and a promising value
system, start sponsoring national and international sport, music, and cultural events. Recently it sponsored
the Grammy awards. MCIL became a sponsor for the Man of Match Award for the 13 th edition of the Indian

Premier League (season 2020) hosted in the United Arab Emirates by the Emirates Cricket Board. The Board
of Control for Cricket in India issued a letter of sponsorship in favour of MCIL. For a total of 32 league matches,
USD 800,000 needs to be remitted by MCIL @ USD 25,000 for each match.
Way back in 2018, MCIL decided to diversify the business. Based on expert committee (of board members)
report and multiple rounds of consultation with advisories firms, it decided to venture into th e Real Estate
sector. Another company was formed after the name of the founder of MCIL, ‘Ramaanuj Construction Limited’
(RCL) which is structured as a subsidiary to MCIL. To start with RCL acquire the few running (ongoing)
projects from promoters and stated few others on their own. Since RCL did not own much experience in real
estate, hence failed to deliver the project on time as scheduled in their majority of the project. This delay not
only annoys the allottees but also causing a huge escalation in cost. Some of the projects really stuck in bad
shape. Association of allottees wrote to RCL and talked to representatives of RCL, except promises allottees
are not getting anything concrete hence the association of allottee decides to file the complaint against the
RCL to the state RERA authority. Authority issued the various instructions to RCL, including instruction to
make afresh application for extension of registration for its ongoing projects which should be completed as
of now (considering the schedule) otherwise. RCL fails to comply with the instructions and didn’t make an
application for extension of registration as currently under moratorium (as a result of actions taken under
some other law, in force). RCL is charged with many offences under the Real Estate (Regulation and
Development) 2016 and penalties are also imposed including imprisonment of directors in few cases.
Despite various actions and efforts by allottees things were not moving and finally, allottees decide to move
an application under the Insolvency and Bankruptcy Code, 2016 (IBC 2016) for initiation of Corporate
Insolvency Resolution Process (CIRP) against RCL to recover of advances deposited by them (against the
allocation). They came to know that financial creditors already made the application for initiation of CIRP and
a resolution professional (Mr Raj) also appointed in the second week of April 2021.
Mr. Raj observed serious defaults that took place during the second and third quarter of 2020-2021 not only
causing insolvency to RCL but results in loss to creditors and some of the losses are potential (due to such
defaults). Mr Raj identified that directors of RCL knew that there was no reasonable prospect of avoiding the
commencement of a corporate insolvency resolution process in respect of RCL, still, they indulged and did
not exercise due diligence in minimizing the potential loss to the creditors of the corporate debtor.
Few employees and a couple of directors of RCL are probably part of some suspicious transactions, where
RCL is also a participant in the capacity of either benamidar or beneficial owner. The initiating officer decided
to inquire into the matter hence summoned them to attend his officer under section 19 and call upon the
information under section 21 of the Prohibition of Benami Property Transactions Act, 1988. Some of the
employees are scared (after summon served upon them) from the threat of legal proceeding and abstained,
decided to make an excuse for remain absent. Whereas few out of those, who were present before the
authority, failed to furnish information. Some others furnished the false information knowingly.
Multiple Choice Questions (2 Marks for each correct answer)
4.1 The prescribed Minimum Average Maturity Period (MAMP) is ______and ______in case of ECB raised
for repayment of rupee loans availed domestically for capital expenditure and other than capital
expenditure respectively.
(a) 3 and 3 years respectively
(b) 7 and 5 years respectively
(c) 10 and 7 years respectively
(d) 7 and 10 years respectively
Page 347 10
Compiled By: Shivam Gopal Mundad

4.2 What will the maximum penalty that may be levied by the Competition Commission of India, in case of
cartelisation?
(i) Three times of its profit for the last three preceding financial years
(ii) Ten percent of the average of turnover for the last three preceding financial years.
(iii) Three times of its profit for each year of the continuance of such agreement
(iv) Ten percent of its turnover for each year of the continuance of such agreement
(a) Higher of i or ii
(b) Maximum upto ii only
(c) Higher of iii or iv
(d) Higher of all i, ii, iii or iv
4.3 Offences under the Real Estate (Regulation and Development) Act, 2016 are-
(a) Cognizable, Non-bailable, and Compoundable
(b) Non-cognizable, Bailable, and Compoundable
(c) Cognizable, Non-bailable, and Non-compoundable
(d) Non-cognizable, Bailable, and Non-compoundable
4.4 Few among allottees take RCL to NCLT for initiation of Corporate Insolvency Resolution Process (CIRP)
against it. Advance payment against allotment by allottees to RCL shall be considered as
(a) Operational debt
(b) Financial debt
(c) None of the operational or financial debt, it is a mere advance
(d) Financial debt, if overdue for more than one year
4.5 For a total of 32 league matches USD 800,000 need to be remitted. For remittance of prize
money/sponsorship abroad, MCIL requires approval of
(a) Ministry of Finance (Department of Economic Affairs)
(b) Reserve Bank of India
(c) Ministry of HRD (Department of Youth Affairs and Sports)
(d) Ministry of External Affairs
Descriptive Questions
4.6 In the given case, can Mr. Raj took action against the directors of the corporate debtor for carrying out
the business in a negligent way, which may be resulting in loss to a creditor, explain in light of provisions
of IBC. (4 Marks)
4.7 If State Metro Rail Corporation moved to Competition Commission of India, considering what factors
commission will decide the nature is anti-competitive and has the appreciable adverse effect on
competition. In your opinion ‘is there anything anti-competitive’? (2 + 4 Marks)
4.8 If any person fails to furnish information under the Prohibition of Benami Property Transaction Act, 1988 then
what penalty can be imposed upon such person? Will it make any difference if there is reasonable cause,
due to which person from whom the information is called, fail to furnish the information? (3 Marks)
What will be the penalty for giving false information? Will it make any difference if any false document
is furnished? (2 Marks)

Page 348 11
Compiled By: Shivam Gopal Mundad

Case Study 5
Tamil-Nadu based Krishnan family owns cotton farms (where cotton is grown, both as a Kharif and as a Rabi
crop), and operates as Hindu Undivided Family. Mr. Raghuvaram Krishnan (Karta of HUF) is the 4 th
generation that engaged in growing and marketing cotton, the 5 th and 6th generation also actively participating
in operations. The quality of cotton produced in Krishnan’s farms is premium, hence exported to many
countries of Europe and the Middle-east apart from sale in the Indian market for domestic, industrial, and
surgical use. Mr. Raghuvaram Krishnan currently out of the country to attend the farming workshop.
Krishnan family supplied raw cotton to Kurl-Well Enterprises Limited (KWEL) which KWEL used for
mattresses and pillows. The financial health of KWEL was under the dark clouds since 2016. In the last month
of 2019, after considering an application of the financial creditors of KWEL, the Corporate Insolvency
Resolution Process has been initiated by National Company Law Tribunal (NCLT) and a moratorium was
ordered. Krishnan family being operational creditor, under the impression that Insolvency and Bankruptcy
Code, 2016 (IBC) is discriminatory and unfair to an operational creditor as compared to the financial creditor.
Mr. Nariman Izaz who is one of the directors at KWEL has been given a personal guarantee against the
borrowings of KWEL. He is of view that after the declaration of moratorium under section 14 of IBC, legal
action against him is barred too. Ms. Jaya Subramanian is another director and who has given a loan of INRs
80 Lakh to KWEL, which remained outstanding when the Corporate Insolvency Resolution Process was
ordered.
Krishnan family got a long-term forward procurement order from Dignity Surgical Products Limited (DSPL) for
its proposed factory. The price offered by DSPL is premium than prevailing in the market. In response to the
financial newspaper, DSPL clarifies the intent to ensure smooth supply and procure only high-quality cotton
(monitoring during cotton grown in farms), because it wishes to offer the best quality pure surgical cotton
(sterilized & free from bacteria) to its customers. DSPL is a famous brand for surgical products and currently
owned nearly 18% shares of the relevant market.
DSPL registered tremendous growth in the recent past, with organic and inorganic means. It currently
conducting due diligence to make an agreement of acquiring a stake in Alvira Naturals and Surgical Limited
(ANSL), which has import agreements and tie-up with foreign vendors from whom it procures necessary raw
material to manufacture double-layer N-95 marks and oximeter. The directors of both the companies decided
that both the companies shall operate independently after the acquisition. It is also decided that part of the
consideration will be paid in cash and a larger part in stocks.
The consultancy and advisory firm hired by DSPL estimates that the value of DSPL will be doubled, whereas
the EPS(earning per share) will improve significantly, P/E(price to earning ratio) expected to decline. After
the acquisition, the market share of both the company putting together will be around 32% of the relevant
market. Since this information found satisfactory, hence DSPL and ANSL enter into an agreement to give
effect to acquisition. The advisory firm suggests that since the threshold prescribed in section 5 of the
Competition Act 2002 has crossed, hence a notice needs to be furnished to the Competition Commission of
India (CCI) in the form as may be specified.
Soon after the acquisition, on 24 th June, 2020, ANSL placed an import order worth US$ 248,000 to
manufacture the oximeter and N-95 marks. The entire order received through a single shipment on 3 rd July,

2020.
It was found by the authority under the Prohibition of Benami Property Transaction Act, 1988, that the
Krishnan family illegally helps the DSPL to acquire the agricultural land for its factory. Since there are certain
restrictions on acquiring agricultural land for industrial purposes; hence DSPL acquired the land in n ame of
the Krishnan family and above mentioned long-term forward procurement order is only a way to oblige back.
Authority issue the notice to HUF to clear their role in acquiring, holding, or aid in acquiring and holding the
property. After considering the information and records furnished in front of the authority, it is easily concluded
that some of the property held in name of HUF and its members are meant for benefits of DSPL and
Page 349 12
Compiled By: Shivam Gopal Mundad

Consideration was also paid by funds provided by DSPL. Authority desires to confiscate the property so held
benami.
The youngest grandson (Mr. Murli) of Raghuvaram Krishnan moved to the States (US) in 2008 for his
graduation, which he completed in 2011. Afterward that he did master there and since then he was engaged
in research on the 7 essential plant micro-nutrient elements boron (B), zinc (Zn), manganese (Mn), iron (Fe),
copper (Cu), molybdenum (Mo), chlorine (Cl). His research helps in proving that these micro -nutrients
constitute in total less than 1% of the dry weight of most plants, basically, he tries to identify the correct
amount of Micro-nutrients which shall be used by farmers in agriculture according to different topography and
environment. He holds an Indian passport. He completed his doctorate there.
Mr. Murli decided to come back to India. After returning back, he joined ICAR (Indian Council of Agricultural
Research) as assistant director and deputed for a joint program with UN, posted at Rajaji Bhawan office of
ICAR at Besant Nagar, Chennai. He booked a flat for himself there in Chennai at Ramaniyam Advaitham. He
was told by promoter ‘Ramaniyam Real Estate Builders’ that he (Murli) shall participate in the formation of an
association of the allottees. He has to pay 8% as a deposit after signing the agreement to sell.
Mr. Murli in order to transfer the deposit money to the promoter and open a salary account for him visited
SBI’s Adyar Branch at Kasturba Nagar, Chennai. There he came to know that, Aadhaar card is needed to
open a bank account and to perform bank transaction (which he don’t possess as he moved out of India when
a law requiring Aadhaar was not promulgated and he is back in India just a month back). Bank denied Mr.
Murli to transact in the absence of furnishing the Aadhaar Card as proof of identity.
Multiple Choice Question (2 Marks for each correct answer)
5.1 Under the Prohibition of Benami Property Transaction Act, 1988, if an order in respect of any property
has been passed holding such property to be a benami property then which authority a mong followings
can pass the order of confiscation?
(a) Initiating Authority
(b) Adjudicating Authority
(c) Administrator
(d) Approving Authority
5.2 Under the Real Estate (Regulation and Development) Act 2016, in absence of local law the association
of allottees shall be formed within a period of three months from the-
(a) Issue of the completion certificate.
(b) Issue of the occupancy certificate.
(c) Majority of allottees having booked their plot or apartment or building, as the case may be.
(d) Majority of allottees occupied their plot or apartment or building, as the case may be.
5.3 In the given case, which one out of the following mentioned forms shall be filed under section 6 of the
Competition Act 2002 with the Competition Commission of India?
(a) Form I
(b) Form II
(c) Form III
(d) Form IV
5.4 Notice in case of Krishnan Family (HUF) under the Prohibition of Benami Property Transaction Act 1988,
can be served on-
(a) Karta only

Page 350 13
Compiled By: Shivam Gopal Mundad

(b) Karta or any male member of HUF


(c) Karta, but if Karta is not available (due to any reason) then to any other member of HUF
(d) Karta or any member of HUF
5.5 As a financial creditor, whether Ms. Jaya can be a part of the Committee of Creditors (CoC) after she
submitted her claim in ‘Form C’.
(a) Yes, she can be a part of the CoC as she had given a loan to KWEL
(b) Yes, she can be a part of the CoC, if Interim Resolution Professional permitted her despite the fact
that she was a director of KWEL.
(c) Yes, she can be a part of the CoC, if Interim Resolution Professional sought permission of a
minimum of 75% of the shareholders of the company carrying voting rights.
(d) No, she is a director of KWEL, hence can’t be a part of the CoC.
Descriptive Questions
5.6 Is the Insolvency and Bankruptcy Code, 2016 (IBC) unconstitutional in the manner it is discriminatory
and unfair to an operational creditor as compared to the financial creditor ? (3 Marks)
5.7 Can the bank deny Mr. Murli to transact in the absence of furnishing the Aadhaar Card as proof of
identity? What remedy is left with Mr. Murli for purpose of verification by the bank? (3 Marks)
5.8 Is the credence of Mr. Nariman Izaz valid? Support your opinion in detail. (6 Marks)
5.9 Till what date, remittances against imports should be completed by ASNL? (3 Marks)

Page 351 14
Compiled By: Shivam Gopal Mundad

Test Series: April, 2021


MOCK TEST PAPER
FINAL (New) GROUP – II
PAPER – 6D: ECONOMIC LAWS
Suggested answers
Case Study 1
1.1 (d)
1.2 (a)
1.3 (c)
1.4 (d)
1.5 (d)
1.6 The Real Estate (Regulation and Development) Act 2016 (herein-after RERA) under its section 14
provides the adherence to sanctioned plan and project specifications by the Promoter.
Sub-section 1 provides the proposed project shall be developed and completed by the promoter
following the sanctioned plans, layout plans and specifications as approved by the competent
authorities.
Sub-section 2 has an overriding effect and its clause (i) provide the promoter shall not make any
additions and alterations in the sanctioned plans, layout plans and specifications and the nature of
fixtures, fittings and amenities described therein in respect of the apartment, plot or building, as the case
may be, which are agreed to be taken, without the previous consent of that person who agrees to take
one or more of the said apartment, plot or building, as the case may be.
Here it is worth noting that the promoter may make such minor additions or alterations as may be
required by the allottee, or such minor changes or alterations as may be necessary due to architectural
and structural reasons duly recommended and verified by an authorised Architect or Engineer after
proper declaration and intimation to the allottee.
For this clause, "minor additions or alterations" excludes structural change including an addition to the
area or change in height, or the removal of part of a building, or any change to the structure, such as
the construction or removal or cutting into of any wall or a part of a wall, partition, column, beam, joist,
floor including a mezzanine floor or other support, or a change to or closing of any required means of
access ingress or egress or a change to the fixtures or equipment, etc.
Since in the given case certain structural changes (in the sanctioned plan of the project) relating to
height is required, hence the changes in sanctioned plan are not minor in nature.
Further clause (ii) of Sub-section 2 provides the promoter shall not make any other alterations or
additions in the sanctioned plans, layout plans and specifications of the buildings or the common areas
within the project without the previous written consent of at least two-thirds of the allottees, other than
the promoter, who have agreed to take apartments in such building.
It is worth noting here that for this clause, the allottees, irrespective of the number of apartments or
plots, as the case may be, booked by him or booked in the name of his family, or in the case of other
persons such as companies or firms or any association of individuals, etc., by whatever name called,
booked in its name or booked in the name of its associated entities or related enterprises, shall be
considered as one allottee only.
In the given case all 120 units’ subscribed/booked by allottees except 2 Flats kept by Mr. Vivek
(promoter). Out of 118, Mr. Tirlochan Negi booked 3 floors one in his own name, an other one in the
name of his daughter in law and the third one in name of his company, whereas Mr. Dabral booked a
Page 352 1
Compiled By: Shivam Gopal Mundad

flat and a villa (both in his name); rest all allottee booked one unit each. Hence the total number of
allottee for purpose of section 14(2)(ii) is 115 (118-2-1) considering Mr Tirlochan (3) and Mr Dabral (2)
as a single allottee each. At least 2/3 allottee shall be 77 (2/3rd of 115 – round up to next whole integer),
whose previous written consent is required; before making changes to sanctioned plan.
Hence the opinion of Mr. Vivek in the context of the provision contained in RERA, 2016 is untenable and
incorrect.
1.7 Investments are considered as capital account transactions, hence governed by section 6 of the Foreign
Exchange Management Act, 1999 read with The Foreign Exchange Management (Permissible Capital
Account Transactions) Regulations 2000 (herein-after regulations).
Clause (b) of regulation 4 of such regulations describe the prohibitions. Although regulation 4 (b) (iv)
provides no person resident outside India shall invest in India, in any form, in any company or partnership
firm or proprietary concern or any entity, whether incorporated or not, which is engaged or proposes to
engage in real estate business. But explanation 1 provides a certain exclusion from real estate business,
explanation read as ‘for this regulation, 'real estate business shall not include development of townships,
construction of residential/commercial premises, roads or bridges and real estate investment trusts
(REITs) registered and regulated under the SEBI (REITs) Regulations, 2014.
Hence repatriation of funds in India as Investment into the real estate project (construction of residential
apartments) in Kharar (Mohali, Kharar) can be seen as a permissible capital account transaction under
clause (a) to schedule II of regulations.
1.8 As per clause (a) to regulation 8 of the Foreign Exchange Management (Acquisition and Transfer of
Immovable Property in India) Regulations, 2018, a person referred to in sub-section (5) of Section 6 of
the Act, or his successor shall not, except with the general or specific permission of the Reserve Bank,
repatriate outside India the sale proceeds of any immovable property referred to in that sub -section.
Whereas section 6(5) of the Foreign Exchange Management Act, 1999 provides a person resident
outside India may hold, own, transfer or invest in any immovable property situated in India if such
property was acquired, held or owned by such person when he was resident in India or inherited from a
person who was resident in India.
Since in the given case father of Mr. Onkar acquired the property through inheritance from his father
who was resident in India, hence fall within the scope of section 6 (5). Therefore, with the permission of
RBI, he can repatriate the sale proceed of ancestral property inherited by him to Canada from India.
Case Study 2
2.1 (a)
2.2 (b)
2.3 (d)
2.4 (b)
2.5 (c)
2.6 NRE account stands for Non-Resident External Account. This account facilitates non-resident to park
their foreign earnings to transfer that to India.
NRO account stands for Non-Resident Ordinary Account. This account also facilitates non-resident to
manage their income (such as rent, dividend, pension, interest, etc.) that is earned in India.
Basis NRE Account NRO Account
Purpose It is an account of an NRI to It is an account of an NRI to manage the
transfer foreign earnings to India income earned in India
Taxability Interest earned is tax free Interest earned is taxable
Page 353 2
Compiled By: Shivam Gopal Mundad

Repatriation Can repatriate Can repatriate the interest amount, the


principle amount can be repatriated
only up to USD 1 million in a financial
year
Joint Account Can be opened by two NRIs Can be opened by an NRI along with an
Indian citizen or another NRI
Deposits and Can deposit in foreign currency, Can deposit in foreign as well as Indian
Withdrawals and withdraw in Indian currency currency, and withdraw in Indian
currency
Exchange Rate Prone to risk Not prone to risk
Risk
Joint Account- NRE account can be jointly opened and operated by two NRIs only, whereas an NRO
account can be opened by an NRI along with an Indian citizen or another NRI; hence M/s An na can
open and operate a joint NRO account with his father.
2.7 Section 5 of the Competition Act 2002 provides the thresholds beyond which an acquisition will result in
combination. Vide notification number S.O. 675(E) dated 4th March 2016 the threshold (w.e.f. 4th March
2016) under section 5 shall be as tabled below;
Parties/enterprises after combination have Enterprises Level Group Level
Joint Assets In India Rs. 2,000 Cr Rs. 8,000 Cr
Joint Turnover Rs. 6,000 Cr Rs. 24,000 Cr
Joint Total Assets In India or US$ 1000 Million US$ 4000 Million
Minimum Indian Component Outside Rs. 1000 Cr Rs. 1000 Cr
Joint Total Turnover US$ 3000 Million US$ 12000 Million
Minimum Indian Component Rs. 3000 Cr Rs. 3000 Cr
In the given case TTPL had a global turnover of INRs 2735 crores in the immediately preceding year
with an asset base of worth INRs 960 crores in India. Whereas TTTPL had a turnover of US$ 3400
million in the immediately preceding year with an asset base of US$ 1200 million across the globe
including asset worth INRs 80 crores in India, this can be summarised as;
Joint Total Assets In India or US$ 1200 Million + (INRs 960 Crores)
Including Indian Component Outside Rs. 1040 Cr (960+80)
Joint Total Turnover US$ 3400 Million
Including Indian Component Rs. 2735 Cr + (Revenue of TTTPL in India is not
given)
Since the joint total assets are more than US$ 1000 million including asset in India more than INRs 1000
crores hence the acquisition of TTTPL by TTPL results in the formation of combination under
Competition Act 2002.
No, there is not complete prohibition; but there is a restriction on the formation of combination
under the Competition Act 2002. Section 6 of the Act provides no person or enterprise shall enter into
a combination that causes or is likely to cause an appreciable adverse effect on competition within the
relevant market in India and such a combination shall be void.
2.8 Section 45 of the Prevention of Money Laundering Act 2002 (here-in-after act), has the title ‘offence to
be cognizable and non-bailable.

Page 354 3
Compiled By: Shivam Gopal Mundad

Through Finance (No. 2) Act 2019 an explanation is inserted in section 45 that the expression "Offences
to be cognizable and non-bailable" shall mean and shall be deemed to have always meant that all
offences under this Act shall be cognizable offences and non-bailable offences notwithstanding anything
to the contrary contained in the Code of Criminal Procedure, 1973, and accordingly, the officers
authorised under this Act are empowered to arrest an accused without a warrant, subject to the fulfilment
of conditions under section 19 and subject to the conditions enshrined under this section.
Further sub-section 1 to section 45 provides notwithstanding anything contained in the Code of Criminal
Procedure, 1973 (2 of 1974), or any other provision of this Act, no police officer shall investigate into an
offence under this Act unless specifically authorised, by the Central Government by a general or special
order, and, subject to such conditions as may be prescribed.
Section 19(1) of the act provides If the Director, Deputy Director, Assistant Director, or any other officer
authorised in this behalf by the Central Government by general or special order, has based on material
in his possession reason to believe (the reason for such belief to be recorded in writing) that any person
has been guilty of an offence punishable under this Act, he may arrest such person and shall, as soon
as may be, inform him of the grounds for such arrest.
It can be concluded that the officer is empowered to arrest without a warrant if the conditions entailed
in section 19 and 45 are fulfilled. But he can’t investigate unless authorised by the central government.
Hence the argument of Ms Rita, Captain of the ship, and Director that the officer is not empowered to
arrest them without a warrant is not legally tenable; whereas the argument of the captain of the ship
and Director that the officer is not empowered to investigate them without prior authorisation from the
government is legally tenable.
Students are advised to note;
The second proviso to section 45 (1) of the act provided that the Special Court shall not take cognizance of
any offence punishable under section 4 except upon a complaint in writing made by the Director; or any officer
of the Central Government or State Government authorised in writing in this behalf by the Central Government
by a general or a special order made in this behalf by that Government.
Cognizance of offence by the court (to initiate trial) shall not be confused with the cognizable nature of the
offence for the arrest of the accused by designated authorities under section 19 of the Act.

Case Study 3
3.1 (d)
3.2 (d)
3.3 (c)
3.4 (c)
3.5 (d)
3.6 As per explanation (b) to section 4 of the Competition Act, 2002, the predatory price means the sale of
goods or provision of services, at a price which is below the cost, as may be determined by regulations,
of production of the goods or provision of services, to reduce competition or eliminate the competitors.
In the given case it was mentioned that due to the low cost of production, PRS was able to offer a deep
discount to its customer, especially first-time customers.
The Standard portable saw machine of 16 th Inches which PRS was able to manufacture under INRs
3500/- and sold for in a range of INRs 3800-4000, other competitors able to manufacture the same in
and around INRs 4000/-

Page 355 4
Compiled By: Shivam Gopal Mundad

Since the price offered by PRS is in the range of INRs 3800-4000 which is not less than the cost
determined under the regulation is INRs 3800/- hence the PRS is not abusing the dominance through
predatory pricing of their product.
Further as per section 19 (4) of the Competition Act, 2002, The Commission while inquiring whether an
enterprise enjoys a dominant position or not under section 4, shall have due regard to all or any of the
following factors, namely:—
(a) Market share of the enterprise;
(b) Size and resources of the enterprise;
(c) Size and importance of the competitors;
(d) Economic power of the enterprise including commercial advantages over competitors;
(e) Vertical integration of the enterprises or sale or service network of such enterprises;
(f) Dependence of consumers on the enterprise;
(g) Monopoly or dominant position whether acquired as a result of any statute or by virtue of being a
Government company or a public sector undertaking or otherwise;
(h) Entry barriers including barriers such as regulatory barriers, financial risk, high capital cost of entry,
marketing entry barriers, technical entry barriers, economies of scale, high cost of substitutable goods
or service for consumers;
(i) Countervailing buying power;
(j) Market structure and size of market;
(k) Social obligations and social costs;
(I) Relative advantage by way of the contribution to the economic development, by the enterprise enjoying
a dominant position having or likely to have an appreciable adverse effect on competition;
(m) Any other factor which the Commission may consider relevant for the inquiry.
Students are advised to note;
Dominance is not prohibited, abuse of dominance is prohibited. Even in the given case, it seems that PRS
holds dominance over the relevant market in the relevant product segment, it can’t be inferred that it violates
any of the provisions of the Competition Act 2002 regarding the prohibition of abuse of dominance.
3.7 Clause viii to paragraph 4.2 of the master direction No.5 (dealing with External Commercial
Borrowings) dated 26 th March 2019, contain the negative list, for which the ECB proceeds cannot be
utilised.
Further clause v to paragraph 4.2 of same directions, which deals with minimum average maturity period
contains certain exception to negative list of end uses contained in clause viii.
(i) As per item c in the negative list, equity investment is not permitted. Hence any form of equity
investment be it direct or indirect (through the purchase of goodwill) is not permitted. Even ECB
can’t be availed of for making a contribution in an LLP.
(ii) Reading both the clauses (v and viii) together it is observed that raising and use of ECB for
repayment of Rupee loans is permitted in some cases.
ECB can be raised and used for repayment of that Rupee loans which was availed domestically, for
purposes both capital expenditure and other than capital expenditure; the only difference is minimum
average maturity period i.e. 7 and 10 years in case of capital expenditure and other than capital
expenditure respectively (provided ECB is not raised from foreign branches/subsidiaries of Indian
banks).
ECB can’t be raised and used for repayment of other than domestically availed Rupee loans.
Page 356 5
Compiled By: Shivam Gopal Mundad

It is worth here to note that ECB can also be raised and used with a minimum average maturity period
of 5 years for repayment of Rupee loans.
3.8 (i) No, an unregistered allottees association can file a complaint with the authority; although individual
allottee can make a complaint in their individual capacity.
It is worth here to quote section 31 (1) of the Real Estate (Regulation and Development) Act 2016, it
read as any aggrieved person may file a complaint with the Authority or the Adjudicating Officer, as the
case may be, for any violation or contravention of the provisions of this Act or the rules and regulations
made thereunder, against any promoter, allottee or real estate agent, as the case may be.
Further explanation to such sub-section provides person shall include the association of allottees or any
voluntary consumer association registered under any law for the time being in force.
Students are also advised to note
As per section 11 (4) (e) promoter shall enable the formation of an association or society or co-operative
society, as the case may be, of the allottees, or a federation of the same, under the laws applicable
(ii) As per section 6 of the Real Estate (Regulation and Development) Act, 2016 the registration granted
under section 5 may be extended by the Authority on an application made by the promoter, due
to force majeure (a case of war, flood, drought, fire, cyclone, earthquake or any other calamity caused
by nature affecting the regular development of the real estate project)
Authority may in reasonable circumstances, without default on the part of the promoter, based on the
facts of each case, and for reasons to be recorded in writing, extend the registration granted to a project
for such time as it considers necessary, which shall, in aggregate, not exceed a period of one year
No application for extension of registration shall be rejected unless the applicant has been given an
opportunity of being heard in the matter.
Hence, Shiva Estate and Realtors can make an application under section 6 for extension of
registration.
Case study 4
4.1 (d)
4.2 (c)
4.3 (b)
4.4 (b)
4.5 (c)
4.6 Sub-section 3 inserted to section 66 of the Insolvency and Bankruptcy Code, 2016 (here-in-after the
code) by the Insolvency and Bankruptcy Code (Second Amendment) Act, 2020 dated 05.06.2020 (w.e.f
05.06.2020), which says notwithstanding anything contained in this section, no application shall be filed
by a resolution professional under sub-section (2), in respect of such default against which initiation of
the corporate insolvency resolution process is suspended as per section 10A.
Sub-section 2 to section 66 provides on an application made by a resolution professional during the
corporate insolvency resolution process, the Adjudicating Authority may by an order direct that a director
or partner of the corporate debtor, as the case may be, shall be liable to make such contribution to the
assets of the corporate debtor as it may deem fit, if (a) before the insolvency commencement date, such
director or partner knew or ought to have known that the there was no reasonable prospect of
avoiding the commencement of a corporate insolvency resolution process in respect of such
corporate debtor; and (b) such director or partner did not exercise due diligence in minimising the
potential loss to the creditors of the corporate debtor.

Page 357 6
Compiled By: Shivam Gopal Mundad

Here it worth noting that filing of the application for initiation of corporate insolvency resolution process
of a corporate debtor under section 7, 9 and 10 of the Code was suspended (under section 10A), who
made the default/s during the period from 25 th March 2020 till 24 th March 2021*. Proviso to section 10A
provides that no application shall ever be filled for the defaults occurring during said period (from 25 th

March 2020 till 24 th March 2021).


*Through SO 4638(E) dated 22 nd Dec 2020, application of section 10A extended for another 3 months
beyond 25 th Dec 2020.
Since the default occurred during the second and third quarter of 2020-2021 (falls in between the
suspension period), hence resolution professional can’t apply to adjudicating authority; due to the effect
of section 66(3) of the IBC 2016.
4.7 As per section 19 (3) of the Competition Act, 2002, the Commission shall, while determining whether
an agreement has an appreciable adverse effect on competition under section 3, have due regard to
all or any of the following factors, namely
(a) Creation of barriers to new entrants in the market;
(b) Driving existing competitors out of the market;
(c) Foreclosure of competition by hindering entry into the market;
(d) Accrual of benefits to consumers;
(e) Improvements in production or distribution of goods or provision of services; or
(f) Promotion of technical, scientific and economic development by means of production or
distribution of goods or provision of services.
Facts given in the case are very similar to facts of case B. P. Khare, Principal Chief Engineer, South
Eastern Railway vs. M/s Orissa Concrete and Allied Industries Ltd. and Ors., wherein A tender
notice was floated by South Eastern Railway for procurement of Anti-Theft Elastic Rail Clips with Circlips
from RDSO approved firms. Responses were submitted by 29 firms, the rate quoted by most of the firms
was @ 66.50 (all-inclusive). The quantity quoted by each of the firms was far less than 50% of the total
tender quantity. It is also alleged that the quoted rate was about 10% higher than the neighbouring
Railways' last purchase rate.
Commission prima-facie noted that the rate was inclusive of freight. Bidders were located across the
country, the cost of freight for supplying the product from different parts of the coun try could not have
been the same hence identical rates, indicative of meeting of minds.
The Director-General during scrutiny of the case found that all the 29 firms have quoted identical bids
which were in the range of Rs.66.49 to Rs.66.51. Further bid documents revealed that the 19 firms, 4
firms, and 2 firms respectively had similar handwriting in which the prices were quoted in their respective
bid documents. 17 bids are supported by a cover letter and the format of the cover letter was the same
in all such 17 cases.
Commission held that conduct of parties was amounting to bid-rigging. Commission issued cease and
desist order.
It is worth mentioning here that as per explanation to sub-section 3 to section 3, bid-rigging means
any agreement, between enterprises or persons referred to in sub-section (3) engaged in identical or
similar production or trading of goods or provision of services, which has the effect of eliminating or
reducing competition for bids or adversely affecting or manipulating the process for bidding.
Since the facts in the given case strongly indicate collusive bidding, such as the rate quoted by
most of the firms was the same per unit; and on an all-inclusive basis despite they from different part of
India (how the price can be same, at least the different amount of the freight will result in the difference

Page 358 7
Compiled By: Shivam Gopal Mundad

of price quoted). The quoted rate is about 25% higher than, the rate at which procurement of similar
battery sets (exactly same specifications) was done by National Capital’s Metr o Rail Corporation very
recently (means by collusion they wish to gain in term of charging a high price). It is also observed that
the quantity quoted by each of the firms was far or less near to 20% of the total tender quantity (so that
all gets the opportunity to deliver the product and make money). Hence such collusive understanding
(meeting of minds) among the bidder, followed by an act of bid-rigging; falls under the scope of
the horizontal anti-competitive agreement.
Students are advised to note
A cease and desist order is issued when a court, tribunal, or quasi-judicial authority intend to direct
someone to stop engaging in illegal activity and not to restart it.
4.8 As per sub-section 1 to section 54A of the Prohibition of Benami Property Transaction Act 1988, any
person who fails to furnish the information as required under section 21 (or comply with summons
issued under section 19(1) shall be liable to pay the penalty of 25000/- for each such failure.
Sub-section 2 to section 54A provides such penalty shall be imposed by the authority who called for the
information. Further sub-section 3 provides such penalty shall be imposed only after the opportunity of
being heard given. Proviso to section 54A acts as a safeguard from the imposition of penalty in cases,
where good and sufficient information prevented the person from furnishing the information.
Further, as per section 54 of the act, any person who is required to furnish information under this Act
knowingly gives false information to any authority or furnishes any false document in any
proceeding under this Act, shall be punishable with rigorous imprisonment for a term which shall
not be less than six months but which may extend to five years and shall also be liable to fine
which may extend to ten percent of the fair market value of the property.
Case Study 5
5.1 (b)
5.2 (c)
5.3 (b)
5.4 (d)
5.5 (d)
5.6 Based upon the judgment pronounced in the case of Swiss Ribbons Private Limited & Anr. Vs. Union
of India & Ors. Insolvency and Bankruptcy Code, 2016 (IBC) is constitutional in entirety and not
discriminatory and unfair to an operational creditor as compared to the financial creditor.
The court held that financial creditors are clearly different from operational creditors and therefore, there
is obviously an intelligible differentia between the two which has a direct relation to the objects sought
to be achieved by the Code. Court also held that the excessive power given to the Committee of
Creditors (CoCs) is controlled through approval/rejection of the plan with the large majority (rather a
simple majority) and NCLT and thereafter NCLAT can set aside the arbitrary decisions of CoCs.
The court held that since there is a difference in the relative importance of two types of debts when it
comes to objects sought to be achieved by the insolvency code, hence article 14 of the Constitution of
India (equality before the law) does not get infracted.
5.7 As per sub-section 1 to section 11A of the Prevention of Money Laundering Act 2002, every reporting
entity shall verify the identity of its clients and the beneficial owner, by—
 Authentication under the Aadhaar (Targeted Delivery of Financial and Other Subsidies, Benefits and
Services) Act, 2016 if the reporting entity is a banking company; or

Page 359 8
Compiled By: Shivam Gopal Mundad

 Offline verification under the Aadhaar (Targeted Delivery of Financial and Other Subsidies, Benefits and
Services) Act, 2016; or
 Use of a passport issued under section 4 of the Passports Act, 1967; or
 Use of any other officially valid document or modes of identification as may be notified by the Central
Government in this behalf.
Further as per sub-section 3 to section 11A of the Prevention of Money Laundering Act 2002, the use of
modes of the identification under sub-section (1) shall be a voluntary choice of every client or
beneficial owner who is sought to be identified and no client or the beneficial owner shall be denied
services for not having an Aadhaar number.
No, the bank shall not deny Mr. Murli to transact in the absence of furnishing an Aadhaar Card as proof
of identity.
Since Mr. Murli holds an Indian passport (which is obviously issued under section 4 of The Passport Act
1967), hence can use his passport as proof of his identity, for purpose verification of identity at the
bank.
5.8 Mr. Nariman Izaz, hold credence that section 14 of the Insolvency and Bankruptcy Code, 2016 (IBC)
would apply to the personal guarantor as well, as a result of which proceedings against the personal
guarantor and his property would have to stay if moratorium declared.
Clause (b) section 14 (3) of the Insolvency and Bankruptcy Code, 2016, read as the provisions of sub-
section (1) shall not apply to a surety in a contract of guarantee to a corporate debtor. It important here
to note that sub-section (1) gave power to adjudicating authority to declare a moratorium.
The credence of Mr. Nariman Izaz seems invalid in light of the pronouncement given by the apex court in
Civil Appeal No. 3595 of 2018, State Bank of India vs. V. Ramakrishnan. The apex court consider the
following facts importantly -
- Report of Insolvency Law Committee dated 26.03.2018 clarified that the period of moratorium under
section 14 is not applicable to personal guarantors,
- Amendment made to the provision of section 14 (substituted vide act 26 of 2018 enforced w.r.e.f. 6 th
June 2018) that clearly states that the moratorium period envisaged in section 14 is not applicable to
a personal guarantor to a corporate debtor.
Since, section 14 (moratorium) of the IBC is not applicable to the personal guarantor, hence the credence of
Mr. Nariman Izaz (that after the declaration of moratorium under section 14, legal action against him is barred
too) is not valid.
Students are also advised to note;
Since in the civil appeal quoted above, question in front of the apex court is much border than what
we are asked to answer here for academic purposes; hence court also observed and record following
in its order (not that much relevant for the answer, but important to note for better understanding)
The Hon'ble Supreme Court first considers the fact that different provisions of the IBC are applicable to the
insolvency of different categories of persons. Section 96 and 101 of the IBC provide for separate provision
for a moratorium for the personal guarantor. Whereas section 14 deals with corporates
Court also observed that different provisions of law brought into effect on different dates and some of the
provisions were not yet enforced (on the date of the judgment). Provisions pertaining to sections 96 and 101
have not been brought into force.

Page 360 9
Compiled By: Shivam Gopal Mundad

5.9 As per para B.5.1 (i) of the ‘Master Direction on Import of Goods and Services’ dated 1 st January 2016

(as amended from time to time), in terms of the extant regulations, remittances against imports should
be completed by not later than six months from the date of shipment, except in cases where amounts
are withheld towards the guarantee of performance, etc.
Vide A.P. (DIR Series) Circular No.33 dated 22 nd May 2020, in view of the disruptions due to the outbreak
of COVID-19 pandemic, with effect from 22 nd May 2020, the time period for completion of remittances
against normal imports (except in cases where amounts are withheld towards the guarantee of
performance, etc.) has been extended from six months to twelve months from the date of shipment
for such imports made on or before 31 st July 2020.
Since the entire order was received through a single shipment on 3 rd July 2020, hence the 12 months

shall be completed on 2 July 2021. So the remittances against imports should be completed by ASNL
nd

within 2 nd July 2021.

Page 361 10
Test Series: October 2019
MOCK TEST PAPER 1
FINAL (NEW) COURSE: GROUP – II
PAPER – 6D: ECONOMIC LAWS
Attempt any four out of five case study based questions.
Each Case Study carries 25 Marks.
Time Allowed – 4 Hours Maximum Marks – 100
Case Study 1
Way back in November, 2011, Mr. Hariharan Reddy, a senior professor of Biology in the University of
Hyderabad, booked a 3BHK apartment in Royal Golf Burg - an impressive and integrated housing project
proposed to be developed by a reputed builder popularly known as Raj Group. The project was large enough
to accommodate 1200 fully-furnished apartments of different sizes spread out in 15 towers; each tower having
80 apartments. In addition, a Golf Course and a three storeyed mall was also to be developed. This project
was to come up near Shankar pally Road, Hyderabad, a non-polluting and posh area having all the facilities
in close vicinity including ultra-modern cinema halls, markets, schools, colleges, hospitals, etc.
As mentioned above, the Raj Group which undertook to develop Royal-Golf-Burg consisted of Dhanraj and
his younger brother Yuvraj, a well-known figure of Hyderabad. Both the brothers were the directors of Eklavya
Estates Private Limited (EEPL) which had registered office at Gachibowli, Hyderabad. EEPL owned the plot
of land where the proposed housing project including mall was to be developed.
The gated residency with a nice and peaceful environment as provided by Royal Golf Burg was meant for
golf lovers who wanted to live a sleek and sporty lifestyle by making golf playing a routine. The glamour which
attracted Mr. Reddy the most was that every apartment owner after occupation would feel the ownership of
the golf course due to its strategic location vis-à-vis each apartment.
The management team of EEPL comprised seasoned architects and professionals who had, in the past, made
luxurious homes possible for every home-buyer and the team was considered to be a dedicated one having
will and honesty as its strong pillars which could build integrated properties with excellent infrastructure and
services. The builders had completed several giant opulent projects in Guwahati, Mumbai and Bhopal earlier.
In case of Royal Golf Burg, the company was to give delivery of fully furnished apartments by December,
2017. Construction cost including the cost of the land was valued at around Rs. 800 crores.
The current project, however, missed the deadline of December, 2017 and on the date of delivery, it was
noticed that only six towers were completed; but the apartments in those towers were yet to be furnished.
Other three towers had been constructed with a skeletal structure. In other six towers, only the foundation
and a negligible wall work had been completed. In other words, the construction work was just at the initial
stage and nothing more than that. However, the construction of Mall was almost complete. This angered the
home-buyers including Mr. Reddy a lot but their repeated visits to the office of promoters did not evoke any
positive response. Six months passed without any significant happening. Nothing was done to furnish the
already constructed apartments or to develop the other nine towers. Disappointed, Mr. Reddy and others
approached Telangana State RERA authorities for redressal of their grievances including filing of complaints
regarding non-delivery of apartments.
Telangana State RERA, after detailed inquiry, found that there were several financial irregularities together
with diversion and siphoning of funds. More than two hundred shell companies were floated in the names of
peons and drivers to divert money. Further, unaccounted money worth crores of rupees was invested in
various other housing projects floated by the Raj Group which sold these flats at throw-away prices on paper
but received black money in cash which was laundered through various shell companies operated by the Raj
Group. This attracted the provisions of Prevention of Money Laundering Act, 2002.

© The Institute of Chartered Accountants of India


t.me/cafinal6d - telegram link
Page 362
In case of 250 apartments built in the six towers, double allotment was also detected where the apartments
were allotted to the persons more than their entitlement at a very nominal amount. It included the person
himself, the spouse or dependent children; and almost in all cases such persons were found to be connected
with the promoters. The double allotment deprived the genuine home-buyers who had parted with their hard-
earned money from getting even the deserving allotment of apartments despite paying around 80% of the
cost. The double allotment was considered to be a form of unfair practice in which the promoters were
involved. Enquiry by the RERA Authority also revealed that ten gardeners and drivers of Raj Group who had
no means of paying the price of ten apartments were allotted the flats though consideration came from the
Raj Group itself.
A show-cause notice was issued by Telangana State RERA authorities to the developers asking them to
provide a satisfactory response within a period of 30 days from the date of the notice as to why the project
should not be de-registered. The response given by the directors, however, was dismal, lacked substance
and was not at all satisfactory.
However, de-registration of a building project is not an ideal choice for the authorities keeping in view the
larger interests of the stakeholders as well as the nation as a whole and it is resorted to only when all the
possible avenues of reaching a comfortable and plausible solution are shut. Therefore, as a last attempt
before going for de-registration, RERA authorities in the interest of the allottees, permitted the developer
EEPL to continue with the project and complete it in the next one year at the most subject, however, to the
payment of a fine equivalent to 10% of Rs.800 crores within next thirty days.
However, the developers, seemed to be not serious at all so far as completion of housing project was
concerned. They did not make use of this golden opportunity; thus, letting the project slip out of their hands.
Citing insufficiency of the funds, they did not cough out the required fine of Rs. 80 crores within next thirty
days and therefore, the Telangana State RERA authorities were forced to de-register the project and an order
to this effect was passed. Thereafter, finally the project was taken over by RERA Authorities.
After take-over, RERA authorities with the concurrence of the State Government imposed various restrictions
and controls on the project and the developer. These included:
• Freezing of various bank accounts due to which the developer was not allowed to make any payment or
withdraw from these accounts without the authority’s approval.
• Debarring the developer EEPL from accessing the web-site of RERA in relation to the project.
• RERA offices in other States and Union Territories were given information about such a revocation.
• As a part of name and shame, the name of the EEPL was mentioned in the list of the defaulters along
with the photographs of Dhanraj and Yuvraj and also relevant information about the case was displayed.
The officials of Telangana State RERA opined that after de-registration, there were several options before
them to solve the issue in favour of home-buyers. The authority could give the first right of completion of the
project to the home-buyers. If case the buyers were not in a position to do so by pooling their resources
together and required RERA to supervise the development work which could be undertaken by another trust -
worthy developer, then the RERA Authority could take steps to develop mechanism to supervise the project.
In case there was not enough money left in the project fund, the Authority could also start proceedings to
recover the diverted funds from the EEPL and it could also explore other possibilities to complete the project
if the home-buyers so wished.
The response from the Home-buyers’ Association of Royal Golf Burg was positive and therefore, a conciliatory
committee was formed. President and Secretary of the Home-buyers’ Association were nominated to the
committee and RERA then appointed Mr. Yudhister Pal, a retired IAS Officer as a conciliator to supervise the
operations of the committee. Another developer Uttam Constructions Private Limited was given the charge
to complete the project within one year under the supervision of RERA Authority represented by
Mr. Yudhishtra Pal.

© The Institute of Chartered Accountants of India


t.me/cafinal6d - telegram link
Page 363
RERA ordered that all money realised from the sale of Mall as well as remaining dues to be given by the
home-buyers would flow into an ‘Escrow Account’ opened solely for the construction of the project and Mr.
Yudhister would release the funds only with the consent of the President and Secretary of Home-buyers’
Association. Proceedings to recover the diverted funds from the EEPL were also started. It was hoped that
the project would be completed as per the new schedule.
Multiple Choice Questions (MCQs) [2 Marks each]
1. RERA Authority passed the order of de-registration of Royal Golf Burg Project against its promoter
EEPL. Within how many days of receipt of order of de- registration, EEPL, as aggrieved party, can file
an appeal with the concerned Real Estate Appellate Tribunal?
(a) 30 days
(b) 45 days
(c) 60 days
(d) 120 days
2. The promoter EEPL did not complete the Royal Golf Burg Project within the projected time-frame as
shared through declaration with RERA Authorities while seeking registration under Section 4. For such
contravention, how much penalty the EEPL is liable to pay?
(a) Penalty may extend up to 2% of the project cost
(b) Penalty may extend up to 5% of the project cost
(c) Penalty may extend up to 10% of the project cost
(d) None of the above
3. In case of Royal Golf Burg project, it is seen that it was de-registered by the RERA Authorities due to
various irregularities. Choose from the given options as to who shall have the first right of refusal for
carrying out the remaining development works in case of such revocation of registration.
(a) Home-buyers’ Association of Royal Burg Golf
(b) EEPL
(c) Mr. Yudhister Pal, Head of Conciliatory Committee
(d) RERA Authority
4. In the above case study relating to Royal Golf Burg, who is Benamidar?
(a) Gardeners and drivers employed by Dhanraj and Yuvraj
(b) Dhanraj and Yuvraj themselves
(c) Hariharan Reddy
(d) None of the above
5. In case the Real Estate Appellate Tribunal admits the appeal of EEPL against de-registration of the
Project, then within how much maximum time such appeal must be disposed of?
(a) Within 30 days from the receipt of appeal
(b) Within 45 days from the receipt of appeal
(c) Within 60 days from the receipt of appeal
(d) Within 120 days from the receipt of appeal
Descriptive Questions
1. According to the above case study, the Royal Golf Burg promoted by the Raj Group was de-registered
by the Telangana State RERA authorities because the promoters were found to be involved in certain
3

© The Institute of Chartered Accountants of India


t.me/cafinal6d - telegram link
Page 364
unfair and fraudulent practices. You are required to state the various reasons due to which registration
granted to a project under RERA can be revoked and the project stands de-registered. (5 Marks)
2. In the given case study, the Telangana State RERA authorities resorted to de-registration of the Royal
Golf Burg Project due to unfair and fraudulent practices and irregularities followed by its promoters while
developing the project and the development was carried out at such a slow pace that ultimately the
home-buyers could not get the possession of fully furnished apartments well within the promised time.
What are the obligations of the RERA Authority and other matters associated with it if it recourses to
de-registration of a project? (5 Marks)
3. The circumstances stated in the above case study require the RERA Authority to revoke the registration
of Royal Golf Burg Project instead of its extension. State the provisions under which RERA Authority
may be required to extend the registration instead of revoking it. (5 Marks)
Case study 2
In northern and western part of the country production of sugarcane is reasonably good. But the large amount
of pendency of the payments by sugar mills to sugarcane producers is cause of worry. Common platform is
essential requirement to provide solution to this problem. Northwest Agro Produce Cooperative Society
was formed to ensure the timely collection of sale proceeds from sugar mills. Northwest Agro Produce
Cooperative Society developed a charter, in form of memorandum for its members, to regulate and control
supply, price, term of sales of sugar canes, collection of sale proceed and recovery if required. This
memorandum is binding on all the members of society.
Northwest Agro Produce Cooperative Society extend the support to cane grower, by given them offer; to sell
their entire farm produce of canes to society at mutually agreed price; which society will further sale to sugar
mills. But farmer who avail this facility have to sell his entire farm produce to Northwest Agro Produce
Cooperative Society, means farmer can’t sale any portion of his farm produce in open market. In order to
trade with the sugar mills, and deals with regulatory authorities, financial institution etc; Northwest Agro
Produce Cooperative Society decided to promote Limited Liability Company named North West Agro
Limited. The extracts from latest audited financial statements of North West Agro Limited are as follows;
Sr. No. Particular Amount (In Lakhs INRs)
1 Proceed (Net of taxes) from sale of sugar canes 320000
2 Operating assets 72800
3 Paid –up share capital 48900
4 Net Profit 9600
With Passage of time North West Agro Limited became the big hit, for role it play as intermediary; in incredible
transformation in process of sale of sugarcane by cane farmers.
Mr. Vijendra Narang, who is CEO of North West Agro limited, heard about forward integration as method of
expansion and growth strategy. Mr. Narang prepared a proposal, which is duly approved by board of directors
and then by members of company to takeover Sun Sugar Limited, by acquiring controlling stake from open
market. Sun Sugar Limited is running sugar mills, with global presence.
Around 60% of sales by Sun Sugar Limited constitute exports of raw sugar, majorly to IRAN. One year back
Sun Sugar Limited opened one branch office in IRAN, as IRAN starts buying sugar from India, in order to
settle trade balance; because IRAN is blocked from the global financial system; including using U.S. dollars
to transact its oil sales. On such branch office, during last financial year, annual recurring expenditure in
foreign currency out of EEFC accounts; was equivalent to INRs 14000 lakhs.
For last financial year, turnover of Sun Sugar Limited was recorded at INRs 120000 lakhs which was
INRs 10000 lakhs more than year earlier to last financial year; whereas operating assets as on reporting date
were INRs 27000 lakhs. Paid–up share capital was INR 12600 lakhs. After acquisition both the entities were
not merged, both kept respective separate identity.
4

© The Institute of Chartered Accountants of India


t.me/cafinal6d - telegram link
Page 365
Sun Sugar Limited has strong domestic network or tie-up with retail shops and stores through which they sale
their sugar, under brand name 'Meetha', which constitute around 40% of sale. Such retail shops and stores
are provided with instruction not to charge the price more then what is suggested by Sun Sugar Limited
although lower prices can be charged and specific jurisdiction is given to each retailer for resale.
Mr. Nair who is head of marketing at North West Agro Limited, also look after marketing at Sun Sugar Limited,
according to him; in order to acquire substantial market share (in term of new customers), Sun Sugar Limited
has to sell sugar at the prices lower than cost. Ignoring the resistance from the governing body of Sun Sugar
Limited, new pricing policy implemented. Resultantly price decreased from INRs 40 per kilogram to INRs 35
per kilogram. But in order to restrict loss, on account of selling sugar at price lower than cost; Sun Sugar
Limited ask to all the shopkeeper and stores, through whose counter they are sale their sugar produce, not
to bill more than 2 kg of ‘Meetha’ sugar per purchase
Northwest Agro Produce Cooperative Society promotes another company named South West Agro Limited,
whose object clause includes; provide weather research and forecast reports, other nec essary technical
knowledge or guidance to members of parents society apart from conducting market research for North West
Agro Limited.
Out market research conducted by South West Agro Limited, it was found that Moon Sugar Limited, hold
major stake in retail of packed sugar, around 30% across the nation under brand name ‘Aur’ (Price of which
is INRs 40 per Kilogram); which cause stiff competition among the players who sell packed sugar. Since
acquisition of Sun Sugar Limited by North West Agro Limited, remains largely successful; hence showing
trust in un-organic growth, a bear-hug letter sent to senior management of Moon Sugar Limited. For latest
financial year, turnover of Moon Sugar Limited is recorded at INRs 280000 lakhs where as operating assets
are of INRs 56800 lakhs. Paid–up share capital is INR 36400 lakhs.
Since Moon Sugar Limited is already undisputed market leader hence refuse the bear hug offer. North West
Agro Limited with help of South West Agro Limited performs hostile acquisition and both the companies
acquires around 25.5% stake in voting rights each; by tender notice over the stock exchange. Governing body
of Moon Sugar Limited restructured completely. Post acquisitions of Moon Sugar Limited, North West Agro
Limited got the dominance over the market. Since new pricing policy introduced principle buyer of North West
Agro Limited is multifold. Hence company decided to re-price their product, which is renamed also ‘Aur
Meetha’. New price is INRs 42 per Kilogram. To support the price rise, North West Agro Limited starts
restricting supply.
North West Agro Limited also entered in memorandum of understanding with Star Ethanol Limited, which is
$ 20 million (assets base) company for transfer of technology.
Multiple Choice Questions (MCQs) [2 Marks each]
1. Takeover (acquisition) of Sun Sugar limited by North West Sugar limited, will be considered as
combination if
(a) Assets of enterprise created after merger is equal than INRs 1000 crores
(b) Turnover of enterprise created after merger is more than INRs 1000 crores
(c) Turnover of enterprise created after merger is more than INRs 3000 crores
(d) Assets of enterprise created after merger is more than INRs 3000 crores
2. South West Agro Limited and North West Agro Limited will be considered as group because, these are
in capacity of
(a) Exercise 26% or more of the voting right of Moon Sugar Limited
(b) Appoint more than 50% of members of board of directors in the sun limited
(c) Control the management or affairs of the sun Limited
(d) All of above
5

© The Institute of Chartered Accountants of India


t.me/cafinal6d - telegram link
Page 366
3. Exchange Earners' Foreign Currency Account can be open by foreign exchange earner;
(a) Who are not resident in India
(b) Who are resident in India
(c) Which are situated in SEZ
(d) Person of Indian Origin, but residing outside India.
4. Decision of North West Agro Limited, on part of Sun Sugar Limited; not to sell more than 2 kg of sugar
per purchase can be categorized as;
(a) Exclusive supply agreement
(b) Exclusive distribution agreement
(c) Refusal to deal
(d) None of the above
5. Exchange Earners' Foreign Currency Account can be open with
(a) Authorised Dealer - Category I
(b) Authorised Dealer - Category II
(c) Authorised Dealer - Category III
(d) Full Fledged Money Changers
Descriptive Questions
1. Is Northwest Agro Produce Cooperative Society can be considered as ‘Cartel’. (3 Marks)
2. Does North West Agro Limited hold dominance over market, if yes at what instances you feel it abuse
its dominant position? (4 Marks)
3. Explain briefly regulatory aspects of combination in respect in light of case for North West Agro Limited
and regulation thereof under Competition Act, 2002. (5 Marks)
4. Mention provisions regarding expenditure on ‘maintaining office abroad’ under FEMA, 1999 & related
rules. Is expenditure done by Sun Sugar Limited is in violation thereto. (3 Marks)
Case study 3
APPRAY is the Karta of a Hindu Undivided family (HUF) also consisting of his wife LAXMI DEVI, 3 sons,
SUBHASH, GIRISH and RAJESH. The eldest son SUBHASH runs a Sugar Mill taken over from his father
APPRAY.
RAJESH, the third son of APPRAY, always feels ignored by his family, looking for some fast easy money,
joins hands with MOHANLAL, who is a Real Estate Agent, who promises to pay RAJESH, a commission in
cash if he helps MOHANLAL to buy 25 Acres of Land and hold the land in his name on behalf of one of his
customers MANORANJAN in good trust and in good faith. RAJESH agrees and a Purchase Agreement for
25 Acres of Land was registered in the name of RAJESH and one MADHAV RAO. Subsequently, Rajesh
entered into several similar agreements in his name on behalf of others.
In due course of time, RAJESH also formed a Company XYZ Pvt Ltd, primarily for a Hotel business, but the
source of funding was secret drug dealings. The Company accepted illegal monies in cash as legitimate
business transactions with fake income and receipts. The monies were then deposited into the Company’s
Bank accounts as clean money. He kept fraudulent records, which did not demonstrate the current state of
his businesses. Monies in the Bank Accounts of XYZ Pvt Ltd were also often transferred as legitimate
business transactions, to the Bank Accounts of RDX Pvt Ltd, which is also in the similar businesses like XYZ
Pvt Ltd. Original source of money is thus disguised.

© The Institute of Chartered Accountants of India


t.me/cafinal6d - telegram link
Page 367
The Company XYZ Pvt Ltd also mobilized funds from various investors, but were never utilized for which they
were collected. The Funds were transferred to bank accounts of some group companies, which were mainly
paper companies, from where they were systematically siphoned off and were used for purchase of various
properties in India.
RAJESH has also held some properties purchased in the name of his wife SUGUNA from his known income
from legal sources.
MAHESH is a friend of GIRISH, the Second son of APPRAY is a Company Secretary of a listed Public Limited
Company ABC Ltd. MAHESH gives Rs. 5 lacs loan to GIRISH, who in his turn gives loan of Rs. 5 Lacs to his
friend RAGHU for investment in the shares of ABC Ltd. RAGHU trades in shares of ABC Ltd on behalf of
MAHESH.
MAHESH also ensures that some money is passed on to various legitimate Companies to buy the shares of
ABC Ltd so that it results in increase in the price of shares. Intention is to show higher valuation of shares
before proposing to the investors or to discourage the shareholders from applying to the buyback scheme.
RAGHAV is the brother in law of SUBHASH, employed in UAE and a non resident Indian. RAGHAV purchased
some properties in Mumbai for Rs. 75 Lacs. He paid RS. 40 Lacs through his NRE Account, Rs. 10 Lacs
through direct transfer from his salaries account in UAE to the sellers account as advance through normal
banking channels, complying with all the procedural requirements, but balance Rs. 25 Lacs payment was
made though some unknown sources.
RAGHAV also invested in Equity shares of various Listed Companies in India in the name of his wife DIVYA,
who is a Resident in India and himself as joint holders from an account not disclosed to tax authorities in
India. RAGHAV also purchased a Flat in Mumbai in the name of DIVYA and himself as joint holders from his
NRE Account.
SUBHASH has a married daughter MANGALA, who is a UK resident. SUBHASH invested Rs. 1.50 Crores in
a Bank Fixed deposit in the name of MANGALA without her knowledge. Later during the course of enquiries
by Tax officials MANAGALA denies ownership of Bank Fixed Deposit.
Since all of his children are well settled, due to the old age and deteriorating health conditions of APPRAY
and LAXMI DEVI, the family decided to sell off the loss making Sugar Mill. Later after much negotiations, the
Sugar Mill was sold to a person well known to the real estate agent MOHANLAL, but unknown to the APPRAY
Family, at a reasonable price.
Multiple Choice questions (MCQs) [2 Marks each]
1. Purchase of properties in Mumbai by RAGHAV for Rs. 75 Lacs:
(a) Is a Fully valid transaction
(b) Is valid to the extent of Rs.40 lacs
(c) Fully invalid transaction and to be considered as “Benami”
(d) May be Benami to the extent of Rs. 25 lacs, since through some unknown sources.
2. Which one of the following transaction is NOT Benami done by RAJESH?
(a) Transaction in respect of a property, where the person providing the consideration to Rajesh is not
traceable.
(b) An arrangement by Rajesh in respect of a property made in a fictitious name.
(c) Property held by Rajesh in the name of his spouse and consideration paid out of known legal
sources.
(d) A transaction by Rajesh in respect of a property where the owner is unaware of or denies
knowledge of the ownership.

© The Institute of Chartered Accountants of India


t.me/cafinal6d - telegram link
Page 368
3. Share Trading by Raghu on behalf of Mahesh is:
(a) Valid transaction since he is not at all connected with ABC Ltd.
(b) Can be proved as Benami trading in stock markets by Mahesh, the Company Secretary, who has
insider price sensitive information.
(c) The transaction is not at all to be considered as Benami.
(d) Valid transaction if Girish does the share trading on behalf of Mahesh, out of the loan of Rs. 5 Lacs
given by Mahesh
4. Which one among the following statement is correct as per PBPT Act?
(a) Resale of the benami property from Rajesh to one of the real owners is a valid transaction.
(b) Resale of the Benami property from Rajesh to a person acting on behalf of real owner is a valid
transaction
(c) The Benami Act prohibits sale of a benami property by Rajesh to a third person
(d) PBPT Act prohibits resale of the Benami property from the Benamidar to the real owner or to any
person acting on his behalf. Such transactions would be considered as null and void.
5. In respect of transactions done by XYZ Company above, crime money injec ted into the formal financial
system is layered, moved or spread over various transactions in different accounts. This step in money
laundering is referred to as:
(a) Smurfing
(b) Integration
(c) Layering
(d) Placement
Descriptive questions
1. In the context of various property dealings in the above case study, critically analyze the statement “the
provisions of the Prohibition of Benami Property Transactions Act, 1988 (PBPT Act) need not
necessarily applicable only to persons, who try to hide their properties, but may also sometimes apply
to genuine properties acquired out of disclosed funds” (5 Marks)
2. In case of confiscation of properties of Rajesh in the above example, how “Fair market value” in relation
to a property is defined under Sec 2(16) of the PBPT Act? Also interpret the Rule 3 of Prohibition of
Benami Transactions Rules, 2016 when the price is not ascertainable? (5 Marks)
3. With reference to various transactions by XYZ Co Private Ltd in the above example, what are the
provisions relating to attachment of property involved in money laundering under Section 5 of the
Prevention of Money laundering Act 2002? (5 Marks)
Case study 4
Alpha Promoter is the promoter of real estate based at New Delhi. The Alpha Promoter proposed to establish
the residential project named “AASHIYANA” The project plan constituted 50 apartment mix of 3BHK and
2BHK Apartments. It includes 20 3BHK Apartments and 30 2BHK Apartments. The Residential project needs
to be registered under RERA. The residential project is based at East Delhi. The Alpha’s son a Qualified
Chartered Accountant and a RERA Consultant. Mr. Alpha discuss the proposed projects with his son usually
once in the month. Mr. Alpha discussed with his son CA Surendar about the newly established project
“AASHIYANA” based at East Delhi. The Father Mr. Alpha discuss the applicability of various provisions on
the new project “AASHIYANA” based at East Delhi with his son CA Surendar under the Delhi Real Estate
(Regulation & Development) Act, 2016 and rules made thereto. The father inquired about the prior approval
before establishment of new project “AASHIYANA”, if any required by Delhi Real Estate (Regulation &

© The Institute of Chartered Accountants of India


t.me/cafinal6d - telegram link
Page 369
Development) Act, 2016. Further Mr. Alpha inquired about the creation of webpage of the Alpha Promoter on
the website of DELHI RERA Estate (Regulation & Development) Act, 2016, after getting Login ID and
Password.
Also, Mr. Alpha want to publish the advertisement for the residential project “AASHIYANA” for which he
wants to know the content. The advertisement specifies the condition of advance payment first wi thout
entering into agreement for sale. The advance payment (deposit) shall not be less than 15% of the cost of
apartment. Only after payment of such advance of 15% of the cost of the apartment, a promoter will enter
into agreement for sale with allottee of the particular apartment. The promoter made the policy for non-
disclosure of any stage-wise time schedule of the completion of the project, including the provisions for civic
infrastructure like water sanitation & electricity at the time of booking and issue of allotment letter to allottee.
The promoter forgot to include any terms for cancellation of allotment in the agreement of sale. The
construction started and afterwards the promoter make some major alterations in the sanctioned plans &
layout plans as well as nature of fittings without the previous approval from allottees. One of the allottee Mr.
Abhay ask for minor changes in his allotted apartment which was further also approved by authorised architect
and proper declaration and intimation is also made regarding such minor change to the allottee (Mr. Abhay).
Meanwhile, presently the Alpha Promoter intended to transfer the project “AASHIYANA” to Beta Promoter
(third party) without obtaining any approval.
Issue Raised
• Can Alpha Promoter can transfer the project to the Beta Promoter suo moto?
• Is Alpha Promoter mandatorily required to maintain webpage on the website of DELHI RERA
AUTHORITY?
• What are requirements for registration of new residential project “AASHIYANA”?
• Can advertisement published with or without mentioning website address?
• Is any possibility of holding right as promoter of not providing any details on stage-wise time schedule
of completion of the project at the time of booking to the allottee?
• Is promoter having right to demand more than 15% of the cost of apartment before entering into
agreement for sale with the allottee.
Few apartments are purchased in the name of Foreign National (Non-Residents) and few are purchased in
joint name of both Indian National (Resident) and Foreign National (non-residents) in light of the provisions
of the Prohibition of Benami Property Transactions Act, 1988, the following transactions took place for
purchase of Apartment under project “AASHIYANA”. The description of such transaction are as follows:
• Mr. X, a non resident in India purchased residential apartment of “AASHIYANA” residential project
under RERA for INR 50,00,000. The apartment is purchased in the joint name of Mr. X and his sister
but the payment is made from unknown source.
• Mr. Y, a non resident in India purchased residential apartment of “AASHIYANA” residential project
under RERA for INR 60,00,000 in the name of his wife from his NRE Account.
• Mr. Z, a resident in India sold a apartment in India (second hand apartm ent) of some other RERA
registered residential project for INR 50,00,000 which was not in his name and was in unknown person
name. But the sale proceed is deposited in his (Mr. Z) personal bank account.
• Mr. P, a non resident in India purchased residential apartment of “AASHIYANA” residential project
under RERA for INR 50,00,000. The payment is made in parts INR 20,00,000 by P’s NRE account,
INR 29,50,000 from unknown sources and INR 50,000 in cash. The registry was done at value of
INR 45,00,000 which was paid by own cheque (INR 20,00,000) and from unknown sources jointly
(INR 25,00,000).

© The Institute of Chartered Accountants of India


t.me/cafinal6d - telegram link
Page 370
Based on the above mentioned facts of the case and issues raised you are requested to give professional
advice as Corporate Consultant to the Alpha Promoter and allottees of the residential apartment on the below-
mentioned questions:
Multiple Choice Questions (MCQs) [2 Marks each]
1. The Alpha Promoters shall not transfer or assign his majority rights & liabilities in respect of a real estate
project to a third party without obtaining:
(a) Prior written consent from two-third allottees;
(b) Prior written consent from two-third allottees; except the promoters
(c) Prior written consent from two-third allottees; except the promoters and prior written approval of
the Authority.
(d) Prior written consent from two-third allottees; except the promoters and prior written approval of
the Authority. However, such transfer or assignment shall not affect the allotment or sale of the
apartments.
2. The Alpha Promoter needs to get registration of the new project “AASHIYANA” through the
operationalise web based online system for submitting application for registration of the project within a
period of ____________________.
(a) One year from the date of its establishment.
(b) One year from the date of its commencement.
(c) One year from the date of its initiation.
(d) One year from the date of its starting.
3. The promoter Alpha at the time of the booking and issue of allotment letter to the allottee shall be
responsible to make available to the allottee the _________.
(a) Sanctioned Plans
(b) Layout Plans
(c) the stage-wise time schedule of completion of the project.
(d) All of the above.
4. The Alpha Promoter shall be responsible for providing and maintaining the essential services, on
reasonable charges, till the taking over of the maintenance of the project “AASHIYANA” by
_____________________.
(a) The allottees;
(b) The third party;
(c) The association of the allottees;
(d) None of the above.
5. Mr. Z, a resident in India sold a apartment in India (second hand apartment) of some other RERA
registered residential project for INR 50,00,000 which was not in his name and was in unknown person
name. But the sale proceed is deposited in Mr. Z’s personal bank account. The given transaction is
___________________ as per the provisions of Prohibition of Benami Property Transactions Act, 1988.
(a) Benami Transaction.
(b) Not a Benami Transaction
(c) May be a Benami Transaction.

10

© The Institute of Chartered Accountants of India


t.me/cafinal6d - telegram link
Page 371
(d) May not be a Benami Transaction
Descriptive Questions
1. In Light of the provisions of the Section 11 of the Real Estate (Regulation & Development) Act, 2016,
the Alpha Promoter upon receiving Login ID and password from RERA Authority create webpage on the
Website of the authority including all details of the proposed project for public viewing.
Describe in detail what information are required to be disclosed for public view on the webpage of
promoter on the Website of the RERA Authority? (5 Marks)
2. Under the light of the provisions of “Prohibition of Benami Property Transactions Act, 1988”. Examine
whether the below mentioned transaction is considered as the Benami Transactions or not?
Mr. P, a non resident in India purchased residential apartment of “AASHIYANA” residential project under
RERA for INR 50,00,000. The payment is made in parts INR 20,00,000 by P’s NRE account,
INR 29,50,000 from unknown sources and INR 50,000 in cash. The registry was done at value of
INR 45,00,000 which was paid by own cheque (INR 20,00,000) and from unknown sources jointly
(INR 25,00,000). (5 Marks)
3. Examine the following:
(i) Alpha Promoter’s responsibility when the Real estate Project “AASHIYANA” is developed on a
leasehold land. (2 Marks)
(ii) Alpha Promoter can cancel the allotment whether or not such terms of cancellation are included in
the executed agreement of sale. (3 Marks)
Case study 5
Mr Suresh Agarwal, is a Jaipur based banker who joined the Samradhi National bank in the year 2008 as a
clerk. He got his first posting at Jodhpur branch. After five years in job, he was transferred and promoted as
assistant manager, Bikaner branch of Samardhi National bank. One day he met Archana in the bank, who
visited bank to discuss some loan issues of her company with the bank. Slowly they started meeting each
other. Both of them like each other and decided to marry after sometime. Archana is a director in a Newtech
Software company. They decided to get married after one year. With the blessing of their parents they got
married in 2013.
After marriage Archana need to visit America for some business meeting of the company with foreign client.
So, they both planned a tour parallelly with the business trip. Company issued Prepaid Forex Travel Card to
Archana for all his expenses abroad. The card is loaded with amount of USD 2,50,000. After two days of
prolonged business meetings, Archana finally got free to enjoy trip with her husband. They went to Los Vegas
and played some games in casino. They paid USD 2000 with the Forex card. They hardly won any amount
out of it. They enjoyed shopping and had all fun. After 10 days they return back to India. Archana returned
the business card to the company with remaining USD 100000.
Back to daily routine they both got busy with their respective work schedule. As an assistant manager,
Mr Suresh got gifts in form of cash and kind all the year round. He came in contact with many rich and
successful business owners from different walks of life. They all were happy with him as mostly they got their
work done on a single phone call. In the year 2016 post Demonetization, Mr Suresh helped many of his bank
customers to launder their money through their bank accounts. He helped one of the bullion trader, Mr Naval
Kishore, account holder in his bank. He suggested him to take money from his customers and issue cheque
back for the same amount, less 5 per cent. The bullion dealer gave them a purchase bill to show that he had
purchased his gold jewellery or silver utensils from him. Than On the amount of the cheque when they will file
their return, the customer will have to pay no capital gains tax as jewellery and silver utensils are of personal
use, and capital gains do not arise on sale of personal things.

11

© The Institute of Chartered Accountants of India


t.me/cafinal6d - telegram link
Page 372
Mr. Naval Kishore with some agents, in past use to trade illegal gold through illicit shell or front companies
using false or incomplete documents. The gold is often then smuggled through third-party countries and
ultimately sold to refineries in the United States.
Mr Suresh offers to exchange demonetized banknotes for a charge of 10 per cent. Much of this was managed
by using scores of individual PAN and Aadhar details already available with his bank. Mr Suresh helped Mr
A.K. Bajaj, to exchange demonetized cash, through his workers ID's at the bank. Mr. Bajaj employed nearly
200 workers of his manufacturing units to get the demonetized notes exchanged. The Demonetization period
became a blessing in disguise for Mr. Suresh as he was able to amass huge profits out of it.
Chetan singh is a director of XYZ Company Ltd. He use to share a good relation with Mr. Agrawal and he
also started giving him attention when he visited bank for day today business. Chetan one day came to Mr.
Agarwal to sanction him INR 0.50 crores. The documents required to process the loan was incomplete. The
branch manager refused to sanction the loan without completing all the formalities. Chetan had a talk with
Mr. Agarwal and promised him to pay 5% of total sanctioned loan amount. So, both of them arranged some
forged documents to complete the file with help of one person called Piyush Sharma, an accountant friend of
Mr. Agarwal. After the completion of all the formalities Mr. Agarwal gave his clearance regarding completion
of the documents, after which his branch manager sanctioned the loan. The deal bought all of them close to
each other. After that Mr Agarwal, Chetan and Piyush became good friends.
As Piyush was accountant and an employee in financial advising company he used to meet many people. He
got some couple of loans sanctioned with the help of Mr Suresh Agarwal. As a result, on performance basis
evaluation Mr Agarwal got the promotion as Bank manager and transferred to the other branch of Bikaner.
Archana in 2016, was appointed as a director, in Zippy International Company Ltd, a foreign based Company.
For Couple of months she stayed in Italy for board meeting and giving financial advice regarding the business
transactions. In lieu of his service and remuneration she received shares of the company worth USD 3,00,000.
During his period of service in bank, Mr Agarwal was able to accumulate INR 20 lakhs from his unauthorized
sources. With this money he bought a plot in his home town Jaipur. The cost of the plot is INR 50 lakhs. To
cover up his source of income he took a loan of INR 20 lakh from his bank. Likewise, he invested his income
got in his mother's name and got the plot registered.
In 2017, Archana again went to London for company related business. She stayed there for a duration of six
months. With a view to invest abroad, she bought a flat their through outwards remittance. She remitted USD
2,49,000 via LRS route.
Mr Agarwal and his wife Archana went on a tour to Qatar and Dubai in 2018.They booked the flight tickets
online through their credit card, rest all the expense of logging and boarding was borne by Oyster tour and
travel agency. The company recently opened his new branch office in Dubai. The company's AD bank is the
bank in which Mr. Suresh is branch manager. He made all the transactions of Oyster tour and travels easy.
Whenever Exchange is required by the branch office in Dubai he easily released them. The profit earned by
Dubai branch, of Oyster tour and travel was invested, in one company of Dubai engaged into real estate
business and its profits is deposited in foreign bank.
Mr Chetan has a planned to earn money through a new export and import business. He submitted a fake
factory proposal of garment manufacturing to the bank whose branch manager is Mr Agarwal. Mr Chetan had
a plot in the outskirts of the city, where he made some construction to give it a factory look. As per the mutual
understanding between both of them some loan of INR 2 crores was sanc tioned to ship the machines and
other products from abroad. Some fake invoices were prepared to show the dispatched of garment orders on
record but sent almost nothing in return. Mr Agarwal and Mr Chetan jointly did all the invoice manipulation.
As a result siphoning off of foreign exchange against fictitious and bogus imports. The money was invested
abroad in mutual funds and buying properties.

12

© The Institute of Chartered Accountants of India


t.me/cafinal6d - telegram link
Page 373
Multiple choice Questions (MCQs) [2 Marks each]
1. Archana went to Italy regarding his official visits as a company director. She bought a flat in Italy through
outwards remittance. Can she continue to hold immovable property outside India which was acquired
by her when he was a non-resident?
(a) With the permission of Reserve bank of India she can hold the property abroad.
(b) Without the permission of Reserve Bank of India she can hold the property abroad.
(c) The property needs to be sold and the funds transferred should be repatriate back to India.
(d) The property needs to be sold out and transferred should be repatriate to her FCNR account.
2. Let's assumed that the plot is bought by Mr Suresh from his known sourced income in his mother's
name. Mr. Tarun an NRI wants to purchase that plot. State your answer.
(a) Mr. Tarun can buy the plot only with permission of RBI.
(b) Mr. Tarun can buy the plot through normal banking channel of India without RBI permission.
(c) Mr. Tarun can purchase the plot in foreign currency with permission of RBI.
(d) Mr. Tarun can make part payment in foreign currency and partly through his NRO account in India.
3. Archana was allotted USD 300000 shares in the foreign entity in lieu of the professional services
rendered by her in lieu of Director’s remuneration. Can she acquire such shares?
(a) Archana can acquire USD 300000 shares as it is paid against her remuneration.
(b) Archana can acquire USD 300000 shares as her remuneration with permission of RBI.
(c) Archana cannot acquire USD 3,00000 shares as it is not in permissible limit of LRS.
(d) Archana can acquire USD 3,00000 shares under LRS but needs to repatriate the profits back to
India after sale of it.
4. Has Suresh and Archana contravene any provision of law, while they sent USD 2000 playing at the
casino? Choose the correct answer.
(a) They did not contravene any provision of the law to as it was issued to her in official capacity for
her trip.
(b) They have used the card in violates the provisions of FEMA act and hence liable under it.
(c) They did not contravene any provision of the law as it was within the permissible limit of their
expenses.
(d) They were only liable under FEMA Act if they had won more than USD 2000 and if it is remitted
back to India.
5. In post Demonetization period Mr. Suresh helped many of his customers to exchange demonetized
currency through various methods. Is he liable for punishment under any provision of the law in India?
(a) Mr. Suresh is not liable for any punishment under any provision of law applicable in India.
(b) Mr. Suresh is liable to punishment under Part A of the schedule, of prevention of Money Laundering
Act.
(c) Mr. Suresh is only liable to be punished under Part C of the Schedule, of Money Laundering Act.
(d) Mr. Suresh is liable for punishment under Part A, PARAGRAPH 1 and PARAGRAPH 4 of Money
Laundering Act.
Descriptive Questions.
1. Advise in the above mentioned two situations as per the relevant applicable law in India?

13

© The Institute of Chartered Accountants of India


t.me/cafinal6d - telegram link
Page 374
(i) Assuming Mr. Suresh wanted to further lease the plot, he bought in his mother name, to a foreign
entity who wants to open his liaison office in India.
(ii) Can Mr. Suresh Agarwal mother's gift this plot to his niece, an NRI. (8 Marks)
2. Examine the given situations in the light of the PBPT Act:
(i) Explain why the property brought by Mr Agarwal in his mother name is a 'Benami transaction'?
(3 Marks)
(ii) If in case a fraudulent transfer is done by Mr Suresh Agarwal, pending the process of inquiry by
the Initiating Officer, what will be the consequences? (2 Marks)
(iii) State what will the effect if property (the plot in the name of Mr Agarwal's mother name) is
confiscated by Initiating Officer? (2 Marks)

14

© The Institute of Chartered Accountants of India


t.me/cafinal6d - telegram link
Page 375
Test Series: October, 2019

MOCK TEST PAPER 1


FINAL (NEW) COURSE: GROUP – II
PAPER – 6D: ECONOMIC LAWS
Suggested Answer
Case Study 1
Multiple Choice Questions (MCQs)
1. (c)
2. (b)
3. (a)
4. (a)
5. (c)
Descriptive Questions
Answer 1
De-registration of a building project is not an ideal choice for the authorities keeping in view the larger
interests of the stakeholders as well as the nation as a whole and it is resorted to only when all the possible
avenues of reaching a comfortable and plausible solution are shut.
According to Section 7 of the Real Estate (Regulation and development) Act, 2016 (in short ‘RERA Act’)
various reasons responsible for revocation of registration granted to a project under RERA due to which the
building project stands de-registered are stated as under:
(a) Making of default: The promoter makes default in doing anything required by or under the RERA Act
or the rules or the regulations made thereunder;
(b) Violation of terms or conditions of approval: The promoter violates any of the terms or conditions of
the approval given by the competent authority. From the case study it shall be noticed that broadly the
following terms or conditions of the approval were violated by the promoters of the Royal Golf Burg:
(i) Non-construction of all the apartments in fifteen towers till the due date.
(ii) Non-furnishing of apartments though the deadline to handover the furnished apartments passed.
(c) Involvement in unfair practice or irregularities: The promoter is involved in any kind of unfair practice
or irregularities.
Explanation. — For the purposes of this clause, the term “unfair practice” means a practice which, for
the purpose of promoting the sale or development of any real estate project adopts any unfair method
or unfair or deceptive practice including any of the following practices, namely:
(A) the practice of making any statement, whether in writing or by visible representation which, —
(i) falsely represents that the services are of a particular standard or grade;
(ii) represents that the promoter has approval or affiliation which such promoter does not have;
(iii) makes a false or misleading representation concerning the services;

© The Institute of Chartered Accountants of India


t.me/cafinal6d - telegram link
Page 376
(B) the promoter permits the publication of any advertisement or prospectus whether in any newspaper
or otherwise of services that are not intended to be offered;
(d) Fraudulent practices: The promoter indulges in any fraudulent practices. From the case study it shall
be noticed that the fraudulent practices undertaken by the promoters of the Royal Golf Burg included:
(i) Resorting to double allotment due to which the genuine home-buyers were not allotted the
apartments which they very much deserved.
(ii) Diversion of funds meant for constructing the apartments to shell companies.
(iii) Allotment of apartments in the name of the peons and drivers though such allotments were actually
meant for the use of the promoters since the consideration flowed from them.
Answer 2
When a project is de-registered due to revocation of registration, following are the obligations of the RERA
Authority and other matters associated with it in terms of Section 8 of the RERA Act:
(i) Consultation with the Appropriate Government: The concerned RERA Authority may consult the
appropriate Government to take such action as it may deem fit. This will include:
• the carrying out of the remaining development works by competent authority; or
• the carrying out of the remaining development works by the association of allottees; or
• the carrying out of the remaining development works by in any other manner, as may be determined
by the Authority.
(ii) When the direction, decision or order of the Authority shall take effect: It is provided that a
direction, decision or order of the Authority under section 8 shall take effect only after the expiry of the
period of appeal provided under the provisions of the RERA Act.
(iii) First right of refusal rests with association of allottees: It is provided that in case of revocation of
registration of a project under the RERA Act, the association of allottees shall have the first right of
refusal for carrying out of the remaining development works.
Answer 3
(i) According to Section 6 of the RERA Act, under the circumstances relating to "force majeure” and where
there is no default on the part of the promoter, the registration granted under section 5 of the RERA Act
may be extended by the Authority.
The expression "force majeure" means a case of war, flood, drought, fire, cyclone, earthquake or any
other calamity caused by nature affecting the regular development of the real estate project.
For the purpose of extension, the Promoter of the building project affected by "force majeure" shall make
an application to the Authority in the specified form and shall make payment of the specified fee as
prescribed by the RERA Regulations made by the Authority.
The Authority may in reasonable circumstances, without default on the part of the promoter, based on
the facts of each case, and for reasons to be recorded in writing, extend the registration granted to a
project for such time as it considers necessary. However, the extension of registration shall, in
aggregate, not exceed a period of one year.
The Authority shall not reject any application for extension of registration unless it gives the applicant
an opportunity of being heard in the matter.

© The Institute of Chartered Accountants of India


t.me/cafinal6d - telegram link
Page 377
(ii) According to Section 7 (3) of the RERA Act also, the Authority may, instead of revoking the
registration under sub-section (1) of Section 7, permit the registration to remain in force subject to
such further terms and conditions as it thinks fit to impose in the interest of the allottees, and
any such terms and conditions so imposed shall be binding upon the promoter.
Case study 2
MCQ
1. c
2. d
3. b
4. d
5. a
Descriptive Questions
Answer 1
As per section 2(c) “cartel” includes an association of producers, sellers, distributors, traders or service
providers who, by agreement amongst themselves, limit, control or attempt to control the production,
distribution, sale or price of, or, trade in goods or provision of services.
Although, Northwest Agro Produce Cooperative Society was formed to ensure the timely collection of sale
proceeds from sugar mills. But Northwest Agro Produce Cooperative Society also developed a charter, in
form of memorandum for its members, to regulate and control supply, price, term of sales of sugar canes
(even though on behalf cane-growers), collection of sale proceed and recovery if required. This
memorandum is binding on all the members of society. Hence Northwest Agro Produce Cooperative Society
is ‘Cartel’ under Competition Act, 2002.
Answer 2
Yes, North West Agro Limited hold dominance, because as per explanation (a) to section 4 “dominant
position” means a position of strength, enjoyed by an enterprise, in the relevant market, in India, which
enables it to (i) operate independently of competitive forces prevailing in the relevant market; or (ii) affect its
competitors or consumers or the relevant market in its favour.
Instances of abuse of dominance
Predatory Pricing after acquisition of Sun Sugar Limited - North West Agro Limited, acquired substantial
network of retailer after takeover of sun sugar Limited, help of which they tried to penetrate in the market
using predatory pricing [Section 4(2)(a)(ii)]. North West Agro Limited reduce the price of its sugar ‘Meetha’
from INRs 40 to 35 per kilogram, where as other player in market like Moon Sugar limited selling sugar at
INRs 40 per kilogram.
As per explanation (b) to section 4 “predatory price” means the sale of goods or provision of services, at a.
price which is below the cost, as may be determined by regulations, of production of the goods or provision
of services, with a view to reduce competition or eliminate the competitors.
Increase the price after acquisition of Moon Sugar Limited – After hostile acquisition of Moon Sugar
Limited, with help of another group company South West Agro Limited; North West Agro Limi ted raise the
prices of its sugar ‘Aur Meetha’ from INRs 35 to 42 per kilogram; even Moon Sugar Limited, originally selling
its sugar ‘Aur’ at INRs 40 per Kilogram. Section 4(2)(b)(i) says there shall be an abuse of dominant position
under sub-section (1) of section 4, if an enterprise or a group limits or restricts production of goods or market
therefore .

© The Institute of Chartered Accountants of India


t.me/cafinal6d - telegram link
Page 378
Answer 3
Provision of related to combination detailed in section 5 of Competition Act ,2002
Sr. Nature of Case facts Criteria Is combination
No Combination
Acquisition by North West Agro Joint Asset over Yes, Joint turnover is INRs
single acquirer but Limited Takeover INRs 1000 crores 4400 crores (3200+1200)
1 different goods Sun Sugar Limited or Turnover over which is more than INRs 3000
(Section 5(a)(i) INRs 3000 crores crores, whereas joint assets
base is only INRs 998 crores
Acquisition by North West Agro Group Asset over No, Joint Asset base of group
group with similar Limited Acquired INRs 4000 crores is only INRs 1566 crores and
goods (Section Moon Sugar or Turnover over aggregate turnover is also
2 5(b)(ii) Limited, with help of INRs 12000 crores INRs 7200 Crores
another group
company South
West Agro Limited
Not considered MOU between Not eligible to be Not Applicable
as combination North West Agro considered as
3
Limited and Star combination
Ethanol Limited
Regulation of Combinations (Section 6)
No person or enterprise shall enter into a combination which causes or is likely to cause an appreciable
adverse effect on competition within the relevant market in India and such a combination shall be void.
Any person or enterprise, who enter into a combination, give notice to the Commission, disclosing the
details of the proposed combination, within thirty days of -
(a) Approval of the proposal concerned with such merger or amalgamation by the board of directors, or (b)
execution of any agreement acquiring of control
No combination shall come into effect until two hundred and ten days have passed from the day on
which the notice has been given to the Commission or the Commission has passed orders whichever is
earlier.
Answer 4
Vide Foreign Exchange Management (Current Account Transactions) Rules 2000, some restriction on
current account foreign exchange transaction prescribed. A general permission is available for opening of
Bank Account for the purpose of meeting the Branch Expenses abroad.
Authorised Dealer Category – I banks may allow remittance up to ten per cent of the average annual
turnover during the last two financial years. But this restriction shall not be applicable if remittance to
account maintained abroad, made out of the funds held in EEFC account.
In given case turnover for relevant 2 years was INRs 120000 lakhs and INRs 110000 lakhs (i.e. INRs 120000
lakhs - INRs 10000 lakhs). Average of which is INRs 115000 lakhs. Maximum permissible amount of branch
recurring expenditure in case of normal account was 10% of INRs 115000 lakhs i.e INRs 11500 lakhs.
Expenditure incurred by Sun Sugar Limited in given case is INRs 14000 lakhs. But such expenditure will not
be considered in violation of Foreign Exchange Management (Current Account Transactions) Rules 2000,
because amount expended out of EEFC Account .

© The Institute of Chartered Accountants of India


t.me/cafinal6d - telegram link
Page 379
Case study 3
Multiple Choice questions (MCQs)
1. (d)
2. (c)
3. (b)
4. (d)
5. (c)
Descriptive questions
Answer1
The general belief is that the provisions of the Prohibition of Benami Property Transactions Act 1988 (PBPT
Act) apply only to persons, trying to hide their properties and not to genuine properties acquired out of
disclosed funds. But that is not true. Even a property acquired using disclosed funds in a genuine transaction
may sometimes be treated as Benami.
“Benami Property” under Sec 2 (8) means any property, which is the subject matter of a Benami transaction
and also includes the proceeds from such property.Benami Property means property without a name. Here
the person, who pays for the property does not buy it under his own name. The person, who finances the
deal is the real owner of the property. The person in whose name the property has been purchased is
Benamidar.
As per the provisions of Section 2 (9) a Benami transaction means-
1. A transaction or arrangement where a property is transferred to or held by one person for direct or
indirect, immediate or future benefit of another person, who has provided or paid the consideration,
except when-
(i) An HUF is purchasing a property in the name of a Karta, or any other member from known sources;
(ii) A person is holding the property in a fiduciary capacity (e.g. trustee, executor, partner of a
partnership firm, director of a company, a depository participant, etc.);
(iii) An individual is purchasing a property in the name of his spouse or any child provided the
consideration is paid out of the known sources;
(iv) Any person is purchasing any property in the name of his brother or sister or lineal ascendant or
descendant, where he is one of the joint-owners, provided the consideration is paid out of the
known sources; or
2. A transaction or arrangement carried out in a fictitious name; or
3. A transaction or arrangement where the owner of the property is not aware of or denies knowledge of
such ownership;
4. A transaction or arrangement, where the person providing the consideration is not traceable or is
fictitious.
Any transaction where possession of any immovable property is taken as a part performance of a contract is
not a Benami transaction if the contract is registered and consideration as well as stamp duty have been paid.
Property would include asset of any kind, whether movable or immovable, tangible or intangible, and includes
rights or interest as well as proceeds from the property.

© The Institute of Chartered Accountants of India


t.me/cafinal6d - telegram link
Page 380
In the above case study, in one of the cases, SUBHASH invested Rs. 1.50 Crores in a Bank Fixed deposit in
the name of his married daughter, MANGALA, who is a UK Resident, without her knowledge. Later during
the course of enquiries by Tax officials MANAGALA denies ownership of Bank Fixed Deposit. Here, the
transaction is Benami, though the FD is generated using disclosed funds in a genuine transaction.
Answer 2
As per the provisions of Section 2(16) of the Prohibition of Benami Property Transactions Act 1988, "fair
market value", in relation to a property, means—
(i) the price that the property would ordinarily fetch on sale in the open market on the date of the
transaction; and
(ii) where the price referred to in sub-clause (i) is not ascertainable, such price as may be determined in
accordance with such manner as prescribed in Rule 3 of Prohibition of Benami Transactions Rules,
2016
Determination of price in certain cases under Rule 3 of Prohibition of Benami Transactions Rules,
2016
3. (1) For the purposes of sub-clause(ii) of c lause (16) of the section 2 of the Ac t, the price shall be
determined in the following manner, namely:—
(a) the price of unquoted equity shares shall be the higher of,—
(I) its cost of acquisition;
(II) the fair market value of such equity shares determined, on the date of transaction, by a
merchant banker or an accountant as per the Discounted Free Cash Flow method; and
(III) the value, on the date of transaction, of such equity shares as determined in the following
manner, namely:—
The fair market value of unquoted equity shares = (A+B - L) x (PV)/(PE)
where,
A= book value of all the assets (other than bullion, jewellery, precious stone, artistic work,
shares, securities and immovable property) as reduced by,-
(i) any amount of income-tax paid, if any, less the amount of income-tax refund claimed, if
any, and
(ii) any amount shown as asset including the unamortised amount of deferred expenditure
which does not represent the value of any asset;
B= the price that the bullion, jewellery, precious stone, artistic work, shares, securities and
immovable property would ordinarily fetch on sale in the open market on the date of
transaction;
L= book value of liabilities, but not including the following amounts, namely:—
(i) the paid-up capital in respect of equity shares;
(ii) the amount set apart for payment of dividends on preference shares and equity shares;
(iii) reserves and surplus, by whatever name called, even if the resulting figure is negative,
other than those set apart towards depreciation;
(iv) any amount representing provision for taxation, other than amount of income-tax paid, if
any, less the amount of income-tax claimed as refund, if any, to the extent of the excess
over the tax payable with reference to the book profits in accordance with the law
6

© The Institute of Chartered Accountants of India


t.me/cafinal6d - telegram link
Page 381
applicable thereto;
(v) any amount representing provisions made for meeting liabilities, other than ascertained
liabilities;
(vi) any amount representing contingent liabilities other than arrears of dividends payable in
respect of cumulative preference shares;
PE = total amount of paid up equity share capital as shown in the balance-sheet;
PV= the paid up value of such equity shares;
The above provisions of Section 2(16) of the Act Read with Rule 3 is applicable, in case of confiscation of
properties of RAJESH.
Answer 3
Attachment of property involved in money-laundering under Section 5 of the Prevention of Money
laundering Act 2002?.
Section 5 (1) Where the Director or any other officer not below the rank of Deputy Director authorised by
the Director for the purposes of this section, has reason to believe (the reason for such belief to be recorded
in writing), on the basis of material in his possession, that—
(a) any person is in possession of any proceeds of crime; and

(b) such proceeds of crime are likely to be concealed, transferred or dealt with in any manner which
may result in frustrating any proceedings relating to confiscation of such proceeds of crime under
this Chapter,
he may, by order in writing, provisionally attach such property for a period not exceeding one hundred and
eighty days from the date of the order, in such manner as may be prescribed:
Provided that no such order of attachment shall be made unless, in relation to the scheduled offence, a
report has been forwarded to a Magistrate under section 173 of the Code of Criminal Procedure, 1973 (2 of
1974), or a complaint has been filed by a person authorised to investigate the offence mentioned in that
Schedule, before a Magistrate or court for taking cognizance of the scheduled offence, as the case may be,
or a similar report or complaint has been made or filed under the corresponding law of any other country:
Provided further that, notwithstanding anything contained in [first proviso], any property of any person may
be attached under this section if the Director or any other officer not below the rank of Deputy Director
authorised by him for the purposes of this section has reason to believe (the reasons for such belief to be
recorded in writing), on the basis of material in his possession, that if such property involved in money-
laundering is not attached immediately under this Chapter, the non-attachment of the property is likely to
frustrate any proceeding under this Act.
Provided also that for the purposes of computing the period of one hundred and eighty days, the period
during which the proceedings under this section is stayed by the High Court, shall be excluded and a further
period not exceeding thirty days from the date of order of vacation of such stay order shall be counted.
(2) The Director, or any other officer not below the rank of Deputy Director, shall, immediately after
attachment under sub-section (1), forward a copy of the order, along with the material in his possession,
referred to in that sub-section, to the Adjudicating Authority, in a sealed envelope, in the manner as may be
prescribed and such Adjudicating Authority shall keep such order and material for such period as may be
prescribed.

© The Institute of Chartered Accountants of India


t.me/cafinal6d - telegram link
Page 382
(3) Every order of attachment made under sub-section (1) shall cease to have effect after the expiry of the
period specified in that sub-section or on the date of an order made under [sub-section (3)] of section 8,
whichever is earlier.
(4) Nothing in this section shall prevent the person interested in the enjoyment of the immovable property
attached under sub-section (1) from such enjoyment.
Explanation.—For the purposes of this sub-section, "person interested", in relation to any immovable
property, includes all persons claiming or entitled to claim any interest in the property.
(5) The Director or any other officer who provisionally attaches any property under sub-section (1) shall,
within a period of thirty days from such attachment, file a complaint stating the facts of such attachment
before the Adjudicating Authority.
Case study 4
Multiple Choice Questions
1. (d)
2. (a)
3. (d)
4. (c)
5. (a)
Descriptive Questions
Answer 1
As per the provisions of the Section 11 (1) The promoter shall, upon receiving his Login Id and password
under clause (a) of sub-section (1) or under sub-section (2) of section 5, as the case may be, create his web
page on the website of the Authority and enter all details of the proposed project as provided under sub-
section (2) of section 4, in all the fields as provided, for public viewing, including—
(a) details of the registration granted by the Authority;
(b) quarterly up-to-date the list of num ber and types of apartments or plots, as the c ase m ay be,
booked;
(c ) quarterly up-to-date the list of number of garages booked;
(d) quarterly up-to-date the list of approvals taken and the approvals which are pending subsequent to
c ommencement certificate; (e) quarterly up -to-date status of the project; and
(f) suc h other information and doc uments as m ay be spec ified by the regulations m ade by the
Authority.
Thus, the Alpha promoter are required to be disclosed above mentioned information for public view on the
webpage of promoter Alpha on the Website of the DELHI RERA Authority.
Answer 2
The payment for purchase of the flat by Mr. P, a non resident in India is made in parts. Since, the payment
of INR 25,00,000 is made from unknown source for registry of the apartment. So, to the extent of unknown
source of the consideration payment, it may be a Benami Transaction.

© The Institute of Chartered Accountants of India


t.me/cafinal6d - telegram link
Page 383
Answer 3
(i) As per the provisions of the Section 11(4)(c) of the Real Estate (Regulation & Development) Act,
2016, the promoter shall be responsible to obtain the lease c ertificate, where the real estate project
is developed on a leasehold land, specifying the period of lease, and c ertifying that all dues and
c harges in regard to the leasehold land has been paid, and to make the lease c ertificate available
to the association of allottees.
T hus, Alpha Promoter have to fulfil all responsibilities as discussed above.
(ii) As per the provisions of the Section 11(5) of the Real Estate (Regulation & Development) Ac t, 2016,
the promoter may c ancel the allotment only in terms of the agreement for sale.
Provided that the allottee may approach the Authority for relief, if he is aggrieved by such cancellation
and such cancellation is not in accordance with the terms of the agreement for sale, unilateral and without
any sufficient cause.
Thus, the Alpha Promoter cannot cancel the allotment without including such terms of cancellation in
the executed agreement of sale.
Case study 5
Multiple choice Questions
1. b
2. b
3. c
4. c
5. b
Descriptive Questions.
Answer 1
(i) A body c orporate incorporated outside India (including a firm or other association of individuals),
desirous of opening a Liaison Office (LO) / Branch Office (BO) in India have to obtain permission
from the Reserve Bank under provisions of FEM A 1999.Establishment of Project Offices/Liaison
Offices in India is regulated in terms of Section 6(6) of Foreign Exchange M anagement Ac t, 1999
read with Notification No. FEMA 22/2000-RB dated M ay 3, 2000.
So if the foreign c ompany wants to establish a Liaison Office in India, it c annot acquire immovable
property. However, the company needs to acquire property by way of lease not exceeding 3 years,
for its Liaison Office.
(ii) A non-resident Indian (NRI) or person of Indian origin (PIO), can inherit any immovable property in
India, whether it is residential or c ommercial. T he FEM A regulations applicable to transfer of
im movable property permit a NRI/PIO to acquire an immovable property in India by way of gift from
a person resident in India provided the property is not agricultural land/ farm house/ plantation
property. All gifts to NRIs will be considered as income accruing in India and would be taxed as per
the norm al slab rates applicable to resident Indians. T his m eans that the NRI/PIO needs to
m andatory pay the applicable tax
A liaison office of a foreign c ompany in India should be established only with requisite approvals
wherever necessary, and is eligible to take any immovable property on lease, in India which is

© The Institute of Chartered Accountants of India


t.me/cafinal6d - telegram link
Page 384
nec essary for its activities, provided that all such applicable laws, rules, Regulations or directions
in forc e are duly c omplied with.
But in the aforementioned both the cases the property is bought in c ontravention of the provisions
of benami ac t hence it is a benami transaction. It is liable to attract section 3 of Prohibition of
Benami Property T ransactions Ac t. T he property is liable to attac hment by the Adjudicating
Authority.
A liaison office of a foreign c ompany in India should be established only with requisite approvals
wherever necessary, and is eligible to take any immovable property on lease, in India which is
nec essary for its ac tivities, provided that all such applicable laws, rules, regulations or directions
in force are duly c omplied with. Since the property liable to be leased is in brought in c ontravention
to the provisions of law applicable in India, it c annot be leased. If any c ase it is leased without the
knowledge of the company the lease stand terminated. In c ase of gift to an NRI niece the gift stands
null and void.
Answer 2
i. A Benami transaction is an arrangement, where a property is transferred to, or is held by, in the
nam e of a person c alled Benamidar and the consideration for such property has been provided, or
paid by another person c alled beneficiary; and that property (benami property) is held for the
im mediate or future benefit, direct or indirect, by that person who has provided the consideration.
In the particular case the consideration is paid by M r Suresh Agarwal but the property is purchased
in his m other's name.
So it is c lear that, Mr Suresh Agarwal has done this benami transactions in respect of the holding of
the property, with the object of c oncealing the real owner.
ii. In case, if after the issuance of the initial notice by the Initiating Officer, the property in question is
transferred to a third party secretively, than the said transaction would be null and void and confiscation
would take effect, notwithstanding such a transfer.
However, in c ase where the person proves that, the property is held or ac quired by a person from
the Benamidar for:
a. adequate consideration, prior to the issuance of show-cause notice by the Initiating Officer
b. Without his having knowledge of the benami transaction in such a case the transfer may be valid.
iii. Effect of confiscation of benami property:
Where an order of c onfiscation has been made, all the rights and title in suc h property will vest
absolutely in the Central Government. It will be free of all encumbrances and no c ompensation will
be paid in respect of such confiscation.

10

© The Institute of Chartered Accountants of India


t.me/cafinal6d - telegram link
Page 385
Test Series: March, 2019
MOCK TEST PAPER
FINAL (NEW) COURSE: GROUP – II
ELECTIVE PAPER 6D: ECONOMIC LAWS

Attempt any two out of three case study based questions.


Each Case Study carries 50 Marks.
Time Allowed – 4 Hours Maximum Marks – 100

QUESTION 1
Mr. Mukesh Kumar, a wealthy merchant, having inherited his business from his forefathers, ran
a reputed jewellery shop titled as M/s. Kumarsons Jewarat in Dariba Kalan at old Delhi. He had
two daughters Reena and Ritika who were married and happily settled. He also had three sons.
Customers from far and near came to his shop to purchase gold ornaments, gold coins and
other items made of gold and silver. He then thought of expanding his business and thereby
purchased a shop on ground floor in up-scale South Extension area of South Delhi. The payment
of Rs. 3.20 crore was made by cheque and title deeds of the property were registered in his
name after making payment of appropriate stamp duty.
The eldest son Dheeraj Kumar managed the jewellery shop in old Delhi while the second son
Prashant Kumar helped him in new shop opened at South Extension. The youngest son Ronit
Kumar was undergoing a two-year jewellery designing course from Manchester Metropolitan
University, UK.
Mr. Mukesh Kumar was desirous of purchasing a residential property in New Friends Colony, a
high-end area of South Delhi. For this purpose, he engaged a property dealer and within a month
was able to finalise a house on ground floor. It was registered in his name and payment towards
purchase consideration, stamp duty etc. to the extent of Rs. 7.20 crore was made by cheque.
After purchasing the property he felt content and happy that he was the owner of a property
which was situated in a prime area of Delhi. He spent some funds in renovating the bungalow,
furnished it properly and thereafter gave it on lease to a nationalized bank for the purpose of
Guest House where bank’s officials visiting Delhi could be accommodated.
Time flew by. Encouraged by the loyalty of the customers he entered into diamond jewellery
business also at his South Extension shop. He entrusted this section independently to his
second son Prashant Kumar.
During the course of the years, he accumulated lots of funds which he wanted to invest safely
with a view to earn higher returns. He, therefore, consulted his close friend Mr. Roopesh Dutt

© The Institute of Chartered Accountants of India


t.me/cafinal6d - telegram link
Page 386
who was a practicing lawyer. He advised him to purchase properties in UP or Rajasthan because
investment in properties was a safe bet and over a period of time it would give higher returns
also. Mr. Dutt also told Mr. Mukesh regarding existence of the Benami Transactions (Prohibition)
Act, 1988 which was recently renamed as the Prohibition of Benami Property Transactions Act,
1988 (PBPT Act). The re-naming of the Act was done by enacting the Benami Transactions
(Prohibition) Amendment Act, 2016 with effect from 1st November, 2016. He also apprised him
of that benami transaction provisions were attracted to any property, shares, debentures, fixed
deposits and bank accounts if they were held by one person for another who actually paid for
the transaction. Anyway, he purchased a plot in Greater NOIDA for Rs. 65 lacs in the name of
his wife Mrs. Rama Devi by paying lesser amount of stamp duty as a relaxation was given to
the women. His wife was a homely lady with a religious bent of mind. Her time was spent in
running the house and looking after the needs of her family. In her spare time she used to read
religious books.
His youngest son Ronit, who also did a part time job in the hospitality sector besides studying
for his jewellery designing course in the UK used to remit his savings into his NRE Account
maintained in India. As Mr. Mukesh Kumar was always enterprising in nature, he was
contemplating to use those funds also for investment in property. Ronit was also agreeable to
this proposition. They searched for a suitable property in Dwarka area of Delhi. As luck would
have it, their neighbour Mr. Narain had a society flat in Dwarka which he wanted to sell. Mr.
Mukesh Kumar came to know of this fact and arranged a visit to the flat along with his two sons.
It was an east-facing flat and they liked the location. By this time Ronit had also come to India
for a short duration. The matter was discussed at length and all agreed to pay Rs. one crore
and ten lacs to Mr. Narain being the amount demanded by him. To fund Rs. 1.10 crore, Ronit
contributed Rs. 10 lacs from his NRE account while his elder brothers Dheeraj and Prashant
agreed to pay Rs. 20 lacs each. Dheeraj arranged funds by taking pre-mature payment of his
two fixed deposits kept in the bank while Prashant sold his quoted shares in NSE. They
managed to procure a cheque of Rs. 30 lacs from somewhere while remaining Rs. 30 lacs were
paid in cash by Mr. Mukesh Kumar. The property got registered for Rs. 80 lacs jointly in the
names of all the three brothers and accordingly stamp duty was paid.
Mr. Mukesh Kumar still had lot of spare funds for investments but such funds were not shown
as his income from business while filing the income tax returns from time to time. He had a
trusted cook by the name Rampal in his family who would follow the instructions of his master
tooth and nail without judging the consequences of following such instructions. Mr. Mukesh
Kumar was keen to purchase another property in Rajasthan with a view to rent it out to some
wealthy person. His close friend Mr. Vinay Seth informed about availability of a commercial
property in Jaipur. After negotiations with the owner, he purchased the same for Rs. 52.45 lacs
in the name of Rampal who could barely sign without knowing the import of signing the
documents. Mr. Mukesh Kumar was of the opinion that he would get the property transferred in

© The Institute of Chartered Accountants of India


t.me/cafinal6d - telegram link
Page 387
his name from Rampal after some time. Within no time the property was let out to an influential
person at a handsome rent.
I. Objective Type Questions (2 marks each)
Select the most appropriate answer from the options given for each question:
1. Mr. Ram, a NRI, purchased a flat for Rs. 30, 00, 000 and paid 20,00,000 in by account
payee cheque of his own account and rest in cash. The registry was done at a value of Rs.
20,00,000 which was paid by cheque. State the correct statement in the light of the given
facts:
(a) Said transaction is benami transaction due to Registry made on partial price of the
property.
(b) Due to partial payment of cash, the said transaction is benami transaction.
(c) Mr. Ram paid the consideration and owned the property in his name , so it is not a
benami transaction.
(d) Both (a) & (b)
2. Benami Property is -
(a) any property which is not the subject matter of a benami transaction.
(b) any property which is the subject matter of a benami transaction and also includes
the proceeds from such property.
(c) any property which is not the subject matter of a benami transaction and also does
not include the proceeds from such property.
(d) None of the above
3. A person who finances the deal in respect of benami property:
(a) is not the beneficial owner of the property
(b) is the beneficial owner of the property
(c) is not the possessor of the property
(d) None of the above
4. How much of the property purchased in Dwarka could be considered as Benami Property:
(a) Rs. 30 lacs
(b) Rs.60 lacs
(c) Rs. 80 lacs
(d) Rs. 1.10 crore

© The Institute of Chartered Accountants of India


t.me/cafinal6d - telegram link
Page 388
5 Benamidar is -
(a) a fictitious person who does not make payment
(b) a person in whose name the benami property is transferred or held but he does not
make payment
(c) a person who lends his name but does not make any payment.
(d) All of the above
6. Beneficial Owner is -
(a) a person, whether his identity is known or not, for whose benefit the benami property
is held by a benamidar.
(b) a person, who is benamidar and holds the property.
(c) Both (a) and (b)
(d) person who is valid legal holder of the property.
7 Under which Act, Ronit is maintaining his NRE account-
(a) Prevention of Money Laundering Act, 2002
(b) Real Estate (Regulation and Development) Act, 2016
(c) Foreign Exchange Management Act, 1999
(d) Prohibition of Benami Property Transactions Act, 1988
8 Why does a person enter into a Benami transaction?
(a) To avoid statutory dues
(b) To avoid payment to creditors
(c) To defeat the provisions of any law.
(d) All of the above
9 According to Section 53 of the Act who shall be punishable in the case where Mr. Mukesh
Kumar purchased property in the name of his cook Rampal:
(a) Rampal
(b) Mukesh Kumar
(c) Vinay Seth
(d) Both (a) and (b)
10 Can Mukesh Kumar get the property purchased in name of his cook Rampal, re-transferred
in his name:
(a) Yes, he can.
(b) No, he cannot.

© The Institute of Chartered Accountants of India


t.me/cafinal6d - telegram link
Page 389
(c) Yes, he can if Rampal agrees
(d) None of the above
II Descriptive Questions (10 marks each)
Q.1 (a) Can the property purchased by Mr. Mukesh Kumar in the name of his wife Mrs. Rama
Devi be considered as ‘Benami Property’? Justify. (6 Marks)
(b) Whether the property purchased in the name of Rampal by Mr. Mukesh Kumar can
be considered as ‘Benami Property’. State with reasons. (4 Marks)
Q.2 Identify beneficial owner and benamidar in all the above property transactions taken place
in this case study. (10 Marks)
S. No. Property Transaction at

1. South Extension

2. New Friends Colony

3. Greater NOIDA

4. Dwarka

5. Jaipur

Q 3 Mr. Raj entered into a purchasing of a property in the name of his wife. This transaction
was made in consultation with his legal counsel. He suggested him various ways of
nonpayment of statutory dues levied on the transaction. Examine the following given legal
situations –
Who shall be liable in the above transaction?
Punishment to levied
What formality shall be required for the sanction of the punishment? (10 Marks)

© The Institute of Chartered Accountants of India


t.me/cafinal6d - telegram link
Page 390
QUESTION 2
Madhuram Shantaraman is the Chairman and Managing Director (CMD) of M/s. Pearl
Stevedores Pvt. Limited (PSPL) having its registered office in Hyderabad, Andhra Pradesh. In
his earlier years, after obtaining his Bachelor of Science degree from Osmania University,
Hyderabad, he had gone to Australia to join Australian Maritime College (AMC) where he did a
three-year course of ‘Bachelor of Global Logistics and Maritime Management’ under the
supervision of his grandfather Shri T.S. Sukumar who was a part of senior management cadre
in a leading shipping company. Thereafter, he joined a shipping company in Australia itself on
a handsome salary and obtained vast experience. However, he always had a profound love for
India and also had an intense desire to become an entrepreneur on Indian soil. Therefore, he
returned back to India to start PSPL. After becoming resident in India, he opened a Residence
Foreign Currency Account with Canara Bank, Hyderabad and credited all his foreign earnings
into this account. After about four years of running PSPL successfully and keeping various
expansion plans in his mind, he purchased a commercial property in Australia out of the funds
kept reserved in the RFC account. For the time being, he let out that property to an individual
who was well known to his grandfather. However, his ultimate intention was to make this
property the overseas office of PSPL in future.
Madhuram’s company PSPL, inter-alia, is engaged in the business of stevedoring and intra-port
transportation of various types of cargo that are imported to and exported from India within the
premises of Visakhapatnam Port and for which the company has valid stevedoring license. To
conduct stevedoring operations PSPL deployed its own equipments like grabs, tugs, slings and
shackles but arranged dumpers and hywas either from Dumper Owners’ Union (DOU) active at
Visakhapatnam Port or if necessary, from alternate sources also. It also has operations but at
lower scale at Chennai Port and at L&T Port, Kattupally. Madhuram’s younger brother Shibu
Shantaraman is another director who looks after day-to-day affairs of the company whose
authorised and paid-up share capital is Rs. 25 lacs.
Vishakhapatnam Port is one of the leading ports in India and has three harbours viz., outer
harbour, inner harbour and the fishing harbour. The Port administration has passed through
different departments and Ministries of the Government of India till its transfer to the Port Trust
in February, 1964 under Major Port Trusts Act 1963. Thus, Vishakhapatnam Port Trust (VPT) is
the sole authority managing all activities being carried out in the port premises.
DOU at Visakhapatnam Port is an association of 250 dumper owners registered under the
Societies Registration Act, 1860 and has been working as a facilitator between the licensed
stevedores, whether listed or unlisted with the DOU and owners of dumpers and hywas for
handling of cargo and their intra-port transportation. Being a major entity for sourcing dumpers
and hywas by the licensed stevedore companies, it enjoyed monopoly position since it used to
give preference to the listed stevedores while supplying dumpers and hywas owned by its

© The Institute of Chartered Accountants of India


t.me/cafinal6d - telegram link
Page 391
members. The unlisted stevedores off and on had tough time to obtain dumper services from
the DOU and at the same time it was equally difficult for them to arrange large number of
dumpers from alternate sources when DOU did not oblige them.
Due to space constraints and other operational reasons VPT had capped the number of dumpers
to be deployed in the Visakhapatnam Port premises at 320 and all those dumpers belonged to
the 250 members of the DOU.VPT was issuing entry permits to the members of DOU as well as
non-members for using their dumpers and hywas in the port premises for intra-transportation of
cargo. However, DOU unofficially had a say in getting issued the entry permits to its members.
The business of PSPL was more or less going on smoothly except for some abrupt happenings.
For every standard shipment, PSPL required 40 to 45 dumpers and hywas at a time but DOU,
on one pretext or the other, never obliged it because the company was not enlisted with DOU
and consequently it was highly arduous on its part to engage such large number of dumpers,
that too at higher rates than usual, from different sources by contacting the individual dumper
owners. From time to time, PSPL requested DOU to enlist it but the company was never enlisted.
This reflected monopolistic and exploitative behaviour of the DOU and led to anti-competitive
practices in the VPT.
This issue once again took a sharp upsurge when M/s. Chandernath Steel and Power Ltd.
(CSPL) appointed PSPL for its unloading and intra-port transportation of cargo comprising
49,500 MT of coking coal imported from Australia through the vessel MV Palateia. PSPL
requested DOU to supply dumpers but its request was turned down. As a result, the vessel was
stranded inside the Port and the cargo could not be unloaded and transported immediately.
PSPL was forced to hire dumpers from some alternate sources by paying 5% more than the
usual price. While PSPL was continuing with the discharge work from the vessel, a dumper
belonging to DOU reversed at a high speed and deliberately hit PSPL’s payloader which caused
huge damage and also broke the windshield of one of the dumpers engaged by it.
As this was not enough, another unpleasant incident took place after a gap of one month. This
time PSPL was appointed by Gopal Enterprises Pvt. Ltd. (GEPL) for intra-port transportation of
cargo comprising 10,000 MT of iron-ores imported through the vessel MV AquaMarine from
Japan. PSPL requested the President of DOU to supply thirty dumpers for unloading but once
again there was no positive response and therefore, PSPL took up the matter with authorities
of VPT which warned DOU not to refuse provision of dumpers to PSPL or otherwise face strict
action for its continuous non-performance of duties. However, the warning fell on deaf ears and
DOU continued with its dominant position activities.
Having no choice, PSPL again decided to engage dumpers from independent sources. DOU at
that time indiscriminately parked about eighty dumpers thereby blocking the roads to restrict the
movement of cargo discharged by the vessel MV AquaMarine and to halt other port operations.
VPT tried to diffuse the situation but of no avail. Due to the likelihood of emergence of a law and

© The Institute of Chartered Accountants of India


t.me/cafinal6d - telegram link
Page 392
order situation at Visakhapatnam Port, a meeting was convened by the Additional District
Magistrate with PSPL, DOU, GEPL and VPT. In that meeting ADM asked GEPL to consider
availing services of another stevedore for removal of cargo. Hence, PSPL had to forego the
execution of cargo-handling service of GEPL.
Madhuram, very much disturbed by these unfortunate happenings, decided to take legal action.
Accordingly, taking recourse to section 19 (1) (a) of the Competition Act, 2002 (in short, the
‘Act’), PSPL filed information in the prescribed manner along with requisite fee with Competition
Commission of India (in short, the ‘Commission’) against DOU alleging that DOU was restricting
and controlling the supply of dumpers for intra-port transportation of cargo at Visakhapatnam
Port and thereby was contravening the provisions of the Act including Sections 3 and 4. It was
further alleged that VPT, being the sole authority for managing the activities within the port
premises, had also contravened the provisions of the Act by issuing entry permits largely to the
members of DOU to operate within the port premises and in fixation of rates of providing services
of dumpers .
PSPL requested the Commission to cancel the registration of the DOU and declare its
arrangement with the VPT inside the Visakhapatnam Port as void; to discard anti-competitive
agreement which it believed to have existed between DOU, its members and VPT; to restrain
both the parties from abusing their dominant position and impose penalty as may be appropriate
keeping in view the wilful and deliberate abuse of dominant position by them; to direct these
parties to pay compensation to PSPL; and to pass such other order(s) as the Commission may
deem fit and proper.
Finding a prima facie case of violation of the provisions of the Act in this case, the Commission,
under section 26(1) of the Act, directed the Director General (in short, ‘DG’) to conduct an
investigation into the matter.
The DG conducted a detailed investigation by taking various submissions and evidences from
PSPL, VPT, DOU and third parties. Following pertinent issues were addressed in the DG report:
examination of the alleged abuse of dominant position by the DOU as per the provisions of
section 4 of the Act; and
examination of existence of any anti-competitive agreement between the DOU, its members and
VPT; fixation of rates of providing services of dumpers/hywas and the alleged contravention of
the provisions of section 3 of the Act.
examination of role of office bearers of DOU.
According to DG, for applicability of section 4, the DOU was to be considered as an enterprise
in terms of section 2(h). DG observed that DOU in itself was not directly engaged in the provision
of the services of dumpers and hywas for intra-port transportation of cargo, which was the
relevant market. Its activities were limited only to allocation of dumpers of its members to the

© The Institute of Chartered Accountants of India


t.me/cafinal6d - telegram link
Page 393
enlisted stevedores who requisitioned them. Even though DOU did not derive any monetary
considerations for the services so rendered and also it was not directly engaged in the provision
of the services of dumpers but the DOU, because of being engaged in the activity of provision
of services of allocation of dumpers, was an 'enterprise' in terms of section 2(h).
However, the DOU as an enterprise was operating in a different market i.e., in the market of
provision of services of allocation of dumpers; not in the market of providing services of dumpers
for intra-port transportation of cargo. Accordingly, DOU was a non-player in the relevant market
and its conduct was not liable to be examined under the provisions of section 4 of the Act.
As regards alleged contravention of the provisions of section 3, DG noted that by forming their
union, the members of DOU had entered into an agreement amongst themselves through a
Memorandum to pursue common objectives stated therein. DOU was allocating dumpers owned
by its members as per its sole discretion and because it was the single point source of supply
of dumpers for stevedoring, it was controlling the provision of dumper services inside the Port
area. Accordingly, when the PSPL and other stevedores had limited option to engage dumpers
from outside sources and when DOU denied services of dumpers to PSPL, the DOU had
restricted and controlled the provision of the said services. This was a clear contravention of
section 3(1) read with section 3(3)(b).
It was independently established by DG and also as contended by PSPL, the members of DOU
had formed a cartel to monopolise the services of dumpers and hywas inside the Port area
which was in contravention of the provisions of section 3 of the Act.
With regard to existence of an arrangement between DOU and VPT, as alleged by PSPL, the
DG reported that there was no evidence which could prove that DOU and VPT had entered into
any anti-competitive agreement to restrict the services of providing dumpers inside the Port
area. Further, VPT was not colluding regarding fixation of the rates of dumpers for intra-port
transportation as it could not derive any benefit out of such collusion. The role of VPT was to
provide necessary infrastructural facilities to the importers and exporters for smooth operations
of the port and nothing else.
DG also observed that DOU along with the enlisted stevedores was involved in the finalising of
rate of dumpers for intra-port transportation of cargo and such rates were made applicable to
all the members of the DOU. Further, the constitution of the DOU itself stated about having
uniform rates for providing dumper services. It further provided for constitution of ‘Rate
Committee’ which was authorised to finalise the rates for different work.
Thus, the members of DOU were not allowed to negotiate rates for providing dumper services
to their customers, i.e., stevedores and were forced to abide by the rates decided by DOU. This
action of DOU closed the independent decision making of individual members. Further, the rates
were enforced on unlisted stevedores also, as alleged by PSPL.

© The Institute of Chartered Accountants of India


t.me/cafinal6d - telegram link
Page 394
Further, DG also investigated the role of all the five office bearers i.e. President, Vice-President,
Secretary, Joint Secretary and Treasurer of the DOU and found that all of them were equally
complicit in the anti-competitive practices of controlling and restricting the provisions of dumper
services and determining the rates of dumpers for intra-port transportation of cargo.
After investigating the matter as above and noting his findings, the DG submitted the
investigation report to the Commission and as per section 35 also authorised the Additional
Director General to appear before the Commission as and when required.
Judgment of the Commission
As regards considering of DOU as an enterprise, the Commission was of the view that to qualify
as an ‘enterprise’, any person or department of government had to be engaged in the activity
specified under section 2(h). It was noted that the DOU was an association of dumper owners
and in itself it was only a facilitator of the services of dumpers and allocated dumpers of its
members. It was further observed that the DOU though engaged in allocating dumpers owned
by its members amongst stevedores, it neither owned them nor received any consideration for
the services of dumpers. Its activity therefore, could not be considered as the activity of an
enterprise as reported by DG.
In view of the fact that there was no evidence provided by PSPL and also that DG denied
existence of any anti-competitive agreement between DOU and VPT, it was held that VPT had
not contravened any of the provisions of section 3 of the Act.
As reported by the DG, DOU was a single point source of supply of dumpers because any
requisition for the dumpers had to be made by the stevedores to it and DOU had the exclusive
right to assign dumpers of its members to them. This way, DOU exercised absolute control over
the supply of dumpers owned by its members. Further, DOU allocated dumpers of its members
to the stevedores who were enlisted with it and refused to supply them to PSPL. This conduct
of DOU clearly established that DOU had restricted and controlled the provision of the said
services in contravention of the provisions of section 3(1) read with section 3(3)(b). Since, due
to space constraints VPT had capped the number of dumpers at 320 and all those dumpers
belonged to the 250 members of the DOU, DG reported that the options for users to source
dumpers from alternate sources were extremely limited and stevedores were mainly dependent
on the DOU for the said services.
Investigations found it correct that PSPL was denied dumpers by DOU for unloading and intra-
port transportation of cargo comprising 49,500 MT of coking coal imported from Australia
through the vessel MV Palateia.
It was submitted by DOU before the Commission that it was not a single point source of supply
of dumper and it was not restricting and controlling provisions of the service of dumpers for
intra-port transportation. Any stevedore could avail the services of dumpers of its members by

10

© The Institute of Chartered Accountants of India


t.me/cafinal6d - telegram link
Page 395
making a simple requisition to it. However, even though DOU might not be a single point source
of supply of dumpers to the stevedores, the Commission observed that the option of alternate
source of supply of dumpers for the stevedores was extremely limited. This was because of the
fact that VPT had capped the number of operational dumpers at 320 and these dumpers were
owned by 250 members of the DOU. Thus, the option available for the non-enlisted stevedores
for the said services was obviously restricted. It was observed that even though there was no
restriction imposed by DOU and VPT on the stevedores for making their private arrangement
for dumper services, the fact could not be denied that the control and management of all 320
dumpers of its members was in the hands of DOU and it used to manipulate issue of entry
permits to its advantage.
From the analysis of details of entry passes issued by VPT during the last one year to the
members of DOU and non-members, it was observed that out of total number of days for which
dumpers, etc., were employed during the said period, the share of non-members was merely
10.74 % against 89.26% share of members of DOU.
As regards observation of DG that the rates were fixed by the DOU for providing services of
dumpers and hywas, the DOU contended that the rates were fixed after the concurrence of
suppliers (i.e. members of DOU) and the consumers (i.e. stevedores firms) but this contention
of the DOU did not appear to be tenable.
Thus, in view of these facts, it was held that the DOU had indulged in the practice of determining
the rates of the provision of dumper services for intra-port transport operations which amounted
to determination of sale price of the services of dumpers. This practice was in violation of the
provisions of section 3(1) read with 3(3)(a). Accordingly, the DOU was found to have
contravened the provisions of section 3(1) read with 3(3)(a) of the Act.
From the sequence of events, circumstances of the case and findings of DG in this regard, it
was amply clear that DOU was not only controlling the services of the provision of dumper inside
the Port area but also restricting the said services by denying it to the PSPL and other
stevedores who were not enlisted with it for the said service and who were dependent on it
because of limited availability of the other sources of supply of dumpers. Accordingly, the
Commission was of the view that the above said acts of DOU amounted to restricting and
controlling the provision of the services of dumpers which was in contravention of the provisions
of section 3(1) read with section 3(3)(b) of the Act.
In view of the findings recorded by the Commission, DOU and its office bearers were to be
directed to cease and desist (i.e. discontinue) from indulging in the acts which had been found
to be in contravention of the provisions of section 3.
Having established that DOU had contravened the provisions of section 3(1) read with sections
3(3)(a) and 3(3)(b) of the Act, as per the provisions of section 48 of the Act, it was to be

11

© The Institute of Chartered Accountants of India


t.me/cafinal6d - telegram link
Page 396
determined whether the office bearers of the DOU could be held liable for the above said
contravention.
In this regard, the Commission viewed in case of union of enterprises, liability for anti-
competitive conduct would be two fold. This implied that besides the union, the anti-competitive
decision of the union could also be attributed to the members who were responsible for running
the affairs of the union and actively participated in giving effect to the anti-competitive decision.
DG investigation had already found that all the five office bearers of the DOU were equally
complicit in the anti-competitive practices of controlling and restricting the provisions of dumper
services and determining the rates of dumpers for intra-port transportation of cargo.
Even though the office bearers of the DOU denied the allegations but they had not brought
anything on record which could absolve them from their responsibility in terms of section 48 of
the Act. Therefore, in concurrence with the DG findings it was to be viewed that the office
bearers were equally responsible along with the DOU in the anti-competitive practices of
controlling and restricting the provisions of dumper services for intra-port transportation of cargo
which was in contravention of the provisions of section 3 (1) read with section 3 (3) (b) of the
Act and determining the rates of providing services of dumpers and hywas for intra-port
transportation which was in contravention of the provisions of section 3 (1) read with section 3
(3) (a) of the Act.
Furthermore, in terms of section 27(b) of the Act, the Commission may impose such penalty
upon the contravening parties, as it may deem fit which shall be not more than ten per cent of
the average of the turnover for the last three preceding financial years, upon each of such person
or enterprises which are parties to such agreements or abuse.
The Commission was of the considered view that the said anti-competitive conduct required to
be penalized to cause deterrence in future among the erring entities. Therefore, it was
imperative that the penalty imposed was adequate enough to create desired level of deterrence.
Accordingly, in absence of any mitigating factor, the Commission, in exercise of powers under
section 27 (b) of the Act, decided to impose penalty on the DOU at the rate of 8% of their
average turnover for the last three preceding financial years which when calculated worked out
to Rs. 2,40,584.
As regards individual liability of the office-bearers of the DOU in terms of section 48 of the Act,
the Commission decided to impose penalty on them because they were found to be responsible
for the conduct of the DOU. Resultantly, the Commission decided to impose penalty @ 5% of
the average income of the last three financial years and the Commission was of the view that
imposition of penalty @ 5% on the office bearers would cause deterrence effect in future among
the erring persons engaged in such acts. The total amount of penalty on each of the office
bearers i.e. President, Vice-President, Secretary, Joint Secretary and Treasurer worked out to
Rs. 32,350, Rs. 31,120, Rs. 33,850, Rs. 13,065 and Rs. 10,795 respectively.

12

© The Institute of Chartered Accountants of India


t.me/cafinal6d - telegram link
Page 397
The Commission also directed that the penalty would be paid by the DOU and its office
bearers within 60 days from the date of receipt of copy of this order.

As per Section 39, if the Commission was of the opinion that it would be expedient to recover
the penalty (as imposed under the Competition Act) from DOU and its office bearers in
accordance with the provisions of the Income-tax Act, 1961, it was authorised to make a
reference to this effect to the concerned income-tax authority under that Act for recovery of the
penalty as tax due under the said Act.
I. Required (MCQ of 2 marks each , Total of 20 Marks)
Select the most appropriate answer from the options given for each question:
1 Whether Dumpers Owners’ Union (DOU) can be considered as an enterprise under section
2(h) because it is a facilitator of the services of dumpers and for that purpose it allocates
dumpers of its constituent members but it neither owns any dumpers nor receives
consideration for the services of dumpers owned by its members.
(a) Yes, it can be considered an enterprise;
(b) No, it cannot be considered an enterprise;
(c) Can’t say
(d) None of the above
2 An agreement which restricts competition in the relevant market is known as:
(a) Fair-competitive agreement;
(b) Competitive agreement;
(c) Pro-competitive agreement;
(d) Anti-Competitive agreement
Q.3 At a port all the dumper owners formed an association which along with its members
decided the rates at which the dumper services for unloading the cargo would be provided
to the licensed stevedores at the port. Whether association and its members is a cartel?
(a) Yes, it is a cartel;
(b) No, it is not a cartel;
(c) Cannot say;
(d) None of the above.
Q.4 DOU was in the market of provision of services of allocation of dumpers which was -------
-----------.
(a) the relevant market;
(b) not the relevant market;

13

© The Institute of Chartered Accountants of India


t.me/cafinal6d - telegram link
Page 398
(c) cannot be ascertained;
(d) none of the above.
5 What did PSPL do to initiate inquiry against DOU as well as VPT for their wrongdoings
which resulted in contravention of the provisions of the Competition Act?
(a) filed a case with the Commission;
(b) filed information with the DG;
(c) filed information with the Commission;
(d) filed information with the Andhra Pradesh Government.
6 Is it mandatory for the DG to appear before the Commission in person?
(a) Yes, it is mandatory;
(b) No, it is not mandatory;
(c) As directed by the Commission;
(d) None of the above.
7 If the Commission is of the view that it is expedient to recover penalty imposed under the
Competition Act as per the provisions of the -----------, it is empowered to make a reference
to this effect to the concerned authority.
(a) FEMA;
(b) Income-tax Act;
(c) PMLA;
(d) None of the above
8 What is the maximum penalty which the Commission could impose on the DOU and its
office bearers for the contravention of the relevant provisions of the Competition Act?
(a) 8% of their average turnover for the last three preceding financial years;
(b) 9% of their average turnover for the last three preceding financial years;
(c) 10% of their average turnover for the last three preceding financial years;
(d) 10% of their average turnover for the last five preceding financial years;
9 While being resident in India after returning back from Australia, Madhuram purchased a
commercial property in Australia utilizing his funds kept in -----------.
(a) Non-resident (External) Account
(b) RFC Account
(c) EEFC Account
(d) NRO Account

14

© The Institute of Chartered Accountants of India


t.me/cafinal6d - telegram link
Page 399
10 The Competition Commission of India, if on receipt of an information under section 19 from
a consumers’ association for alleged contravention of section 3 (1) by the opposite party,
is of the opinion that there exists a prima facie case, it shall direct the ----- to cause an
investigation into the matter.
(a) President of the association giving information;
(b) Director General appointed under section 16;
(c) Representative of the opposite party;
(d) Managing Director.
II. Descriptive Questions of 30 Marks
Q.1 (a) What do you mean by anti-competitive agreements? (5 Marks)
(b) How did DOU and its members cause adverse impact on the competition in the Port
area? (5 Marks)
Q.2 Explain as to why Section 4 relating to ‘abuse of dominant position’ was not made
applicable by the Commission in the above case study? (10 marks)
Q.3 What can be the orders passed by the Competition Commission of India after the inquiry?
Also identify the relevant order passed by the Commission in the above case study.
(10 Marks)

15

© The Institute of Chartered Accountants of India


t.me/cafinal6d - telegram link
Page 400
Question 3
Gupta’s were a famous business class family living in Delhi. Mr. Gupta was involved in
diversified businesses like property, shares, money lending, hotels etc. He had three children,
one daughter and two sons. His daughter Aarushi was the eldest one and after completing her
graduation from Lady Shri Ram College for Women went to London to pursue her M.B.A. After
completion of her course she came back to Delhi and got engaged to Deep. Deep was a Pune
based Civil Engineer involved in construction business. Mr. Gupta’s elder son Aayush was doing
his graduation from Shri Ram College of Commerce and after his graduation he was planning
to go to US for his masters degree. Mr. Gupta’s youngest son Aaditya was studying in class XII
and wanted to become a Chartered Accountant. Mrs. Gupta was a religious lady and a perfect
home maker. Although she was a home maker yet she had a complete knowledge of their family
business and was also a director in some of Mr. Gupta’s companies.
Aarushi’s wedding was fixed for April 17; 2017.As her wedding was approaching, whole Gupta
family was busy in preparing for that. In the month of January 2017 they all went to U.S.A. for
shopping and had a family vacation for 15 days. They came back to India on 1 st February 2017.
At the time of going abroad, Mr. Gupta availed the foreign currency exchange facility for US$
50000 for meeting all the expenses of foreign tour. In India the most expensive banquet hall
was booked as the wedding venue. Shopping of jewellery and clothes was also in full swing.
Payment for both jewellery and clothes were made mostly in cash. During these preparations
one of the jeweler from whom some jewellery was purchased in cash was raided by the Income
Tax Department over cash sale of Gold and jewellery and deposition of high amounts of cash
in banks in different accounts during the period of demonetization.
During this raid information/details pertaining to Gupta family’s cash purchase also revealed.
This information worked as evidence against Gupta’s as the Department was already keeping
an eye on Mr. Gupta. Mr. Gupta too had done cash purchases of gold and jewellery during the
period of demonetization and was also involved in deposition of cash in bank during that period.
Apart from that the Department was also having a tip about Mr. Gupta’s some other business
transactions.
After that jeweler, a raid was also conducted at Mr. Gupta’s house on 23 rd March. As they were
preparing for Aarushi’s marriage a lot of cash and jewellery was lying in the house. Besides
cash in Indian currency around 10000 US $ were also found. Cash amounting to Rs. 2750000/-
, jewellery worth Rs. 3 crores, property papers of nearly 10 properties consisting of plots, flats
and some fixed deposit receipts were seized. A diary consisting of many details of Mr. Gupta’s
various cash transaction was also found.
Due to this raid all the plans of Aarushi’s marriage spoiled and marriage was postponed. Mrs.
Gupta who was a diabetic and high blood pressure patient couldn’t bear this entire incident and
got a paralytic attack. She was rushed to the hospital and doctors suggested that she should be

16

© The Institute of Chartered Accountants of India


t.me/cafinal6d - telegram link
Page 401
taken to U.S.A. for better and fast treatment. Mr. Gupta agreed upon that and asked his younger
brother Sunil, who was settled in U.S.A. to make all the necessary arrangements. Mrs. Gupta
and Aarushi went to America for her treatment. In April 2017 Mr. Gupta arranged for money and
again got US $ 100000 exchanged for her wife’s treatment in USA and remitted the amount to
his brother.
In May 2017 Aayush got admission in a management school in US and for the purpose Mr.
Gupta remitted 20000 US $ as his admission fee to the institute. By the end of May 2017(On
27 th May) Mrs. Gupta and Aarushi came back to India and a sum of US$ 7000 was lying with
them out of the amount remitted for her treatment. In August 2017 Aayush went to USA for
pursuing his studies. Mr. Gupta gave him 2000 US $(out of 7000 US$ unspent and brought back
to India by Mrs. Gupta) for his petty expenses. Mr. Gupta opened a foreign currency account
with a bank in USA for making remittances to Aayush to meet his different expenses there under
Reserve bank of India’s Liberised Remittance Scheme and further a sum of US $ 50000 was
deposited in the same.
Life was coming back to normal. Mrs. Gupta was feeling better now and Aarushi started going
to Mr. Gupta’s office to help him in his business. Mr. Gupta had asked Deep to look for a land
upon which he wanted to construct a small residential tower with all the modern amenities and
facilities. Deep contacted some property dealers of Pune for the land. One of the dealers told
him about a land situated in nearby areas of Pune. He visited the land and liked it very much.
He contacted Mr. Gupta and asked him to visit the land. Mr. Gupta along with Aarushi visited
the land and they too liked it.
After having a meeting with the land owner deal of the land was finalized for Rs. 1.5 crore. Mr.
Gupta decided to pay Rs. One crore through cheque and balance in cash. To arrange for the
said transaction of Rs. 1.5 crore Mr. Gupta decided to sell one of his inherited properties. In
2014 through his mother‘s will he got a plot and a small house in Noida’s prime location. Mr.
Gupta sold the plot for Rs. 1.80 crore. For this deal he took Rs. 70 lacs in cash and balance
through cheque. The cheque amount was utilized to pay for the Pune land through cheque and
cash was utilized for cash payment for the land. A brokerage of Rs. 5 lacs was paid to the
property dealer of Pune and Rs. 3 lacs in cash was paid to Noida’s property dealer who helped
him in getting the buyer and balance cash he gave to Deep to meet out some other expenses.
Since it was intended that the said land will be used for building a residential tower consisting
of 12 units and the area of land was more than 500 square meters a registration under The Real
Estate (Regulation and Development) Act, 2016 was necessary. Deep and Aayushi were given
the full charge of the land and construction upon it. On 1 st July 2017, an online application was
submitted to MAHA RERA for project registration along with all the necessary documents. The
project was named as DEVALAYA and Mr. Gupta and Deep were named as the Promoters.

17

© The Institute of Chartered Accountants of India


t.me/cafinal6d - telegram link
Page 402
On 14 th July the project got its registration and after that the marketing of the project started.
Since the project was equipped with all the modern amenities and Deep’s name was attached
to it, buyers took a great interest in the project and all the 12 units were sold out within few
months of publication of the advertisement. The construction work started on a promised date
i.e.1 st August 2017 and it was promised that by 31 st December 2017, 80%of construction work
will be completed. All the formalities related to agreement of sale with prospective buyers were
duly fulfilled. The selling price of each flat was fixed at Rs. One crore and a legitimate signing
amount was received from the prospective buyers.
Project completion period was 06 months with a grace period of one month. Buyers were
required to pay the amounts as per the stages of completion of the project .As promised by the
promoters by 31 st December project was 80% complete and all the buyers except one Mr.
Bhatia, paid the amount dues as per schedule. Flats were ready for possession by 15 th February
2018.All the buyers paid the full amount towards their flats ,but Mr. Bhatia for one or other
reason did not pay more than 60% i.e. 60 lacs towards his flat.
The promoters of the Devalaya got completion certificate and occupancy certificate in due time
and handed over the possession to the buyers along with all the necessary documents related
to the title and ownership of the flats; except Mr. Bhatia, as he had not made full payment
towards his flat, despite sending him reminders for payment. When Mr. Bhatia came to know
about the possession of the flats he contacted Deep and demanded for possession. Deep asked
Mr. Bhatia to pay the balance amount together with interest for delay period to get the
possession. Mr. Bhatia rejected Deep’s demand of paying the interest saying that he is ready
to pay the balance amount but not the interest as he is legally not bound to do so. Deep tried to
convince him to pay the interest but he continuously refused so after due discussion with Mr.
Gupta, Deep handed over the possession to Mr. Bhatia also when he released the full and final
payment towards the flat purchased by him.
On the other hand in Delhi the raid which was conducted upon Mr. Gupta’s house and cash,
jewellery, US dollars etc. seized, made Mr. Gupta answerable to the Income tax Department.
Diary found at Mr. Gupta’s house revealed many cash transaction of Mr. Gupta. It also had
mention of cash deposits of Rs. 40 lacs during the period of demonetization in about 30 accounts
of different people, which included some of his servants, their spouse and children and some
office staff. All these accounts were opened under Pradhanmantri Jan Dhan Yojna and had a
very few transaction before such deposits. The officials also recovered pass books of the same.
Mr. Gupta could not offer any satisfactory reply for holding pass books of their aids.
As officials had found some property papers also, upon scrutiny of the papers it was found that
he had purchased three plots and three flats for each of his three children to secure their future.
Yet the amount of Rs.30 lacs each for plot and Rs.80 lacs each for flats as shown in Registry
papers was not justifiable considering the locations of the said properties. The fair market value

18

© The Institute of Chartered Accountants of India


t.me/cafinal6d - telegram link
Page 403
of those plots and flats were approx. 55 lacs and 1.2 lacs each respectively. Apart from this he
had purchased two plots amounting to Rs. 30 lacs in the name of one of his female servant.
Although he couldn’t explain source of funds for any of the plots purchased.
Upon asking about cash recovery and cash purchases of jewellery and clothes he could not give
any answer. Although he was having bill for each and every purchase yet all the payments were
made in cash and he could not prove the veracity or legitimate source of cash.
On the basis of information about their bank lockers from the diary, lockers were opened and
many fixed deposit receipts, jewellery and cash was recovered from there. Jewellery found from
the locker had some markings on them; for which he clarified that he used to lend money on the
mortgage of jewellery and those marks were related to that lending. A detail of amount landed
was also recovered from his home.
For 10000 US $ found at his home he said that they had returned from a foreign tour recently
and he was thinking to get them exchanged from Authorised Dealer but could not get time to do
so as was busy in wedding preparations. However he produced a bill of purchase of foreign
currency from an Authorised dealer.
I. Required (MCQ of 2 marks each , Total of 20 Marks)
Select the most appropriate answer from the options given for each question:
1 .How much foreign currency can be held by a resident in India after returning from a
business trip abroad?
(a) 2,000 US $
(b) 5,000 US $
(c) 10,000 US $
(d) None of the Above
2 How much foreign exchange in the form of coins can be held by a resident individual in
India?
(a) 2,000 US $
(b) 5,000 US $
(c) None of the above
(d) Without any limit
3 In the given case how much more foreign currency can be exchanged by Mr. Gupta without
taking prior permission/approval from Reserve Bank of India?
(a) 30,000 US$

19

© The Institute of Chartered Accountants of India


t.me/cafinal6d - telegram link
Page 404
(b) 80,000 US$
(c) There is no such limit
(d) He has exhausted his limit
4. Suppose in the given case if any real estate project was under construction/on going on
the date of commencement of RERA (for which occupancy /completion certificate was not
received) by which date that project should have get its RERA registration?
(a) By 1 st August 2016
(b) By 29 th July 2017
(c) By 1 st May 2017
(d) By 29 th July 2016
5. In the given case Mr. Gupta can be booked under ---
(a) The Real Estate (Regulation & Development)Act,2016
(b) The Foreign Exchange Management Act,1999
(c) Prohibition Of Benami Property Transaction Act,1988
(d) All of above
6 .In the given case Mr. Gupta can be prosecuted under Prohibition of Benami Property
Transaction Act, 1988. What can be the quantum of fine in case of plots purchased by him
and seized during the raid?
(a) Rs. 5,62,500/-
(b) Rs.56,25,000/-
(c) Rs. 2,25,00,000/
(d) Rs.22,50,000/-
7. Which sentence is true about Mr. Bhatia being an allottee-
(a) Mr. Bhatia will have to share the registration charges, municipal taxes, water and
electricity charges, maintenance charges ground rent charges etc.
(b) Mr. Bhatia will have to pay outstanding amount together with interest, at such rates
as may be prescribed.
(c) After paying all the dues, he will have to take the possession of the flat within 2 months
from the receipt of the occupancy certificate.
(d) All of above

20

© The Institute of Chartered Accountants of India


t.me/cafinal6d - telegram link
Page 405
8. With reference to the RERA ; which of the following sentence/s are correct?
(a) The appropriate Government shall, within a period of one year from the date of coming
into force of this Act, establish an Authority
(b) The appropriate Government of two or more states or Union territories can not
establish one single Authority, even if it deems fit
(c) The appropriate Government may not establish more than one Authority in a state or
Union territory, even if it deems fit
(d) The appropriate Government shall establish authority after 1year from the date of
enforcement of the Act.
9. Disputes related to real estate projects covered by the vicinity of RERA can be heard in
which of the following hierarchy—
(a) RERA Authority Appellate Tribunal High Court
(b) RERA Authority Central Advisory Council Appellate Tribunal
(c) Central Advisory Council RERA Authority Appellate Tribunal High Court
(d) RERA Authority Central Advisory Council Appellate Tribunal High Court
10. Cash was found by the Income Tax Authorities at the time of raid conducted at Mr. Gupta’s
house. Under which Act this cash can be seized/ retained by the department?
(a) Prevention of Money laundering Act ,2002
(b) Prohibition of Benami Property Transaction Act,1988
(c) Foreign Exchange Management Act,,1999
(d) Income Tax Act,1961
II. Descriptive Questions (30 Marks)
Q1 In the given case Mr. Gupta and Deep started a housing project in Pune City. Suppose
that you are a Real Estate Regulatory Authority and you are the one who examines
applications received for registration of the real estate projects. Before granting any
registration--
(A) Which key factor will you be required to check or on what basis you will judge any
application?
(B) What are the circumstances under which—
(1) Promoter/s applies for the registration of their project under RERA.
(2) Under which circumstances they are not bound to apply for registration
(10 Marks)

21

© The Institute of Chartered Accountants of India


t.me/cafinal6d - telegram link
Page 406
Q2 A raid was conducted at Mr. Gupta’s residence in relation to cash purchase of jewellery.
(A) Under which Act; he can be questioned? What can be the line of action against him?
(B) During the raid some documents were found and seized by the authorities. Supposing
you are an authority who is examining the case under the respective Act. Examine in
the given situations your power regarding seizing of those documents and
papers. (10 Marks)
Q 3 In India, a person can involve himself in foreign currency transactions, whether on account
of capital or current accounts; but within the limits prescribed by the Reserve Bank of India.
Can such person carry out these transactions on his own or he will require some channel
to deal in foreign currency .What are different provisions related to such channel. In whose
control/direction does this channel work? ( 10 Marks)

22

© The Institute of Chartered Accountants of India


t.me/cafinal6d - telegram link
Page 407
Test Series: March, 2019
MOCK TEST PAPER
FINAL (NEW) COURSE: GROUP – II
ELECTIVE PAPER 6D: ECONOMIC LAWS

SUGGESTED ANSWER
Case Study 1

MCQ’S
1: (c)
2 (b)
3 (b)
4 (a)
5 (d)
6 (a)
7 (c)
8 (d)
9 (d)
10 (b)
Descriptive Answers
Ans.1(a) The property so purchased in the name of Mrs. Rama Devi, whether a ‘Benami Property’
or not, will depend upon the facts/motive/intention of Mr. Mukesh Kumar for purchasing property in
such a way.
The property would be considered as benami property-
• If the intention to purchase property was only to save stamp duty; and
• It was intended not to give any ownership benefit to his wife.
Additionally, it is also to be proved in case of benami property that the owner of the property was not
aware of, or, denied knowledge of such ownership.
The property would not be considered as benami property-
• If the intention was to give ownership rights to Mrs. Rama Devi i.e. gifting of property to her;
and
• his wife was fully aware of purchase of property in her name by her husband.
In this case, as per the facts, the payment of lesser stamp duty is consequential to purchase of
property in the name of a woman. Hence, the property purchased on the name of Mrs. Rama Devi
is not a Benami Property.
Answer 1(b) Yes. It is certainly a case of benami property. The reasons are:
(i) The consideration for the property had not been given by Rampal though title deeds of the
property were registered in his name.

© The Institute of Chartered Accountants of India


t.me/cafinal6d - telegram link

Page 408
(ii) Rampal was not aware that the property was purchased in his name.
(iii) Rampal was not the real owner of the property since he did not make payment.
(iv) The intention of Mr. Mukesh Kumar was to utilize his undisclosed funds without involving
himself as owner of such funds and to enjoy benefits (i.e. rent) emanating from such property.
Answer 2

S. No. Property Transaction at Beneficial Owner Benamidar


1. South Extension Mukesh Kumar N. A.
2. New Friends Colony Mukesh Kumar N. A.
3. Greater NOIDA (i) Mukesh Kumar, if (i) Rama Devi
his intention was
not to give
ownership rights
to his wife.
(ii) Rama Devi, if his
intention was to
give ownership
rights to his wife. (ii) N. A.

4. Dwarka All his three sons Benami property to


extent of Rs. 30 lacs -
Fictitious person from
whom the cheque was
obtained is Benamidar.
5. Jaipur Mukesh Kumar Rampal
Answer 3
As per section 53 of the Prohibition on Benami Property Transactions Ac t, 1988, where any
person enters into a benami transaction in order to defeat the provisions of any law or to
avoid paym ent of statutory dues or to avoid payment to c reditors, the beneficial owner,
benamidar and any other person who abets or induces any person to enter into the benami
transaction, shall be guilty of the offe nce of benami transaction.
Whoever is found guilty of the offence of benami transaction referred to above shall be
punishable with rigorous imprisonment for a term which shall not be less than one year, but
which m ay extend to seven years and shall also b e liable to fine which m ay extend to twenty-
five per c ent of the fair m arket value of the property.
Ac c ordingly, following are the answers:
(a) M r. Raj, beneficial owner, his wife being benamidar and the legal c ounsel who
abets/induces M r. Raj, to enter into a benami transaction, all the three will be liable
under section 53 .
(b) All, in the given c ase entered into a benami transaction with a m otive to avoid payment
of statutory dues, are liable for rigorous im prisonment for a term which shall not be
less than one year, but which m ay extend to seven years and shall also be liable to
fine which may extend to twenty-five per c ent of the fair m arket value of the property.

© The Institute of Chartered Accountants of India


t.me/cafinal6d - telegram link

Page 409
(c ) As per section 55, no prosecution shall be instituted against any person in respect of
any offence committed under sec tions 53 without the previous sanction of the
Board(CBDT).
Case Study 2
MCQ’S
1 (b)
2 (d)
3 (a)
4 (b)
5 (c)
6 (b)
7 (b)
8 (c)
9 (b)
10 (b)
Descriptive Answers
Ans. 1.(a) Section 3 of the Act deals with anti-competitive agreements. Accordingly, any agreement
entered into between enterprises or associations of enterprises or persons or associations of persons
or between any person and enterprise, including cartels, engaged in similar trade of goods or
provision of services, shall be presumed to have an adverse effect on competition and therefore anti-
competitive agreement, if such agreement:
(i) directly or indirectly determines purchase or sale prices;
(ii) limits or controls production, supply, markets, technical development, investment or provision
of services;
(iii) shares the market or source of production or provision of services by way of allocation of
geographical area of market, or type of goods or services, or number of custom ers in the market
or any other similar way;
(iv) directly or indirectly results in bid rigging or collusive bidding.
(b) In this case study, DG noted that by forming their union, the members of DOU had entered
into an agreement amongst themselves through a Memorandum to pursue common objectives
stated therein.
Further, DOU was allocating dumpers owned by its members by applying its sole discretion and also
because it was the single point source of supply of dumpers for stevedoring activities, it was
controlling the provision of dumper services inside the Port area.
Accordingly, when the PSPL and other stevedores had limited option to engage dumpers from
outside sources and when DOU denied services of dumpers to PSPL, the DOU had restricted and
controlled the provision of the said services. This action of DOU and its members caused an adverse
effect on the competition. Thus, the Commission viewed that it was a clear contravention of section
3(1) read with section 3(3)(b).
Ans.2. Section 4 prohibits abuse of dominant position by any enterprise or group. According to the
findings of DG, DOU was to be considered as an enterprise in terms of section 2(h) of the Act.

© The Institute of Chartered Accountants of India


t.me/cafinal6d - telegram link

Page 410
DG observed that DOU in itself was not directly engaged in the provision of the services of dumpers
and hywas for intra-port transportation of cargo, which was the relevant market. Its activities were
limited only to allocation of dumpers of its members to the enlisted stevedores who requisitioned
them. Even though DOU did not derive any monetary considerations for the services so rendered
and also it was not directly engaged in the provision of the services of dumpers but the DOU, because
of being engaged in the activity of provision of services of allocation of dumpers, was an 'enterprise'.
However, it was also observed by DG that the DOU as an enterprise was operating in a different
market i.e., in the market of provision of services of allocation of dumpers; not in the market of
providing services of dumpers for intra-port transportation of cargo. Accordingly, DOU was a non-
player in the relevant market, and therefore its conduct was not liable to be examined under the
provisions of section 4 of the Act which prohibited ‘abuse of dominant position’.
As regards DOU being considered as an enterprise by the DG, the Commission however, did not
view DOU as an enterprise. It was noted by the Commission that the DOU was an association of
dumper owners and in itself it was only a facilitator of the services of dumpers and allocated dumpers
of its members. Further, the DOU though engaged in allocating dumpers owned by its members
amongst stevedores, it neither owned them nor received any consideration for the services of
dumpers. Its activity therefore, could not be considered as the activity of an enterprise as reported
by DG.
Thus, when an entity is not an enterprise, it cannot be charged under section 4 relating to abuse of
dominant position.
Answer 3 Section 27 deals with the orders to be passed by the Commission after the inquiry. If after
inquiry the Commission finds that there is contravention of Section 3 relating to anti-competitive
agreements or section 4 relating to prohibition of abuse of dominant position, it may pass all or any
of the following orders:
(i) direct the involved party to discontinue and not to re-enter anti-competitive agreement or
discontinue the abuse of dominant position, as the case may be;
(ii) impose penalty maximum up to ten per cent. of the average of the turnover for the last three
preceding financial years, upon the defaulting party.
However, if it is the case of a cartel, the Commission may impose upon each producer, seller,
distributor, trader or service provider included in that cartel, a penalty of up to three times of its profit
for each year of the continuance of such anti-competitive agreement or ten per cent. of its turnover
for each year of the continuance of such agreement, whichever is higher.
(iii) direct that the anti-competitive agreement shall stand modified to the extent and in the manner
as specified in the order;
(iv) direct the enterprises concerned to abide by such other orders as the Commission may pass
and comply with the directions, including payment of costs, if any;
(v) pass such other order or issue such directions as it may deem fit.
In the above case study, the Commission passed the orders as under:
(i) directed the DOU and its office bearers to cease and desist (i.e. discontinue) from indulging in
anti-competitive agreement.
(ii) imposed penalty on the DOU at the rate of 8% of their average turnover for the last three
preceding financial years; and in case of each of the office bearers it imposed penalty @ 5% of
the average income of the last three financial years.

© The Institute of Chartered Accountants of India


t.me/cafinal6d - telegram link

Page 411
Case Study 3
MCQ
1: (A)
2. (D)
3: (B)
4: (A)
5: (C)
6: (B)
7: (D)
8: (A)
9: (A)
10: (B)
Descriptive Answer
Answer 1.(A) Mr. Gupta and Deep started a new real estate project in Pune and for that project
they applied for RERA registration. As a Real Estate Regulatory Authority While checking any
application for registration provisions of Section 4 of the Real Estate Regulatory Act (RERA) must
be kept in mind. The different provisions of the Section 4 are as under—
(1) This section provides that every promoter shall make an application to the Authority for
registration of the project in such form, manner, within such time and accompanied by such fee
as may be prescribed.
(2) The promoter shall enclose the following documents along with the application referred to in
sub-section(1), namely—
(a) a brief details of his enterprise including its name, registered address ,type of
enterprise(proprietorship, societies, partnership, companies, competent authority), and
the particulars of registration, and the names and photographs of promoter:
(b) a brief detail of the project launched by him, in the past five years, whether already
completed or being developed, as the case may be, including the current status of the
said projects, any delay in its completion, details of cases pending, details of type of land
and payments pending;
(c) an authenticated copy of the approvals and commencement certificate from the
competent authority obtained in accordance with the laws as may be applicable for the
real estate project mentioned in the application, and where the project is proposed to be
developed in phases, an authenticated copy of the approvals and commencement
certificate from the competent authority for each of such phases.
(d) The sanctioned plan, layout plan and specification of the proposed project of the phase
thereof, and the whole project as sanctioned by the competent authority.
(e) The plan of development works to be executed in the proposed project and the proposed
facilities to be provided thereof including fire fighting facilities, drinking water facilities,
emergency evacuation services, use of renewable energy.

© The Institute of Chartered Accountants of India


t.me/cafinal6d - telegram link

Page 412
(f) The location details of the project, with clear demarcation of land dedicated for the project
along with its boundaries including the latitude and longitude of the end points of the
project;
(g) Proforma of the allotment letter, agreement for sale, and the conveyance deed proposed
to be signed with the allottees;
(h) The number , type and the carpet area of apartment for sale in the project along with the
area of the exclusive balcony or verandah areas and the exclusive open terrace areas
apartment with the appurtenant , if any;
(i) The number and area of garage for sale in the project;
(j) The names and address of his real estate agents, if any, for the proposed project;
(k) The names and address of the contractors, architect, structural engineer, if any and other
persons concerned with the development of the proposed project;
(l) A declaration, supported by an affidavit, which shall be signed by the promoter or any
person authorized by the promoter, stating :--
(A) that he has a legal title to the land on which development is proposed along with
legally valid documents with authentication of such title, if such land is owned by
another person;
(B) that the land is free from all encumbrances, or as the case may be details of the
encumbrances on such land including any rights, title, interest or name of any party
in or over such land along with details;
(C) the time period within which he undertakes to complete the project of phase thereof,
as the case may be;
(D) that 70% of the amount realized for the real estate project from the allottees , from
time to time, shall be deposited in a separate account to be maintained in a
scheduled bank to cover the cost of construction and the land cost and shall be used
only for the purpose;
The promoter shall withdraw the amounts from the separate account, to cover the
cost of the project, in proportion to the percentage of completion of the project.
The amounts from the separate account shall be withdrawn by the promoter after it
is certified by an architect, an engineer, and a chartered accountant in practice
that the withdrawal is in proportion to the percentage of the project.
The promoter shall get his accounts audited within six months after the end of every
financial year by a chartered accountant in practice, and shall produce a statement
of accounts duly certified and signed by such chartered accountant and it shall be
verified during the audit that the amount collected for a particular project have been
utilized for that project and the withdrawal has been in compliance with the
proportion to the percentage of completion of the project.
(E) that he shall take all the pending approvals on time, from the competent authorities;
(F) (F) that he has furnished such other documents as may be prescribed by the rules
or regulations made under this Act; and
(m) such other information and documents as may be prescribed.

© The Institute of Chartered Accountants of India


t.me/cafinal6d - telegram link

Page 413
(3) The Authority shall operationalize a web based online system for submitting applications for
registration of projects within a period of one year from the date of its establishment.
If all the above conditions are satisfied then the registration can be granted.
Answer 1.(B) (1) Promoters of any real estate project applies for registration because the provisions
of Section 3 of RERA require a prior registration with the concerned Real Estate Regulatory Authority.
Section 3 of RERA provides that no promoter shall advertise, market, book, sell or offer for sale, or
invite persons to purchase in any manner any plot, apartment or building, in any real estate project
or part of it in planning area without registering the real estate project with the Real Estate Regulatory
Authority established under this Act.
However, the promoter shall make an application to the Authority for registration of the project that
is ongoing on the date of commencement of this Act and for which completion certificate has not
been issued; within a period of three months from the date of commencement of this Act.
(2) As per the provisions of Section 3(2) in the following situation no registration of the real estate
project is required –
(a) Where the area of the land proposed to be developed does not exceed 500 square meters or
the number of apartments proposed to be developed does not exceed eight (8) inclusive of all
phases;
(b) Where the promoter has received completion certificate for a real estate project prior to the
commencement of this Act,
(c) For the purpose of renovation or repair or re-development which does not involve marketing,
advertising selling or new allotment of any apartment, plot or building, as the case may be,
under the real estate project.
Answer 2.(A) A raid was conducted at Mr. Gupta’s residence by Income Tax Authorities. During the
raid cash, jewellery, property papers and a diary was found. Based upon these and the details as
mentioned in the diary about cash deposition in the bank during the period of demonetization, Mr.
Gupta can be booked under Prohibition of Benami Property Transaction Act, 1988(The Act) .
Details of his diary revealed that during the period of demonetization Mr. Gupta deposited a sum of
Rs. 40 lacs in the different accounts of his servants, their spouse, their children and his office staff.
However he could not offer any satisfactory reply for such deposition and the source of the said
amount. Apart from this, cash amounting to Rs. 27, 50,000 /- was also found, with jewellery worth
Rs. 3 crores. For these also Mr. Gupta did not had any reply. As no legitimate source of cash and
jewellery could be established by Mr. Gupta so whole transaction involved in procuring such cash
and jewellery can be termed as benami transaction and property so acquired by such transaction will
be treated as benami property as per the provisions of Section 2(8) and 2(26) which defines benami
property and property respectively.
Mr. Gupta had purchased three plots and three flats for his three children, but registered value of all
these properties was less than their fair market value so the difference of their fair market value and
registered value (i.e. 25 lacs each in case of plots and 40 lacs each in the case of flats) can be
treated as benami property.
Apart from these he had purchased two plots in the name of his female servant. The definitions of
benamidar and beneficial owner as defined in the Act, makes it clear that in this transaction the
servant becomes benamidar and Mr. Gupta becomes the beneficial owner of such property. So again
transaction of such two plots Rs. 60 lacs is also termed as benami transaction and the property
becomes benami property as defined in the Act.

© The Institute of Chartered Accountants of India


t.me/cafinal6d - telegram link

Page 414
Under the Act Section 3 clearly mentions prohibition on benami transaction. As per Section 3—
(1) No person shall enter into any benami transaction.
(2) Whoever enters into any benami transaction shall be punishable with imprisonment for a term
which may extend to three years or with fine or both.
(3) Whoever enters into any benami transaction on and after the commencement of the Benami
Transaction (Prohibition) Amendment Act, 2016, shall, notwithstanding anything contained in
sub-section (2), be punishable in accordance with the provisions contained in chapter VII.
Section 53 of chapter VII provides for penalty for benami transaction. As per the provisions of this
Section---
Where any person enters into a benami transaction in order to defeat the provisions of any law or to
avoid payment of statutory dues or to avoid payment to creditors, the beneficial owner, benamidar
and any other person who abets or induces any person to enter into the benami transaction, shall be
guilty of the offence of benami transaction.
Whoever is found guilty of the offence of benami transaction referred to above shall be punishable
with rigorous imprisonment for a term which shall not be less than one year, but which may extend
to seven years and shall also be liable to fine which may extend to twenty five percent of the fair
market value of the property.
So it is cleat that as per the provisions of Section 53 Mr. Gupta can be punished with a rigorous
imprisonment for a term not less than one year, extending up to seven years. With this he shall also
be liable to fine upto 25% of the fair market value of the property termed as benami property.In the
given case his fine may extend as under—
Name Value of property Fair market value of property fine (in Rs.)
Cash 40.00 lacs 40.00 lacs 10.00 lacs
Cash 27.50 lacs 27.50 lacs 6.87 lacs
Plots 90.00 lacs 165.00 lacs 41.25 lacs
Flats 240.00 lacs 360.00 lacs 90.00 lacs
Plots 60.00 lacs 60.00 lacs 15.00 lacs
Jewellery 300.00 lacs 300.00 lacs 75.00 lacs
Total 238.12 lacs
In the given case Mr. Gupta can be fined up to Rs. 238.12 lacs.
Answer 2.(B) As an Authority acting under the provisions of the Prohibition of Benami Property
Transaction Act, 1988, the Act has provided with power to impound documents as per the provisions
of Section 22 of the Act. This Section provides for as under---
Where any books of accounts or other documents are produced before the authority in any
proceedings under this Act and the authority in this behalf has reason to believe that any of the books
of accounts or other documents are required to be impounded and retained for any inquiry under this
Act, it may impound and retain the books of accounts or other documents for a period not exceeding
three months (3 months) from the date of order of attachment made by the adjudicating Authority.
Provided that the period for retention of the books of accounts or other documents may be extended
beyond a period of three months from the date of order of attachment made by the Adjudicating
Authority where the authority records in writing the reasons for extending the same.

© The Institute of Chartered Accountants of India


t.me/cafinal6d - telegram link

Page 415
Where the Authority impounding and retaining the books of accounts or other documents, under the
aforesaid is the Initiating Officer, he shall obtain approval of the Approving Authority within a period
of fifteen (15) days from the date of initial impounding and seek further approval of the Approving
Authority for extending the period of initial retention, before the expiry of the period of initial retention,
if so required.
The period of retention of the books of accounts or other documents shall in no case exceed a period
of thirty (30) days from the date of conclusion of all the proceedings under this Act. The person, from
whom the books of accounts or other documents were impounded, shall be entitled to obtain copies
thereof. On the expiry of the period specified, the books of accounts or other documents shall be
returned to the person from whom such books of accounts or other documents shall be returned to
the person from whom such books of accounts or other documents were impounded unless the
Approving Authority or the Adjudicating Authority permits their release to any other person.
Answer 3
In India, if any person wants to enter into foreign currency transaction he can do so, but these
transactions should be within limits as prescribed by Reserve Bank of India. These transactions can
not be carried out on one’s own but can be through a proper channel only. This proper channel is
known as Authorised Person. It means that in India if any person wants to buy or sell foreign currency
in any form he or she can do so through Authorised person only. These Authorized person get
authorization to deal in foreign currency from The Reserve Bank Of India after making application to
the Bank to do so. The provisions regarding Authorisation and their working of any such person are
mentioned in Section 10 of the Foreign Exchange management Act, 1999. The different provisions
of this Section are as under—
(1) The Reserve Bank may, on an application made to it in this behalf, authorize any person to be
known as authorized person to deal in foreign exchange or in foreign securities, as an
authorized dealer, money changer or off- shore banking unit or in any other manner as it deems
fit.
(2) An authorization under this section shall be in writing and shall be subject to the conditions laid
down therein.
(3) An authorization granted under sub-section (1) above may be revoked by the Reserve Bank at
any time if the Reserve Bank is satisfied that:
(a) it is in public interest to do ; or
(b) the authorized person has failed to comply with the condition subject to which the
authorization was granted or has contravened any of the provisions of the Act or any rule,
regulation, notification, direction order made there under ;
Provided that no such authorization shall be revoked on any ground referred to in clause(b)
unless the authorized person has been given a reasonable opportunity of making a
representation in the matter.
(4) An authorized person shall, in all his dealings in foreign exchange or foreign security, comply
with such general or special directions or orders as the Reserve Bank may, from time to time,
think fit to give, and, except with the previous permission of the Reserve Bank, an authorized
person shall not engage in any transaction involving any foreign exchange or foreign security
which is not in conformity with the terms of his authorization under this section.
(5) An authoridsed person shall, before undertaking any transaction in foreign exchange on behalf
of any person, require that person to make such declaration and to give such information as
will reasonably satisfy him that the transaction will not involve, and is not designed for the

© The Institute of Chartered Accountants of India


t.me/cafinal6d - telegram link

Page 416
purpose of any contravention or evasion of the provisions of this Act or of any rule, regulation,
notification, direction or order made there under, and where the said person refuses to comply
with any such requirement or make only unsatisfactory compliance therewith, the authorized
person shall refuse in writing to undertake the transaction and shall, if he has reason to believe
that any such contravention or evasion as aforesaid is contemplated by the person, report the
matter to the Reserve Bank.
(6) Any person other than an authorized person, who has acquired or purchased foreign exchange
for any purpose mentioned in the declaration made by him to authorized person under sub-
section (5) does not use it for such purpose or does not surrender it to authorized person within
the specified period or uses the foreign exchange so acquired or purchased for any other
purpose for which purchase or acquisition of foreign exchange is not permissible under the
provisions of the Act or the rules or regulations or directions or order made thereunder shall be
deemed to have committed contravention of the provisions of the Act for the purpose of this
section.
This channel who deals in foreign currency on behalf of others does its duty as per the directions
given by the Reserve Bank. Powers of Reserve Bank in this regard are mentioned in Section 11 of
the Act. These are as under—
(1) The reserve Bank, for the purpose of securing compliance with the provisions of this Act and
of any rules, regulations, notifications or directions made there under, may give to the
authorized person any direction in regard to making of payment or the doing or desist from
doing any act relating to foreign exchange or foreign currency.
(2) The Reserve Bank may, for the purpose of ensuring the compliance with the provisions of this
Act or of any rule, regulation, notification direction or order made there under, direct any
authorized person to furnish such information, in such manner, as it deems fit.
(3) Where any authorized person contravenes any direction given by the Reserve Bank under this
Act or fails to file any return as directed by the Reserve Bank, the Reserve Bank may, after
giving reasonable opportunity of being heard, impose on the authorized person a penalty which
may extend to ten thousand (10,000) rupees and in the case of continuing contravention with
an additional penalty which may extend to two thousand (2,000) rupees for every day during
which such contravention continues.

© The Institute of Chartered Accountants of India


t.me/cafinal6d - telegram link

Page 417
Test Series: August, 2018
MOCK TEST PAPER
FINAL (NEW) COURSE: GROUP – II
PAPER-6D: ECONOMIC LAWS
Case study 1
Mr. Greed is engaged in the real estate business of development of townships t hrough his company– M/s
Exotic Homes Ltd. During the course of business, he has accumulated enormous amount of wealth in the form
of cash which was generated through illegal businesses. Police cases under several sections of various
Indian laws have also been registered against Mr. Greed; however, police could not take any rigid action due
to his connections.
Mr. Greed has a son Mr. Cute who was residing in India during F.Y. 2015-16. He left for USA on 25 th August
2016 to undergo training for a period of 4 years.
Mr. Honest, brother of Mr. Greed, has a daughter, Ms. Dolly pursuing higher studies in USA. Mr. Honest
intends to:
(a) open a bank account in foreign currency in USA.
(b) remit money from India to his daughter in her account for studies.
Separately, Ms. Dolly has requested Mr. Honest to sponsor a chess tournament in USA which will involve
remittance amounting to USD 95,000. Mr. Honest generally remits money through ABC Bank Ltd. after
complying necessary formalities.
On the other hand, since Mr. Cute's interest lies in India, he intends to invest money in India in the following
manner:
(a) Incorporating a company in India followed by infusion of capital in the said company.
(b) Buying an agricultural farm in his individual capacity.
Above investments require funding which will be sought from Mr. Greed.
From the business of real estate, total wealth generated by Mr. Greed amounts to approx. Rs. 500 Crores. The
said amount was utilized by him in the following manner:
(a) Around Rs.100 crores were used for meeting certain cash expenses and paying bribe.
(b) Rs. 2.25 crores were transferred through hawala transaction to Mr. Cute.
Transferring money through hawala route was chosen by Mr. Greed since the money available with him in his
bank account was not sufficient to remit legally under various provisions of Foreign Exchange Management
Act, 1999. Therefore, he decided to strike a deal with Mr. Hawai, a hawala agent operating in India. Terms of
the deal are as under:
• Mr. Greed will pay Rs. 2.25 crores + commission in cash to Mr. Hawai.
• Mr. Hawai, through his counterparts in USA, will pay equivalent USD to Mr. Cute against invoice for
professional services dated 1 st October 2017.
Mr. Greed and Mr. Honest are promoters and managing directors of M/s Cine World Ltd., a company engaged
in the business of producing films in India.
For a very large upcoming film project, M/s Cine World Ltd. has taken loan from ABC Bank Ltd. amounting to
Rs. 250 crores after mortgaging all the assets of the company including rights related to the film. However,
due to controversies surrounding the film, the Censor Board withheld the certification of the film. Even the
Honorable High Court turned down plea of the producers that the film is not against the interest of the country
or public at large. Mr. Greed used all his contacts and wherever necessary, paid bribe for the said project.
Even these efforts of Mr. Greed could not make his dream possible to release the film.
1

© The Institute of Chartered Accountants of India


t.me/cafinal6d - telegram link
Page 418
Due to the circumstances, the film could not be released and M/s Cine World Ltd. had to suffer huge losses.
Since, crew and the high profile cast were continuously following-up for the payment, Mr. Greed decided to
make payment in cash available with him.
One of the disgruntled crew member filed a complaint against Mr. Greed in police station under Code of
Criminal Procedure for its institution and investigation. The complaint was accompanied with the details of
how Mr. Greed acquired massive amount of wealth and huge properties in his name and also in joint names.
The accused person accumulated movable and immovable properties and assets not only in India but in
abroad also. Those properties were acquired otherwise and were not included in their disclosed assets. Their
criminal acts indicated misappropriation of public money. Accordingly, the complaint was registered under
Indian Penal Code, 1860 and Prevention of Corruption Act, 1988.
Later on, the investigation was taken over by the C.B.I. while the C.B.I. was proceeding with the investigation,
the Enforcement Directorate on the basis of allegation made lodged Enforcement Case Information Report
(ECIR) against Mr. Greed. Similarly, as per the said ECIR when complaint was filed under Section 45 of the
Prevention of Money Laundering Act, 2002, cognizance of the offence was taken against Mr. Greed under
section 3 of the Prevention of Money Laundering Act, 2002, punishable under section 4 of the said Act.
Accordingly, an order was issued by the authorities to provisionally attach properties belonging to Mr. Greed.
Mr. Greed now intends to challenge action taken against him under Prevention of Money Laundering Act,
2002 before the higher authorities.
Answer the following questions:
1. Which of the following is a capital account transaction under Foreign Exchange Management Act, 1999?
(a) Investment in shares of company in India.
(b) Payment of export commission.
(c) Payment towards consultancy services.
(d) None of the above (2 Marks)
2. What is the limit under Liberalized Remittance Scheme?
(a) USD 2,50,000 per financial year per person.
(b) USD 2,50,000 per calendar year per family.
(c) USD 2,50,000 per financial year per family.
(d) USD 2,50,000 per calendar year per person. (2 Marks)
3. Facility under Liberalized Remittance Scheme is available for_____
(a) Studies abroad.
(b) Opening of foreign currency account abroad with a bank.
(c) Only (a)
(d) Both (a) and (b) (2 Marks)
4. Which of the following remittance will require prior approval of Government of India for drawal of foreign
exchange under Foreign Exchange Management Act, 1999?
(a) Payment related to 'call back services' of telephones.
(b) Opening of foreign currency account abroad with a bank.
(c) Remittance of prize money / sponsorship of sports activity abroad by a person other than
International/ National/ State Level bodies, if the amount involved is USD 90,000.
(d) None of the above. (2 Marks)

© The Institute of Chartered Accountants of India


t.me/cafinal6d - telegram link
Page 419
5. Mr. Cute is a person resident in India for financial year______ as per the provisions of Foreign Exchange
Management Act, 1999.
(a) 2016-17
(b) 2017-18
(c) 2018-19
(d) None of the above (2 Marks)
6. As per the provisions of Prevention of Money Laundering Act, 2002, person on whose behalf a
transaction is being conducted is known as:
(a) Client
(b) Financial Institution
(c) Beneficial Owner
(d) Authorized Dealer (2 Marks)
7. Under Prevention of Money Laundering Act, 2002, adjudicating authority consists of following:
(a) 3 persons including chairman
(b) 4 persons including chairman
(c) 2 persons one of whom can be appointed as a chairman
(d) 5 persons including a member from Ministry of Law and Justice. (2 Marks)
8. Among other things, what is the qualification of a person to be appointed as a Public Prosecutor before
the Special Court under the provisions of Prevention of Money Laundering Act, 2002?
(a) Minimum 10 years of experience as an advocate
(b) Minimum 5 years of experience as an advocate
(c) Minimum 7 years of experience as an advocate
(d) Minimum 15 years of experience as an advocate (2 Marks)
9. Prevention of Money Laundering Act, 2002 has an overriding effect on following laws:
(a) Foreign Exchange Management Act, 1999
(b) Companies Act, 2013
(c) Transfer of Property Act, 1882
(d) All of the above (2 Marks)
10. Under Prevention of Money Laundering Act, 2002, property can be provisionally attached for___.
(a) Not exceeding 60 days
(b) Not exceeding 90 days
(c) Not exceeding 180 days
(d) Not exceeding 300 days (2 Marks)
11. Answer the following in light of the provisions of the Foreign Exchange Management Act, 1999:
(a) Advise Mr. Cute whether:
I. he can invest in M/s Exotic Homes Ltd. engaged in the business of building low budget homes.
II. he can buy agricultural farm in his individual capacity.
III. he can make payment through foreign currency notes. (6 Marks)

© The Institute of Chartered Accountants of India


t.me/cafinal6d - telegram link
Page 420
(b) For investing activities in India by Mr. Cute, he approached you on 1 st May 2018 with a notice dated
27 th January 2018 received by him from the office of Enforcement Directorate on 31 st January 2018
directing him to pay penalty. Kindly advise Mr. Cute on timelines to pay the penalty and powers of
the officers to recover the same. Mr. Cute has informed that he doesn't intend to file an appeal.
(4 Marks)
(c) On suspicion of non-compliance of the provisions of the Foreign Exchange Management Act, 1999
by ABC Bank Ltd., the Reserve Bank of India had sent a notice to the bank seeking certain
information on the transactions carried out by Mr. Honest. However, lawyer of ABC Bank Ltd. had
suggested not to provide any response to such notice since such notice is generally issued to every
bank as a part of audit procedure and is routine in nature. Explain the powers of the Reserve Bank
of India in case of non- compliance to notice. (5 Marks)
12. Explain the following in light of the provisions of the Prevention of Money Laundering Act, 2002:
(a) Money Laundering does not mean just siphoning of funds. In light of this statement, explain the
significance and aim of the Prevention of Money Laundering Act, 2002. (5 Marks)
(b) Mr. Greed seeks your advice on the remedy available with him under the Act against the said
attachment order. (4 Marks)
(c) Properties confiscated under the Act shall be available for disposal by Ministry of Finance as and
when necessary. Examine correctness of the statement. (3 Marks)
(d) Notice issued to seek clarification on source of income for acquisition of a particular jointly owned
property shall be given to the majority of the owners. Examine correctness of th e statement.
(3 Marks)
Case study- 2
Speciality Vehicles Limited (SVL) is a manufacturer of passenger cars and commercial vehicles in India. It
sells the cars through single brand dealerships across prominent cities. The dealerships are separate for
passenger cars and commercial vehicles. SVL is lagging behind the competitors in the passenger car
segment due to its cost structure and is losing market share for the last 3 years.
With no revival in sight, the company has decided to exit the passenger car segment and notified its dealers
about shutdown of passenger car manufacturing and sales in India. The service centers for passenger cars
will continue its operations for the next 3 years. Perfect Vehicles Ltd. (PVL) is a passenger car dealer for SVL
in Bhopal with an office cum showroom and no service center.
PVL has bank loans from Bank X and Bank Y for Rs. 15 crore and, Rs. 10.5 crore respectively. Both the banks
have pari-passu charge on the office premise cum showroom. PVL has expressed its inability to repay its
financial obligations to the bankers. One of the bankers, Bank X has filed an application for Insolvency
Resolution Process (IRP) against PVL under IBC, 2016, which is admitted by the adjudicating authority
(NCLT) and consequently appointed Interim Resolution Professional.
SVL has an interest free dealership security deposit of Rs. 2.25 crore since 2010 from PVL with a right to
set-off against any receivables pending from PVL towards SVL. Mr. Right, the nephew of the promoter of PVL
had given a loan of Rs. 0.75 crore to PVL in the last 3 months to pay the utility bills and of fice expenses.
SVL in its claims has demanded Rs. 5.25 crore from PVL against pending receivables. PVL has not paid
wages to the tune of Rs. 0.75 crore to its workmen and statutory employer contributions to the tune of Rs. 0.3
crore.
As per valuers' report, approximated realizable value of office cum showroom is Rs.18 crore. Value of
furniture and equipment is Rs. 0.075 crore. The current receivables on books are Rs. 2.25 crore of which 50%
is doubtful. PC has a general purpose' current account with Bank X having current balance of Rs. 0.225 crore.

© The Institute of Chartered Accountants of India


t.me/cafinal6d - telegram link
Page 421
After admission of the application of Bank X by NCLT, PVL approached one of the Insolvency Professional to
know the applicability of IBC, Infrastructure of Insolvency and Bankruptcy and also the persons who can not
initiate Corporate Insolvency Resolution Process.
PVL also sought opinion on the duties of its officers and employees.
Mr. Right has recently been appointed as one of the member in CCI by the Central Government for 5 years
within the maximum limit of members to be appointed by the Central Government, where a case is pending
against SVL for entering an agreement to limit, restrict or withhold the output or supply of cars or allocate area
or market for the disposal or sale of cars, which was classified as anti-competitive agreement by the
commission and initiated enquiry. Later in view of above, Mr. Right decides to resign from CCI.
Mr. Left, the brother of Mr. Right approached him to have his views on the nature of agreement which he
proposes to enter to sell goods on condition that the prices to be charged on the resale by the purchasers shall
be the prices stipulated by him and also on the number of days notice is to be given to CCI while an enterprise
proposes to enter into a combination, as per the Competition Act, 2002.
Mr. Left also approached the expert to know the procedure for investigation of combinations and the action by
commission after investigation.
Answer the following questions:
1. SVL has a net claim of rupees _______ against PVL.
(a) 5.25 crore
(b) 3.00 crore
(c) 18.00 crore
(d) 28.50 crore (2 Marks)
2. SVL is a/an _____ for its claims against PVL.
(a) A financial creditor
(b) An operational creditor
(c) A financial as well as an operational creditor
(d) Corporate Guarantor (2 Marks)
3. The committee of creditors will constitute of:
(a) Banks X and Y
(b) Banks X, Y and SVL
(c) Banks X, Y, SVL, Nephew of the promoter
(d) Banks X, Y, SVL, Nephew of the promoter, Workmen (2 Marks)
4. Which of the following creditors / groups of creditors cannot reject a proposed resolution plan?
(a) Bank X
(b) SVL
(c) Bank X and Y together
(d) Bank Y (2 Marks)
5. Who proposes the name of the new resolution professional, if the interim resolution professional
appointed by NCLT referred in the case study, is being replaced by the Committee of Creditors?
(a) Adjudicating Authority
(b) Interim resolution professional

© The Institute of Chartered Accountants of India


t.me/cafinal6d - telegram link
Page 422
(c) Insolvency and Bankruptcy Board of India
(d) Committee of Creditors (2 Marks)
6. Which of the following as per the Competition Act, 2002 refers to any agreement to sell goods on
condition that the prices to be charged on the resale by the purchaser shall be the prices stipulated by the
seller unless its clearly stated that the prices lower than those prices may be charged?
(a) exclusive distribution agreement
(b) exclusive supply agreement
(c) resale price maintenance
(d) tie-in arrangement (2 Marks)
7. What is the maximum number of members to be appointed by the Central Government in CCI, as
referred in the case study as per the Competition Act, 2002?
(a) 6
(b) 4
(c) 5
(d) 7 (2 Marks)
8. How many number of days notice is to be given to CCI by Mr. Left while an enterprise proposes to enter
into a combination, as per the Competition Act, 2002?
(a) 15
(b) 45
(c) 7
(d) 30 (2 Marks)
9. Which of the following as per the Competition Act, 2002 refers to any agreement to limit, restrict or
withhold the output or supply of any goods or allocate any area or market for the disposal or sale of
goods as referred in the case study?
(a) exclusive supply agreement
(b) refusal to deal
(c) tie-in arrangement
(d) exclusive distribution agreement (2 Marks)
10. Mr. Right has decided to resign from CCI and therefore he has to submit notice to the:
(a) Chairman of CCI
(b) All other members of CCI
(c) Chairman and all other members of CCI
(d) Central Government (2 Marks)
11. Referring the case study, answer the following as per the provisions of the Insolvency and Bankruptcy
Code, 2016 (IBC):
(a) To whom the IBC is applicable? (2 Marks)
(b) Who are involved in the Infrastructure of Insolvency and Bankruptcy Process under IBC?
(3 Marks)
(c) Who cannot initiate Corporate Insolvency Resolution Process? (3 Marks)

© The Institute of Chartered Accountants of India


t.me/cafinal6d - telegram link
Page 423
(d) What are duties of officers, employees, managers, etc. to report to the Resolution Professional?
What are the consequences if they do not support? (3 Marks)
(e) What is Financial Debt? (4 Marks)
12. Referring the facts provided in the case study, answer the following as per the provisions of the
Competition Act, 2002:
(a) What constitutes competition law and policy? (3 Marks)
(b) What is an anti-competitive agreement? (2 Marks)
(c) When the commission may initiate enquiry into anti-competitive agreements / abuse of dominance?
(2 Marks)
(d) What will the commission do after investigation? (4 Marks)
(e) What is the procedure for investigation of combinations? (4 Marks)
Case study 3
Mr. Bhanu Pratap Taneja is a leading real estate developer based in Delhi. In the last decade, his company
Garvit Bhoomi Developers Pvt. Limited having registered office in Bhikaji Cama Place, New Delhi, had
successfully developed four housing projects – two in Gurgaon and one each in Jaipur and Lucknow. They
had a robust management team. Having been the name behind developing more than five thousand luxurious
apartments with modern amenities, they had the reputation of delivering the projects well within the promised
time.
In the beginning of the year 2015, they launched another project in Indirapuram, UP by the name Omega
Capetown Residency in which 1000 residential units consisting of 2BHK and 3BHK apartments were to be
developed. They were to be completed in all respects by January 2018 and delivered to the consumers by that
date. This project was being carried on smoothly when the Real Estate (Regulation and Development) Act,
2016 came to be enacted w.e.f. 1 st May, 2016. Section 3, which was enacted later w.e.f. 1 st May 2017. Since
Omega Capetown Residency consisted of 1,000 residential units, was required to be registered and so for the
same the company submitted the requisite documents with concerned authorities.
As the application for registration was found to be complete in all respects, the Omega Capetown Residency
Project was granted registration by RERA (UP) within the statutory period and was provided with a registration
number including a log-in ID for assessing the website of the Authority and to create webpage.
In the meantime, Mr. Taneja was approached by some of the influential developers that an understanding had
been reached among them to control the price of apartments to be built by them. However, because of legal
tangles such understanding could not be brought into writing and it was also not intended to be enforced by
legal proceedings. Mr. Taneja did not agree to the proposal because even though the understanding was not
in writing and it was not intended to be enforceable by legal proceedings, it was still illegal as per the
Competition Act, 2002. This revelation made by Mr. Taneja discouraged the intending developers and they
desisted from being a party to this proposal.
Mr. Taneja’s son Garvit, who was a commerce graduate and holder of law degree, had a college friend Rohit
whose father Mr. Dev Kumar dealt in sale, purchase and renting of properties under the title ‘Dev Property
Dealers’ from the Yusuf Sarai market. Since Rohit had joined his father’s business, it was thought prudent to
convert the existing proprietary business into a registered partnership firm titled as ‘Dev & Sons Property
Dealers’. Because of the enactment of Real Estate Act, Rohit consulted his friend Garvit regarding its
implications in case of real estate agents. Accordingly, the firm was got registered as real estate agent with
the help of Garvit’s legal advisor.
Further, Garvit made a proposal to Rohit and his father that they could associate themselves with his Omega
Capetown Residency, a registered RERA project in Indirapuram for facilitating sale of apartments which they
readily accepted. Garvit also cautioned them that as per the Act, since their firm was now a registered real
estate agent they were not supposed to facilitate sale/purchase of any plot, apartment or building in a real
7

© The Institute of Chartered Accountants of India


t.me/cafinal6d - telegram link
Page 424
estate project being sold by the promoter in any planning area, if such project was not registered with RERA
of the concerned State.
In addition, Garvit’s legal advisor told them that as required by Section 10, a registered real estate agent
would maintain and preserve proper books of accounts and other necessary documents. Further, such agent
would not involve himself in any unfair trade practice like making a false statement regarding services to be
provided by him. He would also not permit the publication of any advertisement whether in any newspaper or
otherwise of services that were not intended to be offered. Besides, the agent would also have to help the
intending buyers in getting the required information and documents to which they were entitled, at the time of
booking of any property.
Rohit had a friend Tarun whose father Dr. Sreenivas Sharma was a surgeon in a government hospital and was
residing in a rented government flat in the hospital campus itself. He had an intense desire to have a luxurious
flat of his own. Tarun had joined IBM after doing MBA from IIFT, New Delhi. So, with the combined salary of
both, they decided to buy a flat. Tarun contacted Rohit to help him in searching a suitable apartment for his
family. In turn, Rohit informed him that one particular 3BHK flat at an ideal location was available in Omega
Capetown Residency in Indirapuram as the original allottee had withdrawn from the scheme; otherwise the
booking under this project was already full. Dr. Sharma got interested in the information and went to the
Omega Capetown Residency along with his family to see the concerned apartment. He liked its st rategic
location and gathered more information regarding sanctioned plan, layout plan along with the other
specifications, etc. He then asked for stage-wise time schedule of completion of the project and also enquired
regarding provision of water, sanitation and other amenities. Since, Rohit personally knew Garvit and his
father Mr. Taneja - the promoters of the project - Dr. Sharma and his family had a lively and fruitful meeting
with them. Subsequently, he and his son jointly entered into an agreement for sale with the promoters of the
project and made payment of 75% of the cost of the apartment, while remaining 25% of the cost was to be paid
at the time when the apartments were ready for occupation.
A few months after booking the apartment, Dr. Sharma got a notice from the promoters of Omega Capetown
Residency that due to unforeseen circumstances they were not in a position to complete the project and
needed the allottees’ consent for transferring of their majority rights and liabilities to another rep uted
developer M/s. Sai Developers Pvt. Limited of New Delhi. In case any of the allottees was not agreeable to this
proposal he could get his money refunded. Since Dr. Sharma was very much attached to the location of the
flat, he accepted the proposal after enquiring with Rohit and his father. He also learnt that 95% of the allottees
had already given their written permission. Further, the Authority had given its written approval to the proposal
for transfer and completion of Project by M/s. Sai Developers Pvt. Limited. Dr. Sharma was also assured by
Mr. Bhanu Pratap Taneja, the erstwhile promoter with whom he had earlier interacted satisfactorily, that all the
pending obligations would be fulfilled by the new developer and in no case the date of completion of the
project would be extended; otherwise it would attract penalty. It was also disclosed by Mr. Taneja that the new
promoter would rectify any structural defect if it occurred within a period of five years from the date of handing
over the possession of the apartments. Dr. Sharma, thus felt relieved.
M/s. Sai Developers completed the project on time and received Completion Certificate from the Competent
Authority. As per the agreement for sale, Dr. Sharma made payment of the remaining 25% of the cost.
Thereafter, he received Occupancy Certificate and took physical possession of the apartment well before two
months since the allottees were supposed to take physical possession within statutory period of two months
from the issue of Occupancy Certificate. He was also given other necessary documents and plans, including
that of common areas. He also became a member of the RWA formed by the allottees. In the meantime, the
promoter executed a registered conveyance deed in favour of each of the allottees along with the undivided
proportionate title in the common areas to the RWA.
Select the most appropriate answer from the options given for each question:
1. Registration of a real estate project shall not be required –
(a) where the area of land proposed to be developed does not exceed five hundred square meters or
the number of apartments proposed to be developed does not exceed eight.
8

© The Institute of Chartered Accountants of India


t.me/cafinal6d - telegram link
Page 425
(b) where the area of land proposed to be developed does not exceed five thousand square meters or
the number of apartments proposed to be developed does not exceed eighty.
(c) where the area of land proposed to be developed does not exceed two hundred fifty square meters
or the number of apartments proposed to be developed does not exceed four.
(d) where the area of land proposed to be developed does not exceed three hundred square meters or
the number of apartments proposed to be developed does not exceed three. (2 Marks)
2. Who is required to submit a copy of duly obtained approvals and commencement certificate for getting
the project registered with RERA:
(a) Allottee
(b) Promoter
(c) Real Estate Agent
(d) None of the above (2 Marks)
3. A registered real estate agent shall -
(a) facilitate the sale/purchase of any plot, apartment or building, being sold by the promoter in any
planning area, which is registered with the Authority;
(b) maintain and preserve prescribed books of account, records and documents;
(c) not involve himself in any unfair trade practices
(d) All of the above. (2 Marks)
4. The promoter is required to rectify any structural defect if it occurs within a period of ----- years from the
date of handing over the possession of the apartments to allottees –
(a) Two years
(b) Three years
(c) Four years
(d) Five years (2 Marks)
5. Registration of on-going project for which completion certificate is yet to be received is mandatory -
(a) Yes, if the area of land (developed or to be developed) exceeds five hundred square meters or the
number of apartments (developed or to be developed) exceeds eight.
(b) No, irrespective of the area of land or the number of apartments
(c) Can’t say
(d) None of the above (2 Marks)
6. A real estate developer can leave the project mid-way by selling that project to another developer if he
has taken a written approval of ---------- of allottees along with approval of the Authority.
(a) 2/3rd
(b) 1/3rd
(c) 3/4th
(d) 1/4 th (2 Marks)
7. The time limit within which an allottee is required to take physical possession of the apartment after issue
of occupancy certificate is -
(a) one month
(b) two months
9

© The Institute of Chartered Accountants of India


t.me/cafinal6d - telegram link
Page 426
(c) three months
(d) four months (2 Marks)
8. A certificate certifying that the real estate project has been developed according to the sanctioned plan,
layout plan and specifications as approved by the competent authority under the local laws is called -
(a) Commencement Certificate
(b) Completion Certificate
(c) Occupancy Certificate
(d) None of the above (2 Marks)
9. The flat purchased by Dr. Sharma jointly with his wife Mrs. Neelima Sharma though funded by him would
be held as ‘benami transaction’ under the Prohibition of Benami Property Transactions Act, 1988 -
(a) Yes
(b) No
(c) Can’t say
(d) None of the above (2 Marks)
10 As per section 2 (b) of the Competition Act, 2002 ‘Agreement’ includes any arrangement or
understanding or action in concert:
(a) Whether or not, such arrangement, understanding or action is formal or in writing; or
(b) Whether or not such arrangement, understanding or action is intended to be enforceable by legal
proceedings.
(c) Whether or not, such arrangement, understanding or action is formal or in writing; or whether or not
such arrangement, understanding or action is intended to be enforceable by legal proceedings.
(d) None of the above. (2 Marks)
11. (i) Examine in the light of the RERA, 2016, the following given aspects in reference to the given case
study:
(a) Rights exercised by Dr. Sharma as an allottee.
(b) Duties fulfilled by Dr. Sharma as an allottee.
(c) Right which was not exercised by him and duty which was not required to be fulfilled by
Dr. Sharma.
(ii) The promoters of Omega Capetown Residency transferred majority of rights and liabilities to Sai
DeveIopers Pvt Ltd. for the completion of the project. Advise as to the validity of such transfer of a
real estate project to a Sai Developer’s Pvt Ltd? (10 Marks)
12. In the given case study Omega Capetown Residency has got itself registered under the Real Estate
Regulatory Authority, as it consisted of 1,000 residential units. However, if Omega Capetown Residency
consisted of only 250 residential units, then was it necessary to get itself registered under the Real
Estate (Regulation and Development) Act, 2016. If yes, name the various important documents and
declarations which are required to be submitted by a ‘real estate developer’ while registering a project
with the Real Estate Regulatory Authority (RERA) having only 250 residential units and not 1,000
residential units. (10 Marks)
13. Mr. Bhanu Pratap Taneja was approached by some of the influential developers to join their association
so as to reach an understanding whereby they could inflate the price of the apartments built by them.
Even though the deal was in favour of Mr. Bhanu Pratap Taneja, he rejected the proposal from other
developers. In the light of the provisions of the Competition Act, 2002, discuss whet her the decision of
Mr. Bhanu Pratap Taneja valid? (10 Marks)
10

© The Institute of Chartered Accountants of India


t.me/cafinal6d - telegram link
Page 427
Test Series: August, 2018
MOCK TEST PAPER
FINAL (NEW) COURSE: GROUP – II
PAPER-6D: ECONOMIC LAWS
ANSWERS
Case study 1
1. (a)
2. (a)
3. (d)
4. (d)
5. (a)
6. (c)
7. (a)
8. (c)
9. (d)
10. (c)
11 (a) Under sub section (2) of Section 6, the RBI has issued the Foreign Exchange Management
(Permissible Capital Account Transactions) Regulations, 2000. The Regulations specify the list of
transactions, which are permissible in respect of persons resident in India in Schedule I and the
classes of capital account transactions of persons resident outside India in Schedule II. Further,
on certain transactions, the RBI imposes prohibition. For instance,
- The person resident outside India is prohibited from making investments in India in any form,
in any company, or partnership firm or proprietary concern or any entity whether incorporated
or not which is engaged or proposes to engage:
▪ in real estate business, or construction of farm houses
Explanation: In 'real estate business' the term shall not include development of townships,
construction of residential/commercial premises, etc.
Further, a person resident outside India who is a citizen of India may-
(a) Acquire immovable property in India other than an agricultural property, plantation, or a farm
house:
Provided that in case of acquisition of immovable property, payment of purchase price, if any,
shall be made out of:
(i) Funds received in India through normal banking channels by way of inward remittance
from any place outside India or
(ii) Funds held in any non-resident account maintained in accordance with the provisions of
the Act and the regulations made by the Reserve Bank of India.
Provided further that no payment of purchase price for acquisition of immovable property shall
be made either by traveller's cheque or by foreign currency notes or by other mode other than
those specifically permitted by this clause.
In light of the above provisions, advice to Mr. Cute is as under:
(I) Yes, he can invest in M/s Exotic Homes Ltd. since the company is engaged in the
1

© The Institute of Chartered Accountants of India


t.me/cafinal6d - telegram link
Page 428
business of development of townships.
(II) No, he cannot buy agricultural farm in his individual capacity since it has been
specifically prohibited.
(III) No, he cannot make payment through foreign currency notes since it is specifically
prohibited.
(b) Under Section 14, subject to the provisions of sub-section (2) of section 19, if any person fails to
make full payment of the penalty imposed on him under section 13 within a period of ninety days
from the date on which the notice for payment of such penalty is served on him, he shall be liable
to civil imprisonment under this section.
Further, as per section 14A, save as otherwise provided in this Act, the Adjudicating Authority may,
by order in writing, authorize an officer of Enforcement not below the rank of Assistant Director to
recover any arrears of penalty from any person who fails to make full payment of penalty imposed
on him under section 13 within the period of ninety days from the date on which the notice for
payment of such penalty is served on him.
In light of the above provisions, Mr. Cute is advised to pay the amount of penalty within 90 days.
In case, if he doesn't pay the officer referred, to in Section 14A shall exercise all the like powers
which are conferred on the income-tax authority in relation to recovery of tax under the Income-tax
Act, 1961 and the procedure laid down under the Second Schedule of the said Act shall mutatis
mutandis apply in relation to recovery of arrears of penalty under this Act.
(c) Under Section 12, the Reserve Bank may, at any time, cause an inspection to be made, by any
officer of the Reserve Bank specially authorised in writing by the Reserve Bank in this behalf, of
the business of any authorised person as may appear to it to be necessary or expedient for the
purpose of:
(a) verifying the correctness of any statement, information or particulars furnished to the Reserve
Bank;
(b) obtaining any information or particulars which such authorised person has failed to furnish on
being called upon to do so;
(c) securing compliance with provisions of this Act or of any rules, regulations, directions or
orders made there under.
It shall be the duty of every authorised person, and where such person is a company or a firm,
every director, partner or other office of such company or firm as the case may be, to produce to
any officer making an inspection under sub-section (1), such books, accounts and other documents
in his custody or power and to furnish any statement or information relating to the affairs of such
person, company or firm as the said officer may require within such time and in such manner as
the said officer may direct. Accordingly, advice given by the lawyer is not in line with requirements
laid down under the provisions of the Foreign Exchange Management Act, 1999.
12. (a) “Money Laundering” doesn’t not mean just siphoning of fund. Money Laundering is a moving of
illegally acquired cash through financial systems so that it appears to be legally acquired. Thus,
money laundering is not just the siphoning of funds but it is the conversion of money which is
illegally obtained.
Prevention of Money Laundering Act, 2002 has been enacted with aim for combating channelizing
of money into illegal activities.
Significance and Aim of Prevention of Money Laundering Act, 2002:-The preamble to the Act
provides that it aims to prevent money laundering and to provide for confiscation of property derived
from, or involved in, money laundering and for matters connected therewith or incidental thereto.
In order to further strengthen the existing legal framework and to effectively combat money
laundering, terror financing and cross border economic offences, an Amendment Act, 2009 was
2

© The Institute of Chartered Accountants of India


t.me/cafinal6d - telegram link
Page 429
passed. The new law seeks to check use of black money for financing terror activiti es. Financial
intermediaries like full-fledged money changers, money transfer service providers and credit card
operators have also been brought under the ambit of the Act. Consequently, these intermediaries,
as also casinos, have been brought under the reporting regime of the enforcement agencies. It
also checks the misuse of promissory notes by FIIs, who would now be required to furnish all
details of their source. The new law would check misuse of 'proceeds of crime' be it from sale of
banned narcotic substances or breach of the Unlawful Activities (Prevention) Act. The passage of
the Prevention of Money Laundering (Amendment), 2009 have enabled India's entry into Financial
Action Task Force (FATF), an inter-governmental body that has the mandate to combat money
laundering and terror financing.
(b) Section 25 of the Act, empowers the Central Government to establish an Appellate Tribunal to hear
appeals against the orders of the Adjudicating Authority and the authorities under this Act.
Section 26 deals with the rights and time frame to make an appeal to the Appellate Tribunal. Any
person aggrieved by an order made by the Adjudicating Authority may prefer an appeal to the
Appellate Tribunal within a period of 45 days from the date on which copy of the order is received
by him. The appeal shall be in such form and be accompanied by such fee as may be prescribed.
The Appellate Tribunal may extend the period if it is satisfied that there was sufficient cause for
not filing it within the period of 45 days.
The Appellate Tribunal may, after giving the parties to the appeal an opportunity of being heard,
pass such orders thereon as it thinks fit, confirming, modifying or setting aside the order appealed
against.
The Act also provides further appeal. According to Section 42, any person aggrieved by any
decision or order of the Appellate Tribunal may file an appeal to the High Court within 60 days from
the date of communication of the order of the Appellate Tribunal.
In light of the above provisions of the Act, Mr. Greed is advised to prefer an appeal to Appellate
Tribunal in the first instance.
(c) Under Section 10, the Central Government may, by order published in the Official Gazette, appoint
as many of its officers (not below the rank of a Joint Secretary to the Government of India) as it
thinks fit, to perform the functions of an Administrator.
The Administrator appointed under sub-section (1) shall receive and manage the property in
relation to which an order has been made under sub-section (6) of section 8 in such manner and
subject to such conditions as may be prescribed.
The Administrator shall also take such measures, as the Central Government may direct, to dispose
of the property which is vested in the Central Government under section 9.
Accordingly, an administrator has to be appointed who shall deal with the property in the manner
directed by the Central Government.
In view of above, the statement that the properties under the Act shall be available for disposal by
Ministry of Finance as and when necessary, is correct.
(d) Under Section 8, on receipt of a complaint under sub-section (5) of section 5, or applications made
under sub-section (4) of section 17 or under sub-section (10) of section 18, if the Adjudicating
Authority has reason to believe that any person has committed an offence under section 3 or is in
possession of proceeds of crime, he may serve a notice of not less than thirty days on such person
calling upon him to indicate the sources of his income, earning or assets, out of which or by means
of which he has acquired the property attached under sub-section (1) of section 5, or, seized under
section 17 or section 18, the evidence on which he relies and other relevant information and
particulars, and to show cause why all or any of such properties should not be declared to be the
properties involved in money-laundering and confiscated by the Central Government.

© The Institute of Chartered Accountants of India


t.me/cafinal6d - telegram link
Page 430
Provided that where a notice under this sub-section specifies any property as being held by a
person on behalf of any other person, a copy of such notice shall also be served upon such other
person.
Provided further that where such property is held jointly by more than one person, such notice shall
be served to all persons holding such property.
In view of above, the statement is incorrect and the notice shall be served to all persons holding
such property.
Case study-2

1. (b)
2. (b)
3. (a)
4. (b)
5. (d)
6. (c)
7. (a)
8. (d)
9. (d)
10. (d)
11. (a) The provisions of the IBC, 2016 are applicable to Individuals, Unlimited Partnership Firms, Limited
Liability Partnerships and companies. The provisions relating to Corporate in the Code, i.e., Limited
Liability Partnerships and Companies are notified and in force w.e.f. 1 st December, 2016. The
provisions related to Individuals and Unlimited Partnership Firms – the Part III of IBC, 2016 are yet
to be notified.
(b) The four pillars of supporting institutional infrastructure, to make the Insolvency and Bankruptcy
Process work efficiently are:
(A) The regulator - The Insolvency and Bankruptcy Board of India (IBBI)
(B) Adjudicating Authority (AA):
i. National Company Law Tribunal (NCLT) - For Corporate, i.e., Companies and Limited
Liability Partnerships
ii.. National Company Law Appellate Tribunal (NCLAT) will act as Appellate Authority.
iii. Debt Recovery Tribunal (DRT) - For Individuals and Unlimited Partnership Firms
(C) A private industry of Insolvency Professionals (IPs) with oversight by private Insolvency
Professional Agencies (IPAs)
D. A private industry of Information Utilities (IU)
(c) Section 11 of the IBC, states that the following persons are not entitled to make an application to
initiate Corporate Insolvency Resolution Process:
A. A corporate debtor undergoing the corporate insolvency resolution process
B. A corporate debtor having completed corporate insolvency resolution process twelve months
proceeding the date of application.
C. A corporate debtor or financial creditors who has violated any terms of the resolution plan
which was approved twelve months before the date of making application.

© The Institute of Chartered Accountants of India


t.me/cafinal6d - telegram link
Page 431
D. A corporate debtor in respect of whom a liquidation order has been made.
Here, Corporate debtor includes a Corporate applicant in respect of such Corporate debtor.
(d) The officers and managers of the Corporate Debtor, shall report to Resolution Professional. They
shall provide him all the documents or records as required by him in the course of his duties. Where
any personnel or promoters of Corporate Debtor are not assisting or not co-operating Resolution
Professional, he may file an application to Adjudicating Authority for necessary instructions. Then,
Adjudicating Authority shall direct accordingly.
(e) As per section 5(8) of the IBC "Financial Debt" means, a debt along with interest, if any, which is
disbursed against the consideration for the time value of money and includes-
A. Money borrowed against the payment of interest
B. Any amount raised by acceptance under any acceptance credit facility or its de materialized
equivalent;
C. Any amount raised pursuant to any note purchase facility or the issue of bonds, notes,
debentures, loan stock or any similar instrument.
D. the amount of any liability in respect of any lease or hire purchase contract which is deemed
as a finance or capital lease under the Indian Accounting Standards or such other accounting
standards as may be prescribed;
E. receivables sold or discounted other than any receivables sold on non recourse basis;
F. any amount raised under any other transaction, including any forward sale or purchase
agreement, having the commercial effect of a borrowing;
G. any derivative transaction entered into in connection with protection against or benefit from
fluctuation in any rate or price and for calculating the value of any derivative transaction, only
the market value of such transaction shall be taken into account;
H. any counter-indemnity obligation in respect of a guarantee, indemnity, bond, documentary
letter of credit or any other instrument issued by a bank or financial institution;
I. the amount of any liability in respect of any of the guarantee or indemnity for any of the items
referred to in sub-clauses (a) to (h) of this clause.
12. (a) Competition law and policy is defined as those Government measures that affect the behavior of
enterprises and structure of the industry with a view to promote efficiency and maximize welfare.
There are two elements of such Government measures:- a Competition Policy: Set of policies, such
as liberalized trade policy, relaxed FDI policy, de-regulation, etc., that enhances competition in the
markets. a Competition Law: To prevent anti-competitive practices with minimal intervention.
(b) An anti-competitive agreement is an agreement having appreciable adverse effect on competition.
Anti-competitive agreements include, but are not limited to a agreement to limit production and/or
supply; a agreement to allocate markets; a agreement to fix price; a bid rigging or collusive bidding;
a conditional purchase/sale (tie-in arrangement); a exclusive supply / distribution arrangement; a
resale price maintenance; and a refusal to deal.
(c) The commission may initiate enquiry into anti-competitive agreement Act/ abuse of dominance on
its own on the basis of information and knowledge in its possession,
or On receipt of an information,
or On receipt of a reference from the Central Government or a State Government or a statutory
authority.
(d) After receipt of the investigation report from the Director General, the Commission may forward it
to the concerned parties. If the investigation is on a reference from a statutory authority, the

© The Institute of Chartered Accountants of India


t.me/cafinal6d - telegram link
Page 432
forwarding of report to the concerned authority is mandatory. If the report of the DG does not find
any contravention of the Act, the Commission shall seek objections from the concerned parties.
After considering the objections received, if any, the Commission may accept the report of the DG,
or require further investigation to be made by the DG or make inquiries itself.
In conclusion of the above board process, the Commission shall determine whether it is a case of
anti-competitive agreement or abuse of dominant position or both and after hearing the concerned
parties pass appropriate orders.
(e) If the Commission is of the opinion that a combination is likely to cause or has caused adverse
effect on competition, it shall issue a show cause notice to the parties as to why investigation in
respect of such combination should not be conducted. On receipt of the response, if Commission
is of the prima facie opinion that the combination has or is likely to have appreciable adverse effect
on competition, it may direct publication of details, inviting objections from the public and hear
them, if considered appropriate. It may invite any person, likely to be affected by the combination,
to file his objections. The Commission may also inquire whether the disclosure made in the notice
is correct and combination is likely to have an adverse effect on competition.
Case study 3

1. (a)
2. (b)
3. (d)
4. (d)
5. (a)
6. (a)
7. (b)
8. (b)
9. (b)
10. (c)
11. (i) (a) With reference to Section 19, Dr. Sharma, as an allottee, exercised the following rights:
(i) Obtained the information relating to sanctioned plans, layout plans along with the
specifications as approved by the competent authority.
(ii) Demanded to know stage-wise time schedule of completion of the project, including the
provisions for water, sanitation, electricity and other amenities.
(iii) Claimed physical possession of the said apartment.
(iv) Obtained the necessary documents and plans, including that of common areas, after getting
the physical possession of the apartment from the promoter.
(b) With reference to Section 19, Dr. Sharma, as an allottee, fulfilled the following duties:
(i) Made necessary payments within the time as specified in the agreement for sale.
(ii) Became a member of the RWA formed by the allottees.
(iii) Took physical possession of the apartment within a period of two months from the issue of
Occupancy Certificate.
(iv) Participated towards registration of the conveyance deed of the apartment.
(c) (i) With reference to Section 19, Dr. Sharma, as an allottee, did not exercise the following
right:
6

© The Institute of Chartered Accountants of India


t.me/cafinal6d - telegram link
Page 433
The right to claim the refund of amount paid along with prescribed rate of interest. It was
so because the promoter was able to give possession of the apartment in accordance
with the terms of agreement for sale.

(ii) With reference to Section 19, Dr. Sharma, as an allottee, was not required to fulfill the
following duty:
The duty to pay interest at prescribed rate for delay in making any payment. It was so
because he had made the payments in accordance with the terms of agreement for sale.
(ii) Yes, as per section 15 of the Act, a promoter is permitted to transfer his majority rights and liabilities
in respect of a real estate project to a third party.
The provisions given below are to be adhered to by the promoter for transfer:
(i) Obtain prior written consent from two-third of allottees. Such consent will not include the consent
given by the promoter.
(ii) Also obtain prior written approval of the Authority.
Note: It is to be ensured that such transfer shall not affect the allotment or sale of the apartments,
plots or buildings, as the case may be, in the real estate project developed by the promoter.
(iii) After obtaining the required consent of both allottees and the Authority, the new promoter shall be
required to independently comply with all the pending obligations under the provisions of the Act
or the rules and regulations made thereunder.
(iv) The new promoter is also required to comply with the pending obligations as per the agreement
for sale entered into by the erstwhile promoter with the allottees.
(v) Further, the new promoter must note that any transfer so permitted shall not result in extension of
time to him to complete the real estate project.
Note: In case of default, he shall be liable to the consequences for delay, as per the provisions of
the Act or the rules and regulations made thereunder.
Since in the given case study, 95% of the allottees had already given their written permission.
Further, the Authority had given its written approval to the proposal for transfer and completion
of Project by M/s. Sai Developers Pvt. Limited in compliance with the requirements given in
the said provisions.
12. According to proviso to section 3 of the Real Estate (Regulation and Development) Act, 2016, projects
that are on going on the date of commencement of the Act, and for which the completion certificate has
not been issued, the promoter of the project are required to make and application to the concerned
Authority for the registration of the said project within a period of 3 months from the date of
commencement of the Act.
Further, the section provides that no registration of real estate project shall be required where the area
of land proposed to be developed does not exceed 500 square meters or the number of the apartments
proposed to be developed does not exceed 8 inclusive of all phases.
Hence, the Act requires registration of on-going projects where completion certificate was yet to be
obtained as well as new projects, if the area to be developed exceeded 500 sq. mtrs. or apartments to
be built under the project exceeded eight. Thus, registration of Omega Capetown Residency was must
with the Real Estate Regulatory Authority of UP (RERA, UP), as consisted of 1,000 residential units.
Further, even if Omega Capetown Residency consisted of only 250 residential units (i.e. more than 8
units), it will be compulsory to get itself registered under the Act.
The process of registering a project with the Real Estate Regulatory Authority (RERA) which consists of
1,000 units or 250 units is same which is given under section 4 of the Act. With reference to Section 4,
7

© The Institute of Chartered Accountants of India


t.me/cafinal6d - telegram link
Page 434
various important documents and declaration required to be submitted while registering a project with
RERA are as under:
• Details of the project such as name, address, type, names and photographs of the promoters, etc.
• Details of the project which were already launched by the real estate developer in the preceding 5 years
and their present status.
• Approvals and commencement certificates obtained from the competent authority for each phase of the
project separately.
• Sanctioned layout plan, the development plan for the project and details of basic facilities being made
available like drinking water, electricity etc.
• Proforma of allotment letter, agreement for sale and conveyance deed to be signed with the consumers.
• Location of the project with clear demarcation of the land for the project.
• Number, type and carpet areas of units to be sold.
• The details of open areas if any like terraces, balconies etc.
• Details of associated engineers, contractors, architects and intermediaries in the project.
• a declaration, duly supported by an affidavit, stating the following important matters:
o that the promoter has a legal title to the land and it is free from all encumbrances along with legally
valid documents;
o the time period required for completion of the project;
o that seventy per cent. of the amount realised from the allottees, from time to time, shall be
deposited in a separate escrow account and shall be used only for the purpose of completion of
project;
o that the promoter shall get his accounts audited within six months after the end of every financial
year by a chartered accountant in practice; and shall take all the pending approvals on time from
the competent authorities; etc.
13. According to section 2(b) of the Competition Act, 2002, ‘Agreement’ includes any arrangement or
understanding or action in concert:
(i) Whether or not, such arrangement, understanding or action is formal or in writing or
(ii) Whether or not such arrangement, understanding or action is intended to be enforceable by legal
proceedings.
Further, section 2(c) of the Competition Act, 2002, "Cartel" includes an association of producers,
sellers, distributors, traders or service providers who, by agreement amongst themselves, limit,
control or attempt to control the production, distribution, sale or price of, or, trade in goods or
provision of services.
An association for the welfare of the trade or formed for any other purpose not mentioned in the
aforesaid definition will not be a cartel Thus, it is only when an association, by agreement amongst
themselves, limits control or attempts to control the production, distribution, sale or price of, or,
trade in goods or provision of services, that it will be a cartel.
Hence, an agreement which prohibits an enterprise or person or their association for entering into
an agreement in respect of production, supply, distribution, storage, acquisition or control of goods
or services, which causes or is likely to cause an appreciable adverse affect on competition. Such
agreements entered in contravention of the above are void. These agreements are presumed to
have an appreciable adverse affect on competition.
Here, in the given situation, the agreement between Mr. Bhanu Pratap Taneja and other builders
would have fallen into the ambit of section 2(b) and 2(c) of the Competition Act, 2002 as the aim
8

© The Institute of Chartered Accountants of India


t.me/cafinal6d - telegram link
Page 435
of the association was to increase the price of the apartments. Thus, such an association would
be void.

© The Institute of Chartered Accountants of India


t.me/cafinal6d - telegram link
Page 436
Test Series: March, 2018
MOCK TEST PAPER
FINAL (NEW) COURSE: GROUP – II
ELECTIVE PAPER 6D: ECONOMIC LAWS

Attempt any two out of three case study based questions.


Each Case Study carries 50 Marks.
Time Allowed – 4 Hours Maximum Marks – 100

QUESTION 1
Mrs. Shakuntala Bisht was a dynamic woman entrepreneur running her factory of manufacturing
designer candles and other items made of wax as a proprietary concern in Dehradun (Uttrakhand)
titled as M/s. Bisht Designer Candles since 2003. She had appointed a number of dealers pan India
for selling her designer products.
She was residing in a bungalow on Subhash Road in Dehradun along with her family. Her husband
Mr. O. P. Bisht was joint-secretary in Uttrakhand Sachivalaya. Her son Varun had done his B.E.
(Bachelor of Engineering) from IIT, Kharagpur and thereafter MBA from IIM, Kolkata in the year 2013.
Her daughter Latika was pursuing B.Sc. (Hons.) in Physics from DIT University, Dehradun.
Varun, being a brilliant student, secured a job in Accenture through Campus placement. He attended
a three months’ residential training programme and joined as Assistant Manager (Operations) in
Pune branch of the company. He took a one BHK flat on rent at Hinjewadi locality, purchased some
furniture and other daily household items and got himself settled in the new atmosphere. He was
happy and content as the package offered to him was very lucrative.
Mrs. Bisht had high aspirations and was desirous of expanding her business further. Therefore, in
the year 2013, she thought of exporting her products to various countries crossing the borders of
India. After discussing with her family members, she decided to convert her proprietary concern into
a private limited company. Accordingly, she got registered her company under the title Bisht Designer
Candles Pvt. Ltd. in which she and her daughter were directors while all of the four family members
were shareholders. Thereafter, she completed various formalities required for exporting her product
which, inter-alia, included obtaining a ten digit importer-exporter code (IEC) number from Directorate
General of Foreign Trade (DGFT).
In the year 2015, she sent her first export consignment of designer candles to a foreign buyer in
Berlin, Germany. The order amounted to € 20800 and the importer was required to make payment
in three months after shipment. As per the terms and conditions a Letter of Credit (L/C) was opened
by the Deutsche Bank on behalf of the importer. Before shipping goods, Mrs. Bisht had to fill requisite

© The Institute of Chartered Accountants of India


t.me/cafinal6d - telegram link
Page 437
export declaration form since the consignment did not fall in exempted category as mentioned in
Regulation 4 of the FEM (Export of Goods and Services), Regulations, 2015. After shipment of
goods, she submitted the documentary bill of exchange drawn under L/C to Syndicate Bank,
Dehradun and got it discounted under her sanctioned bills discounting limit. On the due date
Syndicate Bank received the export payment and squared off her liability.
Subsequently, she explored candle market in the USA and came in contact with M/s. Williams’ Art
Gallery in Boston which had a five storey departmental store. In this store, one of the floors was
meant only for designer candles and other items made of wax. After due negotiations with the CEO
Mr. Williams, she managed to get an advance of 50,000 USD being 50% of the total export value. It
was well within her knowledge that in case an advance was received against export to a foreign
buyer, the shipment of goods was to be made within one year of receipt of advance and the export
documents were required to be routed through the same authorised dealer which received the
advance on her behalf. She shipped the goods much before one year and also got payment well
within the statutory period of nine months from the date of export.
On the of successful settlement of her first export consignment to M/s. Williams’ Art Gallery of Boston,
she took steps to complete another export order from the same party for USD 1,00,500. However,
this time no advance payment was made by the importer and on the basis of his firm order, she
dispatched the consignment of designer candles. After shipment of goods, she submitted the
documentary B/E to Syndicate Bank, Dehradun for discounting. As per the agreement, the importer
was to make payment on the completion of five months from the receipt of consignment at his
godown. However, by the time five months were over, the importer could make payment of only 40%
of the total export value.
Being in need of funds, she started raising and collecting funds from various sources. In one of the
cases she had given an unsecured loan of Rs. 5 lacs to a private limited company in which a distant
relative of her husband was a director. However, when she demanded her loan back from the
company, it was transpired that the company was under liquidation process before the National
Company Law Tribunal under Insolvency and Bankruptcy Code, 2016.
Varun was doing his job at Pune to the complete satisfaction of his superiors. In the next three years’
time after joining Accenture, Varun could save a lot of money as he was a man of few needs. One
day, a casual talk with the local grocer Ajay Gupta gave him an idea to buy a flat in a housing society.
Ajay gave him the phone number of a known property dealer, Mr. Rajnikant. Thereafter, a meeting
was fixed in the office of Mr. Rajnikant where he noticed a Certificate of Registration hanging on the
wall of his office. On enquiry, he was told that now it was mandatory for the property agents to get
themselves registered under Real Estate (Regulation and Development) Act, 2016. After seeing the
certificate Varun could conclude that he was dealing with a genuine person. After due negotiations,
a ground floor 2BHK apartment was finalised in Vayudoot Apartments at a cost of Rs. 62.35 lacs. He
himself arranged Rs. 30 lacs out of his savings; obtained a housing loan of Rs. 20 lacs from Axis

© The Institute of Chartered Accountants of India


t.me/cafinal6d - telegram link
Page 438
Bank while the remaining amount of Rs. 12.35 lacs was given by his father out of his personal
savings. The title deeds got registered in his name after making payment of stamp duty and other
statutory dues. On an auspicious day Varun shifted to his new flat.
About after a month of shifting to his own flat, Varun’s boss called him and informed that recognizing
his hard work and devotion towards the company he was being transferred to Boston, USA on
promotion as Manager (Operations). He was beaming with happiness and thanked his boss from the
bottom of his heart. He was supposed to join within next one month.
He went back to Dehradun, completed various formalities including obtaining of visa, packed his
belongings and bade goodbye to his family. On the advice of his father he leased out his flat on rent
to a reputed private company and then flew to Boston and joined his job. Over there, he was provided
with a furnished apartment by the company in the suburbs of Boston. As daily commuting was a bit
difficult, he purchased a second-hand SUV. Slowly and gradually he settled in his new home, new
office and new country.
Here in Dehradun, Mrs. Bisht was pursuing vigorously to obtain export payment from M/s. Williams’
Art Gallery because the statutory period of nine months was over long back and the remaining
payment was yet to be received. In the meantime, the authorities at Syndicate Bank also started
pressurizing Mrs. Bisht to get the foreign exchange realised at the earliest since the statutory period
of nine months was already over. They opined that in cases of default the Reserve Bank of India
may also issue appropriate directions for the purpose of securing the payment if the goods were sold
in USA or if they were still unsold to get them re-imported into India within the specified period.
Though the Reserve Bank had not so far issued any directions but according to her bankers,
omission on the part of RBI to give directions did not absolve her from the consequences of
committing the contravention. Therefore, she was duty bound to realise the export payment as early
as possible.
Besides taking various steps, she also persuaded her son Varun who was already in Boston to follow
the matter vigourously and advised him to meet Mr. Williams personally and settle the case. A
meeting was fixed and during conversation, it was transpired that though Mr. Williams had sold whole
of the consignment, the purchaser was yet to make payment because of some mismanagement.
However, on the vigourous persuasion of Varun, Mr. Williams exerted pressure on the local
purchasers and within next one month, remaining payment along with interest was realised and
repatriated to India.
Varun had a school friend Raman Verma in India who had done MBA from Symbiosis, Pune after
his graduation from Dehradun and had joined sales team of LIC at Shimla. From time to time after
joining Accenture in Boston, Varun was persuading him to visit Boston and nearby areas along with
his wife Vaishnavi Verma. At last, Raman and his wife agreed for the foreign visit and both of them
obtained visa.

© The Institute of Chartered Accountants of India


t.me/cafinal6d - telegram link
Page 439
Raman approached Canara Bank, Shimla for purchase of USD 12,000 for a private visit to the USA.
The bank without much formalities gave him the required amount in foreign currency since it was
well within USD 2,50,000, i.e. an amount which could be remitted by a resident individual in a
financial year under Liberalised Remittance Scheme (LRS). Moreover, the foreign currency was not
required to be remitted for any prohibited current account transaction [mentioned in Schedule I to the
FEM (Current Account Transactions) Rules, 2000] like participation in lottery schemes or lottery like
schemes existing under different names like money circulation scheme or remittances for the
purpose of securing prize money/awards, etc. He was asked to submit a simple letter containing the
basic information, viz., his name, address and that of beneficiary (i.e. self), SB account number,
amount to be remitted and the purpose of remittance along with a cheque of equivalent amount in
rupees. In no time, both of them reached the USA.
Varun received Raman and his wife with open heart at the Logan International Airport, Boston and
all of them drove to his residence. The next ten days were full of fun and frolic. They visited a number
of famous sites which included John F. Kennedy Presidential Museum & Library, Boston Public
Library which was opened in 1852 as the first free publicly-supported municipal library in America,
Museum of Fine Arts having world's most comprehensive art collections, Boston Public Garden
famous for its Swan Boats and having over 600 varieties of trees, Old North Church & Historic Site
where the two famous signal lanterns were hung launching the American Revolution, New England
Holocaust Memorial where its six glass towers represented the six million Jews who perished in the
holocaust, Bunker Hill Monument, etc. In between, they had an overnight stay at New York as well.
Varun helped them in purchasing some nice dresses, chocolates, perfumes, cosmetic items and also
some souvenirs for their relatives and friends in India. They enjoyed their trip to USA to the fullest
and flew back to India with nice memories.
Raman still had with him unspent amount of USD 3500. On enquiry with his bankers regarding
surrender of this amount he was informed that he could surrender to the bank any unspent foreign
exchange within a period of 180 days from the date of his return to India. Even if he approached the
bank after this period, the bank would not refuse to purchase unspent foreign exchange merely
because the prescribed period of 180 days had expired. He was further informed that he was
permitted to retain with him foreign currency notes up to USD 2000 and foreign coins without any
ceiling beyond 180 days and he could utilize this amount for his subsequent visit abroad.
Varun wanted to be inform regarding sale of his flat in Pune if he was to settle down in the USA
permanently since his family at Dehradun was not that much inclined to keep the flat. He once again
approached Mr. Rajnikant and enquired whether he, as NRI, could sell his flat. Mr. Rajnikant after
obtaining necessary information from one of his lawyer friends, informed him that he was permitted
to sell his flat in India to a person resident in India. Further, he could also sell the flat (since it was
not an agricultural or plantation property or farm house) to a person resident outside India who is an
Indian citizen or to a person of Indian origin resident outside India. Such permission was available

© The Institute of Chartered Accountants of India


t.me/cafinal6d - telegram link
Page 440
under Regulation 3 of FEM (Acquisition and Transfer of Immovable Property in India), Regulations,
2000.
As regards purchase of immovable property at Boston, Varun was informed that FEMA did not restrict
such acquisition by a non-resident Indian and he had to follow local laws in this respect. However, if
his family members in India remitted to him funds under the Liberalised Remittance Scheme (LRS)
for purchasing immovable property outside India, then the said property should be in the name of all
the members who made the remittances. Even as per Section 6(4) of the FEMA, if he becomes a
person resident in India in future, he would be allowed to hold, own or transfer the immovable
property situated outside India because such property was acquired by him when he was resident
outside India.
I. Objective Type Questions (2 marks each)
Select the most appropriate answer from the options given for each question:
1. Which of the following remittance would require prior approval of the Reserve Bank of India?
(a) Donation exceeding 0.5% of foreign exchange earning during the previous three financial
years or USD 40,00,000, whichever is less for contribution to funds promoted by educational
institutes,
(b) Commission per transaction to agents abroad for sale of commercial plots in India of USD
20,000 or 4% of the inward remittance whichever is more,
(c) Remittance exceeding USD 10,00,000 per project for other consultancy services procured
from outside India.
(d) Remittance of 4% of investment brought into India or USD 90,000 whichever is higher, by
an entity in India by way of reimbursement of pre-incorporation expenses.
2. Mr. O. P. Bisht’s name does not appear in the registration papers relating to Pune apartment
purchased by Varun though he contributed Rs. 12.35 lacs towards the cost of the apartment.
(a) It is a benami transaction to the extent of Rs. 12.35 lacs.
(b) It is wholly a benami transaction.
(c) It is not a benami transaction
(d) None of the above
3. Export of the following goods/software would require furnishing of the declaration under FEMA,
1999?
(a) Goods imported free of cost on re-export basis
(b) Publicity material supplied free of payment
(c) By way of gift of goods accompanied by a declaration by the exporter that they are of six
lakh rupees in value

© The Institute of Chartered Accountants of India


t.me/cafinal6d - telegram link
Page 441
(d) Unaccompanied personal effects of travellers
4. An exporter receiving advance payment against exports from the foreign buyer is required to
make the shipment of the goods within ---------- of receiving advance payment, if export
agreement does not mention anything to the contrary regarding time period:
(a) 6 months
(b) 9 months
(c) One year
(d) One and a half years
5. An Indian citizen resident outside India is permitted to transfer his agricultural property in India
to:
(a) any person resident in India
(b) any person resident outside India if he is a citizen of India or a person of Indian origin.
(c) Neither (a) nor (b)
(d) both (a) and (b)
6. Foreign exchange purchased from an authorised dealer by a resident individual, if remains
unspent, needs to be surrendered to the authorised dealer within ------------ of purchase or date
of his return to India:
(a) 60 days
(b) 90 days
(c) 120 days
(d) 180 days
7. In case of goods valuing up to Rs. 5,00,000 as declared by the exporter and sent by way of
gift to an importer in a foreign country:
(a) an export declaration need to be furnished
(b) an export declaration need not be furnished
(c) furnishing of export declaration depends upon the discretion of the authorised dealer who
handles export documents
(d) furnishing of export declaration depends upon the discretion of the Custom authorities
8. The term 'Moratorium' in the Insolvency and Bankruptcy Code, means-
(a) A temporary prohibition on an activity by the competent authority.

© The Institute of Chartered Accountants of India


t.me/cafinal6d - telegram link
Page 442
(b) A period declared by the NCLT, during which no action can be taken against the
Company or the assets of the Company.
(c) Suspension order of the Board on the debtor's operations.
(d) Order issued by the NCLT prohibiting an action against the creditor.
9. Is it possible for a non-resident Indian to acquire immovable property outside India:
(a) No, it is not possible
(b) Yes, it is possible
(c) Yes, it is possible but subject to the permission of RBI
(d) None of the above
10. As per the Insolvency and Bankruptcy Code, 2016, an Interim Resolution professional
approved by the committee of Creditors:
(a) Can never be replaced until the conclusion of the resolution process
(b) Has a fixed term of 180 days
(c) Can be replaced with 75% voting in favour of the decision and approval of the Board
(d) Can be replaced with 75% voting in favour of the decision.

II Descriptive Questions (10 marks each)


1. Analyse the following situations under the Foreign Exchange Management Act, 1999:
(i) Forex Dealers Ltd. is an Authorised Person within the meaning of Foreign Exchange
Management Act, 1999. Reserve Bank of India issued certain directions to the said Autho rised
Person to file certain returns, which it failed to file. You are required to state the penal provisions
to which the said Authorised Person has exposed itself.
(ii) Mr. Shekhar resided for a period of 150 days in India during the Financial year 2016-2017
and thereafter went abroad. He came back to India on 1st April, 2017 as an employee of a
business organization. What would be his residential status during the financial year 2017-
2018?
(iii) ‘Printex Computer’ is a Singapore based company having several business units all over the
world. It has a unit for manufacturing computer printers with its Headquarters in Pune. It has a
Branch in Dubai which is controlled by the Headquarters in Pune. What would be the residential
status under the FEMA, 1999 of printer units in Pune and that of Dubai branch?
2. Examine with reference to the provisions of the Foreign Exchange Management Act, 1999
whether there are any restrictions in respect of the following:-

© The Institute of Chartered Accountants of India


t.me/cafinal6d - telegram link
Page 443
(i) Drawal of Foreign Exchange for payments due for depreciation of direct investment in the
ordinary course of business.
(ii) A person, who is resident of U.S.A. for several years, is planning to return to India
permanently. Can he continue to hold the investment made by him in the securities issued
by the companies in U.S.A.?
(iii) A person resident outside India proposes to invest in the shares of an Indian company
engaged in construction of farm houses.
(iv) A person, who is resident of Canada, is planning to acquire an immovable property in
Mumbai.
3. Analyze the following situations under the Real Estate (Regulation and Development) Act,
2016:
(i) Mr. Ram booked a 4 BHK flat under the Gateways project. The project is under supervision
of Mr. Pankaj. Mr. Pankaj without telling the allottees reduced the number of rooms from
4 to 3 himself. Whether this is allowed under the Act and what remedies does the Allottees
have.
(ii) Mr. Vivaan booked a 4 BHK flat under the Flower Valley project for a total cost of Rs. 2
Crore. The project is under supervision of Mr. Shyam. Mr. Shyam put a condition to pay
Rs. 50 Lakhs as an application fee before entering into a written agreement for sale with
Mr. Vivaan. Decide whether the contention of Mr. Shyam is valid?

© The Institute of Chartered Accountants of India


t.me/cafinal6d - telegram link
Page 444
QUESTION 2
Ronit Chawla was a Fellow Chartered Accountant (FCA) practicing in the field of corporate and
economic laws. He represented his clients before Company Law Board (CLB) and thereafter in
National Company Law Tribunal (NCLT). After coming into force of Insolvency and Bankruptcy Code,
2016 w.e.f. 28 May, 2016, he learnt about the Limited Insolvency Examination (LIE) for becoming
Insolvency Professional (IP). Since he had about eleven years of experience as practicing CA, he
attempted the very first examination of LIE conducted by Insolvency and Bankruptcy Board of India
(IBBI) in December 2016 and successfully cleared it. He then enrolled himself with a reputed
Insolvency Professional Agency (IPA) and got registered with IBBI by fulfilling the requisite formalities
including payment of non-refundable application fee of Rs. ten thousand.
His father Roopesh Chawla, a resident of Green Park, New Delhi, was recently posted as Chief
Manager in Bank of India, Delhi which was a full-fledged Foreign Exchange (FX) branch though
Roopesh, being unable to get a chance to work in a FX branch, had very little knowledge of rules
relating to Foreign Exchange. Therefore, he used to consult his son Ronit in the matters of foreign
exchange from time to time. His mother Rukmani Chawla was a senior teacher in Kendriya
Vidyalaya, New Delhi, taking commerce classes.
Rajnish Sinha, a close friend of Roopesh, was heading a Delhi branch of Punjab National Bank
(PNB) and knew that Roopesh’s son Ronit besides being a Chartered Accountant was also an
Insolvency Professional. Rajnish, on behalf of PNB, wanted to initiate corporate insolvency resolution
process (CIRP) before NCLT in the case of its customer Manohar Masale Pvt. Ltd. (MMPL) of Delhi
which had defaulted in repaying the dues of the bank totaling approximately Rs. 23.00 lacs.
Accordingly, PNB being financial creditor, while making an insolvency resolution application to NCLT
proposed the name of Ronit as Interim Resolution Professional (IRP). MMPL was sanctioned cash
credit limit of Rs. 10.00 lacs against hypothecation of stock of raw material and finished goods and
another bill discounting limit of Rs. 5.00 lacs against actionable claims. MMPL was registered with
an authorised capital of Rs. 25.00 lacs but its paid up capital was to the tune of Rs. 10.00 lacs.
Initially started as a registered partnership concern (Manohar Masale & Co.) by two brothers, namely,
Ram Manohar and Shyam Manohar, it did profitable business and keeping an eye on future business
growth, it was converted into a private limited company with Ram, Shyam and Shyam’s elder son
Shivam as directors. Shyam’s younger son Dwapam, an alumnus of IIFT, Delhi and also a law
graduate, did not have any interest in the family business and was more inclined to continue with his
current employment in a German MNC having its office in Gurugram.
MMPL’s factory in Okhla Industrial Area was located on the one-fourth portion of the plot which was
co-owned by the brothers. However, the bank had created an equitable mortgage on the plot as well
as factory building while sanctioning the working capital limits to the company. The elder brother
Ram Manohar was the anchoring person who steered the company to newer heights due to his sheer
business acumen and inherent managerial skills but one day, all of a sudden, he had a massive

© The Institute of Chartered Accountants of India


t.me/cafinal6d - telegram link
Page 445
heart attack resulting in his untimely death. Since he was not married, the business of ‘masale
making’ was now run by Shyam and his son Shivam.
However, the father-son duo could not manage the business properly because of the lack of
foresight, faulty inter-personal relations and poor organisational skills. Their authoritative style of
leadership resulted in demotivation of workers which led to labour unrest and all sort of other
conflicts. The paternalistic approach towards them which Ram always displayed was missing
altogether. Needless to say, the output started declining and wastage of raw material turned north.
Since there was no vigourous follow-up as well, the debtors to the tune of around Rs. 12.00 lacs
were long overdue. Consequently, the company started suffering losses and also defaulted on dues
from the bank.
When PNB, even after repeated reminders to MMPL, could not realise its dues and the liability
touched the height of around Rs. 23.00 lacs (including normal and overdue interest), Rajnish Sinha,
on behalf of PNB, decided to file corporate insolvency resolution application duly supported by ledger
extracts and other specified evidences (services of Information Utility could not be used as by the
time application was filed there was no IU registered with IBBI) with Adjudicating Authority i.e. NCLT,
New Delhi for initiating CIRP against MMPL.
NCLT considered the corporate insolvency resolution application along with the proposed name of
Ronit as Interim Resolution Professional (IRP). Within next 10 days of receipt of application (which
was lesser than the statutory period of 14 days) NCLT ascertained that there existed default because
the defaulted amount was much more than the minimum required of Rs. one lac. Since the CIRP
application was complete in all respects, NCLT admitted it and within the statutory period of next
seven days after admission, it conveyed its order of commencement of CIRP to the financial creditor
(i.e. PNB) and the corporate debtor (i.e. MMPL).
The order of NCLT confirmed the proposed appointment of Ronit as IRP for 30 days, for Ronit had
a clean record without any disciplinary proceedings pending against him. It was also stated in the
order that a moratorium period of 180 days had become applicable during which all suits and legal
proceedings, etc. against MMPL (i.e. corporate debtor) were to be held in abeyance so as to give
time to the ailing company to resolve its status. MMPL was also barred from transferring or disposing
of any of its assets or any legal rights therein. However, the supply of specified essential goods and
services to the MMPL as mentioned in the order, were not to be interrupted during moratorium period.
In the meantime, Ronit’s father Roopesh faced a peculiar problem related to the foreign exchange
matter at his branch. His FX officer brought to his knowledge that one of their exporter customers
who had received an advance of USD 75,000 from an importer based at California, USA against
export of ready-made jeans had not shipped the requisite items worth USD 2,00,000 by utilizing the
advance so received. The exporter, not willing to ship the goods, wanted to refund the advance to
the importer along with interest for which permission of Roopesh was required. Roopesh did not
allow the refund immediately and in turn, advised the FX officer to gather more knowledge about FX

10

© The Institute of Chartered Accountants of India


t.me/cafinal6d - telegram link
Page 446
provisions whether refund along with interest was permissible. At the same time he also discussed
the matter with his son Ronit who advised him to refer FEM (Export of Goods and Services)
Regulations, 2015. A scrutiny of the relevant banking records revealed that 14 months had already
expired since advance of USD 75,000 was received. Further, he came to know that if goods were
not shipped within one year of receipt of advance, such advance could not be refunded without the
permission of the RBI. Accordingly, he advised the customer to seek permission of RBI through his
branch.
After his appointment as IRP, Ronit assumed full control of the affairs of MMPL. Since powers of the
board of directors stood suspended he was empowered to exercise such powers. Accordingly, he
took immediate custody and control of all the assets of the MMPL including its business records.
Following the orders of NCLT, Ronit took steps to make a public announcement within three days
from the date of his appointment regarding the initiation of CIRP against MMPL.
Public announcement, included the following aspects:
 Name and address of the corporate debtor (i.e. MMPL) and its registration/incorporating
authority.
 His details as IRP and the fact that he would be vested with the management of the
corporate debtor and be responsible for receiving claims.
 Penalties for false or misleading claims.
 The last date for the submission of the claims.
 The date on which the CIRP would end.
After the expiry of last date for submission of claims, a Committee of Creditors was constituted which
included PNB and five trade creditors who had cumulative dues of Rs. 3.00 lacs. Within seven days
of its constitution, the first meeting of the committee was called. In the meantime, Ronit electronically
submitted an Information Memorandum to the creditors after they had given an undertaking
regarding maintaining of confidentiality. This Information Memorandum contained details of assets
and liabilities of the MMPL with their estimated values, audited financial statements for the last two
financial years and provisional financial statements for the current financial year made just eight days
earlier from the date of the application, a list of creditors and the amounts claimed by them which
were duly admitted and other prescribed information.
In the meeting of the Committee of Creditors it was resolved to let Ronit continue as full-fledged
Resolution Professional (RP) since he was eligible to be appointed as an independent director and
was not a related party of the MMPL and such decision was conveyed to the NCLT as well as MMPL.
As RP, Ronit assumed all those powers which were conferred on him as IRP. He was required to
manage the operations of the MMPL during the CIRP period.

11

© The Institute of Chartered Accountants of India


t.me/cafinal6d - telegram link
Page 447
Based on the Information Memorandum, Rajnish on behalf of PNB as resolution applicant undertook
to prepare a resolution plan as per the provisions of the Code for onward submission to Ronit. Before
finalizing the resolution plan, he along with his two officers took up the matter with Shyam and his
son Shivam regarding the revival of MMPL and repayment of long outstanding dues or face
liquidation if they were not inclined to revive the company. The fear psychosis of liquidation made
them think frantically to save their company from imminent death. Having woken up from their
slumber they started exploring ways to bring in short term finance and also to rope in some
professional who would help the company in its revival.
Shyam saw a ray of hope in his younger son Dwapam and persuaded him to participate in the
management of the affairs of the company at least for the first three months to which he ultimately
agreed. In the meantime Shyam, with a view to raise short term finance, consulted his elder sister
Rama Devi to lend at least Rs. 5.00 lacs for a short period of about one year and also convinced his
daughter Ria, her husband Dushyant as well as Dwapam to invest at least Rs. 3.00 lacs each in the
share capital of the company. Shivam who had invested funds in the share market agreed to sell his
securities to raise Rs. 3.00 lacs against which he was to be allotted shares in the MMPL. As per the
advice of the bankers, Shyam also started inter-acting with long overdue debtors for recovery who
eventually agreed to pay 50% of Rs. 12.00 lacs in the current month and remaining amount in the
next month. Out of the raised amount, the operational creditors were to be paid fully while dues of
PNB were to be satisfied to the extent of Rs. 12.00 lacs. Further, Rs. 2.50 lacs were to be allocated
towards insolvency resolution process costs including fee of RP and remaining amount was to be
utilized as working capital. Since both the directors of MMPL had consented to repay Rs. 12.00 lacs
in one lump sum, Rajnish on behalf of PNB assured them that he would take up the matter of waiving
of overdue interest up to Rs. 2.00 lacs with his Dy. General Manager and would also seek permission
to revive MMPL’s limits which were currently frozen.
Based on the experience he gathered while working with two MNCs, Dwapam assumed the role of
a leader to set the company on rails. He took note of the prevailing situation from which the ailing
MMPL was passing through. He observed that the current as well as liquid ratios were much far away
from the standard norms of 2:1 and 1:1 respectively. The turnover ratios were also unhealthy and at
the same time the operating ratio was very high - not a good sign for any business. An investment
of about Rs. 5.00 lacs was tied up in raw material like whole red chillies, coriander seeds, turmeric,
black pepper, dry mango, etc.
Since currently the business of spices was run in a traditional manner, Dwapam decided to take the
following short, medium and long term measures:
Short term measures:
 to understand the needs and wants of customers in the target market;
 to apply the principles of scientific management;
 to set standards for raw material, wastage, working conditions, etc.;

12

© The Institute of Chartered Accountants of India


t.me/cafinal6d - telegram link
Page 448
 to conduct time and motion studies;
 to provide financial incentives and to adopt social security plans for the workers;
 to secure registration with FSSAI immediately;
 to appoint an Administrative Officer and, if need be, to appoint another one in future;
 to devise competitive pricing strategy;
 to create a corporate brand identity by assigning the product a brand name ‘Manohar Uttam
Masale’ which would help in building a brand image;
 to design an attractive package and label by using a graphic design of spices combining
green, yellow and red colours for different varieties of masale;
 to promote the masale by advertising initially in leading newspapers and depending upon
income generation in future, to advertise on FM radio, TV as well as cinema halls;
 to adopt sales promotion measures like free gift offers, contests, free sample distribution,
etc.
 to select the similar channels of distribution as used by the competitors;
 to conduct SWOT analysis of MMPL and important competitors;
 to create an effective Website of the company;
 to take decisions regarding various activities under physical distribution of masale like order
processing, transportation, warehousing and inventory control;
 to adopt strict credit policy by reducing debtors’ days with a regular follow-up;
 to use an accounting software;
 to submit various Government Returns within the prescribed time limits so that avoidable
hefty penalties are not levied.
Medium and Long Term Measures:
 to stop heavy expenditure on repairs and maintenance by installing new machines and
grinders;
 to establish direct contacts with the cultivators for obtaining raw material which would help
in avoiding middlemen and their high commissions;
 to develop the remaining three-fourth portion of the plot and rent out some of the developed
portion to a commercial establishment;
 to renovate the factory building.
 to manufacture more types of different spices like Rajma Masala, Pindi Chana Masala,
Shahi Paneer Masala, Dal Makhni Masala, Mushroom Matar Masala, etc;
 to diversify MMPL’s operations by manufacturing Jams and Ketchups;
 To explore offshore markets.
Rajnish prepared a resolution plan containing the above strategies and submitted it to Ronit for his
consideration. Later on, a meeting of committee of creditors was called by Ronit and the resolution
plan was presented for its approval. The plan was duly approved by full majority. Thereafter, Ronit
submitted the approved resolution plan to the NCLT for its approval.

13

© The Institute of Chartered Accountants of India


t.me/cafinal6d - telegram link
Page 449
Since the resolution plan was approved by the committee of creditors much before the statutory
period of 180 days and also met the prescribed requirements, NCLT approved it and passed an order
to this effect. Now the plan was binding on the MMPL and its employees, members, PNB and
operational creditors as well as other stakeholders involved in the resolution plan.
I. Objective Type Questions (2 marks each)
Select the most appropriate answer from the options given for each question:
1. “Default” under the IBC is said to be occurred on the fulfillment of condition/s-
(a) Debts becoming due and payable
(b) Non- payment of the debt
(c) Liability /obligation in respect of a claim which is due
(d) Both (a) & (b)
2. In the case study PNB initiated Corporate Insolvency Resolution Process against MMPL for the
default in the capacity of-
(a) Corporate debtor
(b) Operational debtor
(c) Financial creditor
(d) Resolution applicant
3. If the goods against which an advance payment is received from a foreign buyer are not shipped
within one year and there exists no agreement regarding timing of shipment, the advance
payment:
(a) shall be refunded within reasonable time without prior approval of Reserve Bank.
(b) Shall be refunded within one year from the date of receipt of advance payment without the
prior approval of Reserve Bank
(c) Shall be refunded within one year from the date of receipt of advance payment with the prior
approval of Reserve Bank
(d) Shall be refunded after one year from the date of receipt of advance payment on the basis
of reasonable cause.
4. PNB through an assignment agreement, assigned here the debt to the X trust. X trust filed the
petition for initiation of corporate Insolvency resolution process (CIRP) against MMPL. State the
correct statement with respect to the competency of the X trust in the filing of the petition in the
above situation-
(a) X Trust is not a competent applicant as per section 6 of the IBC
(b) X Trust is being authorized by the PNB to file an application

14

© The Institute of Chartered Accountants of India


t.me/cafinal6d - telegram link
Page 450
(c) X Trust in the capacity of financial creditor can file a valid petition.
(d) None of the above
5. As per the Insolvency & Bankruptcy Code, 2016, resolution plan is prepared by ----------- is
submitted to ----------------- for examination and submission to ------------- for approval.
(a) Committee of Creditors, Adjudicating Authority, Resolution Professional
(b) Resolution applicant, committee of creditors, Adjudicating Authority
(c) Resolution applicant, Resolution Professional, Committee of Creditors
(d) Committee of Creditors, Resolution Professional, Adjudicating Authority
6. The maximum duration during which the appointment of Interim Resolution Professional (IRP)
is valid shall not exceed -------- days.
(a) 10
(b) 20
(c) 30
(d) 40
7. In the case study, the expenses of public announcement shall be borne by the-
(a) MMPL
(b) Ronit
(c) Roopesh
(d) PNB
8. In the case study, committee of creditors of MMPL was constituted on 17.3.2018. Time limit,
within which the first meeting of committee of creditors should be held, is ----------------.
(a) 20.3.2018
(b) 22.3.2018
(c) 24.3.2018
(d) 31.3.2018
9. Ronit, being an Insolvency Professional can be appointed as Resolution Professional, if:
(a) he is eligible to be appointed as an independent director under section 149 of the Companies
Act, 2013
(b) he is not a related party of the corporate debtor
(c) only (a)
(d) Both (a) and (b)

15

© The Institute of Chartered Accountants of India


t.me/cafinal6d - telegram link
Page 451
10. MMPL finds material irregularity in exercise of the powers of the Ronit during the corporate
insolvency resolution period. Remedy available to MMPL-
(a) File a complaint to the adjudicating authority
(b) Complain to the committee of creditor’s
(c) Complaint filed before the IBBI
(d) File an appeal against the order of adjudicating authority against the approval of resolution
plan.
II. Descriptive Questions (10 marks each)
1. Suppose the resolution plan prepared by Rajneesh was delayed in approval by committee of
creditors. Ronit, further presented the said resolution plan, before NCLT after 180 days of
insolvency commencement date.
Answer the following-
(i) What step shall be taken by NCLT on such presented resolution plan.
(ii) What, if MMPL contravened the resolution plan which effected its employees and stake
holders.
(iii) What consequences be there where liquidator continued the business of MMPL during
liquidation process.
2. Ronit in an examination of sale of property of MMPL finds that a transaction was made by the
MMPL to Rama devi (the elder sister of Shyam) in 6 months preceding the Insolvency
Commencement date, was undervalued.
Give the following answers in reference to the above situation-
(i) State the validity of the conduct of such transaction by MMPL to Ramadevi.
(ii) What will be the consequences when resolution professional determines such transactions
undervalue and fails to report that same to NCLT?
(iii) What order NCLT shall pass when MMPL entered into an undervalued transaction?
3. (a) Discuss the legal position and liability of Mr. Shyam in the following given situations-
(i) Where Mr. Shyam fraudulently transferred his holding of shares in favour of his sister
of Rs.1 lakh within 1 year immediately preceding the insolvency commencement date.
(ii) Mr. Shyam makes false entry in the books of account of MMPL to defraud creditors on
insolvency commencement date.
(iii) Shyam permitted Shivam to provide information for initiation of CIRP which is false in
material particular and omits material fact related to a books of accounts of a specified
period in the application.
(b) What course of action can be taken by NCLT against the directors of the MMPL for
transactions defrauding creditors?

16

© The Institute of Chartered Accountants of India


t.me/cafinal6d - telegram link
Page 452
QUESTION 3
During March 2017, XMC Pvt. Ltd., a car manufacturing company, launched its TXI model of car
with a lot of advertisements and promotions in all types of media platforms, inter alia, highlighting
the Ex-showroom price of the said car model in Mumbai as Rs. 6.25 lacs.
Mr. Nazir, a prospective buyer of the said model, visited an authorised dealer of XMC Pvt. Ltd.
i.e. M/s Ratan Lal & Sons located at Bandra, Mumbai and after due consultation/ discussion
with the representatives of M/s Ratan Lal & Sons, booked a vehicle of the aforesaid model on
11 th May, 2017 on payment of Rs. 100,000/-. M/s Ratan Lal & Sons in turn provided the money
receipt for the aforesaid transaction with serial number ABC/1010 as well as booking reference
number 218/ 2017 to Mr. Nazir. He was assured by the representatives of M/s Ratan Lal & Sons
that the booked vehicle will be delivered within three months from the date of booking i.e. by
10 th August, 2017. However, the representative of M/s Ratan Lal & Sons have stated to Mr.
Nazir that as per XMC Pvt. Ltd.’s policy, five months’ time is given in writing so as to keep some
buffer for delays which may arise due to unforeseen exigencies or transportation of vehicle or
other logistic problems. Mr. Nazir, inter alia, noted the conditions in the booking document that
“the vehicle would be delivered within six months from the date of booking”. Believing the
assurance given by the representative of M/s Ratan Lal & Son, Mr. Nazir accepted the terms of
the booking and thought that he will get the vehicle within three months from the date of booking
as assured by the representatives of M/s Ratan Lal & Son and in worst scenario he will get
delivery of the vehicle within six months from the date of booking as per the terms and conditions
of booking of the vehicle.
However, within three months of booking of the vehicle, M/s Ratan Lal & Son failed to deliver
the vehicle to Mr. Nazir despite repeated request and after 10 th August, 2017, Mr. Nazir
contacted the representatives of M/s Ratan Lal & Sons many times for delivery of the vehicle
and they kept on giving assurances that the delivery of the vehicle will be done within six months
from the date of booking as per the conditions of booking. After five months, on 15 th February

2018, Mr. Nazir written an e-mail to XMC Pvt. Ltd. highlighting the issue of delay in delivery of
the booked vehicle, but did not get any response. Then he wrote an email to the President of
XMC Pvt. Ltd. and got the reply that his grievances will be looked into by the sales team of the
Company and the concerned dealer.
Despite the assurance of the president of XMC Pvt. Ltd., the booked vehicle was not delivered
to Mr. Nazir. Rather, through M/s Ratan Lal & Sons, he was informed that due to delay in
production of the said model, the Company is not able to deliver the same and he was asked to
wait for some more time. Subsequently, he received a letter from XMC Pvt. Ltd wherein, inter
alia, it was informed that due to unprecedented number of bookings for the said model the
delivery of the car will be delayed for two months. Through the said letter, it was also informed
that the price of the booked car will be revised and it will be effective from the date of booking
by dealer to the customer.
About the market and the state of competition

17

© The Institute of Chartered Accountants of India


t.me/cafinal6d - telegram link
Page 453
As per Mr. Nazir, XMC Pvt. Ltd. is a big player in the car manufacturing market. Its financial
strength and brand name is much more compared to other players in the market. Also, it
commands largest market share in terms of sales and revenue compared to its competitors and
in the last financial year XMC Pvt. Ltd. acquired a loss making car manufacturing company i.e.
Trisha Ventures Pvt. Ltd. As per Mr. Nazir, XMC Pvt. Ltd. has taken recourse to terms and
conditions of the booking documents to enforce price hike and also not honouring the
commitment made for the delivery within the given time period despite repeated
correspondence. XMC Pvt. Ltd. and its dealer at Mumbai M/s Ratan Lal & Sons started the
gimmick of non-delivery due to production delay and started informing that there will be higher
price of the vehicle. Mr. Nazir alleged that he and other similarly situated consumers are being
not given with delivery of the vehicle in due time and the delay tactics done by XMC Pvt. Ltd. is
to increase the price of the vehicle and to exploit the consumers by not giving the benefit of
initial launch price which is not fair in a competitive market.
Concerns raised
As per Mr. Nazir, XMC Pvt. Ltd. has abused its powers to fix the price of the vehicle. It has
initially priced attractively and launched with heavy advertisements and promotions to lure the
customers and take maximum bookings by taking interest free amount of Rs. 100,000/- as
booking amount. By doing this XMC Pvt. Ltd. has been able to not only generate huge amounts
of cash which is interest free but also create buzz in the market because of publicity in the media
regarding heavy bookings of the said vehicle. It is stated that XMC Pvt. Ltd. has arbitrarily
increased the price of the vehicle to encash on the market demand. Not only that, the Company
has also not passed on the benefit of recent GST reduction on the passenger cars by
Government to the consumers in the said car model. However, it has passed on the benefit of
the GST reduction on its other car models to the customers which are not in such demand. Most
of the other car manufacturers have duly passed on the GST reduction to the customers. As per
Mr. Nazir, XMC Pvt. Ltd. has indulged in unfair practices in connivance with its dealers by
manipulating its delivery policy and price policy. After seeing huge response because of
attractive initial offer price, it not only delayed in giving delivery of the booked car but also
increased the price which is nearly two times of the offer price at the time of booking. It has not
honored the commitment of delivery and price to the buyer who had booked on the very first day
and first hour of the launch. Mr. Nazir stated that it is not just an individual issue but it involves
the larger interests of car buyers, who do not have any recourse to effective mechanism against
the abuse of dominant position by such auto manufacturers for imposing anti-competitive terms
on the buyers.
Based on the above submissions Mr. Nazir alleged that the aforesaid conduct of XMC Pvt. Ltd.
is not in tandem with the provisions of the Competition Act, 2002 and it has acted in a manner
which can be termed as anti-competitive.

18

© The Institute of Chartered Accountants of India


t.me/cafinal6d - telegram link
Page 454
I. Objective Type Questions (2 marks each)
Select the most appropriate answer from the options given for each question:
1. Which of the following is the appropriate authority for redressal of the grievances of Mr. Nazir?
(a) District Consumer Redressal Forum
(b) Competition Commission of India
(c) Car Manufacturers Association of India
(d) Both (a) and (b)
2. Under which provisions of the Competition Act, 2002, the grievances of Mr. Nazir can be
examined?
(a) Prohibition of horizontal anti-competitive agreement under section 3(3) of the Competition
Act, 2002
(b) Prohibition of abuse of dominant position under section 4 of the Competition Act, 2002
(c) Prohibition of vertical anti-competitive agreement under section 3(4) of the Competition
Act, 2002
(d) Regulation of combination under section 6 of the Competition Act, 2002
3. Mr. Nazir stated that “it is not just an individual issue but it involves the larger interests of car
buyers, who do not have any recourse to effective mechanism against the abuse of dominant
position by such auto manufacturers for imposing anti-competitive terms on the buyers”. What
would be his prime intention in stating so?
(a) The car manufacturer’s conduct towards him is exploitative
(b) The car manufacturer is imposing anti-competitive terms on him.
(c) The conduct of car manufacturer is not conducive to the market as it affects larger
consumers’ interest.
(d) All the above
4. Let, Mr. Nazir approached the Competition Commission India for his grievances and you are the
person in the Commission to take a decision in the matter and according to you the matter
pertains to abuse of dominance. What would be your sequence of analysis of the matter?
(a) XMC Pvt. Ltd. is dominant or not
(b) Whether the alleged conduct is abusive under section 4 of the Competition Act, 2002

19

© The Institute of Chartered Accountants of India


t.me/cafinal6d - telegram link
Page 455
(c) Whether XMC Pvt. Ltd. falls under the definition of enterprise as defined under the
Competition Act, 2002
(d) Define the relevant market where XMC Pvt. Ltd. is operating
5. Let Mr. Nazir approached the Competition Commission India for his grievances and you are the
person in the Commission to take a decision in the matter and according to you the matter
pertains to vertical restraint under section 3(4) of the Competition Act, 2002. What would be your
sequence of analysis of the matter?
(a) Whether XMC Pvt. Ltd. and M/s Ratan Lal & Sons have entered into an agreement
(b) Whether XMC Pvt. Ltd. and M/s Ratan Lal & Sons are placed at vertical level.
(c) Whether there is any appreciable adverse effect on competition because of anti-
competitive agreement between XMC Pvt. Ltd. and M/s Ratan Lal & Sons.
(d) Whether XMC Pvt. Ltd. and M/s Ratan Lal & Sons have agreed on some issues which
are anti-competitive in terms of section 3(4) of the Competition Act, 2002.
6. If you think delineation of relevant market is necessary to examine the fact of the case, then
what should be the relevant product market in this case?
(a) Market for passenger car
(b) Market for dealership services for passenger car
(c) Market for motor vehicle
(d) Market for non-commercial passenger car
7. Mr. Nazir submitted that XMC Pvt. Ltd. is a dominant market player in the relevant market, if you
agree with his submission, what would be your reasoning?
(a) Market share of XMC Pvt. Ltd. is largest
(b) Competitors of XMC Pvt. Ltd. have lesser financial strength
(c) XMC Pvt. Ltd. is a known brand
(d) Consumers are dependent on XMC Pvt. Ltd.
8. Given the facts that XMC Pvt. Ltd. and M/s Ratan Lal & Sons, in connivance with each other,
have delayed the delivery of the booked passenger car to Mr. Nazir and revised the price of the
said car, it cannot be a case of cartelization. What would be the possible reason?
(a) The fact does not reveal any exclusive agreement between XMC Pvt. Ltd. and M/s Ratan
Lal & Sons.

20

© The Institute of Chartered Accountants of India


t.me/cafinal6d - telegram link
Page 456
(b) The fact does not reveal any agreement of XMC Pvt. Ltd. with other car manufacture in
fixing the price
(c) The fact does not reveal that M/s Ratan Lal & Sons is involved in price fixation of delay in
giving delivery of the car to Mr. Nazir
(d) None of the above
9. In case the Competition Commission of India ordered that Mr. Nazir should approach in the
appropriate forum, what would be your reaction?
(a) The Competition Commission of India is rightly ordered so because the allegations of Mr.
Nazir do not raise any competition concerns in any market.
(b) The order of the Competition Commission of India should be challenged in National
Company Law Appellate Tribunal as it failed to address the concerns of Mr. Nazir in terms
of the provisions of Competition Act, 2002.
(c) Since it is grievance of an individual consumer, Consumer Redressal Forum is the
appropriate authority to deal this matter.
(d) None of the above
10. If you think that XMC Pvt. Ltd. has abused its dominant position, then which of the following
conduct of XMC Pvt. Ltd. is abusive in terms of Section 4 of the Competition Act, 2002?
(a) not giving delivery of the booked car within the assured time
(b) The President of XMC Pvt. Ltd. vide its mail to Mr. Nazir informed that the price of the
booked vehicle will revised and it will be applicable on the date of invoice by dealer to the
customer
(c) XMC Pvt. Ltd has not passed the benefit of tax deduction to the consumers
(d) None of the above.
II. Descriptive Questions (10 marks each)
1. Do you think that the concerns raised by Mr. Nazir can be examined through the
provisions of the Competition Act, 2002? If yes, explain the steps through which the matter
can be examined.
2. Do you think that not giving delivery of the booked car within the assured time without
enabling provisions in the booking form is tantamount to imposition of unfair conditions
and revision of price of the vehicle with effect from the date booking tantamount to
imposition of unfair price on Mr. Nazir? Examine the given situations in terms of the
provisions of the Competition Act, 2002.

21

© The Institute of Chartered Accountants of India


t.me/cafinal6d - telegram link
Page 457
3. What is relevant market? State the provisions of the Competition Act, 2002 to delineate
the relevant market. Delineate the relevant market in the instant case.

22

© The Institute of Chartered Accountants of India


t.me/cafinal6d - telegram link
Page 458
Test Series: March, 2018
MOCK TEST PAPER 1
FINAL (NEW) COURSE: GROUP – II
ELECTIVE PAPER 6D: ECONOMIC LAWS

SOLUTION
ANSWER 1
I ANSWERS TO OBJECTIVE TYPE QUESTIONS

1. (c) [Hints: Refer Regulation 2 of Schedule III of Foreign Exchange Management (Current
Account Transactions) Rules, 2000]
2. (c) [Hint: Refer Section 2 (9) of the Prohibition of Benami Property Transactions Act, 1988.
It is not a benami transaction because all statutory dues have been paid and his father
knew about the transaction. Therefore, it falls under exempted category. The amount so
contributed can be a loan or gift to the son.]
3. (c) [Hint: Refer Regulation 4 of the Foreign Exchange Management (Export of Goods and
Services) Regulations, 2015]
4. (c) [Hint: Refer Regulation 15 of the FEM (Export of Goods and Services), Regulations, 2000]
5. (a) [Hint: Refer Regulation 3 of FEM (Acquisition and transfer of immovable property in India)
Regulations, 2000]
6. (d) [Hint: Refer Regulation 7 of FEM (Realisation, Repatriation and surrender of Foreign
Exchange) Regulations, 2015]
7. (b) [Hint Refer Regulation 4 the FEM (Export of Goods and Services), Regulations, 2015
which has exempted such export transaction from furnishing of export declaration]
8. (b) [Hint: Section 14 of the Insolvency and Bankruptcy Code, 2016, describes moratorium. It
is an order passed by the adjudicating authority (NCLT) declaring a moratorium on the
debtor's operations for the period of the Insolvency Resolution Process, during which no
action can be taken against the Company or the assets of the Company. This operates
as a 'calm period' during which no judicial proceedings for recovery, enforcement of
security interest, sale or transfer of assets, or termination of essential contracts can take
place against the debtor.]
9. (b) [Hint: FEMA does not impose any restriction on acquisition of immovable property outside
India by a non-resident Indian. Further, when at a future date the person concerned
becomes a person resident in India, Section 6(4) even permits him to hold, own or transfer
immovable property situated outside India since such property was acquired by him when
he was resident outside India]
10. (c) [Hint: As per section 22 of the Insolvency and Bankruptcy Code, 2016, an Interim
Resolution professional approved by the Committee of Creditors can be replaced with
75% voting in favour of the decision and approval of the Board.
II. ANSWERS TO DESCRIPTIVE QUESTIONS
1. (i) Section 11(3) of the Foreign Exchange Management Act, 1999 states that where any
Authorised person contravenes any direction given by the Reserve Bank of India under the
said Act or fails to file any return as directed by the Reserve Bank of India, the Reserve Bank

© The Institute of Chartered Accountants of India


t.me/cafinal6d - telegram link
Page 459
of India may, after giving reasonable opportunity of being heard, impose on Authorised
Person, a penalty which may extend to ten thousand rupees and in the case of continuing
contraventions with an additional penalty which may extend to two thousand rupees for every
day during which such contravention continues.
Since as per the facts given in the question, the Authorised person, namely, Forex Dealers
Ltd., has failed to file the returns as directed by the Reserve Bank of India. According to the
above provisions, it has exposed itself to a penalty which may extend to ten thousand rupees
and in the case of continuing contraventions in the nature of failure to file the returns, with an
additional penalty which may extend to two thousand rupees for every day during which such
contravention continues.
(ii) According to the provisions of section 2(v) of the Foreign Exchange Management Act, 1999,
a person in order to qualify for the purpose of being treated as a "Person Resident in India"
in any financial year, must reside in India for a period of more than 182 days during the
preceding financial year. In the given case, Mr. Shekhar has resided in India for a period of
only 150 days, i.e., less than 182 days, during the financial year 2016-2017. Hence, he cannot
be considered as a "Person Resident in India" during the financial year 2017-2018 irrespective
of the purpose or duration of his stay.
(iii) Printex Computer being a Singapore based company would be person resident outside India
[(Section 2(w)]. Section 2 (u) defines ‘person’ under clause (vii) thereof, as person would
include any agency, office or branch owned or controlled by such person. The term such
person appears to refer to a person who is included in clause (i) to (vii). Accordingly printex
unit in Pune, being a branch of a company would be a ‘person’.
Section 2(v) defines a person resident in India. Under clause (iii) thereof person resident in
India would include an office, branch or agency in India owned or controlled by a person
resident outside India. Printex unit in Pune is owned or controlled by a person resident outside
India, and hence it, would be a ‘person resident in India.’
However, Dubai Branch though not owned is controlled by Printex unit in Pune which is a
person resident in India. Hence prima facie, it may be possible to hold a view that the Dubai
Branch is a person resident in India.
2. Capital Account Transactions: All the transactions referred to in the question are capital account
transactions.
Section 6(2) of FEMA, 1999 provides that the Reserve Bank may in consultation with the Central
Government specify the permissible capital account transactions and the limit upto which foreign
exchange will be allowed for such transactions.
(i) Depreciation of direct investments: According to proviso to section 6(2), the Reserve bank
shall not impose any restriction on the drawal of foreign exchange for certain transactions.
One such transaction is drawal of foreign exchange for payment due for depreciation of direct
investment in the ordinary course of business. Hence this transaction is permissible without
any restrictions.
(ii) Person resident in USA returning permanently to India: When the person returns to India
permanently, he becomes a resident in India. Section 6(4) provides that a person resident in
India may hold, own, transfer or invest in foreign currency, foreign security, etc. if such
currency, security or property was acquired, held or owned by such person when he was
resident outside India or inherited from a person who was resident outside India. In view of
this, the person who returned to India permanently can continue to hold the foreign security
acquired by him when he was resident in U.S.A.
(iii) Investment in shares of Indian company by non-resident: Reserve Bank issued Foreign
Exchange Management (Permissible Capital Account Transactions) Regulations, 2000.
Regulation 4(6) of the said Regulations prohibits a person resident outside India from making

© The Institute of Chartered Accountants of India


t.me/cafinal6d - telegram link
Page 460
investment in India, in any form, in any Company or partnership firm or proprietary concern
or any entity, whether incorporated or not, which is engaged or proposes to engage in
construction of farm houses. Hence it is not possible for a person resident outside India to
invest in the shares of a company engaged in construction of farm houses as such investment
is prohibited.
(iv) Acquisition of immovable property by person resident outside India: Reserve Bank
issued Foreign Exchange Management (Permissible Capital Account Transactions)
Regulations, 2000. The regulations specify the classes of capital account transactions of
persons resident outside India in Schedule II. Under this schedule, acquisition and transfer of
immovable property in India by a person resident outside India is permissible. Hence, the
person resident of Canada can acquire the immovable property in Mumbai.
3. (i) Adherence to sanctioned plans and project specifications by the promoter (Section 14)
The proposed project shall be developed and completed by the promoter in accordance with
the sanctioned plans, layout plans and specifications as approved by the competent
authorities.
Notwithstanding anything contained in any law, contract or agreement, after the sanctioned
plans, layout plans and specifications and the nature of the fixtures, fittings, amenities and
common areas, of the apartment, plot or building, as the case may be, as approved by the
competent authority, are disclosed or furnished to the person who agree to take one or more
of the said apartment, plot or building, as the case may be, the promoter shall not make —
(1) any additions and alterations in the sanctioned plans, layout plans and specifications
and the nature of fixtures, fittings and amenities described therein in respect of the
apartment, plot or building, as the case may be, which are agreed to be taken, without
the previous consent of that person.
Provided that the promoter may make such minor additions or alterations as may be
required by the allottee, or such minor changes or alterations as may be necessary due
to architectural and structural reasons duly recommended and verified by an authorised
Architect or Engineer after proper declaration and intimation to the allottee.
Explanation.—For the purpose of this clause, "minor additions or alterations" excludes
structural change including an addition to the area or change in height, or the removal
of part of a building, or any change to the structure, such as the construction or removal
or cutting into of any wall or a part of a wall, partition, column, beam, joist, floor including
a mezzanine floor or other support, or a change to or closing of any required means of
access ingress or egress or a change to the fixtures or equipment, etc.
(2) any other alterations or additions in the sanctioned plans, layout plans and specifications
of the buildings or the common areas within the project without the previous written
consent of at least two-thirds of the allottees, other than the promoter, who have agreed
to take apartments in such building.
Explanation.—For the purpose of this clause, the allottees, irrespective of the number of
apartments or plots, as the case may be, booked by him or booked in the name of his
family, or in the case of other persons such as companies or firms or any association of
individuals, etc., by whatever name called, booked in its name or booked in the name of
its associated entities or related enterprises, shall be considered as one allottee only.
In case any structural defect or any other defect in workmanship, quality or provision of
services or any other obligations of the promoter as per the agreement for sale relating to
such development is brought to the notice of the promoter within a period of five years by the
allottee from the date of handing over possession, it shall be the duty of the promoter to rectify
such defects without further charge, within thirty days, and in the event of promoter's failure

© The Institute of Chartered Accountants of India


t.me/cafinal6d - telegram link
Page 461
to rectify such defects within such time, the aggrieved allottees shall be entitled to re ceive
appropriate compensation in the manner as provided under this Act.
Hence, in the instant case, reducing the number of rooms does not come under minor
additions or alterations. The promoter i.e. Mr. Pankaj Gupta shall not make any additions and
alterations in the sanctioned plans, layout plans and specifications within the project without
the previous written consent of at least two-thirds of the allottees, other than the promoter,
who have agreed to take apartments in such buildings.
(ii) No deposit or advance to be taken by promoter without first entering into agreement
for sale
According to section 13 of the said Act, a promoter shall not accept a sum more than ten per
cent of the cost of the apartment, plot, or building as the case may be, as an advance payment
or an application fee, from a person without first entering into a written agreement for sale
with such person and register the said agreement for sale, under any law for the time being
in force.
In the instant case, the cost of the flat is Rs. 2 crore and Mr. Shyam put a condition to pay
Rs. 50 Lakhs as an application fee before entering into a written agreement for sale with Mr.
Vivaan. This is invalid as a promoter can accept only Rs.20 Lakhs (10% of Rs. 2 Crore) as an
advance or an application fee without first entering into a written agreement for sale.
ANSWER 2
I. ANSWERS TO OBJECTIVE TYPE QUESTIONS
1. (d) [Hints: As per section 3(12), Default means non-payment of debt when whole or any part
or instalment of the amount of debt has become due and payable and is not repaid by the
debtor or the corporate debtor, as the case may be]
2. (c) [Hints: Financial creditor means any person to whom a financial debt is owed and includes
a person to whom such debt has been legally assigned or transferred to;{section 5(7)}]
3. (b) [Hint: Refer Regulation 15 of the FEM (Export of Goods and Services), Regulations, 2000]
4. (c) [Hint: Refer Section 5 (7) of the Code]
5. (d) [Hint: Refer Section 5 (21) of the Code]
6. (c) [Hint: Refer Section 16 of the Code]
7. (d) [Hint: Refer Section 15 of the Code]
8. (c) [Hint: Refer Section 22 (1)]
9. (d) [Hint: Refer Regulation 3 of the Insolvency and Bankruptcy (Insolvency Resolution
Process for Corporate Persons) Regulations, 2016]
10. (d) [Hint: section 61(3) of the IBC]

II ANSWERS TO OBJECTIVE TYPE QUESTIONS

1. (i) According to section 33 of the Insolvency and Bankruptcy Code, 2016, where the Adjudicating
Authority before the expiry of the insolvency resolution process period does not receive a
resolution plan as approved by the committee of creditors, it shall—
(a) pass an order requiring the corporate debtor to be liquidated as per the relevant
provisions
(b) issue a public announcement stating that the corporate debtor is in liquidation; and
(c ) require such order to be sent to the authority with which the corporate debtor is
registered.

© The Institute of Chartered Accountants of India


t.me/cafinal6d - telegram link
Page 462
According to section 12 of the Insolvency and Bankruptcy Code, 2016, the corp orate
insolvency resolution process (CIRP) shall be completed within a period of one hundred and
eighty days from the date of admission of the application to initiate such process.
As per the facts, Ronit, presented the approved resolution plan, before NCLT after the
prescribed period for the completion of CIRP i..e, after 180 days of insolvency commencement
date.
According to the above stated provisions, NCLT, shall pass an order requiring the corporate
debtor (MMPL) to be liquidated. It shall issue a public announcement of its liquidation and
send such order to the Registrar of companies.
(ii) As per Section 33(3) of the Insolvency and Bankruptcy Code, 2016, where the resolution plan
approved by the Adjudicating Authority is contravened by the concerned c orporate debtor,
any person other than the corporate debtor, whose interests are prejudicially affected by such
contravention, may make an application to the Adjudicating Authority for a liquidation order
as referred above. Accordingly, the employees and the stakeholders of MMPL, whose
interests are affected by contravention in compliances of the resolution plan, may make an
application to NCLT for initiation of liquidation. On receipt of an application, if the Adjudicating
Authority determines that the MMPL has contravened the provisions of the resolution plan, it
shall pass a liquidation order.
(iii) As per section 33(7) of the Insolvency and Bankruptcy Code, 2016, the order for liquidation
shall be deemed to be a notice of discharge to the officers, employees and workmen of the
corporate debtor.
However, where the business of the corporate debtor when continued during the liquidation
process by the liquidator, it shall not be deemed to be notice of discharge to the officers,
employees and workmen of the corporate debtor.
So the Conduct of business of MMPL during liquidation process by the liquidator is tenable
and shall not be deemed to be notice of discharge to the officers, employees and workmen of
the MMPL.
2. (i) Validity of the conduct of undervalued transaction : As per the provisions given in section
45 of the Insolvency and Bankruptcy Code, 2016, Ronit, on an examination of the
transactions of the MMPL, determines that certain transactions were made by MMPL with a
related party (Rama devi) within the period of two years preceding the insolvency
commencement date (in 6 months preceding the Insolvency Commencement date), which
were undervalued. Ronit, shall make an application to the NCLT to declare such transactions
as void and reverse the effect of such undervalued transaction and requiring the person who
benefits from such transaction to pay back any gains he may have made as a result of such
transaction.
(ii) Failure to report to NCLT of undervalued transactions: As per the stated facts given in
the light of the provisions laid in Section 47 of the Insolvency and Bankruptcy Code, an
undervalued transaction has taken place and Ronit (Resolution Professional) has not reported
it to the NCLT, in such case, a creditor , member or a partner of a MMPL, as the case may
be, may make an application to the NCLT to declare such transactions void and reverse their
effect in accordance with the relevant provisions of this Code.
(iii) Order of NCLT: Where the NCLT, after examination of the application made above, is
satisfied that undervalued transactions had occurred; and Ronit (RP) after having sufficient
information or opportunity to avail information of such transactions did not report such
transaction, there it shall pass an order of —
(a) restoring the position as it existed before such transactions and reversing the effects
thereof in the manner as laid down in section 45 and section 48 of the Code. The order
of the Adjudicating Authority may provide for the following:—

© The Institute of Chartered Accountants of India


t.me/cafinal6d - telegram link
Page 463
(1) require any property transferred as part of the transaction, to be vested in the
corporate debtor(MMPL);
(2) release or discharge (in whole or in part) any security interest granted by the
corporate debtor (MMPL);
(3) require any person to pay such sums, in respect of benefits received by such
person, to the Ronit (RP), as the Adjudicating Authority may direct; or
(4) require the payment of such consideration for the transaction as may be determined
by an independent expert.
(b) requiring the Board(IBBI) to initiate disciplinary proceedings against Ronit.
3. (a) (i) As per the provisions given in section 68 of the Code, Mr. Shyam, Director (an officer in
default) has within the twelve months immediately preceding the insolvency
commencement date, fraudulently transferred his holding of shares in favour of his sister
of Rs.1 lakh (which is more than value of ten thousand rupees ). So, Mr. shyam, shall
be punishable with imprisonment for a term which shall not be less than three years but
which may extend to five years, or with fine, which shall not be less than one lakh rupees,
but may extend to one crore rupees, or with both: However, he shall not be liable to any
punishment under this section if he proves that he had no intent to defraud or to conceal
the state of affairs of the corporate debtor.
(ii) According to section 71 of the Code, on and after the insolvency commencement date,
Mr. Shyam, makes a false entry in the books of account of MMPL with an intent to
defraud or deceive any person, he shall be punishable with imprisonment for a term
which shall not be less than three years, but which may extend to five years, or with fine
which shall not be less than one lakh rupees, but may extend to one crore rupees, or
with both.
(iii) As per Section 77 of the Code, as Shyam permitted Shivam to provide informations in
the application under section 10,which is false in material particular and omits material
fact related to a books of accounts of a specified period, so he shall be punishable
with imprisonment for a term which shall not be less than three years, but which
may extend to five years or with fine which shall not be less than one lakh rupees,
but which may extend to one crore rupees, or with both.
(b) As per section 69 of the Code, on or after the insolvency commencement date, where the
directors of the MMPL—
(a) has made transfer of, or charge on, or has caused or connived in the execution of a
decree or order against, the property of the corporate debtor;
(b) has concealed or removed any part of the property of the corporate debtor within two
months before the date of any unsatisfied judgment, decree or order for payment of
money obtained against the corporate debtor,
such directors of MMPL, shall be punishable with imprisonment for a term which shall not be
less than one year, but which may extend to five years, or with fine, which shall not be less
than one lakh rupees, but may extend to one crore rupees, or with both.
However, directors of MMPL, shall not be punishable under this section if the acts mentioned
in clause (a) were committed more than five years before the insolvency commencement date;
or if he proves that, at the time of commission of those acts, he had no intent to defraud the
creditors of the corporate debtor.

© The Institute of Chartered Accountants of India


t.me/cafinal6d - telegram link
Page 464
ANSWER 3
I. ANSWERS TO OBJECTIVE TYPE QUESTIONS
1. (a) [Hint: The matter relates to concerns of an individual consumer regarding non-delivery of
booked vehicle in the given time]
2. (b) [Hint: The allegations essentially relate to abuse of dominance by a car manufacturing
company, directly or through its authorized dealer]
3. (c) [Hint: The tried to portray his issue as an issue of consumer exploitation to draw the attention
of the competition authority]
4. c, d, a, b [Hint: To examine a case under section 4 of the Competition Act, 2002, it is to be seen
first whether the alleged entity is an enterprise or not before defining the relevant market,
assessment of its position of dominance in the relevant market and examination of its conduct]
5. b, a, d, c [Hint: To examine a case under section 3 (4) of the Competition Act, 2002, first it is to be
seen whether the alleged two entities are in a vertical chain and whether they have entered into
any agreement as defined under the Competition Act, 2002. Then it is to seen whether such
agreement is anti-competitive and it has appreciable adverse effect on competition]
6. (a) [Hint: Essentially, the allegations relate to passenger car market]
7. (d) [Hint: All are the factors prescribed under section 19 (6) the Competition Act, 2002 to assess
dominance of an enterprise in a relevant market]
8. (b) [Hint: Cartelization requires agreement amongst players placed at horizontal level]
9. (a) [Hint: No competition concerns raised in the matter as delay in giving delivery to a consumer
or not passing the benefit of tax reduction to consumer or increasing the price cannot said to
be anti-competitive]
10. (d) [ Hint: Refer section 4 of the Competition Act, 2002]
II. ANSWERS TO DESCRIPTIVE QUESTIONS
1. Even though the concerns raised by Mr. Nazir cannot be redressed by the competition authority
as it essentially relates to grievances of an individual consumer of a passenger car manufactured
by XMC Pvt. Ltd, however if the matter is placed before the competition authority it will be examined
in terms of section 4 of the Competition Act, 2002. It is so because the allegations of Mr. Nazir
essentially relate to abuse of dominance by XMC Pvt. Ltd, directly or through its authorized dealer
M/s Ratan Lal & Sons.
To examine the matter under section 4 of the Competition Act, 2002, it is to be seen first whether
the alleged entity is an enterprise or not before defining the relevant market, assessment of its
position of dominance in the relevant market and examination of its conduct.
Enterprise: Yes, XMC Pvt. Ltd. is an enterprise in terms of Section 2 (h) of the Act.
Relevant Product Market: The market for passenger car [section 2 (t)]
Relevant Geographic Market: whole of India [see section 2 (s)]
Relevant Market: the market for passenger car in India [section 2 (r)]
Assessment of Dominance of XMC Pvt. Ltd.: Appear to be dominant in the market for
passenger car in India as it has highest market share and financial strength besides its brand name
and dependence of the consumer on it.
Assessment of the alleged conduct of XMC Pvt. Ltd.: Not appear to be abusive. Delay in giving
delivery of a product to a consumer or not passing the benefit of tax reduction to consumer or
increasing the price cannot said to be anti-competitive in terms of section 4 of the Competition Act,
2002.

© The Institute of Chartered Accountants of India


t.me/cafinal6d - telegram link
Page 465
2. To examine the matter under section 4 of the Competition Act, 2002, it is to be seen first whether
the alleged entity is an enterprise or not before defining the relevant market, assessment of its
position of dominance in the relevant market and examination of its conduct. (all the steps of
answer no. 1 above to be followed)
Delay in giving delivery of a product to a consumer without enabling provisions in the booing
document may be an issue of breach of contract between two parties. It may not be a case of
imposition of unfair condition in term of the provisions of section 4 of the Competition Act, 2002.
Further, increasing price is a commercial decision of an enterprise which is taken considering the
market demand conditions of the product. If market is competitive then excess price, if any, can be
wiped out in the long run, no intervention of the competition authority is required. However, if the
company raised the price after negotiation with the consumer, it can be challenged in other
appropriate forum.
3. As per section 2(r) of the Act, ‘relevant market’ means the market which may be determined by the
Commission with reference to the relevant product market or the relevant geographic market or
with reference to both the markets. Further, the term ‘relevant product market’ has been defined in
section 2(t) of the Act as a market comprising all those products or services which are regarded as
interchangeable or substitutable by the consumer, by reason of characteristics of the products or
services, their prices and intended use. And, the term ‘relevant geographic market’ has been
defined in section 2(s) of the Act to mean a market comprising the area in which the conditions of
competition for supply of goods or provision of services or demand of goods or services are
distinctly homogenous and can be distinguished from the conditions prevailing in the neighbouring
areas.
In order to determine the ‘relevant product market’, the Commission, in terms of the factors
contained in section 19(7) of the Act, is required to have due regard to all or any of the following
factors viz. physical characteristics or end- use of goods, price of goods or service, consumer
preferences, exclusion of in-house production, existence of specialized producers and
classification of industrial products. Similarly in order to determine the ‘relevant geographic market’,
the Commission, in terms of the factors contained in section 19(6) of the Act, is required to have
due regard to all or any of the following factors viz., regulatory trade barriers, local specification
requirements, national procurement policies, adequate distribution facilities, transport costs,
language, consumer preferences and need for secure or regular supplies or rapid after - sales
services.
As stated above, as per the provisions of the Competition Act, 2002 the relevant market comprises
of the relevant product market and relevant geographic market. In the instant matter, the relevant
product market may be considered as the ‘market for passenger car’. It may be noted that the
allegations of Mr. Nazir pertains to purchase and after sale service of a passenger car which cannot
be substitutable with other type of vehicle in terms of price, end use, characteristics, etc. The
relevant geographic market in this matter may be considered as ‘India’ because the condition of
competition in passenger car market in India is homogeneous throughout India. A consumer c an
buy a passenger car from any part of India with similar competitive condition. Thus, the market for
passenger car in India may be considered as the relevant market in this case.

© The Institute of Chartered Accountants of India


t.me/cafinal6d - telegram link
Page 466
vol 24 NO. 12 pages 36 May 2021 50

Page 467
Contents
Board of Studies
(Academic) (2021-22)
INSIDE
Chairman 03 President’s Communication
CA. Jay Chhaira
Vice-Chairman
CA. Ranjeet Kumar Agarwal 04 Vice-President’s Communication
Members
CA. Nihar N Jambusaria, President (Ex-officio)
CA. (Dr.) Debashis Mitra, Vice-President (Ex-officio) 05 Chairman’s Communication
CA. Anil S Bhandari
CA. Tarun J Ghia
CA. Nandkishore C Hegde
06 CA Final: Economic Laws
CA. Chandrashekhar V Chitale
CA. Shriniwas Y Joshi
CA. Durgesh Kumar Kabra
CA. Aniket S Talati 18 CA Intermediate: Enterprise Information Systems
CA. Babu Abraham Kallivayalil
CA. Prasanna Kumar D
CA. G Sekar 32 CA Foundation: Business and Commercial Knowledge
CA. M P Vijay Kumar
CA. Sushil Kumar Goyal
CA. Pramod Kumar Boob
34 CA Examination Toppers + News Updates
CA. Anuj Goyal
CA. Kemisha Soni
CA. Satish Kumar Gupta
CA. Hans Raj Chugh 36 Crossword
CA. Atul Kumar Gupta
CA. Rajesh Sharma
CA. Charanjot Singh Nanda President and Editor-in-Chief Chairman and Editor
Adv. Vijay Kumar Jhalani CA. Nihar N Jambusaria, Mumbai CA. Jay Chhaira
Co-opted Members
CA. (Dr.) G. S. Grewal Vice President Vice-Chairman
CA. Prashant Shekhar Panda CA. (Dr.) Debashis Mitra, Kolkata CA. Ranjeet Kumar Agarwal, Kolkata
CA. Dhanajay R. Barve
CA. Atul Bheda
CA. Ajay Alipuria Director- Board of Studies (Academic) Editorial Support
CA. Girish Samant CA. Vandana D Nagpal Dr. Ruchi Agarwal, Assistant Secretary
Ms. Taruna Kohli, Assistant Secretary
CA. R. N. Singh
Office
CA. Manish Dafria
Board of Studies, The Institute of Chartered, Accountants of India, ICAI Bhawan,
Special Invitee A-29, Sector-62, Noida-201 309. Phone : 0120-3045907
CA. Harshwardhan Sudhir Patil
Head Office
The Institute of Chartered Accountants of India, ICAI Bhawan, Indraprastha Marg,
Total Circulation: 2,54,795 New Delhi-110 104.
Check your Address: All students should
check their mailing address printed on back Editor: CA. Jay Chhaira
cover. In case, there is any change or the PIN Printed and published by CA. Vandana D Nagpal, on behalf of The Institute of
Code (Postal Index Code) is either missing Chartered Accountants of India, New Delhi.
or is incorrect, kindly inform immediately Published at the Institute’s Office at Indraprastha Marg, New Delhi and
the concerned Regional Office, giving full printed at Spenta Multimedia Pvt. Ltd., Plot 15,16 & 21/1, Village Chikhloli,
particulars of your address along with correct Morivali, MIDC, Ambernath (West), Dist. Thane
PIN Code. This would enable us to ensure
regular and prompt delivery of the Journal. The views and opinions expressed or implied in THE CHARTERED ACCOUNTANT
STUDENT are those of the authors and do not necessarily reflect those of ICAI. Unsolicited
Correspondence with regard to advertising and articles and transparencies are sent at the owner’s risk and the publisher accepts no
writing articles liability for loss or damage. Material in this publication may not be reproduced, whether in
Email: writesj@icai.in part or in whole, without the consent of ICAI.
Non-receipt of Students’ Journal DISCLAIMER: The ICAI is not in any way responsible for the result of any action taken on the basis of
Email: nosj@icai.in the advertisement published in the Journal.

Cover Image Courtesy: www.shutterstock.com


Inside image: www.shutterstock.com SWACHH BHARAT - A STEP TOWARDS CLEANLINESS

02 May 2021 The Chartered Accountant Student

Page 468
PRESIDENT’S COMMUNICATION

education and profession. The committee will identify the issues,


elicit opinions/suggestions from the stakeholders, critically
analyse the feedback and table its recommendations.
Subject capsules: Resource for revision
This issue features comprehensive capsules comprising Business
and Commercial Knowledge in Foundation, Enterprise
Information Systems in Intermediate and Economic Laws in
Final. These capsules present vital concepts succinctly through
Dear Students, vivid visuals and illustrations. I urge you to refer to these for
quick revision.

A
t the outset, I would like to congratulate all who have
recently qualified the CA Intermediate examination held
Campus placements
in January 2021. As you enter the final phase of your It is indeed heartening to note that despite the challenges posed
academic journey i.e. CA Final and Articleship commencement, by the pandemic, our placement initiative received an excellent
you must further gear up and accelerate your efforts. This is the response from the industry. With a total of 93 organisations
stepping stone that prepares you for the giant leap from being a participating in the ongoing Campus Placement Programme
discerning student to a conscientious professional. organised across the country at nine major centres and ten
smaller centres, more than 5000 jobs have already been offered
BoS: Partner in your holistic development and till now. It is a matter of pride for all of us that our esteemed
progress Institute has become a popular destination amongst reputed
The ICAI through its BoS is truly dedicated towards your welfare organisations at the national as well as international level who are
and well-being. We are assiduously endeavouring to facilitate a looking for confident and competent entry-level professionals in
whole gamut of learning resources and initiatives to ensure that our various profiles. You would be pleased to know that even during
students are able to constantly learn and augment their knowledge the pandemic time, the highest salary offered for domestic and
as well as enrich their skills simultaneously. Whether it is knowledge international postings were Rs. 25 lacs and Rs. 32.7 lacs per
creation and value addition with regard to providing periodically annum respectively. This stands testimony to the fact that our
updated publications or harnessing contemporary technology to young professionals are highly sought after and valued by the
reach out to our students in every nook and corner of the nation to industry.
knowledge delivery at the doorstep in the form of complimentary As you know, Covid-19 cases are on the rise again, and it is
Virtual Coaching Classes (VCC), the BoS has delivered it all. high time that you all must stay indoors and take all necessary
VCC, in particular, is aimed at providing the much-needed precautions. While going out, strictly adhere to Covid protocol
constant support and hand-holding during the present times such as wearing of masks, wearing of sanitisers and observing
as physical classroom teaching is out of bounds. I am confident social distancing.
that you all must have made the best use of most of the learning We, at ICAI, are closely monitoring the situation with regard
resources/publications and genuinely benefitted from the classes. to the ongoing pandemic. I assure you that we shall always place
Most of you must have also attempted both the series of the Mock your best interest first and BoS will be coming up with certain
Test Papers to assess your learning and strengthening your basic announcements shortly. You must closely follow the website in
concepts. the coming days.
BoS is currently in the process of getting a Mobile Application Stay positive, motivated and focused. Your strength lies
developed which will serve as a one stop destination to provide in your efforts. The more you work hard, the stronger will be
complete information regarding your status as a student, learner the determination to succeed, further motivating you to work
and article. This will also serve as a repository of knowledge, storing harder to achieve your full potential. Stern discipline, tenacious
recorded lectures, guidance notes, E-Books et al and apprise you perseverance and relentless persistence will lead you to the path
about the current and upcoming BoS events and initiatives. to success. So, tread ahead with confidence without losing sight
Our commitment: Strengthening accounting of your goal.
education and training Remember- The strongest factor to success is self-belief.
As a premier Accounting Body, the ICAI persistently strives Believing you can do it, believing you deserve it and believing
to strengthen the system of accounting education and training you will get it! So etch your own success story on the rock of
with an objective to equip our professionals with competence your life!
and capacity to cater to the changing paradigms in industry Best Wishes!
and practice. In pursuance of our commitment, Committee for
Review of Education and Training (CRET) has been constituted
to review the existing scheme and system of education and
training at ICAI in order to determine its adequacy/efficacy
vis-à-vis tenets of the National Education Policy 2020 as well as CA. Nihar N Jambusaria
emerging business and technological trends affecting accounting PRESIDENT, ICAI

The Chartered Accountant Student May 2021 03


"It's never too late to be what you might've
Page 469 been.” – George Eliot
VICE PRESIDENT’S COMMUNICATION

(VCC) from July 2020 targeting ensuing CA examinations


for the students of Foundation, Intermediate and Final. Many
students have benefited from these classes.
 Mock Test Papers
In view of the prevailing COVID-19 pandemic and the ongoing
lock-down at specific places, Mock Tests papers Series –I & II
were conducted in physical/virtual mode(s) from March 24,
2021 onwards. For students who have attempted the Mock
My Dear Students, Test virtually, the Question Papers for each subject were

W
uploaded at BoS Knowledge Portal on www.icai.org
e are going through an unprecedented global  Revision Test Papers
predicament today. The entire world has been Revision Test Papers have been uploaded for all levels for
adversely affected by the coronavirus pandemic May/June 2021 examinations on the BoS Knowledge Portal
whereby our economy, our families, community and available on CDS as well.
and our entire way of life has undergone a metamorphosis. In  Computer Based Test for Assessment of Practical Training
these difficult times, Information Technology (IT) plays a crucial In order to create a seriousness amongst the students for
role at both the interfaces, i.e., receiving the latest and relevant pursuing their 3 years practical training and also to ensure
information and disseminating to the end users. The Institute has that the training is imparted in all important areas required
also been leveraging technology to serve the students in a better for a student to learn, ICAI conducted online MCQ-based
manner. I would advise you all to visit the Institute’s website tests from September, 2018 onwards for students who have
regularly for all the latest information pertaining to your studies. completed first and second year of practical training. These
The same shall enable you to be well abreast of the newest and tests help our students to assess the knowledge gained during
contemporary amendments, notifications, announcements etc. practical training.
relevant to your respective examination attempts.
Key to success: Confidence, hard work and
Optimum utilisation of BoS Knowledge Portal commitment
Board of Studies, the academic wing of ICAI, is fully aware of its If success is easy, everyone would have it. Every successful person
prime responsibility to provide the quality professional education had put in enough efforts and hard work before they attained the
to you. The Study Material and other educational inputs provided success. So the hard work, together with the time it takes, is a
to you are being meticulously prepared with an aim to provide must on your journey to success. The harder you work on your
you the required background enabling you building a strong dream, the more confident you become.
conceptual base. Therefore, it is of utmost importance that you
read the Study Materials and understand the concepts thoroughly. “Confidence and hard work is the best medicine to kill the disease
The concepts have been explained with elucidating illustrations, called failure. It will make you a successful person.”
explanatory examples and numerous exercises for practice.
-A P J Abdul Kalam
Let me enumerate some initiatives taken by BoS for your benefit
and your development. Success and accomplishment are not limited to those who are
lucky. They are available to those who are disciplined, focussed,
 Chartered Accountancy Qualification Equivalent to Post committed, positive and enthusiastic. I hope that you have started
Graduate Degree doing revision of your respective courses. Be regular, disciplined
In a major decision, University Grant Commission (UGC) has and resolute towards your set objectives. Do not develop the
granted the academic equivalence to Chartered Accountancy habit of procrastination, it can dwindle your professional growth.
qualification with the Post Graduate degree. As a result, the My advice is, “never do tomorrow what you can do today.
Chartered Accountant members can now pursue Ph.D. in all Procrastination is the thief of time.” With this note, I would like
the Indian universities as against 107 universities, 2 IITs and to conclude and wish you best of luck for your future endeavours.
7 IIMs which presently recognise the Chartered Accountancy
qualification for the specific purpose of registration to Ph.D. Please, do take good care of yourselves and your families.
Programme. Stay safe, stay healthy.
The Grant of equivalence of Chartered Accountancy course May the blessings and grace of the Almighty always remain with
with the Post Graduate degree will make the Chartered each one of you and your family.
Accountants eligible to appear in National Eligibility Test
(NET) conducted by University Grants Commission. It
will make the Chartered Accountants eligible for award of
Junior Research Fellowship/ Assistant Professorship in the
Universities across India.
 Live Coaching Classes CA. (DR.) DEBASHIS MITRA
Board of Studies commenced free Virtual Coaching Classes VICE PRESIDENT, ICAI

04 May 2021 The Chartered Accountant Student

“The secret of getting ahead is getting


Page 470started.” – Mark Twain
CHAIRMAN’S COMMUNICATION

My Dear Students,

I
convey my heartiest congratulations to all of you who recently Build your confidence
qualified the CA Intermediate examination held in January Confidence is built on accomplishments, however small or
2021. Even as you rejoice, plan ahead for the next rung. You big. Every time you overcome a challenging task that seems
must realise that you need to step up your efforts as you insurmountable or beyond your current capacity, such as solving
register for CA Final and simultaneously commence your practical a difficult problem, taking a difficult decision or even planning a
training. complex task, it builds your confidence. Every time you achieve
As I write this communication, many parts of the country something as big as a significant milestone or even good remarks/
are grappling with steep rise in COVID cases, making the marks, as a result of your hard work, it builds your confidence. It is
situation quite grim. I extend my best wishes to the entire student a continuous exercise that you keep performing until it becomes
community and urge you to follow the administrative guidelines your character trait. Being confident makes you confront
intently to keep yourself safe and sound. At the same time you challenges upfront, handle critical situations with ease and above
must ensure that you use your time productively as most of you all, deal with uncertainties in your life.
are homebound. It is important for you to maintain a steady pace As students, you must realise that the most essential virtues
in studies so that you can build on the hard work done earlier and or facets you need to inculcate are resolute determination,
reap benefits in the times to come. earnest diligence and stern discipline. Following a regimented
BoS: Count on us! approach persistently and mindfully with utmost conviction and
dedication and being able to accomplish your daily goals/targets
We at BoS are mindful of your predicaments as you have been that you envisaged, over a period of time gradually builds your
conducting your studies, largely on your own with little or no confidence. Remember- Your confidence builds your character
physical interaction, with either teachers or your peer group. and your life!
We assure you to stand by you, even as we strive to devise novel
modes and methods to create opportunities for more interactions Stretch your limits
with the faculty, subject experts and your peers. We shall leave Being students of this extremely coveted and highly dynamic
no stone unturned to ensure that you stay ahead of the learning profession, you must prepare for a fulfilling and exciting career
curve to register constant progress on the study front. The BoS is ahead. Do not limit yourself to your comfort zone, instead
consistently working towards your best interest to organise: constantly challenge yourself with complex problems that exhort
• Revision/Recap Sessions: In order to help you to recapitulate you to delve deeper into the subject and explore answers after
the vital details and significant topics in all subjects pertaining to considerable research. Condition your mind to surpass your limits
CA Intermediate and Final, putting everything in perspective with by constantly motivating yourself with positive reinforcements.
respect to examinations. These sessions will encompass every This will help to overcome your fears, inspiring you to give your
aspect extensively with comprehensive discussions including: best to the relatively weaker subjects thus building your capacity
to do more to accomplish more.
o Syllabus/skill-wise/section-wise weightage;
o Pattern of past year questions with topic and mark wise Capsule contents
breakup; As part of the permanent feature, this issue presents integrated
o Major concepts from each chapter/topic in brief with subject capsules for Foundation based on Business and Commer-
specific reference to questions from RTPs/ MTPs/previous cial Knowledge, Intermediate on Enterprise Information
year questions; Systems and Final on Economic Laws. These present vital
o Case study and scenario-based questions; contents vividly. You may refer to these for revision purposes.
o Common errors committed by students; Remember: You are braver than you believe, stronger than
o Utilising BoS publications for revision; you seem and smarter than you think. Push harder than
o Dos and Don'ts vis-à-vis examination. yesterday for a better tomorrow!
• Virtual Coaching Classes Batch-3: Continuing the All the Best!
complimentary/free of cost learning facility for all levels,
enabling you to grasp the entire syllabus and engage you
actively while you stay put indoors. Lectures are rendered by
subject experts with a provision to view them live or recordings
on Youtube anytime.
Announcements and schedules will be notified on the website CA. Jay Chhaira
shortly. CHAIRMAN, BOARD OF STUDIES (ACADEMIC)

The Chartered Accountant Student May 2021 05


“Trust yourself that you can do it and
Page 471get it.” – Baz Luhrmann
Economic Laws
CA FINAL - PAPER 6D - ECONOMIC LAWS
This capsule on Paper 6D: Economic Laws, Final (New) course is another step of the Board of Studies in its endeavour
to provide quality academic inputs to Final course students of Chartered Accountancy course. As students are aware
that this is an open book examination and the duration is 4 hours. The question paper would comprise of five case
studies of 25 marks each, out of which the student would be required to attempt any four. Students must divide
their four hours between four case studies to be answered meticulously. Once the case studies have been opted, give
them a comprehensively reading while attempting the same. Some of the illustrative case studies have been provided
below for practice purpose. Students are suggested to solve the same in examination condition and check for the
answers only after attempting the case studies.

Case Study 1
Ms. Drishel Patel is a young dynamic IT professional and currently An understanding has been reached among the manufacturers
resides in America. She holds the NRI status. Ms. Drishel works of cement to control the price and supply of cement, but the
for Blip LLC, which has a wholly owned subsidiary Blip India understanding is not in writing and it is also not intended to be
Private Limited (here-in-after referred to as Blip). Blip deals in enforced by legal proceedings.
the mobile operating system. Blips’ operating system ‘Diordna’ is Rock Solid Private Limited (RSPL) is the substantial supplier
widely popular among the mobile phone manufacturers in India. of clay, slate, blast furnace slag, silica sand which are essential
Blip also offers proprietary applications and services (such as Blip raw materials of cement, and a shortage of same observed in
Maps, Blip Internet Explorer, and Blip Tube, etc.). Blips Mobile the market. Mr. Arya on behalf of SCPL has executed a supply
Services (BMS) is a bundled suite of Blips’ applications and agreement with RSPL on 20th October 2020 wherein it is provided
services and such apps and services are not available in isolation. that RSPL will not supply these raw materials to any other cement
In trade parlance, the mobile OS is different from OS designed manufacturer, against this the purchase commitment has been
for desktop as they have additional handheld use features. 80% made from SCPL for all their (RSPL) output at price mentioned
of mobile phone, which are in use has Diordna as an operating in such agreement.
system. Solid Cement Limited (SCL) who is another cement
If a mobile manufacturer wants to manufacture a ‘bare’ manufacturer is not happy with the RSPL, because RSPL has
Diordna mobile, it needs to only pass technical tests and accept not supplied the slate and silica power to SCL against the PO
the Diordna License Agreement; but in bare Diordna mobile (Purchase Order) placed by SCL dated 18th October 2020, hence
manufacturer are not permitted to include any of BMS such as board of directors of SCL is considering taking legal remedy
Blip Maps, Blip Internet Explorer, Blip Tube. If a manufacturer against RSPL in the capacity of the consumer. SCL has borne
wants to manufacture a mobile having Diordna with pre- loss on account of the stock-out situation emerged from the
installed BMS, he has to enter into two additional agreements non-availability of raw material. It was found that only half of
with Blip i.e. Mobile Application Distribution Agreement and the consideration was paid and 30 days credit was available for
Anti Fragmentation Agreement. BMS couldn’t be availed directly making payment of the remaining balance, regarding which
by the end-users, in case it is not pre-installed. payment promise is made by SCL.
Ms. Drishel got married to Mr. Joe Harris around a year back. Mr. Alok who is co-owner in SCPL with Mr. Arya, conducts
The marriage took place in a traditional saptapadi ceremony in the market study and concluded that the RMC (Readymix
the backyard of Harris’ residence where only close relatives were Concrete) segment has favourable opportunities because
present. Marriage was registered six months later due to a widely currently competition is relatively less in RMC and RMC
observed lockdown to prevent the widespread of COVID-19. based block segments. Moreover, RMC based block has wide
Indian traditions have a deep-rooted impact on Harris acceptance as an economical replacement of the brick-based
family because the grandmother of Joe is from India. Joe’s structure. Hence SCPL must diversify into the RMC segment.
grandfather is also influenced by Indian culture, hence willing Mr. Arya expresses his concerns over the availability of funds
to migrate to India along with Joe’s grandmother to spend the for the same. Mr. Anil the third member of SCPL, advices
rest of their life. Considering this in the month of January 2021, both the co-owners to float capital through the capital market.
Drishel and Joe acquired a luxurious apartment in joint name in After numerous rounds of discussions, SCPL decided to go for
India, so that Joe’s grandparent can stay there comfortably. Half public issue and listing of its equity shares, largely for business
of the consideration was paid by Ms. Drishel out of the Non- expansion, initially with setting up a new large scale RMC plant.
Resident Account maintained by her, and the remaining half by Mrs. Patel, the mother of Ms. Drishel, who also resides
Joe through proper banking channel, and that too in the manner with her daughter and son-in-law in States and holds NRI
prescribed. To identify the flat and fulfill the legal requirement status, acquired two immovable properties (one farmhouse for
for registration of the same, Ms. Drishel took the help of her elder residential purposes and another an agricultural land, because
cousin Mr. Arya Patel, who is permanently residing in India. she studied botany during her master and willing to develop
Mr. Arya along with two of his friends owns a cement botanical garden there) in their native place situated near to
manufacturing company in India called ‘Strong Cement Private Rajkot district of Gujarat in India in the year 2020-2021 for total
Limited’ (SCPL). The SCPL supplies cement to various builders consideration equivalent to USD 470,000. She made payment for
and retail consumers through a network of stockist and retailers. the same out of her non-resident account.
06 May 2021 The Chartered Accountant Student

Page 472
Economic Laws
Multiple Choice Questions per the provisions of the Foreign Exchange Management
1. Whether the understanding reached among the Act and relevant regulations made thereunder?
manufacturers of cement be termed as an agreement (iii) Can Joe acquire another property which is agricultural
(a) No, because it is not in writing land, in joint ownership with Drishel for investment
(b) No, because it is not intended to be enforced by legal purposes?
proceedings
(c) No, because it is not in writing and also not intended to Answer to MCQs
be enforced by legal proceedings 1. (d): Reason - As per section 2(b) of the Competition Act, 2002
(d) Yes ‘agreement’ includes any arrangement or understanding or
action in concert whether or not, is formal or in writing, or is
2. The agreement is executed among SCPL and RSPL on 20th intended to be enforceable by legal proceedings.
October 2020, can be categorised as In view of the above definition, an understanding reached
(a) Exclusive supply agreement among the cement manufacturers to control the price and
(b) Tie-in arrangement supply of cement will be an ‘agreement’ even though the
(c) Refuse to deal agreement understanding is not in writing and not intended to be
(d) None of these enforceable by legal proceedings.

3. Can SCL assume the position of the consumer for the 2.(c) : Explanation to sub-section 4 of section 3 of the Competition
purpose of competition laws? Act, 2002 describe five prohibited vertical agreements, and
(a) No, because only half of the consideration paid by SCL here relevant among those are;
(b) No, because SCL is not buying slate and silica sand for Tie in arrangement includes any agreement, requiring a
personal use or direct resale purchaser of goods, as a condition of such purchase, to
(c) No, because only an individual can be a consumer purchase some other goods;
(d) Yes Exclusive supply agreement includes any agreement
restricting in any manner the purchaser in the course of his
4. Which of the following statements is correct regarding the trade from acquiring or otherwise dealing in any goods other
acquisition of immovable property in India by Mrs. Patel? than those of the seller or any other person.
(a) Mrs. Patel is not allowed to acquire any sort of immovable Refusal to deal includes any agreement, which restricts or
property in India is likely to restrict, by any method the persons or classes of
(b) Mrs. Patel is not allowed to acquire farmhouse and persons to whom goods are sold or from whom goods are
agricultural land in India bought.
(c) Mrs. Patel may acquire the farmhouse, but not
agricultural land in India 3.(d): The term ‘consumer’ is defined in section 2(f ) of the
(d) Mrs. Patel may acquire both the farmhouse and Competition Act, 2002. Consumer means any person who
agricultural land in India buys any goods for a consideration which has been paid
or promised or partly paid and partly promised, or under
5. SCPL decided to go for public issue and listing of its equity any system of deferred payment and includes any user of
shares, largely for business expansion, initially with setting such goods other than the person who buys such goods
up a new large scale RMC plant. In the context of shares, for consideration paid or promised or partly paid or partly
which one of the following statements is correct under the promised, or under any system of deferred payment when
Competition Act, 2002? such use is made with the approval of such person, whether
(a) Shares can’t be considered as “goods” because nothing such purchase of goods is for resale or for any commercial
has to do with manufacturing, processing or mining. purpose or for personal use.
(b) Shares shall be considered as “goods” only if fully paid-up.
(c) Shares shall be considered as “goods” after the application 4. (b): As per regulation 3 of Foreign Exchange Management
made for shares since application monies are paid for the (Acquisition and Transfer of Immovable Property in India)
acquisition of shares. Regulations, 2018, an NRI may acquire immovable property
(d) Shares shall be considered as “goods” after allotment. in India other than agricultural land/ farmhouse/ plantation
property subject to two conditions;
Descriptive Questions a. Consideration, if any, for transfer, shall be made out of
1. Decide, whether Blip has dominance and does it abused funds received in India through banking channels by
its dominant position? Support your decision with legal way of inward remittance from any place outside India
backing. or funds held in any non-resident account maintained
in accordance with the provisions of the Act, rules, or
2. In the light of the given facts, evaluate the following situations regulations framed thereunder.
in terms of the FEMA, 1999: b. Provided further that no payment for any transfer of
(i) Can Mr. Joe acquire immovable property in India, immovable property shall be made either by traveler’s
independently? cheque or by foreign currency notes or by any other mode
(ii) Is the acquisition of a flat by Drishel and Joe jointly, valid as other than those specifically permitted under this clause.

The Chartered Accountant Student May 2021 07

Page 473
Economic Laws
Since Mrs. Patel holds the status of NRI, hence not allowed to 2. As per regulation 6 of the Foreign Exchange Management
acquire farmhouse and agricultural land in India (Acquisition and Transfer of Immovable Property in India)
Regulations, 2018, a person resident outside India, not being
5.(d): Goods under section 2 (i) of the Competition, Act 2002 a Non-Resident Indian or an Overseas Citizen of India, who
means goods as defined in the Sale of Goods Act, 1930 and is a spouse of a Non-Resident Indian or an Overseas Citizen
includes, products manufactured, processed, or mined; of India may acquire one immovable property (other than
debentures, stocks, and shares after allotment; in relation agricultural land/ farmhouse/ plantation property), jointly
to goods supplied, distributed, or controlled in India, goods with his/ her NRI/ OCI spouse, subject to following conditions
imported into India. (1) The consideration for the transfer, shall be made out of
funds received in India through banking channels by
Answers to Descriptive Questions way of inward remittance from any place outside India
1. Facts in the given case are more or less similar to the case or funds held in any non-resident account maintained
(No. 39 of 2018, Competition Commission of India dated in accordance with the provisions of the Act and the
16.04.2019) of Umar Javeed and Google LLC, wherein legal regulations made by the Reserve Bank;
issue is about dominance and its abuse and also the act of (2) No payment for any transfer of immovable property
Google found in violation of Section 4(2) of the Competition shall be made either by travellers’ cheque or by foreign
Act, 2002. currency notes or by any other mode other than those
In the said case, CCI observed to form a prima facie specifically permitted under this clause;
view about the alleged abusive conduct, it would be first (3) The marriage has been registered and subsisted for a
appropriate to define the relevant market and to determine continuous period of not less than two years immediately
the dominance of accused enterprise therein if any. In the preceding the acquisition of such property;
present case, it is clearly mentioned that mobile OS due (4) The non-resident spouse is not otherwise prohibited
to additional handheld use features are different from OS from such acquisition.
designed for desktop hence all OS for other devices such as Following are the answers in the light of the stated
desktop or laptop shall be excluded from the relevant market. provisions:
Blip appears to be dominant in the relevant market as 80% (i) No, Mr. Joe (a person resident outside India, not
of mobile phones, which are in use have Diordna as the being a Non-Resident Indian or an Overseas Citizen
operating system. of India) can’t acquire immovable property in India,
The signing of the Mobile Application Distribution independently.
Agreement and Anti Fragmentation Agreement is a pre- (ii) No, the acquisition of a flat by Drishel and Joe, jointly
condition for mobile manufacturers to pre-install BMS is not aligned (hence legally invalid, and amount to
(while using Diordna as OS). Further, BMS is also a bundled violation) to the provisions of FEMA and relevant
suite of Blips’ applications and services. In this manner Blip regulations made thereunder, because marriage
reduced the ability of device manufacturers to develop viable has been registered and subsisted for a continuous
alternatives with selected applications and services out of the period of fewer than two years immediately
BMS suite, hence dis-incentivize them. Thereby restricting preceding the acquisition of such property.
technical development to the prejudice of consumers in (iii) No, Joe can’t acquire another property being
violation of Section 4 of the Competition Act, 2002. agricultural land in joint ownership with Drishel for
While reading Section 4 with Section 32 of the Competition investment purposes because;
Act, 2002, it is important to note that the conduct of Blip • The acquisition of agricultural land, farmhouse,
to tie or bundle applications and services is an attempt to and plantation property is specifically prohibited;
eliminate effective competition from the market. There exists and
an element of coercion as the mobile manufacturers are • The time since the marriage took place and
coerced to purchase the BMS suite altogether which results subsisted is less than two years; and
in consumer harm through a reduction in choice of products. • There is a maximum ceiling limit of owning one
property

Case Study 2
Rajeshwari Industries Limited (here-in-after referred to as RIL) it requires more funds. RIL took a term loan of R3.5 crore from
manufactures a wide range of electronic heaters under the brand National Bank (here-in-after referred to as bank). Since the newly
‘Glen’. Glen, which was a popular name among the retailers and developed products, fails to make much impact in the market,
customers till a few years back, has been losing the market share; hence RIL faces a financial crunch and not in a position to serve
the major reason for same is stiff competition from emerging the financial debt.
competitors who are offering a complete range of electronic A pandemic causes another jolt to the financial health of the
products and also offers free delivery at customers address. business, hence on 15th April, 2020 (the due date for payment
To sustain the market share RIL decided to expand the of instalment), RIL conveyed to the bank its inability to repay
product range and improve outbound logistic facilities for which the remaining outstanding loan amount. As of 15th April, 2020,

08 May 2021 The Chartered Accountant Student

Page 474
Economic Laws
the total outstanding amount against RIL is R46 lakh (including Multiple Choice Questions
interest). 1. Can the bank file the insolvency proceedings against RIL?
The officers from the recovery cell and the concerned branch (a) No, the bank can’t take the RIL to insolvency proceedings.
of the bank warns the RIL that default may result in insolvency (b) Yes, the bank can take the RIL to insolvency proceedings
proceedings against the RIL. The RIL pleaded that default is because the default is considered as default, willingness
not wilful, instead, this RIL said it really willing to continue its is irrelevant.
business operations and repay the loan amount as and when (c) Yes, the bank can take the RIL to insolvency proceedings
the business conditions improve. But it seems, it will not be in a because the amount of default exceeds R1 lakh
position to repay the loan at-least in the year to come. (d) No, the bank can’t take the RIL to insolvency proceedings
Mr. Anonymous, an employee in the IT and ERP department because the amount of default is less than the threshold
at RIL uses his workstation to hack the IT server of security limit of R50 lakh.
and intelligence services of the country, such as the research
and analysis wing, and capture the top-secret information. The 2. At what stage, is the laundering process when it reached the
information which he captured, if leaked; can put the defence hands of Mr. Kavir?
and sovereignty of India at severe risk. Mr. Anonymous also (a) Integration
indulge in funding and other arrangements for a terror attack in (b) Layering
the financial capital of India ‘Mumbai’. Indian authorities caught (c) Stratifying
hold of Mr. anonymous while he was transmitting such top- (d) Splitting
secret information through the internet and took him to custody.
One of the executive directors at RIL, Mr. Mohan Bhave 3. What shall be the punishment for the wrongdoing done by
sought some funds into his bank account to acquire any Mr. Anonymous?
immovable property in Mumbai for R 2.5 crore. He has around (a) Fine or rigorous imprisonment for a term which shall not
R1.25 crore in his bank accounts and for the balance amount he be less than three years but which may extend to seven
ask to his friend Mr. Maan in Country M. The friend transferred years.
money to Mr. Ganpat’s Account in Country G. Mr. Ganpat (b) Fine and rigorous imprisonment for a term which shall
transferred the half of funds to Ms. Bhosle in Country B and not be less than three years but which may extend to
remaining half to Ms. Indrani in Country I. Ms. Bhosle and Ms. seven years.
Indrani, in turn, transferred the funds to Mr. Kavir in Country K (c) Fine and rigorous imprisonment for a term which shall
and Ms. Sonam in Country S, respectively. not be less than three years but which may extend to ten
Rocky, the son of Mr. Mohan Bhave is a rock star and singing years.
sensation across the South Asian and European countries. (d) Fine upto R 5 lakh and rigorous imprisonment for a term
Rocky performed numerous successful tours abroad. Rocky has which shall not be less than three years but which may
acquired immovable properties abroad from the consideration extend to seven years.
he accepts from organisers of his shows, he recently buys a
luxurious yacht. 4. Who has the authority to provisionally attach the property of
Rocky accepted said money from Mr. Kavir (in Country K) Mr. Mohan Bhave?
and Ms. Sonam (in Country S) as an advance for his singing i. Director
performance at their functions/parties, with the understanding ii Deputy Director
that on a later date prior to the show date Mr. Kavir and Ms. iii Deputy Director authorised by the Director
Sonam express their inability to arrange functions/parties and iv Judicial Magistrate
request to cancel the performance; and money will be forfeited (a) i, ii, and iv
by Mr. Rocky. In this way, Mr. Mohan Bhave will get money to (b) i, iii, and iv
acquire the immovable property. (c) i and ii
Rocky was arrested by the officers of the Enforcement (d) i and iii
Directorate at Delhi Airport on his return to India for an offence
relating to the possessing and disposal of illegally acquired foreign 5. Within how many days, the authority who provisionally
exchange and taken before the Additional Chief Metropolitan attached the property has to file a complaint with
Magistrate, New Delhi on the very next date. Enforcement Officer Adjudicating Authority?
moved the application to seek ‘judicial remand’ (detention) on (a) Within 14 days from the attachment
the ground that it was necessary to complete the investigation. (b) Within 30 days from the attachment
Office of director conducts an inquiry under section 13 of (c) Within 45 days from the attachment
Prevention of Money-Laundering Act, 2002. Mr. Gulati is an (d) Within 60 days from the attachment
officer of the concerned reporting entity and summoned to
attend the proceeding. Mr. Gulati joined the reporting entity Descriptive Questions
just 3 months back whereas the principle matter of inquiry is 1. Examine the legal position of the stated situations in the light
older than that, hence Mr. Gulati finds the summon unjustified. of the given facts under the Prevention of Money Laundering
Mr. Gulati has to attend a global business conference as a guest Act, 2002, whether Enforcement Directorate is competent
speaker which is falling on same day and date which is mentioned to arrest and take judicial remand of an arrested person?
in summon. Whether the Magistrate before whom a person arrested is

The Chartered Accountant Student May 2021 09

Page 475
Economic Laws
produced has jurisdiction to authorise the detention of that difficult to detect the origin of the money, thus, it is the stage
person? of layering.

2. Advise the Banks officials who consulted you ‘is the amount of 3.(b): Section 4 of the Prevention of Money Laundering Act
default is significant criteria to invoke application under the 2002 provides for the Punishment for Money-Laundering
Insolvency and Bankruptcy Code for Insolvency Resolution - Whoever commits the offence of money-laundering shall
and Liquidation for Corporate Persons?’ be punishable with rigorous imprisonment for a term which
shall not be less than three years but which may extend to
3. Comment can Mr. Gulati be summoned? Whether a Mr. seven years and shall also be liable to fine.
Gulati is bound to attend the proceeding in person? State the But where the proceeds of crime involved in money-
nature of proceeding taken here under the case study ? laundering relate to any offence specified under paragraph
2 of Part A of the Schedule (i.e. Offences under the Narcotic
Answer to MCQs Drugs and Psychotropic Substances Act, 1985), the maximum
1.(a): A new section 10A inserted (vide Insolvency and Bankruptcy punishment may extend to ten years instead of seven years.
Code (Second Amendment) Act 2020, subsequent to an Since, offence committed by Mr. Anonymous ‘waging or
ordinance dated 5th June 2020) considering the possible attempting to wage war or abetting waging of war, against
adverse impact of the pandemic on businesses, which read the Government of India’, is covered under paragraph 1 of
as notwithstanding anything contained in sections 7, 9, and Part A of the Schedule, hence he will be liable to fine and
10, no application for initiation of corporate insolvency imprisonment for a term which shall not be less than three
resolution process of a corporate debtor shall be filed, for any years but which may extend to seven years.
default arising on or after 25th March 2020 for a period of
six months or such further period, not exceeding one year 4.(d): Section 5(1) of the Prevention of Money Laundering Act
from such date, as may be notified in this behalf. It is also 2002, provides where the Director or any other officer not
provided that no application shall ever be filed for initiation below the rank of Deputy Director authorised by the Director
of corporate insolvency resolution process of a corporate for the purposes of this section, has reason to believe (the
debtor for the said default occurring during the said period. reason for such belief to be recorded in writing), on the basis
Moreover, Ministry of Corporate Affairs vide notification of material in his possession, that
S.O. 1205(E) dated 24th March 2020, in the exercise of the (a) Any person is in possession of any proceeds of crime;
powers conferred by the proviso to section 4 of the Insolvency and
and Bankruptcy Code, 2016, the Central Government hereby (b) Such proceeds of crime are likely to be concealed,
specifies one crore rupees as the minimum amount of default transferred, or dealt with in any manner which may
for the purposes of the said section. result in frustrating any proceedings relating to the
Thus, since the default is taken place after 24th March 2020 confiscation of such proceeds of crime under this
(falling in the specified period under section 10A) and the Chapter,
amount of default of the company is less than R1 crore, hence He may, by order in writing, provisionally attach such
bank can’t drag the RIL for insolvency proceedings. property for a period not exceeding one hundred and eighty
Note- Vide SO 3265 (E) dated 24th Sep 2020 application of days from the date of the order, in such manner as may be
section 10A extended by a further period of 3 months from prescribed.
25th Sep 2020. Further, vide SO 4638 (E) dated 22nd Dec 2020
application of section 10A once again extended by a further 5.(b): Section 5(5) of the Prevention of Money Laundering Act
period of 3 months from 25th Dec 2020 (Hence period 2002 provides that the Director or any other officer who
specified under section 10A ranges from 25th March 2020 to provisionally attaches any property under sub-section (1)
24th March 2021) shall, within a period of thirty days from such attachment,
file a complaint stating the facts of such attachment before
2.(b) : Money laundering is a single process, however; its cycle the Adjudicating Authority.
can be broken down into three distinct stages
• Placement is the first and the initial stage when the crime Answers to Descriptive Questions
money is injected into the formal financial system. 1. The facts given in the case are similar to the case of
• Layering is the second stage, in this money injected Directorate of Enforcement vs. Deepak Mahajan (SC,
into the system is layered and moved or spread over Criminal Appeal No. 537 of 1990 dated 31.01.1994) wherein
various transactions in different accounts and different while disposing of the SLP (Special Leave Petition), the
countries. Thus, it will become difficult to detect the hon’ble apex court answered the important question of
origin of the money. law ‘Whether the Directorate of Enforcement fall within
• Integration is the third and final stage, in this money the definition of ‘Police Officer’ under Section 167 of CrPC
enters the financial system in such a way that original (Criminal Procedure Code) or not?’ The Supreme Court
association with the crime is sought to be obliterated stated that the pre-requisite of arrest that ‘it should have
so that the money can then be used by the offender or been effected only by a police officer and no one else’ and
person receiving as clean money. ‘there must necessarily be records of entries of a case diary’,
Thus, from the above, when funds reached Mr. Kavir, it is may be dispensed to invoke Section 167(1) of CrPC (Criminal

10 May 2021 The Chartered Accountant Student

Page 476
Economic Laws
Procedure Code). Hence the Supreme Court stated that the It is important to note here, that Ministry of Corporate
Enforcement Officer can be termed as ‘police officer’ for the Affairs vide notification S.O. 1205(E) dated 24th March 2020,
purpose of arrest. in the exercise of the powers conferred by the proviso to
Hence in the given case Enforcement Directorate is competent section 4 of the Insolvency and Bankruptcy Code, 2016, the
to arrest and take judicial remand of an arrested person. Central Government hereby specifies one crore rupees as
Further, the Supreme Court held that “sub-sections (1) and the minimum amount of default for the purposes of the said
(2) of Section 167 are squarely applicable with regard to the section.
production and detention of a person arrested under the Prior to 24th March 2020, this threshold limit was one lakh
provisions of Section 35 of FERA (now the corresponding instead of one crore.
provision of FEMA) and Section 104 of Customs Act and
that the Magistrate has jurisdiction under Section 167(2) to 3. Section 50 of the Prevention of Money Laundering Act 2002,
authorise the detention of a person arrested by an authorized deals with the power of authorities, which they can exercise;
officer of the Enforcement under FERA (now the FEMA) and especially while conducting any inquiry or any proceeding.
taken to the Magistrate in compliance of Section 35(2) of As per sub-section 2 of section 50, the Director, Additional
FERA (now the corresponding provision of FEMA). Director, Joint Director, Deputy Director, or Assistant
Hence in a given case, against the application of the Director shall have the power to summon any person
enforcement officer, the Magistrate before whom a person whose attendance he considers necessary whether to give
arrested is produced has jurisdiction to authorise the evidence or to produce any records during the course of
detention of that person. any investigation or proceeding under this Act. Hence, Mr.
Gulati can be summoned.
2. Yes, the minimum amount of default is significant criteria to As per Sub-section 3 to section 50, all the persons so
invoke the application under the Insolvency and Bankruptcy summoned shall be bound to attend in person or through
Code for insolvency resolution and liquidation for corporate authorised agents, as such officer may direct, and shall be
persons. bound to state the truth upon any subject respecting which
Section 4 of the Code read as ‘This Part (PART II dealing they are examined or make statements, and produce such
with insolvency resolution and liquidation for corporate documents as may be required.
persons) shall apply to matters relating to the insolvency Hence, Mr. Gulati is bound to attend the proceeding; but if
and liquidation of corporate debtors where the minimum the office of the director directs or authorises he can attend
amount of the default is one crore rupees. the meeting through authorised agents rather than in person.
There is a proviso to section 4 which read as ‘the Central Further, as per Sub-section 4 of section 50, every proceeding
Government may, by notification, specify the minimum under Sub-section (2) and (3) shall be deemed to be a judicial
amount of default of higher value which shall not be more proceeding within the meaning of section 193 and section
than one crore rupee’ 228 of the Indian Penal Code.

Case Study 3
XYZ Limited (Corporate Debtor) is undergoing the protested the decision of the Resolution Professional, by filing
Corporate Insolvency Resolution Process (CIRP) under the an application before the Adjudicating Authority with a prayer to
Insolvency and Bankruptcy Code, 2016 (Code or IBC) which direct the Resolution Professional to accept the Resolution Plan
was commenced on 17th July, 2019 and is under a moratorium. filed by the Resolution Applicant 1. In reply to the application filed
The Resolution Professional of the Corporate Debtor invited by the Resolution Applicant 1 before the Adjudicating Authority,
expression of interest (EoI) by publishing relevant form in the the Resolution Professional made the following submissions in
newspapers and subsequently received two expressions of his counter-affidavit filed with the Adjudicating Authority:
interest from prospective Resolution Applicants (Resolution - Resolution Applicant 1 meets the following ineligibilities:
Applicant 1 and Resolution Applicant 2). • The directors of one of the subsidiaries of the
One of directors at XYZ Limited who gave a personal Resolution Applicant 1 are declared as wilful
guarantee against the borrowings of XYZ Limited has credence defaulters
that after the declaration of moratorium under section 14 of IBC, • The step-down subsidiary of the RA has been
legal action against him is barred too. declared as Non-Performing Asset and it remained as
Pursuant to the regulations, the Resolution Professional a Non-Performing Asset for more than one year.
had sent an information memorandum, evaluation matrix, and - The Resolution Applicant 1 had filed an affidavit as
request for a resolution plan to both the prospective Resolution required under the Code and the Regulations made
Applicants. thereunder but had failed to disclose the above-
Resolution Applicant 1 had filed its resolution plan on 20th mentioned ineligibilities in the affidavit thereby
October 2019 and the Resolution Professional had rejected that misleading the Resolution Professional.
resolution plan on 1st November 2019 on the ground that it is in - Since Resolution Applicant 1 meets the ineligibility
violation of the provisions of the Code pertaining to ineligibility criteria as stipulated by the Code, the instant application
of the Resolution Applicant. The Resolution Applicant 1 filed by the Resolution Applicant 1 be dismissed.

The Chartered Accountant Student May 2021 11

Page 477
Economic Laws
In response to the submissions made by the Resolution iii. The ineligibility may be removed if the overdue amounts
Professional, Resolution Applicant 1 stated that as on the date relating to Non-Performing Accounts are paid before
of submission of resolution plan with the resolution professional submission of the resolution plan
it does not meet any of the above-stated ineligibilities and that (a) i only
the Resolution Professional has analysed the position as on (b) ii only
the Insolvency Commencement Date instead of the date of (c) i and iii
submission of the resolution plan and hence his arguments do not (d) ii and iii
hold any water. The matter was pending before the Adjudicating
Authority. 2. Pursuant to the provisions of the Insolvency and Bankruptcy
On the other hand, the resolution plan received from the Code, 2016, what shall be time to obtain the approval of the
other Resolution Applicant, i.e. Resolution Applicant 2 was Competition Commission of India?
forwarded by the Resolution Professional to the Committee (a) After submission of resolution plan but before the
of Creditors for their consideration and evaluation on 1st approval of the same by Committee of Creditors
November 2019. During the evaluation, it was observed that the (b) Before the submission of the resolution plan
resolution plan submitted by Resolution Applicant 2 meets the (c) After approval of Committee of Creditors
criteria prescribed for combinations under the provisions of the (d) After submission of resolution plan but before filing the
Competition Act, 2002. Accordingly, Resolution Applicant 2 filed plan with the Adjudicating Authority
an application before the Competition Commission of India for
its approval of the proposed combination as per the submitted 3. Who among the following can file an application to the
resolution plan. Adjudicating Authority for extension of the period of CIRP?
On 15th November 2019, the Competition Commission (a) Committee of Creditors after passing a resolution with
of India summoned Resolution Applicant 2 for a hearing on more than 66% of voting share in their meeting
the approval of said combination. During the hearing, the (b) Any stakeholder interested in the affairs of the Corporate
Competition Commission of India raised various questions to Debtor
understand if such a combination has any appreciable adverse (c) Resolution Professional upon instructions do so by
effect on relevant product market and relevant geographic resolution passes at the meeting of the Committee of
market in India. Accordingly, Resolution Applicant 2 had filed its Creditors by 66% voting share
reply to the Competition Commission of India both orally during (d) Resolution Professional at its own
the hearing as well as in writing on November 20, 2019. Having
heard the Resolution Applicant 2 and also having gathered 4. Which among the following are the duties of the Resolution
relevant information to understand whether the combination Professional?
causes an appreciable adverse effect on competition in the i. To present to the Committee of Creditors, only those
relevant market in India or not; the competition commission resolution plans which confirm the conditions prescribed
of India had passed its order approving the combination on 3rd under the Code
February, 2020. ii. To present all resolution plans to the Committee of
On 1st January 2020, the committee of creditors negotiated Creditors
with the Resolution Applicant 2 for modifications in the resolution iii. To obtain approval of the Competition Commission of
amount which was duly agreed to by the resolution applicant, India for the resolution plans approved by the Committee
and post-modification of resolution plan, the revised resolution of Creditors
plan of the Resolution Applicant 2 has been evaluated by the (a) i only
members of the committee of creditors. On 10th January, 2020 (b) ii only
the Committee of Creditors decided to vote on the resolution (c) i and iii
plan of Resolution Applicant 2 as one hundred and eighty days (d) ii and iii
from the insolvency commencement date is set to conclude on
13th January, 2020. Accordingly, the committee of creditors had 5. Which of the following shall be considered to ascertain as to
voted on the resolution plan submitted by Resolution Applicant whether the Resolution Applicant and the Corporate Debtor
2 and approved the same with the voting share of 85%. Post meet the definition of combination under the Competition
approval of resolution plan by the Committee of Creditors, the Act, 2002?
Resolution Professional filed the same with the Adjudicating i. Assets
Authority on 13th January, 2020. ii. Net Worth
iii. Turnover
Multiple Choice Questions iv. Control
1. While examining the ineligibility of resolution applicants (a) i, ii, and iv
pursuant to the provisions of the Code, which among the (b) i and iii
following statements are incorrect: (c) ii, iii, and iv
i. The ineligibility shall be as on the date of submission of (d) i, iii, and iv
the Resolution Plan by the Resolution Applicants
ii. The ineligibility shall be as on the insolvency Descriptive Questions
commencement date 1. Clarify how the Competition Commission of India

12 May 2021 The Chartered Accountant Student

Page 478
Economic Laws
investigates combinations (to regulate) before giving its in a gain of control over enterprise by another enterprise
approval under section 31 of the Competition Act, 2002. either individually or in group constituted as a combination.

2. One of the directors at XYZ Limited who gave a personal Extra reference note for students
guarantee against the borrowings of XYZ Limited has It is important to note here that, under section 20 (3) of
credence that after the declaration of moratorium under the Competition Act 2002, the Central Government shall
section 14 of IBC, legal action against him is barred too. Is at the expiry of every two years, in consultation with the
the credence of the director valid? Apart from provisions Commission, by notification, enhance or reduce the value of
from the bare act, support your opinion with settled judicial assets or the value of turnover mentioned above (for purpose
precedent. of section 5 ‘combination’), on the basis of the wholesale price
index or fluctuations in the exchange rate of rupee or foreign
Answer to MCQs currencies. *Vide notification number S.O. 675(E) dated
1. (b) : The opening line of section 29A of the Insolvency and 4th March 2016, in the exercise of the powers conferred by
Bankruptcy Code 2016, and then further of clause ‘c’ in it section 20 (3) the Central Government enhances, the value
clearly states ‘at the time of submission of resolution plan’ of assets and the value of turnover, by hundred percent from
hence point i is correct and point ii is incorrect. the date of publication of this notification in the Official
Further first proviso to section 29A (c), provided that the Gazette. The publication date is also 4th March 2016.
person shall be eligible to submit a resolution plan if such Hence w.e.f. 4th March 2016 above table (threshold under
person makes payment of all overdue amounts with interest section 5) shall be read as;
thereon and charges relating to nonperforming asset Threshold applicable to Enterprises Group Level
accounts before submission of resolution plan hence point iii Level
also correct. In India Joint Assets R2,000 Cr R 8,000 Cr
Joint Turnover R 6,000 Cr R 24,000 Cr
2.(a) : Proviso to section 31 (4) of the Insolvency and Bankruptcy In India Joint Total Assets US$ 1000 US$ 4000
Code 2016, provides where the resolution plan contains a and Million Million
provision for combination, as referred to in section 5 of the Outside Minimum Indian R1000 Cr R 1000 Cr
Competition Act, 2002, the resolution applicant shall obtain Component
the approval of the Competition Commission of India under Joint Total US$ 3000 US$ 12000
that Act prior to the approval of such resolution plan by the Turnover Million Million
committee of creditors. Minimum Indian R3000 Cr R 3000 Cr
Component
3.(c): As per section 12 (2) of the Insolvency and Bankruptcy
Code 2016, the resolution professional shall file an application Answers to Descriptive Questions
to the Adjudicating Authority to extend the period of the 1. Section 6 (1) of the Competition Act 2002, simply prohibits
corporate insolvency resolution process beyond one hundred the person or enterprise from entering into a combination
and eighty days, if instructed to do so by a resolution passed that causes or is likely to cause an appreciable adverse effect
at a meeting of the committee of creditors by a vote of sixty- on competition within the relevant market in India and such
six percent of the voting shares. a combination shall be void.
Further, the review process for a combination under the Act
4.(b) : Section 25 (2) shall be read along with section 30 (3) of involves mandatory notification to the Commission of the
the Insolvency and Bankruptcy Code 2016, the combined proposed combination. To give effect to this section 6 (2)
reading of these signifies that the resolution professional provide, any person or enterprise proposing to enter into a
shall present all resolution plans at the meetings of the combination shall give notice (as prescribed in section 30) to
committee of creditors. the Commission in the specified form disclosing the details
Further as per section 30 (6), the resolution professional shall of the proposed combination within 30 days of the approval
submit the resolution plan as approved by the committee of of the proposal relating to merger or amalgamation by the
creditors to the Adjudicating Authority. board of directors or of the execution of any agreement or
other document in relation to the acquisition, as the case
Extra reference note for students may be.
As per proviso to section 31 (4), where the resolution plan Further, as per section 20 (1), the Commission may, upon
contains a provision for combination, as referred to in section its own knowledge or information relating to acquisition
5 of the Competition Act, 2002, the resolution applicant shall referred to in clause (a) of section 5 or acquiring of
obtain the approval of the Competition Commission of India control referred to in clause (b) of section 5 or merger or
under that Act prior to the approval of such resolution plan amalgamation referred to in clause (c) of that section, inquire
by the committee of creditors. into whether such a combination has caused or is likely to
cause an appreciable adverse effect on competition in India.
5.(d): Section 5 of the Competition Act 2002, provide the Here it worth noting that the Commission shall not initiate
thresholds relating to the value of assets and amount of any inquiry under this subsection after the expiry of one year
turnover, beyond which the merger and acquisition resulting from the date on which such combination has taken effect

The Chartered Accountant Student May 2021 13

Page 479
Economic Laws
Further section 20 (2) [inquiry in response to notice under Clause (b) section 14 (3) of the Insolvency and Bankruptcy
section 6(2)] read with section 31 (framing of opinion to Code, 2016 (IBC), read as the provisions of sub-section (1) shall
pass an order) and the Competition Commission of India not apply to a surety in a contract of guarantee to a corporate
(Procedure in regard to the transaction of business relating to debtor. It important here to note that sub-section (1) gave
combinations) Regulations, 2011 the Commission shall form power to Adjudicating Authority to declare a moratorium.
firstly prima facie opinion as to whether the combination is The validity of directors’ credence can be denied based on
likely to cause or has caused an appreciable adverse effect on the State Bank of India vs. V. Ramakrishnan (Supreme Court,
competition within the relevant market in India or not. For Civil Appeal No. 3595 of 2018), wherein the facts are largely
this, investigation by director-general can be ordered under similar to the present case.
section 29. The Hon'ble Supreme Court first considers the fact that
Section 20 (4) laid down factors to be considered by the different provisions of the Insolvency and Bankruptcy Code
Commission while evaluating the appreciable adverse effect are applicable to the insolvency of different categories of
of Combinations on competition in the relevant market persons. Section 96 and 101 of the Code provide for separate
include the following: provision for a moratorium for the personal guarantor,
(a) Actual and potential level of competition through whereas section 14 deals with corporates.
imports in the market; Court also observed that different provisions of law brought
(b) Extent of barriers to entry into the market; into effect on different dates and some of the provisions were
(c) Level of concentration in the market; not yet enforced (on the date of the judgment). Provisions
(d) Degree of countervailing power in the market; pertaining to sections 96 and 101 have not been brought into
(e) Likelihood that the combination would result in the force.
parties to the combination being able to significantly Further, the apex court makes observations on relevant
and sustainably increase prices or profit margins; sections. The court observed that Section 14 of the Code
(f ) Extent of effective competition likely to sustain in a authorizes Adjudicating Authority to pass an order of
market; moratorium during which there is the prohibition on the
(g) Extent to which substitutes are available or are likely to institution of suits or continuation of pending suits against
be available in the market; the corporate debtor, transfer of property of the corporate
(h) Market share, in the relevant market, of the persons debtor, or any action to foreclose or enforce any security
or enterprise in a combination, individually and as a interest.
combination; The apex court also consider the following facts importantly
(i) Likelihood that the combination would result in the - Report of Insolvency Law Committee dated 26.03.2018
removal of a vigorous and effective competitor or clarified that the period of moratorium under section
competitors in the market; 14 is not applicable to personal guarantors,
(j) Nature and extent of vertical integration in the market; - Amendment Ordinance dated 6th June 2018, which
(k) Possibility of a failing business; amended the provision of section 14 and proviso
(l) Nature and extent of innovation; clearly states that the moratorium period envisaged in
(m) Relative advantage, by way of the contribution to the section 14 is not applicable to a personal guarantor to a
economic development, by any combination having corporate debtor. (Note – this ordinance later enacted
or likely to have an appreciable adverse effect on as act 26 of 2018 – and enforced w.r.e.f. 6th June 2018)
competition; Hence, as the provisions of section 96 and 101 have not been
(n) Whether the benefits of the combination outweigh the brought into force, the personal guarantor is not entitled to
adverse impact of the combination if any. a moratorium period under the Insolvency and Bankruptcy
Code.
2. The Director of XYZ Limited, hold credence that section 14 Hence, the credence of the Director of XYZ Limited that
of the Insolvency and Bankruptcy Code, 2016 (IBC) would ‘that section 14 of the Insolvency and Bankruptcy Code,
apply to the personal guarantor as well, as a result of which 2016 (IBC) would apply to the personal guarantor as well’ is
proceedings against the personal guarantor and his property not tenable. (Even before 6th June 2018 when sub-section 3 to
would have to stay if moratorium declared. section 14 substituted).

Case Study 4
Mr. Aman Chawla belongs to Delhi based business family and joined an MNC in the role of system engineer after college. But
has ancestral roots in Kharar, a Town in the Sahibzada Ajit Mr. Aman is inspired by constructing the buildings, towers,
Singh Nagar (Mohali) district in the state of Punjab (around 15 landscapes, hence decided to quit the job to pursue his passion.
KMs away from Chandigarh). Chawla family owns the chain of Despite the Chawla family owning a major stake in the
restaurants, snacks points, and Ice-Cream parlours across the business, the business model is unlike to autocratic monarchy.
nation. Few of these are owned properties, but a large number It is managed professionally and listed on the stock exchange.
are leased properties. The holding company is Chawla Snacks and Family members (father, grand-mother and elder brother of Mr.
Refreshment Limited (CSRL). Mr. Aman is an electrical engineer, Aman) are part of the Board of Directors, whereas few other

14 May 2021 The Chartered Accountant Student

Page 480
Economic Laws
family members are also engaged with CSRL but in form of investor from the States (US) based on showing growth prospect
employment (or in a professional capacity). in his business to his investor. The investor was a good friend
Mr. Aman joined his brother-in-law, Mr Vivek, in his of Mr. Dipan and originally from Mohali named Mr. Tarun and
construction business, Mr. Aman assists Mr. Vivek in ongoing settled in Philadelphia (Pennsylvania, US). Mr. Tarun agreed to
projects, and one among them is Rishi Enclave whose centre of invest US$1 Million in the said real estate project.
attraction is state of art yoga centre which will be one of its type The money got transferred from an overseas branch in
in the world apart from the common area which is turned into Philadelphia of some Indian bank (through banking channel) to
with mesmerising landscapes. The project is located near Jolly the Kharar branch (Mohali, India). The Branch Manager in India
Grant Airport on out-skirt of the holy town of Rishikesh. Rishi is the friend of an elder brother of Mr. Aman and was excited
Enclave (Project) consists of 120 units of 2BHKs, 3BHKs (Flats to get one project in Mohali and thus approved the investment
and Floors), and Independent Houses or Villas in totality. The without any opinion from any Finance Professional.
project is registered under the RERA. All 120 units’ subscribed/ CSRL witnessed the bad jolts (of financial turbulence) as
booked by allottees except 2 Flats kept by Mr. Vivek (promoter). revenue vanished and reserves are socked to meet maintenance
Mr. Tirlochan Negi booked 3 floors one in his own name, another costs of properties & employee cost due to lock-down and
one in the name of his daughter in law and the third one in name of afterword restrictions. The financial cost and lease rentals not
his company. Mr Dabral also booked a flat and a villa (both in his only erode the working capital but also forces the CSRL to land
name). Rest all allottee booked one unit each. Soon allottees form into a debt trap situation wherefrom meeting financial obligations
a residential association. Considering the latest NGT decisions seems near to impossible. The only way left to management is
and amendments in policy about the environment (applicable for restructuring of business hence board decided to shut a few
civil construction in hill or foothill area concerning the height points and parlours (to reduce lease rental obligation, and free-
of the building), certain structural changes relating to the height up one-two owned properties so that sale proceed can be infused
and common area landscape is required in sanctioned plan of the as working capital)
project. Mr. Vivek is of opinion since the alteration in sanctioned One of the properties sold by CSRL, acquired by Ms. Vijeta in
plan enforced by changes in policy matter hence the approval of name of her mother-in-law (as she is a senior citizen female – to
allottees is not required. bear less registration cost in form of stamp duty), consideration
Mr. Aman recently visited Kharar after a long time to meet for which is paid out of the known sources of the Mr. Vijeta.
his friends Mr. Onkar Singh and Mr. Dipan Ahuja of early Despite the best efforts made by management at CSRL,
childhood. They all admitted that the town has developed still, the bottom line is in deep red; resulting in default in
substantially especially the townships and Skyscrapers as tri-city repayment of financial debts and such default continues since
(Mohali, Chandigarh, and Panchkula) turns into metropolitan the 2nd quarter of Fiscal 2020-21. Management gave assurance
and hub of service entities. The lifestyle of people also improves. to financial creditors that soon it will overcome the solvency
Mr. Onkar is settled in Canada and holding a Canadian passport issue and they already took corrective measures. On 19th, March
and citizenship as his family migrate there when he was in school 2021, one of the financial creditors moved an application for
only. In Canada, he own a transport business. Currently, he is initiation of corporate insolvency resolution proceeding (CIRP)
on a visit to India to attend the marriage of a relative. Mr. Dipan whose outstanding claim is of INRs 120 lakh. On 26th March
Ahuja is a supplier of construction materials and planning to 2021, another financial creditor file an application to NCLT for
venture into the solar panel business under make in India drive, initiation of CIRP against CSRL in their case amount of default
considering the enhancing role of solar energy for household and is INRs 35 lakh and such default took place in the 3rd Quarter of
commercial uses. Mr. Dipan believes Mr. Aman (considering his fiscal 2020-21.
electrical engineering background) should join him in his solar
panel venture. Multiple Choice Questions
The ancestral property of Mr. Onkar’ family has been 1. Regarding the state of art yoga centre and common area
unoccupied for a long, hence turned into a mud house. Mr. Onkar situated in Rishi Enclave, which of the following statement is
offered Mr. Aman to develop residential apartments on such correct;
property after the name of his grand-father ‘Satnam Apartments’. (a) Promoter will keep the possession and title both
A chunk of land on the backside of such property is also available (b) Promoter may handover physical possession of these to
for sale at a reasonable price because it has no connectivity. Mr. the association of allottees or competent authority as per
Aman found it a good idea to develop the residential apartments the local laws
as backside land can be acquired at a cheaper rate than prevailing (c) In absence of any local law promoter shall hand over
in the market. Mr. Onkar talked to his father [property inherited, within thirty days after obtaining the occupancy
hence registered in his name in land revenue records after the certificate.
death of grandfather (who was resident in India) of Mr. Onkar] (d) In absence of any local law promoter shall hand over within
and ready to transfer (sale) the property for INRs 2.5 Crore. thirty days after obtaining the completion certificate.
The Father of Mr. Onkar is a resident outside India who never
registered as OCI. Mr. Aman after communicating with Mr. 2. State the legal position of mother-in-law of Ms. Vijeta as
Vivek agreed to deal. benamidar in the case study-
Mr Aman heard about the importance of keeping capital (a Yes, the mother-in-law of Ms. Vijeta is benamidar
low to generate more wealth and attain high ROI (Return on (b) No, the mother-in-law of Ms. Vijeta is not benamidar as
Investment). He decided to borrow money from a private she is covered under the exceptions stated

The Chartered Accountant Student May 2021 15

Page 481
Economic Laws
(c) No, mother-in-law of Ms. Vijeta is not benamidar as Answer to MCQs
consideration is paid out of the known source of Ms Vijeta 1. (d) : Reason - As per section 17 (2) of the Real Estate (Regulation
(d) Both b and c above. and Development) Act 2016, it shall be the responsibility of
the promoter to handover the necessary documents and
3. Which of the following statements is correct regarding the plans, including common areas, to the association of the
acquiring, holding, owning and transfer of property, in a case allottees or the competent authority, as the case may be, as
by the father of Mr. Onkar in India- per the local laws:
(a) Being a person resident outside India he can acquire, Provided that, in the absence of any local law, the promoter
hold, own and transfer any immovable property in India, shall hand over the necessary documents and plans,
but with RBI permission only including common areas, to the association of the allottees
(b) Being a person resident outside India he can acquire, hold, or the competent authority, as the case may be, within thirty
own and transfer any immovable property in India, but days after obtaining the completion certificate.
only in joint ownership with any person resident in India
(c) Being a person resident outside India he can acquire, 2.(a): Reason – As per clause (9) to section 2 of the Prevention
hold, own and transfer any immovable property in India, of Benami Property Transaction Act 1988, the transaction is
if inherited by him from the person who was a resident a benami transaction under sub-clause (A) because the same
of India is not covered under exception iv. Since the transaction is
(d) Being a person resident outside India he can acquire, benami hence the property become benami under section 2
hold, own and transfer any immovable property in India, (8), hence benamidar under 2 (10).
if inherited by him when he himself was resident in India
3.(c): Reason – As per section 6(5) of the Foreign Exchange
4. Whether the application moved on 19th March 2021 can be Management Act, 1999 a person resident outside India may
admitted by NCLT to initiate CIRP against CSRL-. hold, own, transfer or invest in any immovable property
(a) Yes, because CSRL made default in repayment of situated in India if such property was acquired, held or
financial debts owned by such person when he was resident in India or
(b) Yes, because the amount of default is more than one crore inherited from a person who was resident in India.
(c) No, because management gave assurance to financial Here is worth noting that regulation 3 and 6 of the Foreign
creditors that soon it will overcome the solvency issue Exchange Management (Acquisition and transfer of
and they already took corrective measures immovable property in India) Regulation 2018 gave the
(d) No, because an application for initiation of CIRP shall right to NRI and OCI (in case of regulation 3) and with
not be filled. the exclusion of other than agriculture land/farmhouse/
plantation property (both in case of regulation 3 and 6)
5. Whether the application moved on 26th March 2021 can be
admitted by NCLT to initiate CIRP against CSRL. 4.(d): Reason – As per section 10A of the Insolvency and
(a) Yes, because CSRL made default in repayment of Bankruptcy Code 2016 notwithstanding anything contained
financial debts in sections 7, 9 and 10, no application for initiation of
(b) Yes, because the application for initiation of CIRP may be corporate insolvency resolution process of a corporate
filled by the financial creditor as a period of suspension debtor shall be filed, for any default arising on or after 25th
of section 7 is over. March 2020 for a period of six months or such further
(c) No, because the amount of default is less than one crore period, not exceeding one year from such date, as may be
(d) No, because default occurred during a period of notified in this behalf.
suspension. On 24th September 2020 vide S.O. 3265(E) the Central
Government hereby notifies a further period of three months
Descriptive Questions from the 25th September 2020 for the purposes of section
1. Mr. Vivek is of opinion since the alteration in sanctioned plan 10A. Hence application can’t be filled under section 7 by the
enforced by changes in policy matter hence the approval of financial creditor till 24th March 2021.
allottees is not required. Are the changes in sectioned plan
minor in nature? Evaluate the opinion of Mr. Vivek in the 5.(d): Reason – As per section 10A of the Insolvency and
context of the provision contained in the RERA 2016? Bankruptcy Code 2016 notwithstanding anything contained
Support your answer with reason and calculation if any. in sections 7, 9 and 10, no application for initiation of
corporate insolvency resolution process of a corporate
2. What would be your opinion related to the repatriation of debtor shall be filed, for any default arising on or after 25th
funds in India as an Investment of US$1 million into the real March 2020 for a period of six months or such further
estate project in Kharar (Mohali, India)? period, not exceeding one year from such date, as may be
notified in this behalf.
3. Can the father of Mr. Onkar repatriate the sale proceed of Further the proviso to said section provided that no
ancestral property inherited by him to Canada from India? application shall ever be filed for initiation of corporate
Elucidate in the light of the relevant provision of applicable insolvency resolution process of a corporate debtor for the
law, the stated legal issue. said default occurring during the said period.

16 May 2021 The Chartered Accountant Student

Page 482
Economic Laws
On 24th September 2020 vide S.O. 3265(E) the Central booked in its name or booked in the name of its associated
Government hereby notifies a further period of three months entities or related enterprises, shall be considered as one
from the 25th September 2020 for the purposes of section allottee only.
10A. In the given case all 120 units’ subscribed/booked by allottees
Hence application can’t be filled under section 7 by a financial except 2 Flats kept by Mr. Vivek (promoter). Out of 118, Mr.
creditor for the default that occurred till 24th March 2021. Tirlochan Negi booked 3 floors one in his own name, another
Candidates also advised to note the explanation provided to one in the name of his daughter in law and the third one in
section 7(1), for the purposes of subsection (1) to section 7, a name of his company, whereas Mr. Dabral booked a flat and a
default includes a default in respect of a financial debt owed villa (both in his name); rest all allottee booked one unit each.
not only to the applicant financial creditor but to any other Hence the total number of allottee for purpose of section
financial creditor of the corporate debtor. Hence option C is 14(2)(ii) is 115 (118-2-1) considering Mr Tirlochan (3) and
not correct and mind it 10A is an overriding section. Mr Dabral (2) as a single allottee each. At least 2/3 allottee
shall be 77 (2/3rd of 115 – round up to next whole integer),
Answers to descriptive questions whose previous written consent is required; before making
1. The Real Estate (Regulation and Development) Act 2016 changes to sanctioned plan.
(herein-after RERA) under its section 14 provides the Hence the opinion of Mr. Vivek in the context of the provision
adherence to sanctioned plan and project specifications by contained in RERA, 2016 is untenable and incorrect.
the Promoter.
Sub-section 1 provides the proposed project shall be 2. Investments are considered as capital account transactions,
developed and completed by the promoter following the hence governed by section 6 of the Foreign Exchange
sanctioned plans, layout plans and specifications as approved Management Act, 1999 read with The Foreign Exchange
by the competent authorities. Management (Permissible Capital Account Transactions)
Sub-section 2 has an overriding effect and its clause (i) provide Regulations 2000 (herein-after regulations).
the promoter shall not make any additions and alterations in Clause (b) of regulation 4 of such regulations describe the
the sanctioned plans, layout plans and specifications and the prohibitions. Although regulation 4 (b) (iv) provides no
nature of fixtures, fittings and amenities described therein in person resident outside India shall invest in India, in any
respect of the apartment, plot or building, as the case may be, form, in any company or partnership firm or proprietary
which are agreed to be taken, without the previous consent concern or any entity, whether incorporated or not, which
of that person who agrees to take one or more of the said is engaged or proposes to engage in real estate business.
apartment, plot or building, as the case may be. But explanation 1 provides a certain exclusion from real
Here it is worth noting that the promoter may make such minor estate business, explanation read as ‘for this regulation, 'real
additions or alterations as may be required by the allottee, or estate business shall not include development of townships,
such minor changes or alterations as may be necessary due to construction of residential/commercial premises, roads or
architectural and structural reasons duly recommended and bridges and real estate investment trusts (REITs) registered
verified by an authorised Architect or Engineer after proper and regulated under the SEBI (REITs) Regulations, 2014.
declaration and intimation to the allottee. Hence repatriation of funds in India as Investment into the
For this clause, "minor additions or alterations" excludes real estate project (construction of residential apartments)
structural change including an addition to the area or change in Kharar (Mohali, Kharar) can be seen as a permissible
in height, or the removal of part of a building, or any change to capital account transaction under clause (a) to schedule II of
the structure, such as the construction or removal or cutting regulations.
into of any wall or a part of a wall, partition, column, beam,
joist, floor including a mezzanine floor or other support, or a 3. As per clause (a) to regulation 8 of the Foreign Exchange
change to or closing of any required means of access ingress Management (Acquisition and Transfer of Immovable
or egress or a change to the fixtures or equipment, etc. Property in India) Regulations, 2018, a person referred to in
Since in the given case certain structural changes (in the sub-section (5) of Section 6 of the Act, or his successor shall
sanctioned plan of the project) relating to height is required, not, except with the general or specific permission of the
hence the changes in sectioned plan are not minor in nature. Reserve Bank, repatriate outside India the sale proceeds of any
Further clause (ii) of Sub-section 2 provides the promoter immovable property referred to in that sub-section.
shall not make any other alterations or additions in the Whereas section 6(5) of the Foreign Exchange Management
sanctioned plans, layout plans and specifications of the Act, 1999 provides a person resident outside India may hold,
buildings or the common areas within the project without own, transfer or invest in any immovable property situated in
the previous written consent of at least two-thirds of the India if such property was acquired, held or owned by such
allottees, other than the promoter, who have agreed to take person when he was resident in India or inherited from a
apartments in such building. person who was resident in India.
It is worth noting here that for this clause, the allottees, Since in the given case father of Mr. Onkar acquired the
irrespective of the number of apartments or plots, as the case property through inheritance from his father who was resident
may be, booked by him or booked in the name of his family, in India, hence fall within the scope of section 6 (5). Therefore
or in the case of other persons such as companies or firms or with the permission of RBI, he can repatriate the sale proceed
any association of individuals, etc., by whatever name called, of ancestral property inherited by him to Canada from India.

The Chartered Accountant Student May 2021 17

Page 483
ENTERPRISE INFORMATION SYSTEMs
CA INTERMEDIATE - PAPER 7A: ENTERPRISE INFORMATION SYSTEMS
The capsule on Intermediate Paper 7A: Enterprise Information Systems that covers the Chapter 4 - “E-Commerce,
M-Commerce and Emerging Technologies” and Chapter 5 -“Core Banking Systems” of the subject is another step of
Board of Studies in its endeavour to provide quality academic inputs to the students of Intermediate Level of Chartered
Accountancy Course. This capsule intends to assist students in their quick revision and should not be taken as a
substitute for the detailed study of the aforesaid chapters. Students are advised to refer to the relevant Study Material
and Revision Test Paper for comprehensive study and revision.

Chapter 4: E-Commerce, M-Commerce And Emerging Technologies


This chapter provides an insight about meaning, Nature of Goods can be Goods cannot be
components and architecture of E-Commerce, various purchase inspected physically inspected physically
risks and controls associated with e-commerce and before purchase. before purchase.
applicable laws and guidance governing e-commerce. The Customer Face-to-face Screen-to-face
chapter further deals with the emerging technologies like interaction
Cloud Computing, Mobile Computing, Green Computing, Business Scope Limited to particular Worldwide reach
Internet of Things etc. and their perspectives. area
Information No uniform platform Provides a uniform
E-Commerce is the process of doing business electronically. exchange for exchange of platform for
It refers to the use of technology to enhance the processing of information. information exchange.
commercial transactions between a company, its customers and its Resource focus Supply side Demand side
business partners. In e-commerce, the buyers and sellers interact
Payment Cash, Cheque, Credit Credit card, fund
electronically using telecommunication networks rather than
card, etc. transfer, Cash in
through physical contact or exchange.
Delivery, Payment
BASE FOR TRADITIONAL E-COMMERCE Wallets, UPCI
COMPARISON COMMERCE application etc.
Definition Traditional commerce E-Commerce means Delivery of Instantly Takes time depending
includes all those carrying out commercial goods upon the address.
activities which transactions or exchange
Fraud Relatively lesser as Lack of physical
encourage exchange, of of information,
there is personal presence in markets
goods / services which electronically on the
interaction between and unclear legal issues
are manual and non- internet.
the buyer and the give loopholes for
electronic.
seller. frauds.
Location It requires a It requires market-
Process Because of manual Automated processing
marketplace to space.
processing of business of business transactions
operate.
transactions, chances minimizes the clerical
Size Type of items, size of Size of business of clerical errors are errors.
items, and the number model is influenced high.
of customers influence by products and
Profit Impact The cost incurred By increasing sales,
the size of the store. customers. Online
on the middlemen, cutting cost and
Stores expecting stores expecting heavy
overhead, inventory streamlining operating
heavy traffic need traffic need enough
and limited sales processes; the profits
to choose a location bandwidth, processing
reduces the profit of margin of manufacturers
with adequate parking power, and data
the organization. is increased, and
and entrances and storage capacity to
customers get better
walkways large enough provide proper service
prices.
to accommodate such to their customers.
traffic. Performance of online
stores is affected by the Illustration of E-Commerce Transaction
bandwidth capacity.
Marketing As stores have a To acquire a new Step 2: Select Step 3: Select the
physical presence customer, they have Step 1: Go to
Product type you desired product
known to potential to invest more money, the website.
wish to buy. from product list.
customers, no need to time and effort to
spend much to acquire advertise their presence
new customers as more aggressively on
compared to online internet. This is also Step 6: Payment Step 5: Review Step 4: Make
companies. This is called as One-to-one options like CoD,
the final price final choice and
also called as One-way marketing. Net banking,
credit card etc. and confirm the make payment
marketing. payment. online.
displayed to user.
Transaction Manual Electronically
Processing
Availability for For limited time. 24×7×365
commercial Step 7: Select the Step 8: Based on
transactions payment option and delivery terms, the
get directed to the product is delivered
payment gateway. to you.

18 May 2021 The Chartered Accountant Student

Page 484
ENTERPRISE INFORMATION SYSTEMs
Benefits of E-Commerce S. e – Market Description
No. models
A. Benefits to Customer / Individual / User
7 e-marketing e-marketing (Electronic Marketing) is the
• Convenience Time saving process of marketing a product or service
• Various products and services Options Easy to find reviews using the Internet. For example, they can
• Coupon and Deals Anytime Access compare prices of products by rival firms.
B. Benefits to Business / Sellers The internet changes the relationship
• Increased Customer Base Recurring payments made easy between buyers and sellers because market
information is available to all parties in the
• Instant Transaction Provides a dynamic market
transaction.
• Reduction in costs Efficiency improvement
• Creation of new markets Easier entry into new markets 8 e-procure- e-procurement is the management of all
ment procurement activities via electronic means.
• Better quality of goods Elimination of Time Delays
E-procurement infomediaries specialize
• Low barriers to entry in providing up-to-date and real-time
C. Benefits to Government information on all aspects of the supply of
• Instrument to fight corruption materials to businesses.
• Reduction in use of ecologically damaging materials 9 e -distribution e-distributor is a company that supplies
products and services directly to individual
business. An example of a firm specializing
• Internet Connection in e-distribution is www. wipro.com that
• High start-up costs uses the internet to provide fully integrated
• Legal issues e-business-enabled solutions that help to
Disadvantages unify the information flows across all the
• Some business processes may never
of E-business major distribution processes including
lend themselves to e-commerce
sales and marketing automation, customer
• Cultural impediments to e-business service, warehouse logistics, purchasing and
• Security Concerns inventory management, and finance.

Various e-Market models


Some Business Models for E-Commerce
S. e – Market Description
No. models Models Definition Examples
1 e - Shops Practical hands on training of the ERP Business- B2C is typically used to This may involve
System should be provided so that the to- refer to online retailers who Direct Sellers like www.
transition from old system to ERP system is Consumer sell products and services cisco.com; Online
smooth and hassle free. (B2C) to consumers through the intermediaries like
2 e -Malls The e-mall is defined as the retailing model Internet. Generally, this www.amazon.com.
of a shopping mall, a conglomeration of supports the activities
different shops situated in a convenient within the consumer chain
location in e-commerce. e.g., Yahoo! Stores. that focuses on sell-side
3 e-Auctions Electronic auctions provide a channel of activities.
communication through which the bidding Business- This supports the supply A wholesaler places
process for products and services can take
to- chain of organizations an order from a
place between competing buyers. E-auction
Business that involves commerce company's website
has become an increasingly popular tool
for the buyer to access the lowest price the (B2B) between a company and its and after receiving the
suppliers are willing to charge. Example – suppliers or other partners. consignment, sells the
www.onsale.com, www.ebay.com. A website sells its products end-product to the final
to an intermediate buyer customer who comes to
4 Portals Portal is a website that serves as a gateway who then sells the product buy the product at one
or a main entry point on the internet to a to the final customer. of its retail. Example -
specific field of interest or an industry. It is
www.indiamart.com.
a website that is positioned as an entrance
to other sites on the internet. Some major Consumer- With C2C e-business A consumer selling his/
general portals include Yahoo, Excite, and to- model, consumers sell her asset like residential
Netscape. Consumer directly to other consumers property, cars,
5 Buyer The Buyer Aggregator brings together (C2C) via on-line classified motorcycles etc. or rent
Aggregators large numbers of individual buyers so that ads and auctions, or by a room by publishing
they can gain the types of savings that are selling personal services their information on
usually the privilege of large volume buyers. and expertise on-line. the relevant website.
Example - www.zomato.com. C2C e-commerce allows Example - www.eBay.
6 Virtual Virtual Community is a community of unknown, untrusted com.
Communities customers who share a common interest parties to sell goods and
and use the internet to communicate with services to one another.
each other. Amazon.com provides websites
for the exchange of information on a wide
range of subjects relating to their portfolio
of products and services.

The Chartered Accountant Student May 2021 19

Page 485
ENTERPRISE INFORMATION SYSTEMs
Models Definition Examples Government This allows consumers Services including land
to to provide feedback or searches, and vehicle
Consumer In this model, consumers The comparison Consumer ask information about ownership searches and
to Business set prices and companies of interest rates of (G2C) government authority from disputes such as non-
(C2B) bid to offer products and personal loan/car public sector. The aim is to payment of tax or tax
services. loan provided by reduce the average time for refunds are resolved
various banks via fulfilling citizen’s requests through online support
websites. A business for various government on the government
organization who services. platforms. Example
fulfills the consumer's – e-Seva (Andhra
requirement within Pradesh)
the specified budget, Business to B2G model is a variant of Such websites are
approaches the Government B2B model. Such websites accredited by the
customer and provides (B2G) are used by governments government and
its services. Example- to trade and exchange provide a medium to
information with various businesses to submit
www.bankbazar.com
business organizations. application forms to
Consumer This covers all the www. incometaxindia. the government. For
example – Business pay
to e-commerce transaction gov.in
taxes, file reports, or sell
Government between consumers and
goods and services to
(C2G) government. Government agencies.

Components of E-Commerce

TECHNOLOGY INFRASTRUCTURE

Electronic Document Interchange


T
E
Electronic Fund Electronic Data v
C Transfer (EFT) Interchange (EDI)
E
H Information Marketing,
Sharing Advertising N
N Corporate Electronic D
Digital E-Commerce
O Publishing
Library Collaborative Sales, Customer O
L Work Support
R
O Email Fax
S
G
Y Electronic Messaging

USERS/INTERNET/WEB PORTAL /MOBILE APPS

• User: The user may be individual / organization or anybody using the e-commerce platforms.
• E-Commerce Vendors: This is the organization / entity providing the user, goods/ services asked for. E-commerce vendors
further needs to ensure Supply Chain Management, Warehouse operations, shipping and returns. E-commerce catalogue and
product delivery, Marketing and loyalty programs, Showroom and offline purchase, different ordering methods, guarantees,
privacy policy and security for better, effective and efficient transaction.
• Technology Infrastructure: E-commerce is technology driven. The technology used in e-commerce should be scalable, easy to
use and convenient for customers. The components of technology infrastructure may include computer, servers and database,
mobile apps, digital library and data interchange.
• Internet/Network: This is the critical enabler for e-commerce. The faster net connectivity leads to better e-commerce. The
success of e-commerce trade depends upon the internet capability of organization.
• Web Portal: This shall provide the interface through which an individual/organization shall perform e-commerce transactions.
• Payment Gateway: Payment gateway represents the way e-commerce/m-commerce vendors collects their payments. It is a
system of computer processes that authorizes, verifies, and accepts or declines credit/debit card transactions on behalf of the
merchant through secure Internet connections.

20 May 2021 The Chartered Accountant Student

Page 486
ENTERPRISE INFORMATION SYSTEMs
Architecture of Networked Systems
Architecture is a term to define the style of design and method of construction, used in general, for buildings and other physical structures.
In e-commerce, it denotes the way network architectures are build.
Advantages Two-Tier Architecture Disadvantages
• The system • Performance deteriorates if
performance is number of users increases.
higher because Client Applications • There is restricted flexibility
business logic and choice of DBMS since
and database are data language used in the
physically close. Two Tier Architecture server is proprietary to each
• More users could Data vendor.
interact with
system.
• It is easy to setup
and maintain
entire system
smoothly.

Database Tier

Presentation Tier

• Presentation Tier (Client Application/Client Tier): This is the interface that


allows user to interact with the e-commerce / m-commerce vendor.
• Database Tier (Data Tier): The product data / price data / customer data and
other related data are kept here.
Advantages Three Tier Architecture Disadvantages
• Clear separation • Increased need for network
of user-interface- Presentation Tier traffic management, server
control and data load balancing, and fault
presentation from tolerance.
application-logic. • Current tools relatively
• Dynamic load immature and more complex.
balancing possible • Maintenance tools currently
if bottlenecks inadequate.
in terms of Client(s) Desktop Laptop Workstation
performance
occurs.
• C h a n g e
management is
easy and faster. Application Tier

Server

Database
Tier Database Database

• Presentation Tier: Occupies the top level and displays information related to
services available on a website.
• Application Tier: Also, called the Middle Tier, Logic Tier, Business Logic
or Logic Tier; it controls application functionality by performing detailed
processing.
• Database Tier: This tier houses the database servers where information is
stored and retrieved.

The Chartered Accountant Student May 2021 21

Page 487
ENTERPRISE INFORMATION SYSTEMs
E-Commerce Architecture Vide Internet
Shopping Cart Credit Cart is charged
Order placed by
User

Order is
completed
Client End/User

Shipment
sent to
Customer Application Layers

Shopping carrier picks


up shipment Sent to Warehouse for fulfillment Email is sent to Customer
& Merchant

Database layers

• Client / User Interface: This layer e-commerce connects to help the customer e-commerce merchant.
• Application Layer: Through these application’s customer logs to merchant systems. This layer allows customer to check the
products available on merchant’s website.
• Database Layer: This layer is accessible to user through application layer.

E-Commerce Architecture Vide Mobile Apps


M-Commerce (Mobile Commerce) is the buying and selling of goods and services through wireless handheld devices such as
cellular telephone and personal digital assistants (PDAs). M-commerce enables users to access the Internet without needing to find
a place to plug in. The layers of e-commerce architecture vide internet is similar to architecture vide Mobile Apps except client/
user interface includes mobile web browser and internet in later.

Risk is possibility of loss. The same may be result of intentional or un-intentional action by individuals. Risks associated with e-commerce
transactions are high compared to general internet activities. These include the following:

Needs access
Delay in goods Security and
Privacy and Quality to internet and
and Hidden Credit Card
Security issues lack of personal
Costs issues
touch

Lack of Data Loss


Problem of Repudiation of
Infrastructure authenticity of or theft or
anonymity contract
transactions duplication

Non-recognition
Attack from Denial of Lack of audit
of electronic
hackers Service trails
transactions

Problem of
piracy

22 May 2021 The Chartered Accountant Student

Page 488
ENTERPRISE INFORMATION SYSTEMs
Controls are necessary for all persons in an e-business environment. These include the following:

Sellers/Buyers/ Merchants: Must have Government: To


Users: To ensure
proper framweork in place to ensure ensure all products/
genuinity of user.
success of business. services sold are legal.

Technology Service Providers: To


Network Service Providers:
ensure availaibility and security of
To ensure availaibility and
other service providers other than
security of network.
network service providers.

Payment Gateways: To ensure Logistics Service Providers:


that payment gateways are Repsonsible for timely prodct
efficient, effective and foolproof. deliveries.

Guidelines for E-Commerce: All entity going for e-commerce /


m-commerce business needs to create clear policy guidelines for the
following:
Commercial Laws Governing E-Commerce
Billing Income Tax Act, 1961 Companies Act, 2013
Foreign Trade Act, 1992 The Factories Act, 1948
The issues are - Format of bill, the details to be shared in The Customs Act, 1962 The Goods and Services Tax
bills, applicable GST. Act, 2017
Indian Contract Act,1872 The Competition Act,
2002
Foreign Exchange Consumer Protection Act,
Product guarantee/ warranty Management Act, 1999 1986

Proper display of product guarantee / warranty online as


well as documents sent along with the products.
Special Laws governing E-Commerce
I. Information Technology Act, 2000
Shipping The objectives of IT Act, 2000 can be summarized as follows:
 To grant legal recognition for transactions carried out by means
of electronic data interchange or electronic commerce in place of
The shipping time, frequency of shipping, the packing at time
paper based method of communication.
of shipping, all these needs to be put in policy documents.
 To give legal recognition to digital signature for authentication of
any information or matter, which requires authentication under
any law.
Delivery  To facilitate electronic filing of documents with Government
departments.
 To facilitate electronic storage of data.
Policy needs to be defined for which mode of delivery to be
chosen, when deliveries to be made and where deliveries to  To provide legal sanction to transfer fund electronically to and
be made? between banks and financial institutions.
 To provide legal recognition for keeping books of account in
electronic format by bankers.
 In order to amend the Indian Penal Code, Indian Evidence Act,
Return 1972, Bankers Book Evidence Act, 1891 and RBI Act, 1934.
 To provide legal infrastructure to promote e-commerce and
Policy for return of goods needs to be put in place secure information system.
defining:Which goods to be accepted in return, and the  To manage cyber-crimes at national and international levels by
number of days within which returns can be accepted. enforcing laws.
 This Act governs all internet activities in India and is applicable
to all online transactions in India, and provides for penalties,
Payment prosecution for non-compliances.
II. Reserve Bank of India Act, 1934
Policy guidelines need to be created for the issues related to Reserve Bank of India (RBI), from time to time, frames guidelines
mode of payment and for which product specific payment to be followed by e-commerce / m-commerce merchants allowing
shall be made. online payments through various modes.

The Chartered Accountant Student May 2021 23

Page 489
ENTERPRISE INFORMATION SYSTEMs
Digital Payment is a way of payment which is made through digital New Methods of Digital Payment
modes. In digital payments, payer and payee both use digital modes
to send and receive money. It is also called electronic payment. No • Unified Payment Interface (UPI) is a payment
hard cash is involved in the digital payments. UPI mode which is used to make instant fund transfers
Apps from the sender’s bank account to the receiver’s bank
Advantages of Digital Drawbacks of Digital account through the mobile app.
Payment Payment
• Easy and convenient • Difficult for a Non-technical • Immediate Payment Service (IMPS) is an instant
interbank electronic fund transfer service through
• Pay or send money from person IMPS
mobile phones.
anywhere • Risk of data theft
• Discounts from taxes • Overspending • Mobile Apps such as Bharat Interface for Money
• Written record • Disputed transactions Mobile (BHIM) is a Mobile App that facilitates e-payments
• Less Risk • Increased business costs Apps directly through banks and supports all Indian banks
• The necessity of internet access which use that platform.

Traditional Methods of Digital Payment • A mobile wallet is a type of virtual wallet service that
I. Cards are provided by banks to their account holders. These have Mobile can be used by downloading an app on smartphone
Wallet and registering for the service. There are mobile
been the most used digital payment modes till now. Various types
wallets like Paytm, Freecharge, Buddy, Mobikwik,
of cards are as follows: State bank buddy etc.
• Credit Cards: A small plastic card issued by a bank, or issuer etc.,
allowing the holder to purchase goods or services on credit. Credit • Aadhar Enabled Payment Service (AEPS) is an
Card issuer charge customers per transactions / fixed amount as Aadhaar based digital payment mode. AEPS allows
AEPS bank to bank transactions. Customers will need to
transaction fees.
link their AADHAR numbers to their bank accounts.
• Debits Cards: It is also a small plastic card with a unique number
linked with bank account number. It is required to have a bank
• Unstructured Supplementary Service Data
account before getting debit card from bank. It enables cardholder (USSD) is a revolutionary idea, where to make
to pay for his/her purchases directly through his/her account. USSD payments through mobiles; there is neither any need
• Smart Cards: Smart card is a prepaid card that has a small for internet nor any smart phone.
microprocessor chip embedded in it. It has capacity to store
customer’s personal information such as financial facts, private • Mobile banking is a service provided by a bank or
encryption keys, credit card information, account information, Mobile other financial institution that allows its customers
Banking to conduct different types of financial transactions
and so on.
remotely using a mobile device such as a mobile phone
II.Net Banking: In this mode, the customers log to his / her bank or tablet.
account and makes payments. All public sectors, large private
sector banks allow net banking facilities to their customers. • A cryptocurrency is a medium of exchange wherein
Crypto- records of individual coin ownership are stored in a
currency computerized database using strong cryptography.

tion Mobile Cloud


ua liz a Computing Computing
Vir t

Grid Mac
Computing
Computing Le a hi ne
rnin
Technologies g

Web 3.0
ur O wn
Bring Yo YOD)
D e vice (B
Ar t
Inte if ic i al Green
llige
nce Computing

24 May 2021 The Chartered Accountant Student

Page 490
ENTERPRISE INFORMATION SYSTEMs
I. Virtualization III. Cloud Computing: Cloud Computing is both, a combination
of software and hardware-based computing resources delivered
Virtualization means to create Application Areas as a networked service. This model of IT enabled services enables
a virtual version of a device • Server Consolidation anytime access to a shared pool of applications and resources.
or resource, such as a server, • Disaster Recovery These applications and resources can be accessed using a simple
storage device, network or even • Testing and Training front-end interface such as a Web browser, and thus enabling users
an operating system where the • Portable Applications to access the resources from any client device including notebooks,
framework divides the resource • Portable Workspaces desktops and mobile devices.
into one or more execution
environments. This refers to Characteristics Advantages
technologies designed to provide • Elasticity & Scalability • Achiever economies of scale
a layer of abstraction between • Pay-Per-Use • Reduce spending on technology
computer hardware systems and • On-demand infrastructure
the software running on them. • Resiliency • Globalize the workforce
• Multi-Tenancy • Streamline business processes
Types of Virtualization
• Workload Movement • Reduce capital costs
Hardware Network Storage • Pervasive accessibility
Virtualization Virtualization Virtualization • Monitor projects more effectively
• Less personnel training is needed
This refers to the It is a method of It is the apparent
• Minimize maintenance &
creation of a virtual combining the pooling of data from
licensing software
machine that acts available resources multiple storage
• Improved flexibility
like a real computer in a network devices, even different
with an operating by splitting up types of storage Types of Cloud
system. The basic the available devices, into what
Private Public Cloud Community Hybrid
idea of Hardware bandwidth into appears to be a single
Cloud Cloud Cloud
virtualization is to channels, each device that is managed
consolidate many of which is from a central It resides It is the cloud It is the cloud This is a
small physical independent from console. It helps the within the infrastructure infrastructure combination
servers into one the others, and storage administrator boundaries that is that is of both at least
large physical server each of which can perform the tasks of of an provisioned provisioned for one private
so that the processor be assigned (or backup, archiving, and organization for open use exclusive use (internal)
can be used more reassigned) to a recovery more easily and is used by the general by a specific and at least
effectively. For particular server or and in less time by exclusively public. It may community of one public
example, a computer device in real time. disguising the actual for the be owned, consumers from (external)
that is running It is intended to complexity of a Storage organization’s managed, and organizations c l o u d
Microsoft Windows optimize network Area Network (SAN). benefits. operated by that have shared computing
may host a virtual speed, reliability, P r i v a t e a business, concerns (e.g. environments
machine that looks flexibility, Clouds can academic, or mission security - usually,
like a computer with scalability, and either be government requirements , consisting of
the Linux operating security. private to the organizations, policy, and infrastructure,
system; based organization or some compliance platforms and
software that can be and managed combination of considerations). applications.
run on the virtual by the single them. It may be owned, The usual
machine. organization Ty p i c a l l y, managed, and method of
(On-Premise p u b l i c operated by one using the
P r i v a t e clouds are or more of the hybrid cloud
II. Grid Computing: It is a computer network in which each Cloud) or can administrated organizations in is to have a
computer’s resources are shared with every other computer in be managed by third parties the community, private cloud
the system. It is a distributed architecture of large numbers of by third party or vendors a third party initially, and
computers connected to solve a complex problem. In the grid (Outsourced over the or some then for
computing model, servers or personal computers run independent P r i v a t e Internet, and combination of additional
tasks and are loosely linked by the Internet or low-speed networks. Cloud). the services are them, and it may resources, the
Benefits Types of Resources Security offered on pay- exist on or off public cloud is
per-use basis. premises. used.
• Making use of • Computation. • Single Sign-on.
Cloud Computing Service Models
Underutilized • Storage. • Protection of
Resources. • Communications Credentials. Infrastructure as a Platform as a Software as a Service
• Resource Balancing. • Software and • Interoperability Service (IaaS) Service (PaaS) (SaaS)
• Parallel CPU Licenses. with local
Capacity. • Special security solutions. IaaS, a hardware- PaaS provides SaaS provides ability to
• Access to additional equipment, • Exportability level service, provides the users the the end users to access
resources. capacities, • Support for computing resources ability to develop an application over the
• Virtual resources and architectures, secure group such as processing and deploy an Internet that is hosted
virtual organizations and policies. communication. power, memory, application on and managed by the
for collaboration. • Support for storage, and networks the development service provider.
• Reliability. multiple for cloud users to run p l a t f o r m
• Management. implementations. their application on- provided by the
demand. service provider.

The Chartered Accountant Student May 2021 25

Page 491
ENTERPRISE INFORMATION SYSTEMs
IV. Mobile Computing: This refers to technology that allows VI. Web 3.0 Technology
transmission of data via a computer without having to be • Known as the Semantic Web, this describes sites wherein the
connected to a fixed physical link.
computers will be generated raw data on their own without
Components Limitations Benefits direct user interaction.
• Mobile Commu- • Insufficient • Mobile workforce • Web 3.0 standard uses semantic web technology, drag and
nication: Refers to Bandwidth with remote access drop mash-ups, widgets, user behavior, user engagement,
infrastructure put in • Security to work order
place to ensure that Standards details. and consolidation of dynamic web contents depending on the
seamless and reliable • Power • Enables mobile sales interest of the individual users.
communication goes consumption personnel to update • Web 3.0 Technology uses the “Data Web” Technology, which
on. • Transmission work order status in features the data records that are publishable and reusable on
• Mobile Hardware: interferences real-time. the web through query-able formats. The Web 3.0 standard
This includes mobile • Potential • Facilitates access to also incorporates the latest researches in the field of artificial
devices/device health corporate services intelligence.
components that hazards and information at
range from Portable • Human any time. Web 3.0 Components
laptops, Smart interface with • Provides remote
Phones, Tablet PCs, device. access to the
and Personal Digital corporate knowledge
Assistants (PDA). base at job location.
• Mobile Software: • Enables to improve Semantic Web Web Services
It is the actual management This provides the web user It is a software system
programme that effectiveness a common framework that that supports computer -
runs on the mobile by enhancing could be used to share and to - computer interaction
hardware and information quality, reuse the data across various over the Internet. For
deals with the information flow, applications, enterprises, example – photo sharing
characteristics and and ability to control
and community boundaries. website.
requirements of a mobile workforce.
mobile applications.
Example: The application that uses content management systems
along with artificial intelligence. This helps to achieve a more
IV. Green Computing: Green computing or Green IT refers to
connected open and intelligent web applications using concepts of
the study and practice of environmentally sustainable computing
or IT. In other words, it is the study and practice of establishing natural language processing machine learning, machine reasoning
/ using computers and IT resources in a more efficient and and autonomous agents.
environmentally friendly and responsible way.
Best • Develop a sustainable Green Computing plan
Practices • Recycle System of interrelated computing
• Make environmentally sound purchase decisions devices, mechanical and digital
• Reduce Paper Consumption machines, objects, animals or people that
• Conserve Energy are provided with unique identifiers.

V. BYOD (Bring Your Own Device): This refers to business


policy that allows employees to use their preferred computing Risks Ability to transfer
devices, like smart phones and laptops for business purposes. It To product manufacturer VII. data over a network
means employees are welcome to use personal devices (laptops, To user of these products Internet without requiring
smart phones, tablets etc.) to connect to the corporate network to of Things human-to-human or
Technology Risk human-to-computer
access information and application. (IoT)
Environmental Risk interaction.
Advantages Emerging BYOD Threats
 Happy • Network Risks: It is normally exemplified and
Employees hidden in ‘Lack of Device Visibility’. As BYOD
permits employees to carry their own devices Application Areas
 Lower IT
(smart phones, laptops for business use), the Home Appliances
budgets
IT practice team is unaware about the number Office Machines
 IT reduces of devices being connected to the network. As Smart Gids
support network visibility is of high importance, this Smart Supply chain
requirement lack of visibility can be hazardous.
 Early • Device Risks: It is normally exemplified and
adoption hidden in ‘Loss of Devices’. A lost or stolen VIII. Artificial Intelligence may be defined as the ability to
of new device can result in an enormous financial use memory, knowledge, experience, understanding, reasoning,
Technologies and reputational embarrassment to an imagination and judgement to solve problems and adapt to new
 Increased organization as the device may hold sensitive situations. Applications Areas include Medical diagnosis, in
employee corporate information. cancer research. Predicting the chances of an individual getting ill
efficiency • Application Risks: It is normally exemplified by a disease; Creating art such as poetry; Proving mathematical
and hidden in ‘Application Viruses and theorems; Playing games (such as Chess or Go) and predicting the
Malware’. Organizations are not clear in outcomes etc.
deciding that ‘who is responsible for device
security – the organization or the user’. IX. Machine Learning is a type of Artificial Intelligence (AI) that
• Implementation Risks: It is normally provides computers with the ability to learn without being explicitly
exemplified and hidden in ‘Weak BYOD programmed. Machine learning focuses on the development of
Policy’. The effective implementation of computer programs that can change when exposed to new data.
the BYOD program should not only cover The process of machine learning is similar to that of data mining.
technical issues mentioned above but For example: Machine learning has been used for image, video,
also mandate the development of a robust and text recognition, as well as serving as the power behind
implementation policy. recommendation engines.

26 May 2021 The Chartered Accountant Student

Page 492
ENTERPRISE INFORMATION SYSTEMs
CHAPTER 5: CORE BANKING SYSTEMS
This chapter deals with components and architecture of Guarantees
Core Banking Systems (CBS) and impact of related risks and • These are required by the customers of banks for submission
controls, and further discusses the functioning of core modules to the buyers of their goods/services to guarantee performance
of banking and business process flow. The chapter also provides of contractual obligations undertaken by them or satisfactory
a detailed understanding on the regulatory and compliance performance of goods supplied by them, or for submission
requirements applicable to CBS such as Banking Regulations to certain departments like excise and customs, electricity
Act, RBI regulations, Prevention of Money Laundering Act and boards, or to suppliers of goods, etc. in lieu of the stipulated
Information Technology Act. security deposit.
Credit Cards
Banking is the engine of economic growth specifically
in a rapidly developing country like India with its diverse • Most credit cards issued by banks are linked to one of the
background, practices, cultures, and large geographic international credit card networks like VISA, Master etc.
dispersion of citizens. The core of banking functions is Debit Cards
acceptance of deposits and lending of money. Further, specific
services such as demand drafts, bank guarantees, letter of • Debit Cards facilitates customers to pay at any authorized
credits, etc. are also provided. The key features of a banking outlet as well as to withdraw money from an ATM from their
business are as follows: account.
• The custody of large volumes of monetary items, including Other Banking Services
cash and negotiable instruments, whose physical security
should be ensured. • These include Back operations, Retail Banking, High Net-
• Dealing in large volume (in number, value and variety) of worth Individuals (HNI), Risk Management and Specialized
transactions. Services such as insurance broking, claims, underwriting, life
• Operating through a wide network of branches and insurance, non-life insurance, etc.
departments, which are geographically dispersed.
• Increased possibility of frauds as banks directly deal with
Key Modules of Core Banking Systems (CBS)
money making it mandatory for banks to provide multi-
point authentication checks and the highest level of Back End Applications
information security.
• Increased possibility of frauds as banks directly deal with • Back Office: Includes settlements, clearances, record
money making it mandatory for banks to provide multi- maintenance, regulatory compliance, accounting and IT
point authentication checks and the highest level of services.
information security. • Data Warehouse: Takes care of the difficult data management
- digesting large quantities of data and ensuring accuracy.
Major products and services provided and • Credit Card System: Provides customer management,
rendered by commercial banks credit card management, account management, customer
information management and general ledger functions; online
Acceptance of Deposits transaction authorization and service of the bank card in
• Commercial banks accept deposits in various forms such each transaction channel of the issuing bank; support in the
as term deposits, savings bank deposits, current account payment application.
deposits, recurring deposits, saving-cum-term deposits and • ATM Switch: Is convenient, allowing consumers to perform
various others innovative products. quick, self-serve transactions from everyday banking like
deposits and withdrawals to more complex transactions like
Granting of Advances bill payments and transfers.
• Advances constitute a major source of lending by commercial
banks. The type of advances granted by commercial banks Central Server
take various forms such as cash credit, overdrafts, purchase/ • All the bank's branches access applications from
discounting of bills, term loans, etc. centralized data centers/ servers, therefore any deposits made
Remittances in any branch are reflected immediately and customer can
withdraw money from any other branch throughout the world.
• Involves transfer of funds from one place to another. Two
of most common modes of remittance of funds are demand Front End Applications
drafts & Telegraphic/ Mail Transfers (TT/ MT). • Mobile Banking: A service provided by a bank or other
financial institution that allows its customers to conduct
Collections financial transactions remotely using a mobile device such as a
• Collections involve collecting proceeds on behalf of the smartphone or tablet.
customer. Customers can lodge various instruments such as • Internet Banking: An electronic payment system that
cheques, drafts, pay orders, travelers’ cheques, dividend and enables customers of a bank or other financial institution to
interest warrants, tax refund orders, etc. conduct a range of financial transactions through the financial
institution's website accessed through any browser.
Clearing • Phone Banking: A functionality through which customers can
execute many of the banking transactional services through
• This involves collecting instruments on behalf of customers of Contact Centre of a bank over phone, without the need to visit
bank. a bank branch or ATM.
Letters of Credit (LC) • Branch Banking: Creating manual documents capturing
data required for input into software; Internal authorization;
• It is an undertaking by a bank to the payee to pay to him, on Initiating Beginning-Of-Day (BOD) operations; End-Of-Day
behalf of the applicant any amount up to the limit specified in (EOD) operations; and Reviewing reports for control and
the LC, provided the terms and conditions mentioned in the error correction.
LC are complied with.

The Chartered Accountant Student May 2021 27

Page 493
ENTERPRISE INFORMATION SYSTEMs
CBS IT Environment Internet Banking IBCS software stores the name and password
The CBS facilities providing banking services for branches of Channel Server of the entire internet banking customers.
a bank which are networked and connected to common data (IBCS) IBCS server also contains the details about
center. This facilitates staff to process transactions of customers the branch to which the customer belongs.
of any branch. The Server is a sophisticated computer that accepts Internet Banking The Internet Banking Software which is
service requests from different machines called clients. The Application stored in IBAS authenticates customer with
requests are processed by the server and sent back to the clients. Server (IBAS) the login details stored in IBCS.
There are different types of servers used in deploying CBS which Web Server It is used to host all web services and
are as follows: internet related software. Web server is
SERVER FUNCTIONING a program that uses HTTP (Hypertext
Application The application software, resides in the Transfer Protocol) to serve the files that
Server application server and is always the latest form Web pages to users, in response to
version as accepted after adequate testing. their requests, which are forwarded by their
computers’ HTTP clients.
Database Server The Database Server of Bank contains entire
data of Bank which would consist of various Proxy Server It is a computer that offers a computer
accounts of customers & master data. network service to allow clients to make
indirect network connections to other
ATM Channel This server contains the details of ATM network services.
Server account holders. Soon after the facility of
using the ATM is created by the Bank, the Anti-Virus It is used to host anti-virus s/w which is
details of such customers are loaded on to Software Server deployed for ensuring all the s/w deployed
the ATM server. are first scanned to ensure that appropriate
virus/ malware scans are performed.

e-Commerce Transaction flow for approval of payments

Payment Gateway
Order Request to Payment
Gateway

Server Response Payment Gateway


User Placing Merchant’s Web Response
Order Server

Bank Response

Confirmation
Funds Transferred Request for
to Your Account
Merchant

Bank

CBS Implementation
The deployment and implementation of CBS should be controlled at various stages to ensure that banks' automation objectives are achieved.

Planning
Audit
Approval
Updation
Implementation Selection
Support of CBS
Design and Develop/Procure

Maintenance
Testing
Implementation

28 May 2021 The Chartered Accountant Student

Page 494
ENTERPRISE INFORMATION SYSTEMs
Planning Operational Risk is defined Credit Risk is the risk that
as a risk arising from direct an asset or a loan becomes
• Implementation of CBS should be done as per strategic and or indirect loss to the bank irrecoverable in the case of
business objectives of bank. which could be associted with outright default, or the risk
inadequate or failed internal of an unexpected delay in the
process, people and systems. servicing of a loan.
Approval

• The decision to implement CBS must be approved by the Risks associated


Board of Directors as high investment and recurring costs with CBS
are involved. Strategic Risk is defined as the Market Risk refers to the risk
risk that earnings decline due to of losses in the bank's trading
changing business environment book due to changes in equity
Selection or changing demand of prices, interest rates, credit
customers. spreads, commodity prices and
• Bank should select the right solution considering various
other indicators whose values
parameters as defined by the bank to meet their specific
are set in a public market.
requirements and business objectives.

Design and develop or procured Information Technology Risks


• Currently, most of the CBS deployment are procured. Ownership of Data/ Process
There should be appropriate controls covering the design or • Establishes clear ownership so that accountability can
development or procurement of CBS for the bank. be fixed.

Authorization process
Testing • Anybody with access to the CBS, including the
customer himself, can enter data directly. Thus needs
• The testing is to be done at different phases at procurement to have robust authorization process.
stage to test suitability to data migration to ensure all existing
data is correctly migrated and testing to confirm processing
Authentication procedures
of various types of transactions of all modules produces the
correct results. • Usernames and passwords, PIN, OTP are some
of the authentication techniques. If inadequately
implemented, the user entering the transaction may
Implementation not be determinable or traceable.

• CBS must be implemented as per pre-defined and agreed Several software interfaces across diverse networks
plan with specific project milestones to ensure successful • As a Data Centre can have as many as 75-100 different
implementation. interfaces and application software; it must contain
adequate infrastructure, such as power distribution
and supplemental power subsystems and so on.
Maintenance
Maintaining response time
• CBS must be maintained as required. E.g. program bugs • Maintaining the interfacing software and ensuring
fixed, version changes implemented, etc. optimum response time and up time can be challenging.

User Identity Management


Support • This could be a serious issue. Some Banks may have
more than 5000 users interacting with the CBS at once.
• CBS must be supported to ensure that it is working
effectively. Access Controls
• Bank environments are subject to all types of attacks;
thus, a strong access control system is a crucial part of
Updation
a bank’s overall security plan.
• CBS modules must be updated based on requirements of
business processes, technology updates and regulatory Incident handling procedures
requirements.
• Incident handling procedures are used to address
and manage the aftermath of a security breach or
cyberattack. These may not be adequate considering
Audit the need for real-time risk management.
• Audit of CBS must be done internally and externally as
Change Management
required to ensure that controls are working as envisaged.
• This reduces the risk that a new system or other change
will be rejected by the users.

The Chartered Accountant Student May 2021 29

Page 495
ENTERPRISE INFORMATION SYSTEMs
Three stages of Money Prevention of Money
Information Security is critical to mitigate the risks Laundering Act (PMLA)
of Information technology. Security refers to ensure Laundering
 CHAPTER II Offence Of
Confidentiality, Integrity and Availability of information. RBI • Placement: Involves the
Placement of proceeds Money-laundering
has suggested use of ISO 27001: 2013 implement information • Section 3. Offence of money-
derived from illegal activities
security. Banks are also advised to obtain ISO 27001 – the movement of proceeds, laundering
Certification. Many banks have obtained such certification  Chapter IV Obligations
frequently currency, from
for their data centers. the scene of the crime to Of Banking Companies,
a place, or into a form, Financial Institutions And
Information security is comprised of the following sub-
less suspicious and more Intermediaries
processes: • Section 12. Reporting entity
convenient for the criminal.
• Layering: Involves the to maintain records.
• Information Security Policies, Procedures and separation of proceeds from • Section 13. Powers of
practices: This refers to the processes relating to approval illegal source using complex Director to impose fine.
and implementation of information security.  CHAPTER X
transactions designed to
obscure the audit trail and Miscellaneous
• User Security Administration: This refers to security
hide the proceeds. • Section 63. Punishment for
for various users of information systems. The security
• Integration: Involves false information or failure
administration policy documents define how users
conversion of illegal to give information, etc.
are created and granted access as per organization
proceeds into apparently • Section 70. Offences by
structure and access matrix. It also covers the complete
legitimate business earnings companies
administration of users right.
• Application Security: This refers to how security is through normal financial or
implemented at various aspects of application right from commercial operations.
configuration, setting of parameters and security for
transactions through various application controls. Money Laundering is the process by which the
• Database Security: This refers to various aspects of proceeds of the crime and the true ownership of those
implementing security for the database software. For proceeds are concealed or made opaque so that the
example - Role based access privileges given to employees. proceeds appear to come from a legitimate source.
• Operating System Security: This refers to security for
operating system software which is installed in the servers Anti-Money Laundering Financing of Terrorism
and systems which are connected to the servers. (AML) using Technology Terrorist financing is a form of
• Network Security: This refers to how security is provided Banks face the challenge of money laundering. Money to
at various layers of network and connectivity to the addressing the threat of money fund terrorist activities moves
servers. For example - Use of virtual private networks for laundering on multiple fronts through the global financial
employees, implementation of firewalls etc. as banks can be used as primary system via wire transfers and
• Physical Security: This refers to security implemented means for transfer of money in and out of personal and
through physical access controls. For example - Disabling across geographies. business accounts.
the USB ports. With regulators adopting It can sit in the accounts of
stricter regulations on illegitimate charities and be
banks and enhancing their laundered through buying and
enforcement efforts, banks selling securities and other
are using special fraud and commodities or purchasing and
Current risk management software to cashing out insurance policies.
& Savings prevent and detect fraud and
Account integrate this as part of their
(CASA) internal process and daily
processing and reporting.
Internet
Credit Cards
Banking
Banking Regulations Acts
CBS: Core • Negotiable Instruments Act-1881 (NI Act) Under NI Act,
Business Cheque includes electronic image of truncated cheque and a
Processes cheque in the electronic form. The truncation of cheques in
clearing has been given effect to and appropriate safeguards in
this regard have been set forth in the guidelines issued by RBI
Loans and
Mortgages from time to time.
Trade Finance
• The Reserve Bank of India (RBI) was established on April
1, 1935 in accordance with the provisions of the Reserve
Bank of India Act, 1934. The basic functions of the Reserve
Treasury Bank as: "to regulate the issue of Bank Notes and keeping of
reserves with a view to securing monetary stability in India
and generally to operate the currency and credit system of the
country to its advantage."

30 May 2021 The Chartered Accountant Student

Page 496
ENTERPRISE INFORMATION SYSTEMs
Information Technology (IT) Act Key provisions of IT related offences that impact banks

Section 43 Section 65: Section Section 66-B: Section Section


provides Tampering 66: Punishment 66-C: 66-E:
for Penalty with Computer for dishonestly Punishment Punishment
The Information Technology Act was passed in 2000, and Computer Related receiving for identity for
amended in 2008 and the Rules were passed in 2011. compen- Source Offences stolen theft violation
• The Act provides legal recognition for transactions carried sation for Documents computer of privacy
damage to resource or
out by means of electronic data interchange and other means computer, communication
of electronic communication, commonly referred to as computer device
"electronic commerce", which involve the use of alternatives system,
to paper-based methods of communication and storage of etc.
information, to facilitate electronic filing of documents with
the Government.
• The Act provides the legal framework for electronic Sensitive Personal Data Information (SPDI)
governance by giving recognition to electronic records The IT Act has a specific category, “Sensitive Personal Data or
and digital signatures. It also deals with cybercrime and Information,” which consists of password, financial information
facilitates electronic commerce. It also defined cyber-crimes (including bank account, credit card, debit card or other payment
and prescribed penalties for them. details), physical, physiological and mental health conditions,
• The Amendment Act 2008 provides stronger privacy data sexual orientation, medical records, and biometric information.
protection measures as well as implementing reasonable This legally obligates all stakeholders (i.e., any individual or
information security by implementing ISO 27001 or organization that collects, processes, transmits, transfers, stores
equivalent certifiable standards to protect against cyber- or deals with sensitive personal data) to adhere to its requirements.
crimes.
Privacy Policy
Every bank captures Personal Information of customers as per
definition of IT Act. Hence, it is mandatory to ensure security
of personal information. This information must be protected by
maintaining physical, electronic, and procedural safeguards by
using appropriate security standards such as ISO 27001 to ensure
compliance with regulatory requirements.

“Successful and unsuccessful people do not vary greatly in their abilities. They vary in
their desires to reach their potential.”
– John Maxwell

CROSSWORD SOLUTION – april 2021


1
S 2
T 3
O C 4
K 5
E 6
C O 7
N 8
O 9
M 10
Y
11
T R A N S F E R R N O I
12
R A T 13
B F I R I A N E
14
E D S 15
P 16
M E L
17
N E 18
M I 19
G 20
T F 21
C Y D

G 22
A I 23
A 24
P I E
25
T 26
R A N S A C T 27
I O 28
N 29
A 30
L

H S P O P 31
L T 32
C I I
33
S A R 34
T E 35
A R M E

O 36
P P N
37
I N 38
T R O 39
D U C T I 40
O 41
N
42
B I T O 43
N O
44
P 45
R O T 46
O 47
C 48
O L 49
T 50
B
51
C 52
C 53
C A S H C O W 54
S O
55
I A 56
M R M 57
R 58
I L
59
D A T A B A S E 60
C O S T

The Chartered Accountant Student May 2021 31

Page 497
business and commercial knowledge
ca foundation - paper 4B - BUSINESS AND COMMERCIAL KNOWLEDGE
This capsule on Foundation Paper 4 part II: Business and Commercial Knowledge broadly covers the companies
discussed in detail in Chapter 3 of the Study Material. To facilitate easy understanding of the significant changes in
the year 2020, an attempt has been made to give an overview of the significant changes in the companies in tabulated
form.
It may be kept in mind that the capsule is not the replacement of the Study Material. Reading of Study Material is
absolute essential. This capsule is intended to assist you in the process of quick revision.

A. AN OVERVIEW OF SELECTED INDIAN COMPANIES


S. Company’s Name Incorpo- Headquarters Chairman Present Chief Chief Ranking Ranking
No ration Managing Executive Financial in Forbes in Forbes
year Director Officer Officer World’s World’s
Largest Public Best
Corporations Employer’s
List 2020 List 2020
1 Adani Ports 1998 Ahmedabad, Gautambhai Karan Deepak 1754th
and Special Gujarat, Shantilal Gautambhai Maheshwari
Economic India Adani Adani
Zone Ltd.
2 Asian Paints 1942 Mumbai, Amit Syngle Amit Syngle RJ 1596th
Ltd. Maharashtra, Jeyamurugan
India
3 Axis Bank 1993 Mumbai, Amitabh Amitabh Puneet 727th 392nd
Ltd. Maharashtra, Chaudhry Chaudhry Sharma
India
4 Bajaj Auto 1945 Pune, Rahul Bajaj Rajiv Bajaj Rajiv Bajaj Soumen Ray 1531st
Ltd. Maharashtra,
India
5 Bharti Airtel 1995 New Delhi, India Sunil Bharti Gopal Vittal Gopal Vittal Badal Bagri 700th
Ltd. Mittal

6 Bharat 1952 Mumbai, D Rajkumar K Padmakar K Padmakar Neelakantapillai 601st


Petroleum Maharashtra, Vijayagopal
Corporation India
Ltd.
7 Cipla Ltd. 1935 Mumbai, Y K Hamied Umang Vohra Umang Vohra Kedar
Maharashtra, Upadhye
India
8 Coal India 1975 Kolkata, Pramod Pramod Sanjiv Soni 612th
Ltd. West Bengal, Agrawal Agrawal
India
9 Dr. Reddy’s 1984 Hyderabad, Kallam Satish Erez Israeli Parag
Lab. Ltd. Telangana, Reddy Agarwal
India
10 Flipkart 2007 Singapore Kalyan Sriram
(legal domicile) Krishnamurthy Venkataraman
Bengaluru,
Karnataka, India
(Operational
Headquarters)
11 GAIL 1984 New Delhi, India Manoj Jain Manoj Jain Manoj Jain A K Tiwari 1257th 403rd
(India) Ltd

12 HDFC Bank 1994 Mumbai, Deepak S Sashidhar Srinivasan 146th 176th


Ltd. Maharashtra, Parek Jagdishan Vaidyanathan
India
13 ICICI Bank 1994 Mumbai, Girish Sandeep Sandeep Rakesh Jha 255th 523rd
Ltd. Maharashtra, Chandra Bakhshi Bakhshi
India Chaturvedi
14 Indian Oil 1959 New Delhi, India Shrikant Sandeep 389th 452nd
Corporation Madhav Kumar Gupta
Ltd. Vaidya
15 Infosys Ltd. 1981 Bengaluru, Nandan Salil Parekh Salil Parekh Sandeep 602nd
Karnataka, Nilekani Kumar Gupta
India

32 May 2021 The Chartered Accountant Student

Page 498
business and commercial knowledge
S. Company’s Name Incorpo- Headquarters Chairman Present Chief Chief Ranking Ranking
No ration Managing Executive Financial in Forbes in Forbes
year Director Officer Officer World’s World’s
Largest Public Best
Corporations Employer’s
List 2020 List 2020
16 ITC Ltd. 1910 Kolkata, Sanjiv Puri Sanjiv Puri Sanjiv Puri Supratim 907th 461st
West Bengal, Dutta
India
17 Larsen & 1938 Mumbai, Anil S.N. S.N. Shankar 443rd 193rd
Toubro Ltd. Maharashtra, Manibhai Subrahmanyan Subrahmanyan Raman
India Naik
18 NTPC Ltd. 1975 New Delhi, India Gurdeep Gurdeep Anil Kumar 497th 261st
Singh Singh Gautam

19 Oil & Natural 1956 Uttarakhand, Shashi Shashi Subhash 269th


Gas India Shankar Shankar Kumar
Corporation
Ltd.
20 Power Grid 1989 Gurugram, Sreekant Sreekant Mohammed 845th 305th
Corporation Haryana, Kandikuppa Kandikuppa Taj
of India Ltd. India Mukarrum
21 Reliance 1966 Mumbai, Mukesh Mukesh Mukesh Srikanth 58th 81st
Industries Maharashtra, Ambani Ambani Ambani Venkatchari
Ltd. India and Alok
Agarwal
22 State Bank 1806 Mumbai, Dinesh C.S. Setty, Charanjit 171st 252nd
of India Maharashtra, Kumar Khara Ashwani Bhatia, Surinder
India Swaminathan J., Singh Attra
Ashwini
Kumar Tewari
23 Tata Sons 1868 Bombay House, Natarajan Natarajan Eruch Noshir Tata Motors- Tata
Private Ltd. Mumbai, Chandrasekaran Chandrasekaran Kapadia 1037th Group –
Maharashtra, TCS – 375 th
310th
India Tata Steel –
828th
24 Wipro Ltd. 1945 Bengaluru, Rishad Premji Rishad Premji Thierry Jatin Dalal 904th
Karnataka, Delaporte
India
B. AN OVERVIEW OF SELECTED GLOBAL COMPANIES
S. Company’s Name Incorpo- Headquarters Chairman Chief Chief Ranking in Ranking Ranking
No ration Executive Financial Forbes World’s in Forbes in Fortune
year Officer Officer Largest Public World’s Best 500
Corporations Employer’s Companies
List 2020 List 2020 List 2020
1 Amazon 1994 Seattle, Jeff Bezos Jeff Bezos Brain T. 22nd 2nd 2nd
Washington, U.S. Olsavsky
2 American 1850 New York, United Stephen Stephen Jeffery C 88th 291st 67th
Express States of America Squeri Squeri Campbell
3 Apple 1977 California, United Tim Cook Luca Maestri 9th 6th 4th
States of America
4 Goldman 1869 New York, United David M. David M. Stephen 47th 60th
Sachs States of America Solomon Solomon Scherr
5 HP Inc. 1939 California, United Charles Enrique Marie Myers 432nd 681st 109th
States of America Victor Bergh Lores
6 IBM 1911 New York, United Arvind Arvind James J. 51st 3rd 38th
Corporation States of America Krishna Krishna Kovnaugh
7 Intel 1968 California, United Omar Ishrak Patrick Paul George Davis 38th 55th 45th
Corporation States of America Gelsinger
8 Microsoft 1975 Washington, John W. Satya Nadella Amy Hood 6th 4th 3rd
Corporation Thompson
9 Nestle 1866 Vevey, Switzerland Paul Bulcke Ulf Mark Francois- 41st 570th
Schneider Xavier Roger
10 Walmart 1969 Arkansas, United Gregory B. Dough Bret Biggs 19th 1st
States of America Penner McMillon
The Chartered Accountant Student May 2021 33

Page 499
toppers

FROM ICAI TO ALL THOSE WHO CLEARED


THE JANUARY 2021 CHARTERED ACCOUNTANCY EXAM
Toppers of CA Examination January 2021

final OLD Scheme

AIR-1 AIR-2 AIR-3

Bishal Timsina Noothula Naveen Mohammed Shabeeb P


Chennai Karimnagar Malappuram

final new Scheme

AIR-1 AIR-2

Bhramar Jain Vaibhav Hariharan


Raipur Dombivili

34 May 2021 The Chartered Accountant Student

Page 500
toppers
Intermediate (IPC) Scheme

AIR-1

Avilash Gaurav
Guwahati

Intermediate New Scheme

AIR-1 AIR-2 AIR-3

Girish Aswani Naman Maheshwari Ayush Gupta


Ajmer Jaipur Kolkata

“Our greatest weakness lies in giving up. The most certain way to succeed is
always to try just one more time.”
– Thomas A. Edison

NEWS UPDATE
ECONOMIC UPDATEs
• The Monetary Policy Committee in its meeting on (Amendment) Ordinance 2021 on 4th April 2021.
5th, 6th and & 7th April 2021 deliberated on the current Source: https://www.ibbi.gov.in/
macroeconomic situations and decided to keep the Policy
rates unchanged. • The resurgence of Covid cases in India has led Nomura to
Source:https://rbi.org.in/Scripts/BS_PressReleaseDisplay. cut the 2021 gross domestic product (GDP) forecast for
aspx?prid=51380 India to 11.5 per cent from the earlier 12.4 per cent.
Source:https://www.business-standard.com/article/
• The Government of India’s Policy has ensured that foreign markets/nomura-cuts-india-s-2021-gdp-forecast-to-11-5-
capital keeps flowing into the Country. FDI norms are sees-higher-inflation-121041200365_1.html
being relaxed across sectors.
Source:https://www.ibef.org/economy/foreign-direct- • OECD Economic Outlook March 2021 talked about
investment.aspx vaccine rollout which will provide a major boost to
economic activity. Faster and effective vaccination and
• The Insolvency and Bankruptcy Board of India has notified deployment across the world is vital for economic recovery
the pre-packaged insolvency resolution process recently, and consequent growth.
after promulgation of the Insolvency and Bankruptcy Source: https://www.oecd.org/economic-outlook/

The Chartered Accountant Student May 2021 35

Page 501
RNI No. 66180/1997 Postal Registration: DL(C)-01/1280/2021-23, D. No. MH/MR/TECH-47/3/2021 License To Post Without Prepayment,
WPP Licence No.: MR/Tech/WPP-247/DL(C)/2021 Posted at Mumbai Patrika Channel Sorting Office, Mumbai - 400001
Posting Date: Last Three Days of Advance Month (29th, 30th, 31st) and first four days of current month days/ dates only
(1st to 4th of next month), Date of publication: 26th of previous month

Crossword - may 2021


11. A militant organisation.
1 2 3 4 5
14. An original draft, minute, or record of a
6 7 document or transaction.
17. An American multinational technology
8 9 10 11 12 company that specialises in Internet-related
13 14 15
services.
19 A deduction from the usual cost of something.
16 17 24. A person can be independent director in a
maximum of ______ listed entities.
18
25. Relating to government revenue, especially
19 20 taxes.
30. Permitted by law.
21 22 32. Likely or liable to suffer from.
35. A generic name for taxes that are levied on
23 24 25 26
transactions such as sale and purchase.
27 28 29 30 37. Business to Business company that develops
digital ordering and delivery programmes for
31 32 33
restaurants.
34 35 36 37 38
40. Institute appointed to create and maintain a
databank of persons who are eligible and willing
39 40 41 to act as independent directors.
44. An indirect tax used in India on the supply of
42 43 44 45 46
goods and services.
47 48 45. Abbrv. Index for Industrial Production.
46. An investigative agency of Government of
49 50 51
India.

Across 36. The ____ date on which list of members


1. A person who searched for business will be finalised for purpose of
opportunity and starts a new enterprise to distribution of dividend.
make use of that opportunity is called _____ 38. An Indian cricket equipment
______ manufacturer.
6. A sudden increase in trade and economic 39. Common to a whole nation.
activity. 41 Production of receipts/vouchers for
7. An act of leaving a place. reimbursement of travelling charges for
8. A ____ code is a code that is valid for only travel within the city admissible under
one login session or transaction using a daily allowance.
mobile phone: 42. In India ____ is officially called
9. ...... clause is introduced in a treaty to Permanent Account Number:
prevent treaty shopping. 43. Characterised by or capable of clear,
12. The speed at which an event happens. sound reasoning.
13. Without concealment especially where 47. Centre of Excellence in Nano electronics.
these might be expected. 48. An educational Institute famous for
15. An amount that must be paid to obtain business studies.
something. 49. After admission of CIRP by the
16. A fictitious item representing an entire Adjudicating Authority, an IRP makes a
product family. public announcement in Form ______.
18. It is a safe and secure system for funds 50. It is a process that helps ______ water
transfer. purifier removes unwanted ions,
19. One search box, with the power of a dissolved solids and TDS from the water.
thousand apps. 51. Institute of Journalism and
20. Public research university in Mumbai. Communication.
21. Abbrv. International Graduate Programs
Office. DOWNWARD
22. Arrival that has been awaited (especially of 1. ______is a deadly disease caused by a
something momentous). virus.
23. Abbrv. Sri Lankan High Commission. 2. Used as a short substitute for a negative
26. An international organisation that works to clause.
build better policies for better lives. 3. ______ is the study of the forms and
27. A company dealing in leather accessories. features of land surfaces.
28. The largest and most diverse European 4. ______is conducted on behalf of
gathering of scholars and practitioners on University Grants Commission (UGC)
security issues. for determining the eligibility of Indian
29. A __ is a semi-solid that can have properties nationals for the Eligibility of Assistant
ranging from soft and weak to hard and Professorship.
tough. 5. An ______ is a powerful person who
31. Abbrv. Linear Parameter Varying Systems. is responsible for making things run
33. Have the same opinion about something. smoothly. If undelivered, please return to: The
34. ____ magnetic field emissions associated 9. ______ long tributary of the River Trent Institute of Chartered Accountants of
with some major earthquakes occurred in that flows through Nottinghamshire. India, ICAI Bhawan, Indraprastha Marg,
Indian Subcontinent. 10. Top Energy Company in India.
New Delhi-110104

36
Page 502
Economic Laws
Ca final - Paper 6D - Economic Laws
This capsule on Paper 6D: Economic Laws, Final (New) course is another step of the Board of Studies in its endeavour
to provide quality academic inputs to Final course students of Chartered Accountancy course. This is an open book
examination and duration is 4 hours. The question paper would comprise of five case studies of 25 marks each, out
of which the student would be required to attempt any four. Students must divide their four hours between four case
studies to be answered meticulously. Once the case studies have been opted, give them a comprehensive reading
while attempting the same. Some of the illustrative case studies have been provided below for practice purpose.
Students are suggested to solve the same in examination condition and check for the answers only after attempting
the case studies.

Case Study 1
Nadus (P) Ltd. is engaged in the business of real estate since which required the said agents to promote and negotiate deals,
12 years. The company is founded by two friends, Mr. Mayur only, for the units in Suvas and not for any other real estate
Agarwal and Mr. Neerav Sutaria, who are also its directors. Mr. project in Mihan area and for entering into such agreement, a
Urmil Dave, brother in law of Mr. Mayur, is the manager of the lumpsum amount was paid to such agents in cash.
company. Vikrama Builders (P) Ltd.’s business was affected due to such
It had acquired 10% shares of a company in Egypt, named arrangement of Nadus (P) Ltd. and so it filed a complaint with
Belashom LLC which is engaged in the construction of the authority under RERA against such arrangement. The case
commercial premises. Recently, it had received some bonus was assigned to Mr. Sumit Joshi, a RERA member. Mr. Sumit, in
shares from the said company. order to understand the arrangement being made by Nadus (P)
Belashom LLC was looking for a commercial property in Ltd. with the real estate agents, contacted his close friend, Mr.
India for opening its branch office in order to expand its business. Aman who was a real estate agent, and asked him to enter into an
For that purpose, Mr. Franklin, an international real estate agent agreement with Nadus (P) Ltd. as normal and then provide him
in Egypt was contacted by Belashom LLC and he told that one of all the details of such agreement.
his clients in India, a private limited company named Autukya Mr. Aman did the same and provided all the details to Mr.
(P) Ltd., wanted to sale, one of its commercial properties in India. Sumit. Mr. Sumit discussed the matter with the other members
After going through the details of the said property, of the authority under RERA in the meeting of the authority and
Belashom LLC became interested in such property and it was it was decided that such agreements made by Nadus (P) Ltd.
decided to send Mr. James, a director of Belashom LLC to India affected competition in the relevant market and so the case was
to meet the client of Mr. Franklin in India and finalise the deal referred to the Competition Commission of India. However, the
for the property. required quorum was not present throughout the said meeting of
Mr. Neerav who was on a visit to meet his old friend in the authority under RERA.
Bhutan, came to know that Mr. James was going to visit India. So The CCI on receipt of such reference from the authority
he shortened his trip and came to India bringing 30,000 INR in under RERA initiated an inquiry into the matter and formed an
form of currency notes with denominations of R100 and 20,000 opinion on the existence of prima facie case and directed the
INR in form of currency notes with denominations of R500, Director General to cause an investigation into the matter.
respectively, received as a gift from his friend. The Director General, during the investigation, received
Mr. James visited India bringing with him, some amount of certain evidences on affidavit from few employees of Nadus (P)
Egyptian Pounds (EGP) as follows:- Ltd. Certain books and papers of Nadus (P) Ltd. were also called
Particulars EGP for by the Director General which he kept in his custody for 2
Currency Notes 90,000 months.
Bank Notes 30,000 The Director General found that the Company Secretary of
Travelers Cheque 22,500 Nadus (P) Ltd., Mrs. Ridhima Sen, had assisted in drafting the
impugned agreements with the real estate agents. Mr. Urmil,
Mr. Neerav accompanied him. Mr. James met the
the manager, however, pleaded before the Director General, that
representative of Autukya (P) Ltd., Mr. Rajiv and after two
though he knew of such agreements being entered into by Nadus
rounds of discussion between them; the deal for the property was
(P) Ltd., he never gave his consent to such an act of the company.
finalized for R650 lakhs. Autukya (P) Ltd. remitted 4,50,000 EGPs
The copy of the report of investigation was forwarded
to Mr. Franklin as commission amount out of its EEFC account.
by the CCI to Nadus (P) Ltd. and the authority under RERA,
All the expenses incurred by Mr. James in INR on account of his
respectively.
boarding, lodging and travelling in India were paid by Nadus (P)
After making further inquiry, the CCI closed the matter and
Ltd., which was going to be reimbursed later on by Belashom
passed a cease and desist order as well as a penalty order to pay
LLC.
an amount equivalent to 25% of the revenue earned by Nadus (P)
Nadus (P) Ltd. was developing a real estate project in
Ltd. by making such anti-competitive agreements with the real
Mihan area of Nagpur City named ‘Suvas’. It had made certain
estate brokers.
agreements with real estate agents mainly operating in that area
The Chartered Accountant Student December 2021 05

Page 503
Economic Laws
On the basis of the given facts, answer the following questions: but for keeping the books and papers of Nadus (P) Ltd. in
Multiple Choice Questions his custody, prior permission of the CCI was required.
1. Whether Mr. Neerav has validly brought INR currency notes
into India? 5. Which of the following persons would be deemed to be guilty
(a) No, Mr. Neerav has brought in excess R25,000 from the of the contravention committed by Nadus (P) Ltd. of the
prescribed limit. provisions of the Competition Act, 2013?
(b) Yes, as there is no restriction of bringing any amount into (a) Nadus (P) Ltd., Mr. Mayur, Mr. Neerav and Mrs. Ridhima,
India from Nepal or Bhutan. respectively.
(c) No, Mr. Neerav has brought INR currency notes with (b) Nadus (P) Ltd. only.
denominations of R500. (c) Nadus (P) Ltd., Mr. Mayur and Mr. Neerav, respectively.
(d) Yes, if Mr. Neerav has provided declaration in respect of (d) Nadus (P) Ltd., Mr. Mayur, Mr. Neerav, Mr. Urmil and
the same to the Custom Authorities. Mrs. Ridhima, respectively.

2. Whether it was necessary for Mr. James to provide any Descriptive Questions
declaration to the Custom Authorities of India in respect of 6. (i) Whether Nadus (P) Ltd. was having any prohibition on
the Egyptian Pounds brought by him into India, if 1 USD = making investment in Belashom LLC?
15 EGPs? (ii) Whether Nadus (P) Ltd. was required to take any
(a) No, as Mr. James is a person resident outside India permission for receiving bonus shares from Belashom
(b) Yes, as the amount of currency notes exceeded $ 5,000 in LLC?
equivalent
(c) No, as the aggregate of EGPs in all forms did not exceed 7. (i) Whether Nadus (P) Ltd. was permitted to make payment
$ 10,000 in equivalent for meeting expenses of Mr. James in India?
(d) No, as there is no restriction in bringing foreign (ii) Whether Autukya (P) Ltd. was required to have any
exchange, without any limit, in any form in India. permissions for remitting the amount of commission to
Mr. Franklin, if 1 USD = 15 EGPs and 1 USD = R75?
3. Whether it was mandatory for the CCI to forward the copy
of the report of investigation to Nadus (P) Ltd. and the 8. (i) Whether any action can be taken against Mr. Sumit for
authority under RERA, respectively? inducing his friend, Mr. Aman to enter into an agreement
(a) Yes, as based upon such report, Nadus (P) Ltd. would with Nadus (P) Ltd.?
have been able to draft its response to the CCI and (ii) Whether the authority under RERA was having the power
because of reference of the authority under RERA, such to make reference to the Competition Commission of
investigation was caused to be made. India in respect of the case of Nadus (P) Ltd.?
(b) It was optional for the CCI to forward the copy of the report
of investigation to Nadus (P) Ltd. but it was mandatory to ANSWERS TO CASE STUDY 1
forward the same to the authority under RERA.
1. (c) As per Master Direction No. 17 – Import of Goods and
(c) It was optional for the CCI to forward the copy of the
Services:
report of investigation to Nadus (P) Ltd. and in case of
(i) Any person resident in India who had gone out of India
the authority under RERA, report was only required to
on a temporary visit, may bring into India at the time of
be forwarded if it was required by such authority.
his return from any place outside India (other than from
(d) It was mandatory for the CCI to forward the copy of the
Nepal and Bhutan), currency notes of Government of
report of investigation to Nadus (P) Ltd. as it was the
India and Reserve Bank of India notes up to an amount not
party under investigation and in case of the authority
exceeding R25,000 (Rupees twenty five thousand only).
under RERA, report was only required to be forwarded
(ii) A person may bring into India from Nepal or Bhutan,
if it was required by such authority.
currency notes of Government of India and Reserve
Bank of India for any amount in denominations up to
4. Whether the Director General was having the authority to
R100/-.
exercise such powers as were exercised by him during the
Mr. Neerav came to India bringing 30,000 INR in form of
investigation?
currency notes with denominations of R100 and 20,000
(a) He was having the authority to exercise such powers only
INR in form of currency notes with denominations of
if the prior permission of the CCI was obtained in that
R500, respectively, received as a gift from his friend.
regard.
It can be said that Mr. Neerav has not validly brought
(b) He was having the power to receive evidences on
20,000 INR in form of currency notes with denominations
affidavit but was not having the power to keep the books
of R500 into India.
and papers of Nadus (P) Ltd. in his custody.
(c) He was having the power to receive evidences on affidavit
2. (b) As per Master Direction No. 17 – Import of Goods and
as well as to keep the books and papers of Nadus (P) Ltd.
Services:
in his custody, respectively.
Import of Foreign Exchange into India: A person may–
(d) He was having the power to receive evidences on affidavit
06 December 2021 The Chartered Accountant Student

Page 504
Economic Laws
(i) Send into India, without limit, foreign exchange in upon him as well as to keep the books and papers of Nadus
any form (other than currency notes, bank notes and (P) Ltd. in his custody, as he has been vested with the powers
travelers cheques); of an inspector under Section 217 of the Companies Act,
(ii) Bring into India from any place outside India, without 2013.
limit, foreign exchange (other than unissued notes), Note: As per Section 217(3) of the Companies Act, 2013, the
subject to the condition that such person makes, on inspector shall not keep in his custody any books and papers
arrival in India, a declaration to the Custom Authorities produced under sub-section (1) or sub-section (2) for more
at the Airport in the Currency Declaration Form (CDF) than one hundred and eighty days and return the same to the
annexed to these Regulations; company, body corporate, firm or individual by whom or on
Provided further that it shall not be necessary to make whose behalf the books and papers were produced.
such declaration where the aggregate value of the foreign
exchange in the form of currency notes, bank notes or 5. (d) As per Section 48 of the Competition Act, 2002, where a
travelers cheques brought in by such person at any company committing contravention of any of the provisions
one time does not exceed USD 10,000 (US Dollars ten of this Act or of any rule, regulation, order made or direction
thousand) or its equivalent and/or the aggregate value issued thereunder, then following shall be deemed to be
of foreign currency notes (cash portion) alone brought guilty of the contravention; hence liable to be proceeded
in by such person at any one time does not exceed USD against and punished accordingly;
5,000 (US Dollars five thousand) or its equivalent. Every person who, at the time the contravention was
Here, it is given that 1 USD = 15 EGPs and Mr. James has committed, was in charge of, and was responsible to the
brought with him following Egyptian Pounds (EGP):- company for the conduct of the business of the company, as
well as the company.
Particulars EGP Converted to USD
Any such person who is liable to any punishment, if he
Currency Notes 90,000 6,000 proves that the contravention was committed without his
Bank Notes 30,000 2,000 knowledge or that he had exercised all due diligence to
Travelers Cheque 22,500 1,500 prevent the Commission of such contravention, then he will
Total 1,42,500 9,500 not be punishable.
Where it is proved that the contravention has taken place
Thus, it was necessary for Mr. James to provide
with the consent or connivance of or is attributable to any
declaration to the Custom Authorities of India in respect
neglect on the part of, any director, manager, secretary
of the Egyptian Pounds brought by him into India as the
or other officers of the company, then he also be deemed
amount of currency notes exceeded $ 5,000 in equivalent.
to be guilty of that contravention and shall be liable to be
proceeded against and punished accordingly.
3. (b) As per Section 26 of the Competition Act, 2002, the
For the purposes of this section, company means a body
Commission may forward a copy of the report of the Director
corporate and includes a firm or other association of
General to the parties concerned.
individuals, and director in relation to a firm, means a
The Commission shall forward a copy of the report of the
partner in the firm.
Director General to Central Government or the State
Here, the persons that would be deemed to be guilty of the
Government or the statutory authority if the investigation is
contravention committed by Nadus (P) Ltd. of the provisions
caused to be made based on reference received from them.
of the Competition Act, 2002 would be- Nadus (P) Ltd., Mr.
Thus, it was optional for the CCI to forward the copy of the
Mayur, Mr. Neerav, Mr. Urmil and Mrs. Ridhima, respectively.
report of investigation to Nadus (P) Ltd. but it was mandatory
Mr. Mayur, Mr. Neerav and Mr. Urmil are the persons
to forward the same to the authority under RERA.
responsible to the company for the conduct of the business of
the company. Though Mr. Urmil never gave his consent to such
4. (c) As per Section 41 of the Competition Act, 2002, the
an act of the company, however, he was having the knowledge
Director General shall assist the commission in investigating
of such agreements being entered into by Nadus (P) Ltd.
into any contravention of the provisions of this Act or any
Mrs. Ridhima assisted Nadus (P) Ltd. in drafting the
rules or regulations made thereunder when so directed by
impugned agreements with the real estate agents and so it
the Commission.
can be said that contravention has taken place due to her
The Director General shall have all the powers as are
connivance.
conferred upon the commission under section 36(2) i.e.
6. (i) As per Regulation 5 of the Foreign Exchange
power vested with the civil court.
Management (Transfer or Issue of any Foreign Security)
The power vested with inspector under sections 217
Regulations, 2004-
(Production of documents and evidence) and 220 (Seizure
(a) Indian Parties are prohibited from making investment
of documents by the inspector) of the Companies Act, 2013,
(or financial commitment) in foreign entity engaged in
shall available to Director General while investigating or any
real estate (meaning buying and selling of real estate or
other person investigating under his authority.
trading in Transferable Development Rights (TDRs) but
Thus, the Director General was having the power to receive
does not include development of townships, construction
evidences on affidavit, as powers of a civil court are vested
The Chartered Accountant Student December 2021 07

Page 505
Economic Laws
of residential/commercial premises, roads or bridges) residential flats or commercial plots in India exceeding USD
or banking business, without the prior approval of the 25,000 or five percent of the inward remittance whichever
Reserve Bank. is more, by persons other than individuals shall require
(b) An overseas entity, having direct or indirect equity prior approval of the Reserve Bank of India, irrespective of
participation by an Indian Party, shall not offer financial whether it is made through EEFC account or not.
products linked to Indian Rupee (e.g. non-deliverable In the given case, the deal for the commercial property
trades involving foreign currency, rupee exchange rates, was finalized for R650 lakhs and Autukya (P) Ltd. remitted
stock indices linked to Indian market, etc.) without the 4,50,000 EGPs to Mr. Franklin as commission amount, out of
specific approval of the Reserve Bank. its EEFC account.
Here, in the given case, Nadus (P) Ltd. had made 5% of inward remittance from sale of property = R650
investment i.e. acquired 10% shares of Belashom LLC, an lakhs*5% = R32.5 lakhs which is equivalent to USD
Egyptian company which is engaged in the construction 43,333.33 (R32,50,000/R75) and commission amount
of commercial premises. remitted = 4,50,000 EGPs which is equivalent to USD 30,000
As per the aforesaid provisions, there is prohibition in (4,50,000/15).
investing in real estate company abroad but real estate, Thus, Autukya (P) Ltd. was required to have prior permission
for this purpose, does not include construction of of RBI for remitting the amount of commission to Mr.
residential/commercial premises, etc. Franklin as the amount remitted is more than USD 25,000.
Thus, Nadus (P) Ltd. was not having any prohibition on
making of investment in Belashom LLC. 8. (i) As per Section 90 of the Real Estate (Regulation and
(ii) As per Regulation 4 of the Foreign Exchange Development) Act, 2016, no suit, prosecution or other legal
Management (Transfer or Issue of any Foreign Security) proceedings shall lie against the appropriate Government or
Regulations, 2004- the Authority or any officer of the appropriate Government
General permission has been granted to persons resident in or any member, officer or other employees of the Authority
India for purchase / acquisition of securities in the following for anything which is in good faith done or intended to
manner: be done under this Act or the rules or regulations made
(a) out of the funds held in RFC account; thereunder.
(b) as bonus shares on existing holding of foreign currency Here, complaint was filed with the authority under RERA by
shares; and Vikrama Builders (P) Ltd. against Nadus (P) Ltd. in respect
(c) when not permanently resident in India, out of their of the arrangements being made by it with the real estate
foreign currency resources outside India. agents. The case was assigned to Mr. Sumit Joshi, a RERA
General permission is also available to sell the shares so member and Mr. Sumit, in good faith, in order to understand
purchased or acquired. the arrangements being made by Nadus (P) Ltd. with the real
In the instance case study, Nadus (P) Ltd. had received some estate agents took help of his friend, Mr. Aman.
bonus shares from the Belashom LLC for which general Thus, no action can be taken against Mr. Sumit who induced
permission has been granted. So, Nadus (P) Ltd. was not his friend, Mr. Aman to enter into an agreement with Nadus
required to take any permission for the same. (P) Ltd. as it was done in good faith by Mr. Sumit.
(ii) As per Section 38 of the Real Estate (Regulation and
7. (i) As per Master Direction No. 17 – Import of Goods and Development) Act, 2016, where an issue is raised relating to
Services, a person resident in India may make payment in agreement, action, omission, practice or procedure that—
rupees towards meeting expenses on account of boarding, (a) has an appreciable prevention, restriction or distortion
lodging and services related thereto or travel to and from and of competition in connection with the development of a
within India of a person resident outside India who is on a real estate project; or
visit to India. (b) has effect of market power of monopoly situation being
As per Section 2(v) of the FEMA, 1999, person resident abused for affecting interest of allottees adversely, then
in India, inter-alia, means any person or body corporate the Authority, may suo motu, make reference in respect
registered or incorporated in India. of such issue to the Competition Commission of India.
Here in the case study, all the expenses incurred by Mr. James Here, the issue was related to the arrangements being made
in INR on account of his boarding, lodging and travelling in by Nadus (P) Ltd. with the real estate agents which affected
India were paid by Nadus (P) Ltd. for which it was going to be the competition in the relevant market and thus, the authority
reimbursed later on by Belashom LLC. under RERA was having the power to make reference to the
As per the aforesaid provisions, Nadus (P) Ltd. being a Competition Commission of India in respect of the case of
person resident in India, was given general permission for Nadus (P) Ltd.
incurring such expenses.
(ii) As per Schedule III (Transactions which are
prohibited)-Foreign Exchange Management (Current
Account Transactions) Rules, 2000, remittance of
commission, per transaction, to agents abroad for sale of

08 December 2021 The Chartered Accountant Student

Page 506
Economic Laws
Case Study 2
Prahasti Ltd. is an unlisted public company, situated in Chennai,
Financial Turnover of Total amount of
Tamil Nadu, with seven directors on its Board and it has share
Year Sharma & Co. (R) Transactions with
capital of R10 crore with 150 shareholders. It is engaged in the
Tamprabha Ltd.
business of cloth garments manufacturing and wholesaling. Also,
during each F.Y. (R)
it exports outside India.
As part of its export trade policy, it provides trade samples 2016-17 220 lakhs 10 lakhs
free of cost to the prospective customers and if it receives an 2017-18 180 lakhs 8 lakhs
export order of delivering more than 1000 cloth garments, then
2018-19 200 lakhs 9 lakhs
it has to export further 50 cloth garments worth R2 lakhs free of
cost to the customer. 2019-20 190 lakhs 9 lakhs
Recently, in the month of June, it had received an export 2020-21 150 lakhs 8 lakhs
order of delivering 1500 cloth garments to a company in
All the financial creditors of Tamprabha Ltd. were related
Germany for which the full export value declared was R63,00,000
parties and it had 15 operational creditors. Mr. Dev was
(70,000 Euros). However, the said company returned 200 pieces
appointed as the resolution professional (RP) and he sanctioned
of clothes worth R8,40,000 back to Prahasti Ltd. in the month of
a transaction of supply of goods to an associate company of
July. Remaining export value was realized by it and repatriated
Tamprabha Ltd. during the insolvency process for which approval
through the authorised dealer in India.
of the committee of creditors was not obtained by him.
Also, in order to have business security, there is an exclusive
The resolution plan of Tamprabha Ltd. contained a provision
distribution agreement entered into between different exporters
of combination as per Section 5 of the Competition Act, 2002
of cloth garments in Tamil Nadu exporting in Europe whereby and it was approved by the prescribed authorities. As a result
each exporter has been allocated different markets of Europe in of the implementation of the resolution plan, there was change
which they are allowed to do business. in the entire management of Tamprabha Ltd. and its control
One of the directors of Prahasti Ltd., Mr. Karan, had has been handed over to persons who have not been its related
withdrawn 50,000 Euros equivalent to $ 60,000, for the purpose of parties and against whom no legal proceedings are going on
business trip to Germany and Italy, respectively, for which he was under any statue.
going to be reimbursed by Prahasti Ltd. but however due to the Also, Tamprabha Ltd. was liable for an offence committed
reason of Covid-19 pandemic, the trip was cancelled and so after under the provisions of the Companies Act, 2013, prior to the
utilizing 20,000 Euros for studies for her daughter in Germany, commencement of corporate insolvency resolution process.
he returned back the remaining amount to the authorised dealer
within 140 days. In the light of enumerated facts, answer the following:
Prahasti Ltd. was expanding its business for the same
Multiple Choice Questions
purpose, one another corporate office was being searched by
the company in Chennai city only. One of its employees, Mr. 1. On expiry, how many further days from the date of receipt
Raj was searching online for a property and he visited a website, of order of revocation of registration by the promoter, the
named ‘propertylelo.com’, whereby Mr. Raj was asked to enter decision of the authority under RERA for carrying out of the
certain details which were then going to be disclosed with remaining development works should have taken effect?
certain promoters of real estate projects in Chennai for which (a) days
the promoters were charged by the website. Also after taking (b) 60 days
permission of a director by Mr. Raj, on payment of some fees, a (c) 45 days
(d) It shall be immediately effective
virtual 3D tour of a real estate project was arranged by the said
website. The said website portal was not registered as a real estate
2. Is there any contravention of the provisions of the FEMA,
agent.
1999, by Karan?
The company found a property near its location but came to
(a) No, as Mr. Karan has utilized the foreign currency
know later that the registration of such real estate project was
amount for a permissible transaction and within the
revoked by the authority under RERA. The authority under RERA
limits as per the ‘LRS’.
decided to hand over the task of the remaining development
(b) Yes, as Mr. Karan has not utilized the foreign currency
works of the said real estate project to the competent authority
amount for the purpose for which it was acquired.
as the association of allottees had refused to do the same and at
(c) No, as Mr. Karan after utilizing the foreign currency
that time, 45 days had passed from the date of receipt of order of amount for a permissible transaction, has surrendered
revocation of registration by the promoter. the remaining amount with the authorised dealer within
In case of one of the debtors of Prahasti Ltd. named the specified period.
Tamprabha Ltd., corporate insolvency resolution process was (d) No, as Mr. Karan was eligible to utilize the foreign
initiated against it by one of its operational creditor. Mr. Dev currency amount for any other permissible transaction
Sharma, was appointed as the Interim Resolution Professional as the business trip was cancelled due to a genuine
(IRP) who is partner of Sharma & Co., a law consulting firm reason and not because of default on his part.
which had transactions of following amounts with Tamprabha
Ltd. during the last 5 financial years:- 3. Whether Mr. Dev has validly sanctioned the transaction of
supply of goods by Tamprabha Ltd.?
(a) No, he was required to take prior approval of the

The Chartered Accountant Student December 2021 09

Page 507
Economic Laws
committee of creditors before sanctioning such ANSWERS TO CASE STUDY 2
transaction. 1. (a) As per Section 8 of the RERA, 2016, upon lapse of the
(b) No, due to applicability of order of moratorium by the registration or on revocation of the registration under this
Adjudicating Authority, such a transaction should have Act, the Authority, may consult the appropriate Government
not taken place. to take such action as it may deem fit including the carrying
(c) Yes, the IBC, 2016, itself has given authority to the out of the remaining development works by competent
resolution professional to undertake such actions authority or by the association of allottees or in any other
necessary for the continued business operations of the manner, as may be determined by the Authority.
corporate debtor. It is provided that no direction, decision or order of the
(d) Yes, provided the transaction was conducted at arm’s Authority under this section shall take effect until the expiry
length price. of the period of appeal provided under the provisions of this
Act.
4. Which authorities would have approved the resolution plan Time period for filing appeal is 60 days from the date of
of Tamprabha Ltd. and in what sequence? receipt of order by the aggrieved person as per Section 44 of
(a) Committee of Creditors and then Adjudicating the Act.
Authority, respectively. Here, 45 days had passed from the date of receipt of order of
(b) Committee of Creditors, Adjudicating Authority and revocation of registration by the promoter, so, after expiry of
then Competition Commission of India, respectively. further 15 days, the decision of the authority under RERA
(c) Committee of Creditors, Competition Commission of for carrying out of the remaining development works should
India and then Adjudicating Authority, respectively. have taken effect.
(d) Competition Commission of India, Committee of
Creditors and then Adjudicating Authority, respectively. 2. (b) As per the provisions of the FEMA, 1999, if any person,
other than an authorized person, who has acquired or
5. Mr. Dev Sharma would have been ineligible to be appointed purchased foreign exchange for any purpose mentioned in
as the Interim Resolution Professional of Tamprabha Ltd. if:- the declaration made by him to authorized person.
(a) Sharma & Co. would have entered into transaction(s) of • Does not use it for such purpose, or
further amount of R1 lakh or more with Tamprabha Ltd. • Does not surrender it to the authorized person within the
during any of the last 3 financial years. specified period, or
(b) Sharma & Co. would have entered into transaction(s) of • Uses the foreign exchange so acquired or purchased for
further amount of R1 lakh or more during F.Y. 2018-19 any other purpose for which purchase or acquisition of
and transaction(s) of further amount of R50,000 or more foreign exchange is not permissible under the provisions
during F.Y. 2019-20 with Tamprabha Ltd., respectively. of the Act or the rules or regulations or direction or order
(c) Sharma & Co. would have entered into transaction(s) of made there under,
further amount of R3 lakhs or more with Tamprabha Ltd. Such person shall be deemed to have committed
during any of the last 5 financial years. contravention of the provisions of the Act.
(d) Sharma & Co. would have entered into transaction(s) of
further amount of R28 lakhs or more with Tamprabha 3. (a) As per Section 5(24) of the IBC, 2016, an associate
Ltd. during any of the last 3 financial years. company is considered as a related party of the corporate
debtor.
Descriptive Questions According to section 28 of the Code, the resolution
6. (i) Whether Prahasti Ltd. needs to furnish declaration in professional, during the corporate insolvency resolution
case of goods which are exported free of cost as per its process, shall not undertake any related party transaction
trade policy? without the prior approval of the committee of creditors.
(ii) Whether Prahasti Ltd. can be said to have realized full Thus, Mr. Dev has not validly sanctioned the transaction
export value with respect to the export order from the of supply of goods to an associate company of Tamprabha
company in Germany? Ltd. during the insolvency process because approval of the
committee of creditors was required to be obtained by him
7. Whether the agreement made between different exporters of for such transaction as aforesaid.
cloth garments in Tamil Nadu can be considered as an anti-
competitive agreement? 4. (d) As per Section 31 of the IBC, 2016, if the Adjudicating
Authority is satisfied that the resolution plan as approved by
8. Whether the website portal named ‘propertylelo.com’ would the committee of creditors meets the requirements as per
be required to be registered as a real estate agent? (Please section 30(2), it shall by order approve the resolution plan.
support your answer on the basis of a relevant case law) Where the resolution plan contains a provision for
combination, as per section 5 of the Competition Act, 2002,
9. (i) What would have been the constitution of committee of the resolution applicant shall obtain the approval of the
creditors of Tamprabha Ltd.? Competition Commission of India under that Act prior to the
(ii) Whether Tamprabha Ltd. would be prosecuted for approval of such resolution plan by the committee of creditors.
the offence committed under the provisions of the On reading of the aforesaid provisions, the authorities and
Companies Act, 2013, prior to the commencement of the sequence of approval that can be derived is:- Competition
corporate insolvency resolution process? Commission of India, Committee of Creditors and then
Adjudicating Authority, respectively.

10 December 2021 The Chartered Accountant Student

Page 508
Economic Laws
5. (a) As per Regulation 3 of the Insolvency and Bankruptcy into India, within the period specified for realisation of the
(Insolvency Resolution process for Corporate Persons) export value, of the exported goods in respect of which a
Regulation, 2016, an insolvency professional shall be eligible declaration was made under Regulation 3, shall be deemed
for appointment as a resolution professional for a corporate to be realisation of full export value of such goods.
insolvency process if he is not an employee or proprietor or As per Regulation 9 of the Foreign Exchange Management
a partner of a legal or consulting firm that has or had any (Export of Goods and Services) Regulations, 2015, the
transaction with the corporate debtor amounting to five per amount representing the full export value of goods /
cent or more of the gross turnover of such firm in the last software/ services exported shall be realised and repatriated
three financial years. to India within nine months or within such period as may
Financial Turnover of Total amount of be specified by the Reserve Bank, in consultation with the
Year Sharma & Co. (R) Transactions with Government, from time to time, from the date of export,
Tamprabha Ltd. provided.
during each F.Y. (R)
Given Case and Analysis: Full export value declared by
2018-19 200 lakhs 9 lakhs Prahasti Ltd. was R63,00,000 in respect to export order from
2019-20 190 lakhs 9 lakhs the company in Germany.
However, Prahasti Ltd. re-imported 200 pieces of clothes
2020-21 150 lakhs 8 lakhs
worth R8,40,000 from the said company in the month of July
Total 540 lakhs 26 lakhs i.e. within the period specified for realisation of the export
value. So, it shall be deemed to be realisation of full export
Here, 5% of R540 lakhs comes to R27 lakhs and Sharma & value of such goods as per explanation to the Regulation 4 as
Co. has already rendered transaction(s) amounting to Rs. aforesaid.
26 lakhs to Tamprabha Ltd. So, Mr. Dev Sharma would have Also, remaining export value had been realized by Prahasti
been ineligible to be appointed as the Interim Resolution Ltd. and repatriated through the authorised dealer in India.
Professional of Tamprabha Ltd. if Sharma & Co. would have Thus, it can be said that Prahasti Ltd. has realized full export
entered into transaction(s) of further amount of R1 lakh or value with respect to the export order from the company in
more with Tamprabha Ltd. during any of the last 3 financial Germany.
years.
Note: Resolution Professional includes an Interim 7. Legal Position: As per Section 3 of the Competition Act,
Resolution Professional as per Section 5(27) of the IBC, 2002, it shall be unlawful for any enterprise or association of
2016. enterprises or person or association of persons to 'enter' into
any agreement in respect of production, supply, storage,
6. (i) Legal Position: As per Regulation 4 of the Foreign distribution, acquisition or control of goods or provision of
Exchange Management (Export of Goods and Services) services, which causes or is likely to cause an appreciable
Regulations, 2015, export of goods / software may be made adverse effect on competition within India; and such
without furnishing the declaration in the following cases, agreements shall be void.
inter-alia, namely: Sub-section 5 to the said section 3 protects the right of
(a) trade samples of goods and publicity material supplied specific persons by restricting the application of section 3 to
free of payment. their rights, hence become exceptions to section 3.
(b) by way of gift of goods accompanied by a declaration by One of such exceptions is:- Any agreement or part there-of
the exporter that they are not more than five lakh rupees in shall not be considered as anti-competitive, hence not void
value. to the extent it is exclusively related to production, supply,
distribution or control of goods or provision of services for
Given Case and Analysis: As part of its export trade policy, purpose of export of goods from India.
Prahasti Ltd. provides trade samples free of cost to the
prospective customers and if it receives an export order of Given Case and Analysis: Here, in the given case, the
delivering more than 1000 cloth garments, then it exports agreement entered into between different exporters of cloth
extra 50 cloth garments worth R2 lakhs free of cost which is garments in Tamil Nadu exporting in Europe is for the
less than value of R5 lakhs as prescribed. purpose of export goods from India and hence cannot be
Thus, Prahasti Ltd. is not required to furnish declaration in considered as an anti-competitive agreement as it has been
case of aforesaid goods which are exported free of cost as per covered by the exception as aforesaid.
its trade policy.
8. Legal Position: The facts in the given case are similar to the
(ii) Legal Position: As per Regulation 4 of the Foreign case law with citation, MahaRera Order in the Suo Moto
Exchange Management (Export of Goods and Services) Enquiry No.17/2018 dated 03.10.2019, where in it was
Regulations, 2015, unless otherwise authorised by the decided that a digital portal needs to be registered as a real
Reserve Bank, the amount representing the full export value estate agent if it carries out the following functions:-
of the goods exported shall be paid through an authorised
dealer in the manner specified in the Foreign Exchange 1. Portals when they collect the details of the viewer and share
Management (Manner of Receipt and Payment) Regulations, them with advertiser/seller and also disclose the information
2000 as amended from time to time. of promoters to buyers, they introduce the parties to the sale
Explanation—For the purpose of this regulation, re-import transaction.

The Chartered Accountant Student December 2021 11

Page 509
Economic Laws
(b) 1 representative elected by all workmen; and
2. If the portal simply provide the information about the real (c) 1 representative elected by all employees.
estate project, its offering for sale to the public at large, then
they are simply the agencies engaged for advertisement and Given Case & Analysis: Here, all the financial creditors of
when an individual is targeted by contacting and persuading Tamprabha Ltd. were related parties and it had 15 operational
him by the portals for sale and purchase of listed properties creditors, so the committee of creditors constituted would
they come under the legal definition of negotiation. have been as follows:
(a) All the 15 operational creditors (as it has less than 18
3. Web Portals introduce the buyer and seller with each other, operational creditors);
they provide the information of the project to the buyer, they (b) 1 representative elected by all workmen; and
arrange virtual tour of the project and also provide other (c) 1 representative elected by all employees.
information useful for taking an informative decision. Hence,
they facilitate the sale of the real estate project. (ii) Legal Position: As per Section 32A(1) of the Insolvency
and Bankruptcy Code, 2016, notwithstanding anything to
4. Once any monetary gain is derived for the purpose of the contrary contained in this Code or any other law for the
performing any act of the real estate agent by whichever time being in force, the liability of a corporate debtor for
name it amounts the receipt of the fees under the RERA. an offence committed prior to the commencement of the
corporate insolvency resolution process shall cease, and the
5. The Parliament has not carved out any exceptions to the corporate debtor shall not be prosecuted for such an offence
applicability of the provisions of RERA, Hence, we hold that from the date the resolution plan has been approved by the
RERA overrides section 79 of the IT Act. Adjudicating Authority under section 31, if the resolution
plan results in the change in the management or control of
Given Case & Analysis: Here, in the website, named the corporate debtor to a person who was not-
‘propertylelo.com’, Mr. Raj was asked to enter certain details (a) a promoter or in the management or control of the
which were then going to be disclosed with certain promoters corporate debtor or a related party of such a person; or
of real estate projects in Chennai for which such promoters (b) a person with regard to whom the relevant investigating
were charged by the website. authority has, on the basis of material in its possession,
Accordingly, Mr. Raj has been introduced to the sale reason to believe that he had abetted or conspired for the
transaction and he would be contacted by such promoters for commission of the offence, and has submitted or filed a
a property deal. Due to this, the website has earned monitory report or a complaint to the relevant statutory authority or
gain for exchange of information of prospective buyers with Court:
the promoters. Provided that if a prosecution had been instituted during
Also, on payment of some fees by Mr. Raj on permission the corporate insolvency resolution process against such
of director, a virtual 3D tour of a real estate project was corporate debtor, it shall stand discharged from the date of
arranged by the said website. This type of facility helps in approval of the resolution plan subject to requirements of
taking an informative decision to the prospective buyer. this sub-section having fulfilled.
Thus, it can be said that the website portal named ‘propertylelo.
com’ would be required to be registered as it carries out the Given Case & Analysis: Here, it is given that, as a result of the
functions of the real estate agent as explained above. resolution plan, there was change in the entire management
of Tamprabha Ltd. and its control has been handed over to
9. (i) Legal Position: As per Regulation 16 of the IBBI persons who have not been its related parties and against
(Insolvency Resolution Process for Corporate Persons) whom no legal proceedings are going on under any statue.
Regulations, 2016, where the corporate debtor has no
financial debt or where all financial creditors are related It appears from the given facts that conditions as
parties of the corporate debtor, the committee shall be set demonstrated in section 32A(1) has been satisfied by
up in accordance with this Regulation. Tamprabha Ltd. and thus, the liability of Tamprabha Ltd.
The committee formed under this Regulation shall consist of for an offence committed under the provisions of the
members as under – Companies Act, 2013, prior to the commencement of the
(a) 18 largest operational creditors by value: corporate insolvency resolution process shall cease, and it
Provided that if the number of operational creditors is less shall not be prosecuted for such an offence from the date
than 18, the committee shall include all such operational the resolution plan has been approved by the Adjudicating
creditors; Authority under section 31.

"Education is the passport to the future, for tomorrow belongs


to those who prepare for it today." - Malcolm X

12 December 2021 The Chartered Accountant Student

Page 510
Economic Laws
Case Study 3
The Adjudicating authority under the Insolvency and Bankruptcy
Sr. Details Details relating to the Application
Code, 2016, had received different applications during the July
No. of the
month, in respect of certain corporate persons, as follows:-
Applicant
Sr. Details Details relating to the Application 7 TLF (P) TLF (P) Ltd. filed an application for
No. of the Ltd., a realizing the secured asset of Anmoli Ltd.
Applicant secured during the liquidation proceedings for
1 Ukrin Ltd., Ukrin Ltd. submitted a withdrawal creditor of which it faced resistance from Mr. Raj,
operational application on 26th May for consideration Anmoli Ltd. director of Anmoli Ltd. (Note 6)
creditor of by the Committee of Creditors which
Kaptcha was approved by it, by a vote of 92%, on Notes:
Ltd., 1st June and Mr. Tanmay, the Interim 1. Such allottees then filed a complaint against Trees Estate Ltd.
corporate Resolution Professional, then submitted with the Real Estate Regulatory Authority under Section 31
debtor such application to the Adjudicating of the Real Estate (Regulation & Development) Act, 2016.
authority on 5th June on behalf of Ukrin The said authority under RERA passed an order imposing
maximum penalty upon the promoter company, Trees Estate
Ltd.
Ltd. with a direction to compensate the said 30 allottees by
2 Certain 30 allottees out of 310 allottees of Trees returning their cumulative investment amount of R20 crores
allottees Estate Ltd. jointly filed an application for along with total interest of R2 crores. The estimated cost of
of Trees initiating corporate insolvency resolution the real estate project was R200 crores. Trees Estate Ltd. filed
Estate Ltd., process against it, as the said allottees, an appeal with the Appellate Tribunal against the said order
corporate on the basis of model apartment had of Real Estate Regulatory Authority.
debtor purchased the properties in the project
and according to them it was not as 2. It was found by the Adjudicating authority that JLC (P) Ltd.
per model displayed and the promoter had notified vide an e-mail to Turf Enterprise within 10 days
company refused to return the investment of the demand notice, of the dispute that existed, and the said
amount of such allottees. (Note 1) matter was going to be referred for arbitration by JLC (P) Ltd.
and accordingly, the Adjudicating authority passed a penalty
3 Turf Turf Enterprise filed an application along
order with a fine amount of R70,000 against Turf Enterprise,
Enterprise, with the relevant enclosures on 10th after opportunity of being heard, for willful non-disclosure of
an June for initiating corporate insolvency such fact of notice of dispute and also rejected its application.
operational resolution process against JLC (P) Ltd.
creditor of (Note 2) 3. One of such transactions was entered by Saath Ltd. before 19
JLC (P) Ltd.
months preceding the insolvency commencement date with
4 Mr. Ravi, Mr. Ravi filed an application for declaring Janam Ltd. which involved supplying of goods by Saath Ltd.
Resolution two undervalued transactions entered for R4.4 crores which Saath Ltd. would have normally sold
Professional into by Saath Ltd. as void and to reverse for 4.6 crores in its ordinary course of business. Saath Ltd.
of Saath the effect of such transactions. (Note 3) and Janam Ltd. were having two directors in common.
Ltd., The other transaction was entered by Saath Ltd. before 17
corporate months preceding the insolvency commencement date with
debtor Mr. Mahesh which involved sale of property of Saath Ltd.
for R15 crore, the stamp duty value of which was R35 crore.
5 KC & KC & Sons filed an application for
Mr. Mahesh is a house worker of Mr. Sunil, the director of
Sons, an obtaining liquidation order against FAL Saath Ltd. There was a case under the Prohibition of Benami
operational Ltd. on the ground that FAL Ltd. had Property Transactions Act, 1988, going against Mr. Mahesh
creditor of contravened the resolution plan approved and Mr. Sunil, due to acquisition of such property in the
FAL Ltd. by the Adjudicating Authority because as name of Mr. Mahesh and it was held that Mr. Mahesh was
per the said plan, FAL Ltd. had to pay 60% the ‘benamidar’ and Mr. Sunil was the ‘beneficial owner’ and
of pending dues to KC & Sons as a full & the property was ordered to be confiscated and consequently
final settlement amount but it had paid has been disposed off.
only 20% of its pending dues as a full &
final settlement amount. (Note 4) 4. Adjudicating authority passed the liquidation order of FAL
6 Mr. Rohan, Mr. Rohan made an application along Ltd. on the basis of application of KC & Sons. However, KC
Interim with a list of financial creditors for & Sons, afterwards, filed a suit against FAL Ltd. in the City
Civil Court for realizing its dues as per the resolution plan
Resolution appointment of authorised representative
approved by the Adjudicating authority under the Insolvency
Professional to act on behalf of such creditors during
and Bankruptcy Code, 2016.
of Tadan the corporate insolvency resolution
Ltd., process. (Note 5)
5. Mr. Rohan had offered names of three insolvency
corporate professionals to such class of financial creditors to act as its
debtor
The Chartered Accountant Student December 2021 13

Page 511
Economic Laws
authorised representative who belonged to three different 4. Whether the names offered by Mr. Rohan for appointment as
states:- Gujarat, Maharashtra and Rajasthan, respectively. authorised representative can be considered proper and till
The highest number of such creditors of Tadan Ltd. belonged what time the Adjudicating authority should have appointed
to the state of Gujarat. such authorised representative?
(a) Yes, as one name is from Gujarat and other two names
6. The application of TLF (P) Ltd. was approved by the belong to such states which are nearby to Gujarat. The
Adjudicating authority and TLF (P) Ltd. was permitted to Adjudicating authority should have appointed such
realize its security interest in the asset. Accordingly, TLF authorised representative prior to the first meeting of the
(P) Ltd. enforced its security interest and yielded amount of committee of creditors.
R2 crores in excess of its debts due from Anmoli Ltd. (b) No, all the three names offered should have been
Apart from the aforesaid applications received by the from Gujarat. The Adjudicating authority should have
Adjudicating authority during the July month there were few appointed such authorised representative prior to the
other applications received by it in respect of certain corporate first meeting of the committee of creditors.
persons which could not be disposed of within the time periods (c) No, all the three names offered should have been
as specified in the IBC, 2016, for which the reasons were recorded from Gujarat. The Adjudicating authority should have
in writing by the Adjudicating authority. appointed such authorised representative prior to the
formation of the committee of creditors.
Answer the following questions in the light of the given (d) Yes, as at least one name offered should have been
informations: from Gujarat. The Adjudicating authority should have
Multiple Choice Questions appointed such authorised representative prior to the
formation of the committee of creditors.
1. Till what date the Committee of Creditors should have
considered the withdrawal application submitted by Ukrin
5. Who can extend the time period for disposing of the few
Ltd. and till what date, such application should have been
other applications received by the Adjudicating authority
submitted with the Adjudicating authority for approval by
during the July month?
Mr. Tanmay?
(a) The President of the National Company Law Tribunal
(a) 2nd June and 9th June, respectively.
can extend the time periods specified in the Act but not
(b) 31st May and 5th June, respectively.
exceeding ten days.
(c) 9th June and 12th June, respectively.
(b) The Chairperson of the National Company Law Appellate
(d) 2nd June and 4th June, respectively.
Tribunal can extend the time periods specified in the Act
but not exceeding seven days.
2. Whether the application filed by the 30 allottees of Trees
(c) The Chairperson of the National Company Law Tribunal
Estate Ltd. can be considered to be admissible by the
can extend the time periods specified in the Act but not
Adjudicating authority?
exceeding ten days.
(a) No, as an application is already with the authority under
(d) The Chairperson of the National Company Law Appellate
RERA, so simultaneously two proceedings cannot be
Tribunal can extend the time periods specified in the Act
initiated for the same matter.
but not exceeding seven days.
(b) Yes, as the amount of default involved is more than
R1 crore.
Descriptive Questions
(c) No, as the application is filed jointly by lesser number of
6. For contravention of which provisions the penalty would
allottees than prescribed.
have been imposed by the authority under RERA upon the
(d) Yes, such application can be admitted as the RERA Act
promoter company, Trees Estate Ltd. and of what amount?
provides an additional remedy to the homebuyer which
Also, how much amount of pre-deposit would have been made
will not bar other remedies available to the homebuyer.
by it for filing the appeal with the Appellate Tribunal?
3. What minimum fine amount should have been imposed
7. (i) Whether the two transactions entered by Saath Ltd. can
on Turf Enterprise by the Adjudicating authority and what
be said to have entered within the relevant period for
amount of maximum fine it could have imposed on Turf
considering them as undervalued transactions?
Enterprise?
(ii) Whether the two transactions entered into by Saath Ltd., as
(a) Adjudicating authority should have imposed minimum
aforesaid, can be considered as undervalued transactions
fine of R1 lakh on Turf Enterprise and maximum fine of
as contemplated by Mr. Ravi in the application filed with
R3 lakhs could have been imposed by it.
the Adjudicating Authority?
(b) Adjudicating authority should have imposed minimum
fine of R1 lakh on Turf Enterprise and maximum fine
8. Whether KC & Sons should have instituted a suit against
of R1 crore could have been imposed by it. However, it
FAL Ltd. in the City Civil Court and whether such court can
possesses the discretion to impose a lower amount of
entertain such suit?
fine.
(c) Adjudicating authority should have imposed minimum
9. What shall be done by TLF (P) Ltd. with respect to amount
fine of R1 lakh on Turf Enterprise and maximum fine of
of R2 crores yielded in excess of its debts due from Anmoli
R5 lakhs could have been imposed by it.
Ltd. and before realizing such security interest by TLF (P)
(d) Adjudicating authority should have imposed minimum
Ltd., what kind of verification would have been made by the
fine of R1 lakh on Turf Enterprise and maximum fine of
liquidator?
R1 crore could have been imposed by it.
14 December 2021 The Chartered Accountant Student

Page 512
Economic Laws
ANSWERS TO CASE STUDY 3 4. (b) As per section 21(6A) of the IBC, 2016, where a financial
1. (d) Withdrawal of application shall be pursuant to Section debt is owed to a class of creditors other than the creditors
12A of the Code read with Regulation 30A of the IBBI covered above, the IRP shall make an application to the AA
(Insolvency Resolution Process for Corporate Persons) along with the list of all financial creditors, with the name
Regulations, 2016. of an insolvency professional to act as their authorised
Once application is admitted and after Constitution of CoC representative appointed by the Adjudicating Authority
but before issue of Invitation for Expression of Interest prior to the first meeting of the committee of creditors.
(“EoI”):- An application for withdrawal made by the Applicant Authorised Representative from the State or Union
shall be firstly considered by the CoC, within seven days of Territory having highest number of creditors in class
its receipt. Such withdrawal of application shall be approved The Interim Resolution Professional shall offer the names of
by the CoC with ninety percent voting share, upon which three insolvency professionals to be voted upon by the class
the resolution professional shall submit such withdrawal of creditors, who must be from the State or Union Territory,
application along with the approval of the committee, to the which has the highest number of creditors in the class as per
Adjudicating Authority on behalf of the applicant, within records of the corporate debtor.
three days of such approval
Where such State or Union Territory does not have
Ukrin Ltd. submitted a withdrawal application on 26th adequate number of insolvency professionals, the insolvency
May for consideration by the Committee of Creditors. So, professionals having addresses in a nearby State or Union
the Committee of Creditors should have considered such Territory, as the case may be, shall be considered.
application by 2nd June i.e. 7 days from 26th May.
Here, the highest number of such creditors of Tadan Ltd.
The Committee of Creditors approved such application by belonged to the state of Gujarat. So, all the three names
a vote of 92% on 1st June. So, Mr. Tanmay should have been offered should have been from Gujarat by Mr. Rohan.
submitted with the Adjudicating authority for approval by
4th June i.e. 3 days from 1st June. 5. (a) As per Section 64 of the IBC, 2016, where an application
is not disposed of or an order is not passed within the period
2. (c) Section 7 of the IBC, 2016:- specified in this Code, the National Company Law Tribunal
A financial creditor either by itself or jointly with other or the National Company Law Appellate Tribunal, as the
financial creditors, or any other person on behalf of case may be, shall record the reasons for not doing so within
the financial creditor, as may be notified by the Central the period so specified; and the President of the National
Government may file an application for initiating corporate Company Law Tribunal or the Chairperson of the National
insolvency resolution process against a corporate debtor Company Law Appellate Tribunal, as the case may be, may,
before the Adjudicating Authority when a default has after taking into account the reasons so recorded, extend the
occurred. period specified in the Act but not exceeding ten days. No
Provided further that for financial creditors who are allottees injunction shall be granted by any court, tribunal or authority
under a real estate project, an application for initiating in respect of any action taken, or to be taken, in pursuance of
corporate insolvency resolution process against the corporate any power conferred on the National Company Law Tribunal
debtor shall be filed jointly by not less than one hundred or the National Company Law Appellate Tribunal under this
of such allottees under the same real estate project or Code.
not less than ten per cent. of the total number of such
allottees under the same real estate project, whichever is 6. Section 12 of the Real Estate (Regulation & Development) Act,
less. 2016, contains provisions which deal with the obligations of a
promoter regarding veracity of the advertisement or prospectus.
Here, 30 allottees out of 310 allottees of Trees Estate Ltd.
jointly filed an application for initiating corporate insolvency Accordingly, where any person makes an advance or a
resolution process against it. But as the proviso above, 100 deposit on the basis of the information contained in the
allottees or 31 allottees (10% of 310) whichever is less, should notice, advertisement or prospectus, or on the basis of any
have jointly filed such application. model apartment, plot or building, as the case may be, and
sustains any loss or damage by reason of any incorrect, false
So, the application filed by the said 30 allottees of Trees
statement included therein, he shall be compensated by the
Estate Ltd. is not admissible by the Adjudicating authority as
promoter in the manner as provided under this Act.
it is filed jointly by lesser number of allottees than prescribed.
However, if the person affected by such incorrect, false
3. (d) As per Section 76 of the IBC, 2016:- Where an statement contained in the notice, advertisement or
operational creditor has wilfully or knowingly concealed prospectus, or the model apartment, plot or building, as
in an application under section 9 the fact that the corporate the case may be, intends to withdraw from the proposed
debtor had notified him of a dispute in respect of the project, he shall be returned his entire investment along
unpaid operational debt or the full and final payment of the with interest at such rate as may be prescribed and the
unpaid operational debt. compensation in the manner provided under this Act.
Such operational creditor or person, as the case may be, shall In the given case, the 30 allottees on the basis of model
be punishable with imprisonment for a term which shall not apartment had purchased the properties in the project and
be less than one year but may extend to five years or with according to them it was not as per model displayed and the
fine which shall not be less than one lakh rupees but may promoter company refused to return the investment amount
extend to one crore rupees, or with both. of such allottees.

The Chartered Accountant Student December 2021 15

Page 513
Economic Laws
Thus, for contravention of provisions of section 12, as Transaction was entered by Saath Ltd. before 19 months
aforesaid, the promoter company, Trees Estate Ltd. would preceding the insolvency commencement date with Janam
have been penalized. Ltd. and Saath Ltd. and Janam Ltd. were having two directors
As per Section 61 of the Real Estate (Regulation & in common.
Development) Act, 2016, if any promoter contravenes As per Section 5(24) of the IBC, 2016, related party, in
any other provisions of this Act, other than that provided relation to a corporate debtor, inter-alia, means any person
under section 3 or section 4, or the rules or regulations who is associated with the corporate debtor on account of
made thereunder, he shall be liable to a penalty which may having more than two directors in common between the
extend up to five per cent of the estimated cost of the real corporate debtor and such person.
estate project as determined by the Authority. As, Saath Ltd. and Janam Ltd. were having two directors in
Here it is given, that the authority under RERA passed an common, Janam Ltd. would be considered as related party in
order imposing maximum penalty upon the promoter relation to Saath Ltd. and the transaction took place within 2
company, Trees Estate Ltd. and the estimated cost of the years preceding the insolvency commencement date.
real estate project was R200 crores, so the amount of penalty Thus, the said transaction can be said to have entered within
would have been 5% of R200 crores = R10 crores. the relevant period for considering it as an undervalued
As per Section 43 of the Real Estate (Regulation & transaction.
Development) Act, 2016, any person aggrieved by any
direction or decision or order made by the Authority or by an In case of transaction entered by Saath Ltd. with Mr.
adjudicating officer under this Act may prefer an appeal before Mahesh
the Appellate Tribunal having jurisdiction over the matter. Transaction was entered by Saath Ltd. before 17 months
Where a promoter files an appeal with the Appellate preceding the insolvency commencement date with Mr.
Tribunal, it shall not be entertained, without the promoter Mahesh. Mr. Mahesh is a house worker of Mr. Sunil, the
first having deposited with the Appellate Tribunal atleast director of Saath Ltd. There was a case under the Prohibition
thirty per cent. of the penalty, or such higher percentage as of Benami Property Transactions Act, 1988, going against Mr.
may be determined by the Appellate Tribunal, or the total Mahesh and Mr. Sunil, due to acquisition of such property in
amount to be paid to the allottee including interest and the name of Mr. Mahesh and it was held that in the order
compensation imposed on him, if any, or with both, as the passed that Mr. Mahesh was the ‘benamidar’ and Mr. Sunil
case may be, before the said appeal is heard. was the ‘beneficial owner’.
Explanation—For the purpose of this sub-section "person" As per Section 5(24) of the IBC, 2016, Related party, in
shall include the association of allottees or any voluntary relation to a corporate debtor, inter-alia, means — a director
consumer association registered under any law for the time or partner or a relative of a director or partner of the
being in force. corporate debtor
In the given case, the authority under RERA has imposed Now, as per the order passed under the provisions of the
a penalty on Trees Estate Ltd. as well as directed to it Prohibition of Benami Property Transactions Act, 1988, Mr.
to compensate the said 30 allottees by returning their Mahesh was considered as the ‘benamidar’ and Mr. Sunil was
cumulative investment amount of R20 crores along with total considered as the ‘beneficial owner’ and thus, it can be said
interest of R2 crores. that, in substance, the transaction was entered by Saath Ltd.
with Mr. Sunil and not with Mr. Mahesh and Mr. Sunil being
Thus, the amount of pre-deposit that would have been made
a director of Saath Ltd. would be considered as the Related
by Trees Estate Ltd. for filing the appeal with the Appellate
party in relation to Saath Ltd.
Tribunal would be:-
Also, the transaction took place within 2 years preceding the
• 30% of R10 crore = R3 crore or such higher percentage as
insolvency commencement date.
may be determined by the Appellate Tribunal and;
• The total amount to be paid to the allottee including Thus, the said transaction can be said to have entered within
interest and compensation imposed on him i.e. R20 the relevant period for considering it as an undervalued
crores + R2 crores = R22 crores. transaction.

7. (i) As per Section 46 of the IBC, 2016, in an application (ii) As per Section 45 of the IBC, 2016, a transaction shall be
for avoiding a transaction at undervalue, the liquidator or considered undervalued where the corporate debtor —
resolution professional shall determine : (a) makes a gift to a person; or
a) That the transaction was entered within the period of one (b) enters into a transaction with a person which involves
year preceding the insolvency commencement date; or the transfer of one or more assets by the corporate debtor
b) That the transaction was made with a related party for a consideration the value of which is significantly
within a period of two years preceding the insolvency less than the value of the consideration provided by the
commencement date. corporate debtor,
The Adjudicating Authority may require an independent and such transaction has not taken place in the ordinary
expert to assess evidence relating to the value of the course of business of the corporate debtor.
transactions
In case of transaction entered by Saath Ltd. with Janam In case of transaction entered by Saath Ltd. with Janam
Ltd. Ltd.

16 December 2021 The Chartered Accountant Student

Page 514
Economic Laws
Though the transaction has not taken place in the ordinary FAL Ltd. in the City Civil Court due to the restrictions as
course of business of the corporate debtor but the mentioned in the aforesaid provision.
consideration for such supply of goods does not appear to As per Section 63 of the IBC, 2016, no civil court or authority
be significantly lesser than the value of the consideration shall have jurisdiction to entertain any suit or proceedings
provided by the corporate debtor as consideration charged in respect of any matter on which National Company Law
by Saath Ltd. was R4.4 crores which it would have normally Tribunal or the National Company Law Appellate Tribunal
sold for 4.6 crores. has jurisdiction under this Code.
Thus, transaction entered by Saath Ltd. with Janam Ltd. Civil court not to have jurisdiction.
cannot be said to be an undervalued transaction even though
Here, in the given case, the Adjudicating authority i.e. the
it has been entered into with a related party within the
NCLT was having the jurisdiction over the matter with respect
relevant period.
to non-payment to KC & Sons as per the resolution plan by
FAL Ltd. and thus, the City Civil court cannot entertain such
In case of transaction entered by Saath Ltd. with Mr. suit as it is not having the jurisdiction to do the same.
Mahesh
Here, Saath Ltd. had sold a property to Mr. Mahesh for R15
crore, the stamp duty value of which was R35 crore. It can be 9. As per Section 52(7) of the IBC, 2016, where the enforcement
said that consideration charged is significantly less than the of the security interest yields an amount by way of proceeds
value of the consideration provided by the corporate debtor, which is in excess of the debts due to the secured creditor, the
Saath Ltd. secured creditor shall—
Further, Mr. Mahesh is a house worker of Mr. Sunil, the (a) account to the liquidator for such surplus; and
director of Saath Ltd. and also an order under the Prohibition (b) tender to the liquidator any surplus funds received from
of Benami Property Transactions Act, 1988 was passed the enforcement of such secured assets.
against them. So, such transaction also does not appear to
Thus, TLF (P) Ltd. should account to the liquidator surplus
take place in the ordinary course of business of the corporate
sum of R2 crores yielded in excess of its debts due from
debtor, Saath Ltd.
Anmoli Ltd. as well as tender the same to the liquidator.
Thus, the transaction entered by Saath Ltd. with Mr. Mahesh
As per Section 52(7) of the IBC, 2016, before any security
can be said to be an undervalued transaction.
interest is realised by the secured creditor, the liquidator
shall verify such security interest and permit the secured
8. As per Section 33(5) of the IBC, 2016, subject to section
creditor to realise only such security interest, the existence
52, when a liquidation order has been passed, no suit or
of which may be proved either—
other legal proceeding shall be instituted by or against the
corporate debtor. (a) by the records of such security interest maintained by an
information utility; or
Provided that a suit or other legal proceeding may be
(b) by such other means as may be specified by the Board.
instituted by the liquidator, on behalf of the corporate debtor,
with the prior approval of the Adjudicating Authority. Thus, before realizing such security interest by TLF (P) Ltd.
the liquidator should have verified the security interest as
Thus, KC & Sons should not have instituted a suit against
aforesaid.

Case Study 4
Mapple Inc. is an American MNC that designs and markets can be broadly classified as smartphones and featured phones.
consumer electronics, computer software and personal While acknowledging that iPhone is a unique product, there
computers, etc. Mapple India is the Indian subsidiary of Mapple are certain smartphones offered by other brands such as Nokia,
Inc. through which it markets and sells its products in India. Blackberry, and Samsung that have advanced features and which
XPhone and Sintel are leading mobile service providers in could be considered as substitutes for the iPhone.
India, jointly having more than 30 crore Indian subscribers that Mapple Inc. and Mapple India entered into some exclusive
account for almost 52% market share in the GSM market. In contracts/agreements with XPhone and Sintel respectively, for
total, there are around 20 service providers in India but none of the sale of iPhones in India, even prior to its launch. XPhone and
them individually holds more than 30% of the total market share. Sintel are both, cellular data and GSM network service providers
Particular models of iPhones – iPhone 3G and iPhone 3GS, functioning in India. As a result of the agreements, XPhone and
were manufactured by Mapple Inc., launched in India during Sintel got exclusive selling rights for an undisclosed number
August 2008 and March 2010, respectively. During the fiscal year, of years. The iPhones sold by XPhone and Sintel came in the
2010, worldwide sales of iPhones were 73.5 million. compulsorily locked form, thereby meaning, that the handset
Mobile services in India can be offered through two competing purchased from either of them shall work only on their respective
technologies i.e. GSM and CDMA and that, SIM cards of each of networks and none other.
these cellular services are compatible only with those handsets Mapple Inc. permitted iPhone users only those applications on
which deploy their respective technologies and thus not able to their iPhones that have been approved by them and available
substitution. iPhones are based on GSM technology. Handsets through their own online application store namely ‘App Store’.
The Chartered Accountant Student December 2021 17

Page 515
Economic Laws
Further, no other third-party applications can be run on iPhone Reply by XPhone to the report of CCI:
unless the same has been approved by Mapple Inc. If however, The agreement was non-exclusive and iPhones were available
the operating system of jail broken iPhone is upgraded, the in India through a number of other distributors/channels and
iPhone gets re-locked and all the third-party applications are XPhone, being a telecom service provider provided the best tariff
deleted by the servers of Mapple Inc. permanently. XPhone and plans to its customers and XPhone never imposed any restrictions
Sintel refused to accept any iPhone for repairs at their authorized on its customers with respect to using unlocked phones and
service centers if the same is not purchased from them. However, therefore, there it can be said that there is no violation.
an unlocked iPhone can be purchased from abroad. Also, a The tariff plans, as were provided to iPhone customers were
consumer who has purchased a locked iPhone in India and has the same and if not, even better than the normal plans offered
paid the unlocking fees is free to choose the network operator of to other subscribers. Further, the tariff plans, as approved by
his choice after unlocking the iPhone. Mapple Inc. were filed with the TRAI in August 2008 and were
Out of the total market share for smartphones in India, Mapple in full compliance with the TRAI regulations. Additionally, it
India had a market share of 1.5% in the year 2008; less than 1% is important to note that, even if an iPhone specific plan was
in 2009 and 2010 respectively, and 2.4% in 2011. Additionally, at published, the customers always had complete freedom to
the time of the launch of the iPhone in India, there were about choose from other plans which were not iPhone specific and
250 million GSM mobile subscribers which subsequently rose to rather the customer were spoilt for choice, given the range
about 600 million in the year 2011. of plans available to them. Therefore, there is no question of
XPhone, being discriminating with iPhone customers vis-à-vis
An allegation by Ms. Rekha: its other customers.
Ms. Rekha was one of the biggest fans of iPhones. After it was The concept of “collective dominance” is not recognized under
launched in India, she purchased an iPhone but was extremely section 4 of the Competition Act. Both, Sintel and XPhone
disappointed when she realized, that, there were so many are separate legal entities, with no structural links and with
restrictions for using such iPhone which did not appear, value completely different boards of directors and management.
for money. When she investigated more into this, she found out Therefore, the question of “collective dominance” does not arise.
that Mapple India was taking undue advantage of the dominant
iPhones are easily available in the open market and without any
position that it enjoyed in the market. She then approached the
network locking. More importantly, even the iPhones bought
CCI, to file a complaint against such abuse, in violation of section
through XPhone distribution channels were unlocked as and
4 of the Competition Act, 2002. In her complaint, she made the
when a request was made after following the due process. Further,
following allegations -
the TRAI’s MNP (mobile number portability) regulations give
Mapple India enjoys a dominant position in the relevant market a right to the customer to move from one service provider to
for smartphones, both in India as well as internationally, as another freely, and consequently, the same customer can unlock
iPhone, being the largest selling smartphone in the world. The his phone without any hassle. These facts clearly indicate that the
informant also averted that XPhone and Sintel jointly enjoyed allegations in the information are mere speculations and should
a dominant position in the relevant market for GSM mobile be dismissed outright.
telephony services in India. The informant further submitted
that XPhone and Sintel have abused their dominant positions by
imposing unfair conditions on the purchasers of Mapple iPhones. Answer the following questions:
Multiple Choice Questions
Reply by Sintel to the report of CCI: 1. The relevant market(s) that the Director-General will identify
It fails to consider that any dispute in relation to a while making the inquiry is/are
telecommunication service is actionable under the Telecom I. Smart Phones in India
Regulatory Authority of India Act, 1997, and the Competition II. GSM cellular service in India
Act, 2002 cannot be invoked as the CCI does not have any III. Smart Phones in America and India
jurisdiction on the matters of cellular service providers in India (a) Only I
when TRAI is the regulatory body. The bundled offer was in (b) I and II
compliance with the guidelines of TRAI. (c) II and III
The informant failed to make any averment of having purchased (d) I, II, and III
Mapple iPhone 3G/3GS to show that she had any interest in the
matter and has the locus standi to file the information. 2. The iPhones sold by XPhone and Sintel came in the
The informant also failed to state that she had purchased iPhone compulsorily locked form, thereby meaning, that the handset
3G and 3GS from the grey market in India or abroad and purchased from either of them shall work only on their
consequently it is inexplicable as to how she has a grievance in respective networks and none other. This is in the nature of
this regard. (a) Exclusive supply agreement
(b) Horizontal agreement
Mapple iPhone 3GS is being sold since June 2011 without its
(c) Tie in agreement
network being locked. For this reason, the issue raised in the
(d) Refusal to deal
information filed by Ms. Rekha is infructuous. The practice
3. Whether the contention of Sintel that CCI does not have
of locking the network onto the Mapple iPhone, even though
jurisdiction on the matters of cellular service providers in
in accordance with international practice, has long been
India when TRAI is the regulatory body is correct?
discontinued in India.
(a) Yes, TRAI has sole jurisdiction as the industry regulator,
CCI does not have jurisdiction

18 December 2021 The Chartered Accountant Student

Page 516
Economic Laws
(b) No, both have the jurisdiction; but TRAI can supersede 5. (a) Central Government
and has primacy being industry regulator over CCI.
(c) No, both are special Acts and primacy have to be given 6. Legal Position: As per section 19(4) of the Competition Act
to the respective objectives of both the regulators under 2002, the Commission (CCI) shall, while inquiring whether
their respective statutes. an enterprise enjoys a dominant position or not, have due
(d) Can’t say, as information on TRAI regulations is not regard to all or any of the following factors, namely:—
provided. a. market share of the enterprise;
b. size and resources of the enterprise;
4. Assuming that iPhone is not purchased by Miss Rekha from c. size and importance of the competitors;
the Mapple store. Can she file a case, in the forum under the d. economic power of the enterprise including commercial
Competition Act 2002? advantages over competitors;
(a) No, as Ms. Rekha has purchased iPhone from the grey e. vertical integration of the enterprises or sale or service
market i.e. through distributors and thus, has no right to network of such enterprises;
file a case f. dependence of consumers on the enterprise;
(b) No, as Ms. Rekha has not suffered any loss due to tie- g. monopoly or dominant position whether acquired as a
up agreement made by Mapple India with XPhone and result of any statute or by virtue of being a Government
Sintel respectively company or a public sector undertaking or otherwise;
(c) Yes, as Ms. Rekha has used the iPhone and availed the h. entry barriers including barriers such as regulatory
cellular services, so she indirectly gets affected barriers, financial risk, the high capital cost of entry,
(d) Yes, not only Ms. Rekha but any person can file such a marketing entry barriers, technical entry barriers,
case economies of scale, high cost of substitutable goods or
service for consumers;
5. The chairperson and other members of the CCI office shall i. countervailing buying power;
be appointed by: j. market structure and size of the market;
(a) Central Government k. social obligations and social costs
(b) Relevant State Government l. relative advantage, by way of the contribution to the
(c) High Court economic development, by the enterprise enjoying
(d) Central Government and the selection committee a dominant position having or likely to have an
appreciable adverse effect on competition;
Descriptive Questions m. any other factor which the Commission may consider
6. Whether there can be a case of abuse of dominant position relevant for the inquiry.
against Mapple India, XPhone, and Sintel respectively? The dominant position has been defined under explanation
(a) to Sec 4 as a position of strength, enjoyed by an enterprise,
7. Is there an appreciable adverse effect on competition due in the relevant market, in India, which enables it to operate
to the agreement made by Mapple India with XPhone and independently of competitive forces prevailing in the
Sintel respectively? relevant market; or affect its competitors or consumers or
the relevant market in its favour.
8. Briefly states the duties of the CCI and the orders that can
be passed by it after the establishment of infringement of Analysis of the case
section 3 or section 4 respectively? Mapple India had a market share of 1.5% in the year 2008; less
than 1% in 2009, and 2010 respectively and 2.4% in 2011. Prima
Answers to Case study 4 facie, these percentages of market share don’t suggest anything
that tantamounts to the existence of dominance.
1. (b) I and II
XPhone and Sintel are leading mobile service providers in India,
2. (c) Tie in agreement jointly having more than 30 crore Indian subscribers that account
for almost 52% market share in the GSM market. As regards the
3. (c) No, both are special Acts and primacy has to be given dominance of XPhone and Sintel in the relevant market, since
to the respective objectives of both the regulators under both are two separate entities without the evidence of having any
their respective statutes horizontal agreement or cartelization between them that could
The honorable apex court in the Civil Appeal no. 11843 be deemed as anti-competitive. Hence, on the basis of section
of 2017 (CCI vs Bharti Airtel Ltd) recognised that the 19(4) conditions that neither Sintel nor XPhone, individually,
TRAI Act and the Competition Act are both special have any adequate market power so as to be deemed dominant.
Acts and primacy has to be given to the respective Also, the argument that XPhone and Sintel hold nearly 52% of
objectives of both the regulators under their respective the market share in the GSM services in India cannot be accepted
statutes. CCI’s jurisdiction is not excluded by the for the fact that they are horizontal competitors who fight for
presence of sectoral regulators and to that end, the CCI greater market share. Moreover, there is no allegation, qua these
enjoys primacy with respect to issues of competition OPs that they have indulged in anti-competitive conduct among
law. themselves for a common cause.

4. (c) Yes, as Ms. Rekha has used the iPhone and availed the Conclusion
cellular services, so she indirectly gets affected Thus, it can be concluded that since dominance does not get

The Chartered Accountant Student December 2021 19

Page 517
Economic Laws
established, there can be no case for abuse of dominance against the network services can be availed on any mobile handset,
all the three aforesaid entities under Section 4 of the Act. even an unlocked iPhone purchased from abroad. Also, a
consumer who has purchased a locked iPhone in India and
7. According to Section 3(1) of the Act, “No enterprise paid the unlocking fees is free to choose the network operator
or association of enterprises or person or association of his choice.
of persons shall enter into any agreement in respect of Also, there is no evidence to show that entry barriers have
production, supply, distribution, storage, acquisition or been created for new entrants in the markets i.e. in the
control of goods or provision of services, which causes or is smartphone market and mobile services market by any of
likely to cause an appreciable adverse effect on competition the impugned parties. Similarly, existing competitors have
within India”. not been driven out from the market, or that the market
Section 3(4) of the Act, highlights anti-competitive itself has been foreclosed. Hence, the belief that the tie-
agreements between vertically related enterprise as “Any in arrangement has caused serious harm appears untrue.
agreement amongst enterprises or persons at different Hence, there appears no appreciable adverse effect on
stages or levels of the production chain in different markets, competition due to agreement by Mapple India with XPhone
in respect of production, supply, distribution, storage, sale and Sintel respectively.
or price of, or trade in goods or provision of services,
including — 8. As per section 18 of the Competition Act 2002, the duties of
(a) tie-in arrangement; the Commission are as follows:
(b) exclusive supply agreement; (a) To eliminate practices having an adverse effect on
(c) exclusive distribution agreement; competition,
(d) refusal to deal; (b) To promote and sustain competition in markets in India,
(e) resale price maintenance, (c) To protect the interests of consumers and
shall be an agreement in contravention of sub-section (1) if (d) To ensure freedom of trade carried on by other
such agreement causes or is likely to cause an appreciable participants in markets in India.
adverse effect on competition in India”. As per section 27 of the Competition Act 2002, where after
Further, what constitutes appreciable adverse effect on an inquiry under section 19 regarding alleged contravention
competition has been provided for in Section 19(3) of the of entering into an anti-competitive agreement or abuse
Act. of dominance as per procedure detailed in section 26, if
Commission find the allegation true and contravention of
In the above case, some kind of ‘tie-in arrangement’ can be
section 3(1) or 4(1) respectively, it may pass all or any of the
seen which has an adverse implication on the purchaser of
following orders-
iPhones in terms of their ability to choose and switch between
various cellular service providers and data plans. However, Cease and desist order - direct any enterprise or association
none of following - Mapple India / Sintel / XPhone, have a of enterprises or person or association of persons, as the case
dominant position in their respective market, and that there may be, involved in such agreement, or abuse of dominant
has been no intentions and evidence to show that the market position, to discontinue and not to re-enter such agreement
has been foreclosed to competitors or that entry-barriers have or discontinue such abuse of dominant position, as the case
been erected for new entrants in any of the markets by any may be.
of the opposite parties. Mapple India had a share of less than Impose penalty - as it may deem fit which shall be not more
3% in the market of smartphones during the period 2008-11. than ten percent of the average of the turnover for the last
Furthermore, the share of GSM subscribers using Mapple three preceding financial years, upon each of such person or
iPhone to total GSM subscribers in India is minuscule (less enterprises which are parties to such agreements or abuse.
than 0.1%). No operator has more than 30% market share, Modification of the terms of such agreements -
in an otherwise competitive mobile network service market. Agreements shall stand modified to the extent and in the
As none of the impugned operators, (XPhone / Sintel) have manner as may be specified in the order by the Commission;
market-share exceeding 30%, that smartphone market in
To abide - Which direct the enterprises concerned to abide
India is less than a tenth of the entire handset market, and
by such other orders as the commission may pass and comply
that Mapple iPhone has less than 3% share in the smartphone
with the directions, including payment of costs if any.
market in India, it is highly improbable that there would be
an AAEC in the Indian market for mobile phones. Such other order or issue such directions as it may deem fit.
Moreover, the lock-in arrangement of the iPhone to a In case any agreement referred to in section 3 has been
particular network was only for a specific period and not entered into by a cartel, the Commission may impose upon
perpetual, a fact known to prospective customers. It is each producer, seller, distributor, trader, or service provider
difficult to construe consumer harm from entering into a included in that cartel, a penalty of up to three times of its
‘tie-in arrangement' by the horizontally related enterprises. profit for each year of the continuance of such agreement or
It is observed that there is no restriction on consumers to use ten percent of its turnover for each year of the continuance
the network services of XPhone and Sintel to the extent that of such agreement, whichever is higher.

"The roots of education are bitter, but the fruit is sweet." - Aristotle

20 December 2021 The Chartered Accountant Student

Page 518

You might also like